You are on page 1of 603

LGEBRA

Ximena Carreo Campos


Ximena Cruz Schmidt

1-6 (2003) 1 25/11/02, 10:58 AM


EDICIN Y PRODUCCIN:
Departamento Pedaggico Arrayn Editores S.A.
Actualmente compuesto por:
Direccin Editorial
Hctor Hidalgo Gonzlez
Correccin de Estilo
Alejandro Cisternas Ulloa
Direccin Grfica
Leonardo Vilches Robert
Diseo y Diagramacin
Vinka Guzmn Tacla
Francisco Martnez Muoz
Claudio Silva Castro

Participacin Externa
Revisin de Contenidos:
Bernardita Cruz Schmidt

Del texto: Ximena Carreo Campos y Ximena Cruz Schmidt.


Arrayn Editores S.A.
Bernarda Morn 435, Providencia, Santiago de Chile. Telfono: 431 4200
Fax: 431 4282 e-mail: arrayan@arrayan.cl
Consultas: e-mail: editorial@arrayan.cl
Obra: lgebra Arrayn.
I.S.B.N: 956-240-168-5. Inscripcin: 87.879. Segunda edicin, noviembre 2002.
Impreso en Chile por Morgan Impresores.
Prohibida su reproduccin total o parcial, a travs de cualquier sistema de reprografa o tratamiento
informtico, bajo las sanciones establecidas por la ley.

1-6 (2003) 2 25/11/02, 10:58 AM


Introduccin

En este libro de ejercicios de LGEBRA hemos querido proponer una


cantidad de trabajos que va desde los ejercicios ms tradicionales
para el aprendizaje del lgebra hasta los problemas ms modernos
y desafiantes que invitan al estudiante y al maestro a conversar y
discutir en torno a posibles soluciones.
Creemos sinceramente estar haciendo un aporte para colaborar con
aquellos estudiantes que se interesen en afianzar sus conocimientos
y sentar las bases de una slida formacin matemtica.
Estimado lector: queremos invitarlo a recorrer estas pginas en
el orden que usted estime conveniente y de acuerdo con las
necesidades que se le vayan presentando. En estas lneas vamos
a tratar de darle una visin global del mbito de trabajo de la
aritmtica y del lgebra.
Nuestro mundo numrico se fue generando a lo largo de los
siglos segn los hombres iban necesitando de diversos modos de
comunicacin y de acuerdo con los requerimientos de otras reas de
accin, como el comercio, la astronoma, la agricultura, el desarrollo
de las diversas ciencias, la matemtica por s misma y una infinidad
de actividades en que el hombre se ha interesado por crear su
expresin en trminos numricos.
En la pgina siguiente encontrar un esquema que contiene los
distintos conjuntos de nmeros y la forma como los matemticos los
han ido ordenando de acuerdo con distintos criterios; y ms adelante
ver un grfico de los diferentes conjuntos numricos.
El objetivo que nos hemos propuesto al escribir esta introduccin
y proponerle algunas actividades es que usted se forme una idea
global de los distintos mbitos en que se mueve la aritmtica y, como
consecuencia, el lgebra, que no es otra cosa que la descripcin
de modelos matemticos para representar mltiples situaciones de
la naturaleza y/o generaciones abstractas del matemtico. Estos
modelos son las distintas relaciones entre variables, que al asignarles
los valores adecuados y haciendo los anlisis pertinentes nos entregan
potentes herramientas para resolver problemas tradicionales, como
la trayectoria de un proyectil, que se puede describir a travs de una
ecuacin de segundo grado, u otros, como el uso de matrices para
organizar y manipular gran cantidad de informacin.

Introduccin 3

1-6 (2003) 3 25/11/02, 10:58 AM


Le vamos a pedir que observe con mucha detencin el esquema
titulado Conjuntos Numricos y analice con sus compaeros
estudiantes o con sus profesores toda la informacin que pueda
obtener de l. No sera extrao que la primera vez no logre recoger
mucha informacin, pero con el tiempo, y conforme el avance
en sus conocimientos, debera servirle de gran ayuda para tener
una visin global de los mbitos numricos que el hombre ha ido
definiendo y entender por qu los ha ordenado de esta manera
y no de otra.
Lo invitamos a observar el esquema propuesto y a reflexionar en
torno a la informacin que contiene.

A continuacin le entregamos la misma informacin pero con otra


presentacin y lo invitamos a que usted ubique correctamente, en
el conjunto correspondiente del ESQUEMA DE LOS CONJUNTOS
NUMRICOS, los nmeros que listamos ms adelante. El profesor
podr inventar una infinidad de actividades para determinar si sus
alumnos(as) se ubican bien en los distintos conjuntos numricos.
Por ejemplo: Cules fueron los primeros nmeros inventados?;
para qu servan?; cmo se expresa la ausencia de valor?; qu
operaciones aritmticas estn definidas en cada conjunto?; por qu?;
qu conjuntos son subconjunto de otros?; cules son disjuntos?;
qu necesidad del hombre inspir la ampliacin de los Naturales
a los Enteros?; y a los Racionales?; qu ejemplo concreto puede
dar de un nmero irracional?; cmo lo puede ubicar en la recta
numrica?; qu diferencia hay entre una fraccin y una razn?;
cmo se generaron los nmeros Complejos?; dnde y para qu se
usan?; cmo se grafican?; etc.

4 Introduccin

1-6 (2003) 4 25/11/02, 10:58 AM


Aqu hay una cantidad de nmeros y usted deber determinar a qu
conjunto numrico pertenece y ubicarlo en el esquema siguiente.
1
a) 3; 2 ; 2,6; 8
; 3; 5; 12; 36; 15 ; 2i; 0; 3,5;
3 4 3
32
1 ; 0,5; 12;
8
1 12
b) 5; 1; 16; 32; ; 25; (1,3); 2;
4
; 2 5i;
4
3 6 ; 1,32; 5 ; 12 ; 7
8;
7 4 10
1 1 22
c) ; 6; 51; 6 ; 15; 3 2 ; (4, 1); 3 ; 4 + 2i; ;
3 3 11
5
8,3; (1)4; ; 144
6
1 18 3 3 25 1
d) 3,2; 9; ; ; 9 ; 0; 12 ; ; 32; 61; ;
9 5 9 5 2
3 2 ; 0.02; 3%
27 ; 12,3;
100

e) 12; 23; 32; (2)5; (3)12; 6 ; 3 1; 5 ; 12;


3 12 5
36; 5i; 1 ; 0,16; 100%

1
f) 50%;
1
2
; 0,5;
1
4
; ( 4 )1; 3
6
; ( 147 ) ; (( 2 ) ) 1 2 1
2i 2 ;

(( ) )
2
2 1 2
; 2 ;
8 2 4

Esquema de los Conjuntos numricos

C
Q R
Z
N I

Introduccin 5

1-6 (2003) 5 25/11/02, 10:58 AM


Con el objeto de que el estudiante pueda formarse una idea completa
de lo que abarca el lgebra abordada en el texto, le proponemos, a
continuacin, un resumen esquemtico que puede ayudar a tener una
idea general de los contenidos.

Mapa de contenidos del Algebra

FUNCIONES MATRICES
LGEBRA
COMPLEJOS

CANTIDADES
REALES ESCALARES VECTOR EN R2

EXPRESIONES
ALGEBRAICAS CANTIDADES
VECTORIALES
TRMINO
ALGEBRAICO
POLINOMIO
COMPARACIONES

COEFICIENTE GRADO

IGUALDAD DESIGUALDAD
VARIABLE

ECUACIONES INECUACIONES
OPERACIONES
INTERVALO
EN R
LOGARITMO SISTEMAS DE SISTEMAS DE
ADICIN
ECUACIONES INECUACIONES

PRODUCTO RACES
PROBLEMAS DE
POTENCIA OPTIMIZACIN

Obsrvelo, comente con sus compaeros y profesores lo que encuentre


en l; critquelo y enve sus observaciones al correo electrnico
ximenacs@entelchile.net.

En el texto hemos querido entregarle referencias para desarrollar sus


estructuras mentales, pero sin duda esto no se lograr si no se desea y
trabaja con esfuerzo y persistencia. Es probable que alguna vez haya
escuchado decir que el desarrollo del pensamiento es un proceso
interior de la persona. Efectivamente, el mundo circundante, cercano
o lejano fsicamente, las inquietudes personales, las expectativas en
la vida, la disposicin a trabajar son las nicas herramientas que lo
pueden llevar a desarrollar su capacidad de pensar y a enriquecer
sus estructuras mentales. Como usted sabe, el aprendizaje se produce
cuando relacionamos algo novedoso con algo que ya sabemos; por
eso es que la persona cada vez que aprende, potencia ms an su
capacidad de aprender. Ponemos en sus manos este texto con la ilusin
de que sea un medio eficaz para enriquecer sus estructuras mentales
y su aprendizaje en general. En la medida que ello suceda, el texto
estar sirviendo efectivamente como un medio para el aprendizaje,
y as estaremos colaborando en su crecimiento como persona en
este mudo globalizado.

6 Introduccin

1-6 (2003) 6 25/11/02, 10:58 AM


CAPTULO 1

CAPTULO 1
A
lgebra
en los
nmeros reales

Lenguaje algebraico 1.1

El lenguaje algebraico se basa en el uso de letras y relaciones


matemticas para generalizar diferentes situaciones.
Ejemplos:
El permetro P de un cuadrado de lado a P = 4a.

El rea A de un cuadrado de lado a A = a2.

El rea A de un tringulo de base b y altura h A=bh


2
Cada una de las letras involucradas en las frmulas anteriores
es una variable; a cada variable se le pueden asignar diferentes
valores.
En general, una variable es cualquier letra involucrada en
una expresin algebraica.
Expresemos en lenguaje algebraico:
1. El doble de un nmero 2a, 2x, 2m, ...
2. El triple de un nmero 3x, 3y, 3b, ...
p q z
3. La mitad de un nmero , , , ...
2 2 2
4. El cuadrado de p p2
5. a aumentado en b a+b
6. a disminuido en b ab
7. El producto entre a y b ab

Si en alguna expresin no est especificado el trmino,


podemos asignar cualquier variable para representar el enunciado,
como se puede ver en los ejemplos 1, 2, 3 y 4.

lgebra en los nmeros reales 7

7-13.(2003) 7 18/11/02, 10:32 AM


En general,
Son mltiplos de a:

el doble 2a
el triple 3a
el cudruple 4a
el quntuple 5a
:
:

Son fracciones de a:
a o 1
un medio (o la mitad) a
2 2
a o 1
un tercio (o la tercera parte) a
3 3
a o 1
un cuarto (o la cuarta parte) a
4 4
a o 1
un quinto (o la quinta parte) a
5 5
:
:

Son potencias de a:

el cuadrado a2
el cubo a3
la cuarta potencia (o a la cuarta) a4
la quinta potencia (o a la quinta) a5
:
:

Otras expresiones algebraicas:


Un nmero par 2n
Un nmero impar 2n 1

Ejercicios Expresemos en lenguaje algebraico:

resueltos 1. El doble de un nmero, aumentado en la mitad del mismo nmero.


Aqu el nmero no est determinado; asignmosle la variable
x; nos queda:
x
2x +
2
2. El doble de a, aumentado en b
2a + b

3. El doble de a aumentado en b
2 (a + b)
Observe los ejemplos 2 y 3. Cul es la diferencia?

8 lgebra en los nmeros reales

7-13.(2003) 8 18/11/02, 10:32 AM


CAPTULO 1

4. La mitad de a ms el triple de b.
Aqu ya estn asignadas las variables, son a y b. Nos queda:
a
+ 3b
2
5. El doble del cuadrado de a.
2 a2

6. El cuadrado del doble de a.


(2a)2
Observe la diferencia entre los ejercicios 5 y 6.

7. La cuarta parte del triple del cuadrado de b.


3 b2
4
8. El triple de la cuarta parte del cuadrado de b.
b2
3 ( )
4

9. El cuadrado de la cuarta parte del triple de b.


3b 2
(4)
Observe las diferencias entre los ejercicios 7, 8 y 9.

10. La diferencia entre el quntuple de x y la mitad de y.


y
5x
2
11. La suma de tres nmeros pares consecutivos.
(2n) + (2n +2) + (2n + 4)
o
(2n 2) + (2n) + (2n + 2)
Observe la diferencia entre ambas.

12. Tres impares consecutivos.


2n 1, 2n + 1, 2n + 3
2n + 1, 2n + 3, 2n + 5
Observe la diferencia entre ambas y exprese esos tres nmeros
de una manera distinta.

13. La semisuma entre a y b.


a+b
2
14. La semidiferencia entre a y b.
ab
2
15. El producto entre un nmero y su antecesor.
x (x 1)
16. El producto entre un nmero y su sucesor.
x (x + 1)

lgebra en los nmeros reales 9

7-13.(2003) 9 18/11/02, 10:32 AM


Ejercicios

I. Asigne variables y exprese en lenguaje algebraico:

1. La mitad de un nmero.

2. El triple de a, aumentado en el doble de b.

3. El doble del cociente entre a y b.

4. El cubo de la diferencia entre x e y.

5. La diferencia entre el cubo de x y el cuadrado de y.

6. El cuadrado de a equivale a la suma entre el cuadrado de x y el cuadrado


de y.

7. La suma de tres nmeros consecutivos es 213.

8. La suma de tres pares consecutivos es 168.

9. El cubo del cuadrado de la diferencia entre x e y.

10. La cuarta parte del producto entre el cuadrado de a y el cubo de b.

11. El triple de un nmero equivale al doble del mismo nmero aumentado


en 15.

12. El volumen de una esfera de radio r equivale al producto entre cuatro tercios
de p y el cubo del radio.

13. La superficie de un rectngulo cuyos lados miden (a + 3) y (a 3).

14. El volumen de un cubo de arista 2a 1.

15. El volumen del paraleleppedo de la figura

16. La superficie lateral del paraleleppedo de la figura.

17. La suma de los cuadrados de tres nmeros consecutivos.

18. El cuadrado de la suma de tres nmeros consecutivos.

10 lgebra en los nmeros reales

7-13.(2003) 10 18/11/02, 10:32 AM


CAPTULO 1

Soluciones

a 2 3
1. 10. a b
2 4
2. 3a + 2b 11. 3x = 2x + 15
a 4
3. 2
b 12. V = p r3
3
4. (x y)3 13. S = (a + 3) (a 3)

5. x3 y2 14. V = (2a 1)3

6. a2 = x2 + y2
15. V = 2a(2a + 3)(2a + 1)
7. (a 1) + a + (a + 1) = 213
a + (a + 1) + (a + 2) = 213 16. S = 2(2a (2a + 3) + 2a(2a + 1))

8. (2n 2) + 2n + (2n + 2) = 168 17. x2 + (x + 1)2 + (x + 2)2

9. [(x y)2]3 18. [x + (x + 1) + (x + 2)]2

Definicin: Se llama trmino (algebraico) a un con-


junto de nmeros y letras que se relacionan entre s
por medio de la multiplicacin y/o divisin.

3a 5 2 2
Ejemplo: 2a2 b , p , x y z.
7
El trmino algebraico consta de un FACTOR NUMRICO, un
FACTOR LITERAL y un GRADO.
El grado es la suma de los exponentes de las letras que aparecen
en el trmino.
12 6 4 2
Ejemplo: En el trmino a b c el coeficiente numrico es
17
12
; el factor literal es a6b 4 c2 y el grado es 12 (6+4+2).
17

Observacin 1: Si el coefi ciente numrico no est escrito,


entonces es 1.
Observacin 2: Si el grado no est escrito, entonces es 1.
Se llama EXPRESIN ALGEBRAICA a cualquier suma o resta de
trminos algebraicos. Si la expresin tiene dos trminos, entonces es un
BINOMIO; si tiene tres trminos se llama TRINOMIO; si tiene cuatro o
ms, hablamos de POLINOMIOS. (El trmino POLINOMIO se puede
usar en forma general para cualquier expresin algebraica.)

lgebra en los nmeros reales 11

7-13.(2003) 11 18/11/02, 10:33 AM


1.2 Valorizacin de
expresiones algebraicas

Las expresiones algebraicas no representan valores en s, sino


que pueden ser evaluadas para distintos valores que se les asignen a
las letras que las componen.

Ejercicios 1. El valor del monomio a2b cuando a = 2 y b = 5


resueltos es 22 5 = 20.
Reemplazamos directamente las letras a y b por los valores
asignados; en este caso, 2 y 5, y realizamos las operaciones
indicadas.
2. El valor del mismo monomio a2b cuando a = 3 y b = 4 es:
32 ( 4) = 9 4 = 36
3. Si x = 2; y = 5 y z = 4, el valor de
2x + 3y z es:
22+354=
4 + 15 4 = 7
4. Si m es el doble de n, n es el cuadrado de p y p = 3,
determinemos m y n:
Aqu tenemos: m = 2n; n = p2 y p = 3, entonces n = 32 = 9
y m = 2n = 2 9 = 18.
As; n = 9 y m = 18.

Ejercicios
2
Determine coeficiente numrico, factor literal y grado de 7. a b
I. los siguientes trminos algebraicos: 2 4
3a b
8.
2 3 8 5
1. 3 ab 4. 17 p q z
12
2 m n
2. a 5. 0,3 c 9.
5 9
11
2 2 x y
3. 0,02 a b 6. a 10.
4

12 lgebra en los nmeros reales

7-13.(2003) 12 18/11/02, 10:33 AM


CAPTULO 1

II. Si a = 3 y b = 2, IV. Si x = 4, y = 2 y z = 5, determine


determine el valor de: el valor de:

1. 2 ab 1. 2 x + y + z
2 2 2. x y 2z
2. a b
2
3. b a
2 3. x + y x + z

2 2 2 2 2
4. a + ab + b 4. x x + y + z

1 1
5. 2ab 5.
x y
3 3 2 2
6. a b 6. 2 x y 2 x z
5 2
7. b 7. x 1

8. 1 + a + b + ab 2
8. z 2 + z 3
2

2 2
9. a + b a b 9. 3 x yz + 2 x yz
3
b 2 4 z
10. a 6 10. x y +
4 5

Si m = 2 y n = + 3,
III. determine el valor de: V.
1. 2 m 3n
1. Si m + n = 3 y n = 1, determine m.
2
2. m m 2n
2. Si m 3 = 2p y p = 2 determine m.
3. 1 + m
3. p + q r = 12 , r q = 5, determine p.
2 2
4. m n 4. 2a 9 = b y a = 3, determine b.
5. m + n m n
5. 1 + 2a = b 2 y a = 2, determine b.
2 2
6. m + 2 mn + n 6. Si a es el doble de b, b es un tercio de c
y c = 12, determine a y b.
7. 5 mn
7. Si m es la cuarta parte de p y p es el
1 1 cuadrado de 2, determine m.
8.
m n
8. La mitad de a es 1. Cul es el valor de a?
1
9.
m n 9. La tercera parte del doble de m es 4. Cul
es el valor de m?
1
10.
mn 10. Si p + q = 2r, q es el triple que p y p = 5,
cul es el valor de r?

lgebra en los nmeros reales 13

7-13.(2003) 13 18/11/02, 10:33 AM


CAPTULO 1

lgebra en los nmeros reales 13

14-15. 13 18/11/02, 10:40 AM


Soluciones

I. 1. 2. 3. 4. 5. 6. 7. 8. 9. 10.
Coeficiente 2 3 1 1
numrico 3. 0,02 17 0,3 1 1
5 5 9 4
Factor literal ab a a2b2 p2q3z8 c a a2b a2b4 m12n x11y
Grado 2 1 4 13 1 1 3 6 13 12

II. 1. 12 2. 5 3. 5 4. 19 5. 12 6. 19 7. 32 8. 12 9. 8 10. 5
5 1 1
III. 1. 13 2. 12 3. 1 4. 5 5. 5 6. 1 7. 30 8. 9. 10.
6 5 6
3
IV. 1. 11 2. 4 3. 7 4. 180 5. 6. 264 7. 15 8. 45 9. 85 10. 1
4
V. 1. m = 4 2. m = 1 3. p = 17 4. b = 15 5. b = 1 6. a = 8 b = 4 7. m = 1

8. a = 2 9. m = 6 10. r = 10

1.3 Reduccin de trminos


semejantes y uso de parntesis

Definicin: Se llaman TRMINOS SEMEJANTES aquellos que


tienen el mismo factor literal (y por consiguiente el mismo
grado); slo pueden diferir en el coeficiente numrico.

Ejemplo 1. Son trminos semejantes:


a2
a2, 2a2, 3a2, 0,5a2,
4
Ejemplo 2. No son trminos semejantes:

a2 b y ab2, a y a2, 2ab y ab2,


Vemos que en el ejemplo 1, el factor literal de todos ellos es a2;
por esta razn son todos semejantes.
En el ejemplo 2, en cambio, tenemos en los tres casos factores
literales diferentes entre s.
En una expresin algebraica SLO podemos reducir aquellos
trminos que son semejantes y esto se efecta sumando (o restando) los
coeficientes numricos y manteniendo el factor literal.
El uso de parntesis es frecuente en lgebra. Sirve para separar
expresiones algebraicas y se elimina de acuerdo con las siguientes
reglas:
1. Si est precedido de un signo + o no tiene signo escrito, se
elimina sin hacer ningn cambio.
2. Si est precedido de un signo se elimina despus de cambiar
TODOS los signos de los trminos del interior del parntesis. (Es

14 lgebra en los nmeros reales

14-15. 14 18/11/02, 10:40 AM


CAPTULO 1

importante hacer notar que al eliminar el parntesis tambin se


elimina el signo que lo antecede.)
Si una expresin algebraica contiene parntesis, es conveniente
eliminarlo antes de proceder a reducir los trminos semejantes.

1. a + 2a + 3a Ejercicios
Los tres trminos de la expresin son semejantes; por lo resueltos
tanto, sumamos sus coefi cientes numricos y conservamos
el factor literal:
a + 2a + 3a = 6a
2. 2a + 3b 5a + 6b
Aqu los trminos 2a y 5a son semejantes entre s y lo mismo
ocurre con 3b y 6b; entonces los podemos agrupar entre s
y obtenemos:
2a + 3b 5a + 6b = (2a 5a) + (3b + 6b) = 3a + 9b

3. 3x6y 5xy6 7x6y x6y + 11xy6


Agrupamos los trminos segn su semejanza y obtenemos:
(3x6y 7x6y x6y) + ( 5xy6 + 11x y6) = 5x6y + 6xy6
4. 5m + (3m 7n) 2n
Antes de proceder a la reduccin de trminos es necesario eliminar
el parntesis; como ste est precedido de un signo +, lo eliminamos
sin hacer cambios y obtenemos:
5m + 3m 7n 2n = 8m 9n

5. 3x2y (x2y 2xy2) + 3x2y


En este caso, al eliminar el parntesis (y el signo que lo precede)
debemos cambiar los signos de los trminos del interior; nos
queda:

3x2y x2y + 2xy2 + 3x2y


(3x2y x2y + 3x2y) + 2xy2 = 5x2y + 2xy2

6. a + a2 + a3 + a4
Aqu no es posible hacer ninguna reduccin pues no existen
trminos semejantes.
Si en una expresin nos encontramos con parntesis dentro de otros
parntesis, procedemos a eliminarlos desde dentro hacia afuera
atendiendo a la misma regla.

lgebra en los nmeros reales 15

14-15. 15 18/11/02, 10:40 AM


CAPTULO 1

lgebra en los nmeros reales 15

16-17. 15 18/11/02, 10:41 AM


7. 2ab [3a (2ab + 3a) ab]
Eliminamos primero el parntesis interior:
2ab [3a + 2ab 3a ab]
Ahora eliminamos el exterior:
2ab 3a 2ab + 3a + ab
(2ab 2ab + ab) + ( 3a + 3a) = ab

Ejercicios

I. Reduzca las siguientes expresiones:

1. m + 2m
2. a + 2a + 9a
3. m2 2m2 7m2
4. 6x2y2 12x2y2 + x2y2
5. 3a 2b 5b + 9a
6. a2 + b2 2b2 3a2 a2 + b2
7. x2yz + 3xy2z 2xyz2 3xy2z + xyz2 x2yz
8. 2pq + 3p 12q 15q + 7pq 13p
9. 2x 6y 2x 3y 5y
10. 15a + 13a 12b 11a 4b b

a a a
11. + +
2 3 4
2
a2b 2ab2 3ab2 6a b
12. +
5 3 2 5
m 2m m
13. m +
2 3 4
3a b 3a b
14. +
2 5
3 3
15. 2p + q 7p + q
4 2
16. a + a2 + a3 + a4 a 2a2 + 3a3 4a4
17. 0,2 m 0,02n + 1,07m 1,03n m n
18. 0,5x2y 0,4xy2 + 0,3x2y 0,2xy2 + x2y
19. 1,17a 2,15a 3,25a + 4,141a

16 lgebra en los nmeros reales

16-17. 16 18/11/02, 10:41 AM


CAPTULO 1

20. 1 + x + xy 2 + 2x 3xy 3 + 2xy 3x


1 2 2 3 3 2 8
21. m n mn m2n + m n mn
5 3 2 10 3
27 35 1 1
22. p q+ p q
4 6 4 6
2 2 2 2
23. u + uv + v 2u + 3uv v
11 3 2 1 5 1
24. s t+ s s s+ t+ t
3 4 3 3 3 4
25. 0,117a 0,35b 2,25b 1,1b + 3,04a
2 3 3 2
26. 10a + 5a 13a 2a 9a + 16a + a
1 2 3 2 3 7 1
27. pt p t + pt p + t + pt
6 5 4 3 5 4 6
2 2 2 2 2 2 2 2
28. x yz xy z + xy z x y z
3 2 2 2 2 2 1 2
29. a b ab a b 3ab + ab
4 3 2
1 3
30. 0,7m p 0,04m + 0,3p p
7 4

II. Elimine parntesis y reduzca los trminos semejantes:

1. a + b + a b

2. a + b + b a

3. a b + a + b

4. a b a + b

5. 2a 2a 3b b

6. 3x + 2y x x y
7. 2m 3n 2m + n m n

8. a + b c a b + c + a b + c
2 2 2 2 2 2 2
9. x y + 2x 3y x 2x 3y

10. a 2b a + 2b a 3b

11. 3x + 2y 2x 3x 2y 3x 2x y

12. 3y 2z 3x x y z x 2x
1 2 3 4
13. a b a b
2 3 4 3

1 1 2
14. a a aa
5 2 3

lgebra en los nmeros reales 17

16-17. 17 18/11/02, 10:41 AM


CAPTULO 1

lgebra en los nmeros reales 17

18-19.(2003) 17 18/11/02, 10:42 AM


Ejercicios

15. 3 x + 2 y x 2y 1 y 2 x
4 5 5 3

16. a b
a b
2 2

17. a b a b + a + b
2 2
1 a b
18. 1 + a + b + +
2 3 4

11 2 3 2 15 2 3 2 1 2 12 2 9 2
19. x y x x y y y
4 25 4 4 25 25 25
20. Si P = x2 + 3x 2 y Q = 2x2 5x + 7, obtenga P + Q.

21. Si P = 3x x2 y Q = 3x2 x, obtenga Q P y P Q.

22. Si M = 2a2 + 3a3 + a4 y N = a4 3a2 + 2a,


obtenga M + N y M N.

23. Si P = x3 5x2 1; Q = 2x2 7x + 3 y R = 3x3 2x + 2,


obtenga P + Q R y P (Q R).

24. Si P = m6 + m3 m; Q = m5 + 2m4 3m3 + 2m


y N = m6 + m5 2m3 + m, obtenga P + Q N y N P.

25. Si A = ab + 2b; B = a ab y C = a + b + ab,


encuentre A + B + C ; A + B C y A (B + C).
a+b ab
26. Si P = y Q= , entonces encuentre el valor de P + Q.
2 2
1 1 2 2 3 2
27. Si P = a b c y Q= a + b + c, encuentre Q P.
2 3 4 3 2 4
28. Si A = 2x3 + 3x2 2x + 5 y A + B = x3 3x2 + x 4, encuentre B.

29. Si A = 3x3 2x2 + 5x 1; B = 2x3 3x 3


y A B + C = x3 2x2 3x 2, encuentre C.

30. Si P = 1 x3; Q = 1 x2; R = 1 x, determine P (Q + R + 3).

Soluciones

I. 1. 3m 2. 12a 3. 8m2 4. 5x2y2 5. 12a 7b 6. 3a2 7. xyz2


13a
8. 9pq 10p 27q 9. 14y 10. 17a 17b 11. 12. a 2b + 5 ab2
12 6
11m 7 9
13. 14. 3a b 15. 5p + q 16. a2 + 4a3 3a4 17. 0,27m 2,05n
12 10 4
10
18. 1,8x2y 0,6xy2 19. 0.089 a 20. 4 21. m2n mn 22. 7p 6q
3
7 1
23. u2 + 4uv 24. s+ t 25. 3,157a 3,7b 26. 9a + 21a2 22a3
3 2
1 2 19 2 33 83
27. pt p + t 28. x2yz x2y2z2 29. a b ab 30. m p
4 6 50 140

18 lgebra en los nmeros reales

18-19.(2003) 18 18/11/02, 10:43 AM


CAPTULO 1

II. 1. 2a 2. 2b 3. 2a 4. 2b 5. 2b 6. 3x + y

7. 5m 5n 8. a b + c 9. 2x2+ y2 10. a 3b 11. 5x + y 12. 4y 3z x


19a 13 11 3a b
13. 1 a + 2 b 14. 15. y x 16. a + b 17. +
4 3 30 5 12 2 2 2 2
1 2 3
18. + a + b 19. 7 x 2 y 2 20. 3x2 2x + 5
2 3 4 4

21. Q P = 4x2 4x 25. A + B + C = 2a + 3b + ab


P Q = 4x 4x2 A + B C = b ab
A (B + C) = ab + b 2a
22. M + N = 2a4 + 3a3 a2 + 2a
26. P + Q = a
M N = 5a2 + 3a3 2a
27. Q P = 1 a + 11 b + c
23. P + Q R = 2x3 3x2 5x 6 6
P (Q R) = 4x3 7x2 + 5x 2 3 2
28. B = x 6x + 3x 9

24. P + Q N = 2m4 29. C = 11x 4


N P = m5 3m3 + 2m 30. P (Q + R + 3) = x3 + x2 + x 4

Multiplicacin algebraica 1.4

Multiplicacin de potencias.
La expresin an se llama potencia de base a y exponente
n. Se cumple:
an am = an + m
(an)m = an m
a0 = 1 con a 0
(ab)n = an bn

Multiplicacin de 2 o ms monomios.
Multiplicamos los coeficientes numricos y los factores literales
entre s (hacemos uso de las propiedades asociativa y conmutativa
de la multiplicacin).
Multiplicacin de un monomio por un polinomio.
Multiplicamos el monomio por cada trmino del polinomio
(hacemos uso de la propiedad distributiva de la multiplicacin
respecto de la adicin).
Multiplicacin de dos polinomios.
Multiplicamos cada trmino del primer polinomio por cada
trmino del segundo. Siempre que sea posible, es necesario reducir
trminos semejantes.

lgebra en los nmeros reales 19

18-19.(2003) 19 18/11/02, 10:43 AM


CAPTULO 1

lgebra en los nmeros reales 19

20-21. 19 18/11/02, 10:45 AM


Ejercicios 1. a6 a7 = a 6 + 7 = a13
resueltos 2. (ab)4 = a4 b4
3. x5 x9 x4 = x5 + 9 + 4 = x18
4. 2a2 3ab = 2 3 a2 a b = 6a3b
5. 5x2 y4 3x6 2y6 = 5 3 2 x2 x6 y4 y6 = 30x8 y10
6. 4a2b (a2 + ab b) = 4a2b a2 4a2b ab 4a2b ( b)
= 4a4b 4a3b2 + 4a2b2

7. (3m5 2m4 mp) 3m = 3m5 ( 3m) 2m4 ( 3m) mp ( 3m)


= 9m6 + 6m5 +3m2p

8. (2x + y) (3x + 2y) = 2x (3x + 2y) + y (3x + 2y)


= 2x 3x + 2x 2y + y 3x + y 2y
= 6x2 + 4xy + 3yx + 2y2
= 6x2 + 7xy + 2y2

(los trminos 4xy y 3yx son semejantes, por lo tanto deben


reducirse).

Ejercicios
Efecte las siguientes
I. operaciones:
14. m2p m

1. a2 a3 15. abc 2abc

2. m3 m4 m5 16. 3x2y x3y6 y

3. x2 x3 x3 17. 4abc 3a2b2 12ab5c7

4. a ab 18. 2pr 3pr5 pr2 7p3r4

5. xy x2y 19. 6x3 6x3

6. a a2b a3b2 20. 2ax4 3ax5 3a2x4

7. 2a ab6 21. an an + 1

8. 3xy2 5x2y3 22. 2am 3an

9. 2m 5n 23. xp + 1 xp 1

10. ax axy 24. p2x p3x 2 px + 9

11. 2x 3xy 2x 25. 2a 2a 3 2a 9

12. 3a2b 5abc c4 26. a2n 3 a3n 2 a2 3n

13. 7abc 2a2bc8 27. a2x 5 bx + 1 a2x + 2 bx 1

20 lgebra en los nmeros reales

20-21. 20 18/11/02, 10:45 AM


CAPTULO 1

28. pa pa + 2 q2a 3 q5 3a 11. 3a6 b2( ab3 + ab + a4b6) 3a7b3(b2 1)

29. ax 4 bx + 4 c2x ax b2x cx + 2 12. 20 abc(a + b c)

30. (ab)5 a4 b2 13. a5b2 a5(a2 ab + b2)


14. 3x6y4(x2 + xy + y2)
31. (mp)3 (mp)2 mp
15. 3b(2ab + b2 + 5bc)
32. (2x)x + 1 (2x)x + 2 (2x)x 3
16. 7a6b8c9(2abc 5a2b + 4ab2c2 abc3)
33. (m2n)5 m5 n6
17. (x6y21 4xy11 9x10y2) 3x6y2
34. (a2)3 (a3)4 a6
1 3 2
18. 2 x 4 x 3 y
35. 2x (2x)6a 2 (2x)3a + 4
1 2 1 3 2
1 3 1 2 19. a ab + ab
36. a a 5a6 3 2 5
2 3
37.
2 4 3 7
b b
4 4
b 20. 3 x 2y 6 2 xy 4 + 4xy 2 1
3 8 3 4 5
6 3 2 15 6 5 21. 8 p2q 1 pq 1 pq3 + 2pq
38. x y x y
5 4 3 4 5
8 6 4 2 2 3 3 2 5 11
39. a b ab c a b c 22. 1 a 2 b3 c 6 abc 8a 2 b2 c 2
9 5 4 8
40. 0,1a6b7c4 0,02abc4 0,1a2b
23. 3 x 6y 2z 4 1 xyz 4 + 2 x 4y 2z 6
41. 0,03a5b4 1,3a4b8 2,7ab6 5 3
3 7 2 6 2 4 2 6 2
42. 0,5xyz4 2,1x2yz 3,1x6 24.
4
m n 14m n mn m n
3 9
43. 1,03a4b 1,3a3b4 2 2 2 2
25. x y x y xy
5
44. 0,06m2n6p2 0,6mn6p4
2 6 12 26. 1 a 6b4c 3 4 ab2 4 a 3b2 1 a
45. a b 3a4b5 0,5a2b4 2 5 7 4
5 27. 0,03a6b2 (1 a2b2 0,03ab3)
28. 0,5m4n2 ( 0,5m6n 2mn3 + 3,5mn3)
II. Monomio por polinomio:
29. 0,07a4b2 (100ab4 10ab3 2ab)
1. 3a (a 2b) 30. 1,2x6y11 (2,1xy9 1,1x2y2 + 2,1xy8)
2. 5x (2 3x2 5x) 31. 0,5abc (a2 b2 c2) + 4,8abc (a2 b2 c2)

3. 7b (2a b) 32. 2,2x6y3z (1,1xyz 1,2x2y2z2 + 3xyz3)

4. 3x2 (3x6 2x4 + x3 2x + 3) 33. 3 p2qr12 3 p2qr3 + 3 pqr6


4 5 4
5. 6x5y3 (3x2y 4xy4 2x2 y2) 2 11 10 2 35 6 2
34. m n p 10m n m n +2
5 8
6. (4xy 5xy4) 6xy
17 8 6 3 6 11 27 6 8
35. x y 1 x y x y
7. (3m2 2mn + n6) 13m4n2 9 4 34
8 2 4 2 3 4 11
8. 15m2np4 (mn6p2 m4n 4p2 + mnp) 36. 3 x y x y xy + y
9. 6m2(2m 5n) 3m(6m2 + 4n) 12 4 2 7 5 2 11 2
37. a b c a bc 10 abc 4ab
10. p2q4(2pq pq3 1) + 3p3q2 (q3 q5 + p2) 5 4

lgebra en los nmeros reales 21

20-21. 21 18/11/02, 10:46 AM


CAPITULO 1

lgebra en los nmeros reales 21

22-23. 21 18/11/02, 10:47 AM


Ejercicios

III.

1. (x + y) ( x2 + y2) 18. (2p 4) (2p + 7)

2. (2a + b ) (3a 2b) 19. (2x 3y 4z) (x + y + z)

3. (1 x) (1 y) 20. (x2 + y2 z2)(2x 3y 4z)

4. (2x 6y) (x2 2xy) 21. (a + 1) (an + an + 1 + an + 2)

5. (x2 + 3x2y) ( 3xy2 + 4xy3) 22. (a 1) (an 1 + an + an + 1)

6. (4x + y) ( 2x 5xy) 23. (u v) (u2 3uv + v2)

7. (6a 5b) (2b + 7a) 24. (x + y) (x 2 + 2xy + y2)

8. (a + b + 1) (a b) 25. ( 3x + y2) (x2 xy y)

9. (2a 3ab + b2) (b b2) 26. (2y + 3x) (x2 xy + 2y2)

10. (5x2y + 2xy2 3xy) (x y2) 27. ( 3x 2y + z) (x + y 3z)

11. (m2 + n2 mn) (2m 3n) 28. (x y) (x2 + xy + y2)

12. ( 3xy 2xy2) (xy2 5xy) 29. (x + y) (x2 xy + y2)

13. (2p2q + 3pq11 5pq4) ( 3pq + 2p) 30. (a + b) (a4 a3b + a2b2 ab3 + b4)

14. (x2 + 1) (x2 1) 31. (a b) (a3 + a2b + ab2 + b3)

15. (a + b) (a b) 32. (x + y) (xn 1 + xn 2 + xn 3)

16. (x + 4) (x 6) 33. (p2 q2) (pn pnqn qn)

17. (a2 + 5) (a2 + 7)

Soluciones

I. 1. a5 2. m12 3. x8 4. a2b 5. x3y2 6. a6b3


7. 2a2b6 8. 15x3y5 9. 10 mn 10. a2x2y 11. 12x3y 12. 15a3b2c5
13. 14a3b2c9 14. m3p 15. 2a2b2c2 16. 3x5y8 17. 144a4b8c8
18. 42p6r12 19. 36x6 20. 18a4x13 21. a2n + 1 22. 6am + n 23. x2p
24. p6x + 7 25. 23a 12 26. a2n 3 27. a4x 3 b2x 28. p2a + 2 q2 a
29. a2x 4b3x + 4c3x + 2 30. a9b7

22 lgebra en los nmeros reales

22-23. 22 18/11/02, 10:47 AM


CAPITULO 1
CAPTULO

5 11
31. m6p6 32. (2x)3x 33. m15n11 34. a24 35. (2x)9a + 3 36. a
6
1 15 4 9 11 14
37. b 38. 9 xx9 y7 39. a b c 40. 0,0002a9 b9 c8 41. 0,1053a10 b18
3 2 15
3 12 21
42. 3,255x9 y2 z5 43. 1,339a7 b5 44. 0,036m3n12 p6 45. a b
5

II. 1. 3a2 6ab 2. 10x + 15x3 + 25x2 3. 14ab 7b2


4. 9x8 6x6 + 3x5 6x3 + 9x2 5. 18x7y4 + 24x6y7 + 12x7y5
6. 24x2y2 + 30x2y5 7. 39m6n2 26m5n3 + 13m4n8
8. 15m3n7p6 + 15m6n5p6 15m3n2p5 9. 6m3 30m2n 12mn
10. 5p3q5 4p3q7 p2q4 + 3p5 q2 11. 3a10b8 12. 20a2bc + 20ab2c 20abc2
13. a7 + a6b 14. 3x8y4 + 3x7y5 + 3x6y6 15. 6ab2 3b3 15b2c
16. 14a7b9c10 35a8b9c9 + 28a7b10c11 7a7b9c12
3 2 1 1 1 2
17. 3x12y23 + 12x7 y13 + 27x16y4 18. x xy 19. a3b a3b
8 3 6 5
3 3 10 3 2 3 2 8 3 4 16 3 2
20. x y + 3x3 y8 x2 y6 21. pq pq + pq
10 4 3 15 3
1 3 4 7 4 5 8 3 6 2 4 3 7 3 8 2 10 4 10
22. a b c +a b c 23. x y z x y z + x y z
8 5 5 5
21 13 3 1 8 6 1 13 4 2 4 2 2 3 3
24. m n + m n + m n 25. xy xy
2 2 6 5 5
2 7 6 3 2 9 6 3 1 7 4 3
26. abc + abc + abc 27. 0,03a b 0,03a8b4 0,0009a7b5
6 2
5 7 8
1 10 3 7
28. m n + m5n5 m7n3 29. 7a5b6 0,7a5b5 0,14a5b3
4 4
30. 2,52x7y20 1,32x8y13 + 2,52x7y19 31. 5,3a3bc 5,3ab3c 5,3abc3
9 4 2 15 9 3 2 18
32. 2,42x7y4z2 + 2,64x8y5z3 6,6x7y4z4 33. pqr + pqr
20 16
13 11 7 17 12 4 11 10 17 8 6 17 14 17 3 14 14
34. 4m n p + m n p m n p 35. xy + x y + x y
4 5 9 12 2
8 4 7 8 3 8 8 2 15 6 3 8 5 3 18 48 5 4 7
36. xy + xy xy 37. 3a b c + 24a b c + a b c
3 3 3 5
III. 1. x3 + xy2 + x2y + y3 2. 6a2 ab 2b2 3. 1 x y + xy
4. 2x3 10x2y + 12xy2 5. 3x3y2 5x3y3 + 12x3y4 6. 8x2 20x2y 2xy 5xy2
7. 42a2 23ab 10b2 8. a2 b2 + a b 9. 2ab 5ab2 + 3ab3 + b3 b4
10. 5x3y 5x2y3 + 2x2y2 2xy4 3x2y + 3xy3 11. 2m3 5m2n + 5mn2 3n3
12. 7x2y3 + 15x2 y2 2x2y4 13. 6p3q2 + 4p3q 9p2q12 + 6p2q11 + 15p2q5 10p2q4
14. x4 1 15. a2 b2 16.x2 2x 24 17. a4 + 12a2 + 35 18. 4p2 + 6p 28
19. 2x2 xy 3y2 2xz 7yz 4z2
20. 2x3 3x2y 4x2z + 2xy2 3y3 4y2z 2xz2 + 3yz2 + 4z3
21. an + 2an+1 + 2an+2 + an+3 22. an+2 an1 23. u3 4u2v + 4uv2 v3
24. x3 + 3x2y + 3xy2 + y3 25 . 3x3 + 3x2y + 3xy + x2y2 xy3 y3
26. 3x3 x2y + 4xy2 + 4y3 27. 3x2 5xy + 10xz 2y2 + 7yz 3z2 28. x3 y3
29. x3 + y3 30. a5 + b5 31. a4 b4 32. xn + xn1 + xn2 + yxn1 + yxn-2 + yxn3
33. pn+2 pn+2qn p2qn q2pn + qn+2pn + qn+2

lgebra en los nmeros reales 23

22-23. 23 18/11/02, 10:49 AM


1.5 Productos notables
Dentro de la multiplicacin algebraica existen algunos productos
que pueden ser desarrollados en forma directa, es decir, sin multiplicar
trmino a trmino primero, y luego reducir. stos son:
Cuadrado de un binomio.
El desarrollo de este producto corresponde al cuadrado del primer
trmino, ms (o menos) el doble del producto del primer trmino por el
segundo y ms el cuadrado del segundo, es decir:
(a b)2 = a2 2ab + b2
Suma por diferencia.
Es igual a la diferencia de los cuadrados de los trminos, es decir:
(a + b) (a b) = a2 b2
Producto de binomios con un trmino comn.
Es el cuadrado del trmino comn ms el producto del trmino
comn por la suma de los trminos no comunes y ms el producto de
los trminos no comunes, o sea:
(x + a) (x + b) = x2 + x (a + b) + ab
Cubo de un binomio.
Corresponde al cubo del primer trmino, ms (o menos) el triple
del cuadrado del primer trmino multiplicado por el segundo, ms el
triple del primer trmino multiplicado por el cuadrado del segundo y
ms (o menos) el cubo del segundo. As:
(a b)3 = a3 3a2b + 3ab2 b3
Para obtener otras potencias de un binomio podemos determinar
los coeficientes mediante el TRINGULO DE PASCAL, que se obtiene
de la siguiente manera:
Comienza y termina con 1.
Cada coeficiente se obtiene sumando los dos correspondientes
segn el orden en la fila anterior.
La primera fila corresponde a los coeficientes de (a + b)0
La segunda fila corresponde a los coeficientes de (a + b)1
La tercera fila corresponde a los coeficientes de (a + b)2
As, la fila n-sima nos entrega los coeficientes de (a + b)n 1.
Los factores literales se obtienen de la siguiente manera:
En (a + b)n debe haber (n + 1) trminos.
El primer factor literal es an ; el segundo es an 1 b1 ; el tercero
es an 2 b2 y as sucesivamente. El grado del trmino a decrece a
medida que el grado de b aumenta hasta terminar en bn.
(Cada trmino se forma con el coeficiente numrico obtenido
del tringulo de Pascal y el factor literal sealado ms arriba).
24 lgebra en los nmeros reales

24-25.(2003) 24 18/11/02, 10:52 AM


CAPTULO 1

Representacin geomtrica de expresiones algebraicas.


a) La expresin ab representa el rea del c) Observemos el producto de una
rectngulo de lados a y b. suma por su diferencia:
D C
ab a D a I b C

A B
b ab
b) Observemos el cuadrado del binomio
(a+b)2 = a2 + 2ab + b2 a
A ab G b
D K C H B
b b2

a a2 ab E F J

H I A(ABCD) = (a+b) (ab)


b a b b2 Tenemos A(EFGA) = A(HBCI)

A a J b B \ A(ABCD) = A(EFGHIDA) que es a2 b2

1. (2 + x)2 = 22 + 2 2 x + x2 Ejercicios
= 4 + 4x + x2
resueltos
2. (3a 5b)2 = (3a)2 2 3a 5b + (5b)2
= 9a2 30ab + 25b2
3. (2x y) (2x + y) = (2x)2 y2
= 4x2 y2
a a a 2 2
4. + 5y 5y = 5y
2 2 2
2
a 2
= 25y
4
5. (x + 8) (x + 5) = x2 + (5 + 8)x + 5 8
= x2 + 13x + 40
6. (2a + 3) (2a 7) = (2a)2 + (3 7) 2a + 3 7
= 4a2 4 2a 21
= 4a2 8a 21
7. (p + 2)3 = p3 + 3 p2 2 + 3 p 22 + 23
= p3 + 6p2 + 3p 4 + 8
= p3 + 6p2 + 12p + 8
8. (2t r)3 = (2t)3 3(2t)2 r + 3(2t) r2 r3
= 8t3 3 4t2 r + 6t r2 r3
= 8t3 12t2r + 6tr2 r3
9. (a + b)4 = 1a4 + 4a3b + 6a2b2 + 4ab3 + 1b4
a4 + 4a3b + 6a2b2 + 4ab3 + b4
10. (2a + y)5 = 1(2a)5 + 5(2a)4 y + 10 (2a)3 y2 + 10(2a)2 y3 + 5(2a)y4 + 1 y5
= (2a)5 + 5 16a4y + 10 8a3y2 + 10 4a2y3 + 10ay4 + y5
= 32a5 + 80a4y + 80a3y2 + 40a2y3 + 10ay4 + y5

lgebra en los nmeros reales 25

24-25.(2003) 25 18/11/02, 10:52 AM


Ejercicios

I. Cuadrado de binomio.

1. (x + y)2 14. (4pq 3q)2

2. (p q)2 15. (9x2 7y2)2

3. (2p + q)2 16. (8a2b + 7ab6)2

4. (3a + b)2 17. (15x2y 3xy2z6)2

5. (2a 3b)2 18. (2a 3b)2 + (3a 5b)2

6. (x + 1)2 19. (11x 5y)2 (13x + 3y)2 + (x 2y)2


2 2
7. (a 6)2 a b
20. + 2b + 2a
2 2
8. (x + 9)2 21. 3a b
2

5
9. (3p 1)2
2
22. 2 2 3
10. (x + 5)2 x yz
3 5
11. (6x 5y)2 23. (0,1a2 0,2abc)2
12. (2m 1)2 24. (1,5xy2 + 2,5x2y)2
13. (6x2y + 2x)2 25. 3 2 3 3 2
a b ab6
4 5

II. Suma por diferencia.

1. (u v) (u + v)
14. (a + 5x) (a 5x)
2. (x + 2y) (x 2y)
15. ( 9x2 + 5xy) ( 9x2 5xy)
3. (3a b) (3a + b)
16. (13n5p2 + 1) (13n5p2 + 1)
4. (5x2 3y) (5x2 + 3y)
17. (1 a) (1 + a) (1 2a) (1 + 2a)
5. (2x 3xy) (2x + 3xy)
18. (x2 2xy) (x2 + 2xy) + (x2 + 2xy)2
6. (6a + 1) (6a 1)
19. (1 w5) (1 + w5)
7. (9m2 3n) (9m2 + 3n)
20. 3 7 2 4 3 7 2 4
p q p + q
4 5 4 5
8. ( 4a2b + 5b) (4a2b + 5b)
21. abc abc
9. ( 6m2n3 7m) ( 6m2n3 + 7m) + 4x 4x
2x 2x

10. (10a2 1) (10a2 + 1) 22. (0,05x12 2) (0,05x12 + 2)

11. b2 1 b2 +
1 23. (6x5y2z3 1) (6x5y2z3 + 1)
2 2
q q
12. 2a 2a 24. 2p + 2p
5b + 5b 4 4
3 3
13. (2a + b) (2a b) (2a + b)2 25. (0,3x2y 2z) (0,3x2y + 2z)

26 lgebra en los nmeros reales

26-27. 26 18/11/02, 10:54 AM


CAPTULO 1

III. Producto de binomios con trmino comn.

1. (a + 2) (a + 3) 10. (x + 6) (x 2) 19. (3a2 2b) (3a2 5b)


2. (x + 5) (x + 4) 11. (x 3) (x 8)
20. (9a 4) (9a + 11)
3. (t + 2 ) (t 3) 12. (x 13) (x + 2)
21. (6x2 2y) (6x2 7y)
4. (a + 5 ) (a 9) 13. (a 7) (a + 12)
5. (x 8) (x 1) 22. (4a2b 3a) (4a2b + 9a)
14. (x2 + 5) (x2 + 3)
a a
6. (a 7) (a 9) 15. (a2 3) (a2 + 4) 23. 2b 6b
4 4
7. (x + 2) (x 12) 16. (2b + 5) (2b + 9) 3a 3a
24. 5b + 8b
8. (x + 3) (x + 8) 17. (6x 3) (6x + 5) 5 5
3p 3p
9. (x 4) (x 6) 18. (2a + 3b) (2a + 5b) 25. + 3q +q
4 4

IV. Cubo de un binomio.


3
1
1. (a + b)3 10. (1 3y)3 19. a
2
3
2. (p q)3 11. (2 + 3t)3 1
20. x + 2y
2
3. (x + 2)3 12. (3a 2x)3 3
2 1
21. a b
4. (a 3)3 13. (5a 1)3 3 3
3
5. (t + 4)3 14. (3a2 2a)3 22. 5 p + 3 q
2 2
6. (2 a)3 15. (t2 + t3)3 1 1 3
23. m n
10 5
7. (2a b)3 16. (1 + x4)3
3
24. a a
8. (3a 5b)3 17. (2t 3a2)3 3
3
9. (2x + 3y)3 18. (u2 + 5v)3 25. 1 t + 2t 2
2

V. Otras potencias de binomios.


5
1. (2a + b)4 5. (3a + 2)6 8. 1
+a
2
2. (x 2y)5 x y 44 2a 4
6. + 9. 3a
3. (a + b)6 2 2 3

4. (2a 1)7 7. (3a + 4)4 10. (x + 1)5

VI. Representacin geomtrica de expresiones algebraicas.


Investigar de qu manera se pueden representar como suma o resta de reas los
siguientes productos.
3. (xa) (x+b) = x2 + (ba)x ab

1. (ab)2 = a2 2ab + b2 4. (xa) (xb) = x2 (a+b)x + ab


2. (x+a) (x+b) = x2 + (a+b)x + ab 5. (a+b+c)2 = a2 + b2 + c2 + 2ab + 2bc + 2ac

lgebra en los nmeros reales 27

26-27. 27 18/11/02, 10:55 AM


Soluciones

I. 1. x2 + 2xy + y2 2. p2 2pq + q2 3. 4p2 + 4pq + q2 4. 9a2 + 6ab + b2


5. 4a2 12ab + 9b2 6. x2 + 2x + 1 7. a2 12a + 36 8. x2 + 18x + 81
9. 9p2 6p + 1 10. x2 + 10x + 25 11. 36x2 60xy + 25y2
12. 4m2 4m + 1 13. 36x4y2 + 24x3y + 4x2 14. 16p2q2 24pq2 + 9q2
15. 81x4 126x2y2 + 49y4 16. 64a4b2 + 112a3b7+ 49a2b12
17. 225x4y2 90x3y3z6 + 9x2y4z12 18. 13a2 42ab + 34b2
2 2 2
2 6 b
19. 47x2 192xy + 20y2 20. 17a + 17b 21. 9a ab +
4 4 5 25
4 4 4 2 9 2 2
22. x x yz + y z 23. 0,01a4 0,04a3bc + 0,04a2b2c2
9 5 25
9 4 6 9 3 9 9 2 12
24. 2,25x2y4 + 7,5x3y3 + 6,25x4y2 25. a b a b + a b
16 10 25

II. 1. u2 v2 2. x2 4y2 3. 9a2 b2 4. 25x4 9y2 5. 4x2 9x2y2


6. 36a2 1 7. 81m4 9n2 8. 25b2 16a4b2 9. 36m4n6 49m2

10. 100a4 1 11. b4


1 12. 4a2 2 13. 4ab 2b2 14. a2 25x2
4 25b
9
15. 81x4 25x2y2 16. 1 169n10p4 17. 3a2 18. 2x4 + 4x3y 19. 1 w10
2 2 2
a b c
20. 9 p14 4 q8 21. 2
16x2 22. 0,0025 x24 4 23. 36x10y4z6 1
16 2
25 4x
2 q
24. 4p 25. 0,09 x4y2 4z2
16

III. 1. a2 + 5a + 6 2. x2 + 9x + 20 3. t2 t 6 4. a2 4a 45
5. x2 9x + 8 6. a2 16a + 63 7. x2 10x 24 8. x2 + 11x + 24
9. x2 10x + 24 10. x2 + 4x 12 11. x2 11x + 24 12. x2 11x 26
13. a2 + 5a 84 14. x4 + 8x2 + 15 15. a4 + a2 12 16. 4b2 + 28b + 45
17. 36x2 + 12x 15 18. 4a2 + 16ab + 15b2 19. 9a4 21a2b + 10b2
20. 81a2 + 63a 44 21. 36x4 54x2y + 14y2 22. 16a4b2 + 24a3b 27a2
2 2
2 9a 9ab 9p
23. a 2ab + 12b2 24. + 40b2 25. + 3pq + 3q2
16 25 5 16
IV. 1. a3 + 3a2b + 3ab2 + b3 2. p3 3p2q + 3pq2 q3 3. x3 + 6x2 + 12x + 8
4. a3 9a2 + 27a 27 5. t3 + 12t2 + 48t + 64 6. 8 12a + 6a2 a3
7. 8a3 12a2b + 6ab2 b3 8. 27a3 135a2b + 225ab2 125b3
9. 8x3 + 36x2y + 54xy2 + 27y3 10. 1 9y + 27y2 27y3
11. 8 + 36t + 54t2 + 27t3 12. 27a3 54a2x + 36ax2 8x3

28 lgebra en los nmeros reales

28-29. 28 18/11/02, 10:57 AM


CAPITULO 1
CAPTULO

13. 125a3 75a2 + 15a 1 14. 27a6 54a5 + 36a4 8a3

15. t6 + 3t7 + 3t8 + t9 16. 1 + 3x4 + 3x8 + x12 17. 8t3 36t2a2 + 54ta4 27a6

18. u6 + 15u4v + 75u2v2 + 125v3 19. 1 3 a + 3 a2 a3


8 4 2
1 3 3 2 8 3 4 2 2 1 3
20. x + x y + 6xy2 + 8y3 21. a a b+ ab2 b
8 2 27 9 9 27
125 3 225 2 135 2 27 3 1 3 3 1 3
22. p + p q+ pq + q 23. m3 m2 n + mn2 n
8 8 8 8 1.000 500 250 125
8 3 1 3
24.
27
a 25. t3 + t 4 + 6 t 5 + 8 t6
8 2

V. 1. 16a4 + 32a3b + 24a2b2 + 8ab3 + b4

2. x5 10x4y + 40x3y2 80x2y3 + 80xy4 32y5

3. a6 + 6a5b + 15a4b2 + 20a3b3 + 15a2b4 + 6ab5 + b6

4. 128a7 448a6 + 672a5 560a4 + 280a3 84a2 + 14a 1

5. 729a6 + 2.916a5 + 4.860a4 + 4.320a3 + 2.160a2 + 576a + 64

x4 x3 y 3x2 y2 xy3 y4
6. + + + + 7. 81a4 + 432a3 + 864a2 + 768a + 256
16 4 8 4 16
1 5 5 2 5 3 5 4 2.401 4
8. + a+ a + a + a + a5 9. a
32 16 4 2 2 81
10. x5 + 5x4 + 10x3 + 10x2 + 5x + 1

Factorizacin 1.6

Definicin: Factorizar una expresin algebraica (o suma


de trminos algebraicos) consiste en escribirla en forma de
multiplicacin. Veremos los siguientes casos:

1.6.1 Factor comn


(monomio y polinomio)
Aqu, todos los trminos de la expresin presentan un factor
comn, que puede ser un monomio o un polinomio, por el
cual se factoriza, es decir, el trmino comn es uno de los
factores de la multiplicacin. El otro se determina aplicando la
multiplicacin algebraica.

lgebra en los nmeros reales 29

28-29. 29 18/11/02, 10:57 AM


CAPTULO 1

lgebra en los nmeros reales 29

30-31. 29 18/11/02, 10:58 AM


Ejercicios 1. Factoricemos la expresin 2a + 6a2
resueltos Vemos que el trmino 2a est contenido en ambos trminos del
binomio que queremos factorizar; por lo tanto, 2a es el factor
comn y escribimos 2a + 6a2 = 2a (1 + 3a).
El segundo factor se obtiene buscando los trminos por los cuales
hay que multiplicar el factor comn (2a) para obtener los trminos
de la expresin original.

2. Factoricemos la expresin
6xy2 15x2 y + 21x2 y2
El coeficiente numrico contenido en los tres trminos de la
expresin es el tres y el factor literal es xy; por lo tanto, el factor
comn es 3xy. Y escribimos:
6xy2 15x2y + 21x2y2 = 3xy (2y 5x + 7xy).

3. Factoricemos la expresin
6 2 3
5a 10a 20a
2
4
3b 21b 9b

El trmino o factor comn de los numeradores es 5a2 y el de


los denominadores es 3b; por lo tanto, el factor comn de la
2
expresin es: 5a y escribimos:
3b
6 2 3 2
5a 10a 20a 5a a4 2 4a
=
3b
2
21b 9b
4
3b b 7 3b3

4. Factoricemos la expresin m (2a + b) 3n (2a + b).


Aqu podemos considerar el parntesis (2a + b) como un solo
trmino y podemos factorizar por l. Entonces nos queda:
m (2a + b) 3n (2a + b) = (2a + b) (m 3n)

5. Factoricemos la expresin a (p q) p + q
Aqu no encontramos un trmino comn en forma inmediata, pero
podemos hacer una asociacin adecuada y nos queda:

a (p q) p + q = a (p q) (p q)
= (p q) (a 1)

Observacin 1: El proceso est completo si no es posible seguir


factorizando dentro de los parntesis (o factores) obtenidos.

Observacin 2: Por la propiedad conmutativa de la multiplicacin no


importa el orden en que se entregue el resultado.

30 lgebra en los nmeros reales

30-31. 30 18/11/02, 10:59 AM


CAPTULO 1

Ejercicios
Factorice las siguientes expresiones: 30. 3a + 12a 21a
2 3
b b b
1. m2 + 3m 2 2 3 3
31. p q + pq + p q
2. a2 + ab 2ab 2ac 2abc
5 4 3
3. 3a 12ab 32. c c c
5 10 15
4. a2b2 + a3b3 ab
2 2 3 3 2 2
33. a b + a b2 a b3
5. 2pq2 3p2q x x x
20 10 5
6. 6x2y5 12x2y6 18x3y4 34.
m
+
m

m
20 10 5
7. 2ab + 2ac + 2ad 2
35. p q + 2pq 2

8. 26x2y6 13x6y2 36. 3 (a 2) a (a 2)


9. x2y2 xy 37. a (x + 4) + b (x + 4) + c (x + 4)
10. 21a6 14a5 + 56a7
38. x (z2 + a2) + 2 (z2 + a2)
11. a + a2 + a3 + a4
39. m (a c) + a c
12. 3a2b 6a 3 b 12ab 3
40. m (a c) a + c
13. 15mn 10m
41. a (x2 + y2 + z2) x2 y2 z2
14. 2q + 2q2 + 2q6
42. 2a b + 3a (2a b)
15. 10q5 30pq5 15pq6
43. a + ax + ax2
16. 18gh5 4g2h2 8g3h3
44. c (3 5c) 2d (3 5c)
17. 7y6x2 35yx4 28y4 2 2 2 2
a +c a +c 2 2
45. a c
18. 2 2x 2b 2q
46. 3x (2x y) 2x + y
19. a + a2
47. (a + b) (a + c) (a + b ) (a + d)
20. a6 7a5 5a4
48. (1 + a) (x y) (x y)2
21. 4m5r6 6m4r5 16m5r3
49. (a2 + 6) (a2 + b) + a (a2 + b)
22. a2b2c6 a3b5c2 + a7b3c2
50. (2 + a + c) (a c) + (2 + a + c) (b d)
23. x2 x2y2 x2y3 + x2y4
51. x2 + y2 + z2 + 2a (x2 + y2 + z2)
24. 2xyz 2xy
52. a (b + x) + b (b + x) + c (b + x)
25. 6a + 36a6
2 4 16
53. a ab abc
26. t9 + t8 + t5 15 5 25
54. m (x + y z) n (x + y z) p (x + y z)
27. 12ab6 12ab5

28. x6y9z12 + x6y8z6 + x5y8z10 55. 3 a 2 b 3 a 2 b2 3 a 2 b3


4 2 8
2 3 4 2 2
a a a 56. x + y x 2 y 2
29.
2 2 2 9a

lgebra en los nmeros reales 31

30-31. 31 18/11/02, 10:59 AM


CAPTULO 1

lgebra en los nmeros reales 31

32-33. 31 18/11/02, 11:00 AM


Soluciones

1. m (m + 3) 22. a2b2c2 (c4 ab3 + a5b) 37. (x + 4) (a + b + c)


2. a (a + b) 23. x2 (1 y2 y3 + y4) 38. (x + 2) (z2 + a2)
3. 3a (1 4b) 24. 2xy (z 1) 39. (a c) (m + 1)
4. ab (ab + a2b2 1) 25. 6a (1 + 6a5) 40. (a c) (m 1)
5. pq (2q 3p) 26. t5 (t4 + t3 + 1) 41. (x2 + y2 + z2) (a 1)
6. 6x2y4 (y 2y2 3x) 27. 12ab5 (b 1) 42. (1 + 3a) (2a b)
7. 2a (b + c + d) 28. x5y8z6 (xyz6 + x + z4) 43. a (1 + x + x2)
8. 13x2y2 (2y4 x4) 44. (3 5c) (c 2d)
2 1 1
9. xy (xy 1) 29. a 1 a a2 45. a2 + c2 1
2 2b 2q
10. 7a5 (3a 2 + 8a2)
46. (2x y) (3x 1)
11. a (1 + a + a2 + a3) 30. 3a 1 + 4 7 47. (a + b) (c d)
12. 3ab (a 2a2 4b2) b b b2
48. (x y) (1 + a x + y)
2 2
13. 5m (3n 2) 31. pq pq + 1 + p q 49. (a2 + b) (a2 + 6 + a)
2a b c bc
14. 2q (1 + q + q5)
3
50. (2 + a + c) (a c + b d)
15. 5q5 (2 6p 3pq) 32. c c2 c 1
5 2 3 51. (x2 + y2 + z2) (1 + 2a)
16. 2gh2 (9h3 2g 4g2h) 52. (b + x) (a + b + c)
a2 b2 ab 1
17. 7y (y5x2 5x4 4y3) 33.
x 1 + 2 2 1 8bc
x x 53. a 2b
5 3 5
18. 2 (1 x)
m5 m15 m5 54. (x + y z) (m n p)
19. a (1 + a) 34. + 1
5 4 2 3 2 1 1 2
55. a b b b
20. a4 (a2 7a 5) 35. pq (p + 2q) 2 2 4
21. 2m4r3 (2mr3 3r2 8m) 36. (a 2) (3 a) 1
56. x2 + y2 1
9a

1.6.2 Factor comn compuesto


Muchas veces, no todos los trminos de una expresin algebraica
contienen un factor comn, pero haciendo una adecuada agrupacin
de ellos podemos encontrar factores comunes de cada grupo. Veremos,
con ejemplos, cmo procederemos en estos casos.

Ejercicios 1. Factoricemos: ac + ad + bc + bd
resueltos Si observamos, vemos que el primer y el segundo trmino tienen el
factor comn a y el tercer y el cuarto trmino tienen b como factor
comn. Asociamos y factorizamos por parte:
ac + ad + bc + bd = (ac + ad) + (bc + bd)
= a(c + d) + b(c + d)

32 lgebra en los nmeros reales

32-33. 32 18/11/02, 11:01 AM


CAPTULO 1

Ahora nos queda (c + d) como factor comn, por lo tanto, la


expresin original queda factorizada como sigue:
ac + ad + bc + bd = (c + d) (a + b)
2. Factoricemos: ax + bx + cx ay by cy
Aqu podemos asociar el primer y el cuarto trmino, el segundo y
el quinto, el tercero y el sexto y nos queda:
ax + bx + cx ay by cy = (ax ay) + (bx by) + (cx cy)
= a(x y) + b(x y) + c(x y)
= (a + b + c) (x y)
3. Factoricemos: ax + bx + cx + ay + by + cy az bz cz
Asociemos en el orden natural los tres primeros, los tres siguientes
y los tres ltimos:
ax + bx + cx + ay + by + cy az bz cz
= (ax + bx + cx) + (ay + by + cy) (az + bz + cz)
= x(a + b + c) + y (a + b + c) z(a + b + c)
= (a + b + c) (x + y z)
Observacin: La forma de asociar no es nica, pero la factorizacin
s lo es.
En el primer ejemplo podramos haber asociado el primer y
el tercer trmino y el segundo con el cuarto y el resultado habra
sido el mismo.

Ejercicios
Factorice las siguientes expresiones:

1. ac + ad + bc + bd 12. 3 + 15z + 4y + 20yz


2. ax ay + bx by + cx cy 13. a2c2 + a2d2 + b2c2 + b2d2
3. pc + qc + pd + qd 14. 3ax3 2bx3 3ay3 + 2by3
4. rt + rv st sv 15. 1 + b + a + ab
5. 2ac ad + 2bc bd 16. a2x2y2 + b2x2y2 2a2 2b2
6. xu xv yu + yv 17. abc 2abcz xy + 2xyz
7. 2au + 2av 3bu 3bv 18. bd 3bf + 2cd 6cf
8. 3a2x + 3a2y + b2x + b2y 19. xp + 2xq 2yp 4yq + 4zp + 8zq
9. 2ac 2ad + 3bc 3bd 20. 4 + 2c + 2d + 2a + ac + ad + 2b + bc + bd
10. x + y + ax + ay 21. a2x2 + x2y2 x2b + a2y2 + y4 y2b a2 y2 + b
11. 2a 2b + ax bx

lgebra en los nmeros reales 33

32-33. 33 18/11/02, 11:01 AM


CAPITULO 1

lgebra en los nmeros reales 33

34-35. 33 18/11/02, 11:02 AM


Ejercicios
22. a2x2 + b2x2 + c2x2 + a2y2 + b2y2 + c2y2 29. p4 + p2q2 + p2r2 + 2p2q + 2q3 + 2qr2 + p2r + q2r + r3
23. 12ac 6ad 2bc + bd 30. ax bx cx + 2ay 2by 2cy az + bz + cz
24. aq ar + bq br 31. a2u a2v + b2u b2v + u v
25. u + au v av w aw 32. 4 2a 2b + 2x ax bx + 2y ay by
26. 2ax 2ay bx + by 33. x2y2w2 x2y2z2 xyw2 + xyz2
27. 3am2 3at2 5b2m2 + 5b2t2 34. ax + 2bx + 3cx ay 2by 3cy
28. x y + 2ax 2ay + 3bx 3by 35. 2ax + 2bx ay by az bz

Soluciones

1. (a + b) (c + d) 18. (b + 2c) (d 3f)


2. (a + b + c) (x y) 19. (x 2y + 4z) (p + 2q)
3. (p + q) (c + d) 20. (2 + a + b) (2 + c + d)
4. (r s) (t + v) 21. (x2 + y2 1) (a2 + y2 b)
5. (a + b) (2c d) 22. (x2 + y2) (a2 + b2 + c2)
6. (x y) (u v) 23. (6a b) (2c d)
7. (2a 3b) (u + v) 24. (a + b) (q r)
8. (3a2 + b2) (x + y) 25. (u v w) (1 + a)
9. (2a + 3b) (c d) 26. (2a b) (x y)
10. (1 + a) (x + y) 2 2 2
* 27. (3a 5b ) (m t )
11. (2 + x) (a b) 28. (1 + 2a + 3b) (x y)
12. (3 + 4y) (1 + 5z) 29. (p2 + 2q + r) (p2 + q2 + r2)
13. (a2 + b2) (c2 + d2) 30. (x + 2y z) (a b c)
3 3 31. (a2 + b2 + 1) (u v)
* 14. (x y ) (3a 2b)
15. (1 + a) (1 + b) 32. (2 + x + y) (2 a b)
16. (x2y2 2) (a2 + b2) 2 2
* 33. xy (xy 1) (w z )
17. (abc xy) (1 2z) 34. (a + 2b + 3c) (x y)
35. (2x y z) (a + b)
NOTA:
Los ejercicios sealados con * son posibles de factorizar an ms con los mtodos
que veremos a continuacin.

1.6.3 Diferencia de cuadrados


Recordemos que el producto de una suma de dos trminos por
su diferencia es igual a la diferencia de los cuadrados de ambos
trminos.
Aplicamos este resultado en las factorizaciones siguientes:

34 lgebra en los nmeros reales

34-35. 34 18/11/02, 11:02 AM


CAPITULO 1
CAPTULO

1. Factoricemos a2 b2 Ejercicios
Observamos que a2 y b2 son los cuadrados de a y b, resueltos
respectivamente.
As: a2 b2 = (a + b) (a b)
2. Factoricemos 9m2 16p2
9m2 es el cuadrado de 3m y 16p2 es el cuadrado de 4p.
Entonces : 9m2 16p2 = (3m + 4p) (3m 4p)

1 25
3. Factoricemos 2
2
a 4b
Usando el mismo razonamiento anterior vemos que la expresin se
1 25 1 5 1 5
factoriza: = +
a 2 4b2 a 2b a 2b
4. Factoricemos 6a2 24m4
En este ejemplo podemos factorizar primero por 6 (factor comn
monomio).
6a2 24m4 = 6 (a2 4m4)
y ahora, el trmino (a2 4m4) es exactamente una diferencia de
cuadrados y por lo tanto la factorizacin correspondiente es:
6a2 24m4 = 6 (a2 4m4)
= 6 (a 2m2) (a + 2m2)
Observacin: No es importante el orden en que uno presente los
factores, puesto que la multiplicacin es conmutativa, es decir:
(a + b) (a b) = (a b) (a + b)

Ejercicios
9. a2b2 c2d2 20. x2a y2b
Factorice las siguientes
10. 1 x10 21. m2an2b 1
expresiones:
11. b6 + a4 22. 25n16 16m4
1. x2 y2 12. 1 + a2 23. 40 90a4
2. a2 4b2 13. a5 a3 24. 24m2 + 54n12
3. 9m2 16n2 14. 8a4 2b2 25. m6n4p12 a2b2c2
4. 9a2 25p2 15. p2q3 q 26. 2x2 8y2z6
5. x2 0,01y2 16. 49a2b4c6 121m6n10 27. a10 100b10
6. 100a2 64b6 17. 12a6 75b8 28. 144b10 121c6
7. m2n2 p2 18. 45m6 80p8 29. 81c4 9d4
8. m4n6 z2 19. 27x4 48y2 30. 225 a2

lgebra en los nmeros reales 35

34-35. 35 18/11/02, 11:02 AM


CAPTULO 1

lgebra en los nmeros reales 35

36-37.(2003) 35 18/11/02, 11:03 AM


Ejercicios 1
37. 12 12 44. a12
a b 9b6
1 8a
4
2 4 25
31. 121 + 2 38. 45.
2 6 6
y 45b 5 x y
32. 64a2b4c6 + x8y2 39. 32m10 18p4q6 46. a2 b2 2a 2b
33. 16x4 4y16 40.
1
a b
2 2
47. p2 q2 rp + rq
2 2
a b
34. 1 25
2 2
2 2 41. x2 y2 ax + ay 48. a 2 + ac b2 bc
4a b 9x y
4 1
35. 24x8 6 42. 25x 49. m2 n2 pm pn
25
6 2
36. 75m 27n 12 10
m n 50. qr2 q3s2
4 25 43.
2
4
c d

Soluciones

1.(x + y) (x y) 2. (a + 2b) (a 2b) 3. (3m + 4n) (3m 4n)


4.(3a 5p) (3a + 5p) 5. (x 0,1y) (x + 0,1y) 6. (10a 8b3) (10a + 8b3)
7.(mn + p) (mn p) 8. (m2n3 z) (m2n3 + z) 9. (ab cd) (ab + cd)
10. (1 x5) (1 + x5) 11. (a2 b3) (a2 + b3) 12. (a 1) (a + 1)
13. a3(a 1) (a + 1) 14. 2 (2a2 b) (2a2 + b) 15. q (pq 1) (pq + 1)
16. (7ab2c3 11m3n5) (7ab2c3 + 11m3n5) 17. 3 (2a3 5b4) (2a3 + 5b4)
18. 5 (3m3 4p4) (3m3 + 4p4) 19. 3 (3x2 4y) (3x2 + 4y)
20. (xa yb) (xa + yb) 21. (manb 1) (manb + 1)
22. (5n8 4m2) (5n8 + 4m2) 23. 10 (2 3a2) (2 + 3a2)
24. 6 (3n6 2m) (3n6 + 2m) 25. (m3n2p6 abc) (m3n2p6 + abc)
26. 2 (x 2yz3) (x + 2yz3) 27. (a5 10b5) (a5 + 10b5)
28. (12b5 11c3) (12b5 + 11c3) 29. 9 (3c2 d2) (3c2 + d2)
1 1
30. (15 a) (15 + a) 31. y + 11 y 11
32. (x4y 8ab2c3) (x4y + 8ab2c3) 33. 4 (2x2 y8) (2x2 + y8).

1 5 1 5 5m3 3n 5m3 3n
34. + 35. 6 (2x4 1) (2x4 + 1) 36. 3 +
2ab 3xy 2ab 3xy 2 5 2 5
2 2a2
37. 1

1 1
+
1 38. 2 2a 1 +1 39. 2 (4m5 3p2q3) (4m5 + 3p2q3)
a b a b 5 3b 3b
1 1
40. ab + ab 41. (x y) (x + y a) 42. 5x2 1 5x2 +
1
ab ab 5 5
m6 n5 m6 n5 2 5 2 5
43. + 44. a6 + 1 a6
1 45. 3 + 3
2
d2 3 x3 y x3 y
c d c 3b 3b3
46. (a + b) (a b 2) 47. (p q) (p + q r) 48. (a b) (a+ b + c)
49. (m + n) (m n p) 50. q (r qs) (r + qs)

36 lgebra en los nmeros reales

36-37.(2003) 36 18/11/02, 11:05 AM


CAPTULO 1

1.6.4 Trinomios ordenados

Definicin: Llamamos trinomio ordenado (segn el grado) a


una expresin de la forma ax2 + bx + c, donde a, b, c, y x
representan nmeros reales.

En general, los trinomios pueden proceder:


de la multiplicacin de un binomio por s mismo (o un cuadrado
de binomio); por ejemplo:
(a + 7)2 = a2 + 14a + 49
de la multiplicacin de dos binomios con un trmino comn;
por ejemplo:
(a + 2) (a + 6) = a2 + 8a + 12
o de la multiplicacin de dos binomios de trminos semejantes:
(2x + 1) (x + 2) = 2x2 + 5x + 2
Con estas consideraciones, resolvamos los ejercicios presentados
a continuacin:

1. Factoricemos x2 + 10x + 25 Ejercicios


Observamos que el primer trmino (x2) y el ltimo (25) son los resueltos
cuadrados de x y 5, respectivamente, y adems el trmino central
(10x) corresponde al doble del producto de x y 5; entonces la
expresin es un cuadrado de binomio y as:
x2 + 10x + 25 = (x + 5)2
2. Factoricemos a2 8a + 16
Usando el mismo razonamiento anterior, observamos que el trinomio
corresponde al cuadrado del binomio (a 4) y escribimos:
a2 8a + 16 = (a 4)2
El signo del trmino central del trinomio indica el signo que corresponde
al segundo trmino del binomio.
3. Factoricemos y2 + 13y + 36
Aqu vemos que tanto el primer trmino como el tercero corresponden
a cuadrados exactos (de y y de 6, respectivamente), pero el trmino
central (13y) no corresponde al doble del producto entre y y 6 (es
decir, a 12y); en este caso, el trinomio puede corresponder al producto
de dos binomios con un trmino comn, que sera y.
Buscamos entonces dos nmeros cuyo producto sea igual a 36 (el

lgebra en los nmeros reales 37

36-37.(2003) 37 18/11/02, 11:05 AM


CAPTULO 1

lgebra en los nmeros reales 37

38-39. 37 18/11/02, 11:06 AM


Ejercicios ltimo trmino del binomio) y el producto del trmino comn
(y) por la suma de estos nmeros sea igual al trmino central
resueltos (13y). Los nmeros son + 9 y + 4.
En efecto: + 9 + 4 = 36 y 9 + 4 = 13
Entonces: y2 + 13y + 36 = (y + 9) (y + 4).
4. Factoricemos a2 2a 48
Descartamos la posibilidad de cuadrado de binomio pues el ltimo
trmino ( 48) no es cuadrado de ningn nmero.
Buscamos dos nmeros cuyo producto sea 48, y cuya suma
sea 2, la que al multiplicarla por el trmino comn a nos
da el trmino central 2a.
Los nmeros son 8 y + 6 y la factorizacin correspondiente es:
a2 2a 48 = (a 8) (a + 6).
5. Factoricemos x2 5x + 6
No es cuadrado de binomio por la misma razn anterior (el + 6
no es cuadrado de un nmero entero). Corresponde entonces
al producto de dos binomios con un trmino comn, que en
este caso es x. Buscamos dos nmeros cuyo producto sea +
6 y cuya suma sea 5.
Los nmeros son 2 y 3. Por lo tanto, la factorizacin
correspondiente es:
x2 5x + 6 = (x 2) (x 3).
6. Factoricemos la expresin 2x2 3x 2
En este ejemplo, ni siquiera el primer trmino es cuadrado exacto
de un trmino entero.
Amplifiquemos por el coeficiente de x2 (en este caso, por 2)
para obtener un primer trmino como en los ejemplos anteriores,
es decir, un cuadrado exacto.
2
2x 3x 2 2
/
2 2
4x 6x 4
2

Podemos aplicar al numerador el razonamiento de los ejemplos


anteriores (porque el primer trmino ya es un cuadrado exacto)
y entonces trataremos de factorizar como producto de dos
binomios con un trmino comn que en este caso es 2x.
Buscamos dos nmeros que multiplicados sean igual a 4 y
cuya suma sea igual a 3 (pues al multiplicar la suma por el
trmino comn 2x se debe obtener 6x).
Los nmeros son 4 y 1 y as, la factorizacin de la expre-
sin amplificada es:
4x2 6x 4 2x 4 2x + 1
=
2 2

38 lgebra en los nmeros reales

38-39. 38 18/11/02, 11:06 AM


CAPTULO 1

Podemos factorizar el primer trmino por dos y luego simplificarlo


por el denominador, obteniendo:
(2x 4) (2x + 1)
2 x2 3x 2 =
2

2 (x 2) (2x + 1)
2 x2 3x 2 =
2

2 x2 3x 2 = (x 2) (2 x + 1)

7. Factoricemos 3x2 5x + 2
Siguiendo los pasos anteriores, obtenemos:

3 x2 5x + 2 3
/
3
9x2 15x + 6
3
(3x 3) (3x 2)
3
3 (x 1) (3x 2)
3
(x 1) (3x 2)

Ejercicios
Factorice las siguientes expresiones:

1. x2 + 14x + 49 12. 4x2 + 20x + 25 23. x2 x 6

2. x2 + 8x + 16 13. 9x2 6x + 1 24. x2 5x + 6

3. a2 + 18a + 81 14. a2 4ab + 4b2 25. a2 5a 36

4. a2 6a + 9 15. y2 + 6xy + 9x2 26. a2 + a 30

5. y2 24y + 144 16. 4t2 + 12t + 9 27. a2 + 8a + 7

6. x2 + 10x + 25 17. 4x2 + 12xy + 9y2 28. y2 + y 56

7. t2 2t + 1 18. 9x2 30xy + 25y2 29. x4 6x2 + 9

8. z2 + 16z + 64 19. x2 + 14xy + 49y2 30. 4 + 20y2 + 25y4

9. x2 22x + 121 20. x4 + 2x2 + 1 31. x4 + 2x2y2 + y4

10. a2 12a + 36 21. x2 + 5x + 6 32. x6 + 2x3 + 1

11. 1 + 6a + 9a2 22. x2 + x 6 33. a4 4a2b2 + 4b4

lgebra en los nmeros reales 39

38-39. 39 18/11/02, 11:07 AM


CAPTULO 1

lgebra en los nmeros reales 39

40-41.(2003) 39 18/11/02, 11:08 AM


Ejercicios
45. 2x2 + 5x + 2 58. 6a2 + 13a + 6

34. 9m4 30m2p2 + 25p4 46. 2x2 + 5x 3 59. 12a2 23a + 5

35. 9m2 30mp2 + 25p4 47. 3x2 + 14x + 8 60. 8a2 2a 15


2 48. 3x2 + 11x 4 61. 5x2 26x + 5
36. x x + 1
4
2 1 49. 6x2 13x + 5 62. 18a2 18a + 4
37. a + a +
4
2 50. 2x2 +15x + 28 63. a4 + 5a3 + 6a2
38. a + ab + b2
4 51. 7x2 8x + 1 64. x3 3x2 40x
39. a2 23a + 132
52. 6x2 + 5x 4 65. x4 3x2 + 2
40. a2 3a 40
53. 8x2 2x 1 66. 2a3 + 6a2 + 4a
41. a4 + 5a2 + 6 54. 5x2 18x + 9 67. m3 m2 30m
42. 4x2 22x + 30 55. 2x2 + 3x 14 68. n4 + n2 2
43. 9x2 9x 28 56. 3a2 7a + 2 69. p4 + 2p2 + 1

44. 25x2 15x + 2 57. 5a2 + 3a 2 70. p3 p2 p + 1

Soluciones

1. (x + 7)2 2. (x + 4)2 3. (a + 9)2 4. (a 3)2 5. (y 12)2 6. (x + 5)2


7. (t 1)2 8. (z + 8)2 9. (x 11)2 10. (a 6)2 11. (1 + 3a)2 12. (2x + 5)2
13. (3x 1)2 14. (a 2b)2 15. (y + 3x)2 16. (2t + 3)2 17. (2x + 3y)2
18. (3x 5y)2 19. (x + 7y)2 20. (x2 + 1)2 21. (x + 3) (x + 2) 22. (x + 3) (x 2)
23. (x 3) (x + 2) 24. (x 3) (x 2) 25. (a 9) (a + 4) 26. (a + 6) (a 5)
27. (a + 7) (a + 1) 28. (y 7) (y + 8) 29. (x2 3)2 30. (2 + 5y2)2 31. (x2 + y2)2
x 2
32. (x3 + 1)2 33. (a2 2b2)2 34. (3m2 5p2)2 35. (3m 5p2)2 36. 1
2
1 2 a 2
37. a + 2 38. +b 39. (a 12) (a 11) 40. (a + 5) (a 8)
2
41. (a2 + 2) (a2 + 3) 42. 2(2x 5) (x 3) 43. (3x + 4) (3x 7) 44. (5x 1) (5x 2)
45. (2x + 1) (x + 2) 46. (2x 1) (x + 3) 47. (3x + 2) (x + 4) 48. (3x 1) (x + 4)
49. (3x 5) (2x 1) 50. (2x + 7) (x + 4) 51. (7x 1) (x 1) 52. (3x + 4) (2x 1)
53. (4x + 1) (2x 1) 54. (5x 3) (x 3) 55. (2x + 7) (x 2) 56. (3a 1) (a 2)
57. (5a 2) (a + 1) 58. (2a + 3) (3a + 2) 59. (3a 5) (4a 1) 60. (2a 3) (4a + 5)
61. (x 5) ( 5x 1) 62. 2(3a 2) (3a 1) 63. a2(a + 2) (a + 3) 64. x(x + 5) (x 8)
65. (x 1) (x + 1) (x2 2) 66. 2a(a + 1) (a + 2) 67. m(m 6) (m + 5)
68. (n 1) (n + 1) (n2 + 2) 69. (p2 + 1)2 70. (p 1)2 (p + 1)

40 lgebra en los nmeros reales

40-41.(2003) 40 18/11/02, 11:08 AM


CAPTULO 1

1.6.5 Sumas o diferencias de cubos


Los factores de una diferencia de cubos son:
x3 y3 = (x y) (x2 + xy + y2)
Los factores de una suma de cubos son:
x3 + y3 = (x + y) (x2 xy + y2)

1. Factoricemos a3 8 Ejercicios
Observamos que a3 es el cubo de a y que 8 es el cubo de 2. Se resueltos
trata de una diferencia de cubos, por lo tanto:
a3 8 = (a 2) (a2+ 2a + 4)
2. Factoricemos x3 + 27
El trmino x3 es el cubo de x y 27 es el cubo de 3. Aqu
tenemos una suma de cubos y por lo tanto:
x3 + 27 = (x + 3) (x2 3x + 9)
3. Factoricemos 27a3 125b3
El primer trmino es el cubo de 3a y el segundo trmino es
el cubo de 5b, entonces escribimos:
27a3 125b3 = (3a 5b) (9a2 + 15ab + 25b2)
4. Factoricemos a6 b6
Aqu tenemos primero una diferencia de cuadrados, la cual
factorizamos como una suma por su diferencia. Luego, cada
uno de los factores corresponde a una suma o diferencia de
cubos. Procedamos por pasos:
a6 b6 = (a3 + b3) (a3 b3)
= (a + b) (a2 ab + b2) (a b) (a2 + ab + b2)
y sa es la factorizacin requerida.

Ejercicios
Factoricemos las siguientes expresiones:
1. m6 n3 4. t3 64 v3 7. 1 125 a3 10. 216 a3 27 b3
1 1 8 27
2. x3 + p3 5. 27 x3 + y3 8. 3
+ 3 11. 3
3
x y z y
6
n3
3. a3 8 b3 6. 8m 9. 16 x3 54 y3 12. 125 1
8 8a3

lgebra en los nmeros reales 41

40-41.(2003) 41 18/11/02, 11:09 AM


CAPTULO 1

lgebra en los nmeros reales 41

42-43. 41 18/11/02, 11:09 AM


Ejercicios 20. 3 t3 3 27. a6 1 34.
x3
1
y3
13. 3 a3 81 b3 a6
21. 216 a3 + 8 b3 28. 1 b3 35. 0,001 3
14. a2 b3 c6 + a2 d3 b
22. 8 t3 + 64 29. 8 27 36. 216
a3
15. m3 x3 + 1 t6 t3 b3
1 1 1
23. 125 t 3 3 30. p + q9
3
37. + 3
16. a3 b6 c9 + 8 z 125 z
2 16
24. 3 31. m12 + 1 3
38. 64a
1
17. x12 y12 t3 y 216
1
18. m9 1 25. 3
8a + 3 32. a27 + b27 39. m3 n3 p6 8a3
b
26. 1 + a3 33. 1 a9 1 1
19. a3 b12 27 40. +
8z3 27 y3

Soluciones

1. (m2 n) (m4 + m2 n + n2) 2. (x + p) (x2 px + p2)


4. (t 4v) (t2 + 4tv + 16v2)
3. (a 2b) (a2 + 2ab + 4b2)
n2 n4
6. 2m 4m2 + mn2 +
5. (3x + y) (9x2 3xy + y2) 2 4
7. (1 5a) (1 + 5a + 25a2) 8. 1 1 1 1 1
x + y 2
+ 2
x xy y
9. 2(2x 3y) (4x2 + 6xy + 9y2)
10. 27(2a b) (4a2 + 2 ab + b2)
2 3 4 6 9
11.
z y + + 2 12. 5 1 25 + 5 + 1
z2 yz y 2a 2a 4a2
13. 3(a 3b) (a2 + 3ab + 9b2) 14. a2(bc2 + d) (b2c4 bc2d + d2)
15. (mx + 1) (m2x2 mx + 1) 16. (ab2c3 + 2) (a2b4c6 2ab2c3 + 4)
2 6 3
17. (x y) (x + y) (x2 + y2) (x4+ x2 y2+ y4) (x4 x2 y2+ y4) 18. (m 1) (m + m + 1) (m + m + 1)
2
20. 3(t 1) (t + t + 1)
19. (ab4 3) (a2 b8 + 3ab4 + 9)
22. 8(t + 2) (t2 2t + 4)
21. 8(3a + b) (9a2 3 ab + b2)
1 2 1 2 4
1 5t 1 24. 2 + + 2
23. 5t z 25t 2 + + 2 t y t 2
ty y
z z
26. (a 1) (a2 + a + 1)
1 2a 1
25. 2a + 4a2 + 2 28. (1 + b) (1 b + b2)
b b b
27. (a 1) (a + 1) (a4 + a2 + 1) 30. (p + q3) (p2 pq3 + q6)
32. (a + b) (a2 ab + b2) (a6 a3b3 + b6) (a 18 a9b9 + b18)
1 2 4 6
29.
3 t
3 2 + +9 x x2 x
t t t 34. y 1 y2 + y + 1
31. (m4 + 1) (m8 m4 + 1)
a 6a a2
33. (1 a) (1 + a + a2) (1 + a3 + a6) 36. 6 + 36 + 2
b b b
a2 0,1a2 a4
35. 0,1 0,01 + +
2a 1
b b b2 38. 4 a 1 16a2 + +
6 3 36
1 1 1 1 1
37. + + 2
5 z 25 5z z 40. 1 1 1 1 1
+ +
39. (mnp2 2a) (m2n2p4 + 2a mnp2 + 4a2) 2z 3y 4z2 6yz 9y2

42 lgebra en los nmeros reales

42-43. 42 18/11/02, 11:11 AM


CAPTULO 1

Fracciones algebraicas 1.7

1.7.1 Simplicacin
Para simplificar una fraccin es necesario y suficiente que el
numerador y el denominador tengan un factor comn.
En el caso de monomios, la simplificacin se hace en forma
directa; en cambio, si el numerador o el denominador de la
fraccin tienen dos o ms trminos, es necesario factorizar primero
y luego simplificar.

2a2
1. Simplifiquemos
3ab
Ejercicios
Aqu tanto el numerador (2a2) como el denominador (3ab) contienen el resueltos
trmino a como factor. Simplificamos, pues, por l y obtenemos:
2
2a 2a
=
3ab 3b 2 2
6m p q
2. Simplifiquemos 3 2
27 mp q
En este ejemplo, el trmino 3mp2q est contenido en el numerador y
en el denominador. Simplificando, nos queda:
2 2
6m p q 2m
=
3 2
27mp q 9pq
2
3. Simplifiquemos 2a + 2
4a
En este caso no es posible hacer una simplificacin directa, pues en el
numerador hay un binomio (recordemos que no podemos simplificar
trminos que se suman o restan).
Debemos entonces factorizar primero y despus simplificar:
2 2 2
2a + 2 2(a + 1) a +1
= =
4a 4a 2a

Y ya no es posible seguir reduciendo porque el numerador no se


puede factorizar ms.
2
4. Simplifiquemos a + ab
a+b
Usando el mismo razonamiento anterior, factorizamos primero y
luego simplificamos:
2
a + ab a(a + b)
= = a
a+b a+b
x2 + 5x + 6
5. Simplifiquemos
x2 + 3x + 2
Factorizando y luego simplificando obtenemos:
x2 + 5x + 6 (x + 2) (x + 3) x+3
= =
2
x + 3x + 2 (x + 2) (x + 1) x+1

lgebra en los nmeros reales 43

42-43. 43 18/11/02, 11:12 AM


CAPTULO 1

lgebra en los nmeros reales 43

44-45. 43 18/11/02, 11:14 AM


x2 9
6. Simplifiquemos
x2 + 6x + 9
Procediendo como antes:
x2 9 (x + 3) (x 3) x 3
= =
x2 + 6x + 9 (x + 3) (x + 3) x + 3
3x3 3x y2
7. Simplifiquemos
x2 y xy2
3x3 3xy2 3x (x2 y2) 3x (x y) (x + y) 3 (x + y)
= = =
x2 y xy2 xy (x y) xy (x y) y

Ejercicios
Simplifique las siguientes expresiones:

3 p q3 2
1. 2a 13. 2 p q2 25. a + ab 37. ab + ac + xb + xc
5ab 2a b2 c2
3a 15 c7 d8 a2 b2 a b x2 + 7x + 12
2. 14. 26. 38.
6b 35 ab c7 ab 1 x2 + 5x + 6

a2 b 17 m6 n11 2 x4 y2
3. 15. 27. x 25 39.
ab2 51 m4 n9 x+5 x2 + y

6m (a + b)2 1 a2 x2 8x + 12
4. 16. 28. 40.
16pm (a + b) 1+a x2 4x 12

5ad (p2 + 1)3 ab2 ac2 2 2


5. 17. 29. 41. 2 a b 8 a b
10d (p2 + 1)4 b+c 4 a2 b 16 a b2
25 p2q a2 + b2 x+4 ax + bx ay by
6. 18. 30. 42.
15 pq3 2
(a + b ) 2 4 x2 + 8x + 16 x2 y2

2 32 z4 y3 xy 10x2 15xy
7. 2 m np
2
19. 31. 43.
18mn p 96 z3 y4 x2 y2 xy3 4x2 9y2
6 a 3a3 3a2 6a
8. 30 a b 20. 32. 3 abc
44.
21 a6 b 2 144a 2
6 a bc 9 ab c 2
2a3 + 6a2 + 4a
125 x6 y5 z4 121 a 6x2 3xy x4 x2
9. 5xyz
21. 33. 45.
11 ac 4x2 y2 x3 + 2 x2 + x

2ab m4 n4 2 pq x2 11x + 30
10. 22. 34. 46.
8 a6 b6 4mn p2 q pq2 x2 25
4 4
12 a12 b12 5xy + 10x x3 3xy2
11. a b 23. 35. 47.
4ab 6 a6 b6 y2 4 x 4 9y4
12 (a3 b2 )2 2 2 x8 9y4
12. 6 a 24. 36. m 2mn + n 48.
12 a6 5a6 b8 m2 n2 x4 3y2

44 lgebra en los nmeros reales

44-45. 44 18/11/02, 11:17 AM


CAPTULO 1

x2 7x + 6 2
58. x 8x + 15
49.
5ab
52. 55. ac ad bc + bd
25a2 5ab x2 1 c2 d2 x2 9

x2 + x 2 2a2 b 2ab2 2p2 x + 2px2 4p2 4p + 1


50. 53. 56. 59.
ax + 2a x 2 a2 2ab + b2 p2 pq + xp xq 4p2 1

x3 x2 + x 1 6 p2 q 2 pq2 2 2m3 18m


51. 54. 57. x + 10x 11 60.
x2 1 3 p2 q p q 2 x2 + 9x 10 2m2 6

Soluciones

3 5p
1. 2 2. a 3. a 4. 5. a 6. 7. m 8. 10
5b 2b b 8p 2 3q2 9n 7b
a3 b3 a6 3q 8
9. 25x5y4z3 10. 1
11. 12. 13. 14. 3d
4a5 b5 4 2 2 7ab
m2 n2 1 1 z 1
15. 16. a + b 17. 18. 19. 20. 21. 11
3 p2 + 1 2 2 3 3y 144 c
(a + b )
m3 n3 1
22. 23. 2a6b6 24. 25. a + b 26. a b 27. x 5 28. 1 a
4 5b4 2

1 1 1 3x 2
29. ab ac 30. 31. 32. 33. 2x + y 34. p q
x+4 xy y2 2a 3b

5x x2
35. 36. m n 37. a + x 38. x + 4 39. x2 y 40.
y2 m+n bc x+2 x+2

1 a+b 5x 3a 6 x2 x x6
41. 42. x + y 43. 44. 45. 46.
2 2x + 3y 2a + 4 x+1 x+5

x b x2 + 1
47. 48. x4 + 3y2 49. 50. x 1 51. 52. x 6
x2 + 3y 2 5a b a1 x+1 x+1
ab 2px 2p 1
53. 2ab 54. 2 55. 56. 57. x + 11 58. x 5 59.
ab c+d pq x + 10 x+3 2p + 1

m3 9m
60.
m2 3

1.7.2 Multiplicacin y divisin


de fracciones algebraicas
Multiplicamos los numeradores y los denominadores entre s y
hacemos todas las simplificaciones posibles.
En el caso de los monomios las simplificaciones pueden hacerse antes
o despus de multiplicar; en el caso de los polinomios (expresiones con dos
trminos o ms) es conveniente hacer todas las simplificaciones primero
(factorizando por supuesto) y luego las multiplicaciones.
Para dividir fracciones, multiplicamos la primera por el recproco
de la segunda.

lgebra en los nmeros reales 45

44-45. 45 18/11/02, 11:21 AM


CAPTULO 1

lgebra en los nmeros reales 45

46-47. 45 18/11/02, 11:23 AM


Ejercicios 1. Efectuemos el siguiente producto:
resueltos 3ab
2b
2a 3a2
Multiplicando en forma directa obtenemos:
3ab 2b 3ab 2b 6 ab2 b2
= = = 2
2a 3a2 2a 3a2 6 a3 a
2. Efectuemos el producto:
3xy 6ab 5z2

2a 3xz 10b2 x
Multiplicando en forma directa obtenemos:
3xy 6ab 5z2 90 x y a b z2 3yz
2
= 2
=
2a 3xz 10b x 60 a x2 z b 2xb
3. Efectuemos el producto:
a+b a2 2ab + b2
=
ax bx a2 b2
Aqu debemos simplificar antes de multiplicar (de lo contrario
complicamos mucho el ejercicio). Como sabemos, factorizamos
primero, obteniendo:
a+b a2 2ab + b2 a+b ab ab 1

2 2
= =
ax bx a b x ab a+b ab x
Una vez hechas las factorizaciones podemos simplificar un
factor de cualquier numerador con un factor igual de cualquier
denominador.

4. a + 1 a2 4 a2 9

a+2 a2 + 4a + 3 a2 4a + 4
Factoricemos primero:
a+1 a2 4 a2 9 a+1 a+2 a2 a+3 a3
=
a+2 a2 + 4a + 3 a2 4a + 4 a+2 a+1 a+3 a2 a2
el resultado es a 3
a2
5. Multipliquemos:
5a + b ab a

2a b ab 5
Aqu no es posible efectuar ninguna simplificacin; por lo tanto,
procedemos a multiplicar directamente.
5a + b ab a 5a + b a2 b 5a3 b a2 b2
= =
2a b a b 5 2a b 5a 5b 10a2 10ab 5ab + 5b2
Reduciendo trminos semejantes obtenemos finalmente:
5 a3 b a2 b2
10 a2 15 a b + 5 b 2
6. Efectuemos la siguiente divisin:
2ab 2a
3x
: 3x y
Cambiamos el signo de divisin : por el de multiplicacin e
invertimos la segunda fraccin. Nos queda:

46 lgebra en los nmeros reales

46-47. 46 18/11/02, 11:24 AM


CAPTULO 1

2ab 2a 2ab 3xy


3x
: 3xy = 3x

2a

Hacemos las simplificaciones adecuadas y obtenemos:


2ab 3xy
= by
3x 2a

7. Efectuemos la siguiente divisin:


a+b a2 b2
2ab
: 6a2 b
Procediendo como en el ejemplo anterior:
2 2
6a2 b
a+b
: a6a2bb =
a+b

2ab 2ab a2 b2
a+b 6a2 b
=
2ab a + b a b
3a
=
ab
(Aqu fue necesario factorizar el trmino a2 b2 antes de simplificar).

Ejercicios
I. Simplifique las siguientes expresiones:
12x6 y10 6p + 12q x2 + 5x 14
1. 3ab 12. 23. 34.
9a 4x5 y11 p + 2q x2 + 9x + 14
2
2 a+b 1a x4 2 x2 y2 + y4
2. ax 13. 24. 35.
bx a+b 3 1 a2 x4 y4
2ab pq 3 ac + ad + bc + bd
3. 14. 25. 12x 3x 36.
5ab 2p 2q 3x ac ad + bc bd

3a + 3ab 4 x2 2 2
4. 3a 15. 26. 37. 2 a b + 6 a b
18a2 b 1+b 2+x a2 + a b 6 b2
16m2 2xy 2bx2 + 2bxy axy ay2
5. 16. 3x + 15 27. 38.
18n2 5x + 25 x2 y xy2 6abx2 3a2 xy
b a2 + a 2 2
a4 + 3a3 + 2a2
6. 17. 28. 9x 16y 39.
a b2
2
2a + 2 3x + 4y a3 a
6m2 n 20x2 5xy a 9b2
2
2t 2 2t 12
7. 18. 29. 40.
15mn2 4x y 3a + 9b 4t 2 16t + 12
7p2 qr 6 3x2 y 3xy2 m n 2 50 2y2
8. 19. 30. 41.
3pq2 r 5 2x2 2xy m2 n2 4y2 + 44y + 120
3p6 q3 x2 y2 a2 5a + 6 ab ay bx + xy
9. 20. 31. 42.
24p6 q2 xy a2 4 b2 by + bx xy
a2 b2 c2 a2 b2 2 a2 2 b2 2
10. 21. 32. 2 2
43. 2a 10a + 12
2abc a+b a +2ab+b a2 + a 6
2 2
9x2 y7 z11 3a 3b 18 a b + 27a b 4u2 v2
11. 22. 33. 44.
x2 y6 z10 a2 b2 2
4 a + 12 a b + 9 b 2
6u2 v 3uv 2

lgebra en los nmeros reales 47

46-47. 47 18/11/02, 11:27 AM


CAPTULO 1

lgebra en los nmeros reales 47

48-49. 47 18/11/02, 11:33 AM


Ejercicios
p2 x + px 2 2 a2 b2 c2 x3 y3
45. 47. 49.
p2 + 3px + 2x2 4 a2 b c + 2 a b2 c + 2 a b c2 x2 y2

a3 a2 30a x3 y3 m2 4p2
46. 48. 50.
a 11a3 + 30a2
4 x2 + xy + y2 m2 4mp + 4p2

II. Efecte las operaciones indicadas (multiplicaciones):


1 2a + 4 a + 4 a2 8a + 16
1. a ab 11. x2 + 2 x y + y2 21.
2b b x+y 3a 12 a2 16 a+2

2x 1 2a3 a2 1 x2 6x + 5 x2 4
2. y
x 12. 22.
a 1 a2 x+2 x2 1
2 x6 x2 x 2 2x
3. 2m 3mn 13. 9x 1 2x 23.
3n 4 2 x2 x2 9x + 18 x+1
x 3x 1
a2 ab m n m+n m2 mp 4p 1
4. 14. 2 2 24.
b a m n m n 2 2p2 m2 p2 2m

3x 10y 2 a2 3a 18 a2 16 a+2
5. 15. a + a
3a
25.
5y 6x a2 a2 + 2a + 1 2 2
a 2a 8 a 5a 6 a+3
2 2x2 + 6x 2xy 1 x3 y3 3xy
6. 3u v 2 u v 16. 26.
2u 18 v 2 3y x+3 2x 3y x2 + xy + y2

10a2 bc 2b x2 + 5x + 6 x2 a+b ab a2 2ab + b2


7. 17. 27.
4b3 5ac x2 4 x2 9 a2 b2 a+b 3ab
2
7xy m 10m 1+x 1 x2 2 2 2 2
8.
2
18.
2
28. a 25b a 7b + 12b
2m 14x y 2x 1x 1+x a 3b a 5b

a2 ab a+b 2
9. a
2b

4c 19.
2
29. 2a 4 a2 5 a + 6
a
b 3c a 2ab a2 b 6a a 4a+3 a2
xy x2 3x 6 x2 9 1 a3 + 2a2 + a a2 25 1
10. 20. 30.
x xy 2x 6 x2 4 3 a2 + 7a + 10 a + 1 2a2

III. Efecte las siguientes operaciones (divisiones):

15n2 p 3np2 2
b2 c2
1. a a 3
: 5. 2ab 6a
: 2ab 9. : 13. 15a bc : 25a
2 3 3b 2nz 4z 3ab2 bc
a2 x3 x2 y x2 y2
2. 1 : 2 6. x 1 : x10
1 10. 1
: aa + 12 14. :
a a 5 a+2 2xy x+y
x2
: xy 2axy 2x 2
3. y 7. : 3y 11. x 1 : ax2 xa 15. 2 a b : 2 x : 3 b x
3a a1 8 b2 x 1 2 x 2
2 2x 6 2
:x 5x + 6
2
4. m n
: ax 8. a + b : a 2 b2 12. a b : a+b 16.
ax ab a3 b 3
ab 3x2 y 6xy

48 lgebra en los nmeros reales

48-49. 48 18/11/02, 11:36 AM


CAPTULO 1

1
17.
a2 49
: a2 81a + 7 24. 1 : 1 : 1
x x x
2
1 2 a 22x y 11 x y
18. : :
2
25. : 14 : 2x
a a2 7
2 3 2 2
19. a 1 : a2 1 26. a + 3 a : a + 2a
2 2
a2 a 4 a 9 a 5a + 6
2
20. a 3 : a 8a + 15 27. 1
: 1
2 3 2 2
a 5 a 11a + 30 x 6x x 12 x + 36
2 2 2
ac ad bc + bd c d
21. 2a + 8 : a + 2a 8 28. :
2 2 2 2 2
3a 3 6a 6 a b a + 2 ab + b
3 2 3 2 2 2
22. a 5a + 6a : a 3a 29. x + 7 x + 10 x + 2
:
x + 3x 4
2 2 2 2
a + 7a + 12 a 16 x + 2x 3 x + 3 x 25
2 2
a
23.
2
:a 30. 2x x 4 x + 2
2
: 2
b b x2 x +x x 1

Soluciones
I. Simplificacin
2
2. a x 8. 7 pr
8m
1. b 3. 2 4. 1 5. 2
6. 1
7. 2 m 9. q
3 b 5 6 ab 9n a b
2
5n 3q 8

abc 3x 1 1 3 a
10. 11. 9yz 12. 13. 14. 15. 3a 16. 17. 18. 5x
2 y a+b 2 5 2

1
19. 3 y 20. x + y 21. a b 22. 3 23. 6 24. 25. 4 x2 26. 2 x
2 a+b 1+ a

a 3b m n
27. 2
28. 3x 4y 29. 30. m+ n
31. a 3 32. 2 a 2 b
xy 3 a+2 a+b
2 2 2
x2
33. 9 ab
34. 35. x2 y 2 36. c + d 37. 2 ab 38. x+y
39. a + 2 a
2a + 3b x+2 x +y cd a 2b 3a x a1

5 y ax 2a 6 2u + v
40. t+ 2
41. 42. 43. a+3
44. 45. p x
2t 2 12 + 2 y b+ x 3uv p+ 2x

a bc 2 2 m+ 2p
46. a+5
47. 48. x y 49. x + x y + y 50.
2
a 5a 2a + b + c x+y m 2p

II. Multiplicacin
2
a
2
2 2 u a 5m
2
8
1. 2. 3. m 4. a2 5. 1 6. 7. 8. 9. 10. x
2b y 2 6 b 4x
2 3

2
1 1
11. x + y 12. 2a2 + 2a 13. 6 x + 2 14. 15. 3
16. 4x
17.
x m n
2 a+1 3 x3

x+3 2
2x 1
18. 1 19. 1 20. 2x + 4
21. 2
22. x 7 x + 10
23. 24.
2b 3 x+1 x3 mp + p2

a+4 xy ab a2 a2 4 a 5
25. 26. 27. 28. a2 + ab 20b2 29. 30.
a+1 2 3a + 3b 3a 3 2 a2 + 4 a

lgebra en los nmeros reales 49

48-49. 49 18/11/02, 11:40 AM


CAPTULO 1

lgebra en los nmeros reales 49

50-51.(2003) 49 18/11/02, 11:45 AM


Soluciones

III. Divisin

2. 1
3
1. 3. x 4. m
5. 2 ab 6. 2 7. y2
2 2 n 9

1 10 a ab 1
8. 9. 10. a + 1 11. 12. 13.
2 2 p x 2
a + ab + b
2
5b
2
a 2a b + b
14. x 15. 3a 16. 4
17. a1
18. 1 19. a + 2
2y 16 2
x 2x a+7 a+1
2
1
20. a 6 21. 4a + 4 22. a 2 6a + 8 23. 24. 1
x
25. 2
a5 a2 a + 3a ab
2
26. a 2a 27. x6 28. a + b 29. x+4
30. 2x 2
a+2 x
2 c+d x5

1.7.3 Adicin y sustraccin


de fracciones algebraicas
Si las fracciones tienen el mismo denominador, entonces sumamos (o
restamos) los numeradores y conservamos el denominador.
Si los denominadores son diferentes, entonces debemos buscar el
mnimo comn mltiplo (m.c.m.) de ellos y amplificar cada fraccin
por el factor necesario, de modo que todas queden reducidas a un
denominador comn.
El mnimo comn mltiplo de expresiones algebraicas es aquella que
las contiene, como factores, a todas.

Ejercicios 1. Encontremos el m.c.m. entre a y 2a.

resueltos Vemos que a est contenido (como factor) en 2a, por lo tanto,
el m.c.m. es 2a.
2. Encontremos el m.c.m. entre a, 2a y a2.
Aqu ninguno de los tres trminos contiene a los otros dos. Buscamos
el m.c.m. entre los coeficientes numricos, en este caso es 2, y entre los
factores literales, en este caso, como se trata de monomios de la misma
base, es el trmino que tiene el exponente ms alto.
As, el m.c.m. es 2a2.
3. Encontremos el m.c.m. entre 2x, 3xy, x2.
Usando el razonamiento anterior, determinamos el m.c.m. entre los
coeficientes numricos, que es el 6, y entre los factores literales, que es
x2 y. As, el m.c.m. entre 2x, 3xy, x2 es 6x2y.
4. Encontremos el m.c.m. entre a b y a2 b2.
Como sabemos, la factorizacin correspondiente de a 2 b2
es (a b) (a + b); por lo tanto, a b est contenido en a2 b2 y
as el m.c.m. es a2 b2.

50 lgebra en los nmeros reales

50-51.(2003) 50 18/11/02, 11:46 AM


CAPTULO 1

5. Encontremos el m.c.m. entre a + 2 y a + 3.


Aqu ningn trmino est contenido en el otro; por lo tanto, el m. c. m.
es el producto de los dos, es decir, a2 + 5a + 6.
6. Efectuemos las operaciones indicadas.
a + 2 2a + 5
+
3 3
Se trata de una suma con igual denominador, as es que sumamos
los numeradores y conservamos el denominador.
a + 2 2a + 5 a + 2 + 2a + 5 3a + 7
+ = =
3 3 3 3

7. Efectuemos las operaciones indicadas:


4 5 3
+
2
x 2x 5x
Los denominadores son diferentes; por lo tanto, debemos
determinar el m.c.m. entre ellos, que ser el denominador comn.
Este es 10x2. Luego amplificamos cada fraccin por el trmino
adecuado para obtener el m.c.m.
4 5 3 4 10x + 5 5x 3 2
+ 2
= 2
x 2x 5x 10x
40x + 25x 6
= 2
10x
65x 6
= 2
10x
8. Efectuemos las operaciones siguientes:
m+ 1 m+ 1 1
2
2
+
2m + 4m m 4 m 2

Factoricemos los denominadores para encontrar el m.c.m.


m+ 1 m+ 1 1
+
2m m + 2 m 2 m+ 2 m 2
El m.c.m. es 2m (m + 2) (m - 2)
Es conveniente mantener el m.c.m. factorizado, pues as facilita el
proceso de amplificacin de cada fraccin y el de simplificacin,
si es posible, al final.
m+ 1 m+ 1 1
+ =
2m m + 2 m 2 m+ 2 m 2
m 2 m + 1 2m m + 1 + 2m m + 2
=
2m m + 2 m 2
2 2 2 2
m + m 2m 2 2m 2m + 2m + 4m m + m 2
=
2m m + 2 m 2 2m m + 2 m 2
Factorizamos el numerador y hacemos la simplificacin corres-
pondiente:
m+ 2 m 1 m 1
=
2m m + 2 m 2 2m m 2
m 1
=
2
2m 4m

lgebra en los nmeros reales 51

50-51.(2003) 51 18/11/02, 11:47 AM


CAPTULO 1

lgebra en los nmeros reales 51

52-53. 51 18/11/02, 11:48 AM


Ejercicios

I. Determine el mnimo comn mltiplo entre:

1. 2, 3, 5 16. 6m, 3m + 1, 6m + 2

2. 2, 2a, 3a 17. x + a, x2 a2, x a

3. 3x, 3xy 18. 1, x + 1, x + 2

4. 2x, 3xy, 2y 19. a, b, a + b

5. m2, n2 20. a2 b2, a2 2ab + b2

6. m, mn, n 21. x + 3, x2 + 5x + 6, x + 2

7. x2, y2, xy 22. x 3, 2x 4, x2 5x + 6

8. 1, a, a2 23. a2 + a, a2 1, a2 + 2a + 1

9. x2yz, xy2z 24. a + 2, a2 + 4a + 4, a2 4

10. xy2z, xyz2 25. x2 + 9x + 14, x2 4, x2 + 5x 14

11. 4p2q, 5pq2 26. a 1, a2 1, 3a2 3a

12. 5p6q6, 6p5q5 27. p, p + 5, p3 25p

13. a + b, a b 28. 2x + 2, 4x + 4, x2 + 2x + 1

14. 2a + 4, a + 2 29. t 5p, t2 25p2, 5t 25p

15. 3a + 6, a2 4 30. x + y, x2 + 2xy + y2, x2 y2

II. Efecte las operaciones indicadas:

3 4 15 a
2
4a 4
1. + 9. +
11 11 11 2 2 2
a 4 a 4 a 4
3 5 21 3
2. + + +
16 16 16 16 10. 7p 3q 6p 4q + 2p
2 2 2
p +1 p +1 p +1
3. 2 6 + 9 12
a a a a
11. 2a a + 4 3a a + 6 + 2a a 5
2 2 2
4. 3a 1 + 2a 7 2 a 20a a 20a a 20a
5 5 5 3a
2
3a a + 4 4a 4b
9 2 1 12. +
5. + + 3a 4b 3a 4b 3a 4b
3x 4 3x 4 3x 4 2 2
13. x 7x + 1 2x x 3 x +x+2
a + 5b 2a + b 4a + 5b +
+ 2 2 2
6. x + 5x + 6 x + 5x + 6 x + 5x + 6
a + 3b a + 3b a + 3b
2 2 2

7. 2x 2 + 3x 1 4x 4 14. 5x 7x 1 + 2x + 4x 1 3x 6x 2
2 2 2
x+6 x+6 x+6 x + 3x x + 3x x + 3x
2 2 2
3x + 5 5x + 8 x 4
8. + 15. 2x + 3x + 6 + x 6x + 8 3x 4x + 3
2x 3 2x 3 2x 3 2
x 121
2
x 121
2
x 121

52 lgebra en los nmeros reales

52-53. 52 18/11/02, 11:49 AM


CAPTULO 1

2 4 1
16. + 34. 2a + b 2a b a
3 5 2 3a 3b b
2a 1 2a 2 a
17. a + a 35. 2

2 3(a 2) a 4 3
a a a 2x 3x y 2x 4y
18. + +
2 3 4
36. + 2 2
x+y xy x 2xy + y
19. 2b b
a+7 a6 2a 4
37. + 2
3 a6 a5 a 25
2x x+4 2x 3
20. 2x x + x 38. 2
+ 2
+ 2
5 7 35 x + 3x + 2 x 4 x x2

21. a + a + a 39. x + 1 + x 2 x3
+
x+4
2 3 3 2 2 2 2
x 1 x 9 x + 5x + 6 x 4x + 3

22. a + a a 40. 3x 2 2
1
2
2x

6 3x
2
2 3 2x + 8 x 16 x + 5x + 4 x 1
2
23. 3x 2x + x 41. 2x 1 + 3x + 1 5x2 + 19x 2
5 5 10 x+3 x+5 x + 8x + 15
m 1 3m
24. a 2 42. 2
2 3
2
9 m+ m m +m m 1

25. 1 + x 43. m + 1 + m + 2 + 2
2m
x m 4 m 5 m 9m +20
x 2x x2
26. 1 + x 44. + 2
x 2 2x + 3 2x 3 4x 9

27. a 7a 2a
2 1 3
45. 2
+ 2
2
4 3 x + 7x + 12 x 16 x 9

28. 3 4 5 46. x4 2x 4 4 2x
+ + 2
2
2
2a 3a 6a 3x + 12x x + x 12 2x 6x
2
3x 4x 2x 3x
29. x 1 + x 2 + x 47. 2
+ 2
+ 2
4 3 2 3x + 15x x + 8x + 15 3x + 9x
1 1 2 1 2+ x 2 x
30. 48. + 2
2
2 3
a a a x+2 x + 4x + 4 x 4
3x 2x 11x 2x x2 x5
31. + 49. 2
2
+ 2
x 3 2x 6 2 x 2x 24 x 36 x 16
1 1 1 2x x3 x 12 3x
32. + 2
2
50. 2
+
z z z 1 3x + 15 x 25 6x x5

33. 3x 5 + 2x 7 + x 1
2
x1 x 1 x+1

III. Efecte las operaciones indicadas:

2a 6a a+3 1 ab 1 b
1.
a 3 a2 9

2a
2. 1+ : 1 1x 3. a+ +
x a+b 2 2a

lgebra en los nmeros reales 53

52-53. 53 18/11/02, 11:51 AM


CAPITULO 1

lgebra en los nmeros reales 53

54-55. 53 18/11/02, 11:53 AM


Ejercicios
2
3 5 3x x
1 1 + 2+
4. +1 : 1 2 5 5
x x 17. 24.
2 2
x 1
15 3
xy 1 x 10
5. 1
x+y 2y a a ab
a
2 3 a+b
a x a+x 18. 25.
6. + : 2 2
a
+
a
1
b
a+x a+x a + 2ax + x 2 3 a+b
7. 1 x1
ab : 1 1 1
a+b y
x+1 x
19. 26.
1 1 x1 1
8. : 1+ +y
1
x1 x+1
+1 x
x+1
2 1
9. a + ab : 1 1 1 1 1
2a a 27. 1 +
x2 x+2 1
2
20. 1+
10. m + mn 1 1 x
m
: 1+ m 2
x 4 ab a+b
2
11. 2a 3b b 1 a

b 28. a a
5b b 1 2 b a 1
21.
2 1 1 a
12. 2a 4 c 2+ c
2+ c 4b
: 2c
1 a b
x
2
1 2
13. a+b ab 2ab 22. 1 + 29.

2 2 1 x
b a a +b 1+ 3
x 3
14. b a ab 2
a
+ 2
b
: 1
a 1 1
23. x 1
2

x x 2 1 30.
15. + 1 x 1 1 1
x+1 x1 1 1+
1 x+1
1 1
x

2 4
16.
1 1
+
2 4

Soluciones
I.
1. 30 2. 6a 3. 3xy 4. 6xy 5. m2n2
6. mn 7. x2y2 8. a2 9. x2y2z 10. xy2z2
11. 2
20p q 2 12. 30p6q6 13. a2 b2 14. 2a + 4 15. 3a2 12
16. 2
18m + 6m 17. x a2
2 2
18. x + 3x + 2
19. a2b + ab2 20. (a b)2 (a + b) 21. x2 + 5x + 6
22. 2 (x 2) (x 3) 23. a (a 1) (a + 1)2 24. (a 2) (a + 2)2
25. (x + 2) (x 2) (x + 7) 26. 3a (a2 1) 27. p (p2 25)
28. 4 (x + 1)2 29. 5 (t2 25p2) 30. (x y) (x + y)2
II.
14
1. 2. 2 7
3. 4. a 2 12
5.
11 a 3x 4

54 lgebra en los nmeros reales

54-55. 54 18/11/02, 11:55 AM


CAPITULO 1
CAPTULO

a2 3p + q
6. 3 7. x + 1 8. 7x + 17 9. 10.
x+6 2x 3 a+2 2
p +1
3 4x + 3 1
11. 1 12. 4 13. 14. 15.
(x + 2) (x + 3) x+3 x 11

16. 29 17. 3a
18. 13a 19. b 20. 2x
30 2 12 3 7
2
a 18
21. 7a 22. 7a
23. 3x 24. 25. 1 + x
6 6 9 x
10
2 2
26. 2 + x 27. 17a 28. 11 29. 13x 11 30. a + a 2
2x 12 3
3a 12 a
2 3 2
41x 11x z z1
31. 32. 2 2
33. 4x 2x 11
2x 6 z (z 1) 2
x 1
2 2 3 2 3 2 2 2 2 3
34. 3ab + b 5a 35. a + 2 a + a + 4 36. x 3x y + 5xy 2xy + 2x 4y y
3ab 2 2
3(a 4) (x + y)(x y)
2 2 3 2
5x + 2x 2
37. 16a 17a 331 38. 3 2 39. 2x + 17x 2x + 19
2 x + x 4x 4 2
( a 25) (a 6) (x 1) (x 9) (x + 2)
4 3 2
3x 12x + 11x 98x + 186
40. 41. 0
2 2
2 (x 16)(x 1)
3 2 2 2
2m m m + 1 2m 4m 13
42. 2 2
43. 44. 2x 8x 2
(m 4) (m 5) 2
m ( m 1) 4x 9
14x + 63 2 3 2
45. 46. 2x + 11x 12 47. 2x + 22x 6x
x2 16 x2 9 3x (x + 4) (x 3) 3x (x + 3) (x + 5)
3 2 3 2
48. 3 49. 2x + x 68x +148 50. 21x + 76x + 43x 300
2 2 2
x+2 (x 36)(x 16) 6x (x 25)
III.
a
1. 2. x
+1 3. a
+ 2b 4. 1
+x
a3 x1 2 1x
x1
5. 6. a + x a2 b2
7.
2x2
8. x
x+y a+b1 x2 + x 2
a2 + ab
9. 10. m
2 + mn
11. 4a 15b3 15b2
ac
12.
2a 2 m+1 10b b

13. 2 1
14. 15. x2 + 1 1
16.
b 3
75
17. 1
18. 1
19. 20. 4
4 5 x

2x + 1 2 2x + 20
21. a b 22. 23. x 24. 6x

x+1 31 x2
25. a 1 xy
26. x+1
27. 28. 2b
1 + xy 2x + 1
2
29. 12
3x x 2
30.
18 2x x2 + x 2

lgebra en los nmeros reales 55

54-55. 55 18/11/02, 11:57 AM


CAPITULO 1

lgebra en los nmeros reales 55

56-57. 55 18/11/02, 11:59 AM


Prueba de seleccin mltiple

Marque la alternativa correcta.


1. Si a = 1 y b = 2 5. 3p 2pq =
el valor de a ab es:
A. 5p2q
A. 1 B. 6p2q
B. 2 C. 6pq
C. 1 D. 6p2q
D. 3 E. 6pq2
E. 2
6. Si p = 1 y q = 1 entonces
p + q + pq es:
2. Al reducir la expresin
a A. 1
a se obtiene:
2
a B. 1
A.
2 C. 0
a D. 2
B.
2
C. a E. 2

D. 0
7. Si p + q = 6 y q = 2 entonces el valor
E. 1 de p es:
2
a A. 6
3. Al reducir 2a a
2 B. 8
se obtiene:
C. 8
A.
a D. 4
2 E. 4
B. 1
2 8. Si m + 5n = 5 y n = 2 entonces el
C. a valor de m es:
2 A. 15
D. 3a B. 05
2
1 C. 0 5
E.
2 D. 15
E. 10
4. Si m = 2 y p = 3 entonces
m2 p2 es: 9. Si a = 5 y a + b = 5 entonces el valor
de b es:
A. 5
A. 0
B. 5
B. 10
C. 13
C. 05
D. 13
D. 5
E. 2 E. 10

56 lgebra en los nmeros reales

56-57. 56 18/11/02, 11:59 AM


CAPITULO 1
CAPTULO

n
10. Si m = y n = 16 14. La expresin el cuadrado
2
entonces el valor de m de la diferencia entre a obtiene:
es: y b es:
A. 2a
A. 32 A. (a b)2 B. 2a
B. 32 B. a2 b2 C. 2b
C. 8 C. a b2 D. 2b
D. 8 D. 2(a b) E. 0
E. 4 ab
E. 19. Al reducir
2
s 3m [2m (3p + m) p]
11. Si q = 2r, r = y
2 15. El doble del producto
s = 9 entonces el valor se obtiene:
entre a y b corresponde
de q es: A. 2m 4p
a:
B. m + 2p
A. 9
A. 2a2b C. 2m + 4p
B. 9 B. 2ab2 D. 2m + 2p
9
C. C. 2a2b2 E. 4p
2

D. 18 D. a2b2 1 2 1
20. a+ a aa
E. 2ab 2 3 3
9 es igual a:
E.
2
16. Al reducir a
12. La expresin el doble del A.
2a [ a (a 2a) ] 6
cuadrado de a corres- a
se obtiene: B.
ponde a: 6
A. 2a a
A. (2a)2 C.
B. 0 2
B. 2 (a2)2 a
D.
C. a 2
C. 2a2
D. 4a a
D. (2a2)2 E.
3
. a2 E. 4a
21. a a2 a 2 =
13. La expresin el cubo 17. Al reducir
A. a
de la mitad de a corres- (a + b) (a b) se
B. a 3
ponde a: obtiene:
C. a 4
3a3 A. 2b
A. D. a3
2 B. 2b
3
E. a5
B. a
C. 2a
2
a
2 D. 2a 22. ab2 ab2 =
C.
3 E. 0
A. 0
D. a 3
2 18. Al reducir B. a2b2
3a (a b) (a + b) se C. a2b4
E.
2
lgebra en los nmeros reales 57

56-57. 57 18/11/02, 12:00 PM


CAPITULO 1

lgebra en los nmeros reales 57

58-59. 57 18/11/02, 12:02 PM


Prueba de seleccin mltiple

D. a2b4 C. x2y D. a(a + b + 1)


E. 2a2b4 D. 2xy2 E. (a + b) (a + b + 1)
E. 2xy2
23. 2m 3m 4mp2 = 34. Para que la expresin
A. 24m3p3 29. Al factorizar m2mn se
9a2 + 12ab + .....
B. 24m3p3 obtiene:
sea un cuadrado de
C. 24m3p2 A. mn(m 1)
binomio falta:
D. 24m3p2 B. m2(m n)
A. 4b2
E. 9m3p3 C. m(m n)
B. 4b
D. m(1 n)
24. x [2x 3y + (3y 2x)] = C. 4
E. m2 (1 n)
A. 3x 6y D. b2
B. 4x 6y 30. Al factorizar 4 p2 se E. 9
C. 4x + 6y obtiene:
2
D. x A. (2 p)2 35. a b c =
abc
B. (2 p) (2 + p)
E. x 1
C. (p 2) (p + 2) A.
c
25. a (a2 + a3) = D. (4 p)2 B. c
A. a6 E. 2p (2 p) C. 1
B. 2a6
D. abc
31. La expresin 1 p6 es
C. a7
equivalente a: E. 1
D. a2 + a3 abc
E. a3 + a4 A. (1 p3) (1 p2)
B. p3 (1 p2) 36. a+ ab =
26. m(1 + m) m(1 m) = ab
C. (1 p3) (1 + p3)
A. ab
A. m2
D. (1 p3)2
B. 2m2 B. a
E. (1 p2)3
C. m m2 C. a + 1
a
D. m + m2 32. Factorice
D. b + 1
E. 0 m2 n2 m n = b
E. b
27. a(1 + a + a2) a = A. (m n) (m2 + n2)
B. (m + n) (m n 1)
A. a + a2 2 2
37. m n =
B. a + a3 C. (m n) (m n 1) m n

C. a + a2 + a3 D. (m + n) (m n + 1) A. m n
D. a2 + a3 E. (m n) (m n + 1) 1
B.
E. 1 + a + a2 m n
33. (a + b) + (a + b)2 =
C. m + n
28. xy (x + 2y) 2xy2 = A. 3(a + b)
1
D.
A. x2y + xy2 B. 3(a + b)2 m+ n
m+ n
B. xy2 C. 3(a2 + b2) E.
m n

58 lgebra en los nmeros reales

58-59. 58 18/11/02, 12:03 PM


CAPITULO 1
CAPTULO

2
38. a 1 : a 1 = 41. x 11x + 28 = D. 1
B. 3m
a a x7 2 n
E. a 2
A. a + 1 A. x 4 C. 3m
x7 n
B. a 1 B. x 7 3
x +y
3
3m
44. = D.
1
C. x 4
2
x xy + y
2 2n
C.
a +1

D. 1 D. x + 4 A. x + y E. 3m
a1 1 2
E. x + 7 B.
x+y n+ 1 n
E. 1 47. a + a
=
a
2 C. x y n
a
x+5 A. an
42. =
2 2 2
x 25 1
39. 3xy 3x y = D. B. a
3xy A. x + 5 xy
C. a + 1
A. 3 ( x y)
B. x 5 E. x + y D. an+1
B. 3 (y x) xy
E. an1
C. y x 1
C. a b
x+5 45. + = 48. 56 + 56 + 56 + 56 + 56 =
D. x y a+b a+b
E. y 3x 1 A. a A. 530
D.
x5
B. (a + b) B. 57
E. 1 C. 256
2 2 C. (a + b)2
40. a b = 2
x 5 D. 2530
4 4
a b D. 1
1 2 n n+ 1
A.
49. 3 4 4
a 4
2 2 43. = E. a+b =
a b 2 n
1 a + 3a + 2 2
a +b
2
4
B.
a +b
2 2
A. 3 4n+1
A. a 2
C. a2 b2 a+1 B. 2
46. m 2m =
D. a2 + b2 2n n C. 1
B. a + 1
a2 3m D. 1
1
E. A.
ab C. 2 2n E. 0

Soluciones

1. D 8. A 15. E 22. C 29. C 36. D 43. A


2. B 9. B 16. B 23. C 30. B 37. C 44. A
3. C 10. D 17. A 24. E 31. C 38. A 45. D
4. B 11. B 18. D 25. E 32. B 39. C 46. A
5. D 12. C 19. C 26. B 33. E 40. B 47. C
6. A 13. D 20. B 27. D 34. A 41. C 48. B
7. C 14. A 21. A 28. C 35. B 42. D 49. D

lgebra en los nmeros reales 59

58-59. 59 18/11/02, 12:05 PM


CAPTULO 2
E
cuaciones
e inecuaciones
de primer grado

2.1 Ecuaciones

Definicin: Se llama ecuacin a una igualdad que presenta


incgnitas y que es verdadera slo para algunos valores
de la incgnita:

Ejemplos: 2x 5 = 3
4x + 2y 1 = 0
5x2 3x + 2 = 0
ax by = 3 ab
2x3 1 = x2 + 2
Observacin 1: La expresin de la izquierda del signo igual
se denomina primer miembro y la del lado derecho se llama
segundo miembro.
Observacin 2: Una ecuacin puede tener una o ms incgnitas.
Observacin 3: Se llama grado de una ecuacin al grado del
trmino que presenta el grado ms alto, despus que se hayan reducido
los trminos semejantes.
Ejemplos: 2x 1 = 5 es ecuacin de primer grado
x2 x = 7 + x es ecuacin de segundo grado
x4 x + 2 = 0 es ecuacin de cuarto grado
2xy + 5 3x + 2y = 0 es ecuacin de segundo grado
x2 + 5x 1 = (x 2)2 es una ecuacin de primer grado

60 Ecuaciones e inecuaciones de primer grado

60-61.(2003) 60 18/11/02, 12:29 PM


CAPTULO 2

Observacin 4: Se llama raz o solucin de una ecuacin a todo


valor de la incgnita que verifique la igualdad.
Resolver una ecuacin significa encontrar el o los valores de la o
las variables (incgnitas) para que la igualdad sea verdadera.
Para resolver una ecuacin debemos tener presente las siguientes
propiedades de la igualdad.
Propiedad 1: Al sumar o restar la misma cantidad en ambos
miembros de una igualdad, la igualdad persiste.
Propiedad 2: Al multiplicar o dividir por una misma cantidad
distinta de 0 en ambos miembros de una igualdad, la igualdad
persiste.
Propiedad 3: Al elevar a una potencia distinta de 0 ambos
miembros de una igualdad, la igualdad persiste.
Propiedad 4: Al extraer la misma raz, en ambos miembros de una
igualdad, la igualdad persiste.

2.1.1 Ecuaciones de primer grado con


coecientes enteros

1. Resolver la ecuacin 3x + 2 = 7 Ejercicios


Debemos despejar x. Para ello restamos 2 en ambos miembros resueltos
(Propiedad 1).
3x + 2 2 = 7 2
efectuamos las operaciones 3x = 5

3x 5
dividimos ambos miembros por 3 (Propiedad 2) =
3 3
5
efectuamos las operaciones x=
3
Verificamos:
3x + 2 = 7
5
3 +2 =7
3
5+2 =7

77
2. Resolver la ecuacin 3 2x = 5x 9
Despejamos x. Para ello restamos 3 y restamos 5x en ambos miembros.
3 2x 3 5x = 5x 9 3 5x
efectuamos las operaciones
2x 5x = 9 3
7x = 12

Ecuaciones e inecuaciones de primer grado 61

60-61.(2003) 61 18/11/02, 12:30 PM


CAPTULO 2

Ecuaciones e inecuaciones de primer grado 61

62-63. 61 18/11/02, 12:34 PM


Ejercicios multiplicamos por 1
resueltos 7x = 12
dividimos por 7 7x = 12
7 7
12
x =
7
Verificamos: 3 2x = 5x 9
12 12
32 =5 9
7 7
24 60
3 = 9
7 7
21 24 60 63
=
7 7 7 7
3 3
7 7
3. Resolver la ecuacin 2 (x 1) = 3 (x + 2) 5 (x + 3)
Primero resolvemos los parntesis y reducimos trminos
semejantes.
2x 2 = 3x + 6 5x 15
2x 2 = 2x 9
sumamos 2 y sumamos 2x en ambos miembros
2x 2 + 2 + 2x = 2x 9 + 2 + 2x
4x = 7
dividiendo ambos miembros por 4
4x 7
=
4 4

7
x =
4
Verificamos:
2 x 1 =3 x+2 5 x+3
7 7 7
2 1 =3 +2 5 +3
4 4 4
11 1 5
2 =3 5
4 4 4
22 3 25
=
4 4 4
22 22

4 4

4. Resolver la ecuacin
2x2 2 (x + 1) (x 1) = (x 3)2 (x + 2) (x 5) + 1
Primero resolvemos parntesis y reducimos trminos semejantes.
2x2 2x2 + 2 = x2 6x + 9 x2 + 3x + 10 + 1
2 = 3x + 20

62 Ecuaciones e inecuaciones de primer grado

62-63. 62 18/11/02, 12:35 PM


CAPTULO 2

restamos 2 y sumamos 3x en ambos miembros


2 2 + 3x = 3x + 20 2 + 3x
3x = 18
dividimos por 3 en ambos miembros
3x 18
=
3 3
x = 6
Verificamos:
2x2 2 (x + 1) (x 1) = (x 3)2 (x + 2) (x 5) + 1
2(6)2 2(6 + 1) (6 1) = (6 3)2 (6 + 2) (6 5) + 1
2 36 2 7 5 = 32 8 1 + 1
72 70 = 9 8 + 1
2 2

Ejercicios
1. x + 3 = 5 21. 2 (x + 3) = 5 (x 1)
2. 2x 5 = 7 22. (2x 5) 2 = (3 + x) 5
3. 5 2x = x + 2 23. (x + 2) (3x + 2) = 5 (x + 4) + 1
4. 2y + 1 = 3y + 4 24. 5 (1 x) + (x 3) 4 = (x 1) (1 x)
5. 6z 3 = 5 + 2z 25. 2[(3x + 1) 2 (x + 4)] (3x + 5) = 0
6. 4x 5 + x = 3 + 2x + 4 26. 2x 3 (x + 1) = [x + 3 (x + 2)] (x + 4)
7. 4 + 2x x = 3x 4 27. 3 + x 5 [(2x + 4) (x + 2)] = x + 2
8. y + 5y 3 + 4 = y 1 28. 2x 10 [2x (x + 3) + 5] = 0
9. y + 2 = 5y 4 + 3y 1 29. [2 (2 y) (2y 3)] 5y = 4 (y + 3)
10. 4y + 9 y 2y = 16y + 42 30. 3 [5y + 2 (y 1) + 4] = 5 [2 (y3) 3 (y2)]
11. 17y y + 9 = 32 19y + 82 31. x + [12x 3 (x + 1) (3x + 2)] = 15x 16
12. 45x 33x + 19 = 25x + 17 32. 2x [14x 2 (x + 3) (2x + 3)] = 16x + 9
13. z 12 + 44z = 18 15z 33. [2 (3x 3) (1 + x)] [5 (3 2x) (1+ x)] = 0
14. 132x + 25 33x = 10 x + 85
15. 49x 105 + 16x = 48x 301 8
16. 405x 203 + 45x = 102 + 115
17. 18z 42 + 15z = 10z 3 + 32z 39
18. 113x + 16 14x = 12 + 27x + 19
19. 15x 135 + 18x = 45 + 90 18x + 15x
20. 339x + 25 = 5 + 309x + 20

Ecuaciones e inecuaciones de primer grado 63

62-63. 63 18/11/02, 12:35 PM


CAPTULO 2

Ecuaciones e inecuaciones de primer grado 63

64-65.(2003) 63 18/11/02, 12:37 PM


Ejercicios
34. [ 5 + 3x 2 (x + 1) (x + 3)] = [ 2x + 5 (x + 1)] ( 2)
35. 2x + {3x [5x + 2 (x 1) 3 (4x 5)] + 3} 2 = 0
36. 5x {1 + (3x 5) 2 [3 (x + 4) 2x] (x + 5)} = 17
37. 3 {x + 1 2 (x + 3)} {6x [2x + 3 (x 1)] + 1}3 = x + 1
38. 1 { 3 [2x (x + 4) (6 2x) + 5] 3x} + 2 = 0
39. 4 (x + 2) {3 (x 1) [5 (1 x) 6x]}2 = [3 (x 4) + 1]2
40. 26 {32x [1 x + 2 (18x 3) + 6x]} = 12x 4
41. (x + 1)2 = 12 + (x 5)2
42. (x + 3) (x 1) = 5 + (x 2)2
43. (3 x) (x + 4) + 16 = 12x (x + 3)2
44. (x 1) (x + 1) (x + 2) (x + 3) = 5x 1
45. 2 (y 2) (y + 2) (y 5) (y + 3) = (y 1)2 + 2
46. y (2y 3) + (y 1)2 = 2y (y 4) + 3 + y2
47. 3x (x 5) (x 3) (x + 3) = 2 (x + 5)2
48. (2x 5) (4x + 3) (2x 1)2 = 3x2 + (x + 4) (x 4)
49. { [ (x + 5)2 + (x + 1) (x 1)] + (2x 3) (2x + 3)} = (2x + 1)2
50. x (x + 5)2 + 2x3 4x2 = 3x (1 + x)2 + 22

Soluciones

1. x=2 2. x=6 3. x=1 4. y = 3 5. z=2

2 11
6. x=4 7. x=2 8. y= 9. y = 1 10. y=
3 5
2 1 1
11. y=3 12. x= 13. z= 14. x = 2 15. x = 12
13 2
14 1 15
16. x= 17. z=0 18. x= 8 19. x = 20. x=0
15 2
11 5
21. x= 22. x = 25 23. x=3 24. x = 25. x = 19
3 3
1
26. x=1 27. x=3 28. x = 12 29. y = 19 30. y=
5 2
11 7 10
31. x= 32. x=0 33. x= 34. x = 3 35. x=
10 6 9
23 25
36. x=9 37. x=4 38. x=1 39. x = 40. x=
15 3
37 3
41. x=3 42. x=2 43. x= 44. x = 45. y=1
7 5
2 41
46. y=
3
47. x=
35
48. x=0 49. x = 8 50. x=1
3

64 Ecuaciones e inecuaciones de primer grado

64-65.(2003) 64 18/11/02, 12:38 PM


CAPTULO 2

2.1.2 Ecuaciones de primer grado con


coecientes fraccionarios

1. Resolver la ecuacin Ejercicios


0,5x 0,7 + 0,3x 1,5 = 0,6x 4 + 1,7x resueltos
Reducimos los trminos semejantes
0,8x 2,2 = 2,3x 4
sumamos 2,2 y restamos 2,3x
0,8x 2,2 + 2,2 2,3x = 2,3x 4 + 2,2 2,3x
1,5x = 1,8
dividimos por 1,5
1,5x 1,8
=
1,5 1,5

x = 1,2
Verificamos :
0,5x 0,7 + 0,3x 1,5 = 0,6x 4 + 1,7x
0,5 1,2 0,7 + 0,3 1,2 1,5 = 0,6 1,2 4 + 1,7 1,2
0,6 0,7 + 0,36 1,5 = 0,72 4 + 2,04
1,24 1,24
2. Resolver la ecuacin
1 3 5 1
x + x + 2 = x 3
3 4 6 5
El mnimo comn mltiplo de los denominadores es 60.
Multiplicamos ambos miembros por 60 para dejar todos los coefi-
cientes enteros.
1 3 5 1
x 60 + 60 x 60 + 2 60 = x 60 3 60
3 4 6 5

20x + 45 50x + 120 = 12x 180


reducimos trminos semejantes
30x + 165 = 12x 180
restamos 165 y restamos 12x en ambos miembros de la ecuacin.
30x 12x = 180 165
42x = 345
dividimos por 42
345
x=
42

Ecuaciones e inecuaciones de primer grado 65

64-65.(2003) 65 18/11/02, 12:39 PM


CAPTULO 2

Ecuaciones e inecuaciones de primer grado 65

66-67. 65 18/11/02, 12:42 PM


Ejercicios simplificamos por 3
115
resueltos x=
14
Verificamos: 1 3 5 1
x+ x+2 = x 3
3 4 6 5
1 115 3 5 115 1 115
+ +2 = 3
3 14 4 6 14 5 14

115 3 575 23
+ +2 = 3
42 4 84 14
230 63 575 168 138 252
+ + =
84 84 84 84 84 84
114 114
=
84 84

3. Resolver la ecuacin
3 x 2x + 1 x+2
=2
5 3 10

Para eliminar los denominadores multiplicamos por el mnimo


comn mltiplo de 5 , 3 y 10, que es 30

(3 x) 6 (2x + 1) 10 = 60 (x + 2) 3
Resolvemos los parntesis y reducimos trminos semejantes.

18 6x 20x 10 = 60 3x 6
26x + 8 = 54 3x

sumamos 3x y restamos 8

26x + 3x = 54 8
23x = 46

dividimos por 23
46
x=
23

x=2

Verificamos: 3 x 2x + 1 x+2
=2
5 3 10
3 + 2 4 + 1 2 + 2
=2
5 3 10
5 3
+ =2 0
5 3

22

66 Ecuaciones e inecuaciones de primer grado

66-67. 66 18/11/02, 12:43 PM


CAPTULO 2

Ejercicios

3 x
1. x=2 2. =1
4 7

2x x
3. =4 4. =0
5 8

5. x + 1 = 4 2x 3x x 1
6. + =
4 3 5 4 10 4

5x 2x x 9 1 1 1
7. + = +5 8. x x+ =1
3 5 4 20 2 3 2
x 3x 1 31 7x 5x 6x 5 9
9. + =x 10. + + =
3 2 5 30 4 3 5 6 20

11. 3 8x
+
7x
+
3x
=
7 12. 3x x + 1 = x + 1
4 3 5 2 12 4 2 8 3 12
1 x 2x 3x 2
13. 4x + 3x x + 11 = 0 14. + + =
5 4 30 10 5 7 10 175

15. 2 x 6 x + 3 x + 5x = 1 77 16. 0,5x + 3,2x - 5,4x + 0,9x + 1,6 = 0


3 5 4 2 240

17. (x + 3,5) - (4,2 - 2,3x) - (4,8x + 2,1) = -1,3 18. 0,7x + 0,3x - 0,1x + 2,6x + 10,5 = 0

19. 3 x + 0,5x 1 = 3 x 0,3x + 0,9 20.


5 2
x + x + 0,6 = 0,5x + 1,3x
4 5 4 5

21. x 3 1 = x 1 + 1 22. 3 x + 2 x 1 + 1 = x + 1 + 2 x + 6 + 2
4 3 3 8 4 5 3 4 5

23. 2 x 3 4 x 1 + 1 = 3x + 2 x 1 + 17 24. 8 2 x x + 3 26 = 3x + 2 1 x
4 3 3 2 300 3 6 27 3 9
4 3x 2x 3 1 3x 1
25. + + = 26. x 1 + 3x + 2 5x 2 = 11
5 15 2 10 20 3 7 3 105

27. 1 + 4x 3 5x + 2 + 1 = 3x + 5 + 6x + 1 2 28. 4x 2 x + 1 3 = 3x x 1 + 11
2 4 9 12 3 9 3 5 15
x x 3x 7
29. 3 + x +7= 1 x 4x
4 3 2 12 2+ 5
30. 3 2x 2 11 3
+ + =
31. 3 x + 1 4 x 5 13 = 5 2 x 4 x + 11 5 3 60 4
2 3 4 3 12
2x x
4 13 +
2 4 32. 12 x 5 4 1
x+ 8x + =
33. 3 5 5 6 9 18 9
+ =0
5 3 6

x 2 x 2 11
34. (x 5) (x + 5 ) x + = 3x + x 27
4 3 2 12

35. 3 x + 1 4 x 5 = 2 + 5 x + x
4 2 3 3 4 3 2 4

3 2x x
4
36. 5 3x 3 4 8 207 1 4
+ = 8
3 4 30 120 2 5

Ecuaciones e inecuaciones de primer grado 67

66-67. 67 18/11/02, 12:45 PM


CAPTULO 2

Ecuaciones e inecuaciones de primer grado 67

68-69.(2003) 67 18/11/02, 12:46 PM


Ejercicios
37. 12 13x + 5 15x + 17 21x = 3 1
3 4 3 156
38. 5 4x + 3 + 2x 5 3x = 67 1
3 4 1 12
2 5 2
3x 1 4x 2 4 2
+ 1x x+9
39. 4 2 5 3 5 3 3
5
4 = 2
3
+ 20

40. 4 x+1 13 3 3x 1 5 2 2x + 5
3 2
90 5 6
+ 4 = 72 9 4

41. 18 1 3x + 42 5 + 2x = 25 3x + 9 61 x + 1
15 3 45 3 30 8 80 3
13 x + 2 14 x 3 27 x + 4 143
42. + + 33 =0
7 5 4 560

43. 5x 7 6x + 2 = 27x 9 12x + 7 + 2 2


3 5 6 15 15

44. x+2 x+3 2x 1 5x + 3


2
20
= 14
2
2x + 5 x+1 x1 17 x 1 x + 2 2
45. + = 12 3
5 3 15
2
3x 1 x + 4 2x 5 4x + 9
46. =
3 4 12
2 3

47.
x x+5 x1 7x + 3 x 1 51 6 4 x3
4
3
= 2
+ 2
+ 12
3 3
x+1 x1 x+7 27 x + 2 6x 1
48. = 2 + 10 +
5 5 5
2 3 2

49. 2 x 1 x2 x+1 1 5x2 x3 1 7


3
+ 4
4
= 12
+ 4 12
2 2

50.
x 2 x3 9 x2 + 3x 5 81
5
+ 4
= 20
20

Soluciones
8
1. x = 3
2. x = 7 3. x = 10 4. x = 0 5. x = 44
3
6. x = 1 7. x = 3 8. x = 3 9. x = 5 10. x = 1
5 1 2 3
11. x =
7
12. x = 2 13. x =
3
14. x =
5
15. x = 1
4
16. x = 2 17. x = 1 18. x = 3 19. x = 2 20. x = 4
21 12 1 3
21. x =
2
22. x = 1 23. x =
25
24. x = 3
25. x = 2
4
26. x = 5
27. x = 0 28. x = 8 29. x = 5 30. x = 2
1
31. x = 17 32. x = 20 33. x = 2
34. x = 1 35. x = 3
68
12 245
36. x = 6 37. x = 13
38. x = 12 39. x = 3 40. x =
316
3 87
41. x = 2 42. x = 43. x =
97
44. x = 7 45. x = 2
4
46. x = 1 47. x = 4 48. x = 2 49. x = 1 50. x = 3

68 Ecuaciones e inecuaciones de primer grado

68-69.(2003) 68 18/11/02, 12:48 PM


CAPTULO 2

2.1.3 Ecuaciones fraccionarias


de primer grado
Se denominan ecuaciones fraccionarias aquellas que presentan
incgnita en el denominador.
Para resolverlas usamos la tcnica de multiplicar por el mnimo
comn mltiplo de las expresiones que son denominadores o
simplificar al mximo las fracciones.
Debemos poner especial atencin en que la solucin que
aparezca no vaya a indeterminar alguna expresin, es decir, no vaya
a hacer cero algn denominador. En un caso as la ecuacin no
tiene solucin. Ver ejemplo 5.

1 3 1 13
1.
x
+ 2 x = 3x + 12 Ejercicios
Multiplicamos por el mnimo comn mltiplo m.c.m. que es 12x resueltos
12 + 18 = 4 + 13x
despejando x
13x = 26
x=2
Verificamos:

1 3 1 13
+ = +
x 2 x 3x 12

1 3 1 13
2
+ 4 = 6 + 12

5 15
4
= 12

5 5

4 4
2. Resolver la ecuacin
1 3 5
= 2
x + 3 2x 1 2 x + 5x 3
Primero tratamos de factorizar al mximo las expresiones del
denominador. Observamos que 2x2 + 5x 3 = (x + 3) (2x 1)
As, el m.c.m. es 2x2 + 5x 3
multiplicamos ambos miembros de la ecuacin por el m.c.m.
(2x 1) 3 (x + 3) = 5

Ecuaciones e inecuaciones de primer grado 69

68-69.(2003) 69 18/11/02, 12:49 PM


CAPTULO 2

Ecuaciones e inecuaciones de primer grado 69

70-71. 69 18/11/02, 12:51 PM


Ejercicios resolvemos parntesis y reducimos trminos semejantes
resueltos 2x 1 3x 9 = 5
x = 15
x = 15
Verificamos:
1 3 5
=
x +3 2 x 1 2 x2 + 5 x 3
1 3 5
= 2
15 + 3 2 ( 15) 1 2 ( 15) + 5 ( 15) 3
1 3 5
=
12 31 372
5 5

372 372
3. Resolver la ecuacin
2x 1 x+3 x2 3
=
x+2 x+5 x2 + 7x + 10
Primero factorizamos todas las expresiones posibles de factorizar
2x 1 x + 3 x2 3

x+2 x+5
= x+2 x+5
m.c.m. : (x+2)(x+5). Multiplicamos toda la ecuacin por el m.c.m.

(2x 1) (x + 5) (x + 3) (x + 2) = x2 3

desarrollamos los parntesis y reducimos trminos semejantes.


(2x2 + 9x 5) (x2 + 5x + 6) = x2 3
2x2 + 9x 5 x2 5x 6 = x2 3
4x = 8
x=2
Verificamos:
2x 1 x+3 x2 3
=
x+2 x+5 x2 + 7x + 10

3 5 1
=
4 7 28
1 1

28 28
4. Resolver la ecuacin
1
11x2
4x +2 3x + 1 5x 1 2
12
2 x 3 = 4 12x 18
Vemos que 12x 18 = 6 (2x3) y que 4 est contenido en 12, luego,
el m.c.m. de los denominadores es 12 (2x 3).
Multiplicamos por este m.c.m.
(4x + 2) (2x 3) (3x + 1) 12 = (5x 1) (2x 3) 3 (11x2 1) 2
2

70 Ecuaciones e inecuaciones de primer grado

70-71. 70 18/11/02, 12:52 PM


CAPTULO 2

Resolvemos los parntesis y reducimos trminos semejantes.


(8x2 8x 6) (36x + 12) = (30x2 51x + 9) 22x2 + 1
8x2 8x 6 36x 12 = 30x2 51x + 9 22x2 + 1
7x = 28
x=4
Verificamos:
1
11x2
4x + 2 3x + 1 5x 1 2
12
2x 3
= 4

12x 18

1
176
18 13 19 2
12
5 = 4 30
3 13 19 176 1
= 4 30 + 60
2 5
11 11

10 10
5. Resolver la ecuacin
2 2x 3x 4
+ +3= 2
x1 x+1 x 1
Para eliminar los denominadores multiplicamos por el m. c. m / x2 1
2 (2 x) (x + 1) + (3 x) (x 1) + 3 (x2 1) = 4
4x + 4 2x2 2x + 3x 3 x2 + x + 3x2 3 = 4
reducimos trminos semejantes
6x = 6
x=1
pero ocurre que si x = 1 se indeterminan dos de los trminos
de la ecuacin: 2 2 x y 4 ; por lo tanto, la ecuacin plan-
x1 x2 1
teada no tiene solucin.

Ejercicios
1 3 4 5 3
1. 1 = 3 1 6. 2 x 5 + 4 3 2 x = 0 11. =
x 3x 1 2 x + 3 6 x2 + 7x 3
3 1 3 4 2 1 x+4
2. = 1 7. =
5x 1 6 7x
12.
3 x+4
+ 4 x5 = 2
x 2 6 x 6 x 120
4
3. 3=0 1 8. 1 + 3 = 11 13. 1

4
=
3x 1
5x 2x 2 5x 3x 2 5x + 1 15 x2 7x 2
x
1 3 6 3 3 4 6
4. =3 1 9. =0 14. =
2x + 1 4x 5x x2 x + 2 x 2 x2 4
2 3 2
5. =0 10. x + 3 = 2 x 15. 2 2 x + 3 x + 3 = 1
x1 x+3 2x 1 x1 x+1 x2 1

Ecuaciones e inecuaciones de primer grado 71

70-71. 71 18/11/02, 12:53 PM


CAPITULO 2

Ecuaciones e inecuaciones de primer grado 71

72-73. 71 18/11/02, 12:58 PM


Ejercicios
16. 6 3 2 5
+ 2
= 2
3x 5 x 2x + 1 x1 3x 8 x + 5
17. 2 x + 3 6x+4
= 15x + 2
5x 1
18. 3 x + 2
=
2 x2
3
2
x+1 x 1 x1

19. 4x 7 = x 16
12x + 3 3x + 5

20. 3 16 + 2 2 x = x 3
2
x 1 x1 x+1

21. 4x 5 8 x 3 = 0
3x + 2 6x + 5

22. 3 5x 5 x + 2 = 0
1 3x 3x + 20
23. x3 125 x1
2
x+3 = 1 + x
x 2 x 15
2
24. x + 2 x + 1 + x 1 = x3 3
2
+ 2
x+1 x+3 x 2 x 15 2 x 4x 30
2
25. x x+1 5x + x + 4
+
3x + 2 2 x 1
= 2
6x + x 2
26. 2 3 1 5 7
+ + =
3x 3 4x 4 2x 2 6x 6 16
27. 2 2 x 13
=
2
x+2 3x 5 3x + x 10
28. x 1 4x2 + 3x 1
+ =
2x 6 6x + 3 8 x2 20x 12 3
3 2 x+1
29. 6x x+2 3x 1 2
3 2
2
= +
4x 12x x + 3 4x 1 2 x2 7x + 3 2 x2 5x 3
2
2+
2
1 1 1 2+

x 2 x 33. x 36.
2 2x 3x 8
=3 + x+1 + 3 =
30. = 2
2 x1 1 x2
3 1 2

x 3 5 + 3x 2
2
2 2 2
x 37. x + 1 3 = x + 3 11x
x+2 2 x2 2
2x 8
1 1 1 1 1

2 x x 3 1
31. = 1 38. 5x 1 12x2 3 2x
1 3 2 34. x 8 = 4x + 1

1 1
= 15 2 x 6 4x2 11x 3
3 x 3x + 5 +
4 1 12x2 17 5x 1 2x 3
1 1+ 39. = 2 x 6 + 4x + 1
1+ x 2
x 4x 11x 3
32.
1
=x 2 1 1
1 1
1 3x 6x x 40. x 2x 5x2 + 125
1+ 35.
1 4
= 1 2x + 3
+
x 5 2 x2 7x 15
=0
x
1+
2x 6x x

72 Ecuaciones e inecuaciones de primer grado

72-73. 72 18/11/02, 12:59 PM


CAPITULO 2
CAPTULO

Soluciones
1 4 1
1. x = 3
2. x = 6 3. x = 15
4. x = 3
5. x = 9
20 3
6. x = 9 7. x = 14 8. x = 10 9. x = 3
10. x = 5

11. x = 2 12. x = 4 13. x = 1 14. x = 20 15. x = 1


2
4 2 13
16. x = 3
17. x = 7
18. x = 0 19. x = 188 20. x = 3
19 25 No tiene 1 2
21. x = 17
22. x = 33
23. solucin 24. x = 25. x =
2 3
1 Ecuacin 13
26. x = 3 27. x = 28. x = 9 29. de 2 grado 30. x = 2
3 16
45 1 7 1
31. x = 16 32. x = 33. x = 2 34. x = 35. x =
2 23 2
36. No tiene 37. x = 1 38. x = 1 39. No tiene 40. x = 125
solucin solucin

2.1.4 Ecuaciones literales


de primer grado
Se denominan ecuaciones literales de primer grado aquellas que
presentan expresiones literales y/o numricas adems de la incgnita.
Generalmente para la incgnita se utilizan las letras x, y o z y para los
coeficientes u otros trminos, las letras a, b, c, m, n, s, t . . .
Resolver una ecuacin literal es despejar una de las letras que en
ella intervienen, generalmente x, y o z en funcin de las otras y/o los
nmeros que aparezcan. Para ello se usan los mismos procedimientos
que se utilizan para resolver ecuaciones numricas.

1. Resolver la ecuacin Ejercicios


a (x + 1) = a (a + 1) x resueltos
Resolvemos los parntesis y reducimos trminos semejantes
ax + a = a2 + a x
trasponemos trminos dejando todos los que contengan x en el
primer miembro y los que no la contienen en el segundo.
ax + x = a2 + a a
ax + x = a2
factorizamos por x y dividimos toda la expresin por (a + 1)
x (a + 1) = a2
a2
x=
a+1

Ecuaciones e inecuaciones de primer grado 73

72-73. 73 18/11/02, 1:01 PM


CAPTULO 2

Ecuaciones e inecuaciones de primer grado 73

74-75. 73 18/11/02, 1:15 PM


Ejercicios Verificamos:
resueltos a (x + 1) = a ( a + 1) x
a2 a2
a a + 1 + 1 = a a+1 a + 1
a3 a2
+ a = a2 + a / a + 1
a+1 a+1
a + a (a + 1) = (a + a) (a + 1) a2
3 2
a3 + a2 + a = a3 + a2 + a2 + a a2
a3 + a2 + a a3 + a2 + a
2. Resolver la ecuacin
x + b (bx + 1) = (1 + b) b (2x 1)
Resolvemos los parntesis y reducimos trminos semejantes.
x + b2x + b = 1 + b 2bx + b
x + b2x + b = 1 + 2b 2bx
trasponemos los trminos agrupando los que contienen x en el
primer miembro y los que no la contienen en el segundo.
x + b2x + 2bx = 1 + 2b b
factorizamos ambos miembros
x (1 + 2b + b2) = 1 + b
x (1 + b)2 = 1 + b
dividimos ambos miembros por (1 + b)2
1+b
x= 2
1+b
1
x = 1+b
Verificamos:
x + b (bx + 1) = (1 + b) b (2x 1)
1 b 2
1+b
+ b 1+b + 1 = 1+b b 1
1+b
1 b2 2b
+
1+b 1+b
+ b = 1 + b 1+b + b
1 + b2 1 + b 2 2b
=
1+b 1+b
1 + b2 1 + b2

1+b 1+b

3. Resolver la ecuacin
1+x 1+x 1+a 1b
b
+ a = b + a
Eliminamos los denominadores multiplicando la ecuacin com-
pleta por el m.c.m. = ab
a (1 + x) + b (1 + x) = a (1 + a) + b (1 b)
resolvemos parntesis y trasponemos trminos
a + ax + b + bx = a + a2 + b b2
ax + bx = a2 b2

74 Ecuaciones e inecuaciones de primer grado

74-75. 74 18/11/02, 1:16 PM


CAPTULO 2

factorizamos por x y dividimos por a + b


x (a + b) = a2 b2
a+b ab
x =
a+b
x = ab
Verificamos:
1+x 1+x 1+a 1b
b
+ a = b + a
1+ab 1+ab 1+a 1b
b
+ a
= b + a
2 2
a + a ab + b + ab b a + a2 + b b2
ab
= ab
a + a2 + b b2 a + a2 + b b2

ab ab
4. Resolver la ecuacin
m+x n+x
n
m =0

Para eliminar los denominadores multiplicamos toda la ecuacin


por el m.c.m. que es m n
m (m + x) n (n + x) = 0
m2 + mx n2 nx = 0
asociamos los trminos agrupando los que contienen a x
mx nx = n2 m2
factorizamos por x en el primer miembro y factorizamos el
segundo miembro
x (m n) = (n m) (n + m)
x (m n) = (m n) (n + m)
mn n+m
x =
mn
x = nm
Verificamos:
m+x n+x
n
m = 0

m+ nm n+ nm
= 0
n m
n m
m = 0
n
(1) (1) = 0
1 + 1 = 0
0 0
5. Despejar h en la ecuacin
1
A= (b1 + b2) h
2
En este tipo de ecuaciones literales que generalmente se trata
de frmulas que se usan en algn clculo de geometra, fsica
o qumica, se especifica cul de las variables se debe despejar
en funcin de las otras.

Ecuaciones e inecuaciones de primer grado 75

74-75. 75 18/11/02, 1:16 PM


CAPTULO 2

Ecuaciones e inecuaciones de primer grado 75

76-77. 75 18/11/02, 1:22 PM


Ejercicios Aqu debemos despejar h.
resueltos La ecuacin o frmula puede ser escrita as:

multiplicamos por 2
2 A = (b1 + b2) h
dividimos por (b1 + b2)
2A
b1 +b2
=h

con esto qued despejada h, tal como se peda.

Ejercicios

I.
8. ax 1 = bx + 1
x 9. (a 1) x + (b 1) x = (x 1) a + (x 1) b
1. =b
a
2. ax = ab 10. (1 a)2 (a + x)2 = (1 b)2 (b + x)2
3. ax 1 = b 11. a2 (x 2) b2 (x 2) = b (a2 b2)
4. 2ax a = a + 2x 12. m2x = n (1 + n) + x (m2 n2)
5. abx = a x (a2 + b2) b (ax 1) 13. (m 4) x + (m 5) x = (x 5) m + (x 4) m
6. ax 3 = bx 5 14. (a 4x) (2a x) (2x a)2 = (a + x) (a 1)
7. a2b + bx = ab2 + ax 15. (x + m)2 (x + n)2 = (m n)2

16. (a + b + c) x (a + b + x) c = (a + x + c) b (x + b + c) a
17. (x + 1) a (x + 1) b (x + 1) c = (a + b + c) x + a
18. 2 (x a) + 3 (x b) 4 (x c) = 3a + 2b c
19. (x + a) (x b) (x a) (x + b) = a2 b2
20. (a + b + c) (x a) + (a + b + c) (x b) + (a + b + c) (x c) = 0

x
21. x b = 25. 3 a + 1 = 0
a x 2 a

26. x a x b = 2ab 2b2


22. x + x = 1 xb xa x2 ax bx + ab
a b ab 2
9a
6a + b
x x b
23. ax b x = 1 + 1 27. =
b a a b b 3a 6a

24. 1 a = 2 28. 3 a x 2 b x = 2b 6 a
2 2
x x a b a ab

76 Ecuaciones e inecuaciones de primer grado

76-77. 76 18/11/02, 1:23 PM


CAPTULO 2

29. a x bx cx 1 33. 3 x + 2 b 3 = 4b2


a
+ b + c = 3 + ab c x 3b x2 9 b2

xa xb
30. + =2 34. 3x + 5a 6x 2b
= 4x 2a
x+b x+a 2x + b

bx a x 2x 1
31. =b xa +a 35. + =0
a 2m 3n mn

32. bx a
b xa =b 36. 3

2
=
3b
a 2x b 2x + b 4x2 b2

II. En los ejercicios siguientes se trata de despejar la variable indicada.


2
1. Si d = v t , despejar v. 18. Si T m v = 0, despejar m.
R
dv dm dv
2. Si F = m a , despejar m. 19. Si F = m + dt , despejar dt .
dt
v2
3. Si P = 2 r , despejar r. 20. Si x = , despejar g.
2mg
d 4
4. Si v = , despejar t. 21. Si F = , despejar t.
t 1+t 2
5. Si S = R , despejar R. 22. Si L = m r2 w, despejar r.

w 1
6. Si P = , despejar w. 23. Si d = 2
a t2, despejar a.
t
1 24. Si I = M R2, despejar R.
7. Si Ec = m v2, despejar m.
2
m
1
8. Si Ec = m v2, despejar v. 25. Si T = 2k k , despejar m.
2
9. Si Ep = m g h, despejar m. 26. Si L = m k r , despejar r.

2
10. Si m2 = a1 , despejar a1. 27. Si Z = x 1 + R2 , despejar x .
1
m1 a2
11. Si F = m g, despejar g. 28. Si Ec = h ( o), despejar o.

12. Si T = P + m a, despejar m. 29. Si 1 1 = h (1 cos a), despejar f.


f f mcc2

13. Si Fr = m a , despejar a. 30. Si P V = n R T, despejar T.

14. Si F + Fr mg sen = m a, despejar m. 31. Si Q = C (T2 T1), despejar T2.


Q2
15. Si N mg cos F sen = 0, despejar F. 32. Si E1 = , despejar Q2.
Q1 Q2
33. Si Q1 Q2 = 0, despejar T .
2
16. Si Fc = m v , despejar R. 1
R T1 T2
2 4p
17. Si v = g, despejar v. 34. Si T2 = g l , despejar l.
R

Ecuaciones e inecuaciones de primer grado 77

76-77. 77 18/11/02, 1:25 PM


Ejercicios
x1 + x2 + .... + xn
35. Si X = n , despejar n. 38. Si S = So + gt , despejar t.
xm
36. Si Z = , despejar s. 39. Si 1 = 1 _ 1 , despejar s.
s/ n f s s
40. Si 1 = 1 + 1 + 1 , despejar R.
37. Si Z = t s x , despejar t. R R1 R 2 R 3

Soluciones
b+1 a
I. 1. x = a b 2. x = b 3. x = a 4. x =
a1
1 2 2
5. x = 6. x = 7. x = a b 8. x =
a+b ba ab
a+b 1+n
9. x = 10. x = 1 11. x = b + 2 12. x =
2 n
a bc
13. x = m 14. x = 15. x = n 16. x = a
6a1
a+b
17. x = 1 18. x = 5(a + b + c) 19. x = 20. x = a+b+c
2 2 3
2
ab 1 1 aa
21. 22. x = 23. x = 24. x =
a1 a+b ab 2

6a ab 3a b
25. 2
26. x = 27. x = 28. x = 3a + 2b
a 2 2 2
2
-1 a+b a
29. 30. x = 31. x = 32. x = a
ab + bc + ac 2 a+b
2 2
29 5a b 6
33. x = b 34. x = 35. x = 36. x = b
11 7a b 3n + 4m
II. d
3. r = P
d F
1. v = 2. m = 4. t =
t a 2p v
s 2 Ec 2 Ec
5. R = j 6. w = P t 7. m = 2
8. v =
v m
Ep a 2 m2 F TP
9. m = 10. a1= 11. g = m
12. m =
gh m1 a
N m g cos a 2
13. a = Fr 14. m = a +Fg+sen
Fr
15. F = 16. R =
mv
mm a sen a Fc
dm
TR F v2
17. v = m g R 18. m = 19. dv = dt 20. g =
v
2
m 2 mx
dt
2 L 2d I
21. t = 1 22. r = 23. a = 24. R = M
F mw t
2

2 2
T L 2 E
25. m = 26. r = 27. x1 = Z R2 28. fo = f c
4k mk h
mc c2 f PV Q E1 Q1
29. f = 30. T = 31. T2 = + T1 32. Q2 =
h f 1 cos a + mc c2 nR C 1 + E1
Q 1 T2 g T2 x1 + x2 + ... + xn n xm
33. T1 = 34. l = 35. n = 36. s =
Q2 4p X z

S So R1 R2 R3
37. t = zs + x 38. t = 39. s = s f 40. R =
g R2 R3 + R1 R3 + R1 R2
fs

78 Ecuaciones e inecuaciones de primer grado

78-79.(2003) 78 20/11/02, 9:49 AM


CAPTULO 2

2.1.5 Ecuaciones con


valor absoluto
Definicin: x = x si x 0
x si x < 0

1. | x + 2 | = 5 Ejercicios
Solucin resueltos
i i) Si x + 2 0 | x + 2 | = x + 2
x+ 2 = 5 x = 3
ii) Si x + 2 < 0 | x + 2 | = (x + 2)
x 2 = 5 x = 7
2. | 7x + 3 | 2 = 1 x
Solucin
ii) Si 7x + 3 0 | 7x + 3 | = 7x + 3
7x + 3 2 = 1 x 8x = 0 x = 0
ii) Si 7x + 3 < 0 | 7x + 3 | = ( 7x + 3 )
7x 3 2 = 1 x 6x = 6 x = 1
3. 3 = 8x | 9x + 4 |
Solucin
i)) Si 9x + 4 0 entonces | 9x + 4 | = 9x + 4
As, 3 = 8x | 9x + 4 | 3 = 8x ( 9x + 4 )
x = 1 contradiccin
pues 9x + 4 0; por lo tanto x = 1 no es solucin
ii) Si 9x + 4 < 0 entonces | 9x + 4 | = ( 9x + 4 )
As, 3 = 8x | 9x + 4 | 3 = 8x + 9x + 4
7
x = 17 contradiccin
7
pues 9x + 4 < 0; por lo tanto x = no es solucin
17

La ecuacin no tiene solucin

Ejercicios
1. | 2x 4 | = 3 4. 3 = 7x | 10x + 4 |
2. 8 ( 5 + | 4x + 9 | ) = 32 5. 13 + | 6x + 7 | = 13
3. 15 = 15 (12 | 2x + 9 | ) 6. 4 = 8 | 6x + 7 |

Ecuaciones e inecuaciones de primer grado 79

78-79.(2003) 79 20/11/02, 9:50 AM


CAPITULO 2

Ecuaciones e inecuaciones de primer grado 79

80-81.(2003) 79 18/11/02, 1:39 PM


Ejercicios 14. 8x = 2 | 9x + 4 |
7. | 3x 3 | = 16 15. | 19x + 3 8x | 4 = 7x
8. 4 + | 6x + 7 | + 9x = 11x + 10 16. | 22x + 6 10x | 1 = 8x
9. 1 = 1 | 6x 6 | 17. 9 = 9 | 6x + 3 |
10. | 7x + 2 | 1 = 2x 18. 11 ( 6 + | 7x 8 | ) = 44
11. 3 ( 4 + | 4x + 1 | ) = 12 19. 2 + | 6x + 8 | = 14
12. 10 (2 + | 2x 5 | ) = 80 20. | 21x + 6 7x | 5 = 10x
13. 55 = 11 (4 | 3x + 8 | ) 21. 6 = 2x | 8x + 7 |

Soluciones
1 7 5
1. , 2. 2, 3. 2, 11
2 2 2
1 7 7 19 5
4. , 5. 6. ,
3 17 6 6 6
13 19
7. , 8. 1 , 13 9. 1
3 3 4 8
1 1 1
11 1
10. , 11. 12. ,
5 3 4 2 2
1 17 1 7
13. 3 , 3 14. No tiene solucin 15. ,
4 17
1 18 2
16. No tiene solucin 17. 18. ,
2 7 7
2 10 1 11 1 13
19. , 20. , 21. ,
3 3 4 24 6 10

2.2 Problemas

Hay un sinnmero de problemas que pueden resolverse planteando,


con sus datos, ecuaciones de primer grado.
Para resolver un problema recomendamos los siguientes pasos:
1. Leer atenta y comprensivamente el enunciado del problema.
2. Identificar la incgnita y los datos que se utilizarn en la so-
lucin.
3. Relacionar los datos con la incgnita planteando una ecuacin.
4. Resolver la ecuacin.
5. Analizar la solucin de la ecuacin cuidando que tenga rela-
cin con el enunciado del problema.
6. Dar la respuesta.

80 Ecuaciones e inecuaciones de primer grado

80-81.(2003) 80 18/11/02, 1:40 PM


CAPITULO 2
CAPTULO

1. Escribir en forma de expresin algebraica el siguiente enunciado. Ejercicios


a) La mitad de los asistentes a la reunin, ms uno. resueltos
Sea x el nmero de asistentes a la reunin.
x
Entonces la mitad ms uno es: +1
2
b) El doble de un nmero menos el 30% de l.
Sea n el nmero.
30
2n n
100
c) Si la suma de las edades de Juan y Mara es 63 aos, escribir
una expresin para la edad de cada uno:
Sea x la edad de Juan, entonces 63 x es la edad de Mara.
d) La suma de dos nmeros multiplicada por su diferencia.
Sean x e y los nmeros, entonces el producto de su suma
por su diferencia se expresa:
(x + y) (x y)
e) Sumar el cuadrado de un nmero con el doble de su cubo.
Si a es el nmero, la situacin descrita se escribe as:
a2 + 2a3
f) Sumar el cuadrado de un nmero con su duplo al cubo.
Si a es el nmero, la situacin ahora se escribe as:
a2 + (2a)3
g) Escribir tres nmeros enteros consecutivos.
Si n es un nmero entero, la situacin descrita se puede escribir:
n, n + 1, n + 2
n 1, n, n + 1
h) Un viajero debe recorrer m km. y ha recorrido n km. Cunto
le falta por recorrer?
Le falta (m n) km por recorrer.
i) Pedro recibe por un trabajo $ a y de mesada $ b. Con todo el
dinero alcanza a comprar (m + 2) revistas. Si todas las revistas
tienen el mismo valor, cunto le cost cada una?
Cada revista le cost:
a+b
$
m+2

2. En un gallinero hay 5 pavos ms que gallinas y 3 patos ms


que pavos. Si en total hay 49 aves, cuntas gallinas, pavos
y patos hay?
Sea x el nmero de gallinas, entonces:
x + 5 ser el nmero de pavos y
x + 5 + 3 ser el nmero de patos

Ecuaciones e inecuaciones de primer grado 81

80-81.(2003) 81 18/11/02, 1:40 PM


CAPTULO 2

Ecuaciones e inecuaciones de primer grado 81

82-83. 81 18/11/02, 1:42 PM


Ejercicios Como en total hay 49 aves podemos decir que:
resueltos x + x + 5 + x + 5 + 3 = 49
3x + 13 = 49
3x = 36
x = 12
Respuesta: Hay 12 gallinas, 17 pavos y 20 patos.
3. La suma de tres nmeros pares consecutivos es 102. Hallar los
tres nmeros.
La expresin 2n representa un nmero par, el siguiente es 2n + 2
y el que sigue es 2n + 4.
Como los tres nmeros suman 102 podemos decir que:
2n + 2n + 2 + 2n + 4 = 102
6n + 6 = 102
6n = 96
96
n=
6
n = 16
Si n = 16, entonces
2n = 32
2n + 2 = 34
2n + 4 = 36
Los nmeros pedidos son 32, 34 y 36.
4. El permetro de un rectngulo es de 40 m. Si el largo se aumenta
2m y el ancho se disminuye 2 m, su rea disminuye en 12m2.
Calcular sus dimensiones.
El permetro es 40 m, luego el semipermetro es 20 m.
Si llamamos x al largo, entonces 20 x representa el ancho.
rea del rectngulo x (2 x)
El nuevo rectngulo tiene:
largo x + 2, ancho 20 x 2 y rea (x + 2) (20 x 2)
Pero el rea disminuye en 12 m2, luego
x (20 x) (x + 2) (20 x 2) = 12
resolviendo la ecuacin
20x x2 (x + 2) (18 x) = 12
20x x2 18x + x2 36 + 2x = 12
4x = 48
x = 12
El largo es 12 m y el ancho es 8 m.
5. La edad de Pedro es el doble de la edad de Mara. Si en cinco
aos ms la suma de sus edades ser 43 aos, qu edad tienen
actualmente?

82 Ecuaciones e inecuaciones de primer grado

82-83. 82 18/11/02, 1:42 PM


CAPTULO 2

En este tipo de problemas es adecuado hacer un cuadro en el tiempo.


actual 5 aos ms
Edad de Mara x x+5
Edad de Pedro 2x 2x + 5
En cinco aos ms sus edades sumarn 43 aos:
(x + 5) + (2x + 5) = 43
resolviendo la ecuacin
3x + 10 = 43
3x = 33
x = 11
Actualmente Mara tiene 11 aos y Pedro tiene 22 aos.

6. Se tiene una herencia a repartir entre sus herederos y la


1
instruccin es que el mayor debe recibir de ella, el segundo
4
3 1
de ella y el tercero de lo que queda, entregando la otra
8 2
mitad a una institucin de beneficencia. Si esta institucin
recibe $ 815.625, hallar el monto de la herencia y cunto
recibe cada heredero.
Sea x el monto de la herencia, entonces:
1
El mayor recibe 4
x
El segundo recibe 3 x, con esto ya se han repartido
8
1 3 5 3
x + x = x, luego quedan x
4 8 8 8

El tercero recibe
La institucin de beneficencia recibe tambin x, lo que
equivale a $ 815. 625.
x = 815.625
3
x = 815. 625
16
x = $ 4.350.000
que es el monto de la herencia, as:
1
El mayor recibe x = $ 1.087.500
4
El segundo recibe 3 x = $ 1.631.250
8
El tercero recibe x = $ 815.625
7. Un estanque se llena con la llave A en 4 horas y con la
llave B en 8 horas. Cunto tardar en llenarse si se abren
simultneamente las dos llaves?
Si con la llave A tarda 4 horas en llenarse, en 1 hora se habr
1
llenado slo de estanque.
4

Ecuaciones e inecuaciones de primer grado 83

82-83. 83 18/11/02, 1:43 PM


CAPTULO 2

Ecuaciones e inecuaciones de primer grado 83

84-85.(2003) 83 18/11/02, 1:44 PM


Si con la llave B tarda 8 horas en llenarse, en 1 hora se habr
Ejercicios 1
llenado slo de estanque.
resueltos 8
Llamemos x al tiempo que tarda en llenarse con ambas llaves
1
abiertas, entonces en 1 hora se habr llenado slo de
x
estanque.

Podemos entonces plantear la ecuacin 1 + 1 = 1


4 8 x
Resolviendo:
2x + x = 8
3x = 8

x= 8
3
2
Demora 2 3 horas en llenarse el estanque con las dos llaves
abiertas, es decir, 2 horas 40 min.
8. Determinar a qu hora, entre las 5 y las 6 horas, los punteros de
un reloj forman un ngulo de 90.
o
360 o
En cada minuto, el puntero avanza =6
60

En cada minuto, el horario avanza


A las 5 horas el minutero est en las 12 y el horario en las 5 y el
ngulo que forman es de 150 (arco AB).
A
12

C
9 O. 3

5
6
DB
Se pide que el arco CD corresponda a un ngulo de 90.
Si llamamos O al centro del reloj, por cada minuto OA avanza
6 y OB avanza 0,5; por lo tanto, por cada minuto el ngulo
AOB se achica 5,5.
Sea x la cantidad de minutos que deben transcurrir para
que el arco AB corresponda a un ngulo de 90. Entonces
podemos decir:
150 5,5x = 90
60 = 5,5x
10
x = 10
11
Luego, el ngulo formado por los punteros del reloj es de 90
10
a las 5 horas 10 min.
11
El ngulo de 90 ocurrir nuevamente despus que el minutero
haya pasado sobre el horario.
Veamos primero a qu hora el ngulo se har 0.

84 Ecuaciones e inecuaciones de primer grado

84-85.(2003) 84 18/11/02, 1:44 PM


CAPTULO 2

A
12

150 5,5x = 0
9 O . 3
C 150 = 5,5x
3
5 x = 27
6 B 11
D
3
Esto es a las 5 horas 27 min.
11
A partir de este momento el ngulo se va agrandando a razn
90 4
de 5,5 por minuto; luego en = 16 minutos ms, el ngulo
5,5 11
entre los punteros volver a ser de 90, es decir, a las 5 horas
7
43 min.
11

Ejercicios
7. En el tringulo ABC los lados
1
AB = 3 BC y BC = 2 AC. Si su permetro
1. Un nmero multiplicado por 5 sumado es 84 m. Cunto mide cada lado?
con el mismo nmero multiplicado por
seis da 55. Cul es el nmero?
8. Si el lado de un cuadrado se duplica,
su permetro aumenta 40 m. Calcular la
2. Qu nmero se debe restar de p+2 medida del lado del cuadrado.
para obtener 5?

9. Las dimensiones de un rectngulo estn


3. El doble de un nmero aumentado en en la razn 3:5 y su permetro es 140 m.
12 es igual a su triple disminuido en 5. Calcular el largo y el ancho.
Cul es el nmero?

10. Una figura cerrada de seis lados


4. Tres nmeros impares consecutivos tiene cinco de sus lados en la razn
suman 81. Cules son los nmeros? 1:2 :3 :6:4 y sus ngulos son rectos,
excepto el formado por los lados que
estn en la razn 2: 3, que vale 270. Si
5. El doble de un nmero ms el triple de su permetro es 144 m, hallar el sexto
su sucesor, ms el doble del sucesor de trmino de la proporcin y la medida
ste es 147. Hallar el nmero. de cada uno de los lados.

6. La diferencia entre los cuadrados de dos 11. Si el lado de un cuadrado es aumentado


nmeros consecutivos es 103. Cules en 8 unidades, su permetro se triplica.
son los nmeros? Cunto mide el lado?

Ecuaciones e inecuaciones de primer grado 85

84-85.(2003) 85 18/11/02, 1:45 PM


CAPTULO 2

Ecuaciones e inecuaciones de primer grado 85

86-87. 85 18/11/02, 1:46 PM


Ejercicios
12. Con una cuerda de 70 cm de longitud 20. Repartir $ 952 entre tres personas
se pide formar un tringulo cuyos lados de modo que reciban cantidades
estn en la razn 3 : 4 : 7. Cunto proporcionales a 4, 6 y 18.
medir cada lado? Se podr construir?
Y si la longitud de la cuerda es 84 cm,
se podr construir? 21. Se compran 25 lpices, 32 cuadernos
y 24 gomas de borrar y se cancela por
ello $16.990. Si cada cuaderno cuesta
13. Un padre tiene 20 aos ms que su el triple de cada goma, ms $ 20 y
hijo. Dentro de 12 aos, el padre tendr cada lpiz cuesta el doble de cada
el doble de la edad del hijo. Cuntos goma, ms $ 8, cunto cuesta cada
aos tiene cada uno actualmente? material?

14. Las edades de un matrimonio suman 62


22. Hernn tiene el doble de dinero que
aos. Si se casaron hace 10 aos y
Gladys y el triple que Mara. Si Hernn
la edad de la novia era 3 de la edad
4 regalara $14 a Gladys y $35 a Mara,
del novio, qu edad tienen actual-
los tres quedaran con igual cantidad.
mente?
Cunto dinero tiene cada uno?
15. La edad de Pedro excede a la de su
amigo Santiago en 4 aos y a la de su
23. Entre Gastn y Javier juntan $ 2.340.
amigo Juan en 2 aos. Hace 6 aos la
Gastn reparte su capital entre cuatro
razn entre sus edades era 2:3:4. Qu
amigos y Javier lo hace entre tres. Los
edad tienen actualmente?
amigos de Javier reciben cada uno
$10 ms que los amigos de Gastn.
16. La edad de Mara es el triple de la de Hallar el dinero que tenan Gastn
Ester y excede en 5 aos a la edad y Javier.
de Isabel. Si las edades de Ester e
Isabel suman 23 aos, hallar la edad
de cada una. 24. Una persona puede pintar una muralla
en 5 horas, otra lo hace en 6 horas y una
tercera persona tarda 12 horas en pintar
17. Guido tiene la cuarta parte de la edad la misma muralla. Cunto tardaran si
de su padre Andrs y el triple de la la pintaran entre las tres?
edad de su hermano David. Qu
edad tiene cada uno, si sus edades
suman 48 aos? 25. Una llave puede llenar un estanque
en 6 horas. Otra puede hacerlo en
7 horas. Estando lleno, el desage
18. Hace 6 aos un padre tena el cudruplo puede vaciarlo en 10 horas. En cunto
de la edad de su hijo. En 10 aos ms tiempo se llenar el estanque si estando
tendr slo el doble. Hallar la edad vaco y con el desage abierto, se
actual de padre e hijo. abren las dos llaves?

19. Un padre tiene 52 aos y su hijo 16. 26. A qu hora entre las 11 y las 12 los
Hace cuntos aos el hijo tena la punteros del reloj formarn un ngulo
sptima parte de la edad del padre? de 90? A qu hora coincidirn?

86 Ecuaciones e inecuaciones de primer grado

86-87. 86 18/11/02, 1:46 PM


CAPTULO 2

27. A qu hora entre las 3 y las 4 33.Una prueba de matemtica contiene


los punteros de un reloj formaran temes de lgebra y geometra y est
un ngulo extendido? A qu hora estructurada con un 80% de preguntas
coincidirn? de seleccin mltiple. De ellas, el 50%
son de lgebra. De las preguntas con
respuestas abiertas, el 75% son de
28. El numerador de una fraccin excede lgebra. El nmero de preguntas de
en dos unidades al denominador. Si al geometra que no son de seleccin
numerador se le suma 3, la fraccin mltiple es 2.
4
queda equivalente a . Hallar la frac-
3
cin. a) Calcule el nmero de preguntas de
geometra que tiene la prueba.
29. La cifra de las decenas excede en b) Calcule el nmero total de preguntas
5 a la cifra de las unidades de un de la prueba.
nmero de dos cifras. Si el nmero se
divide por la suma de sus dgitos, da c) Qu fraccin de las preguntas de
8. Hallar el nmero. geometra son de seleccin mlti-
ple?
30. Descomponer el nmero 564 en tres d) Si las preguntas de seleccin mltiple
sumandos que sean inversamente valen 0,5 puntos; de las preguntas
proporcionales a 3, 4 y 5. abiertas, las de geometra valen 0,8
puntos y las de lgebra 0,4 puntos,
calcule el puntaje mximo de la
31. Considere el conjunto de alumnos
prueba.
y alumnas de su colegio desde 1
Bsico hasta 4 Medio.
e) En una escala de 1 a 7, cul sera la
Identifique: nota que proporcionalmente le corres-
a) El tamao de la poblacin pondera a un alumno que contesta
b) 3 variables cualitativas para estudio correctamente la mitad de las pregun-
tas de seleccin mltiple y todas las
c) 3 variables cuantitativas para estudio
preguntas abiertas?
d) Los rangos de cada una de las
variables identificadas en b y c. 34. Una persona pinta una piscina en r
horas y otra lo hace en s horas. Cunto
32. En las siguientes situaciones: se demoran en pintar la piscina ambas
personas trabajando juntas?
a) Identifique las variables relevantes
b) Elija parejas de variables que se 35. Para cortar el csped de una cancha de
relacionen en forma inversa y otras que ftbol una persona tarda 4 horas y otra
se relacionen en forma directa. lo hace en 6 horas. Calcule cunto se
c) Identifique el campo de variacin de demoraran si trabajaran juntas.
cada una de las variables elegidas.
El curso quiere organizar una despe- 36. Si un cuadrado de rea a aumenta su
dida a un compaero que se va de lado al doble, halle en qu proporcin
intercambio. aumenta su rea.

El curso decide realizar un regalo 37. Haga un anlisis de qu podra ocurrir


a la profesora de ciencias. con el permetro de un rectngulo si su
El orientador avisa que viene un rea se duplica. El permetro del nuevo
curso de visita para intercambiar rectngulo, depende de la forma como
experiencias en la clase de lgebra. se duplica su rea?

Ecuaciones e inecuaciones de primer grado 87

86-87. 87 18/11/02, 1:46 PM


CAPTULO 2

Ecuaciones e inecuaciones de primer grado 87

88-89.(2003) 87 18/11/02, 1:46 PM


Ejercicios
Observacin: Considere las siguientes Habr otras posibilidades?
alternativas:
38. En un cubo, la longitud de su arista se
a) x x triplica. Cmo vara su volumen?

y y y
39. Se tiene un tringulo de base b y altura
x
x h. Si se disminuye su base a la mitad y
b) se aumenta su altura al doble. Qu
y x ocurre con su rea?
y
y y
40. En un paralelogramo ABCD, P es un
c) x x punto cualquiera de la diagonal AC. Por
P se trazan EF AD y GH AB. Demos-
y trar que el rea del paralelogramo EBHP
es igual al rea del paralelogramo GPFD.
x

Soluciones

1. 5. 15. Pedro 14 aos; Juan 12 aos, y Santiago


10 aos.
2. p 3.
16. Ester 7 aos; Isabel 16 aos, y Mara
21 aos.
3. 17.
17. Andrs 36 aos; Guido 9 aos, y David
4. 25, 27 y 29. 3 aos.

5. 20. 18. Hijo 14 aos y padre 38 aos.

19. Hace 10 aos.


6. 51 y 52.
20. $136, $204 y $612.
7. AB = 42 m, BC = 14 m y AC = 28 m.
21. Lpiz $198, cuaderno $305, goma $95.
8. 10 m.
22. Hernn $126; Gladys $63, y Mara $42.
9. Largo = 43,75 m; ancho = 26,25 m.
23. Gastn $ 1.320, y Javier $1.020.

10. l1 = 6m; l2 = 12 m; l3 = 18 m; 24. 2 horas 13 minutos 20 segundos.


l4 = 36 m; l5 = 24 m; l6 = 48 m.
25. 4 horas 46 minutos 21,8 segundos.
11. 4 u. 10
26. A la 11 horas 10 11 minutos y 11
horas 43 7 minutos.
12. No se puede construir un tringulo cuyos 11
lados estn en la razn 3:4:7 porque Coincidirn a las 12 horas.
3 + 4 > 7. 27. A las 3 horas 49
1
minutos.
11
4
Coincidirn a las 3 horas 16 11
minutos.
13. 8 y 28 aos.
28. 17
15
14. 28 y 34 aos. 29. 72

88 Ecuaciones e inecuaciones de primer grado

88-89.(2003) 88 18/11/02, 1:47 PM


CAPTULO 2

30. 240, 180 y 144. 33. a)18 b) 40 c) 8 d) 20 e) 4.2


9
31. a) n = 487 (nmero de estudiantes 34. r s horas
matriculados. r+s
b) Lugar de nacimiento, color de 35. 2 horas 24 minutos
ojos, sexo. 36. Se cuadriplica
c) Talla, peso, N de hermanos.
d) {Ciudades del mundo}; {caf, negro, 38. Aumenta en un factor 27.
gris, celeste, verde}; {masculino, 39. Se mantiene; el rea de un tringulo
femenino} vara en forma directamente proporcio-
{x E R/ 50 x 210 cm} nal a su base y a su altura.
{x E R/ 10 kg x 130 kg};
{x E Z / 0 x 15} 40. Observe las reas de los tringulos ABC
y CDA; AEP y EGA; PHC y CFP.

Desigualdades e inecuaciones 2.3

El conjunto de los reales R es un conjunto ordenado; por lo tanto,


podemos comparar sus elementos mediante una relacin de orden
y podemos decir que:
Para a, b E R se tiene
a<bab<0
a>bab>0
a=bab=0
El signo < se lee menor que.
El signo > se lee mayor que.
Tambin se usa la combinacin de un signo de desigualdad
con el signo igual:
se lee menor o igual que,
se lee mayor o igual que.
Una relacin entre nmeros o letras que representan nmeros en que
se usan los signos <, >, o se llama desigualdad.
Son desigualdades numricas o literales las siguientes:
1. 3 + 2 < 7
2. x + 5 2
3. (x 1)2 > 0
4. 2 5 4
Una desigualdad puede ser verdadera o falsa.
En los ejemplos:
1. es verdadera
2. es verdadera para x = 2 y es falsa para x = 4
3. es verdadera para x 1 y falsa para x = 1
4. es falsa
Cuando una desigualdad presenta una incgnita se denomina
inecuacin y su valor de verdad (verdadero o falso) depender del valor
que le asignemos a la incgnita.

Ecuaciones e inecuaciones de primer grado 89

88-89.(2003) 89 18/11/02, 1:47 PM


Resolver una inecuacin es encontrar el intervalo de nmeros
reales para el cual la inecuacin se transforma en una desigualdad
verdadera.
Dependiendo del grado que presenta la incgnita, las inecuaciones
pueden ser de primer, segundo, tercer... grado y dependiendo del
nmero de incgnitas diferentes, pueden ser inecuaciones de una,
dos o ms variables.
Aqu demostraremos la veracidad de algunas desigualdades y
resolveremos las inecuaciones de primer grado.
Para desarrollar estos ejercicios hay que tener presentes las siguientes
propiedades de las desigualdades. Las enunciaremos usando el signo <,
pero son vlidas tambin para >, y .

Propiedades de las desigualdades.


Coinciden a, b, c, d E R
1. Al sumar una misma cantidad en ambos miembros de una desigualdad,
la desigualdad se mantiene
a<b a+c<b+c
2. Al restar una misma cantidad en ambos miembros de una desigualdad,
la desigualdad se mantiene
a<b ac<bc
3. Al multiplicar o dividir una desigualdad por una cantidad mayor que
cero, la desigualdad se mantiene
a < b ac < bc c>0
a b
a<b < c>0
c c
4. Al multiplicar o dividir una desigualdad por una cantidad menor que
cero, la desigualdad se invierte
a < b ac > bc c<0
a b
a<b > c<0
c c
5. Al invertir ambos miembros de una desigualdad, sta cambia
de signo
1 1
0<a<b >
a b
6. Al elevar a la misma potencia ambos miembros de una desigualdad,
sta se mantiene
0 < a < b an < bn n E R+
7. Al sumar miembro a miembro desigualdades del mismo signo, la
desigualdad se mantiene
a<b
c<d }a + c < b + d
90 Ecuaciones e inecuaciones de primer grado

90-91.(2003) 90 20/11/02, 4:03 PM


CAPTULO 2

8. La regla de los signos para el producto se expresa en trminos


de desigualdad as
a>0^b>0 ab>0
a<0^b<0 ab>0
a>0^b<0 ab<0
Observacin: Ntese que las propiedades 5 y 6 slo se cumplen si
los trminos que se comparan son ambos mayores que cero (positivos).

2.3.1 Desigualdades

1. Demostrar que si n > 0 entonces

n+
1
2
Ejercicios
n
Sabemos que el cuadrado de un nmero es siempre mayor que
resueltos
cero, luego
(n 1)2 0
1
n2 2n + 1 0 > 0 (prop. 3)
n
1
n2+ 0 +2 (prop. 1)
n
1
n+ 2
n
1 1 2
2. Si x > 0 e y > 0, demostrar que + >
x y x+y
Esto es verdadero si y slo si
1 1 2
+ >0
x y x+y
Entonces 1 1 2
+ =
x y x+y
x+y 2
=
xy x+y
2
x + y 2xy
=
xy x + y
2 2
x + 2xy + y 2xy
=
xy x + y
2 2
x +y
>0
xy x + y
ya que por dato del problema
x>0 e y>0 x+y >0
xy > 0
xy (x + y) > 0
adems x2 > 0
e y2 > 0
x + y2 > 0
2

Ecuaciones e inecuaciones de primer grado 91

90-91.(2003) 91 20/11/02, 4:04 PM


CAPITULO 2

Ecuaciones e inecuaciones de primer grado 91

92-93. 91 18/11/02, 1:49 PM


2 2
Ejercicios luego x +y
>0
xy x + y
resueltos
y 1 1 2
+ >0
x y x+y
1 1 2
luego + >
x y x+y

Ejercicios
1 1. Demuestre que si x > 0 e y > 0 entonces x > y x 2 > y2

1 2. Demuestre que, para a y b positivos, su media aritmtica es


a+b ab
mayor que su media geomtrica, es decir >
2
1 3. Demuestre que si 0 < x < 1 entonces x2 < x

1 4. Demuestre que x2 + 4 > 4x

1 5. Demuestre que x2 + y2 + 1 > 2 (x + y xy)

1 6. Demuestre que a2 + 2a + b2 2b > 2ab 1

1 7. Si b > 0 y d > 0 entonces a < a + c < c con a > c


b b+d d
b d
1 8. Demuestre que si a > 0 y b > 0, entonces a b + ab3 < a4 + b4
3

1
1 9. Demuestre que si a > 0 entonces a2 + 2
2
a
10. Determine para qu valores de a la desigualdad del ejercicio
anterior es igual a 2

Soluciones

1. Datos del ejercicio x > 0 , y > 0


Debe demostrar hacia ambos lados la implicacin:
) Dato : x > y . Por dem : x2 > y2

x>yxy>0 / (x + y) > 0 (por dato)

(x y) (x + y) > 0
x2 y2 > 0
x2 > y2

92 Ecuaciones e inecuaciones de primer grado

92-93. 92 18/11/02, 1:50 PM


CAPITULO 2
CAPTULO

) Dato: x2 > y2 Por dem.: x > y


x2 > y 2 x2 y2 > 0
(x + y) (x y) > 0 pero x + y > 0 (por dato)
xy>0
x>y
2
2. a b >0
a 2 ab + b > 0
a + b > 2 ab
a+b
> ab
2
3. 0 < x < 1 x > 0 y x 1 < 0
x (x 1) < 0
x2 x < 0
x2 < x
4. (x 2)2 > 0
x2 4x + 4 > 0
x2 + 4 > 4x
5. x2 + y2 + 1 2 (x + y x y) =
x2 + y2 + 1 2x 2y + 2xy =
(x + y)2 2 (x + y) + 1=
(x + y 1)2 > 0
x + y + 1 2(x + y x y) > 0
2 2
x2 + y2 + 1 > 2 (x + y x y)
6. a2 + 2a + b2 2b 2ab + 1 =
(a b)2 + 2 (a b) + 1 =
(a b + 1)2 > 0
a + 2a + b 2b 2ab + 1 > 0
2 2
y a2 + 2a + b2 2b > 2ab 1

7. Por hiptesis bd > 0

a a+c ab + ad ab bc
entonces = 2
b b+d b + bd
ad bc
= 2
<0
b + bd
2
1
9. a a 0

8. a3 b + a b3 a4 b4 = a3 (b a) + b3 (a b) a2 2 +
1
0
2
= a3 (b a) b3 (b a) a
= (b a) (a3 b3) a2 +
1
2
2
= (b a) (a b) (a2 + ab + b2) a

= (b a)2 (a2 + ab + b2) < 0


a3 b + ab3 a4 b4 < 0 10. para a = 1 y a = 1
a3b + ab3 < a4 + b4

Ecuaciones e inecuaciones de primer grado 93

92-93. 93 18/11/02, 1:51 PM


CAPTULO 2

Ecuaciones e inecuaciones de primer grado 93

94-95. 93 18/11/02, 1:55 PM


2.3.2 Inecuaciones

Ejercicios 1. Si a < b. Escribir como intervalo y grficamente los siguientes


resueltos conjuntos.
a) {x E R / x a}
b) {x E R / x > a}
c) {x E R / x b}
d) {x E R / x < b}
e) {x E R / a < x < b}
f) {x E R / a x b}
g) {x E R / a < x b}
h) {x E R / a x < b}

a) [a, + ) a

b) (a, + ) a

c) ( , b] b

d) ( , b) b

e) (a, b) a b

f) [a, b] a b

g) (a, b] a b

h) [a, b) a b

2. Resolver la inecuacin.
2x 5 < x + 2
aplicando las propiedades
2x x < 2 + 5
x < 7 o ( , 7)
Lo que grficamente es
7
En efecto, cualquier valor de x menor que 7 hace que la
desigualdad sea verdadera, por ejemplo, si x = 3.
2 3 5 < 3 + 5
1<8 lo que es verdadero

94 Ecuaciones e inecuaciones de primer grado

94-95. 94 18/11/02, 1:55 PM


CAPTULO 2

3. Resolver la inecuacin
x 2x
4 1
3 5
Aplicando las propiedades

x 2x
4 1 / 15
3 5
60 5x 6x 15
1
11x 75 /
11
75
x
11

Ntese que al multiplicar por un nmero negativo, la desigualdad


se invierte.
La solucin se puede entregar como intervalo real ,
grficamente
(75
11
o ]
75

11

75
Verifiquemos para x = 1 <
11
1 2
4 1
3 5

13 7

3 5 lo que es verdadero.

4. Resolver la inecuacin

(x 1)2 x2 +3

Resolviendo parntesis y aplicando las propiedades

x2 2x + 1 x2 +3

2x + 1 +3 /2

4x + 2 x + 6

3x 4 /
4
x
3
4
[
como intervalo , +
3 ) y grficamente

4

3

Ecuaciones e inecuaciones de primer grado 95

94-95. 95 18/11/02, 1:56 PM


Ejercicios
Escriba los siguientes conjuntos Resuelva las siguientes inecuacio-
I. como intervalos: III. nes y escriba su solucin como
intervalo:
1. {x E R / 2 < x 3}
1. x2 4x 1 (x 5)2
2. {x E R / 7 x < 5}

3. {y E R / y > 4} 2. (x + 1) (x 5) (x 2) (x + 5)

4. {x E R / x < 2 V x > 4} 3. 4x (x2 1) < (5 x) (1 + x)

5. {x E R / x 3 V x 2} 4. (3x 1) (x + 2) < x + 3 x2
6. {x E R / x < 1 A x 3}
5. (x + 2) (x 2) > (5 + x) (x + 1)
7. {x E R / x 2 A x 2}
6. (2x 1) (x + 4) (x 1) (2x + 3) 0

7. (3x 2) (x + 5) + (1 x) (4 + 3x) < 0


II. Resuelva las siguientes inecua-
ciones y grafique su respuesta: 8. (x + 2) (x 3) < (x + 4) (x 5)
1. 5x + 2 < 2x 1 9. (2x 1) x (x + 3) 2x
2. 3 4x 3 + 2x
10. (3x + 1) x + (5 x) 3x > 0
3. 2x 1 > 3

4. 5 3x 12 IV.
5. 2 x 1 > 0 1. Encuentre los nmeros enteros positivos
2
6. 4x 2 x + 1 + 1 < 0 tales que su quinta parte ms 3 sea
3 mayor que la mitad de su triple.
4 2x 5 3x
7.
3 4
8. 2 3x 6x + 1 0 2. Encuentre los nmeros naturales cuya
2 3 tercera parte sea mayor que su mitad,
9. 1 x 2 x 1 4x + 2 ms 1.
+ <
2 3 6

Soluciones
I
1. (2, 3] 2. [ 7, 5) 3. ( 4, + )
4. ( , 2) K (4, + ) 5. R 6. [ 3, 1) 7.
II
1. x < 1 2. x 1
1 1
7
3. x > 2 4. x
3
2 7
3

96 Ecuaciones e inecuaciones de primer grado

96-97. 96 20/11/02, 4:57 PM


CAPITULO 2
CAPTULO

1 1
5. x > 6. x <
2 1
2
5
_1
7. x 1 5
1
1
4 9. x >
3
8. x
1
21 4 3
21
III

1. [ 13
3
, +) 2. [ , + )
5
7
3. R (
4. , 1
2 ) (
5. , 3
2 )
6. [ 1
, + ) 7. ( , ) 1
8. 9. [ 0, + ) 10. ( 0, + )
6 2
IV
1. 1 y 2 2. No hay.

2.3.3 Inecuaciones simultneas


Se llaman inecuaciones simultneas aquellas que se satisfacen
simultneamente. Se considera como solucin de ellas aquel intervalo
para el cual se satisfacen todas.

1. Resolver simultneamente las siguientes inecuaciones. Ejercicios


x1> 2 resueltos
x 2 < 2x + 3

Resolviendo la primera
x1> 2
x> 3
Resolviendo la segunda
x 2 < 2x + 3
x< 5
x> 5
Comparando ambas soluciones en la recta numrica vemos que
ambas se satisfacen slo para x > 3

5 3
Luego la solucin es x > 3 o (3, + )
2. Resolver las inecuaciones simultneas.
3x 1 > x + 2
x x
4
+3
2

Ecuaciones e inecuaciones de primer grado 97

96-97. 97 20/11/02, 4:58 PM


Resolviendo la primera
Ejercicios
3x 1 > x + 2
resueltos 2x > 3
3
x > 2

Resolviendo la segunda
x x
+3
2 4
2x x + 12
x 12
Comparando ambas soluciones en la recta numrica, observamos
que slo se satisfacen ambas inecuaciones en la interseccin de
3
ellas, es decir, para < x 12.
2

3 12
2
Luego la solucin es
( ]
3
2
, 12

3. Resolver las inecuaciones simultneas


5x 1 > 0
3x + 3 < x + 1

Resolviendo la primera
5x 1 > 0
5x > 1
x>1
5
Resolviendo la segunda
3x + 3 < x + 1
2x < 2
x <1
Comparando ambas soluciones en la recta numrica vemos
que no hay ninguna solucin comn, luego no existe x que
las satisfaga a ambas.

1 1
5
Luego la solucin es vaca ()

4. Resolver simultneamente las siguientes inecuaciones


x + 3 < 2x 1
x + 3 < 2x + 1
2 4 3
5x 1 > x + 2
4

98 Ecuaciones e inecuaciones de primer grado

98-99. 98 20/11/02, 9:54 AM


CAPTULO 2

En este caso debemos resolver las tres inecuaciones y comparar


sus resultados.
Resolviendo la primera
x + 3 < 2x 1
x<4 / 1
x>4
Resolviendo la segunda
x 3 1
+ <2x+ / 12
2 4 3
6x + 9 < 24 x + 4
18 x < 5
5
x>
18
Resolviendo la tercera
x
5x 1 > 4 +2 / 4
20x 4 > x + 8
19 x > 12
12
x>
19
Comparando las tres soluciones vemos que la solucin para todas
simultneamente es x > 4

5 12 4
18 19
Luego la solucin es (4, + ).

Ejercicios
Resuelva las inecuaciones simultneas siguientes:
5. 6x 2x 1
1. 2x 1 > x + 3 2x x
x
4x 5 < x + 2 < +3
4 3
x 5x
2. + 1 0 9x
2 3 6. x x2
4x 3 x + 2 5 3
3x 2x
+ x+1
3. 2x + 1 x < x + 3 2 3
3x 2x
<0
5 4 1 2x 4x
7. < +1
2 3 3
2x
4. 2x + 1 x + 3 1
3 x 2x + 4
3x + 1 2 4 5

Ecuaciones e inecuaciones de primer grado 99

98-99. 99 20/11/02, 9:56 AM


Ejercicios
12. 2x 3 > x
8. 2 13 + 3 1
4
x 1
3
x
4
x+2 x
0 1 2x > 5
5
4x 1 < 3
9. (x 1) (2x + 3) (x 5) (2 + 2x)
(2 x) (3 x) (8 x) (1 x) 13. (x 1) x x2 + 2
1 + x2 2x + x2 1
10. (x 1) (2x + 3) (x 5) (2 + 2x)
3x 5 < 2x + 4
(2 x) (3 x) (8 x) (1 x)

11. 2x + 1 < 3 14. 5 2x + 1 > 0


x + 2 > 2x 1 2x 3 + x < 0
1 3x < 0 2x + 3 1 < 0

Soluciones
6 5
4. 3, 13
4 7
1. ,
3 3
2. , 3. ( , 0)
13 3

5. [ _ 41 , + ) 6. 30
7 7
,
6
(
1 5
7. 4 , 22 ] 5 4
8. 2 , 7

9. 10. 7 , 1 11. 1
,1 12.
9 2 3

13. [ 2, 1] 14. ( , 1)

2.3.4 Inecuaciones
con valor absoluto
El valor absoluto de un nmero real x representa la distancia
a que ste se encuentra del origen y se denota por | x |

|x|= { x si x 0
x si x < 0

Propiedades de la funcin valor absoluto

1. | x | 0 IxER
2. | x | = 0 x = 0
3. | x |2 = x2 I x E R
4. x2 = | x | I x E R
5. | x | x | x | I x E R
Teorema 1: Sean x, a E R, a > 0, entonces:
ii) | x | a a x a
ii) | x | a x a V x a

100 Ecuaciones e inecuaciones de primer grado

100-101. 100 20/11/02, 9:58 AM


CAPITULO 2
CAPTULO

1. Demostrar el teorema 1 que dice: Sean x, a E R, a > 0, entonces


Ejercicios
ii) | x | a a x a
resueltos
ii) | x | a x a V x a
Demostracin:
i)) si x 0 | x | = x a 1
si x < 0 | x | = x a x a 2
de donde 1 y 2 a x a
ii) si x 0 | x | = x a 1
si x < 0 | x | = x a x a 2
de donde 1 y 2 x a V x a

2. Resolver la inecuacin
| 5x 3 | < 2
Por teorema 1 tenemos
2 < 5x 3 < 2 / sumamos 3
1 < 5x < 5 / dividimos por 5
1
< x<1
5
1
Solucin ,1
5 0 1 1
5
3. Resolver la inecuacin
| 5 + 2x | 3
Por teorema 1
5 + 2x 3V 5 + 2x 3
2x 2 V 2x 8
x1Vx 4 4 1
Solucin: ( , 4] K [ 1, + )

4. Resolver la inecuacin
|x+1|+|x|3
Como se trata de una suma de valores absolutos, no podemos aplicar
el teorema anterior.
Vamos a eliminar las barras de valor absoluto conociendo el signo de
la expresin que est entre barras. Analizaremos la recta numrica a
tramos, teniendo en cuenta en qu puntos (nmeros reales) la expresin
entre barras cambia de signo (o se hace cero).
x + 1 = 0 para x = 1
x = 0 para x = 0
Dividimos la recta numrica en tres tramos:

1 0

Ecuaciones e inecuaciones de primer grado 101

100-101. 101 20/11/02, 9:58 AM


Ejercicios i) Si x E ( , 1), entonces x + 1 < 0
x<0
resueltos
Al aplicar la definicin de valor absoluto:
| x + 1 | = (x + 1) |x|=x
entonces la inecuacin queda:
(x + 1) x 3
x 1 x 3
2x 4
x2
Luego del intervalo ( , 1) es solucin el intervalo [ 2, 1)

ii) Si x E [ 1, 0), entonces x + 1 0


x<0
| x + 1| = x + 1 y |x|=x
entonces la inecuacin queda:
x+1x3
1 3 verdadero, lo que indica que todo el intervalo
considerado es solucin: [ 1, 0)

iii) Si x E [ 0, + ) entonces x + 1 > 0


x0
luego | x + 1 | = x + 1 y |x|=x
donde la inecuacin queda:
x+1+x3
2x 2
x1
Luego del intervalo [ 0, + ) slo es solucin el intervalo [ 0, 1].
Uniendo las soluciones de i, ii, iii tenemos que la solucin de la
inecuacin es S = [ 2, 1) K [ 1, 0 ) K [ 0, 1 ] = [ 2, 1 ]

Ejercicios
Resuelva las siguientes inecuaciones: 6. | 1 4x | 2 0

1. | 2x + 5 | 3 7. 1 | 2x + 5 | + 3 0
3
2. | 5 + 3x | < 3 8. 1 x +25
4 4

3. | 6 4x | 2 9. | 3x + 1 | + 5 0

4. | 2x + 1 | > 5 10. | 1 2x | + 3 0

5. | 7x 4 | 3 11. 3 < | x + 2 3x |

102 Ecuaciones e inecuaciones de primer grado

102-103. 102 20/11/02, 10:00 AM


CAPTULO 2

12. 5 > | 2x 3 | 25. 11 2 | x 1 |

13. 2+x 2
3 26. | 19x | 0

14. x2 10x + 25 2 27. 5 5 | 9x |

15. | x + 3 | x 28. 36 > 4 ( 9 | 6x + 3 | )

16. | 2x 1| | x + 2 | < 3 29. | 19x | > 0

17. 1 | 3 x | + | 2x + 3 | 0 30. | 4x 3 | 12

18. | x | + | 3x 6 | > 5 31. 9 + | 3x 8 | < 1

19. | x 2 | + | x + 3 | < | 2x 2 | 32. 9 > 9 | 9x + 5 |

20. | 2x 1 | | 3x + 2 | 4 33. | 3x + 4 | > 0

21. | 3x + 2 | | 2x 1 | 4 34. 2 + | x 2 | 2

22. 3 > 11 | 9x 3 | 35. 9 9 | 6x 4 |

23. 3 (6 + | 3x + 4 | ) 63 36. 7 (2 + | x 7 | ) 119

24. 63 9 ( 7 | 9x 2 | ) 37. 2 > 3 | 5x + 6 |

Soluciones
13. [ 5, 1 ]
25. [ 14, 12]
1. [ 4, 1 ]
14. ( , 3 ] K [ 7, + )
26. R
2. 23, 17
12 12 15.
27. R
3. [ 1, 2 ]
4 1
16. , 6 28. R
4. ( , 3) K (2, + ) 3 2

17. 5, 1 29. R {0}


( 17] K [1, + )
5. , 3
30. [ 3 75 ; 2,25 ]
,

6. ( , ] K [ , + ) 18. , 1 K 2 3 , +
1 3
4 4
2 4
31. No tiene solucin
7. ( , ] K [ , + ) 19. , 1 1
9 1
5
2 2 2 32. R {0,5} o R 9
8 16 20. R 4
8. , 33. R
3 3 3
9. 21. [ 7, 1] 34. {2}
17 11
22. , 9 K 9 ; + 2
10. R 35.
3
11 19
1 5 23. [ 3,67; 6,33] = ,
11. ,
2
K
2
,+ 3 3 36. [ 8, 22 ]

12. ( 1, 4) 24. R 37. ( ; 2,2) K ( 0,2 ; + )

Ecuaciones e inecuaciones de primer grado 103

102-103. 103 20/11/02, 10:01 AM


Prueba de seleccin mltiple
1. En la expresin 3x 7 = 2, x vale: B. 1
3
A. 1 C. 2
B. 2 D. 1
3
C. 3 E. 1
D. 4
E. 5 7. En la ecuacin
0,2x 0,3x + 3,1 = x 3,5 x vale:
2. La solucin de la ecuacin
5y + 2 = 4y 5 es: A. 0,06
A. 7 B. 0,6
B. 5 C. 6
C. 0 D. 0,6
D. 5
E. 0,06
E. 7
2 x 3x x 1
8. La expresin + =
3. La expresin 3(x + 2) 2(x + 3) = x 3 2 4 2 4
es verdadera para x igual:
es verdadera para:
A. x=0 A. 9
11
B. x=1
9
C. x = 1 B.
11

D. ningn x 11
C.
9
E. todo x real
11
D.
4. En la ecuacin (x1)2+ 2 = x2 1, 9

x vale: 1
E.
9
A. 2
B. 1 9. El valor de x en la ecuacin
C. 0 4x 3 5x + 2 3 6x + 1 3x + 5 2
+ = + es:
4 9 2 3 12 9
D. 1
A. 2
E. 2 B. 1
C. 0
5. La solucin de la ecuacin
D. 1
(2z 1) (z + 2) = 1 + 2z2
E. 2
A. 2
B. 1 10. La solucin de la ecuacin
C. 0 1 2
2x 4x +
2 3 , es:
D. 1 =
3 4
E. 2 A. 2
B. 1
6. En la ecuacin x(1x2)+3 = 2 (x2 + 2)x,
C. 0
x vale:
D. 1
A. 1 E. 2
104 Ecuaciones e inecuaciones de primer grado

104-105. 104 20/11/02, 10:04 AM


CAPITULO 2
CAPTULO

11. La expresin 15. El valor de x que hace D. 2


13x 4x verdadera la expresin E. 2
5 x+2 = 2 x+1 2
3 3 3x + 5
x3= es:
es verdadera para: 3x + 1 19. La solucin de la ecua-
A. 2 cin
A. x = 0 1 3 4
B. 1 =
B. x = 1 x + 2 2 x 1 2 x2 + 3x 2
C. 0 es:
C. x = 1
D. 1 A. 1
D. ningn x
E. 2
E. todo x real B. 2
C. 3
16. La solucin de la ecuacin
12. En la ecuacin x3 3x + 1 D. 4
= 0 es:
2 3 3 2x + 5 6x 4 E. 5
z z+1 z+1 3z+z
= 7
2 3 6 A.
el valor de z es:
39 20. La solucin de la ecuacin
B. 7
39 1 2 1 + 2x
3 + =
A. C.
39 2 2 x 3 3x 6x 6
2 7
5 es:
B. D. 39
2
7
A. 1
C. 1 E. 5
3
B. 2
7
5
D. C. 3
2

E. 3 17. En la ecuacin D. 4

2
1 2 1 E. 5

x x x
13. En la ecuacin = , 21. En la expresin
1 2 2
+
1 1 x 3 x ax + bx = a + b, x vale:
= 0, x vale:
3x 4 x + 2 el valor de x es: A. a
A. 1
A. 4,0 B. b
B. 2
B. 4,5 C. a
C. 3
D. b
C. 5,0
D. 4 E. 1
E. 5 D. 4,5

E. 4,0 22. En la ecuacin


a b
14. La solucin de la ecuacin 2ax 3bx = ,
3 2
2 1 1 18. En la ecuacin x vale:
es: + 2 =
x x 2x 1
A. 1 1 A. 6
x+1 1 , x vale:
=
B. 2 1 3 B. a
3 1+
1
C. 0 1+ C. b
x
D. 1 A. 1 D. 1
6
2
B. 1 E. 1
E. a
3 C. 0
Ecuaciones e inecuaciones de primer grado 105

104-105. 105 20/11/02, 10:05 AM


Prueba de seleccin mltiple
23. La ecuacin 27. En la frmula
2
mx + x = m2 1 B. 4b
es verdadera para x = n a 1 + an c
Sn = ,n=
A. m + 1 2 4b
2
C.
B. m1 2Sn c
A. 2
a1 an b
C. 1m Sn
D.
B. 4c
D. m a1 an
E. 4b2c
E. m2 C.
2Sn
gt
2

a 1 + an 31. Si E = , entonces
2
24. En la expresin t2 es:
Sn
p2 x 2pq = (p q)2, D.
a1 + an A. 2 Eg
x vale: Eg
E. 2Sn B.
p+q 2
A. 2
ana1
p
5x
2
C. 2E
B. 1+
p 28. Si y = h 2
,
q v g
q
C. 1+ entonces x2 = D. E
p
2 2g
p
D. 1+ A.
1
(h y) v2
q 5 E. g
2
q 2E
E. 1+
p
B. 5 (h y) v2

25. El valor de x en la 32. En la expresin


C. 5 (y h) v2
ecuacin 5 = r2, r2 =
px + p2 = 1 + x es: D. 1
(y h) v2
5
A. 1+p A. 5
5
E. hy
B. 1p v
2
p
B.
C. p1 5
29. En la expresin C. 5
D. 1 p 2p
E. 1 y = mx + k, m = D. 5
p
26. En la frmula A. y E. 5
k rp
x
an = a1 + (n 1) d, d =
yk
B.
x 33. La frase el doble de un
an + a1
A. k nmero menos su cuarta
n+1 C. y
x parte se expresa:
an a1 D. y+k
B. n
n1 x A. n
k 4
an a1 E. y+
C. x B. 2n 4n
1n
a1 an C. 8n n
D. 30. En la ecuacin 2
n1
D. 2n
an a1
E. b2 4ac = 0 , a = 4
n+1 7n
E.
b
2 4
A.
4c

106 Ecuaciones e inecuaciones de primer grado

106-107. 106 20/11/02, 10:10 AM


CAPITULO 2
CAPTULO

2
34. El 20% de un nmero sumado con el 38. Un poste est enterrado 5 de su
2
doble de l se expresa: longitud, del resto est bajo
7
agua y sobresalen 3 m. Cul es la
A. n longitud del poste?
B. 2 n A. 6 m.

C. 2,2 n B. 7 m.

D. 22 n C. 9 m.

E. 120 n D. 9,5 m.

35. El 20% de x menos el 50% de y lo E. 10 m.


podemos expresar como: 39. Si un nio tiene el triple de la edad que
tena hace 6 aos, cuntos aos tiene
A. 2x 5y en la actualidad?

B. 2 x 5y A. 3
100
B. 6
C. 2 x + 5y
10 C. 9

D. 0,02x 0,05y D. 12

2 x 5y E. 18
E.
10 40. Un comerciante compr 25 juguetes. Si
36. Un nmero sumado con su quinta parte hubiera comprado 5 juguetes ms por
es 12. La ecuacin que representa esta el mismo valor, cada juguete le habra
situacin es: costado $ 10 menos. Cunto le cost
cada juguete?
A. x + 12 = x
5 A. $ 10
x
B. x + = 12
5 B. $ 30
x
C. 12 + = x
5 C. $ 50
x
D. x = 12
5 D. $ 60
x
E. x 12 = E. $ 80
5

37. La suma de dos nmeros pares con- 41. Son soluciones de la inecuacin
secutivos es 106. sta se representa 2x 3 5 los nmeros:
mediante la ecuacin:
I. 4 II. 5 III. 3
A. 2n + (2n + 1) = 106 A. Slo I
B. 4n + 1 = 106 B. Slo II

C. 4n + 2 = 106 C. Slo III

D. n + n + 1 = 106 D. Slo I y III

E. 2n + 1 = 106 E. I, II y III

Ecuaciones e inecuaciones de primer grado 107

106-107. 107 20/11/02, 10:10 AM


Prueba de seleccin mltiple
42. No son solucin de la inecuacin 47. Al resolver la inecuacin 2 3X < 1 4x
se obtiene que: 5 2
2x 1 x + 3
1
A. x >
I. 5 II. 3 III. 8 14
1
A. Slo I B. x
14
1
B. Slo II C. x <
14
C. Slo III D. x 1
14
D. Slo I y III 1
E. x =
14
E. I, II y III
48. La inecuacin 2 x x 1 3 x es
43. Si x distinto de cero, de las expresiones 3 2 4
equivalente a:
siguientes son verdaderas siempre:
A. 5x7
I. x2 > 0 II. x2 > x III. | x | > 0 B. 5x7
A. Slo I C. 7x5
D. 7x5
B. Slo II
E. 7x5
C. Slo III
D. I, II y II
E. Slo I y III 49. La inecuacin (x 1) (x + 3) (x 2)2
es equivalente a:
44. De las desigualdades siguientes son siempre
verdaderas: A. 67x
1 B. 6x 7
I. x 2+y 2 2xy II. x + >2
x C. 67x
III. x 2 + 4 4 x
A. Slo I D. 6 x 7
B. Slo II E. 6 7x
C. Slo I y III
50. La solucin de la inecuacin
D. Slo III
(2x 1)(2 x) + (1 + 2x) (x + 3) 13 es:
E. I, II y III
A. ( ,1]
B. ( , 1)
45. La solucin de la inecuacin 3 x 1 es:
C. (1, + )
A. [ 2, + )
D. [1, + )
B. [2, + )
E. ( 1, 1)
C. ( , 2]
D. ( , 2] 51. Cuntos nmeros naturales no cumplen
E. [ 2, 2] la condicin de que su tercera parte ms
8 sea menor que su quntuplo?
46. El intervalo solucin de la inecuacin
5x 3 A. 1
< 0 es: 3
2 D. 5
,+ B. 2
A. (3, 5)
E. 3 C. 3
,
3 ,+
B. 5 5
D. Ninguno
C. 3
, E. Todos
5

108 Ecuaciones e inecuaciones de primer grado

108-109. 108 20/11/02, 10:13 AM


CAPITULO 2
CAPTULO

52. Cuntos nmeros naturales cumplen la A. ( , 5)


condicin de que su dcima parte es B. ( , 5]
mayor o igual que su mitad disminuida
1
en 2? C. 5,
4
A.
B.
1
2
D. ( 1
4
,+
)
C. 3 E. [ 1
4
,+ )
D. 4 57. Son solucin simultnea de ambas inecua-
E. 5 ciones: 5xx8

53. La quinta parte de un nmero disminuido 2


en 3 es mayor que el doble de l. Esta
proposicin se escribe algebraicamente I. 1 II. 0 III. 3
como: A. Slo I
x3
A. >2x B. Slo II
5
x C. Slo III
B. 3>2x
5
x3 D. Slo I y II
C. <2x
5 E. I, II y III
x
D. 3<2x
5

E. x 10 < 2 x 58. La solucin simultnea de las siguientes


inecuaciones es:
54. El doble de un nmero natural se aumenta
(x 1) (x + 2) (x 3) ( x + 1)
en 3. El doble de esta expresin resulta
igual a 12. Cul es el nmero? 2x323x
x (x + 2) (1 + x) (x 3)
A. 1
B. 2 1 3
A. ,
3 4
C. 3 1
B. ,1
3
D. 4
3
C. ,1
E. No existe 4

D. 3 1
55. Los nmeros enteros tales que su cuarta ,
4 3
parte es menor que su mitad, disminuida
E. R
en 2, son los nmeros:
59. La solucin de |3 2 x | 5 es:
A. Menores que 8
A. [1, 4]
B. Menores que 8 B. [1, 4]
C. Mayores que 8 C. [ 1, 4]
D. [ 1, 4]
D. Mayores que 8
E. ( 1, 4)
E. No hay
60. La solucin de |2 x + 3 | 7 es:
56. La solucin de las inecuaciones A. ( , 5) K (2, + )
simultneas siguientes es: B. ( , 5] K [2, + )
x22x+3 C. [ 5, 2]
D. ( 5, 2)
3x115 x
E. ( 5, + )

Ecuaciones e inecuaciones de primer grado 109

108-109. 109 20/11/02, 10:14 AM


Soluciones

1. C 2. A 3. E 4. E 5. B
6. D 7. C 8. C 9. C 10. B
11. D 12. D 13. C 14. E 15. B
16. B 17. D 18. A 19. C 20. D
21. E 22. D 23. B 24. E 25. D
26. B 27. C 28. A 29. B 30. D
31. C 32. D 33. E 34. C 35. E
36. B 37. C 38. B 39. C 40. D
41. D 42. D 43. E 44. C 45. D
46. D 47. C 48. D 49. B 50. D
51. A 52. E 53. A 54. E 55. D
56. B 57. E 58. D 59. C 60. B

110 Ecuaciones e inecuaciones de primer grado

110. 110 20/11/02, 10:15 AM


CAPTULO 3
R elaciones
y funciones

Lgica 3.1

Una expresin del lenguaje a la cual puede aplicarse con


sentido uno y slo uno de los calificativos verdadera o falsa
se denomina proposicin.
Es decir, una proposicin es una expresin susceptible de ser
verdadera o falsa.
p: El ser humano es mortal
q: El perro tiene dos patas
Si una proposicin es verdadera, diremos que su valor de verdad es
V y si es falsa, diremos que su valor de verdad es F.
Se llama funcin proposicional o proposicin abierta a una
proposicin en que el sujeto est dado en forma de smbolo y
puede ser reemplazado por alguno de los elementos de un conjunto
fijado con anterioridad.
p (x): x es un nmero natural x E N
Cada vez que el smbolo o variable (x) sea reemplazado por un
elemento del conjunto (en este caso N) la funcin proposicional pasa
a ser proposicin y tiene su valor de verdad.
Si x = 2 2 es nmero natural es V
Si x = 0,5 0,5 es nmero natural es F
Al conjunto al que pertenece la variable se le llama dominio o
universo de la funcin proposicional.
Las funciones proposicionales pueden tener ms de una variable.
q (x, y) : x e y viajaron en el buque Esmeralda el ao 1992.
La negacin de una proposicin es aquella que modifica la
proposicin dndole sentido contrario.
p : 5 es mayor que 2

Relaciones y funciones 111

111. 111 08/11/2001, 15:12


La negacin de p se denota por ~ p
~ p : 5 no es mayor que 2 o
~ p : es falso que 5 es mayor que 2.
Axioma de la negacin: p y ~p tienen valores de verdad contrarios.

p ~p
V F
F V

Las proposiciones (proposiciones simples) que hemos definido dan


origen a nuevas proposiciones (proposiciones compuestas) si stas se
conectan a travs de los conectivos lgicos siguientes:
V:o (disyuncin)
A:y (conjuncin)
: si ................, entonces (condicional)
: si y slo si (bicondicional)
Ejemplo:
Si a b = 0 entonces a = 0 o b = 0
p:ab=0
q:a=0
r :b=0
Son proposiciones simples que dan origen a la proposicin
compuesta enunciada y que simblicamente se escribe:

p (q V r)
es decir (a b = 0) (a = 0 V b = 0)
El condicional p q se puede expresar de las siguientes
maneras:
p implica q
si p, entonces q
q es condicin necesaria para p
p es condicin suficiente para q.
El bicondicional pq se puede expresar de las siguientes
maneras :
p si y slo si q
q si y slo si p
p es condicin necesaria y suficiente para q
q es condicin necesaria y suficiente para p
p es equivalente a q
q es equivalente a p
El valor de verdad de una proposicin compuesta depende del
valor de verdad de las proposiciones simples que la forman segn las
siguientes tablas de verdad.

112 Relaciones y funciones

112-113. 112 08/11/2001, 15:15


CAPTULO 3

Disyuncin Conjuncin
p q pVq p q pAq
V V V V V V
V F V V F F
F V V F V F
F F F F F F

Condicional Bicondicional
p q pq p q pq
V V V V V V
V F F V F F
F V V F V F
F F V F F V

El conectivo disyuncin V se usa con una variante si se considera en


forma excluyente. Se denota Y y su tabla de verdad es:
Disyuncin excluyente
p q pYq
V V F
V F V
F V V
F F F

Se llama tautologa a una proposicin compuesta cuyo valor de


verdad es verdadero (V) para cualquier combinacin de los valores de
verdad de las proposiciones simples que la componen.
Se llama contradiccin a una proposicin compuesta cuyo valor
de verdad es falso (F) para cualquier combinacin de los valores de
verdad de las proposiciones simples que la componen.
Los smbolos y se llaman cuantificador universal y
cuantificador existencial, respectivamente, y se leen para todo
y existe.
El cuantificador existencial presenta una variante que es !, que
significa existe un nico.
Ejemplos:
Si p (x) es una proposicin abierta (o funcin proposicional)
y E es el conjunto en el cual se define (dominio o universo de
la funcin), entonces:
1. ( x E), p(x) se lee: para todo x en E, tal que p (x) o para
cada x de E, p (x).

2. ( x E), p(x) se lee: existe x en E tal que, p (x) o existe por lo


menos un x en E tal que, p (x).

3. (! x E), p(x) se lee: existe un nico x en E tal que, p (x) o


existe slo un x E tal que p (x).

Relaciones y funciones 113

112-113. 113 08/11/2001, 15:15


as podemos decir que:
1. ( x E E) p (x) [p (x1 ) p (x2 ) p (x3 ) .....]
2. ( x E) p (x) [p (x1 ) V p (x2 ) V p (x3 ) V.....]
3. (! x E) p (x) [p (x1 ) Y p (x2 ) Y p (x3 ) Y.....]
Si a una funcin proposicional se le agrega un cuantificador, sta
pasa a ser una proposicin, puesto que tiene un valor de verdad.
Ejemplo:
Sea p (x): x es un mltiplo de 2
E = {x N / x < 10} = {1, 2, 3, 4, 5, 6, 7, 8, 9}
( x E), p (x) es falsa, puesto que hay elementos de E que no
son pares.
( x E), p (x) es verdadera, puesto que hay al menos un elemento
de E que es par.
(! x E), p (x) es falsa, ya que existe ms de un elemento de
E que es par.
Leyes de Morgan para cuantificadores.
1. ~ ( x E), p (x) ( x E), ~ p (x)
2. ~ ( x E), p (x) ( x E), ~ p (x)
En palabras:
1. Es falso que para todo x en E se cumple p (x) es equivalente a
existe algn x en E tal que no se cumple p (x).
2. Es falso que existe x en E tal que se cumple p (x) es equivalente a
para todo x en E, no se cumple p (x).

Ejercicios 1. Determinar cules de las siguientes expresiones del lenguaje son


resueltos proposiciones y determinar su valor de verdad.
a) Qu hora es?
b) El rbol pertenece al reino vegetal
c) El queso es un subproducto de la leche
d) En Chile en invierno la temperatura ambiente pasa de 35
e) Voy a salir, vuelvo ms tarde
f) Cierra la puerta
Solucin:
a) No es proposicin, no podemos asignarle un valor de verdad
b) Es proposicin y es verdadera
c) Es proposicin y es verdadera
d) Es proposicin y es falsa
e) No es proposicin, no podemos asignarle un valor de verdad
f) No es proposicin

114 Relaciones y funciones

114-115. 114 08/11/2001, 15:53


CAPTULO 3

2. Determinar cules de las siguientes expresiones son proposiciones


y cules son funciones proposicionales.
a) Los nmeros mayores que 2 son negativos.
b) El nmero entero x es mayor que 10.
c) Los mltiplos de 3 son infinitos.
d) Los enteros x e y son factores de 12.
Solucin:
a) Proposicin; su valor de verdad es falso.
b) Funcin proposicional; segn el valor que tome x, su valor
de verdad ser verdadero o falso.
c) Proposicin; su valor de verdad es verdadero.
d) Funcin proposicional; segn los valores que tomen x e y
ser su valor de verdad.

3. Sean p : x y > 0 , q: x > 0 , r: y > 0


x, y R, x, y 0.
a) Explicar qu significa ~ q , ~ r , ~ p
b) Escribir en smbolos la siguiente proposicin: el producto
de dos nmeros reales es mayor que cero si y slo si ambos
son positivos o ambos son negativos.
c) Escribir en palabras la siguiente proposicin:
~p (~q r) V (q ~ r)
Solucin:
a) Como x, y R y x, y 0 entonces
~q:x<0 y ~r:y<0
~p: x y<0
b) p : x y > 0 : el producto de dos nmeros x e y reales es
mayor que cero.
q : x > 0 : x es mayor que cero
r : y > 0 : y es mayor que cero
p (q r) V (~q ~ r)
c) ~ p (~q r) V (q ~ r)
El producto de dos nmeros reales es negativo si y slo si el
primero es negativo y el segundo positivo o el primero es positivo
y el segundo negativo.

4. Demostrar que si p, q y r son proposiciones, entonces:


(p q) (q r) (p r)
Para hacer esta demostracin haremos la tabla de verdad de
la proposicin compuesta:

Relaciones y funciones 115

114-115. 115 08/11/2001, 15:53


Ejercicios 1 2 3 4

resueltos p q r p q q r 1 A 2 p r 3 4
V V V V V V V V
V V F V F F F V
V F V F V F V V
V F F F V F F V
F V V V V V V V
F V F V F F V V
F F V F V F V V
F F F F V F V V

Observamos que 3 4 es una manera de escribir ms


brevemente la proposicin.
(p q) (q r) (p r) es verdadera para cualquier
combinacin de valores de verdad de las proposiciones simples
que la componen.

5. Sean las siguientes proposiciones


p: x N, x + 2 > 0
q: x N, x 1 N
a) Determinar su valor de verdad
b) Escribir ~p y ~q
Solucin:
a) p es verdadera, ya que para todo nmero natural x, x + 2 > 0
q es verdadera, porque existe un nmero natural x tal que x 1 no es
natural. Ese nmero es 1, ya que 1 1 = 0 N
b) ~p : x N, x + 2 0
c) ~q : x N, x 1 N

6. Sea A = {x N / x < 5} y B = { y N / y < 4}


Sean las proposiciones:
p: ( x A) ( y B), x + y < 6
q: ( x A) ( y B), x y = 15
a) Determinar el valor de verdad de cada proposicin
b) Escribir ~p y ~q
Solucin: Vemos que A = {1, 2, 3, 4} y B = {1, 2, 3}
a) p es verdadero, ya que para todo nmero de A existe un
nmero de B tal que la suma es menor que 6.
q es falso, ya que no existe ningn nmero en A ni ningn
nmero en B tal que su producto sea 15.
b) ~p: ( x A) ( y B) x + y 6
~q: ( x A) ( y B) x y 15

116 Relaciones y funciones

116-117. 116 08/11/2001, 15:56


CAPTULO 3

Ejercicios
1. Determine si las siguientes expresiones son o no proposiciones.
Si lo son, determine su valor de verdad.
a) p: Rembrandt es un pintor famoso.
b) q: La velocidad se define como distancia recorrida en un tiempo dado.
c) r: El 25% de 400 es 200.
d) s: Las gallinas son mamferos
e) t: Vengan a tomar el t.
f) u: 13 es un nmero par.
g) v: Pintemos esa casa.
h) w: 25 es la dcima parte de 250.
2. Determine cules de las siguientes expresiones son proposiciones y cules son
funciones proposicionales.
a) El conjunto de los nmeros naturales es parte de los nmeros reales.
b) Un nmero racional se puede escribir de muchas maneras.
2
c) Los racionales y 5 son equivalentes.
3 8 3
d) x en Q es equivalente a
4
e) El nmero entero x se puede escribir como un racional.
f) es un ngulo agudo.
g) 60o es el suplemento de 120o.
h) Los enteros x e y son factores de 15.
3. Usando cuantificadores, transforme las funciones proposicionales del ejercicio
anterior en proposiciones verdaderas.
4. Sean p, q, r, proposiciones simples, T tautologa y C contradiccin. Demuestre que
las siguientes proposiciones son tautologas.
a) ~(p V q) ~p ~q
b) ~(p q) ~p V ~q } Leyes de Morgan

c) (p q) (~p V q)
d) p V T T
p T p
pVCp
pCC
} Leyes de Identidad

e) p V p p
ppp } Leyes de Idempotencia

f) [(p q) r] [p (q r)]
[(p V q) V r] [p V (q V r)] } Leyes de Asociatividad

g) p p V q ; p q p
h) ~(~p) p
i) pVqqVp
pqqp } Leyes de Conmutatividad

Relaciones y funciones 117

116-117. 117 08/11/2001, 15:56


Ejercicios
j) [p V (q r)] [(p V q) (p V r)]
[p (q V r)] [(p q) V (p r)] } Leyes de Distributividad
k) (p q) (~ q ~ p)
l) (p q) (~ p ~ q)

5. Sea A el conjunto de los seres humanos.


Sean las siguientes proposiciones:
a) p : (h1 A) (!h2 A) (h2 es padre de h1)
b) q : (h3 A) (h4 A) (h3 es hermano de h4)
c) r : (h A) (h es mortal)
d) s : (h A) (h es mortal)
Escriba cada proposicin en palabras y determine su valor de verdad.

6. Sea N el conjunto de los nmeros naturales.


Sean las siguientes proposiciones:
a) p : (x N ), (y N ) x + y > 10
b) q : (x N), (y N) x y N
c) r : (x N), (y N) x y = 3
d) s : (x N), (y N) x y = y
Escriba cada proposicin en palabras y determine su valor de verdad.

7. Escriba la negacin de las cuatro proposiciones del ejercicio anterior. Determine su


valor de verdad, y si es falsa, muestre un contraejemplo.

8. Sea p (x): x es solucin de la ecuacin x2 4 = 0


Sea E = { 2, 0, 2}
Determine el valor de verdad de las siguientes proposiciones:
a) p : x E, p (x)
b) q : x E, p (x)
c) r : !x E, p (x)
d) s : x E, ~ p (x)
e) t : x E, ~ p (x)

9. Sean las siguientes proposiciones:


p : (x {1, 3, 5}) (y {2, 4, 6}), (x es mayor que y)
q : (x {1, 3, 5}) (y {2, 4, 6}), (x es mayor que y)
a) Escriba p y q en lenguaje corriente.
b) Determine el valor de verdad de p y q.
c) Escriba simblicamente ~ p y ~ q.

10. Sean p (x) : x es mayor que 3 y menor que 2


q (x) : x es mayor que 1 y menor que 4
r (x) : x es mayor que 0 y menor que 1
s (x) : x es mayor que 3 y menor que 5

118 Relaciones y funciones

118-119. 118 08/11/2001, 15:57


CAPTULO 3

ParaPara
las siguientes proposiciones,
las siguientes determine
proposiciones, el valor
determine el de verdad
valor y d yun ejemplo de las
de verdad
verdaderas.
d un ejemplo de las verdaderas
a) (x N), p (x) q (x) e) (x N), ~ (r (x) s (x))
b) (x N), p (x) q (x) r (x) f) (x N), ~ (r (x) V s (x))
c) (x R), p (x) q (x) r (x) g) (x N), ~ p (x) r (x)
d) (x Z), p (x) V s (x) h) (x Z), ~ r (x)

Soluciones
1. a) proposicin verdadera b) proposicin verdadera
c) proposicin falsa d) proposicin falsa
e) no es proposicin f) proposicin falsa
g) no es proposicin h) proposicin verdadera
2. a) proposicin b) proposicin
c) proposicin d) funcin proposicional
e) funcin proposicional f) funcin proposicional
g) proposicin h) funcin proposicional
3
3. d) ( x Q) x =
4
e) (x Z) x se puede escribir como fraccin
f) ( < 90) es agudo
h) (x, y Z), x y = 15
4. a) p q pVq ~(p V q) ~p ~q ~p~q
V V V F F F F
V F V F F V F
F V V F V F F
F F F V V V V

Comparando ambas columnas se ve que ~(p V q) ~p ~ q

b) p q pq ~(p q) ~p ~q ~pV~q
V V V F F F F
V F F V F V V
F V F V V F V
F F F V V V V

Comparando ambas columnas se ve que ~(p q) ~p V ~ q

c) p q pq ~p ~pV q
V V V F V
V F F F F
F V V V V
F F V V V

Comparando ambas columnas se ve que (p q) (~p V q)

Relaciones y funciones 119

118-119. 119 08/11/2001, 15:57


Soluciones

d) p T p VT p T p T
V V V V V V
F V V F V F

p VT T pTp

p C p VC p C pC
V F V V F F
F F F F F F

pVCp pCC

e) p pVp p pp
V V V V
F F F F

pVpp ppp

f) p q r p q (p q) r qr p (q r)
V V V V V V V
V V F V F F F
V F V F F F F
V F F F F F F
F V V F F V F
F V F F F F F
F F V F F F F
F F F F F F F

Comparando ambas columnas observamos que (p q) r p (q r)

p q r p V q (p V q) V r qVr p V (q V r)
V V V V V V V
V V F V V V V
V F V V V V V
V F F V V F V
F V V V V V V
F V F V V V V
F F V F V V V
F F F F F F F

Comparando ambas columnas observamos que (p V q) V r p V (q V r)

120 Relaciones y funciones

120-121.(2003) 120 20/11/02, 10:45 AM


CAPITULO 3
CAPTULO

g) p q p V q p(pVq ) p q p q (pq)p
V V V V V V V V
V F V V V F F V
F V V V F V F V
F F F V F F F V

Observando la ltima columna en cada caso se ve que


p (p V q) y (p q) p son siempre verdaderas.

h) p ~p ~(~p) i) p q p vq q v p p q p q q p
V F V V V V V V V V V
F V F V F V V V F F F
F V V V F V F F
F F F F F F F F
p ~ (~ p)
(p v q) (q v p) (p q) (q p)

j) p q r q r p V q p Vr p V (q r) (pVq) (p Vr)
V V V V V V V V
V V F F V V V V
V F V F V V V V
V F F F V V V V
F V V V V V V V
F V F F V F F F
F F V F F V F F
F F F F F F F F

p V (q r) (p V q) (p V r)

p q r q Vr p q p r p (q V r) (pq)V(pr)
V V V V V V V V
V V F V V F V V
V F V V F V V V
V F F F F F F F
F V V V F F F F
F V F V F F F F
F F V V F F F F
F F F F F F F F

p (q V r) (p q) V (p r)

Relaciones y funciones 121

120-121.(2003) 121 20/11/02, 10:45 AM


Soluciones

k) l)
p q pq ~q ~p ~q~p p q pq ~p ~q ~p~q
V V V F F V V V V F F V
V F F V F F V F F F V F
F V V F V V F V F V F F
F F V V V V F F V V V V

(p q) (~q ~p) (p q) (~p ~q)

5. a) p: para todo hombre existe un nico padre. Verdadero


b) q: existe un hombre que es hermano de todos los hombres. Falso
c) r: todo hombre es mortal. Verdadero
d) s: existe un hombre mortal. Verdadero

6. a) p: para todo nmero natural existe algn nmero natural tal que su suma es
mayor que 10. Verdadero
b) q: para todo par de nmeros naturales, su producto es un nmero natural. Verdadero
c) r: existen dos nmeros naturales tal que su producto es 3. Verdadero
d) s: existe un nmero natural tal que su producto con cualquier nmero natural da este
cualquier nmero natural. Verdadero

7. a) ~p : ( x N) ( y N) x + y 10. Falso
Si y = 10 no hay ningn natural que sumado con 10 sea menor que 10
b) ~q : x N, y N, x y F N. Falso
No existe ningn par de nmeros naturales tales que su producto no sea
nmero natural
c) ~r : x N, y N, x y 3. Falso. 3 1 = 3
d) ~s : x N, y N, x y y. Falso. 1 5 = 5

8. a) p es falso, 0 no es solucin de x2 4 = 0
b) q es verdadero, 2 es solucin de x 2 4 = 0
c) r es falso, 2 y 2 son soluciones
d) s es falso, 2 es solucin de x 2 4 = 0
e) t es verdadero, 0 no es solucin de x2 4 = 0

9. a) p: para cada x en {1, 3, 5} existe un y en {2, 4, 6} tal que x es mayor que y


q: existe x en {1, 3, 5} tal que para cada y en {2, 4, 6} x es mayor que y
b) p es falso (1 no es mayor que ningn y)
q es falso (no hay ningn x que sea mayor que todos los y)
c) ~p : ( x {1, 3, 5}) ( y {2, 4, 6}) (x y)
~q : ( x {1, 3, 5}) ( y {2, 4, 6}) (x y)

10. a) Verdadero, ejemplo: el 1


b) Falso.
1
c) Verdadero, ejemplo
2
d) Verdadero, ejemplo 2
e) Verdadero, r (x) A s (x) es vaco

122 Relaciones y funciones

122-123.(2003) 122 20/11/02, 10:46 AM


CAPITULO 3
CAPTULO

f) Verdadero, ejemplo 2
g) Falso
h) Verdadero, todos los enteros son menores o iguales que 0 o
mayores o iguales que 1

Conjuntos 3.2

3.2.1 Conceptos bsicos


Una teora matemtica se fundamenta y se va construyendo a
partir de esos fundamentos, encadenando los nuevos conocimientos
o proposiciones que se basan en las anteriores. Es as como se
parte de trminos no definidos o conceptos fundamentales. Luego
hay proposiciones que relacionan estos conceptos fundamentales
y que son tan evidentes que se aceptan como verdaderas. Estas
proposiciones se denominan axiomas de la teora. Siguiendo
con la construccin, aparecen las proposiciones, cuya veracidad
debe ser probada o demostrada. Son los llamados teoremas, que
segn su importancia se pueden denominar: proposicin, lema,
corolario o teorema.
En la teora de conjuntos aceptamos como trminos no definidos
las ideas de conjunto, elemento y pertenencia; son tres palabras
que usamos, que entendemos, pero que no definimos.
Los conjuntos se denotan por letras maysculas, sus elementos
por letras minsculas y se escriben entre corchetes.
A = {a, e, i, o, u} B = {1, 2, 3, 4, 5, 6, 7, 8}
La relacin de pertenencia se simboliza por y la negacin
de ella es .
As aA Verdadero
i B Falso
3 A Verdadero
5 B Falso
Los conjuntos se definen por extensin (nombrando todos
sus elementos) o por comprensin (indicando la caracterstica
que poseen sus elementos y que no poseen los elementos que
no son del conjunto).
A = {0, 1, 2, 3, 4, 5, 6, 7, 8, 9} B = {x/x es dgito}
P = {2, 1, 0, 1, 2} Q = {xZ / 3 < x <3}
A y P estn definidos por extensin
B y Q estn definidos por comprensin
Conjunto vaco: es el conjunto que no contiene elementos. En
smbolo {} o .

Relaciones y funciones 123

122-123.(2003) 123 20/11/02, 10:46 AM


Conjunto universo: es el conjunto que contiene a todos los
elementos. Se puede definir de acuerdo con el contexto en que se est
trabajando. Se denota por U.
Subconjunto: dado un conjunto no vaco A, se llama subconjunto
de A a todo conjunto B tal que todo elemento de B est en A.
Se anota B C A.
En smbolos:
B C A (x B) x A
El conjunto vaco es subconjunto de todo conjunto.
Conjunto potencia: se llama conjunto potencia de A y se denota
P(A) al conjunto cuyos elementos son todos los subconjuntos
del conjunto A.
P(A) = {B/B C A}
Ejemplo: si A = {1, 2, 3}
P(A) = { {1}, {2}, {3}, {1,2}, {1,3}, {2,3}, {1,2,3}, }
Se llama cardinalidad de un conjunto al nmero de elementos
que el conjunto tiene.
# A representa la cardinalidad de A.
En el ejemplo anterior # A = 3 y # P(A) = 8
En general, la cardinalidad del conjunto potencia de un conjunto
dado es igual a 2 elevado a la cardinalidad del conjunto. Es decir:
Si # A = n entonces # P(A) = 2n
Igualdad de conjuntos: dos conjuntos A y B son iguales si y slo
si tienen los mismos elementos. En smbolos:
(A = B) (x A), x B (x B), x A
En otras palabras y de acuerdo con la definicin de subconjunto:
(A = B) (A C B) A (B C A)
Equivalencia de conjuntos: dos conjuntos son equivalentes si y
slo si tienen el mismo nmero de elementos (igual cardinalidad).
En smbolos:
(A ~ B) (# A = # B)

Ejercicios 1. Escribir por extensin los siguientes conjuntos:


resueltos a) B = {x Z / 5 x 5}
b) C = {x / x es dgito del nmero 4.552.361}
Solucin:
a) B = {5, 4, 3, 2, 1, 0, 1, 2, 3, 4, 5}
b) C = {4, 5, 2, 3, 6, 1}

124 Relaciones y funciones

124-125. 124 08/11/2001, 16:09


CAPTULO 3

2. Escribir por comprensin los siguientes conjuntos:


a) A = {2, 4, 6, 8}
b) B =
{ 12 , 13 , 14 , 15 , 16 , 17 }
Solucin:
a) A = {x N / x es par x < 10}

b) B = {
1 / n N 2 n 7}
n
3. Hay conjuntos que no se pueden escribir por extensin porque
contienen infinitos elementos. Ejemplo:
A = {x R / 2 x 5}
Este tipo de conjuntos se puede escribir como intervalo y graficar como
subconjunto de R en la recta numrica. Ver cap. 2 (2.3.2).
Graficar los siguientes conjuntos en la recta numrica y escribirlos
como intervalo.
a) A = {x R / x > 3}
b) B = {x R / x 5}
c) C = {x R / x < 8}
d) D = {x R / x 1}
e) E = {x R / 0 x 5}
f) F = {x R / 2 < x < 3}
g) G = {x R / 1 x < 2}
h) H = {x R / 3 < x 1}

Solucin:

A = (3, + )
3
B = [5, + )
-5
C = (, 8)
8
D = (, 1]
-1
E = [0, 5]
0 5

F = (2, 3)
2 3

G = [1, 2)
-1 2

H = (3, 1]
-3 1

4. Dados los conjuntos A = {dgitos de 125} y B = {vocales de la


frase hace fro}

Relaciones y funciones 125

124-125. 125 08/11/2001, 16:09


Ejercicios Determinar el valor de verdad de las siguientes proposiciones:
resueltos Solucin:
a) 2 A a) verdadero porque 2 pertenece a A
b) 5 B b) falso porque 5 no pertenece a B
c) i B c) falso porque i pertenece a B
d) u B d) falso porque u no pertenece a B
e) e B e) verdadero porque e pertenece a B
f) f B f) falso porque f no pertenece a B
g) 125 A g) verdadero porque 125 no pertenece a A
h) 1 A h) falso porque 1 pertenece a A
i) a A i) verdadero porque a no pertenece a A
5. Sea A = {2, 4, 6}. Encontrar todos los subconjuntos de A y formar
el conjunto potencia de A.
Solucin:
Son subconjuntos de A los siguientes conjuntos:
{2}, {4}, {6}, {2,4}, {2,6}, {4,6}, A, .
P(A) = {{2}, {4}, {6}, {2,4}, {2,6}, {4,6}, A, .}
6. Dados los siguientes conjuntos, determinar cules de ellos son
iguales y cules son equivalentes.
A = {a,e,i,o,u}
B = {x N / x 5}
C = {vocales de la frase hace mucho fro}
D = {x N / x + 1 6}
Solucin:
#A = 5 , #B = 5 , #C = 5 , #D = 5
por lo tanto, todos son equivalentes, pero slo son iguales
A=C y B=D
Nota: A = B A ~ B, pero el recproco no se cumple.

Ejercicios
1. Escriba por extensin los siguientes conjuntos:
a) A = {mltiplos de 5 menores que 40}
b) B = {divisores de 36}
c) C = {nmeros primos menores que 20}
n
d) D = {x / x = , n N, n 10}
n+1
e) E = {n2 + 1 / n N}
f) F = {2n / n N}
g) G = {2n 1 / n N}
h) H = {n(n + 1) / n N, n < 5}
i) I = {factores primos de 36}

126 Relaciones y funciones

126-127. 126 08/11/2001, 16:10


CAPTULO 3

2. Escriba por comprensin los siguientes c) BA


conjuntos: d) {} A
e) {x N / 1 x 1} B
a) A = {3, 6, 9, 12, 15}
f) ADB
b) B = 1, { 1 1 1 1
, , , ...
2 3 4 5
} g) BDA
c) C = {1, 3, 9, 27, 81...} h) {1, 2, 3} D A
d) D = {0, 3, 8, 15, 24, 35, 48...} i) A A
j) B B
{ 1 1 1
e) E = 1, , , , ...
2 4 8 16
1
}
6. Sea A = {x / x es divisor de 25}
f) F = {0, 1 2 3 4 5 6
, , , , , ... }
3 4 5 6 7 8 a) Encuentre todos los subconjuntos
g) G = {1, 2, 4, 8} de A.
h) H = {1, 2, 5, 10} b) Escriba el conjunto potencia de A.
i) I = {4, 8, 12, 16, 20, 24}
7. Sea M un conjunto tal que # M = 8
3. Grafique en la recta numrica cada Cuntos subconjuntos de M se pueden
uno de los siguientes conjuntos y formar?
escrbalos como intervalo:
8. Sea la expresin n|m que significa que n
a) A = {x R / x 3}
divide a m. Sean los conjuntos:
b) B = {y R / y 5}
M = {x N / x | 10}
c) C = {x R / x > 4 }
N = {x N / x | 5}
d) D = {x R / x < 2}
P = {x N / x | 2}
e) E = {y R / 1 < y < 7}
f) F = {x R / 3 x 5} a) Escriba M, N y P por extensin.
g) G = {x R / 5 < x 0} b) Determine cules son equivalentes.
h) H = {x R / 3 x < 3} c) Determine el nmero de subconjuntos
de cada uno.
4. Dados los siguientes conjuntos: 9. Sean los siguientes conjuntos:
A = {factores de 36} A = {x R / 2 x 2}
B = {factores de 24} B = {x R / x 1}
C = {x R / x 1}
Determine el valor de verdad de las
siguientes proposiciones: a) Grafique A, B y C en la misma recta
numrica.
a) 6A 6B b) Escriba A, B y C como intervalo.
b) 4 A 4B c) Escriba el conjunto de elementos
c) 12 A V 12 B comunes a los tres conjuntos dados.
d) 9 A 9B d) Escriba el conjunto de elementos
e) 36 A V 36 B comunes a A y B.
f) 8A8B e) Escriba el conjunto de elementos
g) 1 A 1B comunes a A y C.
h) 1 AV1B f) Escriba el conjunto de elementos
5. Dados los conjuntos siguientes: comunes a B y C.
A = {x N / x < 5} 10. Sean los siguientes conjuntos:
B = {x Z / 2 x 2}
M = {x R / 2 x 2}
Determine el valor de verdad de las N = {x R / x > 2}
siguientes proposiciones:
Encuentre el conjunto de elementos
a) A C B
que pertenecen a M y a N.
b) C A

Relaciones y funciones 127

126-127. 127 08/11/2001, 16:10


Soluciones
1. a) A = {5, 10, 15, 20, 25, 30, 35} b) B = {1, 2, 3, 4, 6, 9, 12, 18, 36}
c)
e)
C
E
= {1, 2, 3, 5, 7, 11, 13, 17, 19}
= {2, 5, 10, 17, 26, 37, 50, 65,.....}
d) D = { 1 2 3 4 5 6 7 8 9 10
, , , , , , , , ,
2 3 4 5 6 7 8 9 10 11 }
g) G = {1, 3, 5, 7, 9, 11.....} f) F = {2, 4, 6, 8, 10.....}
i) I = {1, 2, 3} h) H = {2, 6, 12, 20}

1
2. a) A = {mltiplos de 3 menores que 18} b) B = { / n E N}
n
c) C = {3n / n E Z, n 0}
d) D = {n2 1 / n E N}
{ }
n
1
e) E = n E Z, n 0
2
g) G = {factores de 8} f) F = { nn + 11 / n E N}
i) I = {mltiplos de 4 menores que 28} h) H = {factores de 10}

3. a) A = [3, + ) 3

b) B = ( , 5]
5

c) C = ( 4, + )
-4

d) D = ( , 2)
2

e) E = (1, 7)
-1 7

f) F = [3, 5]
3 5

g) G = ( 5, 0] -5 0

h) H = [ 3, 3)
-3 3

4. a) falso, porque 6 E B b) verdadero c) verdadero


d) verdadero e) verdadero f) falso, porque 8 E B
g) verdadero h) verdadero

5. a) falso, 3 E A A 3 F B b) verdadero c) falso, 0 E B A 0 F A


d) falso, F A e) verdadero f) verdadero
g) verdadero h) falso, {1, 2, 3}C A i) verdadero
j) verdadero

6. a) {1}, {5}, {25}, {1,5}, {1,25}, {5,25}, A,


b) P (A) = {{1}, {5}, {25}, {1,5}, {1,25}, {5,25}, A, }

7. 256

8. a) M = {1, 2, 5, 10} N = {1, 5} P = {1, 2}


b) N B P
c) # P(M) = 16 , # P(N) = 4 , # P(P) = 4

128 Relaciones y funciones

128-129.(2003) 128 20/11/02, 10:48 AM


CAPITULO 3
CAPTULO

9. a) B A C d) {x E R / 2 x 1}
-2 -1 1 2 e) {x E R / 1 x 2}
b) A = [ 2, 2] ; B = ( , 1] ; C = [ 1, + )
c) {x E R / 1 x 1} f) {x E R / 1 x 1}
10.

3.2.2 Operaciones entre conjuntos


Los conjuntos en general y sus operaciones suelen graficarse a
travs de una figura llamada diagrama de Venn-Euler. A continuacin
definiremos las operaciones ms usuales y las graficaremos segn el
diagrama de Venn-Euler. Consideraremos el conjunto Universal como
el rectngulo que contiene a todos los dems conjuntos.

Complemento: Dado un conjunto A, lla-


maremos complemento de A y lo denotaremos A
por A', Ac o A al conjunto que contiene A
a todos los elementos del universo que no
estn en A.
A' = {x E U / x F A}

Unin de conjuntos: Dados dos conjuntos A


y B, se llama unin de A y B y se denota por
A K B al conjunto que contiene los elementos A B
de A, los elementos de B y los elementos que
estn en ambos conjuntos.
A K B = {x / x E A V x E B}

Interseccin de conjuntos: Dados dos


conjuntos A y B, se llama interseccin de A y
B y se denota A J B al conjunto que contiene A B
a los elementos que estn simultneamente
en ambos conjuntos:
A J B = {x / x E A A x E B}

Dos conjuntos cuya interseccin es vaca


se denominan conjuntos disjuntos.

Diferencia de conjuntos: Dados dos conjuntos A B


A y B, se llama Diferencia de A y B, se denota
AB al conjunto que contiene a los elementos
de A que no estn en B.
A B = {x / x E A A x F B}
Otra forma de definir el complemento de A
es segn la diferencia.
A' = U A = {x / x E U A x F A}

Relaciones y funciones 129

128-129.(2003) 129 20/11/02, 10:49 AM


Diferencia simtrica de conjuntos:
Dados dos conjuntos A y B se llama di-
A B ferencia simtrica de A y B y se denota
A B al conjunto que contiene a los elementos
que estn en A y no estn en B ms los elementos
que estn en B y no estn en A.

A B = {x / (x E A A x F B) V (x E B A x F A)}
A B = (A B) K (B A)
Propiedades de la operaciones entre conjuntos.
Sean A, B, C conjuntos contenidos en el universo U.
1. (A')' = A
2. U' = ' = U
3. A A = A=A
4. A K = A AJ=
AKU=U AJU=A
AKA=A AJA=A Idempotencia
A K A' = U A J A' =
5. (A K B) K C = A K (B K C)
(A J B) J C = A J (B J C) } Leyes de la
Asociatividad
6. A K B = B K A
AJB=BJA } Leyes de la
Conmutatividad
7. A K ( B J C) = (A K B) J (A K C)
A J ( B K C) = (A J B) K (A J C) } Leyes de la
Distributividad
8. (A K B)' = A' J B'
(A J B)' = A' K B' } Leyes de Morgan

Propiedades de la relacin de inclusin entre conjuntos.


Sean A, B, C conjuntos en un universo U.
1. Si A C B entonces A K B = B A A J B = A
2. Si A C B entonces AB=
3. (I A E U) A C A prop. reflexiva
4. A C B A B C A A = B prop. antisimtrica
5. A C B A B C C A C C prop. transitiva
6. A C B A K C C B K C A
AJC C BJC
Nota: Las propiedades 3, 4 y 5 dan a la relacin de inclu-
sin la caracterstica de relacin de orden (ver punto 3.3).
Observacin: Si A y B son conjuntos disjuntos, entonces:
AJB= ; AB=A ; BA=B

130 Relaciones y funciones

130-131. 130 08/11/2001, 15:17


CAPITULO 3
CAPTULO

1. Sean A = {1, 2, 3} , B = {3, 4, 5} y C = {4, 5, 6} Ejercicios


Hallar: a) AJB resueltos
b) AKB
c) AKBKC
d) AJBJC
e) AB
f) BC

Solucin:
a) A J B = {x / x E A A x E B} = {3}
b) A K B = {x / x E A V x E B} = {1, 2, 3, 4, 5}
c) A K B K C = {x / x E A V x E B V x E C} = {1, 2, 3, 4, 5, 6}
d) A J B J C = {x / x E A A x E B A x E C} =
e) A B = {x / x E A A x F B} = {1, 2}
f) A B = {x / (x E A A x F B) V (x E B A x F A)} = {1, 2, 4, 5}

2. Sean A = {x E R / 2 x 5} , B = {x E R / x 4}
C = {x E R / x < 2}
Hallar a) AJB
b) BKC
c) A'
d) C'
e) (A J B)'
f) (A K B) B
Solucin:
Representemos los tres conjuntos en la recta numrica. U = R

C A B
-2 4 5
a) A J B = {x E R / 4 x 5}
b) B K C = {x E R / x < 2 V x 4}
c) A' = {x E R / x < 2 V x > 5}
d) C' = {x E R / x 2}
e) (A J B)' = {x E R / x < 4 V x > 5}
f) (A K B) B = {x E R / 2 x < 4}

3. Sean P = {1, 2, 3, 4}, Q = {3, 4, 5, 6} considrese el universo


U = {0, 1, 2, 3, 4, 5, 6, 7, 8, 9}
Verificar que: a) (P K Q)' = P' J Q'
b) (P J Q)' = P' K Q'
Solucin:
Tenemos que P' = {0, 5, 6, 7, 8, 9}
Q' = {0, 1, 2, 7, 8, 9}
a) P K Q = {1, 2, 3, 4, 5, 6} luego (P K Q)' = {0, 7, 8, 9}
P' J Q' = {0, 7, 8, 9} \ (P K Q)' = P' J Q'

Relaciones y funciones 131

130-131. 131 08/11/2001, 15:17


Ejercicios b) P J Q = {3,4} luego (P J Q)' = {0, 1, 2, 5, 6, 7, 8, 9}
resueltos P' K Q' = {0, 1, 2, 5, 6, 7, 8, 9} \ (P J Q)' = P' K Q'
4. En el diagrama de Venn-Euler adjunto verificar:
a) listar los elementos
A b) A J (B K C) = (A J B) K (A J C)
a B c) A J (B C) = (A J B) C
c f
Solucin:
d b
i g a) A = {a, b, c, d} B = {b, c, e, g, i, f} C = {b, e, d, h, i, j, k}
h e b) B K C = {b, c, e, d, f, g, h, i, j, k}, luego
j k
C A J (B K C) = {b, c, d}
por otro lado
A J B = {b, c} y A J C = {b, d} entonces
(A J B) K (A J C) = {b, c, d}
\ A J (B K C) = (A J B) K (A J C)
c) B C = {c, f, g}, luego A J (B C) = {c}
por otro lado
A J B = {b, c}, luego (A J B) C = {c}
\ A J (B C) = (A J B) C. Se hace notar que esta igualdad
no siempre es verdadera.
5. Mostrar en un diagrama de Venn-Euler que si A J B = ,
entonces A J B = A.
Solucin:
A B En el diagrama se observa claramente que si se intersecta A con
B, la resultante es A, ya que A C B
6. Demostrar que A J B = A B.
Solucin:
Para demostrar igualdades de conjuntos podemos hacerlo por
doble inclusin, es decir:
1 A J B C A B y 2 A B C A J B
1 Sea x E A J B x E A A x E B x E A A x F B x E (A B)
\ A J B C A B
2 Sea x E (A B) x E A A x F B x E A A x E B x E (A J B)
\ A B C A J B
Luego A J B = A B.
7. Probar que (P K Q) J Q = P si y slo si P J Q = .
Solucin:
Aplicando las propiedades:

132 Relaciones y funciones

132-133. 132 08/11/2001, 16:14


CAPTULO 3

(P K Q) J Q = (P J Q) K (Q J Q) = (P J Q) K = P J Q
Luego el problema se transforma en probar que P J Q = P si y
slo si P J Q = .
Como es una bicondicionalidad, demostraremos primero la implicacin
a la derecha y luego la implicacin a la izquierda.
1
Hip.: P J Q = P
Tesis: PJQ=
Dem.: P J Q = P P C Q P J Q =
2
Hip.: PJQ=
Tesis: P J Q = P
Dem.: P J Q = P C Q P J Q = P
\ P J Q = P P J Q =

8. Probar que A C B B C A
1
Hip.: ACB
Tesis: B C A
Hip.
Dem.: Sea x E B x F B x F A x E A
\ B C A
2
Hip.: B C A
Tesis: ACB
Hip.
Dem.: Sea x E A x F A x F B x E B
\ACB
Luego, de 1 y 2 A C B B C A

9. Demostrar que la diferencia simtrica es conmutativa, es decir:


A B = B A.
Solucin:
A B = (A B) K (B A) = (B A) K (A B) = B A.

10. Demostrar la asociatividad de la interseccin:


(A J B) J C = A J (B J C).
Solucin:
Demostraremos por doble inclusin:
1 Por dem.: (A J B) J C C A J (B J C).

Relaciones y funciones 133

132-133. 133 08/11/2001, 16:14


Ejercicios Sea x E (A J B) J C x E A J B x E C
(x E A x E B) x E C
resueltos
x E A (x E B x E C)
x E A x E (B J C)
x E A J (B J C)
(A J B) J C C A J (B J C)
2 Por dem.: A J (B J C) C (A J B) J C
Sea x E A J (B J C) x E A x E (B J C)
x E A (x E B x E C)
(x E A x E B) x E C
x E (A J B) x E C
x E (A J B) J C
A J (B J C) C (A J B) J C
Luego, de 1 y 2 tenemos A J (B J C) = (A J B) J C

Ejercicios
1. Sean A = { 1, 3, 5, 7} c) A J (B K C)
B = { 1, 3, 5, 7, 9} d) (A K B) J C
C = { 2, 4, 6, 8} e) (A K C) J (B K C)
Considere f) (A J C) K (B J C)
U = {0, 1, 2, 3, 4, 5, 6, 7, 8, 9}
Determine: 4. Sean A = { x R / 2 x 4}
a) A K B e) A B i) (A J B)' B = { x R / 3 x 6}
C = { x R / x 5}
b) A J C f) C B j) C' J A
Encuentre:
c) A K C g) A B k) (A C)'
a) A B d) B (A K C)
d) B J C h) A' l) (B K C)'
b) B C e) B (A J C)
2. Sean P = { x R / 1 < x 5} c) (A K B) C f) A C
Q = { x R / x 2 }
R = { x R / 4 x < 1} 5. Sean P = [6 , 0], Q = [3 , 4] y
R = [3, 5] subconjuntos de R.
Determine: Determine:
a) PJQ e) RKP i) QP a) (P K R)'
b) QJR f) RKQ j) RP b) (P J Q)'
c) RJP g) PJQJR k) RQ c) (P K Q K R)'
d) (P Q) J R'
d) PKQ h) PQ l) QR
e) (R Q) J P'
3. Sean A = {x R / 6 x 2} f) (Q P) (Q R)
B = {x R / 1 x 6} 6. Sean U = {n N / n < 10}
C = {x R / |x| 7}
A = {n U / n < 5}
Encuentre: B = {n U / 4 < n < 8}
a) (A J B) K C b) (A K C) J B C = {n U / 3 < n < 10}

134 Relaciones y funciones

134-135. 134 08/11/2001, 16:16


CAPTULO 3

a) Liste los elementos de A, B, C y U 9. Muestre en un diagrama de Venn-Euler y


b) Haga un diagrama de Venn-Euler d un ejemplo en que se vea que si
c) Verifique que (A J B) = A K B A C B entonces B C A.
d) Verifique que
10. Demuestre que A J B = B A
(A J B) J C = A J (B J C )
7. En una encuesta en la Regin Metropo- 11. Demuestre que si
litana se consulta a 1.000 personas y A J B = A J C A A K B = A K C,
entonces B = C
se obtiene que:
990 personas hablan castellano 12. Demuestre que si P C Q, entonces
626 personas hablan ingls PJRCQJR
134 personas hablan francs
620 personas hablan castellano e ingls 13. Demuestre que A B = B A
130 personas hablan castellano y francs 14. Demuestre que si A C B, entonces
100 personas hablan los tres idiomas A K (B A) = B
a) Haga un diagrama de Venn-Euler
que refleje la situacin. 15. Demuestre que P Q C P K Q
b) Cuntas personas hablan slo 16. Haga un diagrama de Venn-Euler para
castellano?, cuntas slo ingls? y los conjuntos no vacos A, B y C que
cuntas slo francs? cumplan las siguientes propiedades:
c) Cuntas hablan ingls y no hablan a) A C B, B C C
francs? b) A C B, C C B, A J C
d) Cuntas hablan ingls o francs? c) A J C = , B J C , A J B
8. En el siguiente diagrama de Venn- Euler d) A C B, C J B , C J A ,
CDB
verifique que:
a) (A J B) D = (B D) J A 17. Demuestre que A (A B) = A J B
b) (B K C)' = D (B K C)
c) (B K C) (A K D) = (A K D)' 18. Demuestre que
A J (B C) = (A J B) (A J C)
B C
3 5 19. Demuestre que
1 6 7
A 4 A J (B K C) = (A J B) K (A J C)
9
11 12
10 8
20. Demuestre que
2 D A K (B J C) = (A K B) J (A K C)

Soluciones

No se incluye la respuesta de las demostraciones.


1. a) A K B = {1, 3, 5, 7, 9} = B b) A J C = c) A K C = {1, 2, 3, 4, 5, 6, 7, 8}
d) B J C = e) A B = f) C B = {2, 4, 6, 8} = C g) A B = {9}
h) A' = {0, 2, 4, 6, 8, 9} i) (A J B)' = {0, 2, 4, 6, 8, 9} j) C' J A = {1, 3, 5, 7}
k) (A C)' = {0, 2, 4, 6, 8, 9} l) (B K C)' = {0}
2. a) P porque P C Q b) {x E R / 2 x < 1} c) (1 F R A 1 F P)
d) Q porque P C Q e) {x E R / 4 x 5 A x 1} f) {x E R / x - 4}

Relaciones y funciones 135

134-135. 135 08/11/2001, 16:16


Soluciones
g) h) porque P C Q i) {x E R / 2 x 1 V x > 5}
j) R porque R y P son disjuntos k) {x E R / 4 x < 2} l) {x E R / x 1}
3. a) C b) B c) A d) A K B e) C f) A K B
4. a) [2, 3) K (4, 6] b) [3, 5) c) [2, 5) d) (4, 5) e) [3, 6] = B
f) A K C, ya que A J C =
5. a) ( , 6) K (0, 3) K (5, + ) b) ( , 3) K (0, + )
c) ( , 6) K (5, + ) d) [ 6, 3) e) (4, 5] f) [ 3, 0] K [3, 4]
6. a) A = {1, 2, 3, 4} b) U
A 1 2
B = {5, 6, 7}
C = {4, 5, 6, 7, 8, 9} 3 4 75
6 B
U = {1, 2, 3, 4, 5, 6, 7, 8, 9} C 89
7. a) Los nmeros representan la C
cardinalidad de cada conjunto. J
520 6
340
b) Slo castellano 340, slo ingls 100
6 y slo francs 4. 30
c) 526 4
d) 660 F

8. a) ambos son el conjunto {1, 3} b) ambos son el conjunto {2}


c) ambos son el conjunto {5, 6, 7, 12}
9. Ej.: Sea U = {0, 1, 2, 3, 4, 5}
B'
B A = {1, 2, 3} B = {1, 2, 3, 4}
A
A' = {0, 4, 5} B' = {0, 5}
A'
aqu claramente A C B A B' C A'
16. a) b) B c) d) B
A C B
A B A
C A C
C

3.3 Relaciones

3.3.1 Conceptos bsicos


Sean A y B conjuntos no vacos. Se define la operacin producto
cartesiano de los conjuntos A y B que se denota A B al conjunto
de pares ordenados.
A B = {(a, b) / a E A A b E B}
Una relacin R de un conjunto A en un conjunto B es un
subconjunto del conjunto A B.

136 Relaciones y funciones

136-137. 136 08/11/2001, 16:19


CAPTULO 3

(R relacin de A en B) R C A B
Observacin 1: Una relacin es un conjunto de pares ordenados.
Observacin 2: Una relacin R de A en B se denota R : A Q B.
Observacin 3: (a, b) E R a R b
(a, b) F R a R b

Sea R : A Q B una relacin y (a, b) E R


1. a se denomina preimagen.
2. b se denomina imagen de a segn la relacin R. Se denota
b = R (a).
3. Im. (a) es el conjunto de todas las imgenes del elemento a.
Sea R : A Q B una relacin. Se denomina.
Dominio de la relacin al conjunto de todas las preimgenes.
Dom. R = {a E A / H b E B A (a, b) E R } C A
Rango o recorrido de la relacin, al conjunto de todas las
imgenes.
Rang. R = {b E B / H a E A A (a, b) E R} C B.
Una relacin se puede graficar usando un sistema cartesiano o
un diagrama sagital (diagrama de flechas).
Si A = {a, b, c, d, e} B = {a, b, c, d}
R : A Q B, R = {(a, a), (a, b), (b, b), (b, c), (d, c), (e, d)}

GRFICO CARTESIANO GRFICO SAGITAL


B A R B
d
a
c a
b b
b
c
a c
d
d
e
a b c d e A

Si la relacin se define en conjuntos numricos, sus grficos


pueden ser figuras geomtricas.
Cada relacin R : A Q B tiene una relacin inversa R 1 : B Q A
R1 = {(b, a) / (a, b) E R}

1. Sean A = {1, 2, 3, 4} y B = {1, 5, 3}. Escribir tres relaciones Ejercicios


de A en B. resueltos
Se tiene que A B tiene cardinalidad 12, luego hay
12
2 = 4.096 subconjuntos de A B y por lo tanto el mismo
nmero de relaciones que se pueden formar. Tenemos que:

Relaciones y funciones 137

136-137. 137 08/11/2001, 16:19


Ejercicios R : A Q B = {(a, b) / a E A A b E B}
resueltos R1 = {(1, 1), (1, 5), (1, 3)}
R2 = {(1, 1), (2, 5), (3, 3), (3, 5), (4, 3)}
R3 = {(4, 5), (3, 5), (2, 5), (1, 5)}
y as podramos formar 4.093 relaciones distintas a estas tres.

2. Sean A = {1, 3, 5, 7} y B = {2, 4, 6}. Sea R : A Q B una relacin definida


por R = {(x, y), / y = x + 1}. Escribir R por extensin.

R est formada por todos los pares (x, y) de A B tales que su segunda
coordenada y es igual a la primera coordenada x ms 1.

R = {(1, 2) (3, 4) (5, 6)}

3. Sea R : Z Q Z una relacin definida por

R (x) =
{ x1
3
x+1
si
si
si
x>0
x=0
x<0
Hallar R (2), R (0), R ( 3)
sta es una relacin definida a tramos, es decir, tiene una frmula para
las imgenes de los nmeros mayores que 0 y otra para los nmeros
menores que 0. La imagen de 0 est dada y es 3. As:
R (2) = 21 =1 (2 > 0)
R (0) = 3
R ( 3) = 3+1=2 ( 3 < 0)

4. Sea P una relacin de N Q N definida por


P = {(x, y) / y > x}
Hallar las imgenes de dos elementos cualesquiera del dominio.
sta es una relacin donde la imagen y debe cumplir con
ser mayor que la preimagen x, luego, cada x tiene infinitas
imgenes:
Im 1 = {2, 3, 4, 5, ...... }
Im 10 = {11, 12, 13, 14, 15, ...... }

5. Dada la relacin R = {(1, 5) (5, 1) (3, 2) (4, 1)}


Hallar: a) Dom. R b) Rang. R
a) Dom. R = {x / x es preimagen}
= {1, 5, 3, 4}
b) Rang. R = {y / y es imagen}
= {5, 1, 2}
6. Sea R : N Q N una relacin definida por
R = {(x, y) / x + 2 y 10 = 0}
Hallar: a) Dom. R. b) Rang. R.
Primero escribamos R por extensin. Cules son los pares ordenados
de nmeros naturales que cumplen con que la primera coordenada
x ms dos veces la segunda y menos diez es cero?

138 Relaciones y funciones

138-139. 138 08/11/2001, 16:21


CAPTULO 3

x = 10 2y
si y = 1 Q x = 8 Q (8, 1) E R
y = 2 Q x = 6 Q (6, 2) E R
y = 3 Q x = 4 Q (4, 3) E R
y = 4 Q x = 2 Q (2, 4) E R
y=5Qx=0 que no es natural
Luego R = {(8, 1) (6, 2) (4, 3) (2, 4)}
Si hubiramos despejado y en la frmula que define R tendramos:
10 x
y=
2
9
Si x = 1 Q y = 2
FN
x = 2 Q y = 4 Q (2, 4) E R
7
x=3Qy= 2
FN
x = 4 Q y = 3 Q (4, 3) E R
5
x=5Qy= FN
2
x = 6 Q y = 2 Q (6, 2) E R
3
x=7Qy= 2
FN
x = 8 Q y = 1 Q (8, 1) E R
1
x=9Qy= 2
FN
x = 10 Q y = 0 F N
y no hay ms pares en R
Luego R = {(8, 1) (6, 2) (4, 3) (2, 4)}
As Dom. R = {8, 6, 4, 2}
Rang. R = {1, 2, 3, 4}

7. Sea M : A Q B una relacin definida por


M = {(3, 2) (0, 1) (3, 1) (1, 1) (2, 4)}
Hallar a) A b) B c) graficar en un diagrama sagital
d) graficar en un diagrama cartesiano.
a) A = {3, 0, 1, 2}
b) B = {2, 1, 4}
c) M d)
A B
4
0 1 3
1 2
2 1
2
3 4
0 1 2 3

Relaciones y funciones 139

138-139. 139 08/11/2001, 16:21


Ejercicios 8. Dadas las siguientes relaciones definidas grficamente, escribirlas
resueltos sealando sus pares por extensin.
a) b)

R Y
A B
6
1 5
a
3 4
b
3
c 5 2
1

1 2 3 4 5 6 7 8 9 10 X
Solucin:
a) R : A Q B R = {(a, 1), (a, 3), (b, 3), (c, 5)}
b) S : X Q Y S = {(2, 1), (2, 3), (3, 3), (4, 4), (5, 2), (7, 4), (8, 6)}

Ejercicios
1. Sea A = {p, c, d, m} y B = {b, a} 5. En la siguiente figura
Se define T : AQ B tal que Abuelo - Abuela Abuelo - Abuela
T = {(u, v) / u E A A v E B} Anbal Carmen Felipe Tamara
Determine si los siguientes pares perte- scar Mara
necen o no a la relacin T.
Jos Ruth Luis Rosa Hugo
a) (p, b) d) (a, d)
b) (m, a) e) (c, a) Se definen las relaciones siguientes:
c) (b, c) f) (m, b) a) R1 : ser padre de
b) R2 : ser hijo de
2. Sea S = { 8, 7, 6, 5, 3, 2, 1, 4, 7, 8}
y T = { 8, 7, 6, 3, 2} c) R3 : ser hija de
d) R4 : ser esposo de
Se define la relacin R : S Q T
Escriba cada relacin por extensin.
tal que R = {(x, y) / y = x 1}
Escriba R por extensin. 6. Sea H : Z Q Z una relacin definida por

3. Sea A = {b, s, i, h} y B = {n}


Sea T : A Q B
una relacin definida por
T = {(u, v) / u E A A v E B}
H (x) =

Encuentre:
{ 5
1x
5x + 4
si x < 2
si x = 2
si x > 2

Escriba T por extensin.


H (10), H (6), H (0), H ( 2), H ( 10)
4. Sea P = { 5, 9, 4, 8} y
7. Sea H : Z Q Z definida por:
Q = {2, 2, 4, 1, 3, 5}
Se define S : P Q Q
tal que S = {(u, v) / u 2 = v}
Escriba S por extensin.
H (x) =
{ 7
1x
7x4
si x < 2
si x = 2
si x > 2

140 Relaciones y funciones

140-141. 140 08/11/2001, 16:22


CAPTULO 3

Determine: 17. Dada la relacin


H (- 14), H (- 6), H (1), H (0), H (3) F = {(0, 3), (- 3, - 6), (3, - 1), (0, 0), (1, 6)}
8. Sea H : Z Q Z una relacin definida por Determine:

{
4 si x < 3 a) Dom F b) Rang F
H (x) = 4-x si x = 3
4x + 5 si x > 3 18. Dada la relacin
Determine: F = {(3, 3), (- 8, - 8), (- 5, 2), (0, 6), (- 6, 8)}
H (14), H (- 5), H (1), H (3), H (0) Determine:
a) Dom F b) Rang F
9. Sea R : Z Q Z una relacin definida por
R = {(x, y) / y = 4x} Busque: 19. Sea F = {(- 6 , 3), (- 7, 8), (- 5, - 7), (0, - 12),
R (- 11), R (6), R (3), R (0), R (1) (- 5, - 8)} una relacin.
Determine:
10. Sea P : R Q R una relacin definida por a) Dom F b) Rang F

{
P = (x, y) / y = 6x 7
4 }
Determine: 20. Sea R : N Q N una relacin definida por
P (0), P (5), P (- 1), P (3), P (- 2) R = {(u, v) / 2u + 3v - 12 = 0}
Determine:
11. Sea R : Z Q Z una relacin definida por a) Dom R b) Rang R
R = {(x, y) / y = 2x} Halle:
R (- 7), R (3), R (14), R (0), R (5) 21. Sea R : N Q N una relacin definida por
R = {(s, t) / 3t + 3s = 12}
12. Sea G : N Q N una relacin definida Halle:
por G = {(x, y) / y < x} a) Dom R b) Rang R
Encuentre:
a) una imagen de 7 22. Sea R : N Q N una relacin definida por
b) una imagen de 20
c) una imagen de 1
{
R = (p, q) / q =
10 3p
2 }
Determine:
13. Sea G : N Q N una relacin definida a) Dom R b) Rang R
por G = {(x, y) / y < x}
23. Escriba las siguientes relaciones por
Encuentre:
extensin:
a) Im (4) b) Im (14) c) Im (10)

14. Sea S : Z Q Z una relacin definida por a) R


S = {(- 4, - 2), (- 2, - 4), (- 4, - 5), A B
(- 5, - 4), (- 4, 1), (1, - 4)} r 1
Determine: q 3
a) Im (- 4) b) Im (- 5) c) Im (- 2) p 5
u
0
15. Sea S : Z Q Z una relacin definida por
S = {(6, 8), (8, 6), (6, 3), (3, 6), (6, 2),
b) T
(2, 6)} E F
Encuentre: Im (6)
9
16. Sea S : Z Q Z una relacin definida por
3
S = {(3, 5), (5, 3), (3,8), (8,3), (3,7), (7,3)} 0
6
Encuentre: Im (3) 0

Relaciones y funciones 141

140-141. 141 08/11/2001, 16:22


Ejercicios a) R = {(x, y) / x2 + y2 = 25}
b) S = {(x, y) / x y = 5}
c) N c) T = {(x, y) / 2x y = 1}
5
4 26. Determine la relacin inversa R1 dada
3 la relacin R.
2
a) R = {(1, 2), (1, 3), (1,4), (2,1),
1
(2, 2), (2, 3)}
1 2 3 4 5 6 7 N b) R = {(a, b), (a, c), (a, a)}
c) R = {(1, 1), (2, 2), (3,3), (4,4),
(3, 5), (5, 3)}
d) N
5 27. Determine la relacin inversa R1 dada
4 la relacin R.
3 a) R = {(x, y) E N N / x = 2y}
2
b) R = {(x, y) E N N / x + 2y = 22}
1
c) R = {(x, y) E R R / y = 4x 2}
1 2 3 4 5 6 7 N d) R = {(x, y) E R R / x + y 4 = 0}
28. Dado el conjunto U = { 2, 2, 4}
24. Grafique las siguientes relaciones de N
Halle la relacin inversa R1 de cada
en N usando un grfico cartesiano.
relacin R en U y proceda a listarla.
a) R = {(x, y) / x + y = 9}
a) R = {(x, y) / x y}
{
b) S = (x, y) / y =
10 x
2
} b) R = {(x, y) / x es mltiplo de y}
c) T = {(x, y) / 5y + x = 26} c) R = {(x, y) / x es la mitad de y}
d) R = {(x, y) / x mayor o igual que y}
25. Grafique las siguientes relaciones reales
e) R = {(x, y) / y = x }
en un grfico cartesiano.

Soluciones
1. a) s b) s c) no d) no e) s f) s
2. R = {( 6, 7), ( 2, 3), ( 5, 6), ( 7, 8)}
3. T = {(b, n), (s, n), (i, n), (h, n)}
4. S = . No hay ningn par de P Q que satisfaga la relacin definida.
5. a) R1 = {(Anbal, scar), (Felipe, Mara), (scar, Jos), (scar, Ruth), (scar, Luis),
(scar, Rosa), (scar, Hugo)}
b) R2 = {(scar, Anbal), (scar, Carmen), (Jos, scar), (Jos, Mara), (Luis, scar),
(Luis, Mara), (Hugo, scar), (Hugo, Mara)}
c) R3 = {(Mara, Felipe), (Mara, Tamara), (Ruth, scar), (Ruth, Mara), (Rosa, scar),
(Rosa, Mara)}
d) R4 = {(Anbal, Carmen), (Felipe, Tamara), (scar, Mara)}
6. H (10) = 46 H (6) = 26 H (0) = 4 H ( 2) = 1 H ( 10) = 5
7. H ( 14) = 7 H ( 6) = 7 H (1) = 11 H (0) = 4 H (3) = 25
8. H (14) = 61 H ( 5) = 4 H (1) = 4 H (3) = 1 H (0) = 4
9. R ( 11) = 44 R (6) = 24 R (3) = 12 R (0) = 0 R (1) = 4

142 Relaciones y funciones

142-143. 142 08/11/2001, 16:24


CAPTULO 3

7 23 13 11 19
10. P (0) = 4
P (5) = 4
P ( 1) = 4
P (3) = 4
P ( 2) = 4

11. R ( 7) = 14 R (3) = 6 R (14) = 28 R (0) = 0 R (5) = 10


12. a) cualquier nmero natural menor que 7. Ejemplo el 5.
b) cualquier nmero natural menor que 20. Ejemplo el 3.
c) cualquier nmero natural menor que 1. No hay.
13. a) {3, 2, 1} b) {13, 12, 11, 10, 9, 8, 7, 6, 5, 4, 3, 2, 1} c) {9, 8, 7, 6, 5, 4, 3, 2, 1}
14. a) { 2, 5, 1} b) { 4} c) { 4}
15. {8, 3, 2}
16. {5, 8, 7}
17. a) { 3, 0, 1, 3} b) {3, 6, 1, 0, 6}
18. a) {3, 8, 5, 0, 6} b) {3, 8, 2, 6, 8}
19. a) { 6, 7, 5, 0} b) {3, 8, 7, 12, 8}
20. a) {3} b) {2}
21. a) {1, 2, 3} b) {1, 2, 3}
22. a) {2} b) {2}
23. a) {(r, 0), (q, 1), (p, 3), (u, 5)} b) {(9, 0), (3,0), (6, 0), (0, 0)}
c) {(1, 3), (2, 2), (3, 1), (3, 3)} d) {(1,1), (1, 2), (1, 3), (2, 3), (2, 4), (3, 4), (4, 4), (5, 4)}

24. a) b)
8 8
7 7
6 6
5 5
4 4
3 3
2 2
1 1

1 2 3 4 5 6 7 8 9 1 2 3 4 5 6 7 8 9

c)
5
4
3
2
1

1 2 3 4 5 6 7 8 9 10 11 12 13 14 15 16 17 18 19 20 21

25. a) y b) c) y
y
5
1
x
5 x 2

5 5 x 1

5
5

Relaciones y funciones 143

142-143. 143 08/11/2001, 16:24


Soluciones
26. a) R 1 = {(2, 1), (3, 1), (4, 1), (1, 2), (2, 2), (3, 2)}
b) R 1 = {(b, a), (c, a), (a, a)}
c) R 1 = R
27. a) R 1 = {(x, y) E N N / x es la mitad de y}
22 y
b) {
R 1 = (x, y) E N N / x = 2 }
R 1 = {(x, y) E R R / y = }
x+2
c) 4

d) R 1 = {(x, y) E R R / y = 4 x} = R
28. a) R 1 = {(x, y) / x y} = {( 2, 2), ( 2, 4), (2, 2), (2, 4), (4, 2), (4, 2)} = R
b) R 1 = {(x, y) / x es factor de y} = {(2, 2), (2, 2), (2, 4), ( 2, 4), ( 2, 2), (4, 4)}
c) R 1 = {(x, y) / x es el doble de y} = {(4, 2)}
d) R 1 = {(x, y) / x y} = {( 2, 2), ( 2, 2), (2, 2), (2, 4), (4, 4), ( 2, 4)}
e) R 1 = {(x, y) / y = x2} = {( 2, 4), ( 2, 4)}

3.3.2 Relacin de equivalencia y de orden


Una relacin definida de un conjunto A en s mismo se denomina
relacin en A.

(R relacin en A) R C A A
Observacin 1
A El grfico sagital de este tipo de relaciones se hace dibujando una
sola vez el conjunto y uniendo mediante flechas los elementos
a relacionados.
b
c Si R = {(a, b) (a, c) (a, a)} es una relacin definida en el conjunto
A = {a, b, c}, su grfico sagital es el adjunto.
Observacin 2
Una relacin definida en el conjunto R se denomina relacin real
y se grafica en el sistema cartesiano.

Propiedades de una relacin en A

Sea R una relacin en A.


(R es refleja) (I x E A, x R x)
(R es simtrica) (x R y y R x)
(R es antisimtrica) ((x R y A y R x) x = y)
V ((x y A x R y) y R x)
(R es transitiva) ((x R y A y R z) x R z)
Observacin 3
Una relacin R en A se llama relacin de equivalencia si y slo si
R es refleja, simtrica y transitiva.

144 Relaciones y funciones

144-145. 144 08/11/2001, 16:26


CAPTULO 3

Observacin 4.
Una relacin R en A se llama relacin de orden si y slo si R es
refleja, antisimtrica y transitiva.
Sea R una relacin de orden en un conjunto A. Sean x e y
elementos de A tal que (x, y) R (x R y). En este caso se dice que
x e y son comparables segn la relacin R.
Si en un conjunto A todos los pares de elementos son comparables
segn una relacin de orden R, el conjunto A se dice totalmente
ordenado y la relacin R se dice de orden total. Si al menos un
par de elementos de A son no comparables, entonces A se dice
parcialmente ordenado y R es una relacin de orden parcial.

1. Sea A = {1, 2, 3, 4, 5} y R la relacin. Ejercicios


R = {(1, 2) (1, 3) (3, 1) (2, 3) (2, 1) (1, 1) (2, 2) (3, 3) (4, 4) (5, 5) resueltos
(3, 4) (1, 4) (2, 4)}
Determinar si R es refleja, simtrica, antisimtrica y/o transitiva.
a) R es refleja porque I x E A; x R x
b) R no es simtrica porque 2 R 3 y 3 R 2
c) R no es antisimtrica porque 1 R 3 A 3 R 1 pero 3 1
d) R no es transitiva porque 3 R 1 A 1 R 2 pero 3 R 2
2. Sea A = {alumnos de un colegio}. Se define la relacin R en A tal
que: x R y x est en el mismo curso que y. Probar que R es
una relacin de equivalencia en A.
a) R es refleja porque x est en el mismo curso que x. Esto se
cumple para todos los alumnos del colegio.
b) R es simtrica porque si x est en el mismo curso que y, esto
implica que y est en el mismo curso que x.
c) R es transitiva porque si x est en el mismo curso que y, e y
est en el mismo curso que z, entonces tenemos que x est en
el mismo curso que z.
Observaciones:
R clasifica a todos los alumnos del colegio en diferentes cursos.
Cada curso es una clase de equivalencia.
Esta es una clasificacin porque:
a) Cada alumno est en un curso.
b) Ningn alumno est en dos cursos.
c) Todo curso tiene algn alumno.
3. Sea A = {1, 2, 3, 4, 5} y R la relacin en A definida por
R = {(a, b) / a b es mltiplo de 2}
a) Listar los elementos de R.
b) Demostrar que R es relacin de equivalencia.

Relaciones y funciones 145

144-145. 145 08/11/2001, 16:26


Ejercicios c) Se llama clase de a y se denota [a] al conjunto:
[a] = {x / (x, a) R}
resueltos scribir la clase de cada elemento de A.
d) Escribir la clasicacin que la relacin R determina en el
conjunto A.
Solucin:
a) R = {(1, 1) (2, 2) (3, 3) (4, 4) (5, 5) (1, 3) (3, 1) (1, 5) (5, 1) (2, 4)
(4, 2) (3, 5) (5, 3)}
b) R es reeja porque (a, a) R a A.
R es simtrica porque si ab es mltiplo de 2, b a tambin lo
es.
R es transitiva porque:
a b mltiplo de 2 a b = 2n
b c mltiplo de 2 b c = 2m
sumando: a c = 2 (n + m)
a c, tambin es mltiplo de 2. Luego, por ser reeja, sim-
trica y transitiva, R es relacin de equivalencia.
c) [1] = {x A / (x, 1) R} = {1, 3, 5}
[2] = {x A / (x, 2) R} = {2, 4}
[3] = {x A / (x, 3) R} = {3, 1, 5}
[4] = {x A / (x, 4) R} = {4, 2}
[5] = {x A / (x, 5) R} = {5, 1, 3}

d) Se observan dos clases:

1
2
2
4
5

[1] = [3] = [5] y [2] = [4]


Nota: Cualquier elemento es representativo de su clase.

Antes de resolver los siguientes ejercicios diremos que una relacin


de orden se llama as porque ordena los elementos del conjunto
donde se dene:
Representaremos el orden segn la relacin R en un diagrama as:
y cuando (x, y) R
x c
b
cuando (a, b), (b, c), (a, c) R
a
4. Sea A = {a, b, c, d, e} y R la relacin en A denida por:
R = {(d, b), (d, a), (b, e), (b, c), (a, c), (d, c), (d, e), (a, a), (b, b),
(c, c), (d, d), (e, e)}
a) Determinar si R es una relacin de orden.
b) Si R es relacin de orden hacer el diagrama de orden.

146 Relaciones y funciones

146-147..indd 146 01-02-2006 16:20:10


CAPTULO 3

Solucin:
a) R es reeja porque x R x x E A.
R es antisimtrica porque si x y (x, y) R entonces ( y, x) FR.
R es transitiva porque (x, y) R (y, z) R (x, z) R, luego, R
es relacin de orden.
b) El diagrama de orden segn R es:
c e Se observa que d es el menor elemento
a b segn R en el conjunto A y no hay un
d elemento que sea mayor que todos.

ste es un orden parcial.

5. Sea A = { x IN / x es par x < 13}


R = {(x, y) / x es divisor de y}
a) Determinar si R es relacin de orden.
b) Hacer el diagrama de orden.
Solucin:
Escribamos A y R por extensin.
A = {2, 4, 6, 8, 10, 12}
R = {(2, 4), (2, 6), (2, 8), (2, 10), (2, 12), (4, 8), (4, 12), (6, 12),
(2, 2), (4, 4), (6, 6), (8, 8), (10, 10), (12, 12)}
a) R es relacin de orden porque es reeja, antisimtrica y transi-
tiva.
b) 12 8 10 no es menor que 12 porque 10 no
divide a 12. 4 no es menor que 6. 2 es
10 6 4 menor que todos (los divide a todos), no
hay un elemento mayor que todos (que
sea dividido por todos).
2
ste es un orden parcial.

6. Sea A = {(8, 6, 2, 7} y S la relacin denida por


S = {(8, 6) (6, 2) (7, 7) (8, 2) (6, 6) (2, 2) (8, 8) (8, 7) (6, 7) (2, 7)}
a) Determinar si S es relacin de orden.
b) Hacer el diagrama de orden en A segn S.
Solucin:
a) S es relacin de orden porque es reeja, antisimtrica y transitiva.
b) El diagrama de orden en A segn la relacin S es:

7 8 es el menor de todos, 7 es el mayor de


2 todos y todos los pares de elementos de A
6 son comparables.
8 ste es un orden total.

Relaciones y funciones 147

146-147..indd 147 01-02-2006 16:20:13


Ejercicios

1. Sea A = {1, 2, 3, 4} se definen las relaciones siguientes. Determine si stas son


reflejas, simtricas, antisimtricas y/o transitivas.
R1 = {(1, 1) (2, 2) (3, 3) (4, 4) (2, 3) (3, 2)}
R2 = {(1, 2) (3, 2) (4, 2) (2, 2) (2, 1) (2, 3) (2, 4)}
R3 = {(1, 1) (2, 2) (3, 3) (4, 4)}
R4 = {(1, 2) (1, 3) (1, 4) (2, 3) (2, 4) (3, 4)}
R5 = {(2, 1) (2, 2) (3, 1) (3, 2) (3, 3) (4, 1) (4, 2) (4, 3) (4, 4) (1,1)}

2. Dadas las siguientes relaciones en Z, determine si stas son reflejas, simtricas,


antisimtricas y/o transitivas.
a) x R y x es mayor o igual a y
b) x R y x y0
c) x R y x = y
d) x R y x y = 1
e) x R y x y es un nmero par
f) x R y x + y = 10
g) x R y | x y | > 2
h) x R y | x y | < 3

3. Sea A = {0, 1, 2, 3, 4, 5, 6, 7, 8, 9}. Se defi nen en A las siguientes relaciones.


Determine las propiedades que tienen dichas relaciones.
a) x S y x y = 2
b) x S y x divide a y
x
c) x S y el cociente tiene resto 2
y
d) x S y y x < 3
e) x S y x y = 36
f) x S y x + y = 9

4. Determine las propiedades que posee la relacin de inclusin entre conjuntos.


ARB A C B A y B conjuntos no vacos.

5. Sea R una relacin simtrica que cumpla la condicin


aRb bRc cRa. Demuestre que R es transitiva.

6. En el conjunto de pases A = {Japn, Sudn, India, China} se define la relacin


S = {(Japn, Japn), (Sudn, Sudn), (India, India), (China, China), (Sudn,
Japn), (Japn, Sudn)}
a) Es S relacin de equivalencia?
b) Si lo es, haga la clasificacin correspondiente.

7. Dados los siguientes conjuntos A y la relacin R definida en ellos


a) Determine si R es relacin de equivalencia
b) Si lo es, haga la clasificacin correspondiente.

148 Relaciones y funciones

148-149. 148 08/11/2001, 16:28


CAPTULO 3

ii i) A= {9, 2, 6, 5} R = {(2, 9), (9, 9), (6, 6), (9, 2), (2, 2), (5, 5)}
ii) A = {3, 9, 1, 8} R = {(9, 3), (3, 3), (1, 1), (3, 9), (9, 9), (8, 8)}
iii) A = {1, 6, 9, 2} R = {(6, 1), (1, 1), (9, 9), (1, 6), (6, 6), (2, 2), (9, 2), (2, 9)}
iv) A = {4, 1, 9, 7} R = {(1, 4), (4, 4), (9, 9), (4, 1), (1, 1), (7, 7)}
v) A = {Alhu, Rapel, Purn, Maule}
R = {(Alhu, Alhu), (Rapel, Rapel), (Purn, Purn), (Alhu, Rapel), (Maule,
Maule), (Rapel, Alhu), (Alhu, Maule), (Maule, Rapel), (Rapel, Maule),
(Maule, Alhu)}
vi) A = {Coat, Koala, Hiena, Lirn}
R = (Coat, Koala), (Koala, Hiena), (Lirn, Lirn), (Coat, Hiena), (Koala, Koala),
(Coat, Coat), (Hiena, Hiena), (Lirn, Hiena)}
vii) A = {Coat, Lemur, Koala, Okapi}
R = {(Lemur, Okapi), (Okapi, Okapi), (Koala, Koala), (Lemur, Lemur), (Okapi,
Lemur), (Coat, Coat)}

8. Dados los siguientes conjuntos A y las relaciones S definidas en A


a) Determine si S es b) Haga el diagrama c) Determine si el or-
relacin de orden. de orden en A segn S. den es total o parcial.
i) A = {Sirio, Aries, Urano, Virgo}
S = {(Aries, Sirio), (Sirio, Sirio), (Urano, Urano), (Aries, Aries), (Sirio,
Aries), (Virgo, Virgo)}
ii) A = {5, 2, 4, 7}
S = {(5, 4), (5, 7), (2, 4), (2, 7), (4, 7), (5, 5), (2, 2), (4, 4), (7, 7)}
iii) A = {2, 5, 4, 1}
S = {(2, 4), (2, 1), (5, 4), (5, 1), (4, 1), (4, 2), (2, 2), (5, 5), (4, 4), (1, 1)}
iv) A = {7, 6, 8, 3}
S = {(7, 6), (6, 8), (3, 3), (7, 8), (6, 6), (7, 7), (8, 8), (3, 8)}
v) A = {Talca, Ancud, Alhu, Achao}
S = {(Talca, Alhu), (Talca, Achao), (Talca, Talca), (Ancud, Alhu), (Ancud,
Achao), (Alhu, Achao), (Alhu, Alhu), (Ancud, Ancud), (Achao,
Achao)}
vi) A = {5, 1, 6, 7}
S = {(5, 5), (6, 6), (1, 1), (5, 6), (7, 7), (6, 5), (5, 7), (7, 5), (6, 7), (7, 6)}
vii) A = {plomo, verde, caoba, rojo}
S = {(plomo, verde), (verde, caoba), (rojo, rojo), (plomo, caoba), (verde, verde),
(plomo, plomo), (caoba, caoba), (rojo, caoba)}
viii) A = {5, 7, 4, 2}
S = {(5, 5), (5, 4), (4, 4), (7, 7), (7, 4), (2, 4), (2, 2)}
ix) A = {7, 9, 6, 4}
S = {(7, 6), (7, 4), (9, 6), (9, 4), (6, 4), (7, 7), (9, 9), (6, 6), (4, 4)}
x) A = {4, 8, 5, 2}
S = {(4, 4), (4, 5), (5, 5), (8, 8), (8, 5), (2, 5), (2, 2)}

Relaciones y funciones 149

148-149. 149 08/11/2001, 16:28


Ejercicios
xi) A = {Libra, Virgo, Orin, Urano}
S = {(Libra, Orin), (Libra, Urano), (Libra, Libra), (Virgo, Orin),
(Virgo, Urano), (Orin, Urano), (Orin, Orin), (Virgo, Virgo), (Urano,
Urano)}
xii) A = {4, 1, 8, 9}
S = {(4, 1), (1, 8), (9, 9), (4, 8), (1, 1), (4, 4), (8, 8), (4, 9),
(1, 9), (8, 9)}
xiii) A = {1, 8, 6, 4}
S = {(1, 8), (6, 4), (1, 1), (8, 1), (6, 6), (4, 6), (8, 8)}

9. Sea el conjunto de rectas en el plano.


Determine qu propiedades posee la relacin ser perpendicular a.
10. En un conjunto de personas determine las propiedades de la relacin tener
los mismos aos que.
11. En un conjunto de circunferencias de un plano se define cRc' si y slo si c es
tangente con c'. Determine las propiedades de esta relacin.
12. Determine las propiedades de la relacin ser hermano de en un conjunto
de personas.
13. Determine las propiedades de la relacin ser hijo de en un conjunto de
personas.
14. En un conjunto de jvenes determine las propiedades de la relacin ser
ms alto que.
15. En el conjunto de polgonos determine las propiedades de la relacin tener
igual nmero de lados que.

Soluciones

1. R1: Refleja, simtrica, transitiva c) Antisimtrica


R2: Simtrica d) Refleja
R 3 :Refleja, simtrica, antisimtrica, e) Simtrica
transitiva f) Simtrica
R4: Antisimtrica, transitiva 4. Refleja, antisimtrica y transitiva
R5: Refleja, antisimtrica, transitiva
2. a) Refleja, antisimtrica, transitiva China Sudn
b) Simtrica 6. a) S b)
India Japn
c) Refleja, simtrica, antisimtrica,
transitiva
d) Antisimtrica 5 2
e) Refleja (0 es par), simtrica, transi- 7. i) a) S b)
6 9
tiva
f) Simtrica 8 9
ii) a) S b) 1 3
g) Simtrica
h) Refleja, simtrica
3. a) Antisimtrica iii) a) S b) 6 9
b) Refleja, antisimtrica, transitiva 1 2

150 Relaciones y funciones

150-151.(2003) 150 20/11/02, 10:51 AM


CAPTULO 3

iv) a) S b) 4 9 Caoba
vii) a) S b) c) Orden
1 7 rojo
verde parcial.
plomo
v) a) S b) Alhu
Purn Rapel 4
viii) a) S b) c) Orden
Maule parcial.
5 2 7
vi) a) No, no es simtrica.
b) No se puede clasificar. ix) a) S b)
4
c) Orden
vii) a) S b) Lemur Coat 6 parcial.
Okapi Koala 7 9
5
x) a) S b) c) Orden
4 2 8 parcial.
8. i) a) No
7
ii) a) S b) c) Orden
Urano
4 parcial. xi) a) S b) c) Orden
Orin parcial.
5 2
Libra Virgo
iii) a) S b) 1 c) Orden
parcial. 9
4 xii) a) S b) c) Orden
8
total.
2 5 1
4
8
iv) a) S b) c) Orden xiii) a) No es de orden, no es refleja.
6 3 parcial.
9. Simtrica.
7
10. Refleja, simtrica, transitiva. (Es relacin
v) a) S b) Achao c) de equivalencia, las personas quedan
Orden
Alhu parcial. clasificadas segn su edad.)
Talca Ancud 11. Simtrica.
12. Simtrica, transitiva.
vi) a) No es relacin de orden, no es 13. Antisimtrica.
antisimtrica. 14. Antisimtrica, transitiva.
15. Refleja, simtrica, transitiva.

Funciones 3.4

3.4.1 Conceptos bsicos


Dada una relacin F : A Q B, esta relacin es funcin si y slo
si cada elemento de A tiene imagen nica en B.
Dom F = A
(
(F : A Q B funcin) (F(x) = y A F(x) = z y = z )
En un grfico sagital, una relacin es funcin si de todos los
elementos del primer conjunto sale una sola flecha.

Relaciones y funciones 151

150-151.(2003) 151 20/11/02, 10:51 AM


a 1 a 1
b 2 b 2
c 3 c 3
es funcin no es funcin

En un grfico cartesiano una relacin es funcin si al trazar


cualquier paralela al eje y sta corta en un solo punto al grfico
de la relacin.
y y

x x

es funcin no es funcin

Composicin de funciones.
Sea f : A Q B y g : B Q C funciones
(g o f) (x) = g (f (x)) ver ejercicio 5.
Algunas funciones reales interesantes.
Funcin constante Funcin idntica
f (x) = k I (x) = x I(x)

f(x) 2 .
k
1 .
y=k

x
2 1
. 1
1 2 x

. 2

Dom f = R Dom I = R
Rang f = {k} Rang I = R

Funcin valor absoluto Funcin parte entera

y = |x| =
{ x si x 0
x si x < 0
y = [x] = parte entera de x
[x] es el entero que cumple
x 1 [x] x

y=|x | y = [x]
3

2 2

1 1

x -3 -2 -1 1 2 3 4 x
-2 -1 1 2
-1

-2

-3

Dom |x| = R Dom [x] = R


Rang |x| = R+ K {0} Rang [x] = Z

152 Relaciones y funciones

152-153. 152 08/11/2001, 15:18


CAPITULO 3
CAPTULO

1. Sean A = {1, 2, 3} y B = {4, 5, 6}. Dadas las relaciones de A en


B, determinar cules son funciones.
Ejercicios
a) R = {(1, 4) (2, 5) (3, 6)}
resueltos
b) R = {(1, 4) (2, 4) (3, 4)}
c) R = {(1, 4) (1, 5) (1, 6) (2, 4) (3, 6)}
d) R = {(1, 5) (2, 4) (1, 6) (2, 6)}
e) R = {(1, 6) (2, 4) (3, 6)}
Solucin:
a) R es funcin porque todos los elementos de A tienen una
sola imagen en B.
b) R es funcin porque todos los elementos de A tienen una sola
imagen en B, no importa que sta sea la misma para todos.
c) R no es funcin porque 1 A tiene ms de una imagen en B.
d) R no es funcin porque 3 A no tiene imagen en B.
e) R es funcin porque todos los elementos de A tienen una
nica imagen en B.

2. Dada la funcin f = {(1, 3) (2, 5) (3, 7) (4, 5)}


Determinar:
a) Dom f
b) Rang f
c) f (1); f (3)
d) el diagrama sagital
Solucin:
a) Recordemos que el dominio de una funcin es el conjunto
de las preimgenes.
Dom f = {1, 2, 3, 4}
b) Recordemos que el rango de una funcin es el conjunto de
las imgenes.
Rang f = {3, 5, 7}
c) f (1) = 3 , f (3) = 7
d) 1
2 3
3 5
4 7

3. Dada la funcin: f : R R definida por:


1
f (x) = | x2|1
3
a) Graficar f (x). Hallar Dom f y Rang f.
b) Determinar f ( 3), f (6), f (2)
c) Determinar x si f (x) = 1
Solucin:
a) Como se trata de una funcin valor absoluto, veremos para

qu valor de x, | 1 x 2 | se hace cero.


3

Relaciones y funciones 153

152-153. 153 08/11/2001, 15:19


Ejercicios 1
=0
1 x2=0 1 x=2x=6
x2
resueltos 3 3 3
f (6) = 6
3
2 1=0 1=1
Como es un valor absoluto menos 1, la grfica se traslada
una unidad hacia abajo.
Encontraremos, adems, la imagen de dos valores simtricos
con respecto a 6, por ejemplo, 5 y 7.
5 1 2
f (5) = 2 1=
1= 3
3 3
1 2
f (7) = 7 2 1 = 1= 3
3 3
Para determinar en qu valor de x f (x) es 0, hacemos:
1
x2 1=0 1
x2 =1
3 3
x 2=1 V x 2 =1
3 3
x =9 V x=3
Luego el grfico es:
f(x)

3 6 9 12 x
-1

Dom f = R, ya que cada nmero real tiene imagen en R.


Rang f = {y R / y 1}, ya que el menor valor de la
funcin es 1.
1
b) f ( 3) = 3
3 2 1 = | 1 2| 1 = | 3| 1 = 2
1
f (6) = 6 2 1 = |2 2| 1 = 0 1 = 1
3
1 2 6 4 1
f (2) = 3
2 2 1= 1= 1=
3 3 3 3
1 1
c) f (x) = 1 3
x2 1=1 x2 =2
3


x
2 = 2 V x 2 = 2
3
3
x x
= 4 V =0
3 3
x = 12 V x =0
As, si f (x) = 1 entonces x = 12 y x = 0
x+3
4. Sea f : R Q R definida por f (x) =
x3
a) Es f una funcin?
b) Si no lo es, cmo podemos hacer que lo sea.
c) Hallar f (1), f (0), f (3), f ( 3), f (5)
Solucin:
a) f no es funcin, porque 3 no tiene imagen, ya que al
calcular f (3) aparece una indeterminacin.

154 Relaciones y funciones

154-155. 154 08/11/2001, 16:31


CAPTULO 3

b) Si f la definimos de R {3} en R, entonces f es funcin, porque cada


nmero real distinto de 3 tiene una imagen nica en R.
1+3 4
c) f (1) = = = 2
13 2
0+3
f (0) = =1
03
f (3) = no existe
3+3 0
f ( 3) = = =0
33 6
5+3 8
f (5) = = =4
53 2

5. Sean f y g funciones reales definidas por


f (x) = 2x 3 y g (x) = 4 5x. Hallar :
a) (f o g) (3)
b) (g o f) ( 1)
c) Una frmula para f o g y para g o f

Solucin:
a) (f o g) (3) = f (g (3)) = f (4 5 3) = f ( 11) = 25
b) (g o f) ( 1) = g (f ( 1)) = g (2 ( 1) 3) = g ( 5) = 29
c) (f o g) (x) = f (g (x)) = f (4 5x) = 2 (4 5x) 3 = 5 10x
(g o f) (x) = g (f (x)) = g (2x 3) = 4 5 (2x 3) = 19 10x

Ejercicios
1. Sean A = {1, 3, 5, 7} y B = {2, 4, 6}. b) R = {(0, 3), (3, 0), ( 3, 3)}
Determine si las siguientes relaciones de c) R = {(0, 3), (0, 3), (0, 0)}
A en B son o no funciones. d) R = {( 3, 0), (3, 0)}
Justifique la respuesta.
e) R = {( 3, 0), (0, 0), (3, 0)}
a) R = {(1, 2), (3, 4), (5, 6), (7, 6)} f ) R = {( 3, 3), (3, 3), (0, 3), (0, 3)}
b) R = {(1, 2), (3, 4), (5, 6)} g) R = {(3, 0), (3, 3), (3, 3)}.
c) S = {(3, 4), (5, 6), (7, 2), (7, 4)} 3. Sean A = {6, 7, 8} y B = {3, 6, 9}.
d) S = {(1, 4), (3, 4), (5, 4), (7, 4)} Sea f una relacin de A en B definida
e) T = {(7, 6), (5, 4), (3, 2), (1, 2)} en los diagramas dados. Determine
f ) T = {(1, 6), (3, 4), (5, 2), (5, 4), (7, 6)} en qu casos f es funcin. Justifique
la respuesta.
g) R = {(3, 2), (3, 4), (3, 6)} f
a) A B
2. Sea A = { 3, 0, 3}. Determine si las
6 3
siguientes relaciones en A son o no
7 6
funciones. Justifique la respuesta. 8 9
a) R = {( 3, 3), (0, 0), (3, 3)}

Relaciones y funciones 155

154-155. 155 08/11/2001, 16:31


Ejercicios 6. Sea F = {(2,3) (3,5) (4,7) (5,9)} una
f
b) A B funcin. Determine:
6 3 a) Dom F
7 6 b) Rang F
8 9
c) F(2), F(3), F(5)
d) El diagrama sagital
f
A B
7. En cada una de las siguientes funciones
c)
6 3 encuentre las imgenes pedidas.
7 6
8 9 a) f : NQN, f(n) = 5 + n
f(1), f(5), f(10), f(50)
f b) f : ZQZ, f(x) = 6 4x
A B
f(3), f(0), f(5), f(9)
d)
6 3
7 6 c) h : RQR, h(x) = 3x 1
8 9 1
h 2 , h ( 13 ), h(6), h(a + 1)

f d) f : RQR, f(x) = 3x2 + 2x 1


A B
e) f(3), f 1 , f(0), f(0,2)
6 3 2
7 6 e) f : RQR, f(x) = (x 2) (x + 3) (x 5)
8 9 f(1), f(0), f(1), f(2)
f) g : RQR, g(x) = |x2 3|
f
A B g(4), g(1), g(1), g(3)
f) g) f : RQR, f(x) = 4x
6 3
7 6 f(a), f(2a), f(a + ), f(a 3)
8 9
8. Sea f : NQN la funcin definida por
4. Dadas las relaciones siguientes, deter-
mine si son o no funciones. Justifique
f(n) =
{
n + 2 si n 4
n 2 si n > 4

la respuesta en caso de que no lo sean. Encuentre:


12 x f(1), f(3), f(4), f(5), f(10)
a) R = {(x, y) E N N / y = 3
}
9. A = {4, 3, 2, 1, 0, 1, 2, 3, 4}
b) F = {(x, y) E N N / y = x + 1}
Sea f : AQR la funcin definida por
c) R = {(x, y) E N N / y = x 1} 2
x 1
f(x) =
d) F = {(x, y) E N N / y es 2
mltiplo de x} Determine Rang f.

e) G = {(x, y) E N N / y = x2 + x} 10. Sea g : RQR una funcin definida por

f) G = {(x, y) E N N / y = x2 + 1}
g) R = {(x, y) E N N / y = 7 x}
g(x) =
{ x+3
2
x + 3
si x 1
si 1 < x < 3
si x 3
5. De las relaciones del ejercicio anterior a) Grafique la funcin g. Hallar
que no son funciones de N en N, Dom g y Rang g.
cules seran funciones si estuvieran b) Determine
definidas de N en Z? g(0), g(1), g(1), g(2), g(2,5), g(5)

156 Relaciones y funciones

156-157. 156 08/11/2001, 16:32


CAPTULO 3

11. Sea f : RQR una funcin definida por: c) Determine x si g(x) = 1


2 si x 1 d) Determine x si g(x) = 0
f(x) =
{ x1
1
si 1 < x < 2
si x 2
17. Sea f: RQR una funcin definida por
f(x) = 1 + [x]
a) Grafique la funcin f. Halle: Dom f a) Grafique f. Halle Dom f y Rang f
y Rang f. b) Determine:
b) Encuentre f(1), f(1,3), f(1,5), f(0) f(0,2), f(3),
7 4 f(3,5), f(3,8)
f
3
, f(1), f(0), f(1), f(2), f
3 c) Determine: x tal que f(x) = 2
12. Sea g: RQR una funcin definida por d) Determine: x tal que f(x) = 4

g(x) =
{ 2x + 6
3
x+2
si x < 2
si x 2
a) Grafique la funcin g. Halle: Dom g
18. Sea f: RQR una relacin definida por

f(x) =
x3
x+3
a) Es f una funcin de R en R?
y Rang g
b) Si no lo es, cmo puede hacer que
b) Determine
lo sea?
g(3), g(1), g(0), g(2), g(3), g(10)
c) Determine: f(3), f (3), f(0), f (2)
13. Sea g: RQR una funcin definida por
19 . Sea f: RQR una relacin definida por
g(x) = |2x 1|
1
a) Haga un grfico. f(x) = .
x
Halle Dom g y Rang g.
b) Determine g(2), g(2), g(5) a) Es f una funcin de R en R?
c) Determine x si g(x) = 9 b) Si no lo es, cmo puede hacer que
d) Determine x si g(x) = 0 lo sea?

14. Sea f: RQR una funcin definida por c) Determine: f(0), f(1), f(2), f
1
2 (1)
1
f(x) = | x + 3| + 1
2
f( 1), f(2) y f
2 ( )
a) Haga un grfico de f(x). d) Bosqueje un grfico de f
Halle Dom f y Rang f. 20. Sea f: RQR una relacin definida por
b) Determine f(2), f(5), f(6), f(7), f(0)
1
c) Determine x si f(x) = 5 f(x) =
x2
d) Determine x si f(x) = 0
a) Es f una funcin de R en R?
15. Sea f: RQR una funcin definida por b) Si no lo es, cmo puede hacer que
f(x) = 1 |x 3| lo sea?
a) Haga un grfico de f(x). c) Determine: f(2), f(3), f(4), f(1),
Halle Dom f y Rang f.
b) Determine f(3), f(2), f(4), f(0), f(6)
f ( 32 ), f(0 )
c) Determine x si f(x) = 0 d) Bosqueje un grfico de f
d) Determine x si f(x) = 2 21. Sean f(x) = 2x 1 y g(x ) = x + 2
16. Sea g: RQR una funcin definida por dos funciones reales.
g(x) = 2 |2x + 1| Halle:
a) Haga un grfico de g. a) (f o g) (2)
Halle Dom g y Rang g. b) (g o f) (2)
b) Determine c) (f o f) (3)
1 d) Una expresin para (f o g) (x) y
g ( 2 )
, g(0 ), g(1), g(1), g(2)
para (g o f) (x).

Relaciones y funciones 157

156-157. 157 08/11/2001, 16:33


Ejercicios

22. Sean f(x) = 2x, g(x) = x2 y h(x) = x+2 Responda las siguientes preguntas:
tres funciones reales. Determine: a) Una familia consumi 25 m3 en el
a) (f o g o h) (3) mes. Cunto debe cancelar?
b) (g o h o f) (2) b) Al mes siguiente se queda una llave
c) (g o h o g) (1) goteando y el medidor arroja 52
d) una frmula para (h o g o f) (x) m3. De acuerdo con el grfico de la
1 x+2
tarifa, pagar poco ms que el doble
23. Sean f(x) = , g(x) = x 2 dos fun- que el mes anterior o mucho ms
x
ciones reales. Determine: que el doble? A qu se debe?
a) Dom f y Dom g c) Cunto debe cancelar por metro
b) (f o g) (3) cbico una familia que gasta entre
c) (f o g) (2) 70 y 80 m3 al mes?
d) (g o f) (1) d) Hasta cuntos metros cbicos
1 puede gastar una familia para no
e) (g o f) ( )
2 cancelar ms de 1 US$ por m3?
f) una expresin para f o g
e) Si en la cuenta se lee un cobro fijo de
g) una expresin para g o f US$ 3 y el total a cancelar es de US$
h) Dom f o g y Dom g o f 105. Qu cantidad de m3 consumi
24. Sean f(x) = 2x2 x + 3 y g(x) = x2 1 esa familia en el mes?
dos funciones reales. Determine: f) Defina la funcin del grfico a
a) (f o g) (5) travs de una frmula y calcule su
b) (g o f) (2) dominio y su recorrido.
c) (g o g) (3)
1
d) (f o f) (1) 27. Si f(x) = x+1
2
e) una expresin para f o g y g o f a) Grafique f(x)
b) Calcule y grafique f(x + 1)
25. Sean h(x) = x3 + 1 y g(x) = x2 dos c) Calcule y grafique f(x 1)
funciones reales. Determine: d) Calcule y grafique f(x) + 1
a) (h o g) (1) e) Calcule y grafique f(x) 1
b) (g o h) (3) f) Compare los resultados y este-
c) (h o g) (x) blezca alguna conjetura.
d) (g o h) (x)
28. Escriba algunas funciones y = f(x) e
26. Suponiendo que el grfico siguiente investigue qu ocurre con f(x k) y
representa los valores en US$ por f(x) k para k E N.
metro cbico del consumo de agua 1 1
potable en una ciudad desrtica. Ej. 1: f(x) = ; f(x+2) = ;
x x+2
[US$] 1
f(x) + 2 = +2
3 x
2,4
2 Ej. 2: f(x) = x + 3 ; f(x5) = x 2 ;
1,6 f(x) 5 = x + 3 5
1
0,6 Grafique y determine intersecciones con
los ejes, y determine dominios y rangos.
10 20 30 40 50 60 70 80 90 100 110 120130140 [m3]

158 Relaciones y funciones

158-159. 158 08/11/2001, 16:34


CAPTULO 3
Soluciones

1. a) R es funcin porque cada elemento d) No es funcin, cada nmero


de A tiene imagen nica en B. natural tiene infinitas imgenes.
b) R no es funcin porque 7 E A no e) Es funcin.
tiene imagen en B. f) Es funcin.
c) S no es funcin porque 1 E A no g) No es funcin, los nmeros naturales
tiene imagen en B y adems 7 E A mayores que 6 no tienen imagen en
tiene ms de una imagen. N.
d) S es funcin porque cada elemento 5. c, y g,
de A tiene imagen nica en B. 6. a) Dom F = {2, 3, 4, 5}
e) T es funcin porque cada elemento b) Rang F = {3, 5, 7, 9}
de A tiene imagen nica en B. c) F (2) = 3; F (3) = 5; F (5) = 9
f) T no es funcin porque 5 E A tiene
d)
ms de una imagen en B.
g) R no es funcin porque hay elementos 2 3
de A que no tienen imagen en B 3 5
y adems 3 E A tiene ms de una 4 7
imagen en B. 5 9
2. a) R es funcin porque cada elemento 7. a) f (1) = 6, f (5) =10, f (10) = 15,
de A tiene imagen nica en A. f (50) = 55
b) R es funcin porque cada elemento b) f (3) = 18, f (0) = 6, f (5) = 14
de A tiene imagen nica en A. f (9) = 30
c) R no es funcin porque 0 E A tiene
1 5 1
ms de una imagen en A y adems c) h = ,h = 0, h (6) = 17
2 2 3
hay elementos de A que no tienen h (a + 1) = 3a + 2
imagen.
d) R no es funcin porque 0 no tiene d) f (3) = 20, f 1 = 5 , f (0) = 1
2 4
imagen. f (0,2) = 0,48
e) R es funcin porque cada elemento e) f (1) = 36, f (0) = 30, f (1) = 16
de A tiene una nica imagen en A. f (2) = 0
f) R no es funcin porque 0 E A tiene f) g (4) = 13, g (1) = 2, g (1) = 2
ms de una imagen en A. g (3) = 6
g) R no es funcin porque hay elementos
g) f (a) = 4a, f (2a) = 8a, f (a + E) = 4a+4E
de A que no tienen imagen en A y f (a 3) = 4a 12
adems el 3 E A tiene ms de una
8. f (1) = 3, f (3) = 5, f (4) = 6, f (5) = 3,
imagen en A.
f (10) = 8
3. a) Es funcin.
15 3 1
b) Es funcin. 9. Rang f = , 4, , 0,
2 2 2
c) No es funcin, 6 no tiene imagen. 10. a) Y
d) No es funcin, 7 tiene ms de una
4
imagen, 6 y 8 no tienen imagen.
e) Es funcin.
f) Es funcin.
4. a) No es funcin, hay nmeros natu- -1 1 3 X
rales que no tienen imagen en N,
ejemplo: Dom g = R
11 Rang g = {y E R / y 4}
si x = 1 entonces y = F N.
3
b) Es funcin. b) g (0) = 3, g (1) = 4, g (1) = 2
c) No es funcin, 1 no tiene imagen en N. g (2) = 2, g (2,5) = 2, g (5) = 2

Relaciones y funciones 159

158-159. 159 08/11/2001, 16:34


Soluciones
f(X)
15. a) 1
11. a) Y
1 1 2 3 4 5 6 X
-1
-2
2 X
-2

Dom f = R Dom f = R
Rang f = { y E R / 2 y 1} Rang f = { y E R / y 1}
7
b) f ( ) 3
= 2, f ( 1) = 2, f (0) = 1, b) f (3) = 1, f (2) = 0, f (4) = 0,
4 1
f (1) = 0, f (2) = 1, f
3
=
3 ( ) f (0) = 2, f (6) = 2
12. a) c) x = 4 V x = 2
Y
d) no existe x ya que f (x) = 2 F Rang f.
4

g(x)
2 16. a)
2
1
-3 2 X
-3
-1 -1 1 x
2 2 2

Dom g = R
10
Rang g = { y E R / y V y 4} Dom g = R
3
Rang g = {y E R / y 2}
b) g( 3) = 0, g (1) = 4 , g (0) = 2,
3
1
g (2) = 4, g (3) = 5, g (10) = 12 b) g ( 2 )
= 2, g (0) =1, g (1) = 1,
13. a) g (1) = 1, g (2) = 1
g (X)
c) x = 0 V x = 1
1 3
d) x = V x=
2 2
1 X
17. a)
Dom g = R f(x)
Rang g = { y E R / y 0}
b) g (2) = 3, g (2) = 5, g (5) = 9
c) x = 5 A x = 4 x
f(X)
d) x = 1
2 4
14. a)

1
Dom f = R
6 X
Rang f = Z
Dom f = R
Rang f = { y E R / y 1} b) f (1) = 2, f (1, 3) = 2, f (1, 5) = 2,
3 3 f (0) = 1, f (0, 2) = 1, f (3) = 2,
b) f(2) = 5, f(5) = , f(6) = 1, f(7) =
2 2 f (3, 5) = 3, f (3, 8) = 3
f(0) = 4
c) x = 6 c) x E [1, 2)
d) H x E R d) x E [ 5, 4)

160 Relaciones y funciones

160-161.(2003) 160 20/11/02, 10:53 AM


CAPTULO 3

18. a) f no es funcin de R en R porque 21. a) 7 b) 5 c) 9 d) (f o g) (x) = 2 x + 3


f (3) no existe. (g o f) (x) = 2 x + 1
b) definiendo f de R { 3} en R.
22. a) 50 b) 4 c) 9 d) 4 x2 + 2
c) f (3) = 0, f (3) = no existe,
1
f (0) = 1, f (2) =
5 23. a) Dom f = R {0}, Dom g = R {2}

19. a) f no es funcin de R en R porque b) 1 c) no existe d) 3 e) no existe


5
f (0) no existe. x2 1+2x
f) g)
b) definiendo f de R {0} en R. x+2 12x
1 h) Dom f o g = R { 2}
c) f (0) = no existe, f (1) = 1, f (2) = ,
2
f ( 12 ) = 2, f (1) = 1, f (2) = 1
2
, Dom g o f = R { 12 }
1
f (
2)
=2
24. a) 1.131 b) 80 c) 63 d) 31
d)
f(x) e) (f o g) (x) = 2 x4 5 x2 + 6;

2
(g o f) (x) = 4 x4 4 x3 + 13 x2 6x + 8

1
1
25. a) 2 b) 676 c) x6 + 1 d) x6 + 2 x3 + 1
2

-1 1 1 2
2 x
26. a) US$ 15
b) Debe cancelar US$
52, lo que repre-
senta mucho ms
del doble, porque
el consumo cay
20. a) f no es funcin de R en R porque
en el tramo entre
f (2) no existe. 30 y 60 m3, cuyo
b) definiendo f de R {2} Q R valor es de US$ 1
1 por m3.
c) f (2) = no existe, f (3) = 1, f (4) = ,
2 c) US$ 1,6
3 1
f (1) = 1, f ( )
2
= 2, f (0) =
2
d) 60 m3
e) 63,75 m3
d)
f(x)

{
0,6 si 0 < x 30
1 si 30 < x 60
f) f(x) =
1,6 si 60 < x 100
2,4 si 100 < x ;

2 3 4 x Dom f = R+ K {0}
Rang f = {0,6 , 1 , 1,6 , 2,4}

Relaciones y funciones 161

160-161.(2003) 161 20/11/02, 10:53 AM


Soluciones
27. f(X)+1= 12 x + 2
f(X+1)= 12 x + 32
f(X)= 12 x + 1 f(X)= 12 x + 1
f (X1)= 12 x + 12 2 f(X)1= 12 x
1
f) f(x+1) est desplazada 1
0
3 2 1 unidad a la izquierda de f(x).
f(x) + 1 est desplazada 1
unidad hacia arriba de f(x).

3.4.2 La funcin de primer grado


(Ecuacin de la recta)
y = f (x) = a x + b, a 0, a, b E R
Dom f = R Rang f = R
Su grfico es una recta y depende de los valores que tomen a y b.
b se llama coeficiente de posicin y (0, b) es el punto donde
la recta intersecta al eje y.
a, coeficiente de x, es el valor de la pendiente de la recta.
Si en una recta (x1, y1) y (x2, y2), con x1 x2 son puntos
de ella, entonces.
y2 y1 y
Su pendiente es m = =
x2 x1 x
y2 y1
Su ecuacin es: (y y1) = (x x1)
x2 x1
la cual se puede escribir en la forma: y = m x + k, donde m es
la pendiente y k el coeficiente de posicin.
Si los puntos (x1, y1) y (x2, y2) son tales que x1 = x2, entonces
la recta es paralela al eje y, su pendiente es indeterminada y
su ecuacin es x = x1
Se llama ngulo de inclinacin (a) de una recta al ngulo que la
recta forma con la parte positiva del eje x.

Relacin entre el ngulo de inclinacin () y la pendiente (m) de una recta L


=0 0 < < 90 = 90 90 < < 180
m m=0 m>0 Indeterminada m<0
G Y Y Y Y
R
L L

F
X X X X
I
C L
O

162 Relaciones y funciones

162-163.(2003) 162 20/11/02, 10:56 AM


CAPTULO 3

Relacin entre las pendientes y la posicin de dos rectas en el plano.


Dos rectas L1 y L2 son paralelas si y slo si sus pendientes son iguales.
(L1 // L2) (m1 = m2)

Dos rectas L1 y L2 son perpendiculares si y slo si el producto


de sus pendientes es 1
(L1 L2) (m1 m2 = 1)

1. Dada una ecuacin de primer grado en dos variables, siempre es


posible despejar una de las variables en funcin de la otra.
Ejercicios
As, dada la ecuacin: ax + by + c = 0 (b 0) resueltos
podemos escribir:

y = ax c
b
a c
y = x
b b
a c
si hacemos =m y =k
b b
escribimos y = mx + k, que es una funcin de primer grado y
representa una recta de pendiente m y coeficiente de posicin k.
a se llama coeficiente de x.
b se llama coeficiente de y.
c se llama trmino libre.
Dada la ecuacin 5x + 3y 6 = 0
Hallar:
a) el coeficiente de x.
b) el coeficiente de y.
c) el trmino libre.
d) la pendiente m de la recta que representa.
e) el coeficiente de posicin k de la recta que representa.
Solucin:
a) coeficiente de x = 5
b) coeficiente de y = 3
c) trmino libre = 6
d) debemos escribir la ecuacin en forma de funcin de primer
grado:
5
y = x+2
3
5
luego m =
3
e) En la misma funcin del punto d) k = 2
2. Escribir la ecuacin 2x 1 = 0 como una funcin de primer
grado y = f (x).
Solucin:
No es posible escribir la ecuacin 2x 1 = 0 como una funcin
de primer grado y = f (x) porque la variable y no aparece en la
ecuacin y esto significa que su coeficiente es 0.

Relaciones y funciones 163

162-163.(2003) 163 20/11/02, 10:56 AM


Ejercicios 3. Dada la ecuacin 5x 3y + 8 = 0 encontrar dos puntos que
pertenezcan a la recta que representa y graficarla.
resueltos
Solucin:
Recordemos que un punto pertenece a una recta si y slo si
satisface su ecuacin.
Dando valores a una de las variables podemos obtener el valor
correspondiente a la otra, de modo que el par (x, y) sea un punto
de la recta representada por la ecuacin 5x 3y + 8 = 0.
As, si x = 2 5 2 + 3y + 8= 0
3y = 18
y = 6 Y
2
(2, 6) es punto de la recta cuya 3

-2 2
ecuacin es 5x + 3y + 8 = 0 X

Si x = 2 5 (2) + 3y + 8 = 0
3y = 2
2
y =
3
2 -6
(2, ) es punto de la recta cuya
3
ecuacin es 5x + 3y + 8 = 0
Sabemos que dados dos puntos de una recta, sta queda definida
(ver grfico).
4. Graficar las rectas representadas por las siguientes ecuaciones:
a) x 2y + 6 = 0
b) x 1 = 2
c) 1 2y = 5
Solucin:
a) Para graficar la recta cuya ecuacin es x 2y + 6 = 0 buscaremos
dos puntos de ella.
Si x = 2 2 2y + 6 = 0 Y

2y = 8 5
y = 4 4

(2, 4) es un punto de la recta


Si x = 4 4 2y + 6 = 0
2y = 10 2 4 X
y = 5
\ (4, 5) es un punto de la recta
(ver grfico).
b) La ecuacin x 1 = 2 es Y

equivalente a la ecuacin x = 3;
representa todos los puntos del
plano cuya abscisa vale 3 (es 3 X
paralela al eje Y) .
c) La ecuacin 1 2y = 5 es equi- Y
valente a la ecuacin y = 2; repre-
senta todos los puntos del plano
X
cuya ordenada es 2 (es paralela -2
al eje X).

164 Relaciones y funciones

164-165. 164 08/11/2001, 16:39


CAPTULO 3

5. Hallar los puntos en que la recta cuya ecuacin es 4x 6y + 8 = 0


intersecta a los ejes coordenados. Graficarla.
Solucin:
Interseccin al eje X: Como todos los puntos del eje x son de
la forma (x, 0), debemos encontrar el punto que satisface la
ecuacin de la recta y tiene ordenada y = 0
Si y = 0 4x + 8 = 0 x = 2
La recta intersecta al eje X en (2, 0)
Interseccin al eje Y: Como todos los puntos del eje y son Y
de la forma (0, y), debemos encontrar el punto que satisface
la ecuacin de la recta y tiene abscisa x = 0
4 4
Si x = 0 6y + 8 = 0 y = 3
3

La recta intersecta al eje Y en 0,


4
3
-2 X

Con los dos puntos de interseccin podemos trazar


su grfico.
6. Hallar la ecuacin de la recta que pasa por los puntos (3, 2) y
(1, 1). Graficarla.
Solucin:
Si (x1, y1) y (x2, y2) son puntos de una recta, su ecuacin es:
y2 y1
y y1 = (x x1) (*)
x2 x1
y2 y1 Y
donde = m (pendiente).
x2 x1
2
Sea (x1, y1) = (3, 2) y (x2, y2) = ( 1, 1)

y2 y1 1 2 3 3 1 3 X
= = = -1
x2 x1 13 2 2
Luego en (*) queda:
3
y2 = (x 3) /2
2
2y 4 = 3x 9
3x 2y 5 = 0 es la ecuacin pedida (ver grfico).
7. Hallar la ecuacin de una recta cuya pendiente es 3 y pasa
por el punto (2, 5)
Solucin:
m = 3 y (x1, y1) = (2, 5)
La ecuacin pedida es:
y y1 = m ( x x1)
y + 5 = 3 ( x 2)
y + 5 = 3x + 6
3x + y 1 = 0 es la ecuacin buscada.
8. Determinar si los tres puntos (3, 2), (2, 0) y (1, 6) son colineales.
Solucin:
Para que tres puntos sean colineales, la pendiente de la recta
definida por dos de ellos debe ser igual a la pendiente de la recta

Relaciones y funciones 165

164-165. 165 08/11/2001, 16:39


Ejercicios definida por el tercer punto y cualquiera de los dos anteriores:
resueltos Pendiente m1 entre (3, 2) y (2, 0)
02
m1 = = 2
2+3
Pendiente m2 entre (2, 0) y (1, 6)
60
m2 = = 2
1+2
Como m1 = m2, los tres puntos son colineales.

9. Determinar si las rectas determinadas por los siguientes pares de


ecuaciones son paralelas, perpendiculares o slo son secantes.
a) L1: 2x 3y + 6 = 0 L2 : 9y 6x = 0
b) L1: 2x + y 1 = 0 L2 : x 2y + 3 = 0
c) L1: 1 4x = y L2 : 2x + 5y 3 = 0

Solucin:
Para responder hay que calcular y comparar las pendientes
de las rectas.
2
a) L1: 2x 3y + 6 = 0 y = x + 2
3
mL1 = 2
3
6 2
L2: 9y 6x = 0 y = 9
xy= x
3
mL2 = 2
3

luego, como mL1 = mL2, entonces L1 // L2

b) L1: 2x + y 1 = 0 y = 2x + 1
mL1 = 2
1 3
L2: x 2y + 3 = 0 y = x+
2 2
mL2 = 1
2

luego, como mL1 mL2 = 2


1
2
= 1, entonces L1 L2

c) L1: 1 4x = y y = 4x + 1
mL1 = 4
2 3
L2: 2x + 5y 3 = 0 y = x+
5 5
mL2 = 25
Como mL1 mL2, las rectas no son paralelas.
Como mL1 mL2 1, las rectas no son perpendiculares.
Luego podemos decir que son rectas secantes. Con ayuda de
algn concepto de trigonometra podramos calcular el ngulo
formado por las rectas.
10. Encontrar la ecuacin de la familia de rectas que pasan por el
punto (5, 3). Dibujar 3 de ellas.

166 Relaciones y funciones

166-167. 166 08/11/2001, 16:45


CAPTULO 3

Solucin:
Para definir una recta necesitamos conocer dos parmetros, los que
pueden ser dos puntos de ella o un punto y su pendiente.
En este caso slo tenemos un parmetro, el punto (5,3), por ello
slo podemos encontrar la ecuacin de la familia de rectas (todas
las rectas) que pasan por el punto (5,3) dndole a la pendiente
un valor m.
Entonces la pendiente es m y (x1, y1) = (5,3). Y
As, la ecuacin de la familia es:
8 m=1
y y1 = m (x x1)
y 3 = m (x 5)
mx y 5m + 3 = 0
3 m=0
Ahora, para dibujar 3 de ellas debemos asignar
3 valores distintos a su pendiente m.
si m = 1 x y 2 = 0 2 5 8 X
si m = 0 y + 3 = 0 2 m = -1
si m = 1 x y + 8 = 0

Ejercicios
1. Determine coeficiente
l) 0,5x = 1 + 2,5y 3. Escriba dos soluciones
de x, coeficiente de y,
de cada una de las ecua-
y trmino libre en cada m) 3x = 2y + x 1
ciones siguientes:
una de las siguientes
n) ax = 2by + c
ecuaciones. a) 4x y + 2 = 0
o) ax bx + aby b = 0
a) 4x + 6y + 1 = 0 b) 6x 2 = y
p) y = mx + k
b) 3x y + 5 = 0 c) 3x 1 = 5
q) ax by = 2a + b
c) 2x = 5 + y d) 6x y = 0
r) x = 1 y + x 3k
d) 6y 2x = 3 e) x 2y + 9 = 0
s) 2x ky + y 3 = 1
e) 4x + y = 0 f) x + 1 = 0
t) x + 2y 4 = y + k g) y = 2
f) 6x = 2y
g) 4x 1 = 0 1 3
h) x y=1
2. Exprese cada una de las 2 4
h) 1 5x = 0 ecuaciones del ejercicio 1
i) 9x y 0,5 = 0
anterior como una fun- 2
i) 1 = 6y
cin de primer grado j) 12x y + 1 = 0
j) 2y 3 = 0
y = f (x). Si no es posible,
k)
1 x+ 1 y1=0 indique por qu. 4. Determine cules de los
3 4 pares de valores indica-

Relaciones y funciones 167

166-167. 167 08/11/2001, 16:45


Ejercicios c) x + ky 5 = 1
q) x + y = x 3
(3, 1)
dos satisfacen la ecuacin dada. r) 2x + 3y = 3y 6
d) kx y = 2k
a) 6x y + 3 = 0
(5, 1) 8. Determine la interseccin
(1, 1), (0, 3), (1, 3)
con cada uno de los ejes
e) kx + 2ky + k = 0
b) y + 2x 5 = 0 coordenados de las rectas
(1, 1)
representadas por las ecua-
( 12 ,4,) ,(2, 1), (3, 1) f) kx + 2ky + k = 0 ciones siguientes:
c) 6y = 12 (1, 1)
(2, 1), (1, 2), (2, 2) a) x + y = 5
g) 3x k + 2y = 5
d) 4x y = 0 (0, 0) b) 6x 2y + 3 = 0
(0, 0), (1, 4), (2, 7) c) x 5y + 10 = 0
h) 1 2k + x y = 0
e) x = 2y 1 (3, 1) d) 2x y + 6 = 0
(1, 1), (3, 2), (2, 3)
i) 3kx + y 1 = 0 e) 3x 5 = 0
f) 1 = x + 2y (4, 1)
(1, 1), (1, 1), (1, 1) f) 4y + 2 = 0
7. Grafique cada una de las g) 2x 5y = 0
5. Determine cules de las siguientes ecuaciones:
siguientes ecuaciones repre- h) 4y x = y + 3x 1
sentan una recta que pasa a) 3x y = 1
i) 6x 5 = 0
por el origen (el pun to
b) x + 2y 5 = 0 j) 4y = 2
(0, 0)).
c) 6x + 2y = 0 k) ax by = ab, a, b 0
a) 6x y = 0
d) 3x = 5 l) 4x 3y + 1 k = 0
b) 4x + 1 = y
e) 4y 1 = 0 m) 1 2y = 4 x
c) x 2y + 1 = 1
x + 2y 1 n) x 1 = y 2
d) 1 x 5y + 3 = 4 f) =2
3
1 o) x + 2y 1 = 0
e) 2x 5y = x y g) 1 x = y x+y y
3 p) =1+
f) 1 2y = x + 2 3 1 2 2
h) x + y 1 = 0 q) 3x 4 = 6y 1
g) 6x 2y = 5 4 3
x y
3 r) 1= +2
h) 1 + 2y = 1 3x i) 0,5x + y 0,2 = 0 3 2
4
i) x 2y + 4 = y + 1 x+2 y1 9. Sin hacer clculos, indique
j) = cul(es) de las siguientes
3 2
j) x y 1 = 1
k) 3x + y 1 = 0 ecuaciones corresponde(n)
a rectas que son paralelas
6. Determine el valor de k l) 24x + 8y 16 = 0 al eje X, paralelas al eje
en la ecuacin para que la
Y, pasan por el origen o
recta que representa pase m) y + 1 = 3x 2
intersectan a ambos ejes
por el punto dado.
n) 4x + 2y 5 = x + y en puntos distintos.
a) x 2y + k = 0
(1, 1) o) 2x 3 = 2y 4 a) 4x y = 0

b) 3x + 2y k = 0 6x 1 + 3y b) 2x y = 1 y
p) =0
(2, 3) 4

168 Relaciones y funciones

168-169. 168 08/11/2001, 16:47


CAPTULO 3

de cada una de las rectas j) (7, 1) , (7, 1)


c) x + 2y 5 = 0 representadas por las
k) (3, 4) ,(4, 3)
ecuaciones siguientes:
d) 4x 5y = 6 l) (0, 0), (1, 5)
e) 2x 1 = 0 a) 3x + y 2 = 0
13. Encuentre la ecuacin de
f) 3y 4 = 0 b) x + 5y 5 = 0
la recta dada su pendiente
g) x + 2y = 1 + x c) x + y 3 = 0 m y un punto de ella.
h) x + 1 = 3y + 1 d) 3x = y + 5 a) m = 1 (2, 3)
i) x + y 2 = 2 3
e) x + 5y = 2 b) m = (1, 1)
4
j) 4 6x = y + 4 c) m = 3 (2, 5)
f) x + 2 = y 3
k) 2x 3y + 5 = 5 x y 1 d) m =
1 (3, 1)
g) + =0
2 3 6 2
l) 4x 2y + 1 k = 0
h) 0,3x 0,5y = 0 e) m = 0 (12, 5)
m) y + 2x = 10 1
i) 2x 3 = y 3 f) m = (3, 1)
2
n) 3x y = 1 + y
j) 5x = x 4 g) m = 5 ( 6, 0)
o) 4y 6 = x
k) x 1 = 1 y h) m = 2 (0, 3)
p) x + 3 = 0
l) y = y + 2x 3 14. Encuentre la ecuacin
q) 2 (x 1) = 3 (x + y)
de la recta que corta al
r) 3 (x + 2) = 2 ( y + 3) m) y 1 = 0 eje X en (5, 0) y al eje
n) 3x 2y + 1 = x + y 1 Y en (0, 3).
10. Determine la pendiente
de la recta que pasa por x 2y 1 15. Encuentre la ecuacin
o) + =0
4 3 2
los puntos dados. de la recta cuya pen-
p) y = 6x x + 2
diente es 3 y pasa por
a) (4, 1) , (2, 3) 4
el origen.
b) (5, 1) , (1, 4) 12. Encuentre la ecuacin
de la recta que pasa
16. Encuentre la ecuacin
c) (2, 5) , (2, 5) de la recta que pasa
por los puntos dados.
d) (3, 8) , (3, 3) Graficarla. por el punto (3, 1) y
que corte al eje X en un
e) (0, 0) , (1, 1) a) (1, 1) , (3, 3) ngulo de 45.
f) (0, 0) , (1, 1)
b) (1, 5) , (3, 2) 17. Encuentre la ecuacin
g) (3, 5) , (6, 8) de la recta que pasa por
c) (6, 6) , (6, 1)
h) (1,5 ; 2,5), (3,5 ; 6,5) el punto (4, 2) y forma
3 4 6 3
d) (4, 3) , (3, 4) un ngulo de 150 con
i) , , ,
4 3 3 2
e) (3, 9) , (0, 9) el eje X.
j) (6, 1) , (1, 1)
f) (0, 0) , (5, 3) 18. Encuentre la ecuacin de
k) (3, 5) , (5, 3) la recta cuya pendiente
l) (0, 5) , (3, 0) g) (3, 1) , (5, 1) es igual a la pendiente de
la recta cuya ecuacin es
h) (4, 3) , (4, 7)
11. Determine la pendiente y 2x 5y 3 = 0 y que
el coeficiente de posicin i) (1, 12) , (5, 12) pasa por el origen.

Relaciones y funciones 169

168-169. 169 08/11/2001, 16:47


Ejercicios
b) m = 2 k=3 a) (5,1) (3, 2) ( 5, 6)
1
19. Encuentre la ecuacin de c) m = 5 k=
2 b) (2,1) (3, 5) (6, 3)
la recta cuyo coeficiente 3
c) (1, 6) (2, 5) (3, 4)
d) m = 1 k=
de posicin es 2 y forma 4
1
un ngulo de 135 con e) m = k=6 d) ( 1, 3) (0, 1) (2, 0)
2
el eje X. 4 e) (4, 0) (4, 1) (4, 3)
f) m= k=2
3
g) m =
3
k=2 f) ( 7, 8) ( 1, 4) (8, 10)
20. Encuentre la ecuacin de 5
la recta que pasa por el h) m = 0 k=1 g) ( 1, 8) ( 7, 4) (2, 10)
origen y forma un ngulo
i) m=5 k=0 h) (0, 0) (1, 1) (1, 2)
de 30 con el eje X.
1 1
j) m= k=
2 2 28. Determine la ecuacin y
21. Encuentre la ecuacin de 3
k) m = k=3 grafique la recta paralela
la recta que pasa por el 4
origen y forma un ngulo l) m=1 k=1 al eje X que pase por el
de 60 con el eje X. punto (3, 5).

25. Determine la pendiente,


22. Encuentre la ecuacin de la ecuacin y el coefi- 29. Encuentre la ecuacin de la
la recta que pasa por el recta que sea paralela al eje
ciente de posicin de la
origen y forma un ngulo X y que pase por el punto
recta que pasa por los
de 120 con e l eje X. (7, 3). Grafquela.
puntos (3, 2) y (2, 5).
Grafique.
23. Grafique la recta dados un 30. Determine la ecuacin de la
punto y su pendiente. recta que sea paralela al eje
26. Dada la ecuacin de una
Y y que pase por el punto
a) (3, 2) m = 1 recta, determine su pen-
(5, 8). Grafquela.
diente, su coeficiente de
b) (6, 4) m = 2
posicin y grafquela.
2
c) (3, 1) m = 3 31. Encuentre la ecuacin de la
4
a) 2 x y + 1 = 0 recta que sea perpendicular
d) (2, 3) m = 5 al eje X y que pase por el
e) (1, 1) m = 1 b) 4 x 3 y 2 = 0
punto (1, 3).
f) (4, 0) m = 1 c) x + 5 = 1
2
g) (0, 3) m = 5
32. Encuentre la ecuacin de la
h) (0, 0) m = 0 d) 3 x y + 6 = 0
recta que sea perpendicular
i) (1, 5) m = 0
e) 1 x = y + 2 al eje Y y que pase por el
j) (3, 4) m = 3
2 punto (6, 16).
k) (4, 1) m = 5
1 f) 1 = 2 y

l) (6, 1) m = indefi- g) x + 6 = y 33. Determine si las rectas L1


nida y L2 son o no paralelas.
h) 3 x 1 = y 1
Justifique la respuesta.
24. Grafique la recta conocida 27. Determine si los siguien- a) L1 : 2 x 3 y + 4 = 0
su pendiente m y su coefi- tes tros de puntos son L2 : 4 x + 6 y 1 = 0
ciente de posicin k. colineales; justifique la b) L1 : 5 x y = 1
a) m = 3 k=1 respuesta. L2 : 10 x 1 = 2 y

170 Relaciones y funciones

170-171. 170 08/11/2001, 16:48


CAPTULO 3

c) L1 : 2 x + 5 y 1 = 0 37. Halle la ecuacin de (3, 1) y (4, 2) es paralela


L2 : 2 x 5 y + 3 = 0 la recta que pasa por a la recta que une los
el punto (4, 2) y es puntos (1, 4) y (2, 5).
d) L1 : 1 3 x = y perpendicular a la recta
cuya ecuacin es: 44. Pruebe que una recta
L2 : 3 y = x + 2
2x 3y + 1 = 0. de pendiente m = 4 es
3

e) L1 : 4 x y + 1 = 3 paralela a la recta que


L2 : y 4 x = 0 38. Determine la ecuacin de la pasa por los puntos
recta que intersecta al eje X (1, 4) y (3, 7).
f) L1 : 1 3y = x en (3, 0) y es perpendicular
a la recta cuya ecuacin
L2 : 4y x = 5 45. Demuestre que la recta
es: x + y = 3.
que une los puntos
34. Determine si las rectas
(2, 3) y (4, 1) es perpen-
L1 y L2 son perpendicu- 39. Determine el valor de
k en la ecuacin dicular a la recta que pasa
lares. Justifique la res-
5x (1 + k) y 3 = 0 por los puntos (5, 2) y
puesta.
para que represente una (3, 1).
a) L1: 2x y + 3 = 0 recta paralela a la recta
L2: x + 2y 3 = 0 cuya ecuacin es: 46. Demuestre que la recta
x 5y + 8 = 0. cuyo coeficiente de posi-
b) L1: 5x + 2y 1 = 0
cin es 2 y corta al eje X
L2: 0,2x 0,5y = 0
40. Determine el valor de k en (2, 0) es perpendicular
en la ecuacin a la recta de igual coefi-
c) L1: 3x + y 6 = 0
2kx 3y + 1 = 0 para ciente de posicin pero
L2: 3x y + 6 = 0
que represente una recta corta al eje X en (2, 0).
paralela a la recta cuya
d) L1: 2x 3y + 1 = 0
ecuacin es:
L2: 4x 6y + 5 = 0 47. Determine la ecuacin
2x + 5y 3 = 0.
de la familia de rectas
e) L1: x + y = 0 que tiene pendiente 3.
L2: x y = 0 41. Encuentre el valor de k
Grafique tres de ellas.
en la ecuacin
f) L1: x + y + 5 = 0 (2 k) x + 3y 4 = 0
L2: x y + 5 = 0 para que represente una 48. Determine la ecuacin
recta perpendicular a de la familia de rectas
la recta cuya ecuacin perpendiculares a la
35. Determine la ecuacin
es: 6x + y 9 = 0. recta cuya ecuacin es:
de la recta que pasa por
3x 2y + 1 = 0.
el punto (2, 1) y es
paralela a la recta cuya 42. Encuentre el valor de k
ecuacin es: en la ecuacin 49. Determine la ecuacin de
2x + 6y 1 = 0. Grafique. x 2ky 3 = ky + k la familia de rectas que
para que represente una pasa por el punto (3, 1).
36. Determine la ecuacin recta perpendicular a la Grafique tres de ellas.
recta cuya ecuacin es:
de la recta que tiene
2y + 3 = 9x.
coeficiente de posicin 50. Determine la ecuacin de la
3 y es paralela a la familia de rectas que tienen
recta x + y 1 = 0. 43. Demuestre que la recta coeficiente de posicin 3.
Grafique. que pasa por los puntos Grafique tres de ellas.

Relaciones y funciones 171

170-171. 171 08/11/2001, 16:48


Ejercicios
51. En un diagrama cartesiano (sistema de ejes 52. En la figura siguiente fije la escala
coordenados), grafique los siguientes de valores en los ejes coordenados
segmentos de rectas en los intervalos y determine las ecuaciones correspon-
indicados. Grafique adems los puntos dientes a los segmentos que forman
P y Q. la figura.
P (4,2 ; 4,8)
Q (5,8 ; 4,8)

y=1 3,5 x 4
6 x 6,5
x=4 1 y 3
x=6 1 y 3
y = 3,6 4,5 x 5,5
y=3 2 x 8
y=6 4 x 6
3
y= x 2 x 4
2
3 6 x 8
y = x + 15 53. Haga un diseo compuesto por segmen-
2
y = 2x 5 5,5 x 6 tos de rectas y determine las ecuaciones
correspondientes. Comparta el trabajo
y = 2x + 15 4,0 x 4,5
con sus compaeros y compaeras.
x=5 6 y 7

Soluciones

1. a) 4, 6, 1 b) 3, 1, 5 c) 2, 1, 5
d) 2, 6, 3 e) 4, 1, 0 f) 6, 2, 0
g) 4, 0, 1 h) 5, 0, 1 i) 0, 6, 1
1 1
j) 0, 2, 3 k) 3
, , 1 l) 0,5 , 2,5 , 1
4
m) 2, 2, 1 n) a, 2b, c o) a b, ab, b
p) m, 1, k q) a, b, 2a b r) 0, 1, 3k 1
s) 2, 1 k, 4 t) 1, 1, 4 k

2. a) y = 2 x 1
b) y = 3x + 5 c) y = 2x 5
3 6
1 1
d) y = x+ 2
e) y = 4x f) y = 3x
3
1
g) No se puede porque el h) No se puede porque el i) y=
6
coeficiente de y es 0 coeficiente de y es 0

172 Relaciones y funciones

172-173.(2003) 172 20/11/02, 10:58 AM


CAPTULO 3

3 4
j) y = k) y = x + 4 l) y = 0, 2x + 0, 4
2 3
o) y = b a x + 1
1
m) y = x + 2
n) y = a x + c
2b 2b ab a
a 2a + b
p) y = mx + k q) y = x+ r) y = 1 3k
b b
2 4
s) y= x+ t) y=x+4+k
k1 1k

3. a) Por ejem: (0, 2), (1, 6) b) Por ejem: (1, 4), (2, 10) c) Por ejem : (2, a), (2, b)
d) Por ejem : (1, 6), (2, 12) e) Por ejem: (1, 5), (3, 6) f) Por ejem : (1, 2), (1, 3)

g) (a, 2), (k, 2) h) (2, 0) , 4, 4 i) (1, 17), (1, 19)


3
1 1
j) ,4 , ,5
4 3

1
4. a) (0, 3) b) ,4 , (2, 1), (3, 1) c) (1, 2), (2, 2)
2

d) (0, 0), (1, 4) e) (1, 1), (3, 2) f) (1, 1)

5. a, c, d, e, h

1
6. a) 1 b) 12 c) 9 d) e) k puede tomar cualquier valor
7

f) Cualquiera que sea el valor de k, la recta no pasa por (1, 1)


5
g) 5 h) 2
i) 0

3 1
8. a) (0, 5) (5, 0) b) 0, ,0 c) (0, 2) (10, 0)
2 2
5 1
d) (0, 6) (3, 0) e) 3
,0 No corta al eje Y f) 0,
2
No corta al eje X
1 1 5
g) (0, 0) h) 0,
3 4
,0 i) 6
,0 No corta al eje Y
1 1k k1
j) 0, No corta al eje X k) (0, a) (b, 0) l) 0, ,0
2 3 4
3 1
m) 0,
2
(3, 0) n) (0, 1) (1, 0) o) 0,
2
(1, 0)
1
p) (2, 0) No corta el eje Y q) 0, (1, 0) r) (0, 6) (9, 0)
2

9. a) pasa por el origen b) paralela al eje Y c) corta a ambos en puntos distintos


d) corta a ambos ejes en puntos distintos e) paralela al eje Y
f) paralela al eje X g) paralela al eje X h) pasa por el origen
i) corta a ambos ejes j) pasa por el origen k) pasa por el origen.

l) Si k = 1 pasa por el origen. Si k 1 corta ambos ejes en puntos distintos


m) corta a ambos ejes en puntos distintos n) corta a ambos ejes en puntos distintos
o) corta a ambos ejes en puntos distintos p) paralela al eje Y
q) corta a ambos ejes r) pasa por el origen
1
10. a) 1 b) c) 0 (recta eje X) d) Indeterminada (recta eje Y) e) 1
2
2 2 5
f) 1 g) 1 h) 4, 5 i) 15
j) 5
k) 1 l)
3

Relaciones y funciones 173

172-173.(2003) 173 20/11/02, 11:00 AM


Soluciones

1
11. a) m = 3 k=2 b) m = k=1 c) m = 1 k=3
5
1 2
d) m = 3 k = 5 e) m = k= f) m=1 k=5
5 5
3 1 3
g) m =
2
k= 2
h) m = 5
k=0 i) m=2 k=0

j) m = Indeterminada k) m = 1 k=2 l) m = Indeterminada


k = No existe. k = No existe
2 2 3 3
m) m = 0 k=1 n) m = k= 3
o) m = k= 4
3 8

p) m = 5 k=2

12. a) x y = 0 b) 7x + 2y 17 = 0 c) x 6 = 0 d) x + y 7 = 0
e) y 9 = 0 f) 3x 5y = 0 g) x + 4y 1 = 0 h) x 4 = 0
i) y 12 = 0 j) x 7y = 0 k) y + x + 7 = 0 l) y 5x = 0

13. a) x y + 1 = 0 b) 4y 3x + 7 = 0 c) 3x + y 11 = 0 d) 4x + 3y 15 = 0
e) y 5 = 0 f) x + 2y 1 = 0 g) 5x y + 30 = 0 h) 2x y + 3 = 0

14. 3x 5y 15 = 0 15. 3x 4y = 0 16. x y 4 = 0

17. x + 3 y 2 3 4 = 0 18. 2x 5y = 0 19. x + y + 2 = 0

20. x 3 y = 0 21. 3 x y = 0 22. 3 x + y = 0

25. m = 7 7x y 19 = 0 k = 19

c) m = Indeterminada
4 2
26. a) m = 2 k=1 b) m = 3
k= d) m = 3 k = 6
3 k = No existe
e) m = 1 k = 1 f) m=0 k=1 g) m = 1 k=6 h) m = 3 k = 0

27. a) S, los tres satisfacen la ecuacin x + 2y 7 = 0 b) No son colineales


c) S, tomados de dos en dos producen la misma pendiente d) No son colineales
e) S, los tres satisfacen la ecuacin x 4 = 0 f) No son colineales
g) No son colineales h) No son colineales
28. y 5 = 0 29. y + 3 = 0 30. x + 5 = 0 31. x + 1 = 0 32. y 16 = 0
2
33. a) L1 L2 porque m1 = m2 = 3
b) L1 L2 porque m1 = m2 = 5
2 2 1
c) L1 L2 porque m1 = y m2 = 5
d) L1 L2 porque m1 = 3 y m2 =
5 3
1 1
e) L1 L2 porque m1 = m2 = 4 f) L1 L2 porque m1 = y m2 =
3 4

1 5 0,2
34. a) L1 L2 porque m1 m2 = 2 = 1 b) L1 L2 porque m1 m2 = = 1
2 2 0,5
c) L1 L2 porque m1 m2 = 3 3 = 9 1
2 4 8
d) L1 L2 porque m1 m2 = 3

6
= 18
1

174 Relaciones y funciones

174-175.(2003) 174 20/11/02, 11:04 AM


CAPITULO 3
CAPTULO

e) L1 L2 porque m1 m2 = 1 1 = 1

f) L1 L2 porque m1 m2 = 1 1 = 1

35. x + 3y + 1 = 0 36. x + y + 3 = 0 37. 3x + 2y 8 = 0 38. y x + 3 = 0


3 3 3
39. k = 24 40. k = 5
41. k = 2
42. k =
2

43. Se demuestra calculando las pendientes (m1 = m2).

44. Se demuestra calculando la pendiente de la recta que pasa por (1, 4) y (3, 7).

45. Se demuestra calculando las pendientes y verificando que su producto es 1.

46. dem 45. 47. y + 3x k = 0 48. 2x + 3y k = 0 49. y mx + 1 + 3m = 0.


50. y mx 3 = 0.

3.4.3 Tipos de funciones


Funcin inversa
Funcin inyectiva o uno a uno
Una funcin f : AQB se dice inyectiva o uno a uno si y slo si
elementos distintos en A tienen imgenes distintas en B.

((f : AQB es uno a uno


) (
Funcin epiyectiva o sobre

(a b f (a) f (b)
o
f (a) = f (b) a = b )
Una funcin f : AQB se dice epiyectiva o sobre si y slo si todo
elemento de B es imagen de algn elemento de A.
(f : AQB es sobre) (Rang f = B).
Funcin biyectiva
Una funcin f : AQB se dice biyectiva si y slo si f es uno
a uno y sobre.
Funcin inversa
Toda funcin f : AQB posee una relacin inversa f 1 : BQA.
Esta relacin inversa f 1 : BQA es funcin si y slo si f es
funcin biyectiva.
Observacin: (f o f 1) (x) = I(x)
(f 1 o f) (x) = I(x)
La composicin de una funcin con su inversa da la funcin
idntica.

Relaciones y funciones 175

174-175.(2003) 175 20/11/02, 11:04 AM


Ejercicios 1. Sean fi las funciones definidas por los siguientes diagramas.
resueltos Determinar si fi es uno a uno, sobre o biyectiva.
f1 A f2 B
a) A B b
1 5 1 5
2 6 2 6
3 7 3
7
4 8 4

c) f3 d A f4 B
A B
1 5 1 5
2 6 2 6
3 7 3 7
8 4 8

Solucin:
a) f1 no es uno a uno porque 7 es imagen de dos elementos
distintos, el 2 y el 3.
f1 no es sobre porque 6 E B no pertenece al Rang de f1.
f1 no es biyectiva porque no es uno a uno ni sobre.
b) f2 no es uno a uno ya que f2(3) = 7 y f2(4) = 7 y 3 4.
f2 es sobre porque Rang f2 = B.
f2 no es biyectiva porque no es uno a uno.
c) f3 es uno a uno porque imgenes distintas corresponden
a pre-imgenes distintas.
f3 no es sobre porque Rang f3 = {5, 6, 7} B, el 8 no es
imagen de ningn elemento de A.
f3 no es biyectiva porque no es sobre.
d) f4 es uno a uno porque imgenes distintas corresponden
a pre-imgenes distintas
f4 es sobre porque
Rang f4 ={5, 6, 7, 8} = B.
f4 es biyectiva porque es uno a uno y sobre.

2. Sea f : ZQZ la funcin definida por f(x) = 4x 2. Determinar si f


es biyectiva, si no lo es, redefinirla para que lo sea.
Solucin:
f es uno a uno: Sean a, b E Z.
f(a) = f(b) 4a 2 = 4b 2 /+2
4a = 4b /: 4
a =b
luego f es uno a uno.
f no es sobre ya que f(x) = 4x 2 4x = f(x) + 2

x = f(x) + 2
4
luego 1 E Z no es imagen de ningn entero ya que 1 sera imagen

176 Relaciones y funciones

176-177. 176 08/11/2001, 16:56


CAPTULO 3

1+2 3
de x = = Z
4 4
\ f no es biyectiva porque no es sobre.
Para que f sea biyectiva debe definirse sobre su rango y con eso
aseguramos que es sobre:
y+2
Rang f = {y E Z / E Z} = {y E Z / y = 2(2n 1), n E N}
4
f : Z Q { 2 (2n 1), n E N} definida por f(x) = 4x 2 es sobre y
como era uno a uno, ahora podemos decir que es biyectiva.

3. Sea f : RQR una funcin definida por :

{
x 1 si x < 2
f(x) = x + 2
2 si x 2
Determinar si f es biyectiva, si no lo es restringir su dominio
y/o rango para que lo sea. Graficar f(x).
Solucin:
f es uno a uno.
i) Sean a, b < 2
f(a) = f(b) a 1 = b 1 a = b
ii) Sean a, b 2
a+2 b+2
f(a) = f(b) = a=b
2 2
iii) a < 2 a 1 < 2 1 f(a) < 1

b 2 b + 2 2 + 2 f(b) 2
2 2 f(x)
a b f(a) f(b)
3
luego f es uno a uno.
2
f no es sobre porque como veamos: 1
si x < 2 f(x) < 1
1 2 4 x
si x 2 f(x) 2
Luego [1, 2) D Rang f
f no es biyectiva.
Para que la funcin sea biyectiva debemos definirla de RQR [1, 2).

4. Sea f una funcin real definida por


si x 4

{
x+1
f(x) = 2 4 3x
2 si x > 4
Determinar si f es biyectiva. Si no lo es restringir su dominio y/o
su recorrido para que lo sea. Graficar f.
Solucin:
f no es uno a uno.
i) Sean a, b 4
f(a) = f(b) a + 1 = b + 1 a = b
ii) Sean a, b > 4

Relaciones y funciones 177

176-177. 177 08/11/2001, 16:57


2 4 3a 24 3 b
Ejercicios f (a) = f (b) 2
=
2 a=b
resueltos
iii) a 4 a + 1 5 f (a) 5
b > 4 3b < 12 / + 24
24 3b < 12 /:2
24 3b
< 6 f (b) < 6
2
a b f (a) f (b)
en efecto, sea f (a) = 4 5 f (b) = 4 < 6
f (a) = a + 1 = 4 a = 3
24 3b 16
f (b) = 2 = 4 24 3b = 8 3b = 16 b = 3

hemos probado que f (a) = 4 a = 3


16
f (b) = 4 b = 3

as con f (a) = f (b) tenemos a b


luego f no es uno a uno.
f no es sobre, ya que si x 4 Q f (x) 5
si x > 4 Q f (x) < 6
f(x) Rang f = {y E R / y < 6} R
6 para que f fuera biyectiva podramos denirla de
5
( , 4] Q ( , 5] o de (4, + ) Q ( , 6)

4 8 x

5. Sea f una funcin real denida por

{
4 x
2 si x < 2
f (x) =
3 x si x 2
Determinar si f es biyectiva. Si no lo es, redenirla, de modo que lo
sea. Hallar una frmula para f 1. Gracar f y f 1.
Solucin: f es uno a uno:
i) Sean a, b < 2
4 a 4 b
f (a) = f (b) = a=b
2 2
ii) Sean a,b 2
f (a) = f (b) 3 a = 3 b a = b
iii) a < 2 a > 2 /+ 4
4a>2 /: 2
4 a 4 b
=> 1 f(a) > 1
2 2

178 Relaciones y funciones

178-179..indd 178 01-02-2006 9:24:33


CAPTULO 3

b2b2 /+ 3
3 b 1 f (b) 1
\ a b f (a) f (b)
luego f es uno a uno.
f es sobre ya que si x < 2 f (x) > 1
y si x 2 f (x) 1
As todos los elementos de R son imagen de algn elemento de R.
f es biyectiva, lo que implica que f tiene funcin inversa f 1.

Encontremos ahora una frmula para la funcin f 1.


Si x < 2 f (x) > 1
llamemos y = f (x)
4 x
y= 2 2y = 4 x x = 4 2y

x = f 1 (y) = 4 2y si y > 1
Si x 2 f (x) 1
llamemos y = f (x)
y=3xx=3y
x = f 1 (y) = 3 y si y 1
Entonces la frmula para f es

f 1 (y)=
{ 4 2y si

3 2y si
y>1

y1

que usando la variable x queda:


f
f 1

2
f(x)

{
1
4 2x si x>1
f 1 (x)= 1 2 3 x
3 x si x1
y = f (x)
Ntese que los grcos de f y f 1 son simtricos res- I(x)
y = f -1(x)
pecto de la recta x = y.

6. Sea f una funcin real biyectiva denida por:


5x+3
f (x) = 2

Hallar una frmula para f 1.


So lucin:
Sabemos que (f o f 1) (x ) = I (x).
Entonces: (f o f 1) (x ) = f (f 1 (x) = x
1
5f x + 3
2 =x

5f1 (x) + 3 = 2x
5f1 (x) = 2x 3
2x 3
f 1 (x) =
5
sta es la frmula pedida.

Relaciones y funciones 179

178-179..indd 179 01-02-2006 9:24:37


Ejercicios 7. Sea f una relacin real definida por f (x) =
x1
resueltos x+3
Hallar dominio y recorrido de f para que sea una funcin
biyectiva y encontrar una frmula que defina f 1.
Solucin:

Dom f = {x E R / x 3} Si x = 3, f (x) no existe, ya que 4


0
est indeterminado.
Para determinar el rango de f vamos a despejar x en funcin de f (x)
para identificar posibles f (x) que indeterminan la expresin:
x1
f (x) =
x+3
x f (x) + 3 f (x) = x 1
x f (x) x = 1 3 f (x)
x (1 f(x) ) = 1 + 3f (x)
1 + 3 f (x)
x =
1 f (x)
1 + 3y
si f (x)= y Q x =
1y
Es decir, el 1 no es imagen, luego: Rang f = {x E R / x 1}
as f : R - { 3} Q R - {1}
Encontremos ahora una frmula para f 1.
(f o f 1) (x) = I (x) = x
f (f 1(x)) = x
1
f (x) 1
=x
1
f (x) + 3
f 1 (x) 1 = x f 1 (x) + 3x
f 1 (x) x f 1 (x) = 3x + 1
f 1 (x) (1 x) = 1 + 3x
1 + 3x
f 1 (x) =
1x
lo que nos da la misma expresin que encontramos para hallar
el rango de f. Esto se debe a que f 1 est definida del Rang f al
Dom f, luego:
f 1 : R - {1} Q R - {3}

Ejercicios

1. Sean A = {1, 2, 3, 4} y B = {a, b, c, d}


Determine cules de las funciones defi nidas en los
siguientes diagramas son uno a uno, sobre, y/o biyectivas.
Justifi que las respuestas.

180 Relaciones y funciones

180-181. 180 08/11/2001, 15:23


CAPTULO 3

f A f B
a) A B b)
1 a
1
2 b
2 a
3 c
3
4 d

f A f B
b) A B c)
1 a 1
a
2 b 2
b
3 c 3
c
4 d 4

f
c) A B d) A f B
1 a a
2 b b 1
3 c c 2
4 d d 3

A f B
d) e) A f B
1 a a
1
2 b b
2
3 c c
3
4 d

A f B f
e) f) A B

1 a 1 2
2 b 3 4
3 c 5 6
4 d

A f B 3. Sean A = {1, 2, 3, 4} y B = {5, 6, 7}


f)
Se puede definir una funcin biyectiva
1 a de A en B? Represntelo mediante un
2 b
c
diagrama.
3
4 d 4. Sean A = {a, e, i, o, u} y B = {1, 2,
3, 4, 5} Se puede definir una funcin
2. Dadas las funciones defi nidas en biyectiva de A en B? Represntelo
los siguientes diagramas, determine mediante un diagrama.
cules son biyectivas. Justifi que las
respuestas. 5. Sea f : Z Q Z una funcin definida por
a) f
f (x) = 2x 1. Indique si f es uno a uno,
A B sobre o biyectiva.
a 6. Sea f : R Q R una funcin definida
a
b
b 5x 3
c por f (x) = . Si f es biyectiva,
c d 2
encuentre una frmula para f 1.

Relaciones y funciones 181

180-181. 181 08/11/2001, 15:23


Ejercicios A f B
7. Sea g : RQR una funcin definida
por g(x) = 4x + 3. a 1
b 2
Si g es biyectiva, encuentre una
frmula para g 1. c 3
d 4
8. Sea A = {x E Z / 5 x 2}
B = {x E Z / 2 < x < 2}
13. De las funciones definidas en los
C = {x E Z / 3 x < 5}
diagramas siguientes, determine cules
y sean las funciones: poseen funcin inversa.
f1 : A Q Z f1 (x) = x2 1
A f1 B
f2 : B Q Z f2 (x) = x2 + 2 a)
f3 : C Q Z f3 (x) = 32 x2 a 1
b 2
Determine cules de estas funciones c 3
son uno a uno (inyectivas).
d 4
9. Sea A = {x E R / 3 x 3}
A f2 B
Se dan las funciones de A en A b)
definidas por : x 1
f(x) = x2 2 y g(x) = x3 + 1.
y 2
Determine cules de las funciones z 3
dadas son uno a uno y cules son
sobre. Intente bosquejar un grfico
calculando valores. A f3 B
c)
10. Sea f : RQR definida por: a
1
b
2 si x 1 2

{
c
f(x) = 3 d
2x si x < 1
A f4 B
Grafique y determine si f es uno a d)
uno y/o sobre. x u
y v
11. Dadas las siguientes funciones, deter-
z w
mine dominio y recorrido de cada
una. En cada caso determine la
relacin inversa e indique si sta f5
es funcin. A B
e)
f1 = {(1, 3) (2, 4) (3, 5) (4, 6)} 1
2 1
f2 = {(1, 5) (2, 5) (3, 5) (4, 5) (5, 5)} 3
f3 = {(a, 1) (b, 2) (c, 3) (d, 4) (e, 5)}
A f6 B
f4 = {(b, 2) (a, 1) (r, 2) (c, 2) (o, 1)} f)
1 1
2 2
12. Dada la funcin f : A Q B definida
por el diagrama siguiente. 3 3
Es f 1 funcin?
Halle f 1(2), f 1(3) y f 1(4). 14. Sea f : ZQZp, Zp = {nmeros pares}
Haga un diagrama de f 1. definida por f(n) = 2n.

182 Relaciones y funciones

182-183. 182 08/11/2001, 17:00


CAPTULO 3

a) Determine si esta funcin tiene 16. Sea A = {x E N / 3 < x < 9} y f: AQN


funcin inversa. definida por f(x) = x2 1.
b) Si f se definiera de ZQZ, tendra a) Escriba f por extensin.
funcin inversa? b) Escriba f 1 por extensin.
15. Dadas las funciones reales definidas por c) Escriba f 1 (15) y f 1 (63).
los siguientes grficos. Determine cules
d) Es f 1 una funcin?
de ellas poseen funcin inversa.
17. Dada la funcin real f(x) = 4x 8.
Determine si f posee funcin inversa
a)
3 y encuentre una frmula que la defina.
2

1 18. Dada la funcin real definida por

{
x2
si x 3
3 2 1 1 2 3 2
1 f(x) =
2
x 2 si x > 3
3 a) Grafique y determine Rang f.
b) Demuestre que f es uno a uno.
c) Demuestre que f no es sobre.
b)
3
19. Dada la funcin real definida por:
2
2x + 5 si x 1.

3 2 1
1

2
1 2 3
f(x) =
3{ si x > 1.

a) Grafique y encuentre Rang f.


3
b) Determine si f es uno a uno.
c) Determine si f es sobre.
c) d) Encuentre, si es posible,
3
la funcin f 1.
2
20. Dada la funcin real definida por:
1

{
9 2x
3 2 1 1 2 3
si x < 3.
f(x) = 3
1

2 4 x si x 3.
3 a) Grafique.
b) Demuestre que f es biyectiva.

d) c) Encuentre la frmula que define f1.


3
21. D ada la funcin real definida por:
2

{
1 2x + 4 si x 3.
f(x) =
3 2 1 1 2 3 2x 4
1
si x > 3.
5
2

3
a) Grafique.
b) Demuestre que f es biyectiva.

Relaciones y funciones 183

182-183. 183 08/11/2001, 17:00


Ejercicios a) Determine dominio y rango para
que f sea una funcin biyectiva.
c) Encuentre la frmula que define
b) Encuentre una frmula para f 1
f 1.
22. Dada la funcin real definida por: 24. Dada la relacin f : RQR definida por

f(x) = 2 x + 1

{
x+2 si x < 1. x3
f(x) =
2x
si x 1. a) Determine dominio y recorrido de f
3
para que sea funcin.
a) Grafique y determine Rang f. b) Encuentre una frmula para f 1.
b) Demuestre que f es uno a uno.
25. Dada la relacin g : RQR defini -
c) Demuestre que f no es sobre. 1 2x
da por g(x) =
x
23. Dada la relacin f : RQR definida por
x1 a) Determine dominio y recorrido de
f(x) = g para que sea funcin.
x+2
b) Encuentre una frmula para g 1.

Soluciones
biyectiva de A en B porque para ello se
1. a) f no es uno a uno, b es imagen de necesita que ambos conjuntos tengan
1, 2 y 3. la misma cardinalidad (cantidad de
f no es sobre, a, c no son imagen. elementos).
b) f es uno a uno, cada elemento de B es 4. Es posible. Ej.:
imagen de un solo elemento de A.
f es sobre, Rang f = B, f es biyectiva. A f B
c) f no es uno a uno. a 1
f(1) = a A f(2) = a A 1 2. e 2
f no es sobre, b y d no son imagen. i 3
o 4
d) f no es uno a uno. u 5
f(3) = f(4) = d A 3 4.
f no es sobre, a no es imagen.
5. f es uno a uno.
e) f es uno a uno, f(a) f(b) a b.
f es sobre, Rang f = B. f no es sobre (4 no es imagen de ningn
f es biyectiva. elemento de Z).

f) f es uno a uno, f es sobre, f es f no es biyectiva.


biyectiva. 2x + 3
6. f es biyectiva f 1(x) =
2. a) f es uno a uno, f no es sobre. 5
b) f no es uno a uno, f es sobre. x3
7. g es biyectiva g 1(x) =
c) f no es uno a uno, f no es sobre. 4
d) f no es uno a uno, f es sobre. 8. f1 y f3
e) f es uno a uno y sobre, f es biyectiva. 9. f no es uno a uno ni sobre.
f) f es uno a uno y sobre, f es biyectiva. g es uno a uno y no es sobre.
3. No es posible definir una funcin 10. f no es uno a uno, f no es sobre.

184 Relaciones y funciones

184-185. 184 08/11/2001, 17:01


CAPITULO 3
CAPTULO

11. Dom f1 = {1, 2, 3, 4} 17. f posee inversa porque es biyectiva.


Rang f1 = {3, 4, 5, 6} f 1 (x) = x + 8
4 f(x)
f1 es biyectiva. 18. a)
f11 = {(3, 1) (4, 2) (5, 3) (6, 4)}
1
es funcin.
2 3 4 x
Dom f2 = {1, 2, 3, 4, 5}
Rang f2 = {5} f2 no es biyectiva.
1
f2 = {(5, 1) (5, 2) (5, 3) (5, 4) (5, 5)} 1
Rang f = {f(x) E R / f(x) 2 V x > 1}
no es funcin. 19. a) f(x)
3
Dom f3 = {a, b, c, d, e}
Rang f3 = {1, 2, 3, 4, 5} f3 es biyectiva.
f31 = {(1, a) (2, b) (3, c) (4, d) (5, e)}
3 1 x
1
es funcin.
Dom f4 = {b, a, r, c, o}
Rang f = {f(x) E R / f(x) 3}
Rang f4 = {1, 2}
b) f no es uno a uno.
f4 no es biyectiva.
c) f no es sobre.
f41 = {(2, b) (1, a) (2, r) (2, c) (1, 0)}
d) no hay funcin inversa porque f no
no es funcin. es biyectiva.
12. f 1 es funcin porque f es biyectiva.
20. a)
f 1(2) = a f 1(3) = b f 1(4) = c f(x)
1 3
B f A
1
1 a
2 b 3 4 x

3 c
c)

{
4 d 9 3x
si x > 1
f 1(x) = 2
13. f1, f4, f6 4-x si x 1
14. a) f tiene inversa.
21. a) f(x)
b) f no tendra inversa porque no sera
sobre. 4 2 x

15. a) tiene inversa. 2


b) tiene inversa.
c) no tiene inversa.
x4

{
d) no tiene inversa. c) si x 2
2
16. a) f = {(4, 15), (5, 24), (6, 35), (7, 48), f 1(x) =
5x + 4
(8, 63)} si x > 2
2
b) f 1= {(15, 4), (24, 5), (35, 6), f(x)
(48, 7), (63, 8)} 22. a)
2

c) f 1(15) = 4; f1(63) = 8
d) f 1 no es funcin de N Q A pero s es 2 1 2 x
funcin de {15, 24, 35, 48, 63} en A.
Rang f = {f(x) E R / f(x) 1}

Relaciones y funciones 185

184-185. 185 08/11/2001, 17:01


Soluciones

23. a) Dom f = R {2} Rang f = R {1} 25. a) Dom g = R {0} Rang g = R {2}
2x+1
b) f 1 (x) = 1x b) g 1 (x) = x +1 2
24. a) Dom f = R {3} Rang f = R {2}
3x+1
b) f 1 (x) =
x2

3.4.4 Funciones de primer grado simultneas.


Sistema de ecuaciones de primer grado
Una funcin y = f (x) de primer grado representa una recta. Cualquier
ecuacin de primer grado en dos variables ax + by + c = 0 donde b 0
se puede escribir como una funcin y = f (x). Si b = 0 estamos frente
a una ecuacin de primer grado de la forma ax + c = 0 con a 0, que
representa una recta paralela al eje Y.
Rectas secantes son aquellas representadas por ecuaciones que
tienen una solucin comn. Se cortan en un punto.
Rectas paralelas son aquellas representadas por ecuaciones que
no tienen ninguna solucin comn. No se cortan. Sus ecuaciones
constituyen un sistema inconsistente.
Rectas coincidentes son aquellas representadas por ecuaciones que
tienen todos sus puntos en comn, es decir, infinitas soluciones
comunes. Sus ecuaciones forman un sistema indeterminado.

Definicin:
Sean L1 y L2 dos rectas en el plano representadas por las ecuaciones
a1 x + b1 y + c1 = 0 y a2 x + b2 y + c2 = 0.
Las ecuaciones a1 x + b1 y + c1 = 0 y a2 x + b2 y + c2 = 0 se
llaman linealmente dependientes (L.D.) si y slo si
a1 x + b1 y + c1 = k (a2 x + b2 y + c2) con k 0, k E R
En caso contrario las ecuaciones se denominan linealmente
independientes (L.I.).

RECTAS ECUACIONES SISTEMA 2 x 2 SOLUCIONES


secantes L.I.Simultneas determinado nica
coincidentes L.D. equivalentes indeterminado infinitas
paralelas L.I. incompatibles inconsistente no tiene

Para resolver geomtricamente un sistema de ecuaciones se


grafican ambas rectas y luego se leen las coordenadas del punto de
interseccin (si tiene solucin nica).

186 Relaciones y funciones

186-187. 186 08/11/2001, 15:25


CAPTULO 3

Para resolver algebraicamente un sistema hay varios mtodos,


entre ellos: eliminacin por reduccin, eliminacin por sustitucin y
eliminacin por igualacin (ver problemas resueltos 5, 6 y 7).

Dos sistemas se llaman equivalentes si tienen la misma solucin.

Una funcin de primer grado en dos variables z = f(x, y)


representa geomtricamente un plano. As, una ecuacin de
la forma ax + by + cz + d = 0 con c 0 , puede ser escrita
en forma de funcin z = f (x , y); luego, toda ecuacin ax + by + cz +
d = 0 representa un plano.
Tres ecuaciones con tres variables forman un sistema 3 x 3
y sus soluciones sern:

iii) Solucin nica si las tres ecuaciones son L.I. y los planos
tienen un punto en comn.

iii) Infinitas soluciones si las ecuaciones que lo forman son L.D. y


los planos tienen infinitos puntos en comn.

a) los tres planos coinciden.

b) los tres planos se intersectan formando una recta.

iii) Ninguna solucin si las ecuaciones son L.I. y los tres planos son
paralelos, es decir, no tienen ningn punto en comn.

Para resolver este tipo de sistemas se elimina una incgnita


combinando adecuadamente dos ecuaciones distintas dos veces y
con ello se obtiene un sistema 2 x 2 . Conociendo el valor de dos
variables, en cualquiera de las tres ecuaciones se puede encontrar
el valor de la variable eliminada.

Hay otro mtodo, llamado mtodo de eliminacin de Gauss.

1. Determinar si las ecuaciones siguientes son L.I. o L.D. Ejercicios


a) L1 : 5x + 2y 3 = 0 resueltos
L2 : 6x + 3y 4 = 0
5 2 3
Las ecuaciones son L.I., ya que
6 3 4
Las rectas L1 y L2 se intersectan en un punto, puesto que sus
pendientes son distintas. mL1 = 5 mL2 = 2
2
b) L1 : 4x 3y + 2 = 0

L2 : 8x + 6y 5 = 0

Relaciones y funciones 187

186-187. 187 08/11/2001, 15:25


4 3 2
Ejercicios Las ecuaciones son L.I. porque 8 = 6 5
resueltos Las rectas L1 y L2 no se intersectan, ya que representan rectas
paralelas; sus pendientes son iguales.
4 8 4
mL1 = 3
mL2 = =
6 3

c) L1 : 4x + 6y 8 = 0
L2 : 6x + 9y 12= 0

Las ecuaciones son L.D., ya que


4 6 8 3
= = y 6x + 9 y 12 = (4x + 6y 8)
6 9 12 2
Las rectas L1 y L2 son coincidentes.
2. Determinar el o los valores de k en los siguientes pares de
ecuaciones para que sean linealmente independientes.

a) L1 : 5x + 2y 1 = 0
L2 : kx 2y 1 = 0
Para que las ecuaciones sean L.I.
5 2 2 1
o
k 2 2 1
Como la segunda desigualdad es verdadera, las ecuaciones
sern L. I. para cualquier valor real de k.
b) L1 : x + 2y 6 = 0

L2 : 3x 6y + k = 0

Para que las ecuaciones sean L. I.,


1 2 2 6
o
3 6 6 k
Como la primera desigualdad es falsa, debemos asegurarnos
de que la segunda sea verdadera; luego k debe ser distinto
de 18.

3. Determinar el o los valores de k para que el sistema dado sea


determinado. (Tenga solucin nica)
a) 2x ky + 1 = 0
3x + 2y 3 = 0

Para que el sistema tenga solucin nica 2 k


3 2
\k 4
3

b) 4x + ky = 2
kx + 4y = 3

Para que el sistema tenga solucin nica 4 k


k 4
k 16 k 4
2

188 Relaciones y funciones

188-189. 188 08/11/2001, 17:03


CAPTULO 3

4. Determinar si los pares de nmeros reales dados son soluciones


del sistema dado:
1
a) 3x 6y + 1 = 0 (2, 1) , (0, )
6
xy = 3

(2, 1) no es solucin porque 3 2 6 ( 1) + 1 0 no satisface la


primera ecuacin, aunque s satisface la segunda.
1
(0, ) no es solucin porque no satisface la segunda ecuacin,
6
aunque s satisface la primera.
b) 2x 4y + 1 = 0 (1, 3 ), (3, 7 ) , (9, 9)
4 4
4x + 8y = 2

(1, 3 ) es solucin porque satisface ambas ecuaciones.


4
(3, 7 ) es solucin porque satisface ambas ecuaciones.
4
(9, 9) no es solucin, ya que no satisface ninguna de las ecuaciones.

5. Resolver usando el mtodo de eliminacin por reduccin el sistema.

3x5y+4=0 (E1)
2x+6y1=0 (E2)

Este mtodo consiste en multiplicar ambas ecuaciones por factores


tales que una de las incgnitas quede con coeficientes iguales y de
signo contrario, y luego sumar miembro a miembro.

Multipliquemos: E1 ( 2) y E2 (3)

6 x + 10 y 8 = 0
6 x + 18 y 3 = 0

Sumamos miembro a miembro 28 y 11 = 0


11
y =
Multipliquemos ahora E 6 y E 5 28
1 2

18 x 30 y + 24 = 0
10 x + 30 y 5 = 0

Sumamos miembro a miembro 28 x + 19 = 0


19
x=
28
Luego la solucin del sistema es:
19 11
,
28 28

Relaciones y funciones 189

188-189. 189 08/11/2001, 17:03


Ejercicios Comprobamos si el par
19 11
, satisface ambas ecuaciones:
resueltos 28 28
E1: 3x5y+4 = 0 E2 : 2x+6y1 = 0
19 11 19 11
3 5 +4 = 0 2 +6 1 = 0
28 28 28 28
57 38 66
55
28
+4 = 0 + 1 = 0
28 28 28
57 55 112 28
28
+ = 0 1 = 0
28 28 28
112 112
+ = 0 11 = 0
28 28
0 0 0 0

6. Resolver usando el mtodo de eliminacin por sustitucin el


sistema:
5x 3 y + 2 = 0 (E1)
x 4 y + 5 = 0 (E2)

Este mtodo consiste en despejar de una de las ecuaciones


una de las incgnitas en funcin de la otra y sustituir su valor
en la otra ecuacin, obteniendo as una ecuacin con una
sola incgnita.
Despejemos en E2 el valor de x e introduzcamos su valor en E1.

x 4 y + 5 = 0 x = 4 y 5 (E3)

Sustituyendo x en E1 5 (4 y 5) 3 y + 2 = 0

resolviendo: 20 y 25 3 y + 2 = 0
17 y = 23
23
y = 17
Reemplazando y en E3 obtenemos el valor de x:
23
x=4 5
17
92 85
x=
17 17
7
x=
17
7 23
Luego, la solucin del sistema es ,
17 17
7 23
Comprobemos si el par , satisface ambas ecuaciones:
17 17
E1: 5x3y +2 = 0 E 2: x4y+5 = 0
7 23 7 23
5 3 +2 = 0 4 +5 = 0
17 17 17 17
35 69 34 7 92 85
+ = 0 + = 0
17 17 17 17 17 17
34 34 85 85
+ = 0 + = 0
17 17 17 17
00 00

190 Relaciones y funciones

190-191. 190 08/11/2001, 15:29


CAPTULO 3

7. Resolver por el mtodo de eliminacin por igualacin el sistema:


x + 2 y 1 = 0 (E1)
5x 3 y + 4 = 0 (E2)

Este mtodo consiste en despejar la misma incgnita de las dos


ecuaciones e igualar los valores as obtenidos, consiguiendo con
ello una ecuacin con una sola incgnita.
Despejemos x en E1 y E2 obteniendo las ecuaciones E3 y E4.

E1 Q x=2y+1 (E3)
3y4
E2 Q x = (E4)
5
3y4
Igualando E3 y E4 2y+1 = / 5
5
10 y + 5 = 3 y 4
13 y = 9
9
y =
13
9
Reemplazando en E3 (o E4) x = 2 +1
13
18 13
x= +
13 13
5
x =
13
5 9
Luego la solucin es ,
13 13
5 9
Comprobemos que el par , satisface ambas ecuaciones:
13 13

E1 : x+2 y1 = 0 E2: 5x 3y + 4 = 0
5 9 5 9
+2 1 = 0 5 3 +4 = 0
13 13 13 13
5 18 13 25 27 52
+ = 0 + = 0
13 13 13 13 13 13
13 13 52 52
= 0 + = 0
13 13 13 13
00 00

8. Resolver el siguiente sistema:


4x : (y 2) = 3 : 4 (E1)
(x + 2) : (y + 3) = 4 : 8 (E2)
Transformando cada una de las ecuaciones formamos un sistema
equivalente:
4x 3 x+2 4
E1 : = E2: =
y2 4 y+3 8
16x = 3y 6 8x + 16 = 4y + 12
16x 3y + 6 = 0 8x 4y + 4 = 0 /: 4
2x - y + 1 = 0 (*)

Relaciones y funciones 191

190-191. 191 08/11/2001, 15:31


Ejercicios 16x 3y + 6 = 0
resueltos 2x y + 1= 0 / ( 3)

16x 3y + 6 = 0
6x + 3y 3 = 0 sumando

10 x + 3 = 0
3
x=
10
Reemplazando x en (*)

6
y+1=0
10
6
y= 1
10
4 2
y= =
10 5
3 2
Luego, la solucin es ,
10 5
Verifiqumoslo:

E1: 4x : (y 2) = 3 : 4 E2: (x + 2) : (y + 3) = 4 : 8
3 2 3 2
4 : 2 = 3 :4 +2 : +3 = 4 :8
10 5 10 5
12 8 17 17
10
: 5
= 3 :4 : = 4 :8
10 5

= 3 :4 =4 :8
3 3 1 4
=
4 4 2 8
9. Resolver el siguiente sistema: 1 1

2 2
ax (1 + b) y + ab = 0 (E1)
(1 a) x + by ab = 0 (E2)

Despejemos y en ambas ecuaciones:

E1 : (1 + b) y = ax + ab
a x + ab
y=
1+b (E3)
E2 : by = ab (1 a) x
ab 1 a x
y= (E4)
b
Igualando E3 con E4:
ax + ab a b 1 a x
=
1+b b

192 Relaciones y funciones

192-193. 192 08/11/2001, 15:42


CAPTULO 3

Multiplicando cruzado
abx + ab2= ab x + ax + ab2 bx + abx
x (1 a + b) = ab
ab
x=
1 a + b
En (E3)
ab
a + ab
1a+b
y=
1+b
2 2 2
a b + ab a b + ab ab 1 + b
y= 2
=
1 a + b + b ab + b 1+ba 1+b +b 1+b
ab
y=
1a+b
ab ab
As la solucin es: ,
1a+b 1a+b
Verifiqumoslo:
E1: a x (1 + b) y + ab = 0
ab ab
a 1+b + ab = 0
1a+b 1a+b
2 2 2 2
a b ab + ab ab a b + ab
+ =0
1a+b 1a+b 1a+b
0 =0
E2 : (1 a) x + by ab = 0
ab ab
1a +b ab = 0
1a+b 1a+b
2 2 2 2
ab a b ab ab a b + ab
+ =0
1a+b 1a+b 1a+b
0 0
10. Resolver el siguiente sistema:
x + 3y 3z = 16 (E1)
3x + 2y 2z = 4 (E2)
12x 25y 2z = 2 (E3)

Con E1 y E2 vamos a eliminar x


3E1: 3x + 9y 9z = 48
E2: 3x + 2y 2z = 4 sumando
11y 11z = 44 : 11
y z = 4 (E4)
Con E1 y E3 vamos a eliminar x
12 E1:: 12x 36y + 36z = 192
E3: 12x 25y 2z = 2 sumando
61y + 34z = 190 (E3)

Relaciones y funciones 193

192-193. 193 08/11/2001, 15:43


Combinamos E4 con E5 para calcular el valor de y A z.
Ejercicios
yz= 4 (E4)
resueltos
61y + 34z = 190 (E5)

34E4 : 34y 34z = 136


61y + 34z = 190

27y = 54
y=2

En E4 si y=2 ,2z=4
z= 2
En E1 si y = 2, z = 2 , x + 3 ( 2) 3 2 = 16
x = 16 + 6 + 6
x =4
Luego la solucin es ( 4, 2, 2)
Verifiquemos:

E1: x + 3y 3z = 16
4 + 3 ( 2) 3 2 = 16
16 16
E2: 3x + 2y 2z = 4
3 ( 4) + 2 ( 2) 2 2 =4
12 4 4 = 4
44
E3: 12x 25y 2z = 2
12 ( 4) 25 ( 2) 2 2 = 2
48 + 50 4 = 2
2 4= 2
22
11. Resolver el mismo sistema del problema 10 pero usando el
mtodo de eliminacin de Gauss.
x + 3y 3z = 16 E1
3x + 2y 2z = 4 E2
12x 25y 2z = 2 E3

Llamaremos operaciones por fila a los siguientes cambios que


podemos hacer con las ecuaciones sin alterar la solucin
del sistema.
Intercambiar dos ecuaciones.
Multiplicar una ecuacin por un factor distinto de cero.
Cambiar una ecuacin por ella misma sumada con otra
que previamente ha sido multiplicada por algn factor
distinto de cero.

194 Relaciones y funciones

194-195.(2003) 194 20/11/02, 11:07 AM


CAPTULO 3

El mtodo se trata de ir ordenando el sistema de modo que los


coeficientes bajo la primera incgnita de la primera ecuacin sean
cero, luego los coeficientes bajo la primera incgnita de la segunda
ecuacin sean cero y as sucesivamente.
Este mtodo es vlido para resolver sistemas de n ecuaciones
de primer grado con n incgnitas y es de fcil implementacin
computacional.

Los explicaremos aplicndolo.

E1 la dejamos igual, ya que el coeficiente de x es 1.


E2 Q E2 + 3 E1 x + 3 y 3 z = 16
E3 Q E3 + ( 12) E1 11 y 11 z = 44
61 y + 34 z = 190

E1 Q E1 x + 3 y 3 z = 16
1
E2 Q 11 E2 yz = 4
E3 Q E3 61 y + 34 z = 190

E1 Q E1 x + 3 y 3 z = 16
E2 Q E2 y z = 14
E3 Q E3 + (61) E2 27 z = 54

En E3: z = 2
En E2: y 2 = 4 y = 2
En E1: x + 3 ( 2) 3 2 = 16 x = 4

Luego, la solucin es ( 4, 2, 2).

12. Resolver el sistema


x+2yz = 3 (E1)
x y + 2z = 0 (E2)
2 x 3 y + 5z = 7 (E3)

Aplicando el mtodo de eliminacin de Gauss.

E1 Q E 1 x+2y z = 3
E2 Q E2 + ( 1) E1 3 y + 3 z = 3
E3 Q E3 + ( 2) E1 7 y + 7z = 1

Relaciones y funciones 195

194-195.(2003) 195 20/11/02, 11:07 AM


E1 Q E1
Ejercicios
x + 2y z = 3
resueltos E2 Q
1
E2
3 yz = 1
1 1
E3 Q E3 yz =
7 7

E1 Q E1 x + 2y z = 3
E2 Q E2 yz = 1
8
E3 Q E3 + (1) E2 0 =
7

Vemos que en la ltima ecuacin se nos produce una


contradiccin, lo que quiere decir que el sistema no tiene
solucin. No hay ningn punto comn a los tres planos.

13. Resolver el sistema:

x + 2y z = 3 (E1)
x y + 2z = 0 (E2)
2x 3y + 5z = 1 (E3)

Aplicando el mtodo de eliminacin de Gauss.


E1 Q E1 x + 2y z = 3
E2 Q E2 + ( 1) E1 3y + 3z = 3
E3 Q E3 + ( 2) E1 7y + 7z = 7

E1 Q E1
x + 2y z = 3
E2 Q 1 E2
3 yz= 1
1
E3 Q E3 yz= 1
7

E1 Q E1 x + 2y z = 3
E2 Q E2 yz=1
E3 Q E3 + (1) E2 0=0

196 Relaciones y funciones

196-197. 196 08/11/2001, 17:05


CAPTULO 3

Ya veamos en el paso anterior que haba dos ecuaciones equivalentes,


lo que significa que el sistema tiene una variable libre y por lo tanto
tiene infinitas soluciones.
Si z = 1 (variable libre), en E2 : y = 2 en E1 : x = 0. Luego (0, 2,
1) es una solucin. Dando otros valores a z se pueden encontrar
otras soluciones.
Si z = a, y = 1 + a x = 1 a
As la solucin general es (1 a, 1 + a, a)

14. Resolver el sistema:

2x y + z 3u = 7 (E1)
x + 2y z + u = 8 (E2)
3x + y 2z u = 6 (E3)
x + y 4z 2u = 3 (E4)

Reordenando
x + 2y z + u = 8
x+y4z2u = 3
2xy+z3u = 7
3x+y2zu = 6
E2 Q E2 + E1
E3 Q E3 + ( 2) E1
E4 Q E4 + ( 3) E1 x + 2y z + u = 8
3y 5z u = 5
5y + 3z 5 u = 23
5y + z 4 u = 18

1
E2 Q E2 x + 2y z + u = 8
3
5 1 5
y 3
z 3
u =
3
5y + 3z 5u = 23
5y + z 4u = 18

E3 Q E3 + 5 E2

E4 Q E4 + 5 E2 x + 2y z + u = 8
5 1 5
y z u = 3
3 3
16 20 44
z u =
3 3 3
22 17 29
z u =
3 3 3

Relaciones y funciones 197

196-197. 197 08/11/2001, 17:06


Ejercicios E3 Q 3 E3 x + 2y z + u = 8
resueltos 2
1
E4 Q 3 E4 y5 z3 u = 5
3 3
8z + 10u = 22

22z + 17u = 29

E3 Q 1 E3 x + 2y z + u = 8
8
5 1 5
y3 z3 u = 3
z+5 u = 11
4 4
22z + 17u = 29

E4 Q E4 + ( 22) E3 x + 2y z + u = 8
y5 z1 u = 5
3 3 3
z+5 u = 11
4 4
21 u = 63
2 2

E4 : u = 3

E3 : z = 11 5 u z = 1
4 4
E2 : y = 5 + 5 z + 1 u y = 1
3 3 3
E1 : x = 8 2y + z u x = 2
Luego la solucin es (2, 1; 1, 3)
15. Determinar el valor de k para que el sistema tenga:
i) solucin nica
ii) ninguna solucin
iii) infinitas soluciones
x + 2y z = 6
4x y + z = 9
x ky + 2z = 5

E2 Q E2 + ( 4) E1
E3 Q E3 + ( 1) E1 x + 2y z = 6
9y + 5z = 33
(k + 2) y+ 3z = 11

E2 Q 1 E2 x + 2y z = 6
9
y 5 z = 33
9 9
(k + 2) y + 3z = 11

198 Relaciones y funciones

198-199. 198 08/11/2001, 15:51


CAPTULO 3

E3 Q E3 + (k +2) E2 x+2yz = 6
5 33
y z =
9 9
(17 5 k) z = 33 33k

396
Si k = 17 E3: 0 = y el sistema no tiene solucin.
5 5
Si k 17 el sistema tiene solucin nica.
5
No existe k para el cual el sistema tenga infinitas soluciones, ya que
175k y 3333k no pueden ser ambas cero.

Ejercicios
1. Determine si los siguientes pares de j) 3y 3 = 0
ecuaciones son L. I. o L. D. 3y + 3 = 0

a) 2x + y + 3 = 0
2. Determine el valor de k para que las
5x + y 9 = 0 ecuaciones dadas sean L. I.

b) 4x + y 7 = 0 a) 2x + 3y k = 0 x+y1 = 0
6x + 5y + 7 = 0 b) x + y 3=0 kx = 3 y
c) 1 2x = y x + 2y = k
c) x + y 12 = 0
5x + 3y + 20 = 0 d) x + ky 2 = 0 xy = 2

d) 2x + y + 1 = 0 3. Determine el valor de k en el sistema


2x y + 3 = 0 para que tenga solucin nica.

e) 2x + 3y 1 = 0 a) x + ky = 2

4 x + 6y 2 = 0 2x + y = 4

f) 2x + 3y 1 = 0 b) 8x + 4y 6 = 0
2x y = 0 12x + ky = 9

g) 2x + 3y 1 = 0
c) kx y = 2
4x + 6y + 7 = 0
x ky = 3
h) 4x + 4y 8 = 0
4. En los casos del ejercicio N 1 en que
6x + y 22 = 0
las ecuaciones sean L. I., determine
si forman un sistema de ecuaciones
i) 2y 3 = 0
simultneas o incompatibles (determi-
y1 = 0 nado o inconsistente, respectivamente).

Relaciones y funciones 199

198-199. 199 08/11/2001, 15:52


Ejercicios b) x+y4 =0
5. Determine, sin resolver, si los siguientes 4x + y 25 = 0
sistemas son determinados, indetermi- (1, 5) ( 7, 3) (1, 29)
nados o inconsistentes.
c) 2x + 7y 1 = 0
a) 2x + 2y + 5 = 0
2x + 7y 13 = 0
x+y+7 = 0
( 24, 5), ( 3, 1) ( 38, 9)
b) 5x + y 17 = 0
4x 8 = 0 d) 3x 5y 25 = 0
6x + 10y + 50 = 0
c) 3x + 3y 1 = 0
x+y+2 = 0 ( 30, 23), (20, 7) (15, 9)

e) 4x + y + 23 = 0
d) 3x + y + 3 = 0 5x + 7y + 23 = 0
9x + 3y + 9 = 0
( 13, 6), ( 6, 1), (8, 9)
e) 5x 3y = 0
f) x + 6y + 32 = 0
25x 15y = 0
3x + 6y + 36 = 0

f) 2x + y 3 = 0 (2, 5), (50, 19), (26, 7)


5x + 4y 15 = 0
7. Determine grficamente la solucin de
g) 4x + y 10 = 0 cada uno de los siguientes sistemas:
5x + y 11 = 0 a) 5x = 4y 36
9x = y 40
h) 7x 5y 4 = 0
14x + 10y + 8 = 0 b) 8x + 11y 4 = 0
3x 2y + 23 = 0
i) x2 y = 0
2x + 4y + 1 = 0 c) x + 12y 31 = 0
4x = 3y 11
j) y+2 = 0
5x + 4y 7 = 0 d) x=y2
4x y 17 = 0

6. Determine si los pares de nmeros reales


e) 2x = 5y + 8
dados son solucin del sistema dado.
4x = 8y + 8
a) 3x + y 4 = 0
6x + 2 y 8 = 0
f) 4x + 4y + 12 = 0
(5, 11) (1, 1) (2, 2) x + 2y 1 = 0

200 Relaciones y funciones

200-201. 200 08/11/2001, 17:07


CAPTULO 3

8. Resuelva usando el mtodo de eliminacin


g) 9x 8y + 5 = 0 por reduccin los siguientes sistemas
6x 7y 5 = 0 de ecuaciones:
a) x + 5y + 21 = 0
h) 11y + 11 = 0
2x + 3y 3 = 0
4x + 6y 10 = 0

b) 5x + 3y + 17 = 0
i) 2x 8y 16 = 0
4x + y + 15 = 0
x = 7y + 17

j) 5x = 7y + 35 c) 3x + 5y + 9 = 0
8x 2y 10 = 0 4x + 2y 2 = 0

k) 3x + 8y 6 = 0 d) x + 3y 13 = 0
2x = 6y + 6
4x + 2y 22 = 0

l) 5x = y 21
e) x + 2y 1 = 0
7x 2y + 43 = 0
2x + 5y 25 = 0
m) x = 14y + 21
4x = 12y 4 f) 3x + 6y 39 = 0
6x + 6y 12 = 0
n) 4x = 3y 8
3x = 7 y + 31
g) 3x + 8y 6 = 0
2x + 2y 18 = 0
o) 2y + 6 = 0
7x + 2y 41 = 0
h) 3x + 3y + 30 = 0
p) 4x = 6y + 2 5x + y + 34 = 0
7x = 11y + 2
i) 2x + 5y + 19 = 0
q) 7x + 12y + 32 = 0
5x + 2y 5 = 0
x + 13y 18 = 0

j) x + 5y + 6 = 0
r) x = 8y + 12
5x + 4y 1 = 0
7x + 5y + 18 = 0

k) 4x + 4y 24 = 0
s) 0=y+4
6x 11y 26 = 0 5x + 3y 24 = 0

t) x + 6y 9 = 0 l) 4x + 7y 26 = 0
x = 7y + 17 2x + 5y 16 = 0

Relaciones y funciones 201

200-201. 201 08/11/2001, 17:07


Ejercicios
9. Resuelva con el mtodo de elimina- 10. Resuelva por el mtodo de elimina-
cin por sustitucin cada uno de los cin por igualacin cada uno de los
siguientes sistemas de ecuaciones: siguientes sistemas de ecuaciones:
a) 4x + 3y 3 = 0 a) x + 2y + 1 = 0
2x + 7y 7 = 0 x + 2y + 15 = 0

b) x + 4y + 4 = 0 b) x + 5y 6 = 0
4x + 6y + 28 = 0 5x + 6y 11 = 0

c) 4x + y 30 = 0 c) x + 8y 39 = 0
2x y 12 = 0 5x + y = 0

d) x + y 10 = 0 d) 2x + 2y 6 = 0
x + 2y 8 = 0 6x + 3y 18 = 0

e) 2x + 5y + 37 = 0 e) x+y3=0
x + 2y + 16 = 0 4x y 6 = 0

f) x + 2y + 3 = 0 f) 2x y 3 = 0
4x + 5y + 14 = 0 6x + 5y + 15 = 0

g) 2x + 4y + 12 = 0 g) x + 2y = 0
3x + 8y + 20 = 0 4x + 7y = 0

h) x + 2y + 1 = 0 h) x + 5y 3 = 0
6x + 2y 48 = 0 2x y + 1 = 0

i) 3x + 5y 22 = 0 i) x + 2y + 3 = 0
5x + 8y 36 = 0 x + y 3 = 0

j) x + 2y + 1 = 0 j) 4x + 7y + 15 = 0
4x + y + 4 = 0 3x + 8y + 25 = 0

k) x + 3y 7 = 0 k) x + 3y 15 = 0
3x + y 5 = 0 2x + y 15 = 0

l) x + 6y + 2 = 0 l) 6x + 4y 28 = 0
2x + 3y 5 = 0 x + 2y 6 = 0

202 Relaciones y funciones

202-203. 202 08/11/2001, 17:07


CAPTULO 3

11. Resuelva los siguientes sistemas de m) 3x + 4y 38 = 0


ecuaciones usando el mtodo que x+ y+1=0
estime ms conveniente:

a) 3x y 12 = 0
12. Resuelva los siguientes sistemas de
x + 4y 5 = 0 ecuaciones:
a) 4x + 5y + 9 = 0
b) 5x + 7y + 8 = 0 20x + 6y 12 = 0
6x + 2y + 32 = 0
b) 20y 5 = 0
c) x + 2y 1 = 0 4x + 12y 15 = 0
3x + 5y = 0
c) 32y + 8 = 0
d) x + 4y + 2 = 0 10x + 4y 9 = 0
2x + 6y + 4 = 0
d) 20x + 6y 8 = 0
8x + 21y + 3 = 0
e) x + 2y + 12 = 0
x + 3y + 13 = 0 e) 12x + 20y 7 = 0
4x + 8y + 6 = 0
f) y6=0
x + 6y 36 = 0 f) 12x + 10y + 2 = 0
3x + 4y + 2 = 0

g) x + 5y 3 = 0
g) 4x + 5y + 11 = 0
3x + 4y + 9 = 0
4x + 3y + 13 = 0

h) 7y + 7 = 0 h) x + 4y + 1 = 0
6x + 5y 25 = 0 x + 8y 10 = 0

i) x + 3y 16 = 0 i) 3x + 9y 2 = 0
6x + 5y 42 = 0 15x + 21y + 6 = 0

j) 2y + 2 = 0 j) 8x + 2y 16 = 0
4x + 2y + 2 = 0 10x + 3y 22 = 0

k) y =0 k) 3x + 5y + 14 = 0
5x + y 2 = 0 4x + 2y + 14 = 0

l) 4x + 6y 4 = 0 l) 4x + 3y 6 = 0
5x + 4y + 28 = 0 16x + 21y 33 = 0

Relaciones y funciones 203

202-203. 203 08/11/2001, 17:07


Ejercicios
7x + 8y 7x 7y
13. Resuelva los siguientes sistemas: h) =
6 6
3y 1 = 0
a) px + y = 1
x qy = 0 i) 5x 4 (7y + 5) = 2
7x 4 ( y 2) = 3
b) px y = 0
pqx 1 = 0 j) 6 ( 3x 7) 5 (4y + 8) = 9
( 6x + 2y) : ( 9x + 2y) = 5 : 2
c) ax b = 0
ax by = 1
k) 6 (5 x) 8 (4y + 2) = 2
(8x + 6) + 2 (5y 4) = 3
d) ax + by = 1
bx + ay = 0
6x 5y 6x + 4y
l) =
2 3
6x + 8 (3 8y) = 0
14. Resuelva los siguientes sistemas de
ecuaciones:
8y x 6y 15. Un cuarto de la suma de dos nmeros
a) 6x + 4 = +2 es 152 y un tercio de su diferencia es
6 6
x 5 ( 3y 4) = 6 66. Cules son los nmeros?

16. Si el doble de un nmero se suma


7y x + 8y con el triple de otro se obtiene 45.
b) 6x + 4= +8 Si al triple del primero se le resta el
4 4
( 4x 2) 7 ( 5y 7) = 4 segundo se obtiene 29. Determine
los nmeros.
c) 7x = 6 17. La suma de las cifras de un nmero
4 ( 5x + 1) 2 (y 8) = 2 de dos cifras es 7. Si se invierten las
cifras, el nuevo nmero es igual a dos
veces el nmero anterior, ms dos
d) 8x : ( 7) = 3y : ( 1) unidades. Calcule el nmero.
4 ( 8x 4) 8 (2 y) = 7
18. Si un nmero de dos cifras se divide
por la suma de sus cifras, el cociente
x + 7y 8 es 7 y el resto es cero. Si a la cifra
e) 5x 4y
=
7
de las decenas se resta la cifra de
8 ( 8x 7) + 3 (6 9y) = 5 las unidades, se obtiene 4. Cul es
el nmero?
f) 5x + 7 (6y 8) = 9 19. Dos nmeros estn en la razn 5 : 5.
( x 4y) : (7x 3y) =3 :6 Si el primero se aumenta en 12 y el
segundo se disminuye en 3, quedan
en la razn de 9 : 4. Cules son
4y 5x + 2y los nmeros?
g) 6x +3 = +3
8 8
8x = 6y 3 20. La suma de dos nmeros es 208 y

204 Relaciones y funciones

204-205. 204 08/11/2001, 17:08


CAPTULO 3

su diferencia es 122. Cules son los la edad de Marcos. Cul es la edad


nmeros? actual de cada uno?

21. Sergio tiene $ 1.950 en monedas de 29. El doble de la edad de ngela sobre-
$ 100 y de $ 50. En total tiene 24 pasa en 14 aos la edad de Sergio. Y
monedas. Determine cuntas son de un quinto de la edad de Sergio es 13
$ 100 y cuntas de $ 50. aos menos que la edad de ngela.
Calcule ambas edades.
22. La suma de las cifras de un nmero de
2 cifras es 4. Si se invirtieran las cifras, 30. Dos nmeros estn en la razn 10 : 5. Si
el nuevo nmero sera igual al doble se resta 20 al primero y se suma 20 al
del nmero anterior, ms 5 unidades. segundo, la razn de ellos se invierte.
Cul es el nmero? Cules son los nmeros?

23. Si un nmero de dos cifras se divide 31. La suma de dos nmeros es 169 y
por la suma de sus dgitos, el cociente su diferencia es 132. Cules son los
es cinco y el resto es cero. Si a la nmeros?
cifra de las decenas se le resta la cifra
de las unidades se obtiene 1. Cul 4
32. de la suma de dos nmeros es
es el nmero? 5 10
igual a 32 y de su diferencia es 10.
9
1 Encuentre los nmeros.
24. La edad de Eliana es de la edad
5
de Miguel y hace 5 aos, la edad de 33. Andrs le pag a Carlos $ 1.550 en
1 monedas de $ 100 y de $ 50. En total
Eliana era de la edad de Miguel.
10 le dio 21 monedas. Cuntas eran de
Determine sus edades actuales. $ 100 y cuntas de $ 50?

25. Un cuarto de la suma de dos nmeros 34. Si un nmero de dos cifras se divide
es 81 y un tercio de su diferencia es por la suma de sus cifras el cociente
54. Cules son los nmeros? es cinco y el resto es trece. Si a la
cifra de las decenas se resta la cifra
26. Si el doble de un entero se suma con el de las unidades se obtiene 1. Cul
triple de otro se obtiene 26. Si al triple del es el nmero?
primero se le resta el segundo se obtiene
28. Determine los nmeros. 35. La edad de Adolfo es 15 aos menos
que el doble de la edad de Teresa. Y la
27. Si una de dos llaves de agua queda sptima parte de la edad de Adolfo es
abierta durante 40 min. y la otra 20 aos menos que la edad de Teresa.
durante 17 min., entregan ambas 1.163 Calcule ambas edades.
litros de agua. Si la primera queda
abierta durante 21 min. y la otra 36. Dos nmeros estn en la razn de
durante 41 min., entregan 1.220 litros 6:4. Si se resta 6 del primero y se
de agua. Cuntos litros de agua por suma 6 al segundo, quedan en la
minuto entrega cada llave? razn 2 : 3.
Cules son los nmeros?
28. Hace 5 aos la edad de Manuel era 9
veces la edad de Marcos. En 5 aos 37. La suma de la cifra de las decenas y la
ms la edad de Manuel ser 4 veces cifra de las unidades de un nmero de

Relaciones y funciones 205

204-205. 205 08/11/2001, 17:08


Ejercicios
i) 8z + 32 = 0
dos cifras es 4. Si al nmero se le resta 18, las x 52y + 30z + 17 = 0
cifras se invierten. Cul es el nmero? 2x + 13y + 7z + 52 = 0

38. Hace 4 aos la edad de Ximena era


j) 21x 14y 13z 73 = 0
8 veces la edad de Matas. En cuatro
14x 21y 6z 115 = 0
aos ms la edad de Ximena ser 4
4x 4y 12 = 0
veces la de Matas. Cul es la edad
de cada uno?
k) 10x + 18y 40z 84 = 0
39. Resuelva los siguientes sistemas: 3x + 30y = 22z 92
5x + 6y = 17z 66
a) 5x 3y 5 = 0
8x + 8y = 4z
2x y + 2z + 2 = 0 l) 4x + 20y = 20z 12
3x + 9z 27 = 0
7x + 26y 2z + 77 = 0
b) 9x + 14y + 25z + 22 = 0
6x + 2y = 5z 19
m) 3y z + 5 = 0
7x + 4y 3z + 26 = 0
16x 20y + 12z + 52 = 0
13x 46y 14z 142 = 0
c) 10x 4y + 12z 28 = 0
22x + 31y = z 29
n) x 2y z = 0
4x + 3y 6z + 15 = 0
19x 29y 12z + 35 = 0
18x + 24y + 15z 15 = 0
d) 2x + 2y + 11z + 23 = 0
y+z =0
) 7x + 5y + 10z + 21 = 0
3x + 4y + 7z + 15 = 0
4x + 6y + 20z + 82 = 0
4x 5y 24z 83 = 0
e) 5x + 25y + 26z + 49 = 0
18x + 20y 51z + 1 = 0
o) 2x + z 3u = 8
15x 9y + 7z + 50 = 0 x 2y + z + u = 4
x + y 3u = 4
f) 3x + 9y = 9z 48 y 4z + u = 2
6x + 4y 4z 8 = 0
12x 25y 2z + 2 = 0 p) x 2y + 5z u =6
x+yz+u =6
g) xy4 =0 x + 2y z u =2
3x + 3y = 4z 4 2x + y + u =7
14x y + 6z 31 = 0
q) 5x 3y + z 2u = 11
h) 15x 8y 11z 9 = 0 x 5y + 3z u = 10
5x 3y + 27z + 29 = 0 2x + y z + 3u = 3
x + y + 5z + 3 = 0 3x 2y + 2z + u = 6

206 Relaciones y funciones

206-207. 206 08/11/2001, 17:09


CAPTULO 3

3 1 2
r) x + 2y z + u = 3 g) x + y z
=2
2x y + 2z u = 8 2 3
x + z =3
x + 3y 2z + u = 7
3x 4y + 3z 2u = 13 1 1 2
x y z =1

40. Resuelva los siguientes sistemas:


2 4
h) x y = 4
a) 1 1 2
x + y + z
=2
3 4 4
x y z
= 1
3 3 3
x y z =3
3 32
2 1 3 x + y z = 4
x y z =3

1 2 4
1 2 =1 i) x y z = 2
b)
x + z
4 3
1 3 3 x +z = 2
x + y +z =0
4 2 3
3 3 x y+z = 1
y + z =2

2 1
j) y z = 1
c) 3 1 + 3 = 4
x y z
1 3
2 2 x z = 0
x +z =1
3 2 2
2 1 3
=2 x y z = 4
x + y z

3 3
d) 1 1 2
=2 k) x y = 0
x +yz
2 3 3 1 2 = 1
x +y =1 x y +z
4 4 1 3 3
x z =1 x y z
= 2

3 2 2
e) 2 1
x y z
=1 l) y + z
= 2
3 2 4 1 1 2
=2
x y z x + y z = 4
1 2 4 2 2 1
x y z =3 x + y z = 3

1 4 3 3 1 2 = 4
f) x y z = 1 m)
x y z
2 4 4 1 3 1
x y z =0 x y + z = 2
1 3 2 2 1 1
x + y z =1 x + y z = 3

Relaciones y funciones 207

206-207. 207 08/11/2001, 17:11


Ejercicios 44. Resuelva el sistema.
3 1 3
n) x y z =3 a) x y + 5z = 3
2 3 1 3x + 3y 15z = 8
x y + z = 2
2x y + z = 2
4 1
x z =1
1
b) 2x y z =2
2
3 4 x + y 2z = 3
o) y z =1 1 1
x y z =5
3 1 2 4
y + z = 0
2 1 4
x y z =4 45. La suma de las tres cifras de un nmero
es 16. Al sumarle 9 se intercambia la cifra
41. Encuentre los valores de k en el sistema de las unidades con la de las decenas
siguiente para que ste tenga: y al sumarle 99 se intercambia la cifra
de las unidades con la de las centenas.
iii) Solucin nica. Encuentre este nmero.
iii) Infinitas soluciones.
iii) Ninguna solucin. 46. Andrs, Arturo y Carlos juntan $ 50.000.
Si Andrs tiene la mitad del dinero de
a) x + 2y z = 1 Arturo y tres veces el de Carlos, determine
x+y+z =0 el dinero que posee cada uno.
x + ky 2z = 2
47. Encuentre la medida de los ngulos
interiores de un tringulo si el doble
b) x 2y + z = 3 del primero menos el segundo es 81 y
el triple del segundo, menos el tercero
kx y + 2z = 3
es 9.
2x y 2z = 1
48. Entre Matas y Javier renen $ 9.000
menos que lo que tiene Daniel. Si Javier
c) 2x 3y + kz = 5
le diera $ 2.000 a Matas, tendran lo
x 2y + 4z = 2 mismo y si Daniel tuviera $ 6.000 ms,
x + ky 9z = 1 tendra el triple de lo que tiene Matas.
Encuentre el capital de cada uno.
42. Resuelva el sistema.
49. Determine la medida de los ngulos
xy =2 interiores de un tringulo si la suma del
primero con el segundo, menos el tercero
2x z = 1
es 72 y el doble del primero menos
y+z =3 la suma del segundo con el tercero es
168.
43. Resuelva el sistema. 50. Encuentre tres nmeros sabiendo que su
x 2y + 1= 0 suma es 8. Si a la suma del doble del
x + 2y 1= 2 primero ms el segundo se le resta el
tercero da 3 y la suma de los dos primeros
2x 4y + 2= 0
menos el triple del tercero es 4.

208 Relaciones y funciones

208-209. 208 08/11/2001, 17:13


CAPTULO 3

51. La suma de las edades de tres amigas el triple del primero es 36 y la suma
es 80 aos. Si la edad de Regina es el de los tres es 26.
doble de la edad de Gloria ms tres 54. Encuentre la medida de tres ngulos
aos y la edad de Susana es el doble sabiendo que estn en la razn 3 : 6 : 4 y
de la edad de Gloria menos tres aos, los dos primeros son suplementarios.
encuentre las edades de las tres.
55. Tres compradores pagaron las sigu-
52. Determine tres nmeros tales que al ientes facturas por compra de ropa
sumar el primero con el doble del en una tienda: el primero cancel $
segundo se obtiene 4. El doble del 60.700 por la compra de 5 camisas, 3
tercero incrementado en 38, ms el chalecos y dos pares de calcetines. El
triple del primero es 4 y la suma de segundo cancel $ 17.700 y compr
ellos es 22. una camisa, un chaleco y un par de cal-
cetines. El tercero adquiri 2 camisas, 1
53. Determine tres nmeros enteros tales chaleco y dos pares de calcetines y pag
que al sumar el primero con el tercero $ 22.700.
se obtiene 1. El doble del tercero ms Determine el valor de cada prenda.

Soluciones

1. a) L. I. b) L. I. c) L. I d) L. I. e) L. D. f) L. I. g) L. I. h) L. I.
i) L. I. j) L. D.
2. a) I k E R b) k 1 c) I k E R d) k 1
1
3. a) k b) k 6 c) k 1, 1
2
4. Determinado: a, b, c, f, h; inconsistente: d, g, i.

5. a) Inconsistente b) Determinado c) Inconsistente d) Indeterminado


e) Indeterminado f) Determinado g) Determinado h) Indeterminado
i) Inconsistente j) Determinado
6. a) (5, 11) S b) (1, 5) No c) ( 24, 5) No d) ( 30, 23) S
(1, 1) S ( 7 3) S ( 3, 1) S (20, 7) S
(2, 2) S (1, 29) No ( 38, 9) No (15, 9) No
e) ( 13, 6) No f) (2, 5) S
(6, 1) S (50, 19) No
(8, 9) No (26, 7) No
7. a) ( 4, 4) b) ( 5, 4) c) ( 5, 3) d) (5, 3) e) ( 6, 4) f) ( 7 , 4)
g) (5, 5) h) ( 4, 1) i) (4, 3) j) (0, 5) k) ( 6, 3) l) ( 5, 4)
m) ( 7, 2) n) ( 1, 4) o) (5, 3) p) ( 5, 3) q) ( 8, 2) r) ( 4, 2)
s) (3, 4) t) ( 3, 2)
8. a) ( 6, 3) b) ( 4, 1) c) (2, 3) d) ( 4, 3) e) (5, 3) f) ( 3, 5)
g) ( 6, 3) h) ( 6, 4) i) (3, 5) j) ( 1, 1) k) (3, 3) l) (3, 2)
9. a) (0, 1) b) (4, 2) c) (7, 2) d) (4, 6) e) ( 6, 5) f) ( 1, 2)
g) (4, 1) h) ( 7, 3) i) ( 4, 2) j) ( 1, 0) k) ( 1, 2) l) ( 4, 1)
10. a) ( 7, 4) b) (1, 1) c) ( 1, 5) d) (1, 4) e) ( 3, 6) f) (0, 3)

Relaciones y funciones 209

208-209. 209 08/11/2001, 17:13


Soluciones
g) (0, 0) h) ( 2 7
,
11 11 ) i) (3, 0) j) (5, 5) k) ( 6, 3) l) (2, 4)

11. a) ( 5313 , 133 ) b) ( 4, 4) c) (5, 3) d) (2, 0) e) (2, 5) f) (0, 6)

g) (3, 0) h) (5, 1) i) (2, 6) j) (0, 1) k) , 0 ( 2


5 ) l) (4, 2)
m) ( 6, 5)

12. a) (64, 3) ( 14 )
b) 3, ( 14 )
c) 1, d) , ( 1
2
1
3 ) e) 1, 1
4
( ) (3 )
f) 2, 1
3
h) (4, ) i) ( , ) l) ( , 1)
3 4 2
g) (1, 3) j) (1, 4) k) (3, 1)
4 3 3 4

13. a) ( q
,
1
1 + pq 1 + pq ) b) ( l , l )
pq q
c) a ,
b
(b b 1) d) ( 2
a
2
b
, 2 2
a b a b )
14. a) ( 904 998
,
541 541 ) b) ( 1.625 1.183
871
,
871 ) c) ( 6 109
,
7 7 ) d) (525 25
,
736 92 )
e) (
561 1.089
,
197 197 ) f) ( 325 585
,
353 353 ) g) ( 3 53
,
61 122 ) h) ( ,)
5 1
14 3

i) (17 101
,
44 176 ) j) ( 637 1.729
,
696 464 ) k) ( 160 177
,
79 158 ) l) ( )
28 120
,
313 313
15. 403 y 205 16. 12 y 7 17. 25 18. 84
19. Ambos nmeros son iguales a 15. 20. 165 y 43
21. Tiene 15 monedas de $ 100 y 9 de $ 50 22. 13 23. 45
24. Eliana tiene 9 aos, Miguel tiene 45 aos. 25. 243 y 81 26. 10 y 2
27. 21 y 19 28. Manuel tiene 59 aos, Marcos tiene 11 aos.
29. ngela tiene 17 aos, Sergio tiene 20 aos. 30. 40 y 20
31. 150,5 y 18,5 32. 24,5 y 15,5 33. 10 de $ 100 y 11 de $ 50
34. 98 35. Teresa tiene 25 aos, Adolfo tiene 35 aos.
36. 18 y 12 37. 31 38. Ximena tiene 52 aos, Matas tiene 10 aos.
39. a) (1, 0, 2) b) (1, 4, 1) c) (0, 1, 2) d) (2, 3, 3) e) (5, 2, 1)
f) ( 4, 2, 2) g) (1, 5, 2) h) (2, 4, 1) i) (1, 2, 4) j) (2, 5, 3)
k) ( 4, 2, 2) l) (3, 2, 2) m) ( 4, 3, 4) n) (5, 0, 5) ) (2, 1, 4)
o) (2, 3, 1, 1) p) (1, 2, 0, 3) q) (1, 1, 1, 1) r) (1, 0, 2, 2)

40. a) , ( 1 1
,
6 18 11
1
) ( )
b) , 6, 2
1
2 (
c) ,
5
6
10 10
,
47 17 ) d) ( 22 11 44
,
17 2
,
23 )
e) ( 4,
8 8
,
41 15 ) f) ( )
29 29 29
,
8 9 5
, g) ( 2 2
, ,
9 15 2
1
) h) ( 14 28 56
, ,
17 11 43 )
i)
19
5 ( 2
, ,
19
6 ) j) ( )
2 12 6
, ,
5 11 5
k) (2, 2, 1) l) ( 1 3 3
, ,
6 8 11 )
(
23 23 23
m) 19 , 29 , 60 ) n) ( 51
17 51
,
5 55), o)
18
23
, 9, 3 ( )
41. a) iii) I k 5 b) iii) k 2 sol. nica c) iii) Si k 3 o k 7 sol. nica
2 5
iii) no existe k iii) no existe iii) no existe k
iii) k = 5 iii) k = 2 iii) k = 3 o k = 7 no tiene sol.
2 5

210 Relaciones y funciones

210-211.(2003) 210 20/11/02, 11:13 AM


CAPTULO 3

42. (2, 0, 3)

43. El sistema tiene infinitas soluciones porque las ecuaciones 1 y 3 representan el mismo
1
plano. Una solucin es (0, , 1)
2
44. a) El sistema no tiene solucin porque b) El sistema no tiene solucin porque
las ecuaciones 1 y 2 representan las ecuaciones 1 y 3 representan
planos paralelos. planos paralelos.
45. 556 46. Andrs: $ 15.000, Arturo: $ 30.000 y Carlos: $ 5.000
47. 57, 33 y 90 48. Matas $ 19.000, Javier $ 23.000 y Daniel $ 51.000
49. 116, 10 y 54 50. 3, 10 y 1
51. Gloria 16 aos, Regina 35 aos, Susana 29 aos.
52. 56, 30 y 48 53. 38, 25 y 39 54. 60, 120 y 80
55. Camisa $ 4.200, Chaleco $ 12.700 y Calcetn $ 800

3.4.5 Inecuaciones con dos variables.


Sistemas y problemas de
programacin lineal.
Una inecuacin de la forma
f(x, y) > 0, f(x, y) < 0, f(x, y) 0, f(x, y) 0
es una relacin que geomtricamente representa un semiplano
determinado por la recta, cuya ecuacin es f(x, y) = 0.
Para determinar el semiplano solucin se aplica el siguiente
teorema:
Teorema:
Sea P un punto de uno de los semiplanos en que la recta
representada por la ecuacin f(x, y) = 0 divide al plano.
Si f(x, y) > 0 en P, entonces f(x, y) > 0 para todos los puntos que
pertenecen al mismo semiplano de P.
En forma anloga se aplica para
f(x, y) < 0, f(x, y) 0 o f(x, y) 0
Si consideramos ms de una relacin de las formas sealadas
estamos frente a un sistema de inecuaciones de primer grado y
su solucin ser la interseccin de los semiplanos, solucin de las
distintas inecuaciones que lo formen.
Una interesante aplicacin de los sistemas de inecuaciones de
primer grado son los problemas de optimizacin o de programacin
lineal. (Ver problema.)

Relaciones y funciones 211

210-211.(2003) 211 20/11/02, 11:13 AM


Ejercicios 1. Graficar el conjunto solucin de la inecuacin 2x y + 1 < 0
Solucin:
resueltos Consideramos la ecuacin 2x y + 1 = 0 y trazamos la recta
que determina.
Consideremos el punto P = (1, 1) en la
y inecuacin dada:
2x y + 1 < 0
n

1
ci

211+1<0
lu

x 2 <0 F
So

1
2
Luego es solucin el semiplano contrario
al que pertenece P.
NOTA: Achuramos el semiplano que no es solucin. Los puntos
sobre la recta 2x y + 1= 0, no son solucin. Se usa dibujar
esa recta discontinua.
2. Graficar el conjunto solucin del siguiente sistema de inecuaciones.
2x + 5y 1 0
x 3y + 5 0
3x + 2y 7 0
Consideremos las rectas
L1: 2x + 5y 1 = 0
L2: x 3y + 5 = 0
L3: 3x + 2y 7 = 0
y
Las graficamos y de la misma forma que
elegimos el semiplano solucin en el
ejercicio anterior lo hacemos ac. Luego
L2
sealamos la interseccin de los tres
2
So
semiplanos.
1 lu
ci
n

-2 3 x
-1
L1

L3

3. Determinar el polgono que se forma al graficar las inecuaciones


dadas y encontrar sus vrtices.
5x 4y 28 0
2x + 3y 25 0
2x 3y + 5 0
5x + 2y 16 0
Solucin:
Consideremos las rectas
L1: 5x 4y 28 = 0 L3: 2x 3y + 5 = 0
L2: 2x + 3y 25 = 0 L4: 5x + 2y 16 = 0

212 Relaciones y funciones

212-213. 212 08/11/2001, 17:17


CAPTULO 3

Graficando las rectas y eligiendo


Y L2
el semiplano solucin como en el L4 L3
problema 1 nos encontramos con el
cuadriltero P Q R S. 5 .R

Resolviendo el sistema

L1: 5x 4y 28 = 0
3 S . . Q

L2: 2x + 3y 25 = 0

Encontramos el punto Q = (8, 3)


2 4 5 8 X
Resolviendo el sistema
L2: 2x + 3y 25 = 0
L3: 2x 3y + 5 = 0
2
P
.
L1
Encontramos el punto R = (5, 5)
Resolviendo el sistema
L3: 2x 3y + 5 = 0
L4: 5x + 2y 16 = 0 Encontramos el punto S = (2, 3)

Resolviendo el sistema
L4: 5x + 2y 16 = 0
L1: 5x 4y 28 = 0 Encontramos el punto P = (4, 2)

4. Se tiene una regin del plano definida por las inecuaciones.


1x y0 (1)
0 x4 (2)
0 y3 (3)

Determinar para qu punto (x, y) de la regin dada, la funcin


F(x, y) = 4x + 3y es mxima y para qu valores es mnima.
y
Solucin:
Si graficamos la regin obtenemos:
1 1xy0
S R
2 0 x4 3

3 0 y3
4x

Todos los puntos de la regin P Q R S T T


+
3y

cumplen con las tres condiciones planteadas


=

Q
0

3
en el sistema de inecuaciones. Si evaluamos P x

la funcin F(x, y) = 4x + 3y en los puntos P,


Q, R, S, T y cualquier otro de la regin F, es
mxima para R y mnima para T.
En efecto: F(x, y) = 4x + 3y 2 2 1

Relaciones y funciones 213

212-213. 213 08/11/2001, 17:17


P = (1, 0) F (1, 0) = 4
Ejercicios Q = (4, 0) F (4, 0) = 16
resueltos R = (4, 3) F (4, 3) = 4 4 + 3 3 = 25
S = (0, 3) F (0, 3) = 9
T = (0, 1) F (0, 1) = 3
4x + 3y es mxima si x=4 e y=3
\ 4x + 3y es mnima si x=0 e y=1
Observamos que la recta 4x + 3y = 0 pasa por el origen. Para
maximizar la funcin F(x, y) = 4x + 3y debemos hallar la recta
paralela a 4x + 3y = 0 que intersecta al eje y en el mayor valor
posible y que pase por la regin dada. sta es la recta paralela a
4x + 3y = 0 que pasa por R.
Para minimizar F(x, y) = 4x + 3y debemos hallar la paralela a
4x + 3y = 0 que intersecte al eje y en el menor valor posible y sta
es la recta paralela a 4x + 3y = 0 que pasa por T.
5. En una fbrica se producen refrigeradores de dos tipos: corrientes
y de lujo. Se trabaja en dos secciones, una de montaje, la cual
dispone de un mximo de 120 horas de trabajo al da, y una de
acabado, que dispone de 180 horas de trabajo diario. Para producir
un refrigerador corriente se necesitan 3 horas de montaje y 3 de
acabado. Para producir uno de lujo debe disponerse de 3 horas
de montaje, pero 6 horas de acabado. La ganancia al producir un
refrigerador corriente es de $ 30.000 y al producir uno de lujo es
de $ 40.000. Cuntos refrigeradores de cada tipo deben producirse
diariamente para obtener ganancia mxima?
Solucin:
La funcin que debemos maximizar es la funcin que expresa la ganancia.
Supongamos que se producen x refrigeradores corrientes e y
refrigeradores de lujo diariamente.
Luego, la ganancia ser: 30.000x + 40.000y
F (x, y) = 30.000x + 40.000y
Veamos ahora qu restricciones debemos observar:
horas de montaje 3x + 3y 120 1
horas de acabado 3x + 6y 180 2
naturalmente x 0, y 0 3
y Graficando las restricciones (ver grfico).
40
La recta F (x, y) = 30.000x + 40.000y = 0 que
30 se puede escribir 3x + 4y = 0 nos da la direccin
P
de maximizacin. As vemos que en P la funcin
F (x, y) es mxima.

3x 40 60 x
+
4y
=
0

214 Relaciones y funciones

214-215. 214 08/11/2001, 17:19


CAPTULO 3

Para hallar P debemos resolver el sistema.


3x + 3y = 120
3x + 6y = 180

x + y = 40
x + 2y = 60

y = 20 A x = 20
Luego para que la ganancia sea mxima deben fabricarse 20
refrigeradores corrientes y 20 refrigeradores de lujo.
Si evaluamos la funcin, observamos que la ganancia es de $ 1.400.000
F (x, y) = 30.000 20 + 40.000 20 = 1.400.000.
Se puede comprobar fcilmente que usando cualquier otro punto
de la regin, definida por las sustracciones, el valor de la funcin
ganancia resultara inferior a $ 1.400.000.
6. En una plantacin se ha detectado una enfermedad y para combatirla
se necesita una mezcla que contenga como mnimo 15 partes de
una sustancia A y 20 partes de otra sustancia B. En el mercado slo
se encuentran dos productos que pueden ser usados mezclados.
Uno tipo x, que contiene 1 parte de A y 5 partes de B y que
cuesta $ 1.000 el litro. Otro tipo y, que contiene 5 partes de A
y 2 partes de B y su valor es de $ 3.000 el litro. Qu cantidad
se debe mezclar de cada uno para satisfacer las necesidades
con un costo mnimo?
Solucin:
Sea x la cantidad de producto x
Sea y la cantidad de producto y
La funcin que queremos minimizar es la funcin que expresa
el costo de la mezcla.
F (x, y) = 1.000x + 3.000 y.
Las restricciones que nos impone el
10
problema son:
1 Debe haber a lo menos 15 partes de
sustancia A en la mezcla.
x + 5y 15 3
2 Debe haber a lo menos 20 partes de 2,4 . 8

sustancia B en la mezcla.
5x + 2y 20 3,1 4 15
x + 3y
=0
3 Naturalmente x 0 e y 0
Graficando las restricciones.

Relaciones y funciones 215

214-215. 215 08/11/2001, 17:19


Ejercicios La funcin F (x, y) = 1.000x + 3.000y debe ser mnima.
Vemos que la ecuacin 1.000x + 3.000y = 0 o x + 3y = 0 nos
resueltos da la direccin de minimizacin y la funcin se hace mnima
en el punto P.
Para hallar las coordenadas de P debemos resolver el sistema:
x + 5y = 15
5x + 2y = 20

70
cuya solucin es x= 3,1
23
55
y= 2,4
23
Luego para que el costo de la mezcla sea mnimo se debe adquirir 3,1
litros del producto tipo x y 2,4 litros del producto tipo y.
Su costo es F (3,1 ; 2,4) = 1.000 3,1 + 3.000 2,4
= 3.100 + 7.200 = 10.300
Si se evala la funcin costo en cualquier otro punto de la regin
definida por las restricciones, se ver que el costo es mayor
que $ 10.300.

Ejercicios
n) x 3y < 9
1. Trace la grfica del conjunto solucin
o) xy2>0
de las siguientes inecuaciones con dos
variables. 2. Grafique la solucin de cada uno de los
a) 2x + y 10 < 0 siguientes sistemas de inecuaciones.
b) x > 3,1 a) 4x + 7y < 5
c) 4x + 3y + 21 < 0 2x 9y + 1 < 0
d) x < 1,3 x > 2,7
e) 2x 3y > 18
f) x < 4,3
g) 7x + y 9 < 0 b) 8x + 13 y 9 > 0
h) x 2y 4 > 0 9x 5y + 16 < 0
i) x > 1,9 x < 2,5
j) 2x 8y > 8
k) x < 3 c) 5x + 10y < 15
l) 3x + 6y 24 3x 6y 9 < 0
m) 10x + y 25 x<3

216 Relaciones y funciones

216-217. 216 08/11/2001, 17:20


CAPTULO 3

c) x+y <0
d) 6x + 5y 7 > 0 7x 10y > 15
9x 4y > 2 y+2 >0
x > 2,9

4. Determine cul o cules de los puntos


e) 6x + 11y 14 < 0
dados pertenecen al conjunto solucin
5x 7y + 6 > 0
del sistema dado:
x < 3,1

f) 3x + 5y + 1 > 0 a) 10x + 10y + 20 > 0


5x 12y 23 > 0 3x 13y + 5 < 0
x + 4,1 > 0
y+3 >0

(1, 0), (0, 1), (1, 1), (1, 1)


g) 6x + 6y + 12 > 0
(1, 1) (3, 3)
9x 8y 4 > 0
x 3,3 < 0
b) 4x + 13y + 11 < 0
h) 10x + 11y 5 0 7x 9y + 3 > 0
9x 8y 4 < 0
y 2 (0, 0), (2, 2), (1, 3), (1, 5),
(1, 1), (3, 0)

i) 2x + 3y 2 > 0 c) 7x + 4y > 6
4x 3y + 8 > 0 9x 9y + 27 < 0
2x + y 2 < 0
(0, 0), (2, 1), (3, 7), (3, 7), (0, 5),
(3, 1)
j) 6x + 8y < 0
9x 11y 8 < 0 5. Determine el polgono que se forma
al graficar las inecuaciones dadas y
x+3 >0
encuentre sus vrtices.

2x y 3 0
3. Escriba a lo menos dos puntos que sean x + 2y 4 0
solucin de cada uno de los siguientes
x 2y + 4 0
sistemas de inecuaciones:
x + 5y + 4 0
a) 6x + 10y < 6
6. Determine el polgono que se forma
5x 11y > 13
al graficar las inecuaciones dadas y
x + 3,1 > 0 determine sus vrtices.

b) 6x + 6y + 18 > 0 5x 6y 26 0
5x 3y > 4 3x + 2y 10 0
x <4 2x y 2 0

Relaciones y funciones 217

216-217. 217 08/11/2001, 17:20


Ejercicios
10. Maximice la funcin F(x, y) = 2x + 3y en
7. Considere la regin del plano definida la regin definida por las restricciones
en el siguiente grfico: siguientes:
y
6x + 5y 10 0
4 7x 3y 15 0
2x 3y + 10 0
x 0, y 0

11. Un artesano fabrica dos tipos de anillos.


2 5 x
Para los del tipo A requiere 1 gr. de
oro y 1,5 gr. de plata y los vende a
a) Determine el sistema de inecua-
$ 4.000 cada uno. Para los del tipo B
ciones que definen la regin.
necesita 1,5 gr. de oro y 1 gr. de plata
b) Maximice la funcin y los vende a $ 5.000 cada uno. Si
F (x, y) = 4x 2y dispone de 750 gr. de oro y 750 gr.
de plata, cuntos anillos de cada tipo
8. Maximice la funcin F (x, y) = 3x + 2y deber fabricar para obtener el mximo
sujeta a las siguientes condiciones. de dinero por su venta?
5y 7x 10
7x 3y 15 12. Se desea contratar movilizacin para
3y 2x 10 trasladar a 400 personas y se dispone
x 0, y 0 de las siguientes alternativas. Hay 8
buses con capacidad para 40 personas
y cada uno cuesta $ 12.000, y 10 buses
9. Maximice y minimice la funcin
con capacidad para 50 personas, con
F(x, y) = 4x + 5y en la regin definida
un valor de $ 16.000 cada uno. Si se
por:
dispone slo de 9 conductores para
12x + 3y 120 esa oportunidad, cuntos buses de
3x + 2 y 90 cada tipo convendra arrendar para
x + 2y 50 que el viaje resulte lo ms econmico
x 0, y 0. posible?

Soluciones

1. a) b) c)
Solucin

10
21
4
So
Sol

lu
ci
uci

n
n

5 3,1 21
3

218 Relaciones y funciones

218-219.(2003) 218 20/11/02, 11:16 AM


CAPTULO 3

d) e) f)

Solucin
9
So

i n
uc
lu

-6
l
So
ci

4,3
n

1,3

9
g) h) i)

Solucin
Solucin 4
n
ci
-9 -1,9
7 u
-2 S ol

j) k) l)
S
o

1
lu

8
c

-4 3
i

n
i
n

-4 c
Solucin lu
So
Solucin

m) n) o)

i n
l uc 9
So
n

2
i
c
lu

-3
o

-2,5 -2
S

Solucin

2. a) 2
b) 2 c)
1
So

1
luc
in

-3 -2 -1 1 2 -3 3
-1
-1 Solucin
-2

Relaciones y funciones 219

218-219.(2003) 219 20/11/02, 11:16 AM


Soluciones

d) e) y f) y

2
1 1
1 Solucin
-1 2 x -1 1 2 3 4 5 x
-3 -1 1 2 -1
uc in
Solucin -2 Sol

y y y
g) h) i) 3

2 2

So
n
i

luc
1 luc

in
o S
-2 1 3 x -1 1 x -2 1 x
-1 -1
Solucin
-2

j) y

1
in
Soluc

-1 1 x
-1

3. a) Entre otros: (1, 2) (0, 5) b) Entre otros: (0, 0) (3, 1)

c) Entre otros: 1, 3 ( 2, 1)
2
4. a) Slo (0, 1) ( 1, + 1) b) (1, 3), ( 1, 1) c) (3, 7) , (0, 5).
5. Se forma un cuadriltero cuyos vrtices son (1, 1), (2, 1), (0, 2), ( 4, 0).
6. Se forma un tringulo cuyos vrtices son: ( 2, 6), (4, 1) y (2, 2).
7. 2x y 0
4x 5y 0 F (x, y) = 4x 2y es mxima para (5, 4) y vale 12.
0y4

8. F (x, y) = 3x + 2y y lo mximo para x = 5, y = 20


3
9. F (x, y) = 4x + 5y es mxima para x = 4,3 y = 22,8 y es mnima para x = 0, y = 0.
20
10. F (x, y) = 2x + 3y es mxima para x = 5, y =
3
11. 300 de cada tipo.
12. Hay que contratar 5 buses de capacidad 40 y 4 buses de capacidad 50.

220 Relaciones y funciones

220-221.(2003) 220 20/11/02, 11:19 AM


CAPITULO 3
CAPTULO

Prueba de seleccin mltiple


1. Dadas las siguientes expre- I) pq 8. Sea
siones del lenguaje, son II) qp A = {x E N / x > 1 A x 4}.
proposiciones. III) Bp q Determinar cul de las
III) Cunt ame un cuento A. Slo I siguientes proposiciones es
III) Gagarin vol a la Luna B. Slo II falsa:
III) La Tierra est en la C. Slo I y II
A. 1 F A
va lctea. D. Slo I y III
E. Slo II y III B. 2 E A
A. Slo I C. 4 F A
B. Slo II 5. Sea p (x) : x es solucin D. 5 F A
C. Slo III de la ecuacin x2 1 = 0. E. 3 E A
D. Slo I y II Sea E = {1, 0, 1}. De las
E. Slo II y III proposiciones siguientes
9. Sean
son falsas:
2. La funcin proposicional A = {factores de 12}
x no es mltiplo de 3 es I) (Ix E E), p(x) B = {factores de 18}
falsa si x vale: II) (Hx E E), p(x)
Determinar cul de las
III) (Hx E E), Bp(x)
A. 1 siguientes proposiciones
B. 2 A. Slo I es verdadera:
C. 3 B. Slo II
C. Slo III A. 4E A A 4 F B
D. 4
E. 5 D. Slo I y II B. 2E A A 2 F B
E. Slo II y III C. 9E A A 9 F B
3. Sea A= {1, 2, 3, 4} D. 6E A A 6 F B
6. Sean
y B= {1, 2, 3}. E. 9 E A A 9 E B
p(x) : x es menor que 3
De las siguientes proposi- q(x) : x es mayor que 1.
ciones son verdaderas: 10. A = {1, 2, 3}
De las proposiciones
siguientes son verdade- La cardinalidad del con-
p: (In E A) (H m E B)
ras: junto potencia de A es:
(m + n) E A
I) (Hx E N), p(x) A q(x) A. 2
q: (In E A) (Im E B)
II) (Ix E N), p(x) V q(x) B. 3
(n m + 1) E A
III) (Hx E N), Bp(x) A B q(x) C. 4
r: (In E A) (Hm E B) D. 8
(n m + 1) E A A. Slo I
B. Slo II E. 16
A. Slo p C. Slo III
B. Slo q D. Slo I y II 11. El conjunto
C. Slo r E. Slo I, II y III F = {n (n+1) 1 / n E N,
D. Slo p y r
n < 6}, escrito por exten-
E. p, q y r 7. Dado el conjunto
A = {1, 2, 3}, entonces: sin es:
4. Sean p = n + m es par y
A. {1, 5, 11, 19, 29, 41}
q = m n es par, dos fun- A. 1 F A
ciones proposicionales en B. {1, 5, 11, 19, 29}
B. 2 C A
el conjunto de los nmeros C. {0, 1, 5, 11, 19}
C. {3} E A
enteros. De las propo- D. {0, 1, 5, 11, 19, 29}
D. E A
siciones siguientes son E. {1, 1, 5, 11, 19}
verdaderas: E. {1} C A

Relaciones y funciones 221

220-221.(2003) 221 20/11/02, 11:19 AM


Prueba de seleccin mltiple
12. Si A = {7, 13, 19, 25, 31}. entonces #A K B es: A. 4
Cul de los siguientes A. 4 B. 8
conjuntos es distinto de B. 7 C. 10
A? C. 8 D. 14
D. 9 E. 16
A. {7n (n 1)
E. 11
/ n E N, n 5} 19. Sean A = {1, 2, 3}
B. {6n + 1 / n E N, n 5} 16. Sea B = {5, 6, 7}
C. {8n (2n 1) A = {x E R De las siguientes relacio-
/ n E N, n 5} / 2 x 2} nes, cules son de A
D. {5n + (n + 1) B = {x E R en B?
/ 0 x 5}
/ n E N, n 5} I {(1, 5) (2, 7) (1, 7)}
C = {x E R
E. {4n + (2n 1) II {(5, 1) (6, 1)}
/ 1 < x 7}
/ n E N, n 5} III {(3, 5) (3, 6) (3, 7)}
El conjunto (A J B) C es:
13. Cul de las siguientes A. Slo I
proposiciones representa A. {x E R / 0 x < 1} B. Slo II
la parte coloreada del B. {x E R / 0 x 1} C. Slo III
diagrama? C. {x E R / 1 < x 2} D. Slo I y II
I) (A J B) (B J C) D. {x E R / 1 x < 2} E. Slo I y III
II) [B J (A K C)] (A J C) E. {x E R / 0 < x < 1}
20. Sean A = {2, 4, 6}
III) [(B J C) K (A J B)]
17. Si A C B y #B = 12 y B = {1, 3, 5, 7}.
(A J C)
y #A = 6, entonces La relacin R : A Q B
se define por:
A B #A J B es:
R = {(x, y) / y = x 1}.
A. 6 Entonces R=
B. 12
A. {(3, 2) (5, 4) (7, 6)}
C. 18
C D. 24 B. {(2, 1) (4, 5) (6, 7)}
E. No se puede C. {(2, 1) (4, 3) (6, 5)}
A. Slo I
determinar D. {(2, 3) (4, 5) (6, 7)}
B. Slo II
E. {(1, 2) (3, 4) (5, 6)}
C. Slo III 18. Se ha consultado a 28
D. Slo I y II personas por el consumo 21. Sea R la relacin en
E. Slo II y III de 3 productos A, B y C, Z x Z definida por
14. La expresin obtenindose el siguiente R = {(x, y) / x 2y}
resultado: Son pares de esta
(A J B) (B J C)
es equivalente a: relacin:
3 consumen A, B y C
A. A' J B J C 7 consumen A y C I) (5, 3)
B. A J B' J C 6 consumen B y C II) (5, 1)
5 consumen A y B III) (5, 2)
C. A J B J C'
D. A K (B J C) 5 consumen slo A A. Slo I
E. (A K B) J C 7 consumen slo B B. Slo I y II
C. Slo I y III
15. Si #A = 4 y #B = 7 Cuntas personas con- D. Slo II y III
y #A J B = 3, sumen el producto C? E. I, II y III

222 Relaciones y funciones

222-223. 222 08/11/2001, 17:25


CAPITULO 3
CAPTULO

22. En la relacin N N 25. De las siguientes relacio- (c, b) (a, b) (d, d)


definida por R = {(x, y) / nes en N, cul de ellas (c, d) (a, d)}
2x + y 10}, el domi- es refleja.
El diagrama de orden
nio y el recorrido son A. x Ry x y 0 definido por S es:
respectivamente:
B. x Ry x y = x2
A. {1, 2, 3, 4} d b
{2, 4, 6, 8} C. x Ry x y = 2 c
A.
D. x Ry x + y es impar a
B. {1, 2, 3, 4} d
{1, 2, 3, 4, 5} E. x Ry x + y = 12 c
B. b
C. {1, 2, 3, 4} a
{1, 2, 3, 4, 5, 6, 7, 8} 26. De las siguientes relaciones
en N, determine cul de c
D. {2, 4, 6, 8} b d
ellas es de equivalencia C. a
{1, 2, 3, 4}
A. x Sy x + y es par d
E. {1, 2, 3, 4, 8, 6, 7, 8}
{1, 2, 3, 4} B. x Sy y = 2x b
D.
C. x Sy x > y a c
23. Si R = {(3, 1), (4, 1), D. x Sy x < y d
(5, 1), (6, 1)} E. x Sy 2x < y
entonces R1 es E. a b c
A. {(3, 1), (4, 1), (5, 1), 27. Sea A = {2, 5, 6, 9} y
30. La relacin R en
(6, 1)} R = {(9, 2), (9, 9), (6, 6),
(2, 9), (2, 2), (5, 5)} A = {1, 2, 3} definida por
B. {(3, 2), (4, 2), (5, 2),
(6, 2)} una relacin de equiva- R = {(3,2) (2,3) (3,3)} es:
lencia en A. Entonces la
C. {(1, 3), (1, 4), (1, 5), A. Refleja y transitiva.
clase del 9 es:
(1, 6)} A. {2} B. Refleja y simtrica.
D. {(2, 3), (2, 4), (2,5), B. {9} C. Simtrica y transitiva.
(2, 6)} C. {2, 9} D. Refleja y
E. {(3,3), (4, 4), (5, 5), D. {5, 6}
antisimtrica.
(6, 6)} E. {2, 5}
E. Transitiva y
24. Si R = {(x, y) E N x N 28. De las siguientes relacio-
antisimtrica.
/ x es el doble de y}, nes en N, determine cul
de ellas es una relacin 31. De las relaciones siguien-
entonces R1 es:
de orden. tes en A = {1, 2, 3},
A. {(x, y) E N x N cules de ellas son fun-
/ x es el doble de y} A. x Ry x divide a y ciones:
B. x Ry x + y = 2
B. {(x, y) E N x N f1 = {(1,1) (2,2) (3,3)}
C. x Ry x + y es par
/ x es la mitad de y}
D. x Ry x = 2y f2 = {(1,1) (2,1) (3,1)}
C. {(x, y) E N x N E. x Ry x es menor f3 = {(1,2) (2,3) (3,1)}
/ y es la mitad de x} que y
D. {(x, y) E N x N A. Slo f1
29. Sea A = {a, b, c, d} y S la
/ y es x ms 2} B. Slo f1 y f2
relacin de orden en A
E. {(x, y) E N x N definida por: C. Slo f2 y f3
/ y es x menos 2} D. Slo f1 y f3
S = {(a, a) (c, c) (b, b) (a, c)
E. f1, f2 y f3

Relaciones y funciones 223

222-223. 223 08/11/2001, 17:25


Prueba de seleccin mltiple
define una funcin real; B. 2x y+3=0
32. En la funcin de 1 es imagen de: C. 2x y3= 0
A = {1, 2, 3} en
A. Slo 1 D. 2x +y4=0
B = {4, 5, 6,}
definida por B. Slo 3 E. 2x +y+4=0
f = {(1,4) (2,4) (3,4)}, C. 1y3 41. De los tros de puntos
el dominio y el rango D. Slo 0 siguientes son colineales:
E. 1 y 3
son: I (3, 2) (1, 0) (1, 2)
A. {1, 2, 3}, {4, 5, 6} 37. Si II (6, 1) (3, 2) (3, 4)
B. {1, 2, 3}, {4, 5} 2 si x 1 III (3, 1) (4, 3) (5, 7)
C.
D.
{1, 2, 3}, {5, 6}
{1, 2, 3}, {4, 6}
{
f (x) = x + 1 si 1 < x <1
1 si x 1
A.
B.
Slo I
Slo II
E. {1, 2, 3}, {4} C. Slo III
define una funcin real;
33. Dada la funcin real su rango es: D. Slo I y II
f (x) = 2x2 3x + 1, E. Slo II y III
A. ( , 1]
la imagen de 1 es: 42. De las ecuaciones siguien-
B. [1, 0]
A. 2 C. [1, 2] tes, la que representa una
B. 1 D. [0, 2] K {1} recta paralela a la recta
C. 0 E. [2, + ) x 2y + 3 = 0 es:
D. 1 A. 2y +x+3=0
38. La recta que pasa por
E. 2 B. 2y x6= 0
los puntos (1, 2) y (-3, 1)
tiene por ecuacin: C. 2y +x+6=0
34. Dada la funcin
D. y+2x 3=0
x 1 si x < 2

{
A. x 4y + 9 = 0
E. y+2x +6=0
f (x) = 3 si x = 2 B. x + 4y 9 = 0
x + 1 si x > 2 C. x + 4y + 7 = 0 43. Una recta perpendicular
D. x 4y 7 = 0 a la recta de ecuacin
Cul de las siguientes
E. x 4y + 7 = 0 3x y + 1 = 0 es la
proposiciones es falsa?
representada por:
A. 2 E Dom f 39. La recta cuya ecuacin es
A. 3x + y + 3 = 0
B. 2 E Rang f 2y x + 1 = 0 intersecta
B. 3y x 2 = 0
C. f (2) = 3 al eje x Y al eje y En los
C. 3y x + 2 = 0
D. f (1) = 0 puntos:
D. 3y + x + 2 = 0
E. f (3) = 4 A. 1 y 1 E. 3x + y 1 = 0
35. Dadas las funciones rea- B. 1 y 1 44. En kx x + y + 3 = 0
les f (x) = x2 1 y C. 1 y 1 el valor de k para que
g (x) = 2x +3. La frmula 2 2
la ecuacin represente a
que define (g o f) (x) es: D. 1 y 1
2 una recta que pasa por el
A. 2x2 1 E. 1 y 1 punto (1, 3) es:
2
B. 2x2 + 3 A. 0
40. La recta cuya pendiente
C. 2x2 + 1 B. 1
es 2 y que pasa por el
D. x2 + 3x + 2 punto (1, 1) tiene por C. 2
E. 4x2 + 12x + 8 ecuacin: D. 1
36. Si f (x) = x2 4x + 4 A. 2x + y 3 = 0 E.2

224 Relaciones y funciones

224-225. 224 20/11/02, 11:22 AM


CAPITULO 3
CAPTULO

45. La pendiente de la recta B. 3x 2y + k =0


A. (1, 3)
que pasa por los puntos C. 2x 3y + k =0
(3, 5) y ( 2, 1) es: D. 2x 3y + k =0 B. (1, 3)
E. 3x + 2y + k =0 C. (1, 3)
A. 4
D. (3, 1)
B. 5 50. De las siguientes funcio-
E. (3, 1)
C. 4 nes en A = {4, 6, 8}
5 determinar cul o cules 54. Los valores de p y q para
D. 5 son biyectivas. que la solucin del sistema
4
4 I) f1 = {(4, 4) (6, 6) (8, 8)} x 2py 5 = 3
E.
5 II) f2 = {(4, 6) (6, 6) (8, 6)} qx + 1 2y = 3
46. De las siguientes rectas, III) f3 = {(4, 6) (6, 8) (8, 4)} sea (4, 3) son respecti-
pasan por el origen: vamente:
A. Slo f 1
I) x + 2y 1 = 1 A. 2 y 2
B. Slo f 3
II) 3x + 2y + 2 = 2 3
C. Slo f 1 y f 2 B. 2 y 2
III) x 5y = 0
D. Slo f 2 y f 3 3
2
A. Slo I C. y 2
E. Slo f 1 y f 3 3
B. Slo II 2
D. 2 y
C. Slo II y III 3
51. La funcin inversa de la 2
D. Slo I y III E. y 2
funcin real 3
E. I, II y III
f(x) = 2x 1 55. El valor de k para que
x+3 el sistema
47. La ecuacin de la recta 3x 1
A. f 1(x) = 5x y +3 = 0
paralela al eje X que 2x
2kx + 3y 1 = 0
pasa por el punto (4, 1) B. f 1(x) = 3x + 1
es: 2x no tenga solucin es:
3x + 1
A. x1 =0 C. f 1(x) =
x2 A. 15
B. x+1=0 3x 1
D. f 1(x) = B. 15
C. y1 =0 x2
x3 C. 2
D. y+1=0 E. f 1(x) =
E. x+y=1 x2 D. 2
52. La funcin inversa de la E. 15
48. La ecuacin de la recta funcin real
2
perpendicular al eje Y 56. La solucin (x, y) del
f(x) = 3x + 5 es:
que pasa por el punto sistema
( 3, 4) es: A. f 1 (x) = 3 (x 5) 2ax by + 2 = 0
A. y 4 = 0 B. f 1 (x) = 3x 5 ax + 2by + 1 = 0 es:
B. y+4=0 x5
C. f 1 (x) = 1 1
C. x + 4 = 0 3 A. ,
a b
D. x 4 = 0 D. f 1 (x) = x 5 1
3 B. a , 0
E. xy=4 x+5
E. f 1 (x) = C.
1
,
1
49. La ecuacin de la fami- 3 a b
lia de rectas que tiene 53. La solucin del sistema 1
D.
a , 0
pendiente 2 es: 2x y 5 = 0
3
x+y4=0 es: E. 1 1
A. 2x + 3y + k = 0 ,
a b

Relaciones y funciones 225

224-225. 225 20/11/02, 11:23 AM


Prueba de seleccin mltiple
57. La solucin del sistema A. Slo I C. Slo III
B. Slo II D. Slo I, y III
x + y z + 2u = 2
x + 2y z u = 1 C. Slo III E. I, II y III
2x y + 2z 3u = 1 D. I, II y III
x 2y + z + 3u = 3 E. ninguno 60. Considerando las restric-
ciones
es: 59. Son soluciones del sis- 2x + y 1 0
A. (0, 1, 1, 1) tema x + 2y 0
B. (1, 0, 1, 1)
2x + y 1 0 x3
C. (1, 1, 0, 1)
x 2y 0
D. (1, 1, 1, 0)
x3 0 la funcin F (x, y) = x + y
E. (1, 1, 1, 1)
es mxima para:
58. Son soluciones de la ine- I) (2, 1)
II) (3, 5) A. 3 1,5
cuacin x 2y + 5 0
los siguientes puntos? III) (2, 3) B. 3 0
C. 2 3
I) (1, 2)
A. Slo I D. 3 2
II) (3, 4)
III) (1, 2) B. Slo II E. 1,5 3

Soluciones

1. E 2. C 3. C 4. C 5. A 6. D 7. E 8. C
9. A 10. D 11. B 12. E 13. E 14. C 15. C 16. B
17. A 18. D 19. E 20. C 21. D 22. C 23. C 24. B
25. B 26. A 27. C 28. A 29. A 30. C 31. E 32. E
33. C 34. B 35. C 36. C 37. D 38. E 39. D 40. A
41. A 42. B 43. D 44. B 45. C 46. C 47. D 48. B
49. C 50. E 51. A 52. C 53. D 54. E 55. E 56. D
57. B 58. D 59. D 60. A

226 Relaciones y funciones

226. 226 08/11/2001, 17:29


4
E
CAPTULO

cuaciones e
inecuaciones
de segundo grado

Ecuacin cuadrtica 4.1

La expresin ax2 + bx + c = 0, donde a, b y c son nmeros


reales cualesquiera y a 0, se llama ecuacin cuadrtica o
ecuacin de segundo grado.
La solucin de esta ecuacin puede obtenerse por factorizacin
o aplicando la frmula general.
Todas las ecuaciones de segundo grado tienen dos soluciones.

4.1.1. Solucin de la ecuacin


por factorizacin
Aplicamos aqu la siguiente propiedad:
a b = 0 P a = 0 o b = 0
(si el producto de dos nmeros reales es cero, entonces al menos
uno de ellos es cero).

1. Resolvamos la ecuacin: x2 3x = 0
Ejercicios
Factorizando obtenemos: x2 3x = 0 resueltos
x (x 3) = 0
y aplicando la propiedad indicada, nos queda:
x=0 o x3=0
de donde obtenemos las soluciones x1 = 0
x2 = 3

Ecuaciones e inecuaciones de segundo grado 227

227. 227 8/11/01, 12:50


Ejercicios 2. Resolvamos 5x2 + 11x = 0

resueltos Factoricemos y apliquemos la propiedad:


5x2 + 11x = 0 x (5x + 11) = 0
x = 0 o 5x + 11 = 0

x1 = 0 ; x2 = 11
5
3. Resolvamos x2 64 = 0
La factorizacin correspondiente es:
x2 64 = 0 (x 8) (x + 8) = 0
x8=0 o x+8=0
x1 = 8 ; x2 = 8
4. Resolvamos x2 5 = 0
Factorizando como suma por diferencia nos queda:
x2 5 = 0 (x 5) (x + 5) = 0
x 5=0 o x+ 5=0

x1 = 5 ; x2 = 5
5. Resolvamos x2 x 30 = 0
Procediendo como antes:
x2 x 30 = 0 (x 6) (x + 5) = 0
x6=0 o x+5=0
x1 = 6 ; x2 = 5

Ejercicios
Resuelva aplicando factorizacin:

1. x2 7x = 0 10. 5x2 + 24x = 0 19. x2 121 = 0 28. 9x2 16 = 0

2. x2 13x = 0 11. 9x2 + x = 0 20. x2 4 = 0 29. x2 15 = 0

3. x2 + 20x = 0 12. x2 + x = 0 21. x2 9 = 0 30. x2 3 = 0

4. x2 + 19x = 0 13. x2 + x = 0 22. x2 100 = 0 31. x2 11 = 0

5. x2 + 6x = 0 14. 11x2 x = 0 23. x2 49 = 0 32. 6x2 24 = 0

6. x2 9x = 0 15. x2 25 = 0 24. 2x2 50 = 0 33. 2x2 6 = 0


7. 7x2 5x = 0 16. x2 36 = 0 25. 3x2 12 = 0 34. 4x2 3 = 0
8. 13x2 + 2x = 0 17. x2 1 = 0 26. 5 5x2 = 0 35. 49x2 1 = 0
9. 20x2 4x = 0 18. x2 16 = 0 27. 4x2 1 = 0 36. x2 5x + 6 = 0

228 Ecuaciones e inecuaciones de segundo grado

228-229. 228 8/11/01, 12:54


CAPTULO 4

37. x2 6x + 5 = 0 49. x2 + 10x + 21 = 0 61. x2 6x + 9 = 0


38. x2 x 12 = 0 50. x2 + 14x + 45 = 0 62. x2 8x + 16 = 0
39. x2 + 7x 18 = 0 51. x2 + 9x 36 = 0 63. x2 + 18x + 81 = 0
40. x2 11x + 30 = 0 52. x2 5x 36 = 0 64. x2 10x + 25 = 0
41. x2 9x 22 = 0 53. x2 + 15x 16 = 0 65. 4x2 + 4x + 1 = 0
42. x2 + 5x 24 = 0 54. x2 9x + 20 = 0 66. 9x2 12x + 4 = 0
43. x2 + 3x 28 = 0 55. y2 y 2 = 0 67. 9x2 6x + 1 = 0
44. x2 9x + 8 = 0 56. y2 13y + 40 = 0 68. 4x2 + 20x + 25 = 0
45. x2 + 15x + 36 = 0 57. y2 + 8y + 12 = 0 69. 9x2 + 24x + 16 = 0
46. x2 + 11x + 30 = 0 58. y2 + 10y + 24 = 0 70. 16x2 24x + 9 = 0
47. x2 x 20 = 0 59. x2 12x + 36 = 0

48. x2 13x + 42 = 0 60. x2 + 2x + 1 = 0

Soluciones
1. x1 = 0 x2 = 7 2. x1 = 0 x2 = 13 3. x1 = 0 x2 = 20
4. x1 = 0 x2 = 19 5. x1 = 0 x2 = 6 6. x1 = 0 x2 = 9
2 1
7. x1 = 0 x2 = 5 8. x1 = 0 x2 = 9. x1 = 0 x2 =
7 13 5
24 1
10. x1 = 0 x2 = 11. x1 = 0 x2 = 12. x1 = 0 x2 = 1
5 9
13. x1 = 0 x2 = 1 14. x1 = 0 x2 = 1 15. x1 = 5 x2 = 5
11
16. x1 = 6 x2 = 6 17. x1 = 1 x2 = 1 18. x1 = 4 x2 = 4
19. x1 = 11 x2 = 11 20. x1 = 2 x2 = 2 21. x1 = 3 x2 = 3
22. x1 = 10 x2 = 10 23. x1 = 7 x2 = 7 24. x1 = 5 x2 = 5
1 1
25. x1 = 2 x2 = 2 26. x1 = 1 x2 = 1 27. x1 = x2 =
2 2
4 4
28. x1 = x2 = 29. x1 = 15 x2 = 15 30. x1 = 3 x2 = 3
3 3
31. x1 = 11 x2 = 11 32. x1 = 2 x2 = 2 33. x1 = 3 x2 = 3
3 3 1
34. x1 = x2 = 35. x1 = 1 x2 = 36. x1 = 2 x2 = 3
2 2 7 7
37. x1 = 5 x2 = 1 38. x1 = 4 x2 = 3 39. x1 = 2 x2 = 9
40. x1 = 5 x2 = 6 41. x1 = 11 x2 = 2 42. x1 = 3 x2 = 8
43. x1 = 7 x2 = 4 44. x1 = 8 x2 = 1 45. x1 = 12 x2 = 3
46. x1 = 6 x2 = 5 47. x1 = 5 x2 = 4 48. x1 = 6 x2 = 7
49. x1 = 3 x2 = 7 50. x1 = 9 x2 = 5 51. x1 = 3 x2 = 12
52. x1 = 9 x2 = 4 53. x1 = 1 x2 = 16 54. x1 = 4 x2 = 5
55. y1 = 2 y2 = 1 56. y1 = 8 y2 = 5 57. y1 = 6 y2 = 2

Ecuaciones e inecuaciones de segundo grado 229

228-229. 229 8/11/01, 12:55


Soluciones

58. y1 = 6 y2 = 4 59. x1 = 6 x2 = 6 60. x1 = 1 x2 = 1


61. x1 = 3 x2 = 3 62. x1 = 4 x2 = 4 63. x1 = 9 x2 = 9
1 2 2
64. x1 = 5 x2 = 5 65. x1 = x2 = 1 66. x1 = x2 =
2 2 3 3
1 1 5 5
67. x1 = x2 = 68. x1 = x2 = 69. x1 = 4 x2 = 4
3 3 2 2 3 3
3 3
70. x1 = x2 =
4 4

4.1.2. Solucin de la ecuacin


cuadrtica aplicando la
frmula general
A partir de la ecuacin general de segundo grado
ax2 + bx + c = 0
podemos obtener las soluciones x1 y x2 aplicando la frmula:

x = b b 4ac
2

2a

Ejercicios 1. Resolvamos la ecuacin x2 + 3x 10 = 0 aplicando la frmula.

resueltos Primero determinamos los coeficientes que son:


a = 1 ; b = 3 y c = 10

y luego reemplazamos estos valores en la frmula.

x = 3 9 + 40
2
3 7
x=
2
obteniendo x1 = 2 y x2 = 5
2. Resolvamos la ecuacin 4x2 + 4x + 1 = 0
Los coeficientes son a = 4, b = 4 y c = 1

Reemplazando en la frmula obtenemos:


4 16 16
x=
8
4 0
x=
8
1
lo cual nos da las soluciones iguales a , es decir,
2
1
x1 = y x2 = 1
2 2

230 Ecuaciones e inecuaciones de segundo grado

230-231.(2003) 230 20/11/02, 11:41 AM


CAPTULO 4

3. Resolvamos la ecuacin 2x2 + 3x 1 = 0


Aplicando la frmula para los valores a = 2 ; b = 3 y c = 1

3 9+8
obtenemos: x= 4
3 17
x=
4
3+ 17 3 17
y obtenemos x1= y x2 =
4 4
Nota: Si la cantidad subradical no es un cuadrado exacto, la dejamos
expresada tal cual aparece, as como en el ejemplo anterior.

4. Resolvamos la ecuacin x2 + x + 2 = 0
Los coeficientes en este caso son a = 1 ; b = 1 y c = 2,
aplicando la frmula obtenemos:
1 1 8
x= 2
1 7
x= 2
y las soluciones son:

1+ 7 1 7
x1 = y x2 =
2 2
Nota: Si la cantidad subradical es un nmero negativo, entonces las
soluciones son nmeros complejos. El captulo de nmeros complejos
est estudiado ms adelante, pero aqu podemos definir:

1 = i unidad imaginaria

Ej. : 2 =i 2
25 = 1 25 = 5 i .......etc.

entonces en el ejemplo anterior, las soluciones pueden ser


expresadas por:
1+ i 7 1 i 7
x1 = y x2 = 2
2

5. Resolvamos la ecuacin x2 + 2x + 5 = 0

Apliquemos la frmula directamente:


2 4 20 2 16 2 4i
x= = = = 1 2i
2 2 2

y las soluciones son x1 = 1 + 2i y x2 = 1 2i

Ecuaciones e inecuaciones de segundo grado 231

230-231.(2003) 231 20/11/02, 11:42 AM


Ejercicios

Aplique la frmula para resolver las siguientes ecuaciones:

1. x2 + x = 0 20. x (x + 5) 3 = 2x (x 6)

2. 3x2 2 = 0 21. 3x (x + 2) = (x + 5) (x 5)

3. x2 + 2x + 1 = 0 22. (x 6) (2 x) = (x + 3)2 (x 2)2

4. x2 x 30 = 0 23. 5x (x + 2) = 2x (x + 1)

5. 2x2 + 3x 1 = 0 24. x (x 6) + 2x (x 1) x (x 3) = 0

6. 3x2 x 2 = 0 25. (1 + x)2 + (2 + x)2 = (3 x)2

7. x2 + 2x + 3 = 0 26. (x 8)2 + (x 5)2 = (x 9)2

8. x2 5x 4 = 0 27. (x + 6) (x 6) (x 5)2 = 0

9. 4x2 + 4x + 1 = 0 28. (3x 1) (x + 2) x (x 4) = 0

10. 2x2 + x 2 = 0 29. a (x a) + b (x b) = x (x a) + x (x b)

11. x2 + 6x + 5 = 0 30. (a + x)2 + (b + x)2 = a2 + b2

12. x2 6x + 5 = 0 31. x2 + ax + b = 0

13. 3x2 + x 2 = 0 32. x2 3abx = 3ab (x 3ab)


14. 2x2 + x 1 = 0
33. 1 1
= ab
15. 6x2 +x+5=0 xa xb

34. 1 1
16. 3x2 x 1 = 0 + =1
x2 x3
17. 9x2 2x + 3 = 0 35. 1 + x 1 x = 3
1 x 1+ x
18. (2x 3) (x + 1) = (x 3) (x + 2)
36. 3 1
=2
19. (x 7)2 + 2x = (2x 1) (x 2) 2x 1 2x + 1

Soluciones
6 6
1. x1 = 0 x2 = 1 2. x1 = 3 x2 = 3. x1 = 1 x2 = 1
3
3 + 17 3 17 2
4. x1 = 6 x2 = 5 5. x1 = x2 = 6. x1 = 1 x2 =
4 4 3
5 + 41
7. x1 = 1 + i 2 x2 = 1 i 2 8. x1 = x2 = 5 41
2 2
1 1 1 + 17 1 17
9. x1 = x2 = 10. x1 = x2 = 11. x1 = 5 x2 = 1
2 2 4 4
2 1
12. x1 = 5 x2 = 1 13. x1 = 1 x2 = 14. x1 = 1 x2 =
3 2

232 Ecuaciones e inecuaciones de segundo grado

232-233. 232 8/11/01, 13:05


CAPTULO 4

1 i 119 1 + 13
15. x1 = 1+ i 119 x2 = 16. x1 = x2 = 1 13
12 12 6 6
1 + i 26 1 i 26
17. x1 = x2 = 18. x1 = i 3 x2 = i 3
9 9

7 + 237 7 237 17 + 277 17 277


19. x1 = x2 = 20. x1 = x2 =
2 2 2 2
3+ i 41 3 i 41
21. x1 = x2 = 22. x1 = 1 + 4i x2 = 1 4i
2 2
8
23. x1 = 0 x2 = 24. x1 = 0 x2 = 5
3 2

25. x1 = 6 2 10 x2 = 6 2 10 26. x1 = 4 + 2 2 x2 = 4 2 2

9 + 97 9 97
27. x1 = 61 x2 = No hay 28. x1 = x2 = 4
10 4
a+b+i ab a+bi ab
29. x1 = x2 = 30. x1 = 0 x2 = (a + b)
2 2
a + a2 4b a a2 4b
31. x1 = x2 = 2
32. x1 = 3ab x2 = 3ab
2
a+b+ a b 2+4 a+b a b 2+4 7+ 5 7 5
33. x1 = x2 = 34. x1 = x2 =
2 2 2 2

2+ 13 1 + 13 1 13
35. x1 = x2 = 2 13 36. x1 = x2 =
3 3 4 4

4.1.3 Ecuaciones bicuadrticas


Estas ecuaciones tienen la forma
ax4 + bx2 + c = 0
y podemos resolverlas haciendo el siguiente cambio de variables
y = x2
Con este cambio, la ecuacin original se transforma en una
ecuacin cuadrtica en la variable y:
ay2 + by + c = 0
y aplicando la frmula general o factorizando podemos obtener los
dos valores de y, que son soluciones de la ecuacin transformada.
A partir de cada valor obtenido para y, usando el cambio
de variable efectuado al comienzo, obtenemos dos valores para
la variable original x, y de este modo las 4 soluciones de la
ecuacin original.
Nota: La ecuacin original es de grado 4 y por lo tanto tiene
4 soluciones.

Ecuaciones e inecuaciones de segundo grado 233

232-233. 233 8/11/01, 13:07


Ejercicios 1. Resolvamos la ecuacin: x4 5x2 + 4 = 0
resueltos Haciendo el cambio de variable y = x2 obtenemos:
y2 5y + 4 = 0
Resolviendo esta ecuacin (por factorizacin o aplicando frmula)
obtenemos las siguientes soluciones: y1 = 1 ; y2 = 4
Pero como y = x2 (recordemos que y es variable auxiliar, nosotros
debemos buscar los valores para la variable original x).
y1 = 1, esto implica x2 = 1, es decir x1 = 1
x2 = 1
y2 = 4, es decir x2 = 4, entonces x3 = 2
x4 = 2
y las cuatro soluciones de la ecuacin original son:
x1 = 1 ; x2 = 1 ; x3 = 2 ; x4 = 2

2. Resolvamos la ecuacin: x4 11x2 + 18 = 0


Hacemos el cambio de variable y = x2 y reemplazamos; nos
queda:
y2 11y + 18 = 0
Podemos factorizar (y 9) (y 2) = 0
y obtenemos las soluciones auxiliares: y1 = 9 ; y2 = 2
Volvemos a nuestra variable original del siguiente modo:
y1 = 9 implica x2 = 9, es decir x1 = 3
x2 = 3
y2 = 2 implica x2 = 2, es decir x3 = 2
x4 = 2

3. Resolvamos la ecuacin x4 3x2 4 = 0


Haciendo y = x2, reemplazando y factorizando obtenemos:
y2 3y 4 = 0
(y 4) (y + 1) = 0
las soluciones auxiliares son: y1 = 4; y2 = 1
y= 4 implica x2 = 4, es decir x1 = 2
x2 = 2
y = 1 implica x2 = 1, es decir x3 = i
x4 = i
Las soluciones pedidas son:
x1 = 2 ; x2 = 2 ; x3 = i ; x4= i

234 Ecuaciones e inecuaciones de segundo grado

234-235.(2003) 234 20/11/02, 11:44 AM


CAPTULO 4

Ejercicios
Resuelva las siguientes ecuaciones:

11. x4 + 10x2 + 9 = 0 12. x4 16 = 0 13. x4 7x2 + 10 = 0

14. x4 5x2 36 = 0 15. x4 7x2 + 12 = 0 16. x4 13x2 + 42 = 0

17. x4 14x2 + 33 = 0 18. x4 + 5x2 6 = 0 19. x4 4x2 + 4 = 0

10. x4 10x2 + 25 = 0 11. 4x4 5x2 + 1 = 0 12. 9x4 10x2 + 1 = 0

13. 2x4 9x2 + 4 = 0 14. 3x4 8x2 + 4 = 0 15. 8x4 6x2 + 1 = 0

Soluciones
1. x1 = i; x2 = i; x3 = 3i; x4 = 3i 9. x1 = 2; x2 = 2; x3 = 2; x4 = 2
2. x1 = 2; x2 = 2; x3 = 2i; x4 = 2i 10. x1 = 5; x2 = 5; x3 = 5; x4 = 5
1 1
3. x1 = 2; x2 = 2; x3 = 5; x4 = 5 11. x1 = 1; x2 = 1;
; x4 = x3 =
2 2
1 1
4. x1 = 3; x2 = 3; x3 = 2i; x4 = 2i 12. x1 = 1; x2 = 1; x3 = ; x4 =
3 3
1 1
5. x1 = 2; x2 = 2; x3 = 3; x4 = 3 13. x1 = 2; x2 = 2; x3 = ; x4 =
2 2
6. x1 = 6; x2 = 6 ; x3 = 7 ; x4 = 7 2 2
14. x1 = 2; x2 = 2; x3 = 3 ; x4 = 3
7. x1 = 3; x2 = 3; x3 = 11; x4 = 11 1 1
15. x1 = ; x2 = ; x3 = 1 ; x4 = 1
8. x1 = 1; x2 = 1; x3 = i 6; x4 = i 6 2 2 2 2

4.1.4 Relacin entre los coecientes de


una ecuacin cuadrtica y sus
races o soluciones y la
naturaleza de ellas

Sean x1 y x2 las soluciones de la ecuacin:


ax2 + bx + c = 0

b c
Se verifica: x1 + x2 = y x1 x2 =
a a
Las soluciones de la ecuacin ax2 + bx + c = 0 estn dadas
por:
b b2 4ac
x=
2a
Llamamos discriminante de la ecuacin a la expresin denotada
por y definida por:
= b2 4ac.

Ecuaciones e inecuaciones de segundo grado 235

234-235.(2003) 235 20/11/02, 11:47 AM


El signo determina la naturaleza de las soluciones de la
ecuacin.
Se verifica:
Si > 0, entonces las soluciones son nmeros reales y distintos.
Si = 0, entonces las soluciones son nmeros reales e iguales.
Si < 0, entonces las soluciones son nmeros complejos.

Ejercicios 1. Determinemos la suma de las soluciones de la ecuacin


3x2 9x 16 = 0
resueltos
Notamos que no es necesario obtener las soluciones para
determinar su suma, pues podemos aplicar directamente la
propiedad
b
x1 + x2 = a para este caso a = 3 y b = 9
9
Entonces tenemos x1 + x2 = =3
3

2. Determinemos el producto de las soluciones de la ecuacin


2x2 + x 15 = 0
Aqu tambin podemos aplicar directamente la propiedad
c
x1 x2 = a para a = 2 y c = 15

y obtenemos: x1 x2 = 15
2

3. Qu valor(es) debe tomar k en la ecuacin


9x2 kx + 1 = 0
para que sus soluciones sean nmeros reales e iguales?
La condicin para que las races sean reales e iguales es que el
discriminante sea igual a cero. En este ejemplo tenemos a = 9;
b = k; c = 1
entonces: = 0 b2 4ac = 0
k2 36 = 0
k2 = 36
k=6
y los valores que puede tomar k son +6 y 6

4. Qu condicin debe cumplir t en la ecuacin


tx2 + 2x + 1 = 0
para que sus races sean nmeros complejos conjugados?

236 Ecuaciones e inecuaciones de segundo grado

236-237. 236 8/11/01, 13:18


CAPTULO 4

Para que las races de una ecuacin sean nmeros complejos


conjugados se debe cumplir que el discriminante sea negativo.
Aqu a = t ; b = 2 y c = 1
entonces: < 0 b2 4ac < 0
4 4t < 0
4 < 4t
1<t
y por lo tanto la condicin pedida es que t sea mayor que 1.

5. Determine una ecuacin cuadrtica sabiendo que sus races son:


x1 = 5 y x2 = 6
Solucin 1:
Aplicando las propiedades que relacionan los coeficientes de una
ecuacin cuadrtica con sus soluciones obtenemos:
b b
x1 + x2 = a 1 = a
c c
x1 x2 = a 30 = a
Podemos asignar a a cualquier valor; en particular, hagamos
a = 1 y entonces obtenemos b = 1 y c = 30 y la ecuacin
pedida es:
x2 + x 30 = 0
Solucin 2:
Si x1 y x2 son las races de la ecuacin, entonces sta se puede
factorizar por (x x1) (x x2) = 0
Aqu x1 = 5 y x2 = 6,
entonces la ecuacin factorizada es (x 5) (x + 6) = 0
y la ecuacin pedida es: x2 + x 30 = 0
NOTA: Cualquier amplificacin que hagamos a una ecuacin cuadrtica
nos dejar invariables las soluciones. sta es la razn que nos permiti
elegir a = 1 en la solucin 1 del ejemplo anterior.

Ejercicios
1. Cul es la suma de las soluciones 3. Cul es la suma de las races de
de la ecuacin: la ecuacin:
3x2 5x 2 = 0? 3x2 5x 1 = 7(x 3)?

2. Cul es el producto de las soluciones 4. Cul es el producto de las races de


de la ecuacin: la ecuacin:
3x2 + 5x + 2 = 0? (x 5)2 = (x 5) (x + 5)?

Ecuaciones e inecuaciones de segundo grado 237

236-237. 237 8/11/01, 13:18


Ejercicios
con coeficientes enteros e irreductibles
15. Determine la suma y el producto de cuyas races sean:
las races de la ecuacin: 1
x1 = 3 y x2 =
2ax2 bx + a2b2 = 0 2
17. Determine una ecuacin cuadrtica
16. Determine la suma y el producto de
con coeficientes enteros e irreductibles
las races de la ecuacin:
cuyas soluciones sean:
(a x)2 + (b x)2 = 0
1
x1 = y x2 = 2
17. Determine una ecuacin cuadrtica- 2
cuyas races sean: 18. Determine una ecuacin cuadrtica
x1 = 2 y x2 = 5 con coeficientes enteros e irreductibles
(Esta ecuacin debe tener coeficientes cuyas races sean:
enteros e irreductibles). 3 2
x1 = y x2 =
5 5
18. Determine una ecuacin cuadrtica
cuyas races sean: 19. Determine una ecuacin cuadrtica
x1 = 0 y x2 = 1 con coeficientes enteros que tenga
como soluciones:
19. Determine una ecuacin cuadrtica 2 3
x1 = y x2 =
cuyas races sean: 7 2
x1 = 0 y x2 = 0
20. Determine una ecuacin de segundo
10. Determine una ecuacin cuadrtica grado con coeficientes enteros que
cuyas races sean: tenga como soluciones:
x1 = 2 y x2 = 2 5 3
x1 = y x2 =
11 4
11. Determine una ecuacin cuadrtica
cuyas races sean: 21. Sin resolver la ecuacin
x1 = 3 y x2 = 3 2x2 + 3x 5 = 0
determine la naturaleza de sus
12. Determine una ecuacin cuadrtica soluciones.
cuyas races sean:
22. Sin resolver la ecuacin
x1 = 5 y x2 = 5
x2 + x + 1 = 0
13. Determine una ecuacin cuadrtica determine la naturaleza de sus races.
cuyas races sean: En los ejercicios 23 Q 30, determine
x1 = 2 y x2 = 2 la naturaleza de las races sin resolver
las ecuaciones.
14. Determine una ecuacin cuadrtica
cuyas races sean: 23. 2 (x 3)2 3 (x + 1)2 = 0
x1 = 6 y x2 = 6 24. (x 6) ( x + 5) 2 (x 7)2 = (x + 3)2
15. Determine una ecuacin cuadrtica 25. 3x2 5x 2 = 3 (x 3) + 2 (x 1)
con coeficientes enteros e irreductibles
cuyas races sean: 26. (1 + x)2 = (1 2x)2
2 27. 6x2 + 7x + 4 = 0
x1 = 2 y x2 =
3
16. Determine una ecuacin cuadrtica 28. 2x (x + 4) x (x 1) = (x 3) (2x 1)

238 Ecuaciones e inecuaciones de segundo grado

238-239. 238 8/11/01, 13:29


CAPTULO 4

x + 5 2x 3 x 3
29. + =
x x x2
35. 3x2 x 2k = 0
30. x 2 x + 3
+ =1 Soluciones reales y distintas.
x+3 x2
En los ejercicios 31 Q 40 determine qu 36. x2 + x + 3k = 0
valores debe tomar k o qu condiciones
Soluciones reales y distintas.
debe cumplir k para que las soluciones
sean como se requiere en cada caso. 37. 4x2 12x k = 0
31. 2x2 + kx 3 = 0 Soluciones reales e iguales.
Soluciones reales y distintas. 38. 3kx2 + 2x 1 = 0
32. 3x2 kx + 3 = 0 Soluciones complejas conjugadas.
Soluciones reales e iguales. 39. 3x2 2kx + 2 = 0
33. kx2 + kx 2 = 0 Soluciones reales e iguales.
Soluciones reales e iguales. 40. 3kx2 2x + 5 = 0
34. 5x2 + 2x + k = 0 Soluciones reales e iguales.
Soluciones complejas conjugadas.

Soluciones

1. x1 + x2 = 5 14. x2 6 = 0 29. > 0 Reales y distintas.


3
2 15. 3x2 8x + 4 = 0 30. < 0 Complejas conju-
2. x1 x2 =
3 16. 2x2 7x + 3 = 0 gadas.
3. x1 + x2 = 4 17. 2x2 3x 2 = 0 31. k2 > 24; cualquier k
4. Tiene 1 sola raz. real.
18. 25x2 25x + 6 = 0
32. k = 6
5. x1 + x2 =
b 19. 14x2 17x 6 = 0
2a 33. k = 0 o k = 8;
20. 44x2 + 13x 15 = 0
ab2
x1 x2 =
2 21. > 0 Reales y distintas. k2 + 8k > 0
6. x1 + x2 = a + b 22. < 0 Complejas conju- 1
34. k >
a2 + b2 gadas. 5
x1 x2 =
2 23. > 0 Reales y distintas. 35. k > 1
24
7. x2 + 7x + 10 = 0
24. < 0 Complejas conju-
36. k < 1
8. x2 x = 0 gadas. 12
9. x2 = 0 25. < 0 Complejas conju- 37. k = 9
gadas.
10. x2 4 = 0 38. k < 1
26. > 0 Reales y distintas. 3
11. x2 9 = 0
27. < 0 Complejas conju- 39. 6 < k < 6; k 2 < 6
12. x2 25 = 0 gadas.
1
13. x2 2 = 0 40. k =
28. > 0 Reales y distintas. 15

Ecuaciones e inecuaciones de segundo grado 239

238-239. 239 8/11/01, 13:30


4.2 La funcin cuadrtica
Corresponde a la expresin y = ax2 + b x + c, donde x es la
variable independiente; y es la variable dependiente; a,b, y c son
los coeficientes de la funcin.
La grfica de la funcin cuadrtica es una parbola y puede
tener una de las siguientes seis posiciones.

y y y

x x x

1 2 3
y y y

x x x

4 5 6
Es decir, se puede abrir hacia arriba (figuras 1-2-3) o hacia abajo
(figuras 4-5-6) y puede intersectar al eje x en 2 puntos (figuras 1 y 4);
en 1 punto (figuras 2 y 5) o en ningn punto (figuras 3 y 6).
La concavidad de la parbola o la posicin en que se abre,
(hacia arriba o hacia abajo) est determinada por el signo del
coeficiente de x2 en la funcin y = ax2 + bx + c , es decir, est
determinada por el signo de "a". As:
si a > 0, entonces la concavidad es positiva y la parbola
se abre hacia arriba.
si a < 0, entonces la concavidad es negativa y la parbola
se abre hacia abajo.
NOTA: a no puede tomar el valor 0 (cero) pues entonces la
funcin sera lineal y no cuadrtica.
Las intersecciones de la grfica con el eje X corresponden
a las soluciones de la ecuacin cuadrtica asociada; es decir a;

240 Ecuaciones e inecuaciones de segundo grado

240-241. 240 8/11/01, 13:31


CAPTULO 4

ax2 + bx + c = 0 (cuando y toma el valor cero la grfica est


sobre el eje x).
Como sabemos, los tipos de soluciones de la ecuacin dependen
del signo del discriminante = b2 4ac.
Si > 0, entonces las soluciones son reales y distintas y
por lo tanto hay dos intersecciones con el eje x; stas son los
puntos x1 y x2.
Si = 0, las soluciones son reales e iguales y hay una sola
interseccin con el eje x. Aqu x1 = x2.
Si < 0, las soluciones son complejas conjugadas y entonces
no hay interseccin con el eje x.
La interseccin de la parbola con el eje Y se obtiene haciendo
x = 0 y corresponde por supuesto a y = c.
Todas las parbolas tienen un vrtice que corresponde al valor
mnimo (si la parbola se abre hacia arriba) o al valor mximo
(si se abre hacia abajo).
Las coordenadas del vrtice son: x = b
2a

b b2 4ac
V ,
2a 4a
b
La recta x = es el eje de la parbola.
2a

El dominio de la funcin cuadrtica es R


(no hay restriccin).
El recorrido depende de la concavidad: x1
. .x
2

Si a > 0 entonces c
.
[( ) [
2 V
b 4ac
Rec.(f) = ,+
4a
Si a < 0 entonces

Rec.(f) = ] 2
(
, b 4ac
4a
)]

1. Determinemos la concavidad y el nmero de intersecciones con


Ejercicios
el eje x de la grfica de la funcin:
y = 2x2 + 3x 1
resueltos
En esta funcin tenemos: a = 2, b = 3, c = 1.
Para la concavidad nos basta con analizar el signo de a.
a = 2; a > 0 implica concavidad positiva, la parbola se
abre hacia arriba.

Ecuaciones e inecuaciones de segundo grado 241

240-241. 241 8/11/01, 13:32


Ejercicios Para determinar las intersecciones con el eje X analizamos el signo
del discriminante = b2 4ac
resueltos =9+8
= 17; > 0, es decir, las soluciones son reales y distintas, por lo
tanto hay dos intersecciones con el eje X.

2. Determinemos la concavidad y el nmero de intersecciones de la


grfiica de la funcin: (con el eje X)
y = 3 x2 x + 2
De inmediato; a = 3; a < 0 implica concavidad negativa y la
parbola se abre hacia abajo.
= b2 4ac
= 1 + 24 = 25
> 0 , hay dos intersecciones con el eje X.

3. Determinemos concavidad e intersecciones con el eje X en la


funcin
y = x2 + 6x 9
a = 1 , concavidad negativa, por lo tanto la parbola se abre
hacia abajo.
= 36 36
= 0, es decir, hay un solo punto de interseccin con el eje X.

4. Determinemos, en la funcin
y = x2 4x 32
la concavidad, las intersecciones con ambos ejes, las coordenadas
del vrtice, el dominio y el recorrido y esbocemos la grfica.
Tenemos:
y = x2 4x 32 a = 1 ; b = 4 ; c = 32
a) concavidad
a=1,a>0
b) intersecciones
con eje X : = 144
> 0 2 intersecciones.
Solucionamos la ecuacin para determinar los puntos de
interseccin, que estn dados por las soluciones x1 y x2
x2 4x 32 = 0 (x 8) (x + 4) = 0
x1 = 8 y x 2 = 4
con eje Y:
hacemos x = 0 en la funcin y = x2 4x 32 y obtenemos
y = 32
c) Coordenadas del vrtice.
b b2 4ac
V ,
2a 4a
reemplazando obtenemos: V (2 , 36)

242 Ecuaciones e inecuaciones de segundo grado

242-243.(2003) 242 20/11/02, 4:09 PM


CAPITULO 4
CAPTULO

d) Dom (f) = R
Rec (f) = [ 36, + [ x2 = 4 2 x1 = 8

e) Grfico

32
36 vrtice

5. Dada la siguiente grfica, determinemos


la funcin correspondiente.
Debemos determinar a, b y c. . 28
Tenemos x1 = 7
x2 = 2
c = 28
(c es la interseccin de la grfica con el
eje y)
.
7
.2
b
Sabemos que x1 + x2 = a
c
y x1 x2 = a
c 28
x1 x2 = 2 = a 14a = 28

a :7 a=2

b b
x1 + x2 = ba : 7 + 2 = a 5 = b = 10
2
entonces la funcin pedida es: y = 2x2 10x + 28

Ejercicios
1. Dados los siguientes grficos, determine signo de a, (concavidad)
y tipos de soluciones de la ecuacin asociada:
a) b) c) d)

Ecuaciones e inecuaciones de segundo grado 243

242-243.(2003) 243 20/11/02, 4:10 PM


Ejercicios
c) coordenadas del
En los ejercicios 2 10, 33.
vrtice
determine la concavidad y
el nmero de interseccio- d) recorrido de la
nes con el eje X. funcin
5
2. y = x2 1 e) grfico

3. y = x2 + 1 19. y = x2 + 4x + 3
2 5
20. y = x2 + 5x
4. y = 2x2 3x + 1
21 y = x2 6x + 5 34.
5. y = 3x2 + x + 1
22. y = x2 + 2x + 24
6. y = 5x2 + 2x
23. y = x2 + 6x + 16
7. y = 3x2 +4 4
24. y = 3x2 5x 2
8. y = 6x2 2x 3
25. y = 4x2 9x + 2 8 2
9. y = x2 +x+1
26. y = 4x2 + 9
10. y = 5x2 27. y = 2x2 + 5x + 4 35.

Determine las coorde- 28. y = x2 + 5x


nadas del vrtice de la 4
grfica de las funciones 29. y = 6x2 13x 5
dadas en los ejercicios 30. y = 3x2 5x 6
11 Q 18. 2 2
En los ejercicios 31 Q 42
11. y = x 2 5
determine la funcin corres-
12. y = x2 + 2x + 1 pondiente de acuerdo con
los datos dados:
13. y = 4x2 3x 2
31. 36.
14. y = 2x2 + x + 1

15. y = 3x2 3x + 2 6 .
2
. .4 3
3x 1
16. y = 3x2 + 2
4 32
8 .
17. y = x2 +1
1 2

3x2
18. y = 2x + 5
2
32. 37.
En los ejercicios 19 30 9
determine:

a) concavidad de la
parbola 3 3

6
. .2
b) interseccin con el 1
eje X

244 Ecuaciones e inecuaciones de segundo grado

244-245. 244 8/11/01, 15:16


CAPTULO 4

38. 40. 42.


7
. 1
.
4 4

. 8 . 14

39. 41.

6 .
1
. .4
2 .
3

Soluciones
7. Concavidad negativa; 19. a) positiva
1. a) a < 0 ; 2 intersecciones. b) x1 = 1; x2 = 3
soluciones reales 8. Concavidad positiva; c) V( 2, 1 )
y distintas. 2 intersecciones. d) Rec: [ 1, )
b) a > 0 ;
soluciones 9. Concavidad positiva; 20. a) negativa
complejas 0 intersecciones.
b) x1 = 0 ; x2 = 5
conjugadas. 10. Concavidad negativa;
c) a > 0 ;
soluciones reales
1 interseccin. c) V ( 5 25
,
2 4 )
11. V (0, 5)
y distintas.
d) a < 0 ;
d) Rec: ( ,
25
4
]
12. V ( 1, 0)
soluciones 21. a) positiva
reales e
iguales. 13. V ( ) 3
8
,
41
16
b) x1 = 5
c) V ( 3, 4 )
; x2 = 1

2. Concavidad positiva;
2 intersecciones. ( )
14. V 4 , 8
1 9 d) Rec: [ 4, )

3. Concavidad positiva; 22. a) negativa


0 intersecciones. 15. V ( ) 1 5
,
2 4
b) x1 = 6
c) V (1, 25)
; x2 = 4
4. Concavidad negativa;
2 intersecciones.
5. Concavidad positiva;
16. V
1
( 1)
,
8 64 d) Rec: ( , 25]

0 intersecciones. 17. V (0, 1) 23. a) negativa


b) x1 = 8 ; x2 = 2
6. Concavidad negativa;
2 intersecciones.
18. V ( 2 13
,
3 3 ) c) V (3, 25)

Ecuaciones e inecuaciones de segundo grado 245

244-245. 245 8/11/01, 15:16


Soluciones

d) Rec: ( , 25] 5 7
c) V , 31. y = x2 2x 8
4 8
24. a) positiva 7
d) Rec: [ , ) 32. y = x2 + 9
8
b) x1 = 2 ; x2 = 1 1 2 7
3 28. a) positiva 33. y = x x+5
5 49 2 2
c) V , 4 2 40 64
6 12 b) x1 = 0 ; x2 = 5 34. y = x x
49 9 9 9
d) Rec: [ , ) 5 25
12 c) V , 35. y = x2
2 4
25. a) positiva
25 36. y = x2 4x + 6
1
d) Rec: [ , )
4
b) x1 = 2 ; x2 = 1 2
4 29. a) positiva 37. y = x + 2x + 3
4
c) V 9 49
, 5 2
8 16 b) x1 = ; x2 = 1 38. y = x 4x 8
2 3 2
d) Rec: [ 49 , ) 1 2 3
16 c) V 13 289 39. y = x x 2
, 2 2
26. a) negativa 12 24
3 3 d) Rec: [ 289 , ) 40. (Faltan datos)
b) x1 = ; x2 = 24
2 2
30. a) negativa
c) V (0, 9) 41. y = 2x2 4x + 6
b) no hay 3
d) Rec: ( , 9 ]
5 47 42. y = 2x2 16x 14
c) V ,
27. a) positiva 6 12
b) no hay 47
d) Rec: ( , ]
12

4.3 Inecuaciones de segundo grado

Resolveremos aqu inecuaciones que pueden ser expresadas


en la forma:
ax2 + bx + c 0 o ax2 + bx + c 0
(por supuesto que las desigualdades tambin pueden ser estrictas,
es decir > ).
Usaremos la siguiente propiedad de los nmeros reales:
a b > 0 Pa > 0 A b > 0 o
a<0 A b<0
a b < 0 Pa > 0 A b < 0 o
a<0 A b>0
es decir, un producto de dos factores es positivo si ambos tienen el
mismo signo y es negativo si ambos tienen distinto signo.
Entonces para resolver una inecuacin cuadrtica, la factorizamos
primero (esto es siempre posible determinando las races) y luego
aplicamos la propiedad sealada.

246 Ecuaciones e inecuaciones de segundo grado

246-247. 246 8/11/01, 15:23


CAPTULO 4

1. Resolvamos la inecuacin: x2 5x + 6 > 0


Ejercicios
Factorizndola nos queda: (x 2) (x 3) > 0
resueltos
Aplicando la propiedad, tenemos las siguientes condiciones:
ii) x2>0 A x3>0 o
ii) x2<0 A x3<0
De i) obtenemos x2>0x>2
x3>0x>3
Como deben cumplirse simultneamente, la solucin S1 es la
interseccin de ambas soluciones parciales, es decir Si = ] 3, [
De ii) tenemos el siguiente sistema x 2 < 0 y
x3<0
con lo cual obtenemos las condiciones x < 2 A x < 3

la interseccin de ambas es S2 = ] , 2 [
La solucin final es la unin de S1 y S2 (puesto que i) e ii) son
situaciones independientes), es decir;
S = ] , 2 [ K ] 3, + [
En forma grfica:

2 3

2. Resolvamos la inecuacin x(2x + 4) (x2 + 2x) 35 0


Factorizando tenemos: (x + 7) (x 5) 0
Aplicando la propiedad tenemos dos sistemas, que son:

i) x + 7 0 y ii) x + 7 0
x 50 x50
De i) obtenemos x 7 A x 5 ,
lo cual es una contradiccin pues no hay ningn nmero que
cumpla simultneamente ambas condiciones.
De ii) obtenemos x 7 y x 5
lo que nos da como solucin el intervalo [ 7, 5]
la solucin grfica es:

7 5

3. Resolvamos la inecuacin 2x2 + 9x 5 > 0

Las races de la ecuacin 2x2 + 9x 5 = 0 son

x1 = 1 y x2 = 5,
2
entonces podemos escribir (la ecuacin) en la forma:
1
x x+5 =0
2

Ecuaciones e inecuaciones de segundo grado 247

246-247. 247 8/11/01, 15:24


Ejercicios Amplificndola por 2, nos queda la factorizacin correspondiente a
la inecuacin original, es decir, estudiamos:
resueltos
(2x 1) (x + 5) > 0

i) 2x 1 > 0 Q x > 2
1 { 1
x > 2 (S1)
x+5>0 Qx>5

ii) 2x 1 < 0 Q x < 1 {


2
x < 5 (S2)
x+5<0Qx<5

S = S1 K S2, es decir
1
S = ] - , 5 [ K ] 2 , + [
en forma grfica:

5 1
2

4. Resolvamos la ecuacin 3x2 + 20x 7 0


Procedamos aqu de un modo diferente. Factorizando la expresin
nos queda (3x 1) (x + 7) 0

Las races de la ecuacin correspondiente son x1 = 7 y x2 = 1


3
Ubicamos estos puntos en el eje real, obteniendo tres intervalos.

I II III

7 1
3

Los signos que se obtienen al reemplazar la variable x de la inecuacin


por un nmero real, van intercalados, es decir, cambian de un intervalo
al intervalo siguiente. La razn es obvia.
Por esto slo basta reemplazar la variable x por un valor cualquiera;
esto nos determinar el signo del intervalo donde se encuentra ese valor
y por consiguiente, el signo de los otros intervalos.
Veamos qu pasa con x = 0, (x pertenece al segundo intervalo).
x = 0 Q 3x2 + 20x 7 < 0
Entonces si x pertenece al segundo intervalo, la expresin es negativa
all y por lo tanto es positiva en el primer y tercer intervalo.
1
(y es igual a cero en los puntos 7 y 3 )
+ +

7 1
3

As, la solucin para 3x2 + 20 x 7 0 es


1
S = ] , 7] K [ 3 , + [

248 Ecuaciones e inecuaciones de segundo grado

248-249.(2003) 248 20/11/02, 4:22 PM


CAPITULO 4

5. Resolvamos la inecuacin 3x2 11 x 4 < 0.


1
Las races de la ecuacin asociada son x1 = 3
y x2 = 4
por lo tanto la factorizacin correspondiente es
(3x + 1) (x 4) < 0.
Ubicamos las races en la recta real (en este caso estos valores no
deben estar incluidos en la solucin pues se trata de una desigualdad
estricta) y analizamos lo que pasa para cualquier valor de la variable,
por ejemplo para x = 0 :

I II III Nota:
+ +
Si x = 0, que pertenece al segundo
1 x=0 4 intervalo, la inecuacin queda:
3
3 02 11 0 4 < 0
(x = 0 pertenece al segundo intervalo) 4 < 0
1
Por lo tanto la solucin pedida es: S = ] 3 , 4 [ como esto es verdadero el intervalo
II es solucin.

Ejercicios

Resuelva las siguientes inecuaciones:


1. x2 1 0 12. 3 (2x2 + 1) > 11x
2. 8x2 + 5x 0 13. x (3x 4) > 7
3. x (x 3) 2x (x 2) + 3x < 0 14. 5x2 + 4x 1 0
4. 4x2 < 1 15. (x 2)2 2 (x2 + 2)
5. 3x2 5x < 0 16. x2 10x + 25 < 0
6. x (x 5) 2x (x + 3) + 6 x2 11x 17. 4x (x 4) + 7 0
7. x2 13x + 40 < 0
x+2 x
8. 2x2 + 3 7x 18. 2x 1 x 2 + 2 0
9. 2x2 3x 36 > x2 + 2x 2x x 5
19. + 0
10. 3x2 + 16x 12 < 0 x + 12 x + 3 x + 12 x + 3
x+1 x+2 x+3
11. 4x (x + 3) 5 20. +
x 1 2x + 1 x 1

Soluciones

1 1
1. S = ] , 1] K [1, + [

5 0

] 5
]
2. S = , 8 K [0, + [ 8

Ecuaciones e inecuaciones de segundo grado 249

248-249.(2003) 249 20/11/02, 4:22 PM


Soluciones

3. S = ] , 0 [K] 4, + [ 0 4

4. S = ] 1 1
,
2 2 [ 1
2
1
2

5. S = 0,] 5
3
[ 0 5
3

6. S = ] , 3] K [ 3, + [
3 3

7. S = ] 5, 8 [ 5 8

8. S = [ 12 , 3] 1
2
3

9. S = ] , 4 [ K ] 9, + [ 4 9

10. S = 6, ] 2
3
[ 6 2
3

]
11. S = , 2
5
]K[ 1
2
,+ [ 5
2
1
2

]
12. S = , 3
1
[ K ] 32 , + [ 1
3
3
2

13. S = ] , 1[ K ] 73 , + [ 1 7
3

14. S = 1, [ 1
5
] 1 1
5

15. S = ] , 4] K [0, + [ 4 0

16. S =

]
17. S = , 1
2
]K [ 7
2
,+ [ 1
2
7
2

1
18. S = [ 0, [ K ] 2, 3 ] 0 1 2 3
2 2

19. S = ] , 12 [ K ] 3, 1 ] K [ 5, + [
12 3 1 5

1
20. S = [ 1, [ K ] 1, 4 ] 1
2 1 1 4
2

250 Ecuaciones e inecuaciones de segundo grado

250-251.(2003) 250 20/11/02, 12:00 PM


CAPTULO 4

Sistemas de ecuaciones de 4.4


segundo grado
Un sistema de ecuaciones en dos variables es de segundo grado si
alguna de las ecuaciones contiene alguno de los trminos x2, y2 o xy
(suponiendo que las variables son x e y por supuesto).
No hay mtodos generales que puedan ser aplicados en forma
prctica a todos los sistemas.
Veremos aqu algunos tipos de ellos.

4.4.1 Sistemas que contienen


una ecuacin lineal y una
ecuacin cuadrtica
Para resolverlo, despejamos una de las variables de la ecuacin
lineal y la sustituimos en la ecuacin cuadrtica.

1. Resolvamos: 2x + y = 10 Ejercicios
2 x2 y2 = 12 resueltos
Despejemos la variable y de la primera ecuacin:
y = 10 2x
Reemplacemos en la segunda ecuacin la variable y despejada.
Obtenemos:
x2 (10 2x)2 = 12
x2 (100 40x + 4x2) = 12
Ordenando los trminos tenemos la siguiente ecuacin cuadrtica:
3x2 40x + 112 = 0
28
cuyas soluciones son: x1 = 4 y x2 =
3
x1 = 4 Q y1 = 2
Si 28 26
x2 = Q y2 = 3
3
La solucin de la ecuacin es el conjunto

S= { (4, 2), ( 283 , 263 )}.


Notemos que la solucin de este tipo de sistemas puede estar
formada por 2 puntos, 1 punto o ninguno (geomtricamente representa
la interseccin de una lnea recta con una cnica, o bien, la interseccin
de dos cnicas).

Ecuaciones e inecuaciones de segundo grado 251

250-251.(2003) 251 20/11/02, 12:01 PM


Ejercicios 2. Resolvamos el sistema: x + y =6
resueltos x2 + y2 = 16

Despejamos la variable x (o la variable y) de la primera ecuacin


y obtenemos:
x=6y
y la reemplazamos en la segunda ecuacin:
(6 y)2 + y2 = 16
2y2 12y + 20 = 0 o y2 6y + 10 = 0
las soluciones algebraicas de esta ecuacin son los puntos
y1 = 3 + i y y2 = 3 i
y por lo tanto
x1 = 3 i ; y1 = 3 + i
x2 = 3 + i ; y2 = 3 i
geomtricamente el sistema no tiene solucin.
3. Resolvamos el sistema: x y = 7
xy = 10
Despejando la variable y de la primera ecuacin:
y=x7
y reemplazndola en la segunda: x (x 7) = 10
x2 7x + 10 = 0
las soluciones de la ecuacin son: x1 = 2 y x2 = 5.
Si x = 2 entonces y= 5
x = 5 entonces y= 2
y la solucin del sistema es: S = {(2, 5), (5, 2)}

Ejercicios

Resuelva los siguientes sistemas:

1. x y = 2 4. 2x + y2 = 1 7. 3x 2y = 6
x2 + y2 = 20 2 x y2 =8 2x2 y2 = 23

2. 2x + y = 4 5. 2x y2 = 29 8. 2x + 4y = 18
x2 + y2 = 5 2xy = 40 4xy = 40

3. x 2y = 7 6. 2x + y = 6 9. 5x y = 8
x2 y = 26 x2 + 2y = 0 2xy = 6

252 Ecuaciones e inecuaciones de segundo grado

252-253. 252 8/11/01, 15:31


CAPTULO 4

10. x + 5y = 1 14. 2x + y = 11
x2 + 3xy = 27 x2 xy = 4 18. 2x + 3y = 5
61
11. 3x +9y2 = 2 15. 3x + y2 5 = 0 x2 xy + y2= 36
9x2 9y2 = 8 x2 + y2 xy = 3
16
19. x 2y =
12. x 2y = 0 16. 2x + y2 = 8 15
2xy = 1 2 x2 y2 = 5 x2 + 3xy = 6
25
17. 5x +9y 2= 17
13. 3x + 4y = 0 20. x + y = 12
x2 y2
1 5 =1 x + y xy = 8
4xy = 5
3

Soluciones

1. (4, 2) ; (2, 4) 2. (1, 2) ; ( 11 2


5
,
5 ) 3. (5, 1) ; ( 9 23
2
,
4 )
(
4. (1, 3) ; 7 ,
4 2
5
) 5. (5, 4) ; (4, 5) 6. (2, 2) ; (6, 18)
7. (4, 3) ; (32, 45) 1 3
8. (2, 5) ; (20, ) 9. (1, 3) ; ( 5 , 5)
2
10. (9, 2) ; ( 15 13
,
2 10 ) 1
( ) ( )
11. , 1 ; ,
3
7 3
6 2 ( ) (1
12. 1, 2 ; 1,
1
2 )
(
1 1
13. 3, 4 ;
1 1
,
3 4 ( ) ( )
14. (4, 3) ;
1 35
,
3 3 (
15. (1, 2) ;
22 1
,
3 13 )
16. (3, 2) ; ( 23 22
,
3 3 ) ( , )
17. (3, 2) ;
49 41
12 12 18. (, ) (
3 2
2 3
; 13 , 82
38 57 )
(
19. 5,
2 1
) (
6 31
3 ; 25
,
75 ) 20. (2, 10) ; (10, 2)

4.4.2 Sistemas en que ambas ecuaciones


son de la forma ax2 + by2 = c
(No hay trminos de primer grado, ni el trmino xy).
Lo ms prctico en estos casos es proceder por reduccin de variables y como
sabemos, esto se logra con una adecuada amplificacin de las ecuaciones:

1. Resolvamos el sistema: x2 + y2 = 41
Ejercicios
x2 y2 = 9
resueltos
Podemos reducir la variable y en forma inmediata sumando
ambas ecuaciones.

Ecuaciones e inecuaciones de segundo grado 253

252-253. 253 8/11/01, 15:33


Ejercicios Nos queda: 2x2 = 50
x2 = 25
resueltos
y las soluciones para la variable x son: x1 = 5 y x2 = 5
Si x1 = 5 entonces, reemplazando en la primera ecuacin,
obtenemos: y2 = 41 25
y2 = 16
y = 4
Lo mismo ocurre si x = 5.
La solucin del sistema, entonces, consiste en 4 puntos que son:
5 = {(5, 4), (5, 4), ( 5, 4), ( 5, 4)}

2. Resolvamos el sistema: 2x2 y2 = 1


3x2 + 2y2 = 5

Aqu para eliminar la variable y2 podemos amplificar la primera


ecuacin por 2, y luego sumamos ambas ecuaciones:

+
{ 4x2 2y2 = 2
3x2 + 2y2 = 5

7x2 = 7

Y las soluciones para la variable x son: x1 = 1 y x2 = 1


Si x = 1 entonces, y 1
Si x = 1 entonces, y 1

y la solucin del sistema est dada por:


S = {(1,1), (1, 1), ( 1, 1), ( 1, 1)}

3. Resolvamos el sistema: 3x2 2y2 = 3


x2 3y2 = 13

Amplificando la segunda ecuacin por ( 3) y sumando ambas


ecuaciones obtenemos:

+
{ 3x2 2y2 = 3
3x2 + 9y2 = 39

7y2 = 42
y2 = 6
Las soluciones para la variable y son: y1 = 6 ; y2 = 6
Y sustituyendo estos valores en cualquier ecuacin del sistema
obtenemos para x los valores x1 = 5 x2 = 5
La solucin del sistema es, entonces:
5 = {( 5, 6), ( 5, 6), ( 5, 6), ( 5, 6)}

254 Ecuaciones e inecuaciones de segundo grado

254-255. 254 8/11/01, 15:36


CAPTULO 4

Ejercicios
Resolver los siguientes sistemas:

1. x2 + y2 = 5 6. 5x2 + 3y2 = 172 11. 6x2 5y2 = 3x2 2y2


x 2 y2 = 3 x2 y2 = 60 x2 = 2y2

12. x2 + y2 = 7
2. x2 + 2y2
= 72 7. 2x2 3y2
= 194 4x2 7y2 = 6
x2 y2 = 60 3x2 + y2 = 379

13. 6x2 2y2 = x2 + y2 4


3x2 y2 = 4
3. 2x2 + y2 = 22 8. 4x2 5y2 = 8
x2 2y2 = 1 x2 + 3y2 = 49
x2 + y2
14. = 17
2
4. 3x2 + y2 = 124 9. 2x2 y2 = 1 x2 y2
2x2 + 3y2 = 120 2x2 + y2 = 7 = 4
4

5. 5x2 + y2 = 20 10. 2x2 + 3y2 = 27 3x2 + 2y2 7


15. 4
=
3y2 = 75 x2 y2 = 1 4
x2 + y2 =3

Soluciones
1. S = {(2, 1), (2, 1), ( 2, 1), ( 2, 1)}
2. S = {(8, 2), (8, 2), ( 8, 2), ( 8, 2)}
3. S = {(3, 2), (3, 2), ( 3, 2), ( 3, 2)}
4. S = {(6, 4), (6, 4), ( 6, 4), ( 6, 4)}
5. S = {(3, 5), ( 3, 5), (3, 5), ( 3, 5)}
6. S = {(2, 8), (2, 8), ( 2, 8), ( 2, 8)}
7. S = {(11, 4), (11, 4), ( 11, 4), ( 11, 4)}
8. S = {( 13 , 12 ), ( 13 , 12 ), ( 13 , 12 ), ( 13 , 12 )}

9. S = {( 2, 3), ( 2, 3), ( 2, 3), ( 2, 3)}


10. S = {( 6, 5), ( 6, 5), ( 6, 5), ( 6, 5)}

11. S = {(0, 0)}


12. S = {( 5, 2), ( 5, 2), ( 5, 2), ( 5, 2)}
13. S = {(2, 8), (2, 8), ( 2, 8), ( 2, 8)}

14. S = {(5, 3), (5, 3), ( 5, 3), ( 5, 3)}


15. S = {(1, 2), (1, 2), ( 1, 2), ( 1, 2)}

Ecuaciones e inecuaciones de segundo grado 255

254-255. 255 8/11/01, 15:39


4.4.3 Sistemas formados por una ecuacin de
x2 + y2 = a y la otra ecuacin
la forma
de la forma x y = b.

Una manera prctica de resolver estos sistemas es completando


cuadrados de binomio y reduciendo la solucin del sistema de 2
grado a sistemas lineales (o de 1er grado).

Ejercicios 1. Resolvamos el sistema: x2 + y2 = 34

resueltos y=

Amplificando la segunda ecuacin por 2, y sumando las


ecuaciones, y luego restando, obtenemos:

+
{ x2 + y2 = 34
2xy = 30

{ x2 + y2 = 34
2xy = 30

x2 + 2 xy + y2 = 64 x2 2xy + y2 = 4
(x + y)2 = 64 (x y)2 = 4
x+y = 8 xy =2
y nos quedan cuatro sistemas, de solucin casi inmediata,
que son:
x+y=8 x+y=8 x+y=8 x+y=8
xy=2 xy=2 xy=2 xy=2

y las soluciones son respectivamente:

(5, 3); (3, 5); ( 3, 5); ( 5, 3)

La solucin del sistema es la unin de todas ellas.


S = {(5, 3); (3, 5); ( 3, 5); ( 5, 3)}

2. Resolvamos el sistema: x2 + y2 = 15
xy =

Repitiendo el procedimiento anterior tenemos:

+
{ x2 + y2 = 15
2 xy = 12

{ x2 + y2 = 15
2 xy = 12

x2 + 2 xy + y2 = 27 x2 2xy + y2 = 3
(x + y)2 = 27 (x y)2 = 3
x+y= 27 = 3 3 xy = 3

256 Ecuaciones e inecuaciones de segundo grado

256-257.(2003) 256 20/11/02, 12:04 PM


CAPTULO 4

Los sistemas lineales asociados al sistema original son:


x+y=3 3 x+y =3 3 x+y =3 3 x+y= 3 3
xy= 3 xy = 3 xy = 3 xy = 3

y las soluciones son, respectivamente:

x1 = 2 3 x2 = 3 x3 = 3 x4 = 2 3

y1 = 3 y2 = 2 3 y3 = 2 3 y4 = 3

Y la solucin del sistema es:

S = {(2 3 , 3); ( 3, 2 3 ); ( 3, 2 3 ); ( 2 3 , 3)}

Observacin: Las soluciones de estos sistemas estn en el conjunto de


los nmeros reales, es decir, no se aceptan como soluciones races de
nmeros negativos.

3. Resolvamos el sistema: x2 + y2 = 4
y =

Si procedemos en forma anloga a los ejemplos anteriores, obtenemos:

+
{ x2 + y2 = 4
2 xy = 4

{ x 2 + y2 = 4
2 xy = 4

x2 + 2 xy + y2 = 0 x2 2xy + y2= 8
(x + y)2 = 0 (x y)2= 8

Y los sistemas asociados seran:

x+y = 0 x+y = 0
x y = 8 x y = 8

Y sus soluciones estaran dadas por:


8
x1 = 8 x2 =
2 2
8 8
y1 = y2 =
2 2
las cuales no son nmeros reales. Decimos entonces que el sistema
no tiene solucin en R.

Nota: Una simple inspeccin en el sistema original nos habra determinado


de inmediato la no existencia de solucin real del sistema, pues una
propiedad elemental de los nmeros reales es:

x2 0 I x E R.

Qu conclusin se obtiene?

Ecuaciones e inecuaciones de segundo grado 257

256-257.(2003) 257 20/11/02, 12:06 PM


Ejercicios

Resolver los siguientes sistemas:

1. x2 + y2 = 26 8. x2 + y2 = 97 14. x2 + y2 = 2
xy = 5 xy xy = 6
= 6
6

2. x2 + y2 = 20 15. x2 + y2 = 0
xy = 6 9. 5 x2 +5 y2 = 185 xy = 2
xy
= 2
3
3. x2 + y2 = 30 16. 3 x2 + y2 = 11
2xy = 18 xy = 0
10. x2 y2 = 61
xy
= 6
4. x2 + y2 = 85 5 17. x2 + y2 = 153
3xy = 54 xy
11. x2 + y2 = 6 = 9
4
xy
5. 2 x2 + 2 y2 = 50 = 8
3 18. x2 + y2 = 1
xy = 12
xy = 1
x2 + y2
12. = 15
6. x2 + y2 = 40 3
xy = 18
5 xy = 60

7. x2 + y2 = 58 13. 2x2 2 y2= 100


2 xy = 42 xy = 25

Soluciones
1. S = {(5, 1); (1, 5); ( 5, 1); ( 1, 5)}
2. S = {( 18 , 2 ); ( 18 , 2); ( 2, 18 ); ( 2, 18 )}
3. S = {( 3, 27 ); ( 3, 27 ); ( 27 , 3); ( 27 , 3)}
4. S = {(9, 2); ( 9, 2); (2, 9); ( 2, 9)}
5. S = {(3, 4); ( 3, 4); (4, 3); ( 4, 3)}
6. S = {(6, 2); ( 6, 2); (2, 6); ( 2, 6)}
7. S = {(7, 3); ( 7, 3); (3, 7); ( 3, 7)}
8. S = {(9, 4); ( 9, 4); (4, 9); ( 4, 9)}
9. S = {(1, 6); ( 1, 6); (6, 1); ( 6, 1)}
10. S = {(6, 5); (5, 6); ( 6, 5); ( 5, 6)}
11. S = 12. S = {(3, 6); (6, 3); ( 3, 6); ( 6, 3)} 13. S = {(5, 5); ( 5, 5)}
11 11
14. S = 15. S = 16. S = {(0, 11 ); (0, 11 ); ( , 0)}
3 , 0); ( 3
17. S = {(12, 3); ( 12, 3); (3, 12); ( 3, 12)} 18. S =

258 Ecuaciones e inecuaciones de segundo grado

258-259. 258 8/11/01, 15:50


CAPTULO 4

4.4.4 Sistemas homogneos formados por


ecuaciones cuyos trminos son todos
de segundo grado

(Es decir, contienen trminos x2, y2, xy)


Para resolver estos sistemas usamos el cambio de variable y = x, y as
reducimos el problema de resolver un sistema de segundo grado (con 2
variables) en una ecuacin de segundo grado (con una variable).

1. Resolvamos el sistema: x2 + y2 = 50 Ejercicios


x2 + xy = 56
resueltos
Hagamos el cambio de variable indicado: y = x,
y sustituyamos en ambas ecuaciones.
x2 + 2 x2 = 50
x2 + x2 = 56
Factorizando ambas ecuaciones por x2 y luego dividindolas
obtenemos:

:
{
x2 (1+ 2 ) = 50
x2 (1 + ) = 56
1 + 2 50
=
1+ 56
lo cual da origen a la siguiente ecuacin cuadrtica en la
variable :
56 + 562 = 50 + 50
56 50 + 6 =
2 0
o 28 2 25 + 3 = 0
resolviendo la ecuacin, obtenemos para las siguientes
soluciones:
3 1
1 = y 2 =
4 7
3
Para =
1 tenemos y = 3 x
4 4
sustituyendo en la segunda ecuacin (o en la primera),
3
obtenemos: x2 + x x = 56
4
4 x2 + 3 x2 = 224
7 x2 = 224
x2 = 32
x = 32 = 4 2
Si x1 = 4 2 Q y1 = 3 2 (y = 3 x)
4
Si x2 = 4 2 Q y2 = 3 2

Ecuaciones e inecuaciones de segundo grado 259

258-259. 259 8/11/01, 15:51


1 1
Ejercicios Para l2 =
7
tenemos y = x.
7
resueltos Sustituyendo en la 2a ecuacin, obtenemos:
1
x2 + x2 = 56
7
7x2 + x2 = 392
x2 = 49
x = 7
1
Si x3 = 7 y3 = 1 (y = x)
7
x4 = 7 y4 = 1
La solucin del sistema es, entonces:
S = {(4 2, 3 2) , ( 4 2, 3 2), (7, 1), ( 7, 1)}

2. Apliquemos el mismo procedimiento para resolver:

2x2 + y2 = 33
y2 xy = 15

Primero hacemos la sustitucin y = l x y reemplazamos:

2x2 + l2 x2 = 33
l2x2 lx2 = 15

Factorizamos por x2 y dividimos ambas ecuaciones:

x2 (2 + l2) = 33
x2 (l2 l) = 15
2 + 2
= 33
2 15
Formamos la ecuacin cuadrtica en la variable l:
30 + 15 l2 = 33 l2 33l
18 l2 33 l 30 = 0
6 l2 11 l 10 = 0
2
las soluciones son l1 = 5 y l2 =
2 3
Para l1 = 5 tenemos y= 5x
2 2
sustituyendo en la segunda ecuacin:
25 2 5 2
x x = 15
4 2
25x2 10 x2 = 60
15 x2 = 60
x2 = 4 x1 = 2
x2 = 2
x1 = 2 y1 = 5
x2 = 2 y2 = 5

260 Ecuaciones e inecuaciones de segundo grado

260-261. 260 20/11/02, 12:13 PM


CAPTULO 4

2 2
Para l2 = 3 tenemos: y= 3x

4 2 2 2
sustituyendo: 9
x + 3x = 15
4x2 + 6x2 = 135
10x2 = 135
3
135 27 x3 = 3 2
x2 = = 2 3
10
x4 = 3 2

3 3
x3 = 3 2 y3 = 2 2
3 3
x4 = 3 2 y4 = 2 2

y la solucin est dada por el conjunto:

{
S = (2, 5), ( 2, 5), 3 ( 3
2,2
3
)(
2 , 3
3
2,2
3
2 )}

Ejercicios
Resolver los siguientes sistemas: 5. 2x2 + 3y2 = 4
x2 xy = 12
1. x2 y2 = 7
xy = 12
6. x2 + xy + y2 = 26
x2 y2
2. x2 + y2 = 41 = 4
4
x2 xy = 4

7. 3x2 2y2 = 24
3. 2x2 y2 = 34
2xy + y2
xy = 6 = 14
6

4. x2 3y2 = 13 8. x2 2xy y2 = 41
xy = 10 x2 + 3xy y2 = 131

Soluciones

1. S = {(3, 4), ( 3, 4)} 2. S = {(4, 5), ( 4, 5) ( 0,5, 40,5); ( 0,5, 40,5)}

3. S = {(1, 6), ( 1, 6)} 4. S = {(5, 2), ( 5, 2)}

5. S = 6. S = {( 18 , 2), ( 18 , 2)}

7. S = {(4, 6), ( 4, 6)} 8. S = {(9, 2), ( 9, 2)}

Ecuaciones e inecuaciones de segundo grado 261

260-261. 261 20/11/02, 12:15 PM


4.4.5 Otros sistemas y problemas
Resolveremos aqu algunos problemas cuyo planteamiento
corresponde a un sistema de ecuaciones de segundo grado.
Hay oprtunidades en que las ecuaciones de 2 grado son la
expresin de dos rectas que se intersectan y por lo tanto la solucin
del sistema puede ser nica si el punto en que se intersectan las
cuatro rectas coincide; o vaco en el caso en que la cuatro rectas no
concurran en el mismo punto. (ver ejercicios 4 y 5)

1. La suma de dos nmeros es 11 y la suma de sus cuadrados es 65.


Ejercicios Determine dichos nmeros.
resueltos Sean x e y los nmeros pedidos. Tenemos entonces:
x + y = 11
x2 + y2 = 65
Resolviendo el sistema por sustitucin, nos queda: y = 11 x.
x2 + (11 x)2 = 65
x + 121 22x + x2
2
= 65
2x2 22x + 56 = 0
x2 11x + 28 = 0
las soluciones para x son; x1 = 7 y x2 = 4
Si x1 = 7 Q y1 = 4
x2 = 4 Q y2 = 7
y los nmeros pedidos son x = 7, y=4
(por la naturaleza del problema, no es necesario en este caso tomar
en cuenta la segunda solucin).
2. El permetro de un rectngulo es 20 cm y su rea mide 24 cm2.
Determine sus dimensiones.
Llamemos x al largo del rectngulo e y al ancho. Tenemos el
sistema:
2x + 2y = 20 cm (Permetro)
2
xy = 24 cm (rea)
Despejamos y de la primera ecuacin: y = (10 x) cm
y lo reemplazamos en la segunda:
x (10 x) = 24
10x x2 = 24 o x2 10x + 24 = 0
las soluciones para x son; x1 = 6 y x2 = 4
Si x = 6 Q y = 4
x=4 Q y=6
Por lo tanto las dimensiones del rectngulo son 6 cm de largo
y 4 cm de ancho.
7
3. La suma de dos nmeros es 10 y la suma de sus recprocos es 7.
Determine dichos nmeros.

262 Ecuaciones e inecuaciones de segundo grado

262-263. 262 10/11/2001, 14:28


CAPTULO 4

Sean x e y los nmeros. Entonces:


x + y = 7
10
1 1
x + y = 7

Procediendo algebraicamente tenemos:


7
x+y= 7 Q x+y= Q
10 10
x+y
xy = 7 x+y=7xy

7 7
x+y= Q y= x
10 10
1
xy =
10

7 1 7 1
x 10 x = Q x x2 = Q 7x 10x2 = 1
10 10 10

10x2 7x + 1 = 0 x1 = 1
2
x2 = 1
5

1 7 1 1
Si x1 = entonces y1 = = 5
2 10 2
7 1 1
x2 = 1 entonces y2 = =
5 10 5 2
1
y los nmeros pedidos son; x = 1 e y = 5
2

4. Resolvamos el sistema:

x2 + 4y2 = 0 (1)

9x2 + 16y2 = 0 (2)

Algebraicamente vemos que en (1) x2 = 4y2 y reemplazando en (2):


9 4y2 16y2 = 0
32y2 16y2 = 0
16y2 = 0
y2 = 0
y =0

En (1) x2 4 0 =0
x2 = 0
x =0

Por lo tanto, la nica solucin es (0, 0).

Ecuaciones e inecuaciones de segundo grado 263

262-263. 263 10/11/2001, 14:29


Analticamente vemos que:
Ejercicios (1): x2 4y2 = 0 =
0
4y
resueltos (x + 2y) (x 2y) = 0 3x y = 0
3 2
representa las rectas x
x + 2y = 0 y x 2y = 0 1
4 2 2 4
x+
(2): 9x2 16y2 = 0 3x 2y = 0
(3x + 4y) (3x 2y) = 0 +4
y=
representa las rectas 0
3x + 4y = 0 y 3x 4y = 0

Las cuatro rectas se intersectan en el punto (0, 0)

5. Resolvamos el sistema:

4x2 y2 + 6y 9 = 0 (1)
x2 4y2 + 24y 36 = 0 (2)

Observamos que para eliminar y debemos amplificar la primera


ecuacin por 4 y sumamos:

16x2 + 4y2 24y + 36 = 0 (1)


x2 4y2 + 24y 36 = 0 (2)

15x2 = 0
x2 = 0
x=0

Sustituyendo x por su valor 0, en cualquiera de las ecuaciones del


sistema obtenemos que y = 3
Luego, la nica solucin del sistema es (0, 3)

Otra forma: Analticamente vemos que si


despejamos y en la primera ecuacin:

y2 6y + 9 4x2 = 0

6+ 36 4(9 4x2) 6+ 36 36 + 16x2)


y= =
2 2
+ y1 = 3 + 2x
= 6 4x
2 y2 = 3 2x
ecuaciones que corresponden a dos rectas que se intersectan
en el punto (0, 3).

En forma anloga, podemos ver que la segunda ecuacin,


x2 4y2 + 24y 36 = 0 corresponde a las rectas
y = 1 x + 3 y y = 1 x + 3 , las cuales tambin se
2 2
intersectan en el punto (0, 3).

264 Ecuaciones e inecuaciones de segundo grado

264-265. 264 8/11/01, 16:07


CAPTULO 4

Ejercicios

1. La suma de dos nmeros es 12 y la diferencia de sus


cuadrados es 24. Determine dichos nmeros.

2. La diferencia de dos nmeros es 5 y la suma de sus


cuadrados es 97. Determine dichos nmeros.

3. La diferencia de dos nmeros es 5 y la diferencia de sus


cuadrados es 55. Determnelos.

4. La suma de dos nmeros es 25 y la diferencia de sus


cuadrados es 25. Determnelos.

5. El cuadrado de la suma de dos nmeros es 100 y el producto


de ellos es 24. Cules son?

6. El cuadrado de la suma de dos nmeros es 225 y su cociente


es 4. Cules son esos nmeros?

7. El cuadrado de la diferencia de dos nmeros es 25 y el


producto de ellos es 36. Cules son?

8. Dos nmeros estn en la razn 1:3 y la diferencia de sus


cuadrados es 200. Cules son?

9. Dos nmeros estn en la razn 2:3 y el triple del cuadrado


del primero menos el cuadrado del segundo es 27. Cules
son los nmeros?

10. La suma del cuadrado de un nmero ms el quntuple del


cuadrado de otro es 49, y la diferencia entre el triple del
cuadrado del primero y el cuadrado del segundo es 3.
Cules son los nmeros?

11. El doble del cuadrado de un nmero menos el triple del


cuadrado de otro es 23 y el producto de ambos es 15.
Determine dichos nmeros.

12. Determine las dimensiones de un rectngulo sabiendo que


su permetro es 40 cm y su rea mide 91 cm2.

13. Determine las dimensiones de un rectngulo sabiendo que el


largo es 4 veces el ancho y que el rea mide 25 cm2.

14. Cules son las dimensiones de un rectngulo cuyos lados


estn en la razn 1:3 y cuya superficie mide 6,75 cm2?

Ecuaciones e inecuaciones de segundo grado 265

264-265. 265 8/11/01, 16:07


Ejercicios
15. El rea de un tringulo rectngulo mide 10 cm2 y la hipotenusa mide 41 cm.
Determine la medida de los catetos.
16. La suma de las reas de dos cuadrados es 74 cm2 y la diferencia de sus permetros
es 8 cm. Determine el lado de cada uno.
17. Las reas de dos crculos estn en la razn 1:4 y sus radios suman 9 cm.
Determine los radios.
18. Los catetos de un tringulo rectngulo estn en la razn 5:3 y su superficie mide
120 cm2. Determine la medida de sus 3 lados.
19. Resolver los siguientes sistemas:
a) 9x2 y2 + 2y 4 = 0
4x2 4y2 + 16y + 16 = 0

b) x2 4y2 + 2x + 16y 15 = 0
2x2 y2 + 4x + 4y 2 = 0

c) (x + 1)2 2y 2 = 8
2 6 9
2x2 + 4x + 2 y 1 = 2
3 5

20. Una piscina rectangular cuyas dimensiones son 5 por 10 metros tiene un borde de
ancho uniforme. Si el rea del borde es 16 m2, calcule el ancho del borde.

Soluciones

1. x = 7 , y = 5 2. (x1 = 4 , y1 = 9); (x2 = 9 , y2 = 4)


3. x = 8 , y = 3 4. x = 13 , y = 12
5. (x1 = 4 , y1 = 6); (x2 = 4 , y2 = 6) 6. (x =12 , y = 3); (x2 = 12; y2 = 3)
7. (x1 = 4 , y1 = 9); (x2 = 4 , y2 = 9) 8. (x1 = 5 , y1 = 15); (x2 = 5 , y2 = 15)
9. (x1 = 6 , y1 = 9); (x2 = 6 , y2 = 9) 10. (2, 3), ( 2, 3), ( 2, 3), (2, 3)
11. (5, 3) , ( 5, 3) 12. 7 cm y 13 cm
13. 2,5 cm y 10 cm 14. 1,5 cm y 4,5 cm
15. a = 4 cm , b = 5 cm 16. a = 7 cm , b = 5 cm
17. r1 = 3 cm , r2 = 6 cm 18. a = 20 cm , b = 12 cm
19. a) (0, 2) b) (1, 2) c) (1, 13 ) (3, 13 ) 20. 50 cm
3 3

266 Ecuaciones e inecuaciones de segundo grado

266-267. 266 20/11/02, 12:17 PM


CAPITULO 4
CAPTULO

Prueba de seleccin mltiple


1. La suma y el producto sus races sean complejas 9. La interseccin de la
de las races de la ecua- conjugadas? parbola cuya ecuacin
cin ax2 + x + c = 0 son es y = 2x2 + 3x 2 con el
9
respectivamente: A. k= eje x es en los puntos.
40
1 c
A. y
a
B. 1 y
a
c
B. k<
9
40
A. ( 12 , 2)
a a 9
B. ( , 0) y ( 2, 0)
C. k < 1
1 c 40
C. y a 2
a
c 9
C. (0, ) y (0, 2)
D. x
y D. k > 1
a a 40
2
c 9
E. x y E. k>
D. (0, ) y (0, 2)
a a 40 1
6. La ecuacin cuyas races 2
2. Las soluciones de la ecua-
son 0 y 2 es:
cin x2 + x 20 = 0 son:
A. x2 2 = 0
E. (
1 ,0
2 ) y (2, 0)
A. 5 y 4 10. El vrtice de la parbola
B. x2 + 2 = 0
B. 5 y4 C. x2 2x = 0 cuya ecuacin es
C. 4 y5 D. x2 + 4x = 0 y = x2 2x 24 tiene por
D. 4 y 5 E. x2 + 2x = 0 coordenadas:
E. 10 y2 A. (1, 25)
7. Una de las races de la
B. (1, 25)
ecuacin
3. La ecuacin cuyas races C. ( 1, 25)
ax2 2x 3 = 0
son x1 = 4 y x2 = 6 D. ( 1, 25)
es: 3
es: E. (0, 24)
Cul es el valor de a?
A. x2 4x 6 = 0 11. La funcin
B. x2 + 2x + 24 = 0 1 y = x2 + 2x + 15
A.
9 alcanza su mximo valor
C. x2 2x + 24 = 0
1 para:
D. x2 + 2x 24 = 0 B.
9
E. x2 2x 24 = 0 A. x = 5
C. 1 B. x = 3
3
4. Para que las races de la 1 C. x = 1
D. D. x = 1
ecuacin 4x2 + 12x k = 0 3
sean reales e iguales el valor E. No se puede deter- E. x = 5
de k debe ser: minar.
12. La solucin de la inecua-
A. 9 8. El producto de las races cin x2 2x > 0 est
B. 9 de la ecuacin representada por:
C. 36 2ax2 + 3abx + 4ab2 = 0 es:
A.
A. 3 b
0 2
D. 6
2 B. 0 2
E. 6
B. 2b2 C. 0 2
5. Qu condicin debe C. 4ab2
cumplir k en la ecuacin D. 2b2 D.
0 2
2kx2 + 3x + 5 = 0 para que E. 4ab E.
0 2

Ecuaciones e inecuaciones de segundo grado 267

266-267. 267 20/11/02, 12:19 PM


Prueba de seleccin mltiple
13. La solucin de la ine- C. x = 3 y = 3 A. 2
cuacin x2 5x + 6 0 D. x = 3 y = 3 B. 2
est representada por: E. x = 6 y = 0 C. 5
D. 7
A. 2 3 18. Dado el sistema:
E. 7
B. x+y=9
2 3
xy=1 22. x = 3 es solucin de la
C.
2 3 ecuacin x2 9 = 0. La
D. el valor de 2x es: otra solucin es:
2 3

E. 3 2
A. 10 A. 9
B. 5 B. 9
14. La solucin de la ine-
C. 8 C. 3
cuacin x2 1 0 es:
D. 4 D. 3
A. x 1 E. otro. E. 0
B. x 1
19. Dado el sistema: 23. La suma de las solucio-
C. ] , 1] K [1, [ 2x + y = 7 nes de la ecuacin
D. [ 1, 1] x+y=4 2x2 + 5x 1 = 0 es:
E. ] 1, 1[ 1
el valor de x es: A. 5
15. El conjunto [ 3, 3] es A. 3 1
solucin de la inecuacin: B.
B. 3 2
C. 1 1
A. x2 + 9 < 0 C. 2
D. 1
B. x2 9 0 5
E. 4 D. 2
C. x2 6x + 9 0
20. Si x2 + y2 = 26 5
D. x2 9 0 E.
2
x+y =6
E. x2 + 6x + 9 < 0 24. El producto de las solu-
ciones de la ecuacin
entonces son soluciones
16. La solucin de la inecuacin x2 + ax + b = 0 es:
del sistema:
x2 4 < 0 est dada por:
I. x = 5, y = 1 A. a
A. [ 2, 2] II. x = 1, y = 5 B. b
B. ] , 2[ III. x = y = 5 C. a
C. ] , 2[ K ]2, [ D. b
Son verdaderas:
D. ] 2, 2[ A. Slo I E. b
a
E. ] 2, [ B. Slo II
C. I y II 25. La condicin para que
17. Una solucin del sistema las soluciones de la
D. I y III
ecuacin
3x + y = 6 E. Todas.
kx2 + 3x + 2 = 0 sean
x2 + y2 = 18 21. Si x = 5 es solucin de complejas conjugadas es:
la ecuacin 9
es: A. k >
8
x2 7x + k = 0
A. x = 3 y=3 8
entonces la otra solu- B. k >
B. x = 3 y=3 9
cin es:

268 Ecuaciones e inecuaciones de segundo grado

268-269. 268 8/11/01, 16:14


CAPTULO 4

9 A. 0 C. 42 cm
C. k <
8 D. 56 cm
B. 1
9 C. 1 E. 70 cm
D. k <
8
3
34. El rea de un tringulo
8 D. rectngulo es 24 cm2
E. k < 2
9
1 y la hipotenusa mide
26. Para que las soluciones E.
2 10 cm. Cul es el per-
de la ecuacin
30. Una de las races de la metro?
12x2 + kx + 3 = 0 ecuacin A. 24 cm
sean iguales se debe 2x2 + 17x 9 = 0 B. 34 cm
cumplir: es 9 C. 40 cm
A. k > 12 Cul es la otra raz? D. 60 cm
B. k < 12 A. 9 E. 30 cm
C. k > 12 B. 2 35. El permetro de un rec-
D. k < 12 C. 2 tngulo es 28 cm y su
E. k = 12 1 rea mide 33 cm2. El
D.
2 lado menor mide:
27. La suma y el producto
de las races de una E. 1 A. 11 cm
2
ecuacin cuadrtica son B. 5 cm
31. La suma de dos nmeros
3 y 10 respectivamente. C. 3 cm
es 21 y su producto es
La ecuacin es: D. 6 cm
90. Cul es el nmero
A. x2 3x 10 = 0 mayor? E. 7 cm
B. x2 3x + 10 = 0 A. 15 36. La suma de dos nmeros
C. x2 + 3x 10 = 0 B. 18 es 28 y la diferencia de
D. x2 3x + 10 = 0 C. 9 sus cuadrados es 56. La
E. x2 + 3x + 10 = 0 diferencia de ellos es:
D. 6
E. 12 A. 2
28. Las races de una ecua-
cin de segundo grado B. 1
32. Dos nmeros estn en la
estn en la razn 3:1 y C. 4
razn 3:2 y la diferen-
son ambas positivas. Si D. 8
cia de sus cuadrados es
la ecuacin es: E. 6
20. Cul es el nmero
x2 + ax + 12 = 0 mayor? 37. La funcin que repre-
el valor de "a" es: A. 4 senta la curva dada es:
A. 2 B. 6
B. 4 C. 8
C. 8 D. 10
D. 8 E. 2
E. no se puede deter-
. 2
33. La superficie de una
minar. jaula rectangular es de
29. Qu valor debe tener k 48 cm 2 . Si los lados A. y = x2 + 2
en la funcin estn en la razn 3:4. B. y = x2 2
y= 2x2 3x + k 1 Cul es su permetro?
C. x = y2 + 2
para que el punto (0, 0) A. 14 cm D. x = y2 2
pertenezca a ella? B. 28 cm E. y = x2 2
Ecuaciones e inecuaciones de segundo grado 269

268-269. 269 8/11/01, 16:14


Prueba de seleccin mltiple
38. A partir del siguiente 41. La funcin asociada al 45. La funcin cuya grfica
grfico, podemos afir- grfico es: es la siguiente cumple las
mar que la ecuacin siguientes condiciones:
cuadrtica asociada:
3

1 3

A. y = x2 2x + 3 A. = 0 a>0
B. y = x2 2x 3 B. = 0 a<0
A. Tiene solucin ima- C. y = x2 + 2x + 3 C. = 0 a=0
ginaria. D. y = x2 + 2x 3 D. > 0 a>0
E. y = x2 + 2x + 3 E. < 0 a<0
B. Tiene una raz nega-
tiva. 42. El recorrido de la funcin 46. La grfica de la funcin
C. Tiene races reales y = 16x2 1 es: y = 3x2 2x intersecta
iguales. A. ] , 1] al eje x en:
D. Tiene races reales y B. ] , 1] A. 0 y 2
distintas. C. [1, [
B. 0 y 3
D. [ 1, +[
E. No tiene solucin. 2
E. [ 1, 1] C. 0 y
3
3
39. La grfica de la funcin D. 0 y
43. El recorrido de la fun- 2
cuadrtica cin E. 0 y 2
y = 3x2 2x 5 inter- y = x2 + 2x + 15 es: 3
secta al eje y en: 47. La grfica de la funcin
A. [16, + [
y = x2 x + 1 intersecta
A. 3 B. [ 16, + [ al eje x en:
C. ] , 16] A. x = 1
B. 2
D. ] , 16] B. x = 0
C. 2
E. [ 16, 16]
C. x = 1
D. 5
44. La funcin cuya grfica D. x = 2
E. 5 es la siguiente cumple las
E. No lo intersecta.
siguientes condiciones:
40. La grfica de la funcin 48. Las coordenadas del
y = 3x2 8x 3 vrtice de la parbola
intersecta al eje x en: cuya funcin es
y = 9x2 + 6x 8 son:
1
A. 3 y 3
1
A. ( , 9)
1
3
B. 3 y 3
1 A. > 0; a>0
B. ( , 9)
1
3
C. 3 y 3
1
B. = 0; a<0 C. ( , 9)
1
3
D. 3 y 3
E. 3 y 3
C.
D.
> 0;
< 0;
a<0
a<0
D. ( , 9)
1
3
E. = 0; a>0 ,
E. ( 3 9)

270 Ecuaciones e inecuaciones de segundo grado

270-271. 270 20/11/02, 12:21 PM


CAPITULO 4
CAPTULO

Soluciones

1. B 7. C 13. A 19. B 25. A 31. A 37. B 43. D


2. A 8. D 14. C 20. C 26. E 32. B 38. C 44. C
3. D 9. B 15. D 21. A 27. A 33. B 39. D 45. A
4. B 10. A 16. D 22. C 28. D 34. A 40. A 46. C
5. E 11. D 17. A 23. E 29. B 35. C 41. C 47. E
6. E 12. C 18. A 24. B 30. D 36. A 42. D 48. D

Ecuaciones e inecuaciones de segundo grado 271

270-271. 271 20/11/02, 12:22 PM


CAPTULO 5
P
olinomios
y Teora de
ecuaciones

5.1 Denicin y
operaciones con polinomios

Definicin: Decimos que P es un polinomio en el conjunto de


los nmeros reales R si y slo si P es una funcin de R en R, tal
que P(x) admite una representacin de la forma

P(x) = an xn + an 1 x n 1 + + a2 x2 + a1 x + a0

donde an, an 1, a2, a1, a0 E R

y se denominan coeficientes del polimonio, y n E N

Ejemplo: P(x) = 2 x2 3x + 1
Q(x) = 5x3 2x2 + x 1
R(x) = x5 3x + 2
C(x) = 4x2 1
son polinomios.

Definicin: Llamamos grado de un polinomio P al mayor


exponente que presenta la variable (frecuentemente x) con
coeficiente distinto de cero.

En el ejemplo anterior:
grado P (x) =2
grado Q (x) =3
grado R (x) =5
grado C (x) =2

272 Polinomios y Teora de ecuaciones

272-273. 272 20/11/02, 12:48 PM


CAPTULO 5

Observacin 1: Un nmero real puede entenderse como un


polinomio de grado 0.
Observacin 2: Al coeficiente del trmino de mayor grado del
polinomio se le llama coeficiente principal.
Observacin 3: Dos polinomios P(x) y Q(x) son iguales si y slo
si Ix E R, P(x) = Q(x).

OPERACIONES CON POLINOMIOS.


Sean P(x) = an xn + am xm +...+ a2 x2 + a1 x + a0
y Q(x) = bm xm + .....+ b2 x2 + b1 x + b0

dos polinomios de grados n y m, E N, con n > m.


Estos polinomios podemos escribirlos usando sumatoria
n m
P(x) = a ix i ;
i= 0

Q(x) = i = 0 b i x
i

5.1.1 Suma. (Ver problema 2, pg. 275)


P(x) + Q(x) = anxn + + (am + bm) xm + + (a2 + b2)x2 +
(a1 + b1) x + (a0 + b0)
o bien, usando sumatoria:
n
P(x) + Q(x) =
i= 0
ai + bi x i

5.1.2 Resta. (Ver problema 2, pg. 275)


Previo:
(1) P(x) = anxn amxm a2 x2 a1x a0
luego
P(x) Q(x) = P(x) + (1) Q(x)

5.1.3 Producto. (Ver problema 3, pg. 275)


P(x) Q(x) = este producto se obtiene aplicando la propiedad distributiva
del producto sobre la suma en forma reiterada y considerando las leyes
de multiplicacin de potencias de igual base.
m+n i
P(x) Q(x) = ci x i,
i= 0
donde ci = a k bik
k=0

Teorema 1: El conjunto de los polinomios con coeficientes reales


para la suma y el producto de polinomios tiene una estructura
de anillo:

Polinomios y Teora de ecuaciones 273

272-273. 273 20/11/02, 12:49 PM


Es decir:

{
+ es cerrada
+ es asociativa
(P, +) es grupo abeliano + es conmutativa
+ tiene elemento neutro
+ tiene elemento inverso
es asociativa
es distributiva sobre la suma.

Teorema 2. Sean P(x) y Q(x) dos polinomios con coeficientes


reales.
Si P(x) Q(x) = 0 I x E R,
entonces P(x) = 0 V Q(x) = 0

Teorema 3. Sean P(x), Q(x) y R(x) tres polinomios en R, tales


que P(x) 0.

Si P(x) Q(x) = P(x) R(x) I x E R,


entonces Q(x) = R(x).

5.1.4 Divisin. (Problemas 4 y 5, pg. 276)


En toda divisin de dos polinomios se verifica que
Dividendo = Divisor cociente + resto.
P(x) = Q(x) C(x) + R(x),
lo que tambin se puede escribir como
P(x) R(x)
= C(x) +
Q(x) Q(x)
Observacin 1: Para efectuar la divisin de P(x) por Q(x), Q(x) debe
tener grado inferior o igual a P(x).
Observacin 2: La divisin de polinomios se termina cuando el
resto es de grado inferior al divisor.
Regla de Ruffini o divisin sinttica: Para efectuar la divisin de
polinomios por un divisor de la forma x a es posible trabajar slo
con los coeficientes y mediante este procedimiento determinar el
cociente y el resto. Ver problema 7, pg. 277.
Teorema 4. (Teorema del Resto) Al dividir un polinomio P(x) por
(x a), el resto es P(a).
Demostracin: Recordemos que al dividir un polinomio se obtiene un
cociente C(x) y un resto R(x), donde el resto tiene grado inferior que
el grado del divisor. En este caso el grado del divisor (x a) es 1, por
lo tanto el grado del resto es cero, es decir, el resto es una constante
real que denotaremos por r, R(x) = r.
P(x) r
x a = C(x) + x a o bien
P(x) = C(x) (x a) + r

274 Polinomios y Teora de ecuaciones

274-275. 274 20/11/02, 12:50 PM


CAPTULO 5

Si hacemos x = a nos queda P(a) = C(a) 0 + r

P(a) = r.

Es decir, al dividir P(x) por (x a) el resto est expresado por P(a).

1. Dado el polinomio P(x) = 5x4 3x2 + x 3


Ejercicios
determinar su grado y hallar su valor para x = 1, x = 1 y x = 0. resueltos
Solucin:

El grado de P(x) es 4.

Como P(x) = 5x4 3x2 + x 3


Se tiene que P(1) = 5 14 3 12 + 1 3
= 53+13=0
P(1) = 5 (1)4 3 (1)2 + (1) 3
= 5 3 1 3 = 2
P(0) = 5 04 3 02 + 0 3 = 3

Observar que el valor de un polinomio en x = 0 es equivalente al trmino


independiente de la variable que presente el polinomio.

2. Dados los polinomios P(x) = 6x5 3x2 + x 2


y Q(x) = x4 x3 + 2x2 x + 3.
Hallar P(x) + Q(x); P(x) Q(x).

Solucin:

P(x) + Q(x) = (6x5 3x2 + x 2) + (x4 x3 + 2x2 x + 3)


= 6x5 + x4 x3 x2 + 1
P(x) Q(x) = P(x) + (1) Q(x)
= (6x5 3x2 + x 2) + (x4 + x3 2x2 + x 3)
= 6x5 x4 + x3 5x2 + 2x 5

Observar que para obtener (1) Q(x) se cambia el signo de todos


los trminos de Q(x).

3. Dados los polinomios P(x) = x3 x2 + 2x 1


y Q(x) = x2 x + 3.
Hallar P(x) Q(x).

Solucin:

Para obtener el producto de ambos polinomios debemos multiplicar


cada trmino de P(x) por cada trmino de Q(x) y luego reducir
trminos semejantes.

Polinomios y Teora de ecuaciones 275

274-275. 275 20/11/02, 12:50 PM


Ejercicios P(x) Q(x) = (x3 x2 + 2x 1) (x2 x + 3)
resueltos = x5 x4 + 2x3 x2 x4 + x3 2x2 + x + 3x3 3x2 + 6x 3

= x5 2x4 + 6x3 6x2 + 7x 3

4. Si P(x) = 2x4 x3 + 3x 1
y Q(x) = x2 x + 3
P(x)
Hallar el cociente C(x) = y el resto R(x)
Solucin: Q(x)
Para realizar una divisin de polinomios, primero se deben ordenar
dividendo y divisor de acuerdo al grado de sus trminos. Seguidamente
buscamos el primer trmino del cociente, que es el que multiplicado
por el primer trmino del divisor da el primer trmino del dividendo.
Este trmino se multiplica por cada trmino del divisor y ese producto
se resta del dividendo.
Se obtiene as un nuevo polinomio, el cual se divide nuevamente
por el divisor, con el mismo procedimiento, obteniendo el segundo
trmino del cociente. Anlogamente se obtienen los dems trminos
del cociente.
La divisin se termina cuando al restar aparece un polinomio cuyo grado
es inferior al grado del divisor.

(2x4 x3 + 3x 1) : (x2 x + 3) = 2x2 + x 5


) 2x4 2x3+ 6x2
x3 6x2 + 3x 1
) x3 6x2 + 3x
5x2 1
) 5x2 + 5x 15
5x + 14
2
Luego C(x) = 2x + x 5 es el cociente y
R(x) = 5x + 14 es el resto.

5. Sea P(x) = x6 64
y Q(x) = x3 8
P(x)
Hallar el cociente C(x) = y el resto R(x).
Q(x)
Solucin: (Ver explicacin ejercicio anterior)

(x6 64) : (x3 8) = x3 + 8


) x6 8x3
8x3 64
) 8x3 64
0 0
C(x) = x3 + 8
R(x) = 0

276 Polinomios y Teora de ecuaciones

276-277. 276 20/11/02, 12:51 PM


CAPTULO 5

6. Realizar el cociente entre x5 243 y x 3

Solucin: (Ver explicacin ejercicio 4)


(x5 243) : (x 3) = x4 + 3x3 + 9x2 + 27x + 81
) x5 3x4
3x4 243
) 3x4 9x3
9x3 243
) 9x3 27x2
27x2 243
) 27x2 81x
81x 243
) 81x 243
0 0

Luego (x5 243) : (x 3) = x4 + 3x3 + 9x2 + 27x + 81

7. Efectuar el cociente entre P(x) = 6x4 10x3 4x2 3x + 6


y Q(x) = x 2.
Solucin:
Efectuaremos la divisin aplicando la regla de Ruffini o divisin
sinttica.
Copiamos los coeficientes del dividendo teniendo especial cuidado
de anotar 0 si algn trmino no aparece.
Copiamos al margen derecho el trmino independiente del divisor
con signo contrario (2).
Al dividir un polinomio por (x a) el cociente tendr como grado
una unidad inferior al dividendo y el coeficiente principal del cociente
ser el mismo del dividendo. Copiamos, segn muestra el diagrama el
coeficiente principal (6) bajo la lnea horizontal.
6 10 4 3 6 2

6
Este nmero (6) es el coeficiente principal del cociente. Ahora este
coeficiente principal se multiplica por el factor (2) que est al margen y
su resultado se escribe sobre la lnea horizontal bajo el coeficiente del
segundo trmino del dividendo. Se suman estos dos nmeros obteniendo
as el coeficiente del segundo trmino del cociente (2).
6 10 4 3 6 2
12

6 2
Este nmero (2) se multiplica por el factor (2) que est al margen
y se procede en la misma forma anterior hasta llegar al ltimo
nmero (0).

Polinomios y Teora de ecuaciones 277

276-277. 277 20/11/02, 12:52 PM


Ejercicios 6 10 4 3 6 2
resueltos 12 4 0 6
6 2 0 3 0

Bajo la lnea horizontal se obtienen los coeficientes del cociente, cuyo


grado es inferior en una unidad al grado del dividendo y el ltimo nmero
representa el resto de la divisin. En este caso es 0.

As, el cociente es 6x3 + 2x2 3 y el resto es 0.

8. Efectuar la divisin entre los siguientes polinomios:


P(x) = 5x5 3x2 + 6x + 12 y Q(x) = x + 1
Solucin:
Usaremos divisin sinttica (ver explicaciones del ejercicio anterior).
Dividendo P(x) = 5x5 3x2 + 6x + 12
Divisor Q(x) = x + 1
Coeficientes del dividendo

5 0 0 3 6 12 1 O trmino
independiente del
5 5 5 8 14 divisor con signo
5 5 5 8 14 2 contrario.

X
coeficientes del cociente resto

Luego cociente C (x) = 5x4 5x3 + 5x2 8x + 14


y el resto R (x) = 2

Observar que el resto es independiente de x pues su grado debe ser


menor que el del divisor que en este caso es uno.

9. Determinar el resto que se produce al dividir


P(x) = x6 3x2 + 2x 5 por x 1
Solucin:
Para hallar la solucin basta con evaluar P(1). (Solucin de la ecuacin
x 1 = 0, divisor igual a cero).
P(1) = 16 3(1)2 + 2(1) 5
=13+25=5

10. Sean P(x) = x3 ax2 + x b


y Q(x) = x3 2x2 + ax + b
dos polinomios. Determinar a y b para que P(x) + 2 sea divisible por
x 1 y Q(x) 3 sea divisible por x + 1.
Solucin:
P(x) + 2 = x3 ax2 + x b + 2
Dividiendo P(x) + 2 por x 1:

278 Polinomios y Teora de ecuaciones

278-279. 278 20/11/02, 12:53 PM


CAPTULO 5

1 a 1 b + 2 1
1 1a 2a
1 1a 2a ab+4

ab+4=0 (1) El resto debe ser cero.

Q(x) 3 = x3 2x2 + ax + b 3

Dividiendo Q(x) 3 por x + 1

1 2 a b3 1
1 3 a3
1 3 a+3 a+b6

a+b6=0 (2) El resto debe ser cero.

Resolviendo el sistema (1) y (2).


a+b=+4
a=1yb=5
ab=6

Ejercicios
1. Dados los siguientes polinomios, determine su valor para el nmero
real indicado:
a) P(x) = 5x6 3x4 + 2x2 1 x=2
b) P(x) = x5 2x4 + x3 + x2 3x 1 x = 1
c) P(x) = 4x4 3x + 4 x=0
1
d) P(x) = 36x6 2x5 x 3 x=
2
e) P(x) = 2x4 x3 + 3x2 x + 2 x=4
f) P(x) = x3 x + 2 x = 3
g) P(x) = 3x3 + x2 + 5x x = 2

h) P(x) = 2x2 x + 5 x= 5
1 2
i) P(x) = x4 x2 + x 1 x= 2
2 3
j) P(x) = x5 x4 x3 + x2 + x 1 x=1
k) P(x) = x5 x4 + x3 x2 + x 1 x = 1
l) P(x) = x5 + x4 + x3 + x2 + x + 1 x=1
m) P(x) = x5 + x4 + x3 + x2 + x + 1 x = 1

Polinomios y Teora de ecuaciones 279

278-279. 279 20/11/02, 12:53 PM


Ejercicios
2. Dados los polinomios P(x) = x2 + 2x 1 y Q(x) = x3 x + 2
Determine:
a) P(x) + Q(x) b) P(x) Q(x) c) 2P(x) 3 Q(x)
d) P(x) Q(x) e) P(x) + 2Q(x)

3. Dados los polinomios P(x) = x3 2x2 + x 2 y Q(x) = x 3


Encuentre:
a) P(x) + Q(x) b) P(x) Q(x) c) P(x) + [Q(x)]2
d) P(x) + x Q(x) e) P(x) + P(x) Q(x)
4. Dar ejemplos de sumas y productos de polinomios donde se verifiquen
todas las propiedades del teorema 1.

5. Si P(x) = 3x3 2x2 + x 5 Q(x) = x3 + 3x


R(x) = x2 x + 2 S(x) = x3 + 2x2 3.
Determine:
a) [P(x) + Q(x)] [R(x) + S(x)]
b) P(x) [Q(x) + R(x) + S(x)]
c) [P(x) Q(x)] + [R(x) S(x)]
6. Realice la divisin de los siguientes polinomios; obtenga cociente
C(x) y resto R(x).
a) (5x3 + 2x 3) : (x2 x + 1) =
b) (x5 2x4 + x3 3x2 + 5) : (2x3 2x2 + x 1) =
c) (4x5 + x3 6x + 1) : (x3 2x + 3) =
d) (x4 3x3 + 2x2 x + 4) : (x2 2x + 1) =
e) (x5 + x4 4x3 + 4x2 5x + 3) : (x2 + 2x 3) =
f) (2x 6 2x 5 + x 4 + 4x 3 6x 2 + 6x 5) : (2x 3 x + 5) =
g) (x7 3x6 + 2x4 + 2x3 8x2 + 3x 3) : (x3 3x2 + x 1) =
h) (x7 3x5 + x4 2) : (x5 + x2 3) =
7. Realice la divisin de los siguientes polinomios obteniendo cociente
C(x) y resto R(x).
a) (x4 14x2 + 17x 6) : (x 3)
b) (x3 + 3x2 5x + 2) : (x 1)
c) (x5 + 3x4 + 2x3 3x2 7x 2) : (x + 2)
d) (x4 + x3 3x 1) : (x + 1)
e) (x6 + 3x5 2x4 5x3 + 2x2 3x) : (x + 3)

280 Polinomios y Teora de ecuaciones

280-281. 280 20/11/02, 12:54 PM


CAPTULO 5

f) (x5 2x3 + x2 x) : (x 1) =
g) (x4 + x3 x2 ) : (x2) =
h) (x5 2x3 + x2 x) : (x + 1) =
i) (x5 3x4 4x3 3x + 12) : (x 4) =
j) (x5 2x4 4x3 3x + 12) : (x 3) =
k) (x5 3x4 4x3 3x + 12) : (x 1) =
l) (x4 2x3 34x2 + 4x 5) : (x + 5) =
m) (x3 3x2 + x 1) : (x 1) =
8. Encuentre el resto que se produce al dividir cada uno de los
polinomios dados por x + 3.
a) x4 2x3 + x2 1
b) 3x5 x + 6
c) x3 + x2 8
d) 3x3 + 4x2 9
9. Determine el resto que se produce al realizar las siguientes
divisiones.
a) x5 2x + 3 por x+1
b) x4 x3 + 2x2 x + 3 por x1
c) 4x3 2x2 + x 5 por x+2
d) 2x2 4x + 6 por x3
10. Qu valor debe tomar a, para que al dividir x4 6x2 + x a por
x 2, el resto sea cero?

11. Qu valor debe tomar a, para que al dividir x3 4x2 + ax 3 por


x + 2, la divisin sea exacta?

12. Qu valor debe tomar a, para que al dividir 3x4 x3 + 2x2 ax + 1


por x 1, la divisin d como resto 5?

13. Encuentre un polinomio P(x) tal que dividido por x2 x + 1 d como


cociente x + 2 y como resto x 3.

14. Determine los valores de a, b y c para que


ax2 + bx c
=3 IxER
3x2 + 5x 1
15. Determine el valor de a E R de modo que los polinomios
P(x)= x4 2 ax3 + 3x2 2 ax + 1 y
Q(x) = x2 x + 1
verifiquen la condicin P(x) = [Q(x)]2

16. Determine un polinomio P de segundo grado de modo que


P(0) = 1, P(1) = 0, P(2) = 1

Polinomios y Teora de ecuaciones 281

280-281. 281 20/11/02, 12:54 PM


Ejercicios
17. Pruebe que si x3 + ax b es divisible por x2 x b,
entonces a + b = 1.
18. Sea P(x) = x3 + 2x2 + ax + b. Determine a y b tales que P(x) + 1 sea
divisible por x + 1 y P(x) 1 sea divisible por x 1.
19. Sea P(x) = x3 2x2 + ax b. Determine a y b tales que P(x) + 2 sea
divisible por x 2 y P(x) + 1 sea divisible por x 1.
20. Determine a y b para que P(x) = x3 (a + b) x + 2 y
Q(x) = x2 x + (a b) sean ambos divisibles por 1 x.
21. Determine un polinomio P de segundo grado de modo que
P(1) = 0, P(1) = 6 y P(0) = 1.
22. Encuentre un polinomio de segundo grado P de modo que
P(1) = 1, P(2) = 8 y P(1) = 5.
23. Encuentre los valores de a y b en el polinomio
4x3 3x2 + ax + b para que ste sea divisible por x2 1.
24. Al dividir ax4 + bx3 12x2 + 16x 5 por x2 + 2x 3 se obtiene como
resto 32x + 40. Pruebe que a = 5 y b = 4.

Soluciones

1. a) 279 b) 1 c) 4 d) 3 e) 494 f) 22 g) 30 h) 10

i) 2 2 3 j) 0 k) 6 l) 6 m) 0
3
2. a) P(x) + Q(x) = x3 + x2 + x + 1 b) P(x) Q(x) = x3 + x2 + 3x 3
c) 2P(x) 3Q(x) = 3x3 + 2x2 + 7x 8 d) P(x) Q(x) = x5 + 2x4 2x3 + 5x 2
e) P(x) + 2Q(x) = 2x3 + x2 + 3

3. a) P(x) + Q(x) = x3 2x2 + 2x 5 b) P(x) Q(x) = x3 2x2 + 1


c) P(x) + [Q(x)]2 = x3 x2 5x + 7 d) P(x) + x Q(x) = x3 x2 2x 2
e) P(x) + P(x) Q(x) = x4 4x3 + 5x2 4x + 4

5. a) 5x3 5x2 + 5x 4 b) 3x3 5x2 x 4 c) 3x3 3x2 3x

6. a) C(x) = 5x + 5 R(x) = 2x 8

b) C(x) = 1 x2 1 x 1 R(x) = 5 x2 1 x + 19
2 2 4 2 4 4
c) C(x) = 4x2 + 9 R(x) = 12x2 + 12x 26
d) C(x) = x2 x 1 R(x) = 2x + 5
e) C(x) = x3 x2 +x1 R(x) = 0
f) C(x) = x3 x2 +x1 R(x) = 0
g) C(x) = x4 x2 +3 R(x) = 0
h) C(x) = x2 3 R(x) = 6x2 11

282 Polinomios y Teora de ecuaciones

282-283.(2003) 282 20/11/02, 12:56 PM


CAPITULO 5
CAPTULO

7. a) C(x) = x3 + 3x2 5x + 2 R(x) = 0


b) C(x) = x2 + 4x 1 R(x) = 1
c) C(x) = x4 + x3 3x 1 R(x) = 0
d) C(x) = x3 3 R(x) = 2
e) C(x) = x5 2x3 + x2 x R(x) = 0
f) C(x) = x4 + x3 x2 1 R(x) = 1
g) C(x) = x2 + x 1 R(x) = 0
h) C(x) = x4 x3 x2 + 2x 3 R(x) = 3
i) C(x) = x4 + x3 3 R(x) = 0
j) C(x) = x4 + x3 x2 3x 12 R(x) = 24
k) C(x) = x4 2x3 6x2 6x 9 R(x) = 3
l) C(x) = x3 7x2 + x 1 R(x) = 0
m) C(x) = x2 2x 1 R(x) = 2
8. a) 143 b) 720 c) 26 d) 108
9. a) 4 b) 4 c) 47 d) 24
27
10. a = 6 11. a = 12. a = 0 13. x3 + x2 1
2
14. a = 9 b = 15 c=3
15. a = 1 16. x2 + 2x + 1 18. a = 0 , b = 2

19. a = b = 2 20. a = b = 3 21. P(x) = 2x2 3x + 1


2
22. P(x) = 3x2 + 2x 23. a = 4 y b = 3

Teora de ecuaciones 5.2

5.2.1 Clculo de las races


de un polinomio.
Factorizacin.

Definicin: Un nmero a es raz de un polinomio P(x) si


y slo si P(a) = 0.

Teorema 5: Si a es una raz de la ecuacin polinmica P(x) = 0, es


decir, P(a) = 0, entonces (x a) es divisor de P(x).
Si (x a) es divisor de P(x), entonces a es una raz de la
ecuacin P(x) = 0.
Ver ejercicio 2, (pg. 285).
Teorema 6: Sea P(x) un polinomio no constante, entonces, P(x) tiene
a lo menos una raz, real o compleja.

Polinomios y Teora de ecuaciones 283

282-283.(2003) 283 20/11/02, 12:57 PM


Teorema 7: Sea P(x) un polinomio de grado n 1, entonces P(x)
tiene a lo ms n races distintas.
Algunas races de una ecuacin polinmica pueden ser iguales;
esto es, tienen multiplicidad mayor que 1.
Por ejemplo:
La ecuacin de 6 grado P(x) = (x 2)3 (x 1) (x + 3)2 = 0
tiene por races 2 de multiplicidad 3,
1 de multiplicidad 1
3 de multiplicidad 2,
luego las 6 races son 2, 2, 2, 1, 3 y 3.

RACES COMPLEJAS Y RACES IRRACIONALES.


Si un nmero complejo a + bi es raz de un polinomio P(x)
de coeficientes reales, el complejo conjugado a bi tambin
es raz de P(x).
De aqu se deduce que una ecuacin P(x) = 0 de grado impar
tiene por lo menos una raz real.
Si un nmero real de la forma a + b , con b irracional, es raz de
una ecuacin P(x) = 0 con coeficientes racionales, entonces el nmero
a b tambin es raz de la ecuacin P(x) = 0.

RACES RACIONALES.
p
Si q es una fraccin irreducible que es raz de la ecuacin
P(x) = an xn + an 1 xn 1 + + a2 x2 + a1 x + a0 = 0
entonces p es divisor de a0 y q es divisor de an.

Ejemplo:
Dada la ecuacin 2x4 + 5x5 + x2 + 10x 6 = 0 pueden ser races
racionales de ellas los nmeros formados por un divisor de 6 ( 1,
2, 3, 6) partido por un divisor de 2 ( 1, 2).
As pueden ser solucin o raz los nmeros

1 3
1, , 2, 3, , 6.
2 2
1
En efecto, son soluciones racionales y 3.
2

5.2.2 Relacin entre los coecientes


de una ecuacin p(x) = 0
y sus races
Sea P(x) = an xn + an 1 xn 1 + . . . + a2 x2 + a1 x + ao = 0, an 0
dividiendo la ecuacin por an tenemos:
an 1 a2 2 a1 a0
xn + xn 1 + + x + x+ = 0
an an an an

284 Polinomios y Teora de ecuaciones

284-285. 284 20/11/02, 12:58 PM


CAPTULO 5

donde:
an 1
a = suma de las races.
n
an 2
= suma de los productos de las races
an
tomadas de dos en dos.
an 3
a = suma de los productos de las races
n
tomadas de tres en tres y as sucesivamente hasta
a0
(1) n a = producto de todas las races.
n
Un caso particular:
Si P(x) = ax2 + bx + c = 0
y sus races son r1 y r2

entonces: r1 + r2 = b
a
c
r1 r2 = a
Si P(x) = ax3 + bx2 + cx + d = 0
y sus races son r1, r2 y r3

entonces: r1 + r2 + r3 = b
a
r1 r2 + r1 r3 + r2 r3 = c
a
r1 r2 r3 = d
a
Ver ejercicios 6 y 7, (pgs. 287, 288).

1. Determinar si 3, 2, 1 son o no races del polinomio Ejercicios


x4 + x3 7x2 13x 6 resueltos
Solucin:
a es raz de P(x) si P(a) = 0
P(3) = 34 + 33 7 32 13 3 6 = 0
luego 3 es raz del polinomio.
P( 2) = ( 2)4 + ( 2)3 7( 2)2 13 ( 2) 6 = 0
luego 2 es raz del polinomio
P(1) = (1)4 + 13 7 12 13 1 6 = 24
luego 1 no es raz del polinomio.

2. Demostracin teorema 5.
a) P(a) = 0 x a divisor de P(x)
P(x) = (x a) Q(x) + P(a) (5. 1. 4)
como P(a) = 0
P(x) = (x a) Q(x)

Polinomios y Teora de ecuaciones 285

284-285. 285 20/11/02, 12:59 PM


Ejercicios Luego, (x a) es divisor de P(x).
resueltos b) x a divisor de P(x) P(a) = 0
P(x) = (x a) Q(x) + P(a)
como (x a) es divisor de P(x), al dividir se tiene resto 0,
luego P(a) = 0

3. Escribir un polinomio que tenga por races 2, 3 y 1 de multiplicidad 2.


Solucin:
Como este polinomio tiene cuatro races; 2, 3, 1, 1, su grado
debe ser a lo menos cuatro y (x 2), (x 3) y (x + 1)2 son
factores del polinomio pedido.
Luego un polinomio que cumple las exigencias es
P(x) = (x 2) (x 3) (x + 1)2
= (x2 5x + 6) (x2 + 2x + 1)
P(x) = x 4 3x3 3x2 + 7x + 6

4. Determinar las races racionales de la ecuacin


2x 4 3x3 2x2 18x 9 = 0
Solucin:
Divisores de 9 = {1, 3, 9}
Divisores de 2 = {1, 2}
Posibles soluciones racionales de la ecuacin propuesta:

{ 1 3
1, , 3, , 9,
2 2
9
2 }
Para determinar cules son las soluciones debemos probar ya
sea usando divisin sinttica o evaluando la ecuacin para los
distintos valores posibles.

Para ( 12 ): 2 ( 12 )
( 12 ) 2 ( 12 )
4
3
3 2
( 12 ) 9 = 0
18

luego 1 es raz y ( x + ) es factor de


1
2 2
2x4 3x3 2x2 18x 9
para 3: 2 (3)4 3 (3)3 2 (3)2 18 (3) 9 = 0
luego 3 es raz y (x 3) es factor del polinomio.
La ecuacin 2x4 3x3 2x2 18x 9 = 0 no tiene ms races
racionales.

5. Factorizar el polinomio x 4 4x3 6x2 + 7x 10.


Solucin:
Son posibles races racionales del polinomio los divisores de
10 = {1, 2, 5, 10}
Si aplicamos divisin sinttica y probamos con 5 obtenemos:

286 Polinomios y Teora de ecuaciones

286-287. 286 20/11/02, 1:03 PM


CAPTULO 5

1 4 6 7 10 5 Q (x 5) es factor
5 5 5 10

1 1 1 2 0 Q resto

x3 + x2 x + 2 es el cociente que se obtiene al dividir


el polinomio dado por (x 5) y resto 0.
Luego podemos escribir:
x 4 4x3 6x2 + 7x 10 = (x3 + x2 x + 2) (x 5)

Ahora buscamos algn factor del polinomio x3 + x2 x + 2.


Son posibles soluciones los divisores de 2 = {1, 2}.
Aplicando divisin sinttica y probando con 2 obtenemos

1 1 1 2 2 Q (x + 2)
2 2

1 1 1 0 Q resto

x2 x + 1 es el cociente que se obtiene al dividir


x3 + x2 x + 2 por x + 2 y resto 0.
luego:
x4 4x3 6x2 + 7x 10 = (x2 x + 1) (x + 2) (x 5)
Observamos que x2 x + 1 no es factorizable en R ya que sus races
son complejas conjugadas (discriminante menor que cero).

As podemos decir que: (x2 x + 1) (x + 2) (x 5) es la factorizacin


del polinomio x4 4x3 6x2 + 7x 10
6. Resolver la ecuacin x3 16x2 + 79x 120 = 0 sabiendo que una
de sus races es 7 unidades menor que el producto de las otras dos y
que las tres races son racionales.
Solucin:
De acuerdo con la relacin entre coeficientes y races de una
ecuacin sabemos que:

r1 + r2 + r3 = 16
r1 r2 r3 = 120
y por dato del problema podemos decir que:
r3 = r1 r2 7
as resolviendo el sistema:
r1 + r2 + r3 = 16
r1 r2 r3 = 120
r3 + 7 = r1 r2

Polinomios y Teora de ecuaciones 287

286-287. 287 20/11/02, 1:03 PM


Ejercicios r32 + 7r3 120 = 0
(r3 + 15) (r3 8) = 0
resueltos r3 = 15 o r3 = 8

Si r3 = 15
r1 + r2 = 31 r1 = 31 + 993 y r2 = 31 993
2 2
r1 r2 = 8 que no son racionales.

Si r3 = 8
r1 + r2 = 8 r1 = 3 y r2 = 5
r1 r2 = 15

Por lo tanto, la solucin es r1 = 3, r2 = 5 y r3 = 8


7. Si la ecuacin x3 3x2 x 6 = 0 tiene como races los valores a, b
y . Hallar el valor de las siguientes relaciones:
a) a2 + b2 + 2
b) a3 + b3 + 3
c) 1 + 1 + 1

Solucin:
Sabemos que:
a+b+ = 3 (Relacin entre coeficientes
ab + a + b = 1 y races de una ecuacin)
ab = 6

a) (a + b + )2 = a2 + b2 + 2+ 2ab + 2a + 2b = 9
= a2 + b2 + 2 + 2 (ab + a + b) = 2

1
= a2 +b2 + 2
2=2
Luego: a2 +b + =4
2 2

b) La ecuacin es x3 3x2 + x 6 = 0 y se satisface para todas


sus races, luego:

a: a3 3a2 + a 6 = 0
b: b3 3b2 + b 6 = 0
: 3 32 + 6 = 0
Sumando:

a3 + b3 + 3 3 (a2 + b2 + 2) + (a + b + ) 18 = 0

4 3
a3 + b3 + 3 12 + 3 18 = 0
Luego:

a3 + b3 + 3 = 27

288 Polinomios y Teora de ecuaciones

288-289. 288 20/11/02, 1:05 PM


CAPTULO 5

c) Sabemos que:
( 1 + 1 + 1 ) = + +
6 1

6 ( 1 + 1 + 1 ) = 1
luego: 1 1 1 1
+ + =
6

Ejercicios
1. Dados los polinomios y los nmeros 4. Factorice los siguientes polinomios:
siguientes, determine cul de ellos es
(o son) raz(ces). a) x4 + 10x3 + 35x2 + 50x + 24

a) x3 2x2 5x + 6 2, 3, 0 b) x4 + 2x3 7x2 8x + 12

b) x2 + 3x 10 2, 3, 5 c) 4x4 28x3 + 47x2 + 7x 12

c) x4 + x3 7x2 x + 6 2, 3, 1 d) x5 + x4 + 5x2 x 6

d) x4 + x3 6x2 x + 5 1, 2, 1 e) 2x4 5x3 20x2 22x 15


f) x3 6x2 + 8x 3
e) x4 1 1, 1, 2
2. Escriba un polinomio con coeficientes g) 36x4 13x2 + 1
enteros cuyas races sean: h) x4 + 2x3 3x2 4x + 4
a) 2, 3, 1 i) x4 5x3 + 6x2 + 4x 8
b) 3, 1, 2 de multiplicidad 2 j) x5 + 3x4 5x3 + 7x2 6x + 2
c) 1 de multiplicidad 2 y 1 de 5. Encuentre las soluciones de la ecuacin
x3 7x + 6 = 0, sabiendo que el producto
multiplicidad 3
de dos de ellas es 2.
d) 2, 2, 5
6. Resuelva la ecuacin
e) 1 + 2, 1 2, 1 x3 19x2 + 114x 216 = 0, sabiendo que
1 el cuadrado de una de sus races es igual
f) 0, 2
, 2 de multiplicidad 3
al producto de las otras dos.
g) 3 2i, 5, 3 + 2i
7. Resuelva la ecuacin
3. Determine las races racionales de las
2x3 x2 18x + 9 = 0, sabiendo que la
siguientes ecuaciones:
suma de dos de sus races es cero.
a) x4 6x2 + 7x 6 = 0
8. Resuelva la ecuacin
b) 2x5 + 7x4 + 6x3 + 3x2 + 4x 4 = 0 x3 4x2 17x + 60 = 0, sabiendo que la
c) x4 8x2 4x + 3 = 0 suma de dos de sus races es 1.
d) 3x4 7x3 + 5x2 7x + 2 = 0 9. Resuelva la ecuacin
e) 2x4 + x3 8x2 x + 6 = 0 x3 4x2 11x + 30 = 0, sabiendo que
la suma de dos de sus races es igual
f) x3 + x2 5x + 3 = 0 a la tercera.

Polinomios y Teora de ecuaciones 289

288-289. 289 20/11/02, 1:06 PM


Ejercicios
19. Determine el valor de k en la ecuacin
10. Resuelva la ecuacin x3 11x2 + 38x + k = 0, para que una
x3 15x2 + 71x 105 = 0, sabiendo que solucin sea el doble de otra.
una de sus races es la mitad de la suma
20. Determine el valor de k en la ecuacin
de las otras dos.
x3 3x2 + k = 0, para que tenga una
11. Encuentre las soluciones de la ecuacin raz de multiplicidad 2.
x3 6x2 37x + 90 = 0, sabiendo que
una de sus races es dos unidades menor 21. Determine el valor de k en la ecuacin
que la suma de las otras dos. x3 2x2 4x + k = 0, para que tenga
dos races opuestas.
12. Resuelva la ecuacin
x3 17x2 + 82x 120 = 0, sabiendo que 22. Pruebe que si a + bi es raz de un
el producto de dos de sus races es 2 polinomio P(x), entonces (x a)2 + b2
unidades mayor que la tercera y que las divide a ese polinomio.
tres races son racionales.
23. Sabiendo que 2 i es raz del polinomio
13. Si a, b y g son las races de la ecuacin
x3 7x2 + 15x 9, encuentre un
x3 2x2 6x 8 = 0, encuentre el
polinomio de grado 2 que lo divida
valor de:
(que sea factor). (Aplique ejercicio
a) a + b + g anterior.)
b) a b + a g + b g
c) a b g 24. Si 1 + 3i es raz del polinomio
P(x) = x4 x3 + 6x2 + 14x 20,
d) a2 + b2 + g2
encuentre un polinomio primo de
e) a3 + g3
+ b3
segundo grado que lo divida (Aplique
1 1 1 ejercicio 22).
f) + +

NOTA: Polinomio primo es aquel no
g) 1 + 1 + 1 factorizable en R.

14. Encuentre la suma y el producto de las
25. Si i es raz del polinomio
races de la ecuacin
P(x) = x5 4x4 + 7x3 8x2 + 6x 4,
3x3 4x2 + x 6 = 0
factorice este polinomio en sus factores
15. Encuentre la suma y el producto de las primos. (Aplique ejercicio 22.)
races de la ecuacin
5x5 2x4 + 3x + 20 = 0 26. Determine k en la ecuacin
x3 kx2 + 72x 108 = 0
16. Determine la suma y el producto de las sabiendo que dos races son iguales y
races de la ecuacin todas son enteras.
3x6 2x5 + x 1 = 0
27. Determine la multiplicidad de la raz r
17. Escriba la ecuacin de menor grado
dada en la ecuacin:
de coeficientes reales que tengan por
solucin 1 + i, 2 y 5 a) x8 + 5x6 6x2 = 0; r = 0

18. Encuentre la ecuacin de menor grado b) 2x5 17x4 + 51x3 69x2 + 47x 14 = 0;
que tenga una raz igual a 2 3 r=1

290 Polinomios y Teora de ecuaciones

290-291. 290 20/11/02, 1:07 PM


CAPITULO 5
CAPTULO

Soluciones

1. a) 2, 3 b) 2 c) 2, 3, 1 d) 1, 1 e) 1, 1
2. a) x3 6x2 + 11x 6 b) x4 6x3 + 9x2 + 4x 12
c) x5 + x4 2x3 2x2 + x + 1 d) x3 5x2 2x + 10
e) x3 x2 3x 1 f) 2x5 13x4 + 30x3 28x2 + 8x
g) x3 11x2 + 43x 65
1 1
3. a) 2, 3 b) , 2 de multiplicidad 2 c) 1, 3 d) 3, 2
2
3
e) 1, 1, 2, 2 f) 3, 1 de multiplicidad 2
4. a) (x + 1) (x + 2) (x + 3) (x + 4) b) (x 1) (x + 2) (x + 3) (x 2)
c) (x 3) (x 4) (2x + 1) (2x 1) d) (x + 1) (x 1) (x + 2) (x2 x + 3)
e) (x 5) (2x + 3) (x2 + x + 1) f) (x 1) (x2 5x + 3)
g) (2x + 1) (2x 1) (3x +1) (3x 1) h) (x 1)2 (x + 2)2
i) (x 2)3 (x + 1) j) (x2 2) (1 x)3
1
5. 1, 2 y 3 6. 4, 6 y 9 7. 3, 3 y 8. 5, 3 y 4 9. 5, 2 y 3
2
10. 7, 5 y 3 11. 2, 9 y 5 12. 3, 4 y 10
1
13. a) 2 b) 6 c) 8 d) 16 e) 68 f) 3 g) 4
4 2
14. Suma = producto = 2 15. Suma = producto = 4
3 5
2 1
16. Suma = producto = 3 17. x 7x4 + 10x3 + 4x2 24x + 20 = 0
5
3
18. x4 10x2 + 1 = 0 19. k = 40 20. k = 4 21. k = 8
23. x2 4x + 5 24. x2 2x + 10 25. (x2 2x + 2) (x2 + 1) (x 2)
26. k = 15 27. a) 2 b) 1

Prueba de seleccin mltiple


1. Si P(x) = 4x5 6x2 x + 3 C. 6
es un polinomio, su grado es: D. 4
A. 1 E. 1
B. 3
3. Dados P(x) = 4x2 5x + 3
C. 4
y Q(x) =2x3 x2 + 5, entonces
D. 5
E. 6 P(x) + Q(x) es:

2. En el polinomio A. 6x5 6x3 + 8


Q(x) = 25x6 14x4 + x2 1 B. 2x4 + 3x3 5x + 8
el coeficiente principal es: C. 2x3 + 3x2 5x + 8
A. 25 D. 2x3 + 4x2 5x + 8
B. 14 E. 2x3 3x2 5x + 8

Polinomios y Teora de ecuaciones 291

290-291. 291 20/11/02, 1:08 PM


Prueba de seleccin mltiple
4. Sean P(x) = 5x4 3x + 1 C. 80
y Q(x) = x4 3, entonces P(x) Q(x) = D. 84
E. 152
A. 6x4 3x 2
B. 4x4 3x + 4
9. Si Q(x) = 4x 4 3x 2 + 2, entonces
C. 4x4 3x 2
Q( 2 ) es:
D. 4x4 + 3x 2
E. 4x4 + 3x + 4 A. 10
B. 12
5. Si P(x) = x4 x3 +3 C. 14
y Q(x) = x4 + x3 3, D. 16
entonces 2 P(x) Q(x) es: E. 18

A. 2x4 10. Cul (o cules) de las siguientes expresio-


B. 2x3 +6 nes es(son) polinomio(s) en R.
C. x4 3x3 +9 I) x2 x1 + 1 II) 3x2 x1
D. x4 3x3 9
1
III) x 2x + 3
2

E. x4 3x3 +6 A. Slo I
B. Slo II
6. Si P(x) = 6x5 x4 + 2x3 x2 1, entonces
C. Slo III
P(1) es:
D. Todas
A. 1 E. Ninguna
B. 1
11. Al dividir x4 2x2 6 por x + 3, el
C. 11
resto es:
D. 11
E. 6 A. 69
B. 62
7. Si P(x) = x5 + 2x4 x 2 son soluciones C. 59
o races del polinomio D. 57
I) i II) i III) 2 E. 54

A. Slo I 12. Al dividir x5 6x4 2x3 x + 1 por


B. Slo II
x3 3x2 + 1 el cociente y el resto son
C. Slo III
respectivamente:
D. Slo I y II
E. I, II y III A. x2 + 3x 11 y 34x2 2x + 12
B. x2 3x + 11 y 34x2 + 2x 12
8. Si P(x) = 6x3 x2 + 2x 1, entonces
C. x2 3x 11 y 34x2 + 2x + 12
P(3) es:
A. 156 D. x2 + 3x + 11 y 34x2 2x 12

B. 158 E. x2 + 3x 11 y 34x2 2x 12

292 Polinomios y Teora de ecuaciones

292-293. 292 20/11/02, 1:10 PM


CAPTULO 5

13. Cul es el polinomio que dividido por 18. Para que x3 ax x + b sea divisible
x2 + 1 da como cociente x + 2 y por x2 + x a debe ser:
resto x 3. A. a = b
A. x3 2x2 + 2x 1 B. a=b
B. x3 + 2x2 + 2x 1 C. a+b=1
C. x3 + 2x2 2x 1 D. ab=1
D. x3 + 2x2 + 2x + 1 E. a + b = 1
E. x3 2x2 + 2x + 1
19. Sea P(x) = x3 + 2x2 ax + b 3.
14. Al dividir x4 2x3 + 4x2 x + 1 por x Determinar la relacin que debe cumplirse
2 el resto es: entre a y b para que P(x) + 6 sea divisible
A. 3 por x + 1.
B. 9
A. a+b3=0
C. 15
B. a+b4=0
D. 51
C. a+b+3=0
E. 61
D. a+b+4=0
15. Cul es el valor que debe tener k en el E. ab+4=0
polinomio 4x5 2x3 + kx 2 para que
sea divisible por x 2? 20. Qu valores deben tener a y b para que
5x3 2x2 + ax b sea divisible por x2 + 1.
A. 5
B. 25 A. 5 y 2
C. 50 B. 5 y 2
D. 50 C. 5 y 2
E. 55 D. 5 y 2
E. 4 y 2
16. Qu valor debe tener k en el polinomio
6x3 kx2 + x 1 para que al dividirlo por 21. El polinomio de segundo grado tal que
x2 3, el resto sea 19x 7? P(1)1 = 6, P(2) = 14 y P(2) = 18 es:
A. 1
A. 3x2 x + 4
B. 0
C. 1 B. 3x2 x 4
D. 2 C. 3x2 + x + 4
E. 3 D. 3x2 + x + 4

17. Qu valores deben tomar a y b para que E. 3x2 + x 4


se verifique la igualdad
1 22. El polinomio con coeficientes reales de
2ax3 bx2 + 1 = 2 (bx3 2x2 + )
2 menor grado cuyas races son 3, 4 y 5 es:
I x E R.
A. x3 12x2 + 47x + 60
A. a=4 b=4
B. x3 12x2 47x 60
B. a=2 b= 2
C. a=4 b= 4 C. x3 + 12x2 + 47x + 60
D. a=4 b=4 D. x3 + 12x2 47x 60
E. a = 2 b = 2 E. x3 12x2 + 47x 60

Polinomios y Teora de ecuaciones 293

292-293. 293 20/11/02, 1:10 PM


Prueba de seleccin mltiple
23. El polinomio con coeficientes reales 27. El polinomio x3 7x 6 es equivalente a:
de menor grado cuyas races son 1 y A. (x + 2) (x 3) (x 1)
2 es: B. (x + 2) (x 3) (x + 1)
A. x3 x2 + 2x + 2 C. (x 2) (x + 3) (x + 1)
B. x3 x2 2x 2 D. (x 2) (x + 3) (x 1)
C. x3 + x2 + 2x 2 E. (x + 2) (x + 3) (x + 1)
D. x3 x2 2x + 2 28. Si las races de la ecuacin
E. x3 + x2 + 2x + 2 x3 2x2 + x 3 = 0 son
a, b y g, entonces el valor de
24. El polinomio con coeficientes reales de a+b+g=
menor grado cuyas races son
A. 1
1 3, i y 2 es: B. 2
C. 3
A. x5 + 4x4 + 3x3 + 2x + 4
D. 3
B. x5 4x4 3x3 + 2x + 4
E. 2
C. x5 4x4 + 3x3 2x + 4
D. x5 4x4 + 3x3 + 2x 4 29. Si las races de la ecuacin
E. x5 4x4 + 3x3 + 2x + 4 2x3 6x2 + 8x 10 = 0
son a, b y g, entonces el valor de la
25. Las races racionales del polinomio expresin a2 + b2 + g2 es:
x5 2x4 + x 4 son: A. 3
A. 2, 2 y 4 B. 4
B. 1, 2 y 2 C. 5
C. 1, 4 y 4 D. 1
D. 1, 1 y 4 8
E.
E. no tiene 9

30. El valor de k en la ecuacin


26. Las races racionales del polinomio
x5 4x4 5x3 + 20x2 + 4x 16 son: x3 kx2 + 48x 36 = 0
para que sta tenga una raz de multipli-
A. 1, 1, 2, 2 y 4 cidad 2 es:
B. 1, 1, 2, 4 y 4 A. 10
C. 1, 1, 2, 4 y 4 B. 12
C. 13
D. 1, 2, 2, 4 y 4
D. 24
E. 1, 2, 2, 4 y 4 E. 32

Soluciones

1. D 2. A 3. C 4. B 5. C 6. D 7. E 8. B
9. B 10. E 11. D 12. C 13. B 14. C 15. E 16. D
17. C 18. B 19. D 20. A 21. A 22. E 23. D 24. E
25. E 26. A 27. B 28. B 29. D 30. C

294 Polinomios y Teora de ecuaciones

294. 294 20/11/02, 1:11 PM


CAPTULO 6
Potencias
y Races

Potencias 6.1

6.1.1. Potencias de exponente natural


Definimos an = a a a a (n veces)
La expresin an se llama potencia n-sima de a.
a es la base de la potencia.
n es el exponente de la potencia.

6.1.2. Potencias de exponente cero


y exponente entero negativo
De las propiedades que estudiaremos ms adelante se deduce que:
a0 = 1 para todo valor de a. (con a 0)
1
an = para todo valor de a : a 0 ; n N
an

1. Calculemos el valor de (2)3


Ejercicios
Aplicando la definicin tenemos:
resueltos
(2)3 = (2) (2) (2) = 8
2. Calculemos el valor de 34
Observamos aqu que la base de la potencia es 3 ( y no 3),
expresndola en forma de producto nos queda:
34 = 3 3 3 3 = 81
3. Calculemos (3)4
Aqu la base es (3) y por lo tanto:
(3)4 = (3) (3) (3) (3) = 81
4. Calculemos 2 5

Potencias y races 295

295.(2003) 295 20/11/02, 1:13 PM


Ejercicios Aplicando la definicin (exponente negativo):
resueltos 1 1 1
25 = = =
25 2 2 2 2 2 32
0
5
1 17 7
5. Calculemos 2 3
2 4
Como el exponente es cero ( y la base es distinta de cero ), aqu no es
necesario hacer ningn clculo. El valor es 1.

Ejercicios
I. Aplique la definicin de potencias para calcular :
4 4
3 1
1. 22 13. (11)2 25. (3,5)0 35. 43. 1
4 2
2. 32 14. (5)4 26. (1,7)0 3 6 3
36.
4 44. 2 5
3. 52 15. (3)5 7
27. (2,1)1 3 2
3
4. (2)3 16. (3)5 37. 3
5 45. 3
28. (0,8)2 4
5. 33 17. 63 11 4
38. 46. ( 0,27)1
6. 42 18. (6)3 29. (0,8)2 2
2
7. 101 19. 25 30. (2,5)3 2
39. 3 47. (0,08)2
3
8. 122 20. (2)5 2 3
4 3
31. 1 1
9. (3)2 21. (0,5)2
3 40. 1 48. 1
3
5 16
10. (3)3 22. (1,1)3 32. 3 3 4
2 1 3
2 41. 2 49. 2
3
11. (8)3 23. (0,3)3 33. 2 4
5 2
4 2 3
12. (12)3 24. (2)2 34. 3 42. 3 1
5 50. 1
4 4

II. Aplique la definicin para calcular :


1 4
3 1
1. 2 1 9. (1)5 17. 22. 1 28. (0,06) 1
4 3
2. 2 2 10. (1)6 2 29. (0,04)2
2 23. ( 0,25)0
18. 1
3. (2)2 11. (0,5) 4 3 3
30. 2 4
24. (0,25)0
4. (2)2 12. (0,5)3 3 2 2
19. 4 2 31. 1 1
5. (5) 3 2 15
13. (1,7) 2 25. 5
3
3 2 32. (0,06) 1
6. (5) 4 14. (0,2) 2 20. 4
2 26. 3 1 3 2

7. (5)3 15. (2,1)3


4 33. 3
3 3 5
3
21. 2 27. 2 4
8. 2 3 16. (2,05)0 4 5 34. (0,71)2

296 Potencias y races

296-297. 296 08/11/2001, 10:45


CAPTULO 6

1 0
35. 1 1 2 37. ( 0,04) 1 39. 6
2 16
3 6 3 0
36. (0,04) 1 38. 2 40. 2
2 5

III. Calcule el valor de: 0


1 1 2
1 3 4 1
1 + 31. 3 +2
1. 3 + 32 18.
2 2 2 3 2

2. 23 22 2
+
3
19. 1 21 + 22 23 32.
32 21
3. 20 + 21 + 22 + 23 1 2 3
3 3 4 0
20. + 2 1
2
4. 3 35 4 4 3 33.
5 7
5. (2 3)2 21. (0,02)2 + (0,02)2 1 2
1 1
34. 3 + 2
6. 2 (3)2 1 1 2 2
1 2
22. +
7. (2)2 + (3)2 4 3 1
5 1
2
3 2
3 2 35. +
23. + 5 2
8. (0,2)2 (0,1)2 2 2
0 1
1 1
9. 5 (3)2 3 2
3 2
36. + + 21
24. 2 2
2 2
10. 5 (3)3 1 2 1 1
2
1
2
2 3 1 37. 32 + +
25. + 2 1 3 3
11. (5)2 (2)3 3 2 5
3
4 7 3
12. (3)1 + (3)3 26. 2 3 2
7 4 1 1 1
38. 1+ +
13. 3 43 3 4
2 4 2 2 2
27.
2 3 0
14. 23 32 41 3
1 1 1
28. 32 : 23 39. 3 2
7
+2
15. 26 32 25 32 1 0
3 5 5
1 2
29. 4 1
16. (12)1 + (12)1 3
5
+
2 1 2 3
2 5 2
3 3
4 3
2 1
0 40.
17. + 30. 2 5 2 5
3
4 3 3 11
5

IV. Determine el valor numrico de las siguientes expresiones para los valores
de las variables indicadas:
1. x2 + y2 si x = 1 e y =2
2. x3 3y2 + 2x si x = 2 e y = 1
3. 2a2b 3b2 si a = 3 y b =2
4. 5a 5b si a = 2 y b = 3
5. 3 (a2 + 2ab + b2) si a = 4 y b =3
6. 2 (1 x + x2 x3) si x = 2
7. x 3 + y 3 si x = 1 e y =2

Potencias y races 297

296-297. 297 08/11/2001, 10:46


Ejercicios
8. 2x 4 3y 5 si x = 1 e y = 2
9. a(2a2 a2) si a = 3
10. (2 + x y)2 si x = 2 e y = 3
11. (3x2y4)1 si x = 3 e y = 3
12. (3xy)x + y 2 si x = 2 e y = 1
13. (22 + 33)x si x = 3
14. (a b + b a)1 si a = 2 y b =3
15. 2a(2a3 3a4)2 si a = 2
16. (3x 2 + y 3 2y 4)1 si x = 2 e y = 1
17. 2ab + a2 b2 si a =5 y b =3
18. x 4 + 4 x si x = 1
19. 2x1 3y3 + xy si x = 1 e y = 1
20. x 2 + y 3 si x = 2 e y = 3

Soluciones
I. 1. 4 2. 9 3. 25 4. 8 5. 27 6. 16 7. 10 8. 144 9. 9 10. 27 11. 512
12. 1.728 13. 121 14. 625 15. 243 16. 243 17. 216 18. 216 19. 32
20. 32 21. 0,25 22. 1,331 23. 0,027 24. 4 25. 1 26. 1 27. 2,1 28. 0,64
8 3 4 81 81 729
29. 0,64 30. 15,625 31. 27 32. 3 33.
35. 256 36. 4.096 34.
8 25 256
9 1 4 46 19 14 1 342
37. 38. 915 39. 13 40. 2 41.12 42. 11 43. 5 44. 19 343
25 16 9 625 27 25 16
47 817 49 61
45. 52 46. 0,27 47. 0,0064 48. 1 49. 57 50. 1
64 4.096 256 64

1 1 1 1 1 1
II. 1. 2. 3. 4 4. 5. 6. 625 7. 1 8. 1 9. 1 10. 1
2 4 4 125 125 8
4
19. 4
100 1.000
11. 16 12. 8 13. 14. 25 15. 16. 1 17. 18. 9
289 9.261 3 4 9
4 64 81 9 256 125
20. 21. 22. 23. 1 24. 1 25. 26. 27.
9 1.331 256 361 28.561 2.744
50 4 225 50 25 10.000 4
28. 29. 625 30. 31. 256 32. 3 33. 324 34. 35.
3 11 5.041 9
64
36. 25 37. 25 38. 39. 1 40. 1
117.649

III. 1. 12 2. 4 3. 15 4. 729 5. 36 6. 18 7. 5 8. 0,03 9. 45 10. 135


2 5 5
11. 200 12. 30 13. 192 14. 288 15. 287 16. 0 17. 18. 19.
3 8 8
52 25 1 1 110
20. =1 21. 2.500,0004 22. 5 23. 4 24. 0 25. 26. 0
27 27 2 2 147

298 Potencias y races

298-299. 298 08/11/2001, 10:53


CAPITULO 6
CAPTULO

8 78 9 1
27. 1 28. 29. 1 30. 1 31. 1 32. 24 33. 1 34. 35. 2 36. 3 2
9 175 10
2 57 16
37. 9 38. 1 39. 1 40.
9 64 625
4 1 3 1
IV. 1. 5 2. 9 3. 24 4. 5. 147 6. 30 7. 1 8 8. 2 9. 53 2 10.
125 32 3 9
1 8 1 4
11. 12. 37 13. 29.791 14. 15. 16. 17. 20 18. 5
2.187 71 1.024 9
23
19. 2 20.
108

Propiedades de las potencias 6.2

6.2.1 Multiplicacin de
potencias de igual base
Para multiplicar potencias de igual base mantenemos la base y
sumamos los exponentes, es decir:
an am = am+n

6.2.2 Divisin de potencias


de igual base
En este caso, mantenemos la base y restamos los exponentes,
es decir:
an : am = anm

6.2.3 Elevacin de potencia a potencia


Aqu debemos elevar la base a la multiplicacin de los
exponentes.
(am)n = an m

6.2.4 Multiplicacin de potencias


de igual exponente
Elevamos el producto de las bases al exponente comn.
an bn = (ab)n
Potencias y races 299

298-299. 299 08/11/2001, 10:53


6.2.5 Divisin de potencias
de igual exponente
Elevamos el cociente de las bases al exponente comn.
n
an : bn = a
b ( )
Los recprocos de las propiedades 6.2.4 y 6.2.5 tambin son
vlidos, es decir:

6.2.6 Potencia de un producto


Se eleva cada factor del producto al exponente:
(a b)n = an bn

6.2.7 Potencia de un cociente


n
( )
a
b
n
= an
b

1. Expresemos en forma de potencias:


Ejercicios
1 1 1 1 1
resueltos
2 2 2 2 2
Aqu tenemos el producto del trmino 1 cinco veces (el
2
trmino se repite 5 veces).
As es que :
5
1 1 1 1 1 1
=
2 2 2 2 2 2
2. Efectuemos los productos:
3 5 2 6
a a 15 a11
4 5
La multiplicacin es una operacin conmutativa, por lo tanto :
3 5 2 6 3 2 3
a a 15 a11 = 15a5 a6 a11
4 5 4 5
2

= 9 a22
2
3. Desarrollemos (a2 + a6)2
Se trata de un cuadrado de binomio, por lo tanto,
(a2 + a6)2 = a4 + 2a8 + a12
4. Efectuemos los productos indicados :
am 3 b2 a4 bn + 2
conmutamos los trminos agrupando bases iguales y luego multi-
plicamos.

300 Potencias y races

300-301.(2003) 300 20/11/02, 1:23 PM


CAPTULO 6

am 3 a4 b2 bn + 2 = am + 1 bn + 4
5. Efectuemos las operaciones:
25 5a +1 27 3a + 4
Observamos que el 25 es potencia de 5 (25 = 52) y que 27 es
potencia de 3 (27 = 33). Entonces:
25 5a +1 27 3a + 4 = 52 5a + 1 33 3a + 4
= 5a + 3 3a + 7
6. Multipliquemos: 162 4n + 2
Podemos expresar el 16 como potencia de 4; 16 = 42
162 4n + 2 = 44 4n + 2 = 4n + 6

Ejercicios

I. 19. 5 54
1. a3 a5 = 20. 16 24 + a
2. x2 x3 x6 = 2 5 3 9 10 12
21. p p p
3 5 7
3. 6 a4 5 a3 2a8 22. 0,07a3 0,5a2 11,1a1
4. (a b2)4 (a b2)3
23. 4 m2 p 3 m2 p2
5 5
5. 2 ab (a2 + b2)
24. 2 (a + b)7 5 (a + b)8 4 (a + b) 6
6. nk 3 n4 k
25. 254 + p 1253 p
7. n 2 n k n3 + k
26. 9 3n 2 3n + 1
8. 10 c8 0,25 c 4 2 c6
27. 125 5 2 5 4
9. pn + 1 pn 2
28. 3c4 9c6 81c 4
10. (2 a2 3b2)4
29. 2 4n 83n
11. (a2 + a3 + a4) 2

30. 64 2 6 22
12. (1 + a + a2) a6
1 2n 2 1 2n 3
31. p2n 1 p p
13. (na 1 2na 2 + n) na + 3 2 4
32. am 3 (am 2 a3 m)
14. (y 1 y6 + y9) 2 y2

15. (3an 2 2an 3) a3 33. 128 24n 1


3 n 4 2n
16. (m6 + n6) (m6 n6) 34. m p m p
4 5
17. 32 2k 2 35. 0,4 4 1 + 0,3 3 2 + 0,1 10 3

18. 27 3m + 3 36. (105 + 106) 10 4

Potencias y races 301

300-301.(2003) 301 20/11/02, 1:24 PM


Ejercicios

3 n 5 3 5 n
37. ax + 1 ax 1 a2x 39. p p
4 5
3 7 2 1
38. 12n8 n n 40. 3an 2 bn 3 6a3 b 4
4 5

II.

1. x6 : x2
18. (u 4 : 4u) : u 6
2. a4 :a
19. (a 2 : 3a4) : (a6 : a 6)
3. m16 : m6 2 2
2 3

4. (2p 3q)5 : (2p 3q)3


20.
3
: 3
21. (ex e x) : ex
5. (216 : 24) : 28
22. 3a2 : 6a3
6. a11 : (a3 : a5)
23. m6 c : mc 6
a+b 3
7. 24. x2n 1 : xn 1
a+b
8. x7 : x4 25. 82 3x : 2x + 2
6 5 26. a 2x : ax
9. x5 + x4
x +x a 2
a 3

10. (a 8 a 3) :a 11 27.
4
: 2
11. (p a + p 2a + p 3a) : p 4a
1 n n
28. a : a
bn b1 n
12. (a : ab) : b
29. (m a m b m c) : m
13. (abc : bc) : a
30. (a4 b4) : (a2 b2)
14. 81 : 3a 3
31. (a6 a5) : a5
1 2
2 3 32. (16a8 8a4 4a2) : 2a2
15.
3
: 2
16. a 6 : a 8 33. [a8 : (a4 : a2)] : a3

17. (2p 2 q 3) : 6p 3 q 5 34. [mp + 1 : 2mp + 2] : 2mp 2

III.

5 5
1. 34 24 4. 2 p
3
q 7. (2x + y)3 (2x y)3
3 2
am bm 1 (1,4) 1
2. 5. (1,04) 8. (m + n)6 (m n)6
6n 4
3. ( 2a)4x (3b)4x 6. 36 n 1 9. 2rs 4 2
3 rs

302 Potencias y races

302-303. 302 20/11/02, 1:26 PM


CAPTULO 6

25 3
10. (0,2)5 105 17. 0,6 a 4 0,2 b 4 24.
75 3
4 4
11. 2
3
18. 27 : 37 (3m)a
25.
3 4 ma
1 162
12. 3a2 2 1
19. 63
82 26.
2 a2 6a 3
35
13. 2 6 :3 6 20. 5
27.
2 ay
9
x x 6 ay
a 2y 212
14.
y
21.
492
28. (a2 b2)2 : (a b)2
3a
4 4
15. (b2 4ac)2 (b 2)2 22. 2 : 4 29. 5p 2 : 6q 2
3 9
2a + b
6x 3y
16. (a 3b) 3 (a + 3b) 3 23. 3,2 2 : 1,6 2 30. 2a + b
36x2 9y2

IV.
01. (22)3 02. (32)3 03. [(2)2]4

04. (( 6)3) 1 05. [(8,5)1]2 06. (2a2b)3


1
07. (mamb) ab 08. (23)2 09. (1002)1 104
10. 105 : (104 : 102) 11. [(a b)2 : (a b)]1 12. (x2)m

13. (x2 y 3) 1 x2 y 2 14. [px + 2 qx + 2]


3
3 1 3 2
15. a a 16. (a a2 + a3 a4) a1
2 2
17. (a 1 + a 2 + a 3 + a 4) : a 5 18. [( z 4) : ( z)4]3

19. [125 x6 : (25x3 : 5x)] 2 20. (5a3) 3 : (5a3) 4


21. [(x + y) : (x2 y2)] 2 22. [(a2 b2) : (a b)] 1

23. [( 0,117)0 : ( 3,15)2] 1 24. [(0,03)2 (0,3)2] 1

25. 9x5 : [2x : x5]3 26. [16a3 : 4a2] 2

27. [ax + 1 bx 2]3 x (ab)x 28. [xu + yv] 1


1
2 2
29. [a6 : b5] 2 30.
2
3

20
5 5 3

Soluciones

I. 1. a8 2. x11 3. 60 a15 4. (a b2)7 5. 2a3b + 2ab3 6. n 7. n 8. 5 c10


1
9. p2n - 1 10. 16a8 96a6 b2 + 216a4 b4 216a2 b6 + 81 b8 11.
a4 + 2 a5 + 3 a6 + 2 a7 + a8
12. a6 + a7 + a8 13. n2a + 2 2n2a + 1 + na + 4 14. 2y 2y8 + 2y11 15. 3an + 1 2an
4
16. m12 n12 17. 2k + 3 18. 3m + 6 19. 55 20. 28 + a 21. p26 22. 0, 3885 a6
7

Potencias y races 303

302-303. 303 20/11/02, 1:28 PM


Soluciones

12 4
23. m p3 24. 40 (a + b)9 25. 517 p 26. 32n + 1 27. 5 3 28. 37 c6
25
3
29. 211n + 1 30. 22 31. 1 p6n 6 32. a2m 5 1 33. 24n + 6 34. m p 3n
8 5
18 9
35. 0,13343 36. 10 + 102 37. a4x 38. 39. 40. 18 an + 1 bn 7
5 20

II. 1. x4 2. a3 3. m10 4. (2p 3q)2 5. 24 6. a13 7. (a + b)2 8. x3 9. x


3
3 1
10. a3 a8 11. p3a + p2a + pa 12. b 2 13. 1 14. 37-a 15. 2 16. a2 17. p q2
3
1 1
18. 4 u 19. a 18 20. 2 21. 1 e 2 x 22. 1 a 1 23. m12 2c 24. xn 25. 24 10x
3 3 2
26. a 3x 27. 2a 28. a1 2n b1 2n 29. m a 1 m b 1 m c 1 30. a2 + b2

31. a 1 32. 8 a6 4 a2 2 33. a3 34. 2 2 m1 p

III 1. 64 2. (ab)m 3. ( 6 ab)4x 4. (p q)5 5. (1, 456) 1 6. 1 7. (4x2 y2)3 8. (m2 n2)6
6 x
3 2
9. 4 4 10. 25 11. 2 4 12. 1 13. 14. 15. (b3 2b2 4abc + 8ac)2
3 2 3
7 2

()
2 3 3 4
16. (a2 9b2) 3 17. 0,12 (ab) 4 18. 19. 4 20. 3 5 21. 22.
3 7 2
2
1 5 q
23. 2 2 24. 33 25. 3a 26. a 3 27. 28. (a + b)2 29. 30. (6x + 3y) 2a b
3 6 p
1 1
IV 1. 26 2. 36 3. ( 2)8 4. ( 6) 3 5. (8,5)2 6. 8a6 b3 7. mb na 8. 26
3
9. 1 10. 103 11. (a b) 1 12. x2m 13. y 14. (pq)x + 2 15. 4 a9 16. 1 a + a2 a3
9
17. a4 + a3 + a2 + a 18. z 24 19. 5 4 x 8 20. 5 a3 21. (x y)2 22. (a + b) 1

23. ( 3, 15)2 24. 12.345,679 25. 72x 7 26. (4a) 2


2 + 3x + 3 b x2 + 6x 6
1 125
27. ax 28. xu + yv 29. a 12 b10 30.
48

6.3 Ecuaciones exponenciales


Son aquellas ecuaciones que presentan las variables en el
exponente. Para resolverlas aplicamos la siguiente propiedad
de potencias: ax = ay x = y, para todo a, a 0, a 1.
Si en una ecuacin no resulta posible igualar las bases, la
solucin se obtiene aplicando LOGARITMOS, tema que no
abordaremos en este captulo.

Ejercicios 1. Resolvamos la ecuacin 2x = 4.


resueltos Debemos expresar ambos miembros de la igualdad como potencias
de la misma base; en este caso claramente la base es 2.

304 Potencias y races

304-305. 304 08/11/2001, 11:15


CAPTULO 6

As tenemos: 2x = 4 Q 2x = 22
y aplicando la propiedad indicada al comienzo obtenemos la
solucin x = 2.
2. Resolvamos la ecuacin 3x + 2 = 27.
Procediendo como en el ejercicio anterior tenemos:
3x + 2 = 33 Q x + 2 = 3 Q x = 1
3. Resolvamos la ecuacin 16x = 32
Aqu, tanto el 16 como el 32 son potencias de 2. Entonces:
16x = 32 Q (24)x = 25 Q 24x 25
Q 4x = 5

Q x= 5
4
4. Resolvamos la ecuacin 7x 3 = 1
Recordemos la propiedad de potencias: a0 = 1, es decir, podemos
representar el 1 como una potencia de cualquier base (distinta de
cero) con exponente cero.
As: 7x 3 = 1 Q 7x 3 = 70 Q x 3 = 0
Qx=3

Ejercicios
I.
13. 5x + 4 = 125x 4
1. 2x = 16
14. a2x 1 = a2
2. 2x-5 = 32
15. mx 3 = (m2)2x
3. 36 x = 27x 2
16. (3a)2x 5 = 9a2
4. 32x 2 = 2
17. (p 3)5x = p2 6p + 9
5. 642x 5 = 16x 2
18. (a2 + 2ab + b2)2 = (a + b)2x
6. 125x 3 = 25x 3
19. (1 + a2)x = 1 + a2
7. 3 27x 2 = 9x
20. (a2 + 2a 9)x 3 = 1
8. 5x 3 = 1
21. 81x 6 = 3x 4
9. 162x 4 = 1
10. 2x + 1 = 8 22. 322x 3 = 2x + 3

11. 2x + 1 = 16 23. 125y + 2 = 52y

12. 2x + 1 = 128 24. 256y = 4 42y 3

Potencias y races 305

304-305. 305 08/11/2001, 11:15


Ejercicios
25. m 2 y 5 = (m5)y + 4 37. ux 5y = u0
v3y = 1
26. (2p)6y + 1 = 16p4
x2
2 4
y 38. =
27. 2x + 1 =4 3 9
y
22x = 4 + 1 2x 3
4 5
39. =
5 4
28. ax + 1 = a2
x+y 2x
b =1 1 9
40. 2 =
3 49
29. m4x 2 = m2 x6
px + y = p3 1 216
41. 1 =
5 125
30. u2x 1 = u5 42. (0,1)2x 4 = 10
v2y = vx
43. (0,001)x 4 = (0,01)x
31. mx = my + 1
n2x = ny 3 44. (0,5)x 6 = 1
16
2x 3 0
32. 53x 2 = 125
y
45. 2 3
=
7 4
32x 2 = 16
46. (0,0001)2x = 0,1
33. (a + b)6x = (a2 + 2ab + b2)y 47. ax ay = a4
(2a)x y = 32a5 xy = 3

34. 2x y = 2 48. m2x + 1 m2y + 1 = m18


16y = 32x mx : my = m

49. 23x 22x y = 16


35. 25x 3 = 1
32x + y = 81
px y = p2
1
50. 128 x = 1
36. m2y = m4
mx + y = m4

Soluciones
11 11
I. 1. x = 4 2. x = 10 3. x = 3 4. 5. 6. x = 3 7. x = 5
5 4
8. x = 3 9. x = 2 10. x = 2 11. x = 3 12. x = 6 13. x = 8
3 7 2
14. x = 15. x = 1 16. x = 17. x = 18. x = 2 19. x = 1
2 2 5
20 25
20. x = 3 21. x = 22. x = 2 23. y = 6 24. y = 1 25. y =
3 3
1
26. y = 27. x = 3 ; y=2
2

306 Potencias y races

306-307 306 08/11/2001, 11:24


CAPTULO 6

28. x = 1 ; y = 1 29. x = 1 ; y = 2 30. x = 3 ; y = 3


2
14 32 5 15
31. x = 4 ; y = 5 32. x = ; y= 33. x = ; y=
5 15 2 2
34. x = 4 ; y = 5 35. x = 3 ; y = 1 36. x = 2 ; y = 2

37. x = 0 ; y = 0 38. x = 4 39. x = 1 40. x = 1 41. x = 9


3
42. x = 43. x = 12 44. x = 10 45. x = 3 46. x = 1
2 2 8
47. (x = 3 y = 1) V (x = 1 y = 3) 48. x = 9 ; y = 7
2 2
49. x = 8 ; y = 12 50. No hay solucin en R.
7 7

Races 6.4
n
Definicin: a = b bn = a
n es el ndice de la raz
a es la cantidad subradical

Observaciones:
n
1. Si a > 0 y n es par, entonces a representa un nmero real, es
decir, n
a R.
2. Si a < 0 y n es par, entonces n a representa un nmero complejo,
conjunto que estudiaremos ms adelante.
Es decir, a < 0 y n es par n a R.
3. Las operaciones definidas para las races verifican las propiedades
que se cumplen en los nmeros reales (R).

Propiedades 6.5

6.5.1 Potencia de
exponente fraccionario
Toda potencia de exponente fraccionario se puede expresar como
raz cuyo ndice es el denominador del exponente
m
a n = n am
Potencias y races 307

306-307 307 08/11/2001, 11:24


6.5.2. Multiplicacin de races
de igual ndice
Multiplicamos las cantidades subradicales y conservamos el
ndice.
n n n
a b = ab

6.5.3. Divisin de races


de igual ndice
Dividimos las cantidades subradicales y conservamos el ndice.
n n n
a : b= a :b

6.5.4. Raz de una raz


Conservamos la cantidad subradical y multiplicamos los
ndices.
m n
a = mn a

Ejercicios 1. Determinemos el valor de


3
216
resueltos Aplicando la definicin tenemos:

3
216 = 6, ya que 63 = 216
5
2. Expresemos la raz m3 como potencia de exponente fraccionario.
Aplicando directamente la propiedad tenemos:

3
5
m3 = m5
3 3
3. Obtengamos el siguiente producto: 15a 5a
Se trata de multiplicacin de races de igual ndice;

3 3 3
15a 5a = 75a2

4. Obtengamos la siguiente divisin 18 5 m4 : 3 5 m


Se trata de divisin de races de igual ndice;

18 5 m4
5
= 6 5 m3
3 m

308 Potencias y races

308-309. 308 08/11/2001, 11:29


CAPTULO 6

5. Simplifiquemos 75a3b 4
Aplicando las propiedades tenemos:

75a3b4 = 25 3 a2 a b2 b2 = 5ab2 3a

5 3
6. Expresemos en forma de una sola raz, x2y

Podemos directamente multiplicar los ndices; nos queda:


5 3 15
x2y = x2y

3
7. Expresemos como una sola raz a b
Aqu es necesario introducir el trmino a dentro de la raz cuadrada
antes de multiplicar los ndices. As:
3 3 6
a b = a2 b = a2 b

8. Realicemos las operaciones siguientes:


2 +3 3 +4 2 5 3

Recordemos que slo podemos sumar o restar races que tengan


el mismo ndice y la misma cantidad subradical; nos queda
entonces:
2 +3 3 +4 2 5 3= 2 +4 2 +3 3 5 3
=5 2 2 3

Ejercicios

I. Determine el valor de:


4 3
1. 4 8. 100 15. 256 23. 8.000
4
3 16. 81 5 1
2. 25 9. 8 24.
32
3 1
3 17. 8 5
3. 64 10. 27 25. 1
5
3
18. 32 26. 9,61
3
4. 64 11. 216 3 1 121
19. 27 27. 196
3 3
5. 1.000 12. 0,001 81
20. 49 28. 0,09
3
6. 121 13. 125 3 29. 0,16
21. 512
4 4 16
7. 196 14. 625 22. 841 30. 81

Potencias y races 309

308-309. 309 08/11/2001, 11:31


Ejercicios

II.
1. Escriba los cuadrados de los nmeros naturales del 1 al 20.
2. Escriba los cubos de los nmeros naturales del 1 al 30.
3. Exprese los nmeros naturales del 1 al 20 como races cuadradas.
4. Exprese los nmeros naturales del 1 al 10 como races cbicas.

III. Exprese las siguientes potencias como races:


3 3
4
1 2a 7
2
1. a 17. mn2 3 12. 2 16. 23
3b
1
2 2 1
2. m 18. 3pq 5 1 3
5 6 17. 5a2bc5
4 13.
3. 3
5
3
2a
p
1 19. 5a2 4
18. 4m2n q
6 1
4. 2
14. 1
3
1 y
3
4 10. m n 6 7 8 2 3z z
5. p 19.
2y
1 1
2 p
1 2 2
6. 11.
a 5 15. 1 20.
4m q
2 3 5n6
b

IV. Exprese las siguientes races como potencias de exponente fraccionario.


6
z 5t
1. a3 5. 5a7 19.
5
x2y6 13. 3u
11 x p+q p
2. 5m 6. p10 10. xy 14. 2a
4 n ab
3. 2p2 3 7. 2m4 11. 81 15. 3xy 2a
p 2a
6
4. 2x5 8. 3p6q3 12. 5 16. b
5a2 a

V. Aplique la definicin para despejar la incgnita indicada en cada caso:

4
1. a =5 (a) 6. a+1 = b2 (a) 11. x2 =2 (x)
3 4
2. z =2 (z) 7. 2t + 5 =3 (t) 12. a 1 =4 (a)
5 n 1 1
3. m =n (m) 8. 2x + 1 = m (x) 13. = (a)
2a 2
3 x
4. 3p + 1 = 4 (p) 9. y1 =4 (y) 14. 16 a = n (a)

2n = 1 (n)
n
5. 10. 2p + q = 3q (p) 15. p+2 = 2p (p)
2

310 Potencias y races

310-311.(2003) 310 20/11/02, 1:34 PM


CAPTULO 6

16.
3
18.
6
2a 1 = b (a) 20. 1 (x)
t+5 =3 (t) x 1 =
6
5
17. a2 =b (a) 19. a 2 =1 (a)

VI. Seale qu condicin se debe cumplir en cada caso para que las expresiones
representen nmeros reales

1. x 1 2. x+1 3. 2a + 1 4. a3

5. 1 2x 6. 3 2x 7. a2 1 8. x2 25

1
9. 9 y2 10. 2x 11. x 12. 1+ a 1 a

1 x1 1
13. 2 x2 14. 15. 16. x
x1 x+1

6 x+4
17. 2a2 18. 19. 20. 1 + a2
6 a x+1

VII. Seale a qu conjunto pertenecen las siguientes races.


(R : nmeros reales; k : nmeros complejos)
3 3
1. 2 2. 2 3. 2 4. 3

3 4 1 4 1
5. 3 6. 7. 8. 1 + 2
3 3
3 3 3
9. 2+ 3 10. 2+ 3 11. 2 + 3 12. 6 + 3 + 3

3 4 3 3 3 3
13. 2 3 14. 1 + 2 5. 1 1 16. 3+ 2

VIII. Simplifique las siguientes expresiones:

6 5
1. 9b2 6. p6 q12 r18 11. m20 n15 t 10

4 6 64 a6
2. 16x2 7. 81 m4 n12 12. +a
b12
7
3. 25a2 b2 c2 8. a21 b7 c14 13.
1
t4
t4
5
4. 81a4 b2 9. 32 m25 a8 a6
14. 6
+
b b8
3
5. 125x3 y6 10. x2 2x + 1

Potencias y races 311

310-311.(2003) 311 20/11/02, 1:34 PM


Ejercicios
a2 + 2ab + b2 a2 + 2a + 1
15. 4a2 + 4a + 1 17. a2 2a + 1

25a6 3
16. 18. x3 3x2 y + 3xy2 y3
a2 12a + 36

IX. Ubique las siguientes races entre dos nmeros enteros consecutivos:
3 4 3
1. 3 5. 100 9. 112 13. 334

3 6 4
2. 5 6. 1.125 10. 1.156 14. 112

3 3
3. 20 7. 4.810 11. 124

3 4 3
4. 35 8. 34 12. 1.149

X. Reduzca a trminos semejantes:

1. 2 +2 2 +3 2 11. 3q a 2q b + 5q b q a

2. 3 3 5 3 6 3 + 9 3 12. 3 7 + 2 28 6 63

3. 5 3 5 11 5 + 2 5 13. 11 2 + 3 8 + 13 12

4. 3 a 4 a + 6 a a 14. 3 3 22 75 5 27

5. 3a 2 + 2a 2 a 2 15. 4 2 3 12 + 3 75 6 8

6. 3 p 53 p + 23 p 16. 2 5 13 20 + 5 45 11 5

17. 3 28 2 20 + 5 80 4 63
7. 3 2 2 +5 3 6 2 + 2

18. 45 + 3 20 11 112
8. 4 6 3 5 5 6 + 2 5
4
19. a b 3a b + 3 a2 b a 4 b2
9. a b 3 a a 3 b
1 2 3 1
10.
n
p 2 n p + 18 n p 4n p 20. a a+ a a
2 5 4 3

XI. Efecte las siguientes multiplicaciones:

1. 2 3 3. 3a 2a 6
3
2. 3 12 4. 3x 3 2x 3 16x2

312 Potencias y races

312-313. 312 08/11/2001, 11:52


CAPTULO 6

2
4 4 4
5. 2p3 5p7 7p6 13. 1 + 2 + 3

6. a1 a1 14. 6 2 6+ 2

7. 3a + 2 3a 2 15. 16 3 16 + 3
2 2
3 3
8. 3x2 yz 2x2 y2 z 16. 3 5 + 3+ 5
2 2
9. 3x + 1 3x 1 17. 1 + 2 2 + 2 3 6
2 2
10. 2 + 2 2 2 18. 31 + 3 2
2 2
11. 1 + 5 1 3 19. a + b a b
2
12. 2 3 3 20. a b a+ b

XII. Efecte las siguientes divisiones:

1. 18 : 2 11. 30 6a 27 18a + 18 12a :6 6a

2. 125 : 5 12. x2 4 : x 2 : x+2


3 3
3. 9a6 b12 : ab5 13. 96x3 : 24x

4.
4
x6 y2 z4 :4 xyz 14. 2x + 3 : 2x

n
5.
5
a2 + 5 a6 + 5 a9 : 5 a2 15. am + 6 : n am 6
6.
3
x2 y :3 x y2 16. x2 8x + 7 : x 7

7. 26a : 2a 17. a2 a2 121 : a a 11

3x 3x
8. 3 128a4 : 6 64a2 18. 22a : 2a 1

9. 444a3 : 111a 19. x 2 25 : x 5

10. 12 20 18 15 :6 5 20. a2 6a + 9 : a 3

XIII. Exprese en forma de una sola raz los siguientes trminos:

3 4
1. 3 2. 2 3. 5a 4. 2 2

Potencias y races 313

312-313. 313 08/11/2001, 11:54


Ejercicios
3 x x
5. 2 3 11. 4+ 2 4 2 16.
5 4
3 2

4
6. a a 12.
2x 2x
17.
6
3 2
3 4
4
3 3x 2 3

8 3 12
7. a a a 13. 15 : 5 18.
5
2 3
4 10
3 2

3 4 4 3 3
8. 5 2 14. 3 5: 2 2 19. 2 3 3 2

n m
a b 3
mn m
9. 15. 3 2 :6 5 20.

10. 3 3 3

XIV. Exprese las siguientes races con un ndice comn:

3 3 x y xy
1. 2 y 2 6. a+1 y a+1 11. 4 , 3 y 6

4 6 4 2a a2 a
2. 2 y 3 7. p+2 y p+1 12. 3n , 3m , mn

3 m n 8a 6a 12 a
3. a y a 8. a y a 13. m6 , m3 , m2

a a2 2a 2a 2b
4. 4 9. 14.
5 y 10 2 , 2 , 2 b, a

a b 4 3
5. 9 y 5 10. a , a , a

XV. Efecte las siguientes operaciones:

1. 3 6 + 5 8 2 2 : 4 2 7. 3 2 2+ 3+ 2 2

2. 3 +5 35 2 8. 2 a a 3 b a + ab a 2 ab a

a+b 4
3. a b 9. a2 + 2ab + b2 a+b
a2 2ab + b2

4
4. a b + b a : ab 10. x2 2xy + y2 + x y

3 3
5. a+b a2 + 2ab + b2 11. 3
3a 2a

4
6. 1 + 2 + 5 1 + 2 5 12. 3 2x 2 3x

314 Potencias y races

314-315. 314 08/11/2001, 11:59


CAPTULO 6

3 3
13. 5 2 : 10 5 17. 5 5+ 5

3 3
14. 4 4 a 2 3 b : 6 ab 18. 3+ 3 3 3

3
15. 3 6 2x + 2 4 3x x
3
x 19. 3
x2 2x + 1 x 1 x1

3 4 4
16. 2 2 2 20. 2 x + 2 3 x + 3

Soluciones
I. 1. 2 2. 5 3. 8 4. 4 5. 10 6. 11 7. 14 8. 10 9. 2 10. 3
1
11. 6 12. 0,1 13. 5 14. 5 15. 4 16. 3 17. 1 18. 2 19. 3
2
9 11
20. 21. 8 22. 29 23. 20 24. 1 25. 1 26. 3,1 27. 28. 0,3
7 2 14
2
29. 0,4 30.
3
II. 1. 1, 4, 9, 16, 25, 36, 49, 64, 81, 100, 121, 144, 169, 196, 225, 256, 289, 324, 361,
400.

2. 1, 8, 27, 64, 125, 216, 343, 512, 729, 1.000, 1.331, 1.728, 2.197, 2.744, 3.375, 4.096,
4.913, 5.832, 6.859, 8.000, 9.261, 10.648, 12.167, 13.824, 15.625, 17.576, 19.683,
21.952, 24.389, 27.000.

3. 1, 4 , 9, 16, 25 36, 49, 64, 81, 100, 121, 144, 169, 196, 225,

256, 289, 324, 361, 400.


3 3 3 3 3 3 3 3 3 3
4. 1 , 8 , 27 , 64 , 125 , 216 , 343 , 512 , 729 , 1.000

6 5 3
III. 1.
4
a3 5. 4
p3 09. 4
5a2 3
13. 2a 17. 5a2bc5

1 3 1 q
8 p
2. m 6. 2 10. m6 n7 14. 2 18. 4m2n
y
5 a 2
1 z 3z
3 2y
3. 5
3 4
7. m n2 11. b 15. 3 19.
3 p
7 2a q 4m
6 5 2 3
4. 2 8. 3pq 12. 3b2 16. 22 20. 5n6
3 1 3 1 5 1 7 10 1
IV. 01. a2 2. 5m 2 3. 2 p2 4 4. 2 6 x 6 5. 5 6 a 6 6. p 11 7. 2 2 m2
2 6 1 1
1 3 1 1 1 2a p 5t z
08. 3 2 p3 q 2 9. x 5 y 5 10. x x y x 11. 81 n 12. 5 13. 3 v
p 2 a
p+q b
14. 2 a 15. 3xy 16. 5 a2 b

Potencias y races 315

314-315. 315 08/11/2001, 12:02


Soluciones
1
V. 1. a = 25 2. z = 8 3. m = n5 4. p = 21 5. n = 6. a = b4 1 7. t = 38
8
n
mn 1 3q q
8. x = 9. y = 4x + 1 10. p = 11. x = 18 12. a = 1 13. a = 2
2 2 16
n2 1 33 b6 + 1
14. a = 15. p = 8
16. t = 22 17. a = b5 + 2 18. a =
16 2

19. a = 1 20. x = 36

1 3
VI. 1. x 1 2. x 1 3. a 4. a 3 5. x 1 6. x
2 2 2
7. a E ] 1 ] K [ 1, + [ 8. x E] , 5] K[5, + ] 9. y E [3,3] 10. x 0
11. x > 0 12. a E [1,1] 13. x 2, 2 14. x > 1 15. x E ] , 1[ K [ 1, + [
16. x < 0 17. a = 0 18. a > 6 19. x E ] , 4 [ K ] 1, + [ 20. I a E R

VII. 1. R 2. R 3. R 4. k 5. R 6. R 7. k 8. R 9. R 10. k
11. R 12. R 13. R 14. R 15. R 16. R

VIII. 1. 3b 2. 4x 3. 5abc 4. 9a2b 5. 5xy2

6. pq2r3 7. 3mn3 8. a3bc2 9. 2m5 10. x1


2a 1 a4 a3 a+b
11. m4n3t2 12. +a 13. t2 14. + 15.
b2 t2 b3 b4 2a + 1
5 a3 a+1
16. 17. a1
18. x y
a6

I X. 1. 1 y 2 4. 4 y 3 7. 69 y 70 10. 3 y 4 13. 6 y 7
2. 2 y 3 5. 5 y 4 8. 2 y 3 11. 5 y 4 14. 3 y 4
3. 2 y 3 6. 10 y 11 9. 3 y 4 12. 11 y 10

X. 1. 6 2 6. 2 3 p 11. 2q a + 3q b 16. 20 5

2. 3 7. 6 3 7 2 12. 11 7 17. 16 5 6 7

3. 11 5 8. 5 6 13. 17 2 + 26 3 18. 9 5 44 7

4. 4 a 9. 3 a 4 b 14. 122 3 19. 0


31
5. 4a 2 10. 13 n p 15. 9 3 8 2 20. a
60

3 4
XI. 1. 6 2. 6 3. 6a 4. 96x4 5. 70 p4
6. a 1 7. 9a2 4 8. y 3 6x4 z2 9. 3x 1

10. 2 11.2 + 2 3 + 2 5 12. 31 12 3

316 Potencias y races

316-317.(2003) 316 25/11/02, 9:46 AM


CAPITULO 6
CAPTULO

13. 6 + 2 2 + 2 3 + 2 6 14. 2 15. 253 16. 16

17. 27 8 2 18. 9 2 3 2 6 19. a2 b 20. ab

5 5 3
XII. 1. 3 2. 5 3. 3
9a5 b7 4. 4
x5 y z3 5. 1 + a4 + a7 6. x y 1

2 9
7. 13 8. a 9. 2 a 10. 4 3 3 11. 5 + 3 2 3 12. 1
2 2
n
13. 2x 14. 23 15. a12 16. x1 17. a a + 11 18.
3x
2a+1
19. x+5 20. a3

4 6 n m 24 12
XIII. 1. 3 5. 12 9. am b 13. 3 17. 72
8 45
6 8 4 20
2. 2 6. a3 10. 2.187 14. 32 18. 864
x
6 18
8 8 14 6
3. 5a 7. a7 11. 15. 5 19. 216
40 2n
4.
4
8 8.
6
250 12.
4x
81 22x x 16. 18 20. mn + 1

6 6 mn mn
XIV. 1. 8 , 4 8. an , am

4 4 2a 2 2a 2 2a 2
2. 4 , 3 9. 22a , 4 , 2a
6 6 12 12 12
3. a2 , a3 10. a3 , a4 , a6

4 4 xy xy xy
4. 5 , 100 11. 4y , 3x , 6

2a 2 2a 2 2a 2
5. ab
9b ,
ab
5a 12. 3n a , 3m 2 , mn 2a

6 6 12a 12a 12a


6. a+1 3 , a+1 2 13. m9 , m6 , m2

12 12 2ab 2ab
7. p+2 2 , p+1 3 14. bb , aa

3
XV. 1. 2 + 4 3 2. 484 3. a+b 4. a + b 5. a + b 6. 2 2 2
4
7. 10 8. 4a3b 6a2b2 + 2a3b2 9. a+b 3 10. 2 x y

6 4 1 6 8 12 a 6 b
11. 108a5 12. 6 18x3 13. 14. 4 2 a
2 25 b 2

6
15. 3 2x3 + 212 27x7 6
x5 16.
12
8.192 17. 5 + 3.125
6

3 4
18. 3 9 19. x1 2 20. 6 x+2 x+3 2

Potencias y races 317

316-317.(2003) 317 25/11/02, 9:50 AM


6.6 Racionalizacin

Definicin: El proceso de racionalizacin consiste en expresar


una fraccin cuyo denominador es un trmino irracional,
es decir, tiene raz irreductible, en otra fraccin equivalente
cuyo denominador es un trmino racional, es decir, no
contiene raz.

6.6.1 Tcnicas de racionalizacin


Veremos aqu los casos ms frecuentes de racionalizacin que
son:

a) Denominador irracional monomio:


A
n
, r<n
pr

n
En este caso amplificamos la fraccin por: pn r

n
A pn r A n pn r
y obtenemos: n

n
=
pr pn r p

b) Denominador binomio (de ndice 2).


A
a b

En este caso la amplificacin adecuada es por



a b

es decir, los mismos trminos del binomio pero con la operacin


opuesta. De este modo obtenemos del producto la diferencia de
cuadrados, con lo cual eliminamos las races:

A a b A a b
=
a+ b a b ab

Observacin 1: Se pueden combinar ambas tcnicas en algunos


casos.
Observacin 2: La segunda tcnica se puede utilizar para casos
de sumas o diferencias de cubos, haciendo una
adecuada amplificacin.

318 Potencias y races

318-319. 318 08/11/2001, 12:31


CAPTULO 6

a
1. Racionalicemos la expresin: 5
Ejercicios
b2
5 3
Amplificando por b obtenemos:
5 5
resueltos
a b3 a b3
5

5
=
b2 b3 b ab
2. Racionalicemos la expresin: 7 3
ab
7 4
Amplifiquemos por ab
7 4 7 4
ab ab ab ab 7 4
= = ab
7
ab 3 7
ab 4 ab
xy
3. Racionalicemos la expresin: x+ y

Amplifiquemos por x y:
xy x y xy x y
= = x y
x+ y x y xy
1
4. Racionalicemos la expresin:
2+ 3
En primer lugar amplifiquemos por 2 + 3 (caso de denominador
monomio).
1 2+ 3 2+ 3
=
2+ 3 2+ 3 2+ 3

Ahora amplifiquemos por 2 3 (caso de denominador binomio).

2+ 3 2 3 2+ 3 2 3
= = 2+ 3 2 3
2+ 3 2 3 43

Ejercicios
I. Racionalizar las siguientes expresiones fraccionarias:
2 a b
1. 2. 3 3. 4. 1 5. 6
2 3
3 b a 2 2 2 3
a mn ab ab 2a2 b
6. 7. 5
8. 9. 10.
3 6 7
ab m2 n 7
a3 b2 ab 5 6 a3 b3
6 3a 3
a 2 2+ 3
11. 12. 13. 14.
2+ 2 a +3 2 3
xy pq
ab 4
15. 16. x y y x 17. ab1 c2 18.
p qq p
a b

19. 5a 20. m n 21. 2


22. 3
6 1+ 2 + 3 2 3+ 5
3 2a m n

a 1 5 3 2
23. 24. 25. 26.
3
3
a+ b 2 23 3 3 55 3 4 2

Potencias y races 319

318-319. 319 08/11/2001, 12:33


Ejercicios
27. Calcular el valor aproximado de la expresin x para x E N, 1< x < 10.
x
x+ 1
Observar los valores obtenidos. Comparar x con y determinar cul
expresin es mayor. x x+ 1

x x+ 1
28. Demostrar que < I x E N.
x x+ 1
29. Calcular el valor aproximado de la expresin x para x E N, 1< x < 10.
x
x con x+ 1
Observar los valores obtenidos. Comparar y determinar cul
x x +1
expresin es mayor.

x x+ 1
30. Demostrar que < I x E N.
x x+ 1

Soluciones
3 ab 2
1. 2 2. 9 3. 4. 4 5. 3
b
3
a2 b2 6 1 7
6. 07. 5
m3 n4 8.
7
a 4 b5 9. ab 10. 3 a 66a4b4
b
3
3a a 3 a a b a+ b
11. 6 3 2 12. 13. a 14. 7 + 4 3 15.
a9 a2 b
x y +y x 4
ab3 c2 p q +q p 5a 3 2a 3 + 2a
16. xy 17. 18. pq 19.
bc 9 2a

6 5 1+ 2 3 2 3+ 5 2 6
20. m n m+ n 21. 22.
2 4
3 3 3
a a2 a b +b a b 2 2 +3 3 3 5 +5 3
23. 24. 25.
19
a2 b3 6
3 3
3 2 16 + 4 2 +2 4+ 8
26.
8
27. 1.414; 1.732; 2; 2.236; 2.449; 2.645; 2.828; 3; x < x+ 1
x x+ 1
28. Sug.: Racionalizar ambas expresiones
x x+ 1
29. 0.707; 0.577; 0.5; 0.447; 0.408; 0.377; 0.353; 0.3 ; >
1 x x+ 1
30. Sug.: 1 <
x2 (x + 1)2

6.7 Ecuaciones
irracionales

Definicin: Son aquellas ecuaciones que presentan la


variable como cantidad subradical. Para resolverlas
debemos elevar a la potencia adecuada tantas veces sea
necesario hasta eliminar la raz (o las races).

320 Potencias y races

320-321. 320 20/11/02, 1:51 PM


CAPTULO 6

1. Resolvamos la ecuacin x+7 =5


Ejercicios
Vemos que elevando al cuadrado ambos miembros de la desigualdad
resueltos
la raz se elimina.

x+7 =5 /( )2
x + 7 = 25 /7
x = 18

2. Resolvamos la ecuacin 1 + 5 + 3x + 4 = 2
Procedamos a elevar al cuadrado paso a paso:

1 + 5 + 3x + 4 = 2 /( )2

1+ 5 + 3x + 4 = 4 /1

5 + 3x + 4 = 3 /( )2

5 + 3x + 4 = 9 /5

3x + 4 = 4 /( )2

3x + 4 = 16 / 4
1
3x = 12 / 3
x = 4

3. Resolvamos la ecuacin x +5 x 3 = 2
Elevamos al cuadrado ambos miembros de la igualdad.
Observamos que el primero es un binomio.

x+5 x3 = 2 /( )2

x+5 2 x+5 x 3 +x 3 =4

2x + 2 2 x+5 x 3 = 4
1
2x 2 = 2 x+5 x 3 /
2
x1 = x+5 x 3 /( )2

x2 2x + 1 = x2 + 2x 15 / x2

2x + 1 = 2x 15

4x = 16

x =4

Potencias y races 321

320-321. 321 20/11/02, 1:52 PM


Ejercicios

I. Resuelva las siguientes ecuaciones:

1. x+3 =3 2. 2x 7 = 13 3. 2 15x = 8

4. 1 + 2x = 2 5. 2 2x = 3 x 6. 1 + 7x = 2 2

7. 2 + 3x 6 = 6 8. 5 2 3x = 15 9. 3+ 4+ x 8 = 3

10. 2x + 13 4 = x 5 11. x2 + 5 3 = x 12. x + 9 = x2 5

13. 2x2 + 3 = 5 x 3 14.


3
2x = 2 15.
3
3x + 5 = 1

16. 4+5 x 1 =3 17. 1+ 2 x + 7 = 3 18. 4 x+1 +2 =3 2

19. 3x + 1 + 2x 1 = 7x + 2 20. 2 x 3 + x + 1 = 3
10 5 2
x2 x
21. x+2 = 22. x5=
x+1 x+8
x9 2x x 3x
23. 2x + 6 = 24. + =
x5 5 3 5
25. 2x 1 = x + 3 x 2 26. 1 + x = x + 7

27. 2x 1 + 2x+1=3 28. ax bx = a b b a

29. a x b x = ab 30. x x 1
+ =
a b ab
1
31. 2x 5 1 = 32. 2x 7 x 3 = x + 4
2x 5
3x
33. 2 = x+5 34. 1 + 2x 1 = 1 2x
x+5
35. m 2n = mn 36. a b x 2
= ba x 2
x x

Soluciones
15 8
I. 1. x = 6 2. x = 88 3. x = 2 4. x = 5. x1 = y x2 = 0
5 2 81
22 27
6. x = 7 7. x = 8. x = 9. x = 1.032
3 4
2 43
10. x1 = 6; x2 = 54 11. x = 12. x = 9
3

13. x1 = 13 ; x2 = 6 14. x = 4 15. x = 4 16. x = 2


2 3

17. x = 9 18. x = 15 19. x1 = 2; x2 = 1 20. x = 16


5
2
21. x = 22. x = 40 23. x1 = 7 + 4 10; x2 = 7 4 10
7 3

322 Potencias y races

322-323. 322 20/11/02, 2:00 PM


CAPITULO 6
CAPTULO

1 + 29 1 29
24. x = 0 25. x1 = 2
; x2 =
2
26. x = 9 27. x = 85
72
a2b2 1
28. x = ab 29. x = 2 30. x= 2
ab a+b
13 + 5 13 5 1 + 113 1 113
31. x 1 = 4
; x2 =
4
32. x1 = ; x2 =
2 2

2
9 + 61 9 61 3
33. x1 = ; x2 = 34. x = 35. x = m 2n 36. x = 1
8 8 8 2 2
m n

Prueba de seleccin mltiple


Potencias y Races (Marque la alternativa correcta).
3
4. 2n1 2n + 1 7. 1 2
1. a5 a3 a1 = a
2
A. a15 21 A. 8a6
A. 2n
B. a15 8a5
B. 4n B.
C. a8 n2 1 6
a
C. 2 C. 2
D. a7
D. 42n D.
1 5
a
E. a7 2 1
2
E. 4n 1 6
E. a
2. El valor de 5x2 8
5. El valor de (3x5y2z4)0
Si x = 2 es: Si x = 2 y = 1 z=1 8. 44 + 44 + 44 + 44
A. 20 A. 96 A. 410
B. 20 B. 96 B. 210
C. 100 C. 1 C. 25
D. 100 D. 1 D. 216
E. 10 E. 32 E. 416
3. El cuadrado de 6. 6m3n5 : 2m2n3
9. 93 94 =
3m3 es: A. 3mn2
A. 9m6 A. 35
B. 3m2n
B. 9m9 B. 912
C. 9m6 C. 3mn2
C. 314
D. 9m9 D. 3m2n D. 312
E. 9m3
E. 3mn3 E. 95

Potencias y races 323

322-323. 323 20/11/02, 2:01 PM


Prueba de seleccin mltiple
1
10. El valor de x en 14. 2 3 2 2 2 3 2 1 = E.
x 44x
2 3
= es: A. 83
3 2
B. 218 18. (0.5)x (0.1)x (40)x =
A. 1

B. 1 C. 8 A. 2x
D. 23 B. 4x
C. 0
3 E. 163 C. 20x
D.
2
15. Los valores de x e y en:
E. 2 D. 23x
3 ax + y =a
bx : by = b2 E. Otro
11. (31)2 =
son respectivamente:
A. 9 19. (31)2 (32)1 (31)2 =
1 1 1
B. 1 A. ,
A.
2 2 3
9
C. 3 1 1
B. , B. 9
2 2
1 1
D. 3 1 C.
3 C. , 9
2 2
E. 9 D. 3
3 1
D. ,
2 2
12. De las afirmaciones: E. 9

I an + an = a2n E. 3 , 1
2 2 20. El valor de x en
II an an = a2n 16. Los valores de y x2 2x 1 x
9 = 32 es:
III an an = an para x = 1 son respec- 2

son verdaderas: tivamente: A. 5


1
A. Slo I A. 1 , B. 5
2
1 1
B. I y II B. 1 , 2 C. 5
C. Slo II C. 1 ,2 D. 1
D. II y III 5
D. 1 , 2
E. Todas E. 4
E. 1 , 2
13. El valor de x en 21. El valor de x en
3x + 1 = 9 x es: 17. 4x : 82x =
4 3x 3 x = 27 es:
1
A. 3 A. x A. 1
2
B. 1 1
B. B. 2
24x
C. 2
C. 3
C. 1
D. 1 4x D. 2
E. 3 D. 1
E. 3
22x

324 Potencias y races

324-325. 324 08/11/2001, 12:53


CAPITULO 6
CAPTULO

1 1
22. x3 + x5 = D. 1 D. (a b)u v
3
1 1
A. E. E. Ninguna.
x8 4
2
B. x5
x2 +1 26. Si x = 2 el valor de 30. El valor de
C.
x5 2x + 1 3x 2 2 x 1 es:
2
x +1
D. x2 2y 3x y3
x15 A. 8
si x = 1, y = 2 es:
E. Otro B. 8
2
A.
C. 32 3
23. De las proposiciones: 2
B.
D. 16 3
I an bn = (ab)n 1
C.
E. 24 3
II (a + b)n = an + bn 1
D.
3
III an : am = an + m
27. La solucin de x en E. Otro
Son falsas: (23x 4)0 = 1 es:
3 12
31. =
A. I, II A. 2
4
B. Slo II B. 3 3
4 A.
4 2
C. I y III C. 6
3 B.
2
D. II y III D. No existe solucin
C. 3
E. Ninguna E. Cualquier valor real
D. 6

24. (a + b)x + y : (a + b)x y = 28. 23x 4 3x 5 2 2 3x 33 x =


E. 2 3
A. (a + b)2x 4
A. 9
32. 6 3 =
B. (a + b)2x 9
B.
4 A. 9
C. (a + b)2y 1
C.
36 B. 9 2
D. (a + b)2y
D. 36
C. 2 3
E. (a + b) 1
E.
27 D. 3 2
25. El valor de x en
E. 18
29. (a + b)u (a b)v =
2x 22x + 1 = 64x es:
A. (a2 b2)uv
A. 3
B. (a2 b2)u + v
B. 3

C. 1 C. (a + b)u + v
3

Potencias y races 325

324-325. 325 08/11/2001, 12:54


Prueba de seleccin mltiple
2
33.
3
64 = 37. 1 + 2 = 41. 2 3 6=

A. 3 A. 6
A. 2
3 B. 5 + 2 2 B. 6 6
B. 16
6
C. 3 + 2 2 C. 6
C. 2
6
D. 9 D. 30
D. 8
3 E. 2 6
E. 4 E. 5

38. 3 2 3+ 2 = n
anm =
34. a b a b = 42.
1
A. 1 A. a m
A. ab
1
B. 2 B. a n
B. ab2
m
C. 3 2 C. a n
C. a2 b2
D. 5 D. an
D. a3 b
E. 5 E. am
E. a2 b

39. 3 2+ 3 8 = 43.
3
25 5 =
3

35. a2 b3c4 =
A. 3 6 A.
6
5
A. abc2 b
B. 3 + 6 B. 6
25
B. a2 b c2 b
C. 3 6 C. 5 5
3

C. abc c
6
D. 3+ 6 D. 5 5
D. abc2
E. 3 E. 5
E. abc2 c
2
40. Al racionalizar se 44. Si a = 3, b = 4, entonces
2
36. a a = obtiene: el valor de b2 a2 es:
4 A. 2 2 A. 1
A. a

B.
4
a3 B. 2 B. 5
2
C. a C. C. 7
2
D. a a D. 4 2 D. 7
4
E. a a E. 2 E. 7

326 Potencias y races

326-327. 326 08/11/2001, 13:00


CAPITULO 6
CAPTULO

3
45. Al simplificar a15 b9 49. 3 2 2 18 = 52.
x
6x + 1 =
se obtiene:
A. 6 A. 6x
A. a12 b6 x
B. 36 B. 6
B. a5 b3 x
C. 12 C. 6 6
C. a 5 b3 D. 6 6 D. 6
D. a 12
b 6
E. 6 12 E. Otro
E. a12 b9
4 50. De las afirmaciones 53. La solucin de la
46. Al racionalizar
51 siguientes: ecuacin x + 2 = 5 es:
se obtiene:
I a 2 b2 = a 2 b2 A. 3
A. 5 +1
II a 2 + b2 = a + b B. 23
B. 51
III a b = b a C. 8
C. 4 5 +1
son verdaderas: D. 23
D. 4 51
5 +1 A. Slo I E. 8
E.
4
B. Slo II
54. La solucin de
C. Slo III
47. 3 7 3+ 7 = 1 + 2x 3 = 4 es:
4 D. Todas
A. 2 A. 0
E. Ninguna B. 3
B. 40

C. 40 51. De las afirmaciones, C. 6


m n n m 3
D. 2 I a= a D.
2
m
E. 2 II a n = n am E. Otro

III an m = an am
2b
48. El valor de aa 2 si 55. La solucin de
son falsas: 7 + 2 + x 3 = 3 es:
a = 2 y b = 3 es:
3 A. Slo III
A. 16 A. 4
3 B. I y III
B. 4 B. 3
6 C. II y III
C. 3 C. 5
6
D. Todas
D. 4 D. 7
E. Ninguna
E. Otro E. 9

Potencias y races 327

326-327. 327 08/11/2001, 13:02


Prueba de seleccin mltiple
3 3
56. En x+2 =2 , 58. Para que la expresin B. x < 5
el valor de x es: 2x 3 sea real es
A. 0 necesario y suficiente C. x > 5
3
B. 6 que: 5
D. x < 3
C. 16 A. x 3
2 E. x > 3
D. 62 B. x 5
3
E. 64 3
C. x 4 1
+
3 1
+
1
=
2 60. 81 27 9
x+1 2+ x 1 2 =2 3
57. En D. x 1
2
A. 3
el valor de x es: 2
E. x
3 1
A. 1 B. 9
B. 2 59. Para que la expresin
3 C. 3
C. 0 sea real es ne-
3 5x
D. 3 D. 1
cesario y suficiente que:
3
E. 4 A. x > E. Otro
5

Soluciones
Clave de Respuestas:
1. D 11. A 21. B 31. D 41. C 51. A
2. B 12. C 22. C 32. D 42. E 52. C
3. C 13. D 23. B 33. A 43. E 53. B
4. B 14. C 24. D 34. E 44. C 54. C
5. D 15. D 25. C 35. A 45. B 55. D
6. A 16. A 26. D 36. B 46. A 56. D
7. A 17. B 27. E 37. C 47. D 57. A
8. B 18. A 28. C 38. A 48. B 58. C
9. C 19. C 29. E 39. A 49. B 59. B
10. B 20. A 30. A 40. B 50. E 60. D

328 Potencias y races

328. 328 08/11/2001, 13:05


CAPTULO 7
L ogaritmos

Denicin de logaritmo 7.1

Sean a, x E R+, a 1. Decimos que y es el logaritmo en base a de


x si y slo si x = ay, lo que escribimos y = loga x.
NOTA: y es el exponente al que hay que elevar la base a
para obtener el nmero x.

Observacin 1: y = loga x es una funcin real cuyo dominio es


R+ y su rango o recorrido es R.

y = log2 x
2

11 1 1 2 3 4 5 6 7 8 9
84 2

Solamente se puede calcular logaritmo de nmeros reales positivos.


Observacin 2: Cada valor real positivo distinto de 1 que toma la base
a da origen a un sistema completo de logaritmos.
Si la base es 10 se acostumbra no escribirse y el sistema de
logaritmos de base 10 se llaman logaritmos vulgares, decimales
o logaritmos de Briggs.

Logaritmos 329

329. 329 8/11/01, 17:41


Si la base es e = 2,7128... entonces el sistema se denomina
de logaritmos naturales o neperianos y se acostumbra a anotar
por y = ln x.

Observacin 3: Si la base toma un valor entre 0 y 1, entonces


la grfica queda como sigue:

11 1 1 2 3 4 5 6 7 8 9
84 2

-1

-2

-3
y = log 1 x
2

7.2 Propiedades

1. loga a = 1 El logaritmo de la base es 1.


2. loga 1 = 0 El logaritmo de 1 es 0.
3. loga M N = loga M + loga N.
El logaritmo de un producto es igual a la suma de los logaritmos
de los factores.
M
4. loga = loga M loga N.
N
El logaritmo de un cociente es igual al logaritmo del numerador
menos el logaritmo del denominador.
5. loga MP= p loga M.
El logaritmo de una potencia es igual al exponente multiplicado
por el logaritmo de la base.
logbN
6. loga N = Teorema de cambio de base.
logba
7. En el sistema de logaritmos en base 10, a la parte entera del
logaritmo de un nmero se le llama caracterstica y a su parte
decimal se le llama mantisa.

330 Logaritmos

330-331. 330 8/11/01, 17:51


CAPTULO 7

1. Calcular: log2 128 Ejercicios


Solucin: resueltos
Se debe encontrar el exponente al que hay que elevar la base 2 para que
d 128. Como 27= 128, entonces:
log2 128 = 7

2. Calcular log 1 27
3

Solucin:
1 3
1 2
= 31 y 27 = 27 2= 3
3
Aqu la pregunta es a cunto debemos elevar la base 31 para
3
2
que d 3 ?
3
(3 1)x = 32
3
3 x = 32

x = 3
2
Luego, log 1 27 = 3
3 2
3. Calcular log5 3 125
Solucin:
1
3
Como 125 = (53) 3 = 51 debemos encontrar x tal que 5x = 51 , de donde
3
x = 1. Luego, log5 125 = 1.

4. Calcular log27 1
9
Solucin:
Debemos encontrar la forma de expresar la base del logaritmo y
el nmero al cual se le busca el logaritmo como potencias del
mismo nmero.
1
27 = 33 y = 91 = 32
9
Debemos hallar x tal que (33)x = 32, es decir, 3x = 2 de donde
2
x= .
3
1 2
Luego log27 =
9 3

125
5. Calcular log 3
5 27
Solucin:
125 53 5 3 3 3
= = =
27 33 3 5

Logaritmos 331

330-331. 331 8/11/01, 17:53


x 3
Ejercicios Debemos hallar x tal que
3
=
3
. Es decir, x = 3
5 5
resueltos Luego, log 3
125
=3
5 27
6. Graficar la funcin y = log3 x. En base al grfico responder las
siguientes preguntas.
a) Cul es su dominio?
b) Cul es su recorrido?
c) Qu signo tiene el logaritmo en base 3 de los nmeros menores
que 1?
d) El logaritmo en base 3 de qu nmeros est entre 2 y 2?
e) Graficar la funcin y = log3 (x 2) e indicar su dominio y su
recorrido.
Solucin:
y
2

1 y = log3 x

1 1 1 2 3 4 5 6 7 8 9 x
9 3

a) Dom log3 x = R+
b) Rec log3 x = R

c) Negativo. Si observamos el grfico, el logaritmo en base 3 de cual-


quier nmero menor que 1 es negativo. Por ejemplo, log3 1 = 1
3
1
d) Observamos que log3 = 2 y log3 9 = 2, luego el logaritmo en
9
base 3 de los nmeros que estn entre 1 y 9 estn entre 2 y 2
9
e)
y y = log3 (x 2)
2

1 219 7 3 4 5 6 7 8 9 x
9 3

2
Dom log3 (x 2) = {x E R / x > 2}
Rec log3 (x 2) = R

332 Logaritmos

332-333. 332 8/11/01, 17:46


CAPITULO 7
CAPTULO

1
7. Calcular el valor de log a + log a
Solucin:
1 1
log a + log a = log (a a ) = log 1 = 0
N
8. Si log6 N = r. Determinar el logaritmo en base 6 de 216
Solucin:
N
log6 = log6 N log6 216 = r 3
216
3a b2
9. Desarrollar la expresin log 1 . Escribirla en trminos de
c
log a, log b, y log c.
Solucin:
3a b2
log = log 3ab2 log c1
c 1
= log 3 + log a + 2 log b + log c

10. Escribir como un solo logaritmo la expresin


1 1 1
2 log a 2 log b 2 log c
Solucin:
1 1 1 1 1 1
log a 2 log b log c = log a 2 log b 2 log c 2
2 2
= log a (log b + log c )

= log a log b c
a a
= log = log
b c bc

Ejercicios

1. Escriba como potencia del nmero que se indica los siguientes nmeros:

a) 2, 4, 8, 16, 64, 256 de 2 1


c) 2, 4, 8, 16, 64, 256 de
2
b) 1, 1 , 1 , 1 , 1 , 1 1 1 1 1 1 1
de 2 d) , , , , , de 1
2 4 8 16 64 256 2 4 8 16 64 256 2
2. Escriba los siguientes nmeros como potencia de 10.
1 1 1
a) 10; 100; 1.000.000 b) 10 ; 100; 1.000.000
c) 1; 0,1; 0,001; 0,0000001

3. Escriba como un logaritmo de base 3 los siguientes nmeros:

a) 1; 2; 3; 4 b) 1; 2; 3; 4

Logaritmos 333

332-333. 333 8/11/01, 17:48


Ejercicios
4. Calcule los siguientes 6. Calcule los siguientes m) log 1 2
4
logaritmos: logaritmos:
3
n) log 81
a) log 1 a) log 1 1
2

b) log 10 b) log 1 2 o) log 1 4


2 2

c) log 100 c) log 1 4


2
p) log9 243
d) log 1.000 1
d) log 1
2 2 q) log0,3 0,0081
e) log 1 1
10 e) log 1 4
2 r) log50 1
f) log 1
100 f) log 2
1 8
2 s) log 3
g) log 0,01 2
27
3
g) log 2
h) log 0,0001 1
2
t) log 1 1.000
10
1
h) log 1 8. Encuentre entre qu poten-
i) log 10 2 8
cias de 10 est el nmero
i) log 1 8 cuyo logaritmo decimal es:
j) log 1.000 2
3 1 a) 0,5
5. Calcule los siguientes j) log 1 4
logaritmos: 2 b) 1,2
7. Calcule los siguientes c) 2,8
a) log 2 1 logaritmos:
d) 3,5
b) log 2 2 a) log7 343 e) 0,5
b) log3.459 1 f) 1,7
c) log 2 4
c) log 64 g) 2,3
8
1 3 4 h) 3,7
d) log 2 d) log5
2 6
e) log8 32 9. Encuentre entre qu nme-
1
e) log 2 ros enteros est el logaritmo
4
f) log2 32 decimal de: (bosqueje el
f) log 2 2 grfico de y = log x)
g) log3 125
0.2
a) 7
3
g) log 2 2 h) log9 27
b) 9,27
4
1 i) log4 16 c) 12,58
h) log 2 8
j) log27 1 d) 83,025
3
i) log 2 8 k) log 128 e) 135
16
f) 2.992,16
3 1 1
j) log 2 4 l) log3 g) 0,27
9

334 Logaritmos

334-335. 334 20/11/02, 3:28 PM


CAPTULO 7

h) 0,349 b) 1, 2 c) log (a b) (a2 2 ab + b2)= 2


i) 0,052 c) 2, 2
d) log 1 27 = 3
d) 3, 2
j) 0,0116 3

e) 4, 2 18. Calcule el valor de la


k) 0,0098 incgnita:
f) 5, 2
l) 0,000145
a) log5 25 = x
14. Usando una calculadora
10. Escriba la caracterstica cientfica determine el
de los logaritmos deci- b) log5 x = 3
nmero cuyo logaritmo
males de los nmeros en base 2 es:
del ejercicio anterior. c) logx 27 = 3
a) 0, 2
d) log0,027 x = 1
11. Determine en qu base b) 1, 2 3
el logaritmo de: 1
c) 2, 2 e) log32 =x
2
a) 125 es 3
d) 3, 2 16
f) logx =2
3 36
b) 8 es
2 e) 4, 2 1
g) log0,008 x =
c) 16.384 es 7 3
f) 5, 2
d) 4 es 2 h) logx 1 = 2
4
e) 9 es 2 15. Escriba alguna relacin i) log5 x = 1
f) 16 es 2 entre los resultados del
ejercicio 13 y los del j) logx 8 = 3
4
12. Escriba el logaritmo de ejercicio 14. 2
k) logx 4 =
los siguientes nmeros 3
en funcin de log 2. 1
16. Exprese x en forma de l) log3 x =
3
logaritmo en cada igual 1
a) log 4 m) log2 =x
dad siguiente: 64

b) log 1 a) 4x = 1 n) log81
1
=x
16 3
1 b) 14x = 17
c) log o) log4 x = 3
32 2
c) ax = m n
6
d) log 5 p) log2 x =
5
d) qx = a+b
e) log 125 q) logx 8 = 2

f) log 0,5 17. Escriba en forma de r) logx 8 = 3


13. Usando una calculadora potencia las siguientes
igualdades: s) logx 27 = 3
cientfica determine el
nmero cuyo logaritmo
t) logx 9 = 2
decimal es: a) loga b = p

a) 0, 2 b) log5 4 = p u) log 2 x = 0
3

Logaritmos 335

334-335. 335 20/11/02, 3:29 PM


Ejercicios
1 2
w) logx = 24. Grafique y = log 1 x
4 3
2
1 2
v) log3 1 = x x) logx = y = log x
1
3 3 3

19. Aplicando las propiedades de logaritmo, calcule: 25. Si la base del logaritmo es
mayor que 1, qu signo
1
a) loga 1 + logb bn + logc tienen los lo ga rit mos
cn
de los nmeros mayores
b) loga a2 + logb b3 que 1, y de los nmeros
menores que 1?
c) loga ab + loga a
b 26. Si la base del logaritmo es
3 4
d) loga a + logb b + logc c menor que 1, qu signo
tienen los loga ritmos
e) log1.000 log3 92 de los nmeros mayores
que 1, y de los nmeros
f) log 0,1 log 0,01 menores que 1?

g) log 1 1 + log 2 3 + log9 1 27. Encuentre usando una


4 3 2 3 calculadora:
h) log2 3 +log 1 3 (Sug.: escriba en notacin de potencia) a) El logaritmo en base
2
3 de 4, 6, 8, 10.
20. Aplicando las propiedades escriba los siguientes logaritmos
b) El logaritmo en base
desarrollados:
1
de 4, 6, 8, 10.
4 a2 b3 2
a3 b 3
a) log b c c) log e) log Qu relacin existe entre
a ac c2 a3
los resultados?
3
3 a2 b2
a2 b
b) log d) log 5 b 28. Demuestre que:
c
c
loga x = log 1 x
a
21. Aplicando las propiedades, reduzca las expresiones
I x E R+
siguientes:

a) log a + log b 2 log c 29. Demuestre que:

1 3 1 1 loga b logb c logc a = 1


b) log a + log b log c log d
2 2 2 2
30. Si loga p = 3 y
1 2 1
c) log a log b + log c loga 36 p = 5, calcule a.
3 3 3
1 1 1 31. Calcule el valor de:
d) log x log y log z
2 3 4
1 1
1 loga + logb
22. Demuestre que: log9 x = log3 x a b
2
23. Grafique las funciones siguientes: 32. Calcule el valor numrico
de:
a) y = log2 (x + 1) c) y = 1 + log2 x logx x2
a) 3
b) y = log2 (x 1) d) y = 1 log2 x

336 Logaritmos

336-337. 336 20/11/02, 3:35 PM


CAPTULO 7

b) 4log x x 34. Demuestre que 36. Calcule el valor


1 numrico de:
c) log (log 1010) log + log y = 0
y 3 logb b + logb b2+ logb b5
d) loga aa log a a 2
35. Si log x = y, encuentre
33. Si log 2 = 0,3010 y log 3
= 0,4771, encuentre sin log(10x 2) en funcin
calculadora log 120. de y.

Soluciones

1. a) 21, 22, 23, 24, 26, 28 b) 21, 22, 23, 2 4, 2 6, 2 8

c) 1 1
1 2
1 3
1 4
1 6
1
d) 1 1 1 2 1 3 1 4 1 6 1 8
, , , , , , , , , ,
2 2 2 2 2 2 2 2 2 2 2 2

2. a) 10; 102; 106 b) 101; 102; 106 c) 100; 101; 103; 107

1 1 1 1
3. a) log3 3; log3 9; log3 27; log3 81 b) log3 ; log3 ; log3 ; log3
3 9 27 81
4. a) 0 b) 1 c) 2 d) 3 e) 1

1 3
f) 2 g) 2 h) 4 i) j)
2 2
5. a) 0 b) 1 c) 2 d) 1 e) 2

1 1 3 3 2
f) g) h) i) j)
2 3 2 2 3
6. a) 0 b) 1 c) 2 d) 1 e) 2

1 1 3 3 2
f) g) h) i) j)
2 3 2 2 3

5
7. a) 3 b) 0 c) 4 d) 0 e)
3
5 9 3 1 1
f) g) h) i) j)
2 2 4 2 3
7 4
k) l) 2 m) 0,5 n) o) 4
2 3
5 3
p) q) 4 r) 0 s) 3 t)
2 2
8. a) entre 1 y 10 b) entre 10 y 102 c) entre 102 y 103 d) entre 103 y 104
e) entre 101 y 1 f) entre 102 y 101 g) entre 103 y 102 h) entre 10 4 y 103

9. a) entre 0 y 1 b) entre 0 y 1 c) entre 1 y 2 d) entre 1 y 2

Logaritmos 337

336-337. 337 20/11/02, 3:35 PM


Soluciones

e) entre 2 y 3 f) entre 3 y 4 g) entre 1 y 0 h) entre 1 y 0


i) entre 2 y 1 j) entre 2 y 1 k) entre 3 y 2 l) entre 4 y 3

10. a) 0 b) 0 c) 1 d) 1 e) 2 f) 3
g) 1 h) 1 i) 2 j) 2 k) 3 l) 4

11. a) 5 b) 4 c) 4 d) 2 e) 3 f) 4

5
12. a) 2 log 2 b) 4 log 2 c) log 2
2
d) 1 log 2 e) 3 3 log 2 f) log 2

13. a) 1,58489319 b) 15,8489319 c) 158,489319


d) 1.584,89319 e) 15.848,9319 f) 158.489,319

14. a) 1,148698355 b) 2,29739671 c) 4,59479342


d) 9,18958684 e) 18,37917368 f) 36,75834736

15. En el ejercicio 13, cada nmero obtenido es igual al anterior multiplicado por 10. En el
ejercicio 14, cada nmero obtenido es igual al anterior multiplicado por 2.

16. a) x = log4 1 b) x = log14 17


c) x = loga m n d) x = logq a + b

1 3
17. a) ap = b b) 5p = 4 c) (a b)2 = a2 2ab + b2 d) = 27
3

1
18. a) 2 b) 125 c) 3 d) 0,3 e)
5
2 1
f) g) 0,2 h) 2 i) 5 j)
3 16
1 3 1 1
k) l) 3 m) 6 n) o) 8
8 4
5 2 1 1 1
p) 2 2 q) r) s) t)
4 2 3 3

u) 1 v) 0 w) 8 x) 3 3

13
19. a) 0 b) 5 c) 2 d)
12
3
e) 1 f) 1 g) h) 0
2
3 3
20. a) 4 log b + 2 log c 4 log a b) 6 log a + log b log c
2 2

338 Logaritmos

338-339.(2003) 338 20/11/02, 3:44 PM


CAPITULO 7
CAPTULO

7 1
c) 3 log a 6 log b + 2 log c d) 2 log a + log b + log c
15 5
3 1
e) log a + log b log c
4 4
ab ab3 3 ac x
21. a) log b) log c) log d) log
cd b2 4
c2 3
y z

23.
a) y = log2(x + 1) b) y = log2(x 1)
3
3
2 2
1 1

1 12 1 2 3 4 5 6 7 2 3 4 5 6 7 8 9
1 1

2
2

c) d)
y = 1 + log2x
4

3
3
2 2
1
1 1
-1 21 2 4 5 6 7 8 16
1 -2
1 2 3 4 5 6 7 8
2 -3
-1 y = 1 log2x

-2

-3
24.

3 3

2 2

1
1
1
1 3
1 2 3 4 5 6 7 8 9
1 2 3 4 5 6 7 8
2 -1
-1
-2
-2
-3
-3

25. Positivo, negativo


26. Negativo, positivo

Logaritmos 339

338-339.(2003) 339 20/11/02, 3:45 PM


Soluciones

27. 30. 6 33. 2,0791


a) 1,261859507 b) 1,261859507
1,630929754 1,630929754 31. 2 35. 1 + 2y
1,892789261 1,892789261
2,095903274 2,095903274 32. 36. 0
log3 x = log 1 x x = 4, 6, 8, 10 a) 9 b) 2 c) 1 d) 2a
3

7.3 Ecuaciones exponenciales


y logartmicas

Se llaman ecuaciones exponenciales aquellas ecuaciones que


presentan la incgnita en el exponente:

3x = 1 o 23x 1 = 3x + 2
Se llaman ecuaciones logartmicas aquellas ecuaciones que
presentan la incgnita como argumento de una funcin logartmica:
log x = 2 o log (3x 1) = log (x + 2)

Para resolver ecuaciones exponenciales podemos igualar las


bases y aplicar:

ax = ay x = y

Ejemplo: 3x = 1
3x = 30

x=0
En ocasiones no es posible igualar las bases, y en estos casos
podemos aplicar el concepto de funcin logartmica. Como sta es
biyectiva en su rango se tiene:

log x = log y x = y

Ejemplo: 23x 1 = 3x + 2
log 23x 1 = log 3x + 2
(3x 1) log 2 = (x + 2) log 3
3x log 2 log 2 = x log 3 + 2 log 3
x (3 log2 log 3) = 2 log 3 + log 2
2 log 3 + log 2
x =
3 log 2 + log 3
340 Logaritmos

340-341. 340 8/11/01, 18:25


CAPTULO 7

Para resolver ecuaciones logartmicas tambin debemos aplicar


el concepto de funcin biyectiva:
log x = log y x = y
Ejemplo:
log x = 2 2 = log 100
log x = log 100

x = 100
Ejemplo:
log (3x 1) = log (x + 2)

3x 1 = x + 2
2x = 3
2
x=
3

1. Resolver la ecuacin 22x = 5 Ejercicios


Solucin: Aplicando logaritmo. resueltos
log 22x = log 5 aplicando log ap = p log a
2x log 2 = log 5 despejando x
log 5
x= 2 log 2 con una calculadora

x= 0,69897
20,3010
x = 1,16096
2. Resolver la ecuacin 5 52x 1 = 2
Solucin: Para aplicar logaritmo debemos efectuar primero la
multiplicacin 5 52x 1 = 52x

log 52x = log 2


2x log 5 = log 2

x= log 2
2 log 5
0,3010
x=
20,6989
x = 0,2154
3. Resolver la ecuacin:
3x 1 + 3x 2 + 3x 3 + 3x 4 = 1.080

Logaritmos 341

340-341. 341 8/11/01, 18:25


Ejercicios Solucin:

resueltos 3x (31 + 32 + 33 + 3 4) = 1.080


1 1 1 1
3x 3 + 9 + 27 + 81 = 1.080

40
3x = 1.080
81
1.080 81
3x =
40
3x = 2.187
3x = 37

x= 7

4. Resolver la ecuacin
4x 2 2 x + 3 + 64 = 0
Solucin:
Haciendo 2x = y nos queda una ecuacin de segundo grado.
4x 2 2 x 2 3+ 64 = 0
(2x)2 16 2x + 64 = 0
y2 16y2 + 64 = 0
(y 8)2 = 0
y= 8
Luego 2x = 8 de donde x = 3

5. Resolver la ecuacin
8x 9 8x = 8
Solucin:
8x 9 8x = 8
1
8x 9 = 8 / 8x
8x
82x 9 = 8 8x
82x 8 8x 9 = 0 Sea 8x = y

y2 8y 9 = 0
y1 = 9
(y 9) (y + 1) = 0
y2 = 1

Si y1 = 9 8x = 9 x = log 8 9 x = 0,94639
Si y2 = 1 8x = 1 x no existe.

6. Resolver la ecuacin 6x 32x + 2 =20

342 Logaritmos

342-343 342 8/11/01, 18:38


CAPTULO 7
CAPITULO

Solucin: 6x 32x + 2 = 20
6x 32x = 18 aplicando log
log 6x 32x = log 18
log 6x + log 32x = log 3 + log 6
x log 6 + 2x log 3 = log 3 + log 2 + log 3
x (log 2 + log 3 + 2 log 3) = 2 log 3 + log 2

2 log 3 + log 2
x=
3 log 3 + log 2

7. Resolver la ecuacin

log x + log (x + 1) = log 6


Solucin: Aplicando propiedades de logaritmo el primer miembro
queda log x (x + 1).

log (x2 + x ) = log 6



x2 +x= 6
x2 +x6= 0
(x + 3) (x 2) = 0
cuyas soluciones son x1 = 3 y x2 = 2
Una vez obtenidas las soluciones de la ecuacin de segundo grado
debemos comprobar que los valores obtenidos sean en realidad solucin
de la ecuacin logartmica planteada. Recordemos que el dominio
de la funcin logartmica es R+ y no R, por lo tanto, cualquier valor
obtenido para la incgnita que haga negativo o cero el argumento de algn
logaritmo, no es solucin de la ecuacin planteada. En este caso x = 3 no
es solucin ya que log (3) y log (2) no estn definidos.
x = 2 es solucin ya que:

log 2 + log (2 + 1) = log 6


log 2 + log 3 = log 6
log 2 3 = log 6
log 6 log 6
8. Resolver la ecuacin:
log x log (x + 3) = 1

1 x
Solucin: 1 = log y log x log (x + 3) = log
10 x+3
x 1
luego log = log
x+3 10
x
= 1
x + 3 10

Logaritmos 343

342-343 343 8/11/01, 18:38


Ejercicios 10 x = x + 3
resueltos 9x=3
1
x=
3
que es solucin de la ecuacin planteada.
En efecto:
1 1
log log + 3 = 1
3 3
10
log 1 log 3 log = 1
3
log 1 log 3 log 10 + log 3 = 1
0 1 = 1
1 1
9. Resolver la ecuacin:
2log2 (x + 2) log2 (x + 1) = 2
Solucin:
x+2 2
log2 = log24
x+1

x+2 2
=4
x+1
(x + 2)2 = 4 (x + 1)
x2 + 4x + 4 = 4x + 4
x2 = 0
x =0
comprobando
2log2 2 log21 = 2
{
{

1 0
2 1 0 = 2
22

10. Resolver la ecuacin


log x3 = log2 x 4
Solucin:
3 log x = log2 x 4 Sea u = log x
3u = u2 4
2
u 3u 4 = 0 u= 4
(u 4) (u + 1) = 0
u = 1
Si log x = 4 x = 10.000
Si log x = 1 x = 0,1

Ambos valores satisfacen la ecuacin.

344 Logaritmos

344-345. 344 8/11/01, 18:42


CAPITULO 7
CAPTULO

2x + 2x
11. Despejar x en la expresin y =
2x 2x
Solucin:
Sea u = 2x 2x = 1 1
=
2x u
1
u+
u u
y= 1
/ u
u
u
u2 + 1
y= 2
u 1
u2y y = u2 + 1
u2y u2 = y + 1
u2 (y 1) = y + 1
y+1
u2 =
y1
Como u = 2x entonces u2 = 22x

22x = y + 1 aplicando log


y1
2x log 2 = log (y + 1) log (y 1)
log (y + 1) log (y 1)
x=
2 log 2
12. Resolver el siguiente sistema:
2x+y = 6
2x = 3 2y 1

Solucin: Aplicando logaritmo en ambas ecuaciones:


(x + y) log 2 = log 6
x log 2 = log 3 + (y 1) log 2

x log 2 + y log 2 = log 6


x log 2 y log 2 = log 3 log 2

Sumando: 2 x log 2 = log 6 + log 3 log 2


log 2 + log 3 + log 3 log 2
x=
2 log 2
log 3
x= = log2 3 = 1,5849
log 2
Restando: 2 y log 2 = log 6 log 3 + log 2
log 2 + log 3 log 3 + log 2
y=
2 log 2
y=1
Luego la solucin del sistema es (log2 3, 1)

Logaritmos 345

344-345. 345 8/11/01, 18:43


Ejercicios 13. Resolver el siguiente sistema:
log (x + 1) + log 2 = log y
resueltos
log x + log (y + 1) = log 5

Solucin: Aplicando las propiedades de logaritmo:

log[(x + 1) 2] = log y
log [x (y + 1)] = log 5

2x + 2 = y (1)
xy + x = 5 (2)

Reemplazando y en (2)

x ( 2x + 2) + x 5 = 0

2x2 + 3x 5 = 0
x1 = 1
x = 3
9 + 40
=
4 x2 = 10 = 5
4 2
Si x1 = 1 y1 = 4
5
Si x2 = y2 = 3
2
La segunda solucin obtenida no es solucin del sistema, ya que log
[(x + 1) 2] sera el logaritmo de un nmero negativo.
Por lo tanto, la solucin es (1, 4).

14. Demostrar que:


log a2 + 1 + a + log a2 + 1 a = 0 para todo a 0.

Solucin:

log a2 + 1 + a + log a2 + 1 a =

log a2 + 1 + a a2 + 1 a =
log (a2 + 1 a2) = log 1 = 0

15. Encontrar la base del sistema de logaritmos en que el logaritmo de 80


excede al logaritmo de 5 en 2 unidades.
Solucin: Sea x la base buscada:

logx 80 logx 5 = 2

logx 80 = 2
5
logx 16 = 2
luego, x = 4

346 Logaritmos

346-347. 346 8/11/01, 18:51


CAPITULO 7
CAPTULO

Ejercicios
1. Resuelva las siguientes ecuaciones:
1
a. 3 x = 12 c. 2 x + 2 = 5 x + 1 e. 2 2 x 1 =
3
b. 2 x 1 = 32x 1 d. 3 2x 3 2 4x 1 = 0 f. 3 3x + 1 = 3 2 x + 3
2. Resuelva las siguientes ecuaciones:

a. 2x 1 + 2x + 2x + 1 = 14 e. 2 x + 2x + 1 + 2x + 2 + 2x + 3 = 15

b. 3x 1 + 3x + 3x + 1 = 13 f. 3x + 3x + 1 + 3x + 2 = 39

c. 5x 1 + 5x + 5x +1 = 31 g. 3x + 3x 1 + 3x 2 = 13

d. 5x 1 + 5x + 5x + 1 = 62 h. 21 x + 22 x + 23 x = 7
3. Resuelva las siguientes ecuaciones:

a. 4x 9 2x + 1 + 81 = 0 d. 52x + 3 8 5x + 1 + 3 = 0

b. 9x + 3x 12 = 0 e. 32x + 2 5 3x + 1 + 4 = 0
Nota: Reduzca las ecuaciones precedentes
c. 2 32x 7 3x + 3 = 0 a ecuaciones de segundo grado.
4. Resuelva las siguientes ecuaciones:

a. 10x + 10x = 2 d. ax + 24ax 11 = 0

b. 5x + 16 5x 8 = 0 e. 2x + 5 22 x 9 = 0
Nota: Reduzca las ecuaciones precedentes a
c. 5x + 15 5x 8 = 0
ecuaciones de segundo grado.
5. Resuelva las siguientes ecuaciones:

a. log x + log 3 = log 15 f. 2 log x = 2

b. log 2 log x = log 3 g. 2 log x + log 4 = 2

c. log x 2 log 3 + log 2 = 0 h. log x3 = log 3 + log x2

d. 2 log x = 2 i. log x5 = 3 + log x2

e. 2 log 2 x = 4 j. 2 log 3 x log 3 2 = 2

6. Resuelva las siguientes ecuaciones:

a. log (x + 3) = log (2x 1)


b. log (x + 1) + log (x 2) = log (x 3) + log (x + 5)
c. 2 log (x + 1) log (x 1) = 1
d. log x = 1 + log (11 x)
e. log (3x 4) log (2x + 1) = log (2x 1) log (3x + 4)

f. 2 log (x + 4) log (x 1)2 = log 3

Logaritmos 347

346-347. 347 8/11/01, 18:51


Ejercicios d)
log x 2 log y = 5
g) log (x + 1) = log 3 + log (x 3) 3 log x log y = 3
h) 2 log2 (x + 2) log2 (x + 1) = 2
i) 2 log3 (x + 2) = log3 9 e)

j) 3 log2 (x + 1) = log2 (x + 1)4 log2 (x + 2y) = 1


log3 (2x + y) = 0
7. Resuelva las siguientes ecuaciones:
11. Resuelva el siguiente sistema:
a) 2 log3 x = (log3 x)2 + 1
1 1 log2 (x + y) + log2 (x y) = 6
b) log x 2 = (log x) 2
2x 2y = 16
c) 2 log x = (log x )2

d) log x3 = log2 x + 2
12. Resuelva el siguiente sistema:

8. Resuelva los siguientes sistemas: (x + y)log (x y) = 100


a) d) x2 y2 = 1.000
2x y = 3x 22x 3 = 4y 1
3 2x + 1 = 2y 1 6x 6 = 3y 13. Resuelva el siguiente sistema:
b) e)
log2 (x + y) log2 (x y) = 2
2x 1 = 3y + 2 2 4x y 2 = 2x
3x 3y = 81
3x y =2 3 x + y : 3 = 3y

c) 14. Resuelva el siguiente sistema:


2x + y = 6
2x = 3 2y 1 log3 (x + y) + log3 (x y) = 5
ex : ey = e27
19. Si 3x + y = 2 y 2x y = 3 pruebe que
x + y = log3 2 y que x y = log2 3
15. Determine qu relacin debe existir entre
p y q para que se cumplan las siguientes
10. Resuelva los siguientes sistemas: relaciones:
a) a) log p log q = 1
log (x + 3) + log 2 = log (x + y) b) 2 log p + log q = 2
log y log 3 = log (x + 1)
16. Determine dos nmeros naturales x
b) e y tales que:
2 log x log y = 3 y log x x log y = 0
log x + log y = 1
x2 y2 = 12

c)
17. Determine dos nmeros tales que la suma
log (x + 1) log 2 = log y
de sus cuadrados sea 10.100 y que la suma
log x log (y + 1) = log 3
de sus logaritmos decimales sea 3.

348 Logaritmos

348-349. 348 20/11/02, 3:47 PM


CAPTULO 7

18. Determine dos nmeros sabiendo que 21. Encuentre el sistema de logaritmos (su
la diferencia de sus logaritmos en base base) en que el logaritmo de 108 excede al
2 es 1 y que la suma de sus cuadrados logaritmo de 12 en 2 unidades.
es 1.280.
22. En las siguientes expresiones, despeje la
19. Determine dos nmeros que estn en la incgnita que se indica:
razn 3:1, sabiendo que la suma de sus
logaritmos en base 3 es 7. a) log2 y = x + k ; y
b) log x 2 log y = 0 ; x
20. Encuentre el sistema de logaritmos (su
base) en que el logaritmo de 324 excede al c) ln x = ln x0 t ; x
logaritmo de 81 en 2 unidades. d) log3 k = log3 4 - 2 log3 x ; x
e) ln (30 c) = ln 30 2 t ; c

Soluciones
2 log 2 log 2 log 3 log 5 2 log 2
1. a) 1 + log 3 b) c)
log 2 2 log 3 log 2 log 5
3 log 3 log 2 log 3 3 log 2
d) e) f)
2 log 3 4 log 2 log 2 3 log 3 log 2
1
2. a) 2 b) 1 c) 1 d) e) 0 f) 1 g) 2 h) 1
log 5
2 log 3 log 2 log 3 2 log 5 log 4 log 3
3. a) b) 1 c) 1, d) 1, e) 1,
log 2 log 3 log 5 log 3
log 4 log 3 log 3 log 8 log 5
4. a) 0 b) c) 1, d) , e) 2,
log 5 log 5 log a log a log 2
2 9
5. a) 5 b) c) d) 10 e) 4 f) 0,1 g) 5 h) 3 i) 10 j) 3 2
3 2
13 7+5 3
6. a) 4 b) c) 4 5 d) 10 e) 3 f) 2
g) 5 h) 0 i) 1 j) 0
3

7. a) 3 b) 1, 10.000 c) 1, 100 d) 10, 100


2 log 2 + log 3 2 log 2 + log 3 log 2 log 3 2 log 3 + log 2 2 log 3 + log 2 log 2
8. a) , b) ,
log 3 log 2 log 3 log 2 log 3 log 2 log 3 log 3

log 3 + 2 log 6 3 log 6


c)
( log 3
log 2
,1 ) d) ,
2 (log 3 log 6) 2 log 6 + 2 log 3
e) (1, 1)

9. Sugerencia: resuelva el sistema formado con las dos condiciones dadas.


3 1
10. a) 3 15
, b) 10 10 , 3
2 2 10
c) No tiene solucin porque los valores que resultan para x e y hacen que
algn argumento de logaritmo sea menor o igual que cero

d) 5 5 e) (0, 1)
0.1 , 0.01 0.01
11. (10, 6) 12. (55, 45) 13. (10, 6) 14. (18, 9)

Logaritmos 349

348-349. 349 20/11/02, 3:47 PM


Soluciones

15. a) p 10q = 0 18. 16 y 32 22. a) y = 2x + k

b) p2 q = 100 b) x = y2
19. 81 y 27
c) x = x0 e t
16. 4 y 2 20. base 2 k
d) x = 2
k
17. 100 y 10 21. base 3 e) c = 30 (1 e 2 t)

Prueba de seleccin mltiple

1. Si log k = x, entonces 4. Si log x = a, entonces 7. log 27 =


3
log 100k = log x =
A. 1
A. 100 + k A. a B. 3
B. 100 + x C. 6
B. 2a
C. 2+k
C. 1 a D. 9
D. 2+x 2
E. 2x D. a E. 12
1
E. a 2
2. Si log x = 0,3495, 8. log 81 9 =
entonces log x2 = 5. Si log x = y, entonces
A. 2
A. 0,3495 log 10x3 =
B. 1
B. (0,3495)2
A. 1 + 3x 1
C.
C. 2 0,3495 2
B. 1 + 3y
D. 4 0,3495 1
D.
C. 10 + 3x 2
E. 4,3495
D. 10 + 3y E. 1
3. Si log x = a y
E. 30y
log y = b, entonces
1
log 3 xy = 9. log 27 =
6. Si e y = 3, entonces y = 3
A. log 3 A. 3
A. 3 a + 3 b
B. ln 3 B. 1
B. 3 a b
1 1 C. log 3 log e 1
a+ b C.
C. 3 3 3
D. ln 3 ln e 1
D.
D. 1 a b 3
3
E. 3
a+b E. ln 3 E. 1
e

350 Logaritmos

350-351. 350 8/11/01, 19:46


CAPITULO 7
CAPTULO

1 18. Si y = alog ax,


10. log + log x = 14. Si log x2 y = a
x
x entonces x vale:
y log = b,
1
A. log x y2
x A. loga y
B. log x entonces log y =
1 B. logy a
C. 1 A. (a 2b)
3 C. y
1
D. 0 B. (2a + b)
5 D. 0
E. 1 1
C. (a + 2b) E. 1
3
1 19. La expresin
D. (2a b)
11. El valor de 5 2 log a x log a y
1 a a = 1 es
logq p logp r logr q es: E. (a 2b)
5 equivalente a:
A. pq r 15. Si log x + log 3 = A. 2 loga x + loga y = 0
1 log 60 log 20,
B.
pqr B. 2 loga x loga y = 0
entonces x =
C. p + q + r
A. 0 C. 2 loga x loga y = 1
D. 1 B. 1
D. 2 loga x + loga y = 1
E. 0 C. 3
D. 10 E. y = x2
12. La expresin loga b logb c E. 33
es equivalente a: 20. De las siguientes expresio-
16. Si log n log x =
nes; son equivalentes:
A. logb c 2 log y 1, entonces n = I. blog b x b2 log b y = blog b 1
B. logc b x2y II. logb x + logb y2 = 0
A.
C. loga c 10 III. xy2 = 1
xy2
D. loga bc B. A. Slo I y II
10
E. logb ac C. 10 x2 y B. Slo II y III
C. Slo I y III
D. 10 x y2
a D. Todas
13. La expresin log es
b2c E. x (y 1)2 E. Ninguna
equivalente a:
17. Si px 2 = qx + 1,
21. La expresin
A. log a 2 log b + log c entonces x = 5 loga a loga a4 + loga a2
A. 2 log p vale:
B. log a 2 log b + 2 log c
B. 2 log p + log q A. 2
C. log a 2 log b log c C. 2 log p log q B. 1
2 log p + log q C. 0
D.
D. log a 2 log b 2 log c log p log q D. 1
2 log p + log q
E. log a + 2 log b + log c E. E. 2
log q log p

Logaritmos 351

350-351. 351 8/11/01, 19:46


Prueba de seleccin mltiple
22. La expresin 25. Si 2 x 2 + 2x + 2 = 17, 28. En la expresin
loga 5 + log 1 5 vale: entonces x vale: log 3 x = p + q, x vale:
a
A. 2 A. 2 A. 3p + q
B. 1 B. 1 B. 3q + p
C. 0 C. 0 C. 3p + q
D. 1 D. 1 D. 3p q
E. 1 E. 3p + 3q
E. 2
1
23. El valor de log3 8+ log3
8 29. Si ln y = ln y0 t,
es igual a: 26. Si 31 x 3x 1 = 8 3 1,
entonces x vale: entonces y vale:
A. 2
A. 2 A. e t y0
B. 1
B. 1 B. e t y0
C. 0
D. 1 C. 0 C. e t + y0

E. 2 D. 1 D. e t y0

E. 2 E. et + y0
24. La expresin
1 1 log x 2 log y
log2 + log3 es: 27. Si log (x + 3) log (x +2) 30. Si = ,
3 2 2 3
= log 2, entonces x vale: entonces :
A. log2 3 log3 2
A. 2 A. x3 y4 = 0
B. log2 3 log3 2
B. 1 B. x3 + y4 = 0
C. log2 3 + log3 2 C. 3x 4y = 0
C. 0
D. log2 3 + log3 2 D. 1 D. 3x + 4y = 0

E. 2 E. x = 3 y
E. log2 5

Soluciones

Clave de respuestas

1. D 6. B 11. D 16. B 21. B 26. C


2. D 7. C 12. C 17. D 22. C 27. D
3. C 8. C 13. C 18. C 23. C 28. C
4. C 9. D 14. E 19. A 24. B 29. A
5. B 10. D 15. B 20. D 25. A 30. A

352 Logaritmos

352. 352 8/11/01, 19:53


CAPTULO 8
T rigonometra

Sistemas de medicin
de ngulos

Una unidad de medida para ngulos es el grado sexagesimal o sim-


plemente grado. El ngulo obtenido por una revolucin completa en
sentido opuesto a las agujas del reloj mide 360 grados; por lo tanto,
1
un grado es 360 de una circunferencia.

Otra unidad de medida de ngulos es el radin.


Un radin es la medida del ngulo central de una circunferencia que
subtiende un arco de la misma longitud que su radio.

Relacin entre ambos sistemas:


1 =
( )
180
rad
Nota: En general se omite la
1 rad =
( )
180
palabra rad; as, un ngulo
puede medir 2 (en vez de
3
2 rad).
3

Trigonometra 353

353-354-355(2003) 353 20/11/02, 3:52 PM


Razones trigonomtricas para
ngulos agudos
Sea a un ngulo agudo en el tringulo rectngulo ABC, de catetos
a y b y de hipotenusa c. Las razones trigonomtricas son: seno,
coseno, tangente, cosecante, secante y cotangente y se definen:

B c
a cosec a = = hipotenusa
sen a = = cateto opuesto a cateto opuesto
c hipotenusa
c hipotenusa
b sec a = =
cos a = = cateto adyacente b cateto adyacente
c hipotenusa c
a
b
a cot a = = cateto adyacente
tg a = = cateto opuesto a cateto opuesto
b cateto adyacente

a
C b A

Identidades trigonomtricas

Definicin. Una identidad es una igualdad que se verifica


para todos los valores posibles de la variable.

Son identidades bsicas:

1
1. cosec a =
sen a 5. ctg a = cos a
sen a
2. sec a = 1 6. sen2 a + cos2 a = 1
cos a
1 7. 1 + tg2 a = sec2 a
3. ctg a =
tg a
sen a 8. 1 + ctg2 a = cosec2 a
4. tg a =
cos a

Su demostracin es consecuencia directa de la definicin de razones


trigonomtricas en 8.2 (ver ejercicio resuelto n 10).

354 Trigonometra

353-354-355(2003) 354 20/11/02, 3:52 PM


CAPTULO 8

Funciones trigonomtricas de un
ngulo cualquiera

Sea a un ngulo cualquiera en un sistema de coordenadas rectangulares


y sea P(x,y) un punto cualquiera de su lado terminal.

Si r = x2 + y2, definimos:

y
sen a = r II I
x
cos a = r
y P (x, y)
y
tg a = x
y

r a
csc a = y
o x
III IV
r
sec a = x

x
ctg a = y

Funciones trigonomtricas de 60, 30


y 45, 0, 90, 180 y 270

funcin
ngulo sen cos tg cosec sec ctg
Los valores de las funciones
3 1 3 2 3 3
o 60 2 trigonomtricas de los ngulos
3 2 2 3 3
de 30, 45, 60 y de los ngu-
3 3 2 3 los cuadrangulares (su lado ter-
1 3
o 30 2 3 2 3 minal coincide con un lado de
6 2
un cuadrante del sistema car-
2 2 2 2 tesiano) son usados frecuente-
o 45 2 2 1 1 mente, y se presentan en la
4
tabla adjunta: (ver ejercicios
0 o 0 0 1 0 indef. 1 indef. resueltos nos 12 y 13)


o 90 1 0 indef. 1 indef. 0
2
o 180 0 -1 0 indef. -1 indef.

3
o 270 -1 0 indef. -1 indef. 0
6

Trigonometra 355

353-354-355(2003) 355 20/11/02, 3:52 PM


Funciones peridicas

Una funcin f es peridica si existe un nmero real positivo P tal que


f(x + P) = f(x) para todo valor de x en el dominio de f.
El nmero real P se llama perodo de la funcin f.
Las funciones seno y coseno, secante y cosecante son peridicas de
perodo 2 (360).
Las funciones tangente y cotangente son peridicas de periodo (180).

Ejemplos:
a) sen a = sen (a + 2) b) tg a = tg (a + 180)
7 3 3
sen = sen = tg 30 = tg 210 =
3 3 2 3

Funciones pares e impares


Una funcin f es par si f(x) = f(x) para todo valor de x en el dominio de f.
Una funcin f es impar si f(x) = f(x), para todo valor de x en el dominio
de f.
Las funciones coseno y secante son funciones pares.
Las funciones seno, cosecante, tangente y cotangente son funciones
impares.
Ejemplos:
a) cos a = cos ( a) b) sen a = sen ( a)

cos
6
= cos (6
)= 2
3
( 12 ) =
sen 30 = sen (30) =
1
2

Ecuaciones trigonomtricas

Definicin. Una ecuacin trigonomtrica es aquella que con-


tiene la variable dentro de una expresin trigonomtrica. Las
soluciones de estas ecuaciones son ngulos expresados en
grados o radianes.

Ejemplos:
1
a) sen x = b) 4tg x 2cos x 4 + 2 =0
2
c) tg (2x + ) = 1

Ver ejercicios resueltos nos 17 al 22.

356 Trigonometra

356-357(2003) 356 20/11/02, 3:54 PM


CAPTULO 8

Resolucin de tringulos
no rectngulos

Se llama resolver un tringulo a determinar la medida de sus tres lados


y de sus tres ngulos interiores. Los siguientes teoremas se verifican en
todo tipo de tringulos.

8.9.1 Teorema del seno (o de los senos)


Sean a, b y g los ngulos interiores de un tringulo ABC cualquiera
y sean a, b y c los respectivos lados. Se cumple:
C
sen a sen b sen g g
= =
a b c
a
b
a b
A c B

8.9.2 Teorema del coseno


(o de los cosenos)
Sean a, b y g los ngulos interiores de un tringulo ABC cualquiera y
sean a, b y c los respectivos lados. Se cumple: C
a2 = b2 + c2 2bc cos a g

b2 = c2 + a2 2ac cos b
a b
c2 = a2 + b2 2ab cos g A B

8.9.3 Angulos de elevacin y depresin


OBJETO
a es ngulo de elevacin
Se llama ngulo de elevacin al ngulo
formado por la horizontal y la recta que
une al observador con el objeto cuando el
objeto est sobre el observador.
a
OBSERVADOR

Se llama ngulo de depresin al ngulo


formado por la horizontal y la recta que OBSERVADOR b
une al observador con el objeto cuando el
objeto est bajo el observador.
b es ngulo de depresin. OBJETO

Ver ejercicios resueltos 14 al 16, 23 y 24

Trigonometra 357

356-357(2003) 357 20/11/02, 3:54 PM


Ejercicios 1. Exprese en radianes la medida de los ngulos:
resueltos ( )

a) 120 = 120 180 rad = 2 rad
3

b) 54 = 54
( ) rad = 3 rad
180 10

2. Exprese en grados la medida de los ngulos siguientes:



a)
4
3
=
4
3

( ) 180

= 240 b) 11
6
=
11
6

( ) 180

= 330

2
3. Sea a un ngulo agudo y tg a = .
3
Determinemos las dems funciones.
B cateto opuesto
Solucin: Como sabemos, tg a =
cateto adyacente
Dibujamos un tringulo ABC y a es uno de sus
ngulos agudos. Asignamos el valor de la tan-
2 gente, como en la figura, y luego determina-
mos la hipotenusa, aplicando el Teorema de
Pitgoras.

AB = 9+4 = 13
a
C 3 A
2 13
As tenemos: sen a = csc a =
13 2
3 13
cos a = sec a =
13 3
2 3
tg a = ctg a =
3 2
4. Determine las funciones trigonomtricas del ngulo b sabiendo que
2+b
sec b =
b
hipotenusa
Solucin: Sabemos que sec b =
cat. adyacente
A
Aplicamos Teorema de Pitgo-
ras para determinar cateto AC
2+b
AC 2 + b2 = (2 + b)2

AC 2 = 4 + 4b + b2 b2

b AC = 4 + 4b = 2 1+b
C b B
2 1+ b 2+ b
Por lo tanto: sen b = cosec b =
2+ b 2 1+ b
b 2+b
cos b = sec b =
2+ b b
2 1+ b b
tg b = ctg b =
b 2 1+ b

358 Trigonometra

358-359 358 09/11/2001, 10:43


CAPTULO 8

4
5. Determine las funciones trigonomtricas del ngulo a si sen a =
3

cateto opuesto
Solucin: Como sen a =
hipotenusa
Vemos que no es posible asignar dichos valores a los lados de un
tringulo rectngulo, pues un cateto no puede ser mayor (ni igual) a la
4
hipotenusa. Por lo tanto, no existe ningn ngulo cuyo seno sea .
3

Desafo: Averige qu son funciones acotadas y entre qu valores


estn acotadas las funciones trigonomtricas.

6. Demuestre que sen a ctg a = cos a.

Analizamos el lado izquierdo de la igualdad y aplicamos las identida-


des que sean necesarias hasta obtener la expresin del lado derecho.

sen a ctg a = sen a cos a


sen a
= cos a

7. Demuestre que sen a (csc a sen a) = cos2 a

Procediendo como en el caso anterior, tenemos que:

sen a (csc a - sen a ) = sen a ( 1 sen a)


sen a

= sen a sen2 a
sen a
= 1 sen2 a
= cos2 a (identidad 6 pg. 354)

8. Demuestre que tg a = tg a 1
1 ctg a
En este caso analizaremos el lado derecho de la igualdad (es ms
sencillo simplificar una expresin trigonomtrica que amplificarla).
Aplicaremos las identidades que sean necesarias hasta obtener la
expresin del lado izquierdo.
tg a 1 = tg a 1
1 ctg a 1 1
tg a

= tg a 1
tg a 1
tg a

= tg a - 1 tg a = tg a
tg a 1

Trigonometra 359

358-359 359 09/11/2001, 10:43


Ejercicios 9. Demuestre que cos4 b + sen2 b = cos2 b + sen4 b
resueltos Procediendo como en los casos anteriores, analizamos el primer miem-
bro de la igualdad y aplicamos las identidades correspondientes:

cos4 b + sen2 b = (cos2 b)2 + sen2b

= (1 sen2b)2 + sen2b

= 1 2 sen2b + sen4 b + sen2b

= 1 sen2b + sen4 b

= cos2 b + sen4 b

10. Demuestre que sen2 a + cos2 a = 1

B Consideremos el M rec-
tngulo ABC y a uno de sus
ngulos agudos.
Por Teorema de Pitgoras se
c cumple:
a
a2 + b2 = c2 /:c2

a
C b A () ()
a 2+ b 2=1
c c

Aplicando las definiciones de razones trigonomtricas, tenemos:

sen2 a + cos2 a = 1

11. Sea a un ngulo en posicin estndar en un sistema de ejes coordena-


dos, esto es, el lado inicial de a coincide con la parte positiva del eje
x, el vrtice del ngulo es el origen del sistema, y sea P(-4,3) el punto
del lado terminal de l. Determine todas las funciones trigonomtricas
de a.

Solucin: Primero deter-


minemos la longitud del P
segmento OP = r. 3

3
r= 32 + 42 r = 5
a
4 0
4

360 Trigonometra

360-361 360 09/11/2001, 10:48


CAPTULO 8

y ahora aplicamos las definiciones de las funciones trigonomtri-


cas de a: (x = 4; y = 3; r = 5)

y 3 r 5
sen a = = cosec a = =
r 5 y 3
x 4 r 5
cos a = = sec a = =
r 5 x 4
y 3 x 4
tg a = = cot a = =
x 4 y 3


12. Encuentre las funciones trigonomtricas para a = 60 ( o a = )
3
C
Solucin: Consideremos
el M ABC equiltero de
lado 2 y sea CD su altura.
En M rectngulo ADC se 2 2
tiene que h2 + 1 = 4 . h

h= 3 a
y DAC = 60 A
D B
1
3
Entonces: sen 60 = cosec 60 = 2 = 2 3
2 3 3
1
cos 60 = sec 60 = 2
2
1 3
tg 60 = 3 cot 60 = =
3 3

Desarrolle el mismo ejercicio pero para un tringulo equiltero de lado


3, 5, a.

13. Encuentre las funciones trigonomtricas para a = 90 (o )
2
Solucin.

El punto (0,1) pertenece al (0, 1)


lado terminal de a.
a
As: x = 0; y = 1; r = 1

y r
sen 90 = =1 cosec 90 = = 1
r y
x r
cos 90 = =0 sec 90 = = indefinida
r x
y x
tg 90 = = indefinida cot 90 = =0
x y

Trigonometra 361

360-361 361 09/11/2001, 10:48


Ejercicios 14.Desde un punto P situado a nivel el suelo, el ngulo de ele-
resueltos vacin de la cima de una torre es de 30. Si la distancia
entre el punto P y la base de la torre es 12 metros, deter-
mine la altura de sta.

30

P
12 m
La figura ilustra la situacin planteada.
El tringulo determinado es rectngulo; un cateto es informa-
cin dada y el otro cateto es la incgnita. Una funcin que
relaciona los dos catetos es la tangente (la otra es la cotan-
gente)

h
As tg 30 = 12

h
Pero tg 30 = 1 entonces = 1
3 12 3
despejando nos queda: h = 12
3
y racionalizando : h = 4 3

Solucin: la torre mide 4 3 metros, aproximadamente 6,9


metros.

15. Desde un punto P situado a nivel del suelo se observa


la punta de una chimenea bajo un ngulo de elevacin
de 30 y acercndose 20 metros desde otro punto Q el
ngulo de elevacin es de 60. Determine la altura de la
chimenea y la distancia desde sta hasta el primer punto
de observacin (P).

362 Trigonometra

362-363 362 20/11/02, 3:55 PM


CAPTULO 8

La figura muestra la situacin


planteada.
Debemos determinar
h y d = x + 20. h
h
Tenemos tg 60 = x
60 30
h
y tg 30 = x + 20
x Q P
20 m
3 1
Como tg 60 = y tg 30 = se tiene
3
h 3 y (2) h 1
(1) x = x + 20 = 3
x 3 1
de (1) h = x 3 reemplazando en (2) =
x + 20 3
As 3x = x + 20
2x = 20
x = 10
Por lo tanto, h = 10 3

Solucin. La chimenea mide 10 3 metros (17,3m) y la distancia


desde ella al primer punto de observacin es 30 metros.

16. Un nio eleva un volantn con


una cuerda tensa que forma un
ngulo de elevacin de 60 con
la horizontal. A qu altura se
encuentra el volantn del suelo
si la longitud de la cuerda es de
18 m x
18 metros y el nio mide 1.50
metros (o el nio tiene la cuerda
a 1.50 m del suelo)? d
60

La figura representa la situacin


planteada. En este caso, una 1.5 m
informacin es el largo de la
cuerda, lo que corresponde a la
hipotenusa del tringulo. Enton-
ces aplicamos la funcin seno.

x x 3
sen 60 = 3
18 18 = 2 x=9

La distancia desde el volantn al suelo es: d = x + 1,5


d = 9 3 + 1,5
d 17 m.

Desafo: Averige y construya los grficos de las funciones trigonomtricas.

Trigonometra 363

362-363 363 20/11/02, 3:55 PM


Ejercicios 1
17. Resuelva la ecuacin: sen x =
resueltos 2

1
Solucin: Sabemos que sen 30 = ; por lo tanto, una solucin particular
2
es x1 = 30.
Pero adems en la circunferencia geomtrica (de radio 1) la funcin
seno queda definida por el eje y, y tambin es positiva en el 2 cua-
drante; por lo tanto, ah hay otra solucin particular.
y
Observando la circunferencia
unitaria (de radio 1), vemos 1
que: 2
150 30
x2 = 150 tambin es solucin x
de la ecuacin
Ntese que 150 = 180 30

As, las soluciones particulares de la ecuacin son:

x1 = 30
( 6 )
o

y x2 = 150
( 6 )
o 5

y para obtener las soluciones generales, agregamos a cada solucin


particular, mltiplos del periodo.


Y las soluciones generales son: x1 = + 2k
6

x2 = 5 + 2k
6

y
2
18. Resolver la ecuacin: cos x =
Solucin: Sabemos que para
2
4

x1 = 45 (o )
4
2
el coseno vale
2
Tambin sabemos que el co- 2 x
seno es positivo en el 1 y 4 2
cuadrante (pues queda deter-
minado por la coordenada x),
7
entonces tambin hay una solu-
4
cin en el 4 cuadrante, que es
x2 = 315 (360 - 45)

Y las soluciones generales son: x1 = + 2k
4

x2 = 7 + 2k
4

364 Trigonometra

364-365 364 20/11/02, 3:57 PM


CAPTULO 8

19. tg x = 3
Determinamos la solucin particular primera, o el ngulo de referen-

cia, este es x = 60 (o )
3
Y luego determinamos el otro
cuadrante donde la tangente es
positiva, este es el tercer cua-
drante. En ese cuadrante est
la 2a solucin particular y es
x1 = 60
x2 = 240 = 180 + 60

Las soluciones son: x1 = + k
3
4
x2 = + k
3
20. 2cos 2x = 3
3
Nos queda cos 2x =
2
3
Llamemos 2x = y, tenemos cos y =
2
De acuerdo con los anlisis anteriores, obtenemos:
y1 = 30 e y2 = 330

y1 = + 2k y2 = 11 + 2k
6 6
pero y = 2x, entonces las soluciones son:

x1 = + k y2 = 11 + k
12 12

21. 4 sen2 x tg x tg x = 0
Factorizamos: tg x (4 sen2 x 1) = 0
tg x (2 sen x 1) (2 sen x + 1) = 0
Un producto de 3 factores es cero si cualquiera de ellos es cero. As:
y
tg x = 0 (tg a = ) x1 = 0
x
x2 = 180
1
2 sen x = 1 sen x = x3 = 30
2
(ngulo de referencia 30) x4 = 150

1
2 sen x = 1 Q sen x = Q x5 = 210
2
(ngulo de referencia 30) x6 = 330

6

Solucin: {k, + 2k; 150 + 2k; 210 + 2k; 330 + 2k}
6

Trigonometra 365

364-365 365 20/11/02, 3:57 PM


Ejercicios 22. 1 2 sen2 x = cos x

resueltos Aplicamos identidades para expresar todas las funciones en trminos de


coseno.

1 2(1 cos2 x) = cos x


1 2 + 2 cos2 x = cos x
2 cos2 x 1 = cos x

(esta es una ecuacin de 2 grado en cos x)

2 cos2 x cos x 1 = 0

Factorizando, obtenemos:

(2 cos x + 1) (cos x 1) = 0

Entonces:
1
2 cos x + 1 = 0 Q cos x = Q x1 = 240
2
(ngulo de referencia 60) x2 = 300

cos x 1= 0 Q cos x = 1 Q x = 0
(coordenada x)
60 60

Solucin: {4 + 2k, 5 + 2k, 2k}
3 3

23. Resolver el tringulo ABC, dados:


a = 36 b = 64 a = 12

Segn el teorema del seno.


a b 12 b
= =
sen a sen b sen 36 sen 64
Usando calculadora, obtenemos:
12 b
=
0,5878 0,8988
de donde b = 18,35

Ahora, como a = 36 y b = 64, concluimos que g = 80 y aplicamos


nuevamente teorema del seno:
a c 12 c
= =
sen a sen g sen 36 sen 80
Usamos la calculadora para obtener:
12 c
=
0,5878 0,9848
y tenemos c = 20,1

366 Trigonometra

366-367 366 10/11/2001, 16:41


CAPTULO 8

24. Resolvamos el tringulo ABC, dados


a = 18, b = 25 y c = 12.
Por teorema del coseno:
a2 = b2 + c2 2bc cos a

de donde:
b2 + c2 a2
cos a =
2bc
625 + 144 324 445
cos a = = = 0,7417
600 600
Aqu obtenemos a = 4210

(en este caso, al resolver la ecuacin resultante slo debemos conside-


rar la solucin entre 0 y 180)

Para determinar b podemos aplicar nuevamente el teorema del coseno:

b2 = a2 + c2 2ac cos b
a 2 + c2 b 2
cos b =
2ac
324 + 144 625 157
cos b = = = 0,3634
432 324
De donde obtenemos b = 11120
Para determinar g, calculamos el suplemento de a + b.
g = 180 (4210 + 11120)
g = 2630

Ejercicios
1. Demuestre la equivalencia entre ambas unidades de medicin de ngulos.
2. Exprese en radianes la medida de los siguientes ngulos:
a) 45 e) 225
b) 15 f) 210
c) 150 g) 60
d) 300 h) 135
3. Exprese en grados la medida de los siguientes ngulos:

a) 3 d) g) 1
4 16
b) 2 e) h) 1
9 3 3
c) 7 f) 5 i) 2
5 6

Trigonometra 367

366-367 367 10/11/2001, 16:41


Ejercicios

4. Determine las funciones trigonomtricas del ngulo a sabiendo que sen a = 1


3
5. Determine las funciones trigonomtricas del ngulo a sabiendo que sec a = 5
3
6. Determine las funciones trigonomtricas del ngulo b sabiendo que ctg b = 5

7. Determine las funciones trigonomtricas del ngulo g sabiendo que cosec g = b


1+b
8. Determine entre qu valores estn acotadas las funciones seno y coseno.

9. Determine entre qu valores estn acotadas las funciones tangente y cotangente.

10. Determine entre qu valores estn acotadas las funciones secante y cosecante.

11.Verifique las identidades trigonomtricas a partir de las funciones dadas.

a) (1 sen2 a) sec2 a = 1

b) sen a tg a = sec a - cos a

c) cos b cot b = cosec b - sen b

2
d) sec2 j = cosec2 j
tg j
e) sen g (cosec g - sen g) = cos2 g

f) 1 + sec2 b = cosec b
tg b + sen b
g) tg2 a - sen2 a = tg2 a sen2 a

h) cosec2 a tg2 a - 1 = tg2 a

i) sen2 a sec2 a = sec2 a -1

j) (1 + tg2 a) (1 sen2 a) = 1

k) 1 + cos m + sen m = 2 cosec m


sen m 1 + cos m

l) 1 = cosec t cot t
cosec t + cot t
m) sen2 a - cos2 a = sen4 a - cos4 a

n) 1 + cos a + sen a = cos a + 1


sen a cos a sen a cos a

o) 1 sen d = cos d
cos d 1 + sen d

368 Trigonometra

368-369 368 09/11/2001, 11:18


CAPTULO 8

12. Determine las


funciones trigonomtricas 2
del ngulo a de la figura
a

5
13. Determine las funciones trigonomtricas del 1
ngulo b de la figura.

b
14. Determine las funciones 2 3
trigonomtricas del C
ngulo g de la figura.

M ABC issceles con AB = 10,


AC = BC = 12.
Sugerencia : trace la altura hc.
g
A B

15. Si sen a = 0,3 determine las dems funciones trigonomtricas de a.

16. Si cot b= 1,2 determine las dems funciones trigonomtricas de b.

17. Si tg g = 1 determine el valor de: sen g cos g.


b

18. Si sec d = 1 + a determine el valor de: cos2 d 1.


1a

19. Verifique que: 2 sen cos tg = 1


4 4 4

20. Verifique que: sen 30 cos 60 + cos 30 sen 60 = tg 45.

21. Determine las funciones trigonom- y coincide con la recta de ecua-


tricas del ngulo b sabiendo que b cin x 3y = 0.
est en posicin estndar y que el
punto P(1,-2) pertenece al lado ter- 24. Determine las funciones trigonom-
minal de l. tricas del ngulo b sabiendo que el
lado terminal est en el 4 cuadrante
22. Determine las funciones trigonom- y es paralelo a la recta de ecuacin
tricas del ngulo b sabiendo que el 3x + 2y + 3 = 0
punto Q(2,-5) pertenece al lado ter-
minal de l. 25. Determine las funciones trigonom-
tricas del ngulo b sabiendo que
23. Determine las funciones trigonom- el lado terminal est en el 2 cua-
tricas del ngulo b sabiendo que el drante y es perpendicular a la recta
lado terminal est en el 3er cuadrante de ecuacin x y = 3

Trigonometra 369

368-369 369 09/11/2001, 11:19


Ejercicios
Nota: Para resolver ejercicios que inclu-
26. A partir del ejercicio resuelto 12, verifi- yan ngulos cuyas funciones trigono-
que el valor de las funciones trigonom- mtricas se desconozcan se debe hacer
uso de la calculadora.
tricas del ngulo de 30 o (
6 )
39. Desde lo alto de un edificio de 25m
27. Compruebe el valor de las funciones tri- de altura se obtiene una medicin de

gonomtricas del ngulo de 45 o (4
. ) 35 para el ngulo de depresin de un
Sugerencia: considere un tringulo rec- quiosco situado en el mismo plano del
edificio. A qu distancia se encuentra
tngulo issceles de lado (catetos) 1.
el quiosco del edificio?
28. Determine las funciones trigonomtricas del
40. Desde lo alto de un acantilado se obser-
(
ngulo de 180 (o ) y de 270 o 3 .
2
) van dos botes bajo ngulos de depresin
de 20 y 30, respectivamente. Deter-
29. Evale: sen 30 + cos 30
mine la altura del acantilado sabiendo
que la distancia entre los botes es 35m.
30. Evale: 1 tg2 45
41. Un avin despega en un ngulo de 10
1 + tg 60 m
31. Evale: y vuela con una velocidad de 75 .
1 tg 60 seg
Cunto tardar en alcanzar una altitud
32. Verifique la igualdad: de 15.000 metros?
ctg 30 1
= ctg 30 42. Una escalera de 8 metros se encuentra
1 tg 30
apoyada en una pared y forma con sta
2
33. Verifique 1 + sen 45 = sec2 45 + tg2 45 un ngulo de 40. Calcule la distancia
cos2 45 entre la pared y el pie de la escalera.

2 tg
34. Verifique tg = 6 Desafos.
3 1 tg2
6
43. Grafica todas las funciones trigonom-
tricas asignando algunos valores y apli-
35. Verifique cos 3 = cos2 6 sen2 6
cando las propiedades de periodicidad,
36. Un cohete es lanzado a nivel del suelo, paridad e imparidad y acotaciones.
en un ngulo constante de 60 hasta una
distancia de 3.000 metros. Determine a
qu altura se encuentra del suelo. 44. Resolver las siguientes ecuaciones
(Hallar todas las soluciones menores o
37. Sabiendo que el ngulo de elevacin iguales a 2):
del sol, a cierta hora del da es de 30,
1) tg x = 1
determine la longitud de la sombra que
proyecta una persona que mide 1,6 m. 2) sen x = 1
2
3) sen x + 1 = 0
38. Desde un punto P situado a 12 metros
de un edificio se observa un letrero 4) cos x 1 = 0
luminoso que est en una ventana del
5) tg2 x 1 = 0
edificio, bajo un ngulo de elevacin
de 30, y desde el mismo punto P se 6) sec y = 2
observa el techo del edificio bajo un 7) 2 cos t + 1 = 0
ngulo de elevacin de 60. Calcule la
altura del edificio. 8) 2 sen2 x sen x = 0

370 Trigonometra

370-371 370 09/11/2001, 11:26


CAPTULO 8

9) 2 cos x = 3 45. Resolver los tringulos ABC, dados:

10) sec2 t 2 = 0 1) a = 30 b = 35 b = 35

11) 2 sen x = csc x 2) a = 25 c = 94 a = 57

12) 4 sen x = 3 csc x 3) a = 64 a = 28 b = 34

13) 2 cos x = ctg x 4) a = 15 b = 55 c = 104

14) sec2 x = 3 tg2 x 1 5) b = 58 b = 58 g = 18


15) sen x = cos x 6) a = 100 b = 100 c = 40
16) tg x = 3 ctg x
7) a = 70 b = 15 g = 60
17) tg2 t + tg t = 0
8) b = 10 c = 20 a = 40
18) sen2 t + sen t cos t = 0
9) a = 14 b = 28 c = 50
19) 3 tg2 y=0 (analizar)

20) 2 cos2 x + cos x = 0 46.Una pequea embarcacin debe diri-


girse desde una isla a un puerto en el
21) sen2 x sen x 6 = 0 continente, que se encuentra a 240 km
(indicacin: analice soluciones) de la isla. Debido a la fuerte corriente,
despus de navegar un tiempo, la embar-
22) cos x + 2 sec x + 3 = 0
cacin se encuentra a 140 km de la isla
y a 35 direccin N.E. Determine a qu
23) sen2 x + 2 sen x + 1 = 0
distancia aproximada se encuentra del
24) 2 sen2 x 3 sen x + 1 = 0 puerto y qu direccin debe tomar para
corregir el curso.
25) 2 cos 2 x sen x 1 = 0
47. Para llegar a casa Juanito debe cruzar un
26) cos 3x = 1 ro de 30 m de ancho; l se encuentra
justo frente a su casa, pero en la orilla
27) 2 ctg2 x + csc2 x 2 = 0 opuesta. La corriente lo desva 28 ro
arriba. A qu distancia se encontrar de
28) 2 cos2 t + 3 cos t + 1 = 0 su casa cuando logre atravesar el ro?

29) 1 sen t = 3 cos t 48. Un avin vuela 250 km desde un punto


(indicacin: eleve al cuadrado y ana- A en direccin 70 y luego 120 km en
lice soluciones) direccin 220. A qu distancia aproxi-
mada se encontrar del punto A?
30) 4 sen2 u 1 = 0
49. Dos vehculos salen de una ciudad al
31) sen x + cos x = 1 mismo tiempo y circulan en carreteras
(indicacin: eleve al cuadrado y ana- rectas que forman entre s un ngulo
lice situaciones) de 70. Si viajan a 90 km/h y 110
km/h respectivamente, a qu distancia
32) 3 tg2 x sec2 x 5 = 0
se encontrarn despus de 40 minutos?

Trigonometra 371

370-371 371 09/11/2001, 11:26


Ejercicios
50. Un bote pesquero utiliza un equipo de ondas sonoras para detectar un banco de peces
que se encuentra a 3 km de la embarcacin y que se mueve a razn de 12 km/h en
direccin 35 N.E. Si el bote avanza a razn de 20 km/h, determine cunto tiempo
tardar en alcanzar el banco de peces.

32
51. Al observar el sol desde la tierra se ve bajo un ngulo de 32 (minutos, es decir, grados)
60

0,53
TIERRA

SOL

Si la distancia entre la tierra y el sol es aproximadamente 150.000.000 km, calcule el


dimetro del sol.

Soluciones

2. a) e) 5
4 4

b) f) 7
12 6

c) 5 g)
6 3

d) 5 h) 3
3 4
3. a) 135 d) 11,25 g) 57,32
b) 40 e) 60 h) 19,1
c) 252 f) 150 i) 114,6

1 2 2 2
4. sen a = cos a = tg a =
3 3 4
3 2
csc a = 3 sec a = ctg a = 2 2
4
4 3 3
5. sen a = cos a = tg a =
5 5 4
5 5 4
csc a = sec a = ctg a =
4 3 3

372 Trigonometra

372-373 372 09/11/2001, 11:29


CAPITULO 8
CAPTULO

5
6. sen a = 1 cos b = tg b = 1
6 6 5

6 6 5
csc b = sec b = ctg b =
5

7. No existe un ngulo agudo en tal tringulo (lo que no existe es ese tringulo
rectngulo)

8. 1 sen a 1
1 cos a 1

9. La tangente y la cotangente varan entre menos infinito e infinito positivo. No tienen


restricciones.

10. sec a 1 o sec a 1

csc a 1 o sec a 1

11. Las demostraciones debe hacerlas el estudiante. Ver ejercicios resueltos n 6 al 10.
21 2 21
12. sen a = cos a = tg a =
5 5 5
5 5 2
csc a = sec a = ctg a =
21 2 21

11 1
13. sen b = cos b = tg b = 11
2 3 2 3

2 3 1
csc b = sec b = 2 3 ctg b =
11 11

119 5 119
14. sen g = cos g = tg g =
12 12 5

3 91 3
15. sen a = cos a = tg a =
10 10 91

10 10 91
csc a = sec a = ctg a =
3 91 3

5 6 5
16. sen b = cos b = tg b =
61 61 6

61 61 6
csc b = sec b = ctg b =
5 6 5

Trigonometra 373

372-373 373 09/11/2001, 11:29


Soluciones
1b
17.
1+ b2
18.
4a
(1 + a)2
1 1
19. 2 =1
2 2
1 1 3 3
20. + =1
2 2 2 2
2 1
21. sen b = cos b = tg b = 2
5 5
5 1
csc b = sec b = 5 ctg b =
2 2
5 2 5
22. sen b = cos b = tg b =
29 29 2
29 29 2
csc b = sec b = ctg b=
5 2 5
1 3 1
23. sen b = cos b = tg b =
10 10 3
10
csc b = 10 sec b = ctg b = 3
3
3 2 3
24. sen b = cos b = tg b =
13 13 2
13 13 2
csc b = sec b = ctg b =
3 2 3
1 1
25. sen b = cos b = tg b = 1
2 2
csc b = 2 sec b = 2 ctg b = 1
26. Ver ejercicio resuelto n 12 y tabla de la pgina 343.
27. dem.
28. dem.

29. 1+ 3
2
30. 0

31. (2 + 3 )
31 31
32. = = 3 = ctg 30
1 1 31
3 3
1
1+
33. Primer miembro: 2 =3 y Segundo miembro: 2 + 1 = 3
1
2 3 2
3 1 = 3 1
34. 3 = 35. 2 4 4
2
3
1 = 1
3 = 3 2 2

374 Trigonometra

374-375 374 20/11/02, 3:58 PM


CAPITULO 8
CAPTULO

36. 2.598 m es la altura a la que se encuentra el cohete del suelo.


37. La persona proyecta una sombra de 92,3 cm.
38. El edificio mide 20,78 m. (hay informacin de ms en el enunciado).
39. 17,5 m
40. 34,47 m
41. 19 min 11,8 seg.

42. 5,14 m
3 3 5 7
44. 1) 45, 225 2) 210, 330 3) 4) 2 5) , , ,
2 4 4 4 4
5 2 4 3 11
6) , 7) , 8) , , , 9) 150, 210
3 3 3 3 2 2 6 6
3 5 7 2 4 5
10) , , , 11) 45, 135, 225, 315 12) , , ,
4 4 4 4 3 3 3 3
3 3
13) , , , 14) 35,26 ; 144,74 ; 215,26 ; 324,74
2 2 4 4
5 3 7
15) , 16) 60, 120, 240, 300 17) 0, , ,
4 4 4 4
3 7 2 4 5 2 4 3
18) 0, , , 19) , , , 20) , , ,
4 4 3 3 3 3 2 3 3 2
3 3 11
21) No tiene 22) 23) 24) , , ,
2 6 2 2 6
3 3 5 7
25) 26) 0, 120, 240, 360 27) , , ,
2 4 4 4 4
2 4
28) , , 29) 90, 330 30) 30, 150, 210, 330
3 3
31) 0, 90 32) 50,77 ; 129,23 ; 230,77 ; 309,23

45. 1) a = 29,45 g = 115,55 c = 55,05

2) no existe un tringulo con esas medidas 3) g = 118 b = 76,23 c = 120

4) g = 110 a = 28,64 b = 90,66 5) a = 104 a = 66,36 c = 21,13

6) a = b = 78,465 ; g = 23,07 7) a = 71,74 b = 48,26 c = 63,84

8) No existe un tringulo con esas medidas

9) No existe un tringulo con esas medidas (28 + 14 < 50)

46. 149 km 12,5 SE 47. 34 m

48. 158 km 49. 13,35 km


1
50. Si el barco avanza en direccin 33,6 NE alcanza al banco de peces en de hora.
4
1
Estudie usted otras posibilidades. Se podr demorar ms de hora? Cul ser el ngulo
2
necesario para alcanzar a los peces en el mnimo de tiempo?

51. 1.387.546,5 km.

Trigonometra 375

374-375 375 20/11/02, 3:59 PM


Prueba de seleccin mltiple

1. Exprese en radianes 5. Si sec a = 3 , 10. Si sen a = cos a


270 cul es el valor de cos2 a? entonces el valor de a es:
2 A. 0 y 180
A.
2 A. 3 B. 45 y 225
1 C. 135 y 315
3 B.
B. 3 D. todo a E R
2
1 E. ninguno
3 C. 1
C. 9 11. Si cos x = entonces
4 2
D. 9
4 el valor de x es:
D. E. otro
3
A.
3
3
E. 6. Si tg a = 0,7
2 cunto vale ctg a? B.
6
2. Exprese en grados A. 0,3

C.
7 4
6
radianes B. 0,7 3
D.
4
A. 210 C. 7
10 E. otro
D. 10
B. 21 7 12. Si ctg x = 1, entonces
E. otro cul es el valor de x?
C. 280
A.
7. Si 1 tg b = 2, 4
D. 420
cunto vale b? B.
2
E. 350 A. 90 y 270 3
C.
2 4
3. Si sen a = B. 0 y 180 5
5 D.
C. 45 y 225 4
entonces tg a =
5
D. 135 y 315 E.
2 3
A.
21 E. 60 y 30 13. Cul de las afirmaciones
2 siguientes es verdadera?
B. 8. Si cosec a = 1,
21 1
cunto vale a? A. sen a =
21 cos a
C. A. 0 B. tg a - cos a = sec a
2
5 B. 90 C. 1 + sec2 a = tg2 a
D.
21 C. 180 D. sen2 90 + cos2 90 = tg2 45
21 D. 270
E. E. sec a - cosec a = 1
5 E. 45
4. sen2 60 + cos2 30 = 14. Cul de las afirmaciones
3 9. Si tg2 a = 2 1, siguientes es falsa?
A. cunto vale sec2 a?
2 A. sen2a + cos2a = sec2a - tg2a
3 A. 1 + 2
B. B. cos b sec b = 1
4 B. 1 2
C. 1 C. tg 30 = ctg 60
C. 2
D. 3 1
D. 2 D. sen2 45 =
2
E. 1+ 3 E. otro E. sec 45 + cosec 45 = 2
4
376 Trigonometra

376-377 376 20/11/02, 4:00 PM


CAPTULO 8

15. A partir de la figura, 19. La expresin 2 cos2 a + sen2 a a b


qu relacin es falsa? es equivalente a: A. =
sen 45 sen 35

A. 2 sen2 a sen 100 sen 45


B. =
2 B. 2 + sen2 a c b
C. 2 sen 100 sen 45
C. =
D. 2 cos2 a c a
b E. 2 + cos2 a sen 35 c
D. =
sen 100 a
a 2 2 20. La expresin sen2 a sec2 a
es equivalente a: a sen 100
E. =
sen 35 c
A. tg a
A. a = b
B. tg2 a 24. En M ABC de la figura,
B. cos a = 2 cosec b =
2 C. ctg a
C. tg b = 1
D. ctg2 a
D. cosec b = 1
2 e) 1 b
E. tg a = cot a a
21. Si sen a = , entonces
a+1
cos a = r
16. Al expresar en radianes, p
a+1
un ngulo de 15 es A.
a
equivalente a:
2a + 1
B.
a+1
A. a
8 a+1
C. q
2a + 1
B.
10 q
C.
D. 2a+ 1 A.
a+1 r
12
2a+ 1 p
D. E. B.
15 a r

E. 22. Si ctg b = 2, entonces sen b = r
30 C.
p
17. Al expresar en grados un 1
5 A. r
ngulo de es equiva- 5 D.
12 q
lente a: 2
B.
A. 300 5 p
E.
1 q
B. 150 C.
3
C. 75 2 25. El valor de
D.
D. 30 3
1 sen + sen 3 es:
E. 15 E. 2 2
2
A. 1
18. La expresin cosec2 a - 1 23. A partir de la figura, qu
es equivalente a: relacin es correcta? B. 2
A. 1 + ctg2
a b
a C. 0
B. ctg2 a - 1
35
C. 1 tg2 a D. 1
D. ctg2 a
c 45 E. no est definido
E. tg2 a

Trigonometra 377

376-377 377 20/11/02, 4:00 PM


Prueba de seleccin mltiple

26. Cul es el valor de 28. Si sec b = 1,5 cul es el 30. Cul(es) de la(s) siguiente(s)
sen 30 cos 30 ctg 30 valor de sen b? relaciones es o son verdade-
ras?
1 3
A. A.
2 I) sen = cos
13 3 6
1 2
B.
4 B.
13 II) tg ctg 3 = csc
4 4 6
C. 2 5
C. III) sen 2 cos2 =1
3 4 4
D. 4
D. 5 A. Slo I
3 2
E. B. Slo II
4 2
e) C. Slo III
3
27. Cul es el valor de
tg 60 ctg 60 sen2 60? 29. Cul es el valor de D. Slo I y II
E. I, II y III
3 tg + tg ?
A. 4 2
2
A. 1
9
B.
2 1
B.
9 2
C.
4 C. 0
3 1
D. D.
4 4
3 E. no est determinado
E.
2

Soluciones

1. B 11. B 21. D
2. A 12. C 22. A
3. B 13. D 23. B
4. A 14. E 24. D
5. B 15. D 25. C
6. D 16. C 26. E
7. D 17. C 27. D
8. B 18. D 28. C
9. D 19. A 29. E
10. B 20. E 30. A

378 Trigonometra

378 378 09/11/2001, 11:44


CAPTULO 9
N meros
complejos

Deniciones y
propiedades 9.1

Definicin: Un nmero complejo es un par ordenado de nmeros


reales.
El conjunto de los nmeros complejos lo simbolizamos por k y
cada nmero complejo por la letra z.
k = {z = (a, b) / a E R, b E R}.
Si z = (a, b) E k, a se llama parte real del complejo z y se
denota por Re(z), y b se llama parte imaginaria del complejo
z y se denota por Im(z).

9.1.1 Igualdad
Dados dos complejos z1 = (a, b) y z2 = (c, d).

z1 = z2 si y slo si { ab == cd
9.1.2 Representacin geomtrica
Geomtricamente el conjunto de los nmeros complejos representa el plano cartesiano.
Existe una relacin biunvoca entre los elementos del conjunto k y el conjunto
de puntos del plano.
Sea z = (a, b) E k
Si b = 0, entonces el complejo z es de la forma (a, 0) y se asimila al nmero real a.
Geomtricamente se representa en el eje horizontal.
Si a = 0, entonces el complejo z es de la forma (0, b) y decimos que es un imaginario
puro. Se representa geomtricamente en el eje vertical.

Nmeros complejos 379

379-381 379 10/11/2001, 16:44


9.1.3 Forma cannica de un complejo
El complejo z = (a, b) se puede escribir en su forma cannica como z = a + bi, donde i es
la unidad imaginaria. i = se cumple i2 = 1.

9.1.4 Operaciones con nmeros complejos


SUMA.
Sean
z1 = (a, b) z2 = (c, d)
z1 + z2 = (a, b) + (c, d) = (a + c, b + d)
Sean
z1 = a + bi z2 = c + di
z1 + z2 = (a + bi) + (c + di) = a + c + ( b + d) i

PRODUCTO.
Sean
z1 = (a, b) z2 = (c, d)
z1 z2 = (a, b) (c, d) = (ac bd, ad + bc)
Sean
z1 = a + bi z2 = c + di
z1 z2 = (a + bi) (c + di) = ac bd + (ad + bc)i

Observacin: Para efectuar las operaciones con nmeros complejos


escritos en su forma cannica se procede como en el conjunto de
los nmeros reales considerando que i2 = 1.

9.1.5 Estructura del conjunto (k, +, )


El conjunto de los nmeros complejos con las operaciones de
suma y producto tiene estructura de Campo.

PROPIEDADES DE LA SUMA.
Sean z1, z2, z3, E k.
1. La suma es cerrada en k
Iz1, z2 E k, z1 + z2 E k.
2. La suma es asociativa en k
Iz1, z2 y z3 E k, (z1 + z2) + z3 = z1 + (z2 + z3).
3. La suma es conmutativa en k
Iz1, z2 E k, z1 + z2 = z2 + z1.
4. Existe un elemento neutro para la suma en k
Iz E k, H! w E k tal que z + w = z.

380 Nmeros complejos

379-381 380 10/11/2001, 16:44


CAPTULO 9

5. Existe un elemento inverso para la suma en k


I z E k, H z` E k tal que z + z` = w (w neutro aditivo).
Nota: El neutro aditivo es el complejo (0, 0) o 0 + 0i = 0
El inverso aditivo de (a, b) es ( a, b). Escrito en forma
cannica, el inverso aditivo de a + bi es a bi.
El inverso aditivo de z se denota por z.
PROPIEDADES DEL PRODUCTO.
Sean z1, z2, z3 E k.
1. El producto es cerrado en k
Iz1, z2 E k, z1 z2 E k.
2. El producto es asociativo en k
Iz1, z2 y z3 E k, (z1 z2) z3 = z1 (z2 z3).
3. El producto es conmutativo en k
Iz1, z2 E k z1 z2 = z2 z1
4. Existe un elemento neutro para el producto en k
Iz E k, H! w E k tal que z w = z.
5. Existe un elemento inverso para el producto en k
Iz E k, H z E k tal que z z = w (neutro multiplicativo).
Nota: El neutro multiplicativo es el complejo (1, 0) o 1 + 0i = 1.
a , 2 b 2
El inverso multiplicativo de (a, b) es ( 2
a +b 2 a +b )
Escrito en forma cannica, el inverso multiplicativo de
a b
a + bi es 2 2 i
a + b2 a + b2

El inverso multiplicativo de z se denota por z1.


El elemento (0, 0) no tiene inverso multiplicativo.
DISTRIBUTIVIDAD DEL PRODUCTO SOBRE LA SUMA
Sean z1, z2, z3 E k.
Iz1, z2 y z3 E k: z1 (z2 + z3) = z1 z2 + z1 z3
Observacin: Sean z y w dos complejos, definimos:
Resta: z w = z + ( w)
Divisin: z = z w1
w

9.1.6 Potencia de i
Las potencias de i son cclicas y cada cuatro vuelven a repetir
su valor.
i =i
i2 = 1
i3 = i2 i = 1 i = i
i4 = i2 i2 = 1 1 = 1
i5 = i4 i = i (i5 = i)

Nmeros complejos 381

379-381 381 10/11/2001, 16:44


i6 = i5 i = i i = i2 =1 (i6 = i2)
i7= i6 i = 1 i = i (i7 = i3)
i8= i7 i = i i = i2 = 1 (i8 = i4)

Ejercicios 1. Dados los complejos z1 = 2 + 3i, z2 = 5 + i


z3 = 2i y z4 = 5.
resueltos
Encontrar el valor de:
a) z1 + z2 z3 b) z2 (z3 + z4)
1 1 z1 + z2
c) + d)
z2 z3 2(z3 z4)
Solucin:
a) z1 + z2 z3 = (2 + 3i) + ( 5 + i) (2i)
= 2 + 3i 5 + i 2i = 3 + 2i
b) z2 (z3 + z4) = ( 5 + i) [(2i) + ( 5)] = ( 5 +i) ( 5 + 2i)
= 25 10i 5i + 2i2 (i 2 = 1)
= 25 10i 5i 2
= 23 15i
c) 1 + 1 = 1 + 1
z2 z3 5 + i 2i
Para dividir por a + bi se amplifica la fraccin por a bi.

5 i 2i
= +
(5 + i) (5 i) (2i) (2i)
5 i 2i
= +
25 i2 4i2
5 i 2i 10 2i 26i
= + +
26 4 52
10 28 7
= i= 5 i
52 52 26 23
z1 + z2
d) = (2 + 3i) + (5 + i) = 3 + 4i =
2(z3 z4) 2 ((2i) (5)) 10 + 4i
(3 + 4i) (10 4i) 30 + 12i + 40i 16i2
= =
(10 + 4i) (10 4i) 100 16i2
14 + 52i 7 13 i
= = =
116 58 29
1 3
2. Dados los complejos z1 = 5 + i, z2 = 3 i,
2 4
z3 = i y z4 = 1, encontrar:

a) Re (z1 z3) b) Im
( z1
2
+ z3
)
382 Nmeros complejos

382-383 ok 382 09/11/2001, 11:59


CAPTULO 9

c) Re ( zz )
4
1
d) 2 Im ( zz )
3
4
Solucin:
(
a) Re (z1 z3) = Re (5 + 1 i) ( i) = Re 5 + 3 i = 5
2 2 ) ( )
(
b) Im 1 + z3 = Im
z2 ) 1
3 3 i
4
[+ (i)

3
]
[( ]
3 + i
= Im 4 i
3 3
3 i 3 + i
4 4 )( )
3 + 3 i 3
= Im
[ 9
4
9 2
16
i
i = Im
3
153
16] [
+ 4 i i
153
16 ]
51(
= Im 16 + 4 ii
51 )
51(
= Im 16 47 i
51
= 47
51 )
5 1 i
z
( )
c) Re z4 = Re
1
[ 5+ 1 i
2
1
] = Re
[( 2

5+ 1 i 5 1 i
2 2 )( ) ]
5 1 i 1
= Re 2

[
1 2
25 i
2
= Re

20
5
101
4
2 i
101
4 ] [ ]
(
= Re 20 + 2 i =
101 101 101 )
z
( )
d) 2 Im z3 = 2 Im i = 2 Im ( i) = 2 ( 1) = 2
4 1 ( )
3. Encontrar x e y para que se cumpla la siguiente igualdad:
2x 3 + xi = x + y 2yi i 1
Solucin:
2x 3 + xi = x + y 1 (2y + 1) i

2x 3 = x + y 1 xy =2
x = 2y 1 x + 2y = 1 x=1 e y=1

4. Sabiendo que el neutro multiplicativo en k es z = 1 + 0i = 1,


demostrar que el inverso multiplicativo de a + bi es:
a b
2 i
a2 + b2 a + b2
Solucin:
Sea z = (x + yi) el inverso pedido. Se debe cumplir que:

Nmeros complejos 383

382-383 ok 383 09/11/2001, 11:59


z (a + bi) = 1
(x + yi) (a + bi) = 1
ax by + (ay + bx) i = 1 + 0 i

ax by = 1
ay + bx = 0

resolviendo el sistema obtenemos:


a b
x= 2 e y= 2 , luego
a + b2 a + b2
El inverso multiplicativo de (a + bi) es:
a b
(
a2 + b2
2
a + b2
i
)
5. Encontrar el valor de:
i5 i7 i18 i133
a = 11 9 b=
i + i i24 + i111
Solucin:
a) i5 = i 4 i = 1 i = i
i7 = i4 i3 = 1 i2 i = i
i11 = i8 i3 = (i4)2 i3 = 12 ( i) = i
i9 = i8 i = i
i5 i7 i (i) 2i
= = =1
i11 + i9 i+ i 2i
b) i18 = (i4)4 i2 = 1
i135 = (i4)33 i3 = i
i24 = (i4)6 = 1
i111 = (i4)27 i3 = i
i18 i135 1 + i
= = (1 + i) (1 + i)
i24 + i111 1 i (1 i) (1 + i)
1 i + i + i 2 2
= = = 1
1 i2 2

6. Encontrar el valor de a) b) (1 + i)16


Solucin:

2 3 + 2 i 2 3 + 2 i 3 2 + 4 3 i
a) =
3 2 4 3 i 3 2 4 3 i 3 2 + 4 3 i

6 6 + 24 i + 6 i + 4 6 i2
=
18 48 i2

2 6 + 30i 6 5
= = + i
66 33 11

384 Nmeros complejos

384-385 384 09/11/2001, 12:02


CAPTULO 9

b) (1 + i)16 = [(1 + i)2]8 = [1 + 2 i + i2]8 = (2i)8 = 256.


7. Sea 2x + (3y 6) i + 3 un nmero complejo.
Determinar los valores de x e y para que la expresin dada sea:
a) un nmero imaginario puro
b) un nmero real
c) cero
d) igual a 2 5i

Solucin:
2x + (3y 6) i + 3 = (2x + 3) + (3y 6) i
a) Para que sea imaginario puro, su parte real debe ser cero:
3
2x + 3 = 0 x =
2
b) Para que sea un nmero real, su parte imaginaria debe
ser cero:
3y 6 = 0 y = 2
c) Para que sea cero, la parte real y la parte imaginaria
deben ser ambas cero:
x= 3 A y=2
2
d) Para que sea igual a 2 5i
1
2x + 3 = 2 x =
2
1
3y 6 = 5 y =
3

Ejercicios
1. Encuentre las races de los siguientes 4. Efecte las siguientes operaciones:
nmeros complejos: a) (2 + 3i) + (5 6i)
a) 25 b) 81 b) (3 i) + (2 4i)

36 144 c) (5 + 4i) + ( 1 i)
c) d)
d) (6 + i) i
2. Efecte las siguientes operaciones:
e) (8 4i) (2 + i)
a) 25 + 4 2 16 =
f) (3 i) (5 + 4i)
b) 3 49 2 25 + 169 = g) 2 (6 2i)
3. Escriba los inversos aditivos de los h) (1 i) (1 + i)
siguientes nmeros complejos: 5. Efecte los siguientes productos:
a) 2 3i b) 1 i c) 6 2i d) i a) (2 3i) (4 i)

Nmeros complejos 385

384-385 385 09/11/2001, 12:02


Ejercicios 10. Calcule el cuadrado de los siguientes
nmeros complejos:
b) (5 + 2i) ( 1 6i)
a) 3 + 2i b) 5 3i c) 1 + i
c) (3 5i) (4 + i)
d) 2 + i e) 1 i f) 2 2i
d) ( 3 2i) ( 1 + 6i) 11. Calcule:
e) ( 2 + i) ( 3 i) a) (1 i2)6 b) (i 22 + i30)4
f) (1 + 2i) (3 i) c) (i5 + i 12)2 d) (i 3 i 5) 2
g) (4 2i) (5 + i) 12. Verifique que los complejos 3 i y
3 + i son solucin de la ecuacin
h) (3 + 2i) (7 i) x2 6x + 10 = 0
6. Efecte los siguientes productos: 13. Calcule:
a) (3 2i) (3 + 2i)
a)
b) (1 5i) (1 + 5i)
c) ( 6 + i) ( 6 i) b)
d) (4 3 i) (4 + 3i)
14. Si z1 = ( 2, 3), z2 = ( 1, 2), z3 = ( 5,
e) ( 1 i) ( 1 + i) 0) y z4 = ( 0, 4), encuentre:
f) ( 5 3i) ( 5 + 3i) a) z1 + z2 z3
g) i ( i) b) 2 z1 3 z2
h) 2i (2i) c) z4 (z1 + z2)
7. Calcule las siguientes divisiones: d) (z1 z2) (z3 + z4)

a) (2 + 5i) : (3 2i) e)
b) (1 4i) : (6 2i)
f) (z1 + z4)
c) (3 2i) : (1 + i)
g)
d) (1 i) : (2 4i)
e) (4 + 2i) : (5 i) h) (z2 z3 + z1 )
f) (2 + i) : (2 i) 15. Si z1 = 3 5i, z2 = 6 + i, z3 = 4 9i
g) (1 i) : ( i) y z4 = 5i, encuentre:
a) z1 z2 + z4
h) (6 + 2i) : i
b) 2 z1 + 5 z3
8. Calcule los inversos multiplicativos de
los siguientes nmeros complejos: c) 4 z3 (z1 z2)
d) 2 z1 z2 3 z3 z4
a) 1 2i b) 1 + 2i c) 4 i
e) (1 z1) (1 + z2)
d) 3 + i e) i f) 2i
f) 2 z1 ( z1 z2 z3)
9. Calcule las siguientes potencias de i:
a) i 1 b) i2 c) i 16 d) i 125 g)

e) i 1.003 f) i 2 g) i 3 h) i 4 h)
i) i 5 j) i 6

386 Nmeros complejos

386-387 386 20/11/02, 5:49 PM


CAPTULO 9

16. Efecte las siguientes operaciones: cunto gira cada uno.


a) (2 2 3i)(3 2 i) 26. Pruebe que
(a + bi) i = b + ai I a, b E R
b)
27. Calcule los siguientes productos:
17. Considerando que al complejo a) (3 2i) i2 b) (2 + i) i2
x + yi corresponde el par ordenado
(x, y) grafique los siguientes nmeros c) ( 4 3i) i2 d) ( 1 + 3i) i2
complejos en el plano cartesiano: 28. Grafique el primer factor y el producto
en los ejercicios del problema anterior.
a) z1 = 2 + 3i b) z2 = 5 2i Una los puntos con el origen de
c) z3 = 8 + i d) z4 = 2 3i coordenadas y observe cunto gira
cada uno.
e) z5 = 5 + 2i f) z6 = 8 i
29. Encuentre a para que el producto
Compare z1 con z4, z2 con z5 y z3 (2a 3i) (5 + i) sea un imaginario
con z6 puro.
18. Determine los nmeros reales x e y 30. Determine un nmero complejo cuyo
que satisfagan la siguiente igualdad: cuadrado sea 8 6i.
31. Determine los nmeros reales x e y que
a) 2x 3i + y = xi 2i + 2yi + 1
satisfagan la siguiente condicin:
b) (2x i) + (y i) = (2 3i) (x + 2yi) (2 + xi) : (1 2i) = y + i
c) (x + i) (y 3i) = 1 7i 32. Calcule el valor de:
d) (2x i) (y + 2i) = 10 + 11i z2 2z + 1 si z = 2 3i
19. Encuentre un nmero complejo cuyo 33. Calcule el valor de:
cuadrado sea 3 4i z2 5z + 4 si z = 1 + i
34. Calcule el valor de:
20. Determine x para que el cociente 2 z2 z 3 si z = 1 3i
35. Resuelva las siguientes ecuaciones:
sea imaginario puro.
a) (1 z) (1 + i) = 2 i

21. Encuentre x para que sea un b) z (1 2i) + 3 = 1 2z + i


nmero real.
c)
22. Determine x para que el producto
de (1 2i) (x 5i) sea un nmero d)
real.
23. Determine x e y tales que: e) 3 + 4i
(x + yi)2 = 16 30i
36. Si z = 4 3i, encuentre la parte
24. Calcule los productos siguientes:
real de
a) (2 3i) i b) (4 + 2i) i
c) (5 3i) i d) ( 2 + i) i 37. Calcule la raz cuadrada de :
a) 3 + 4i b) 21 + 20i c) 15 + 8i
e) ( 3 2i) i
d) 5 + 12i e) 8i f) 2i
25. Grafique el primer factor y el producto
del ejercicio anterior. Una el origen Sugerencia: plantee un sistema de
con el nmero complejo y observe ecuaciones.

Nmeros complejos 387

386-387 387 20/11/02, 5:49 PM


Ejercicios 42. Calcule el valor de:
38. Determine z en la ecuacin: 1 1 1 1
+ + + ..... +
2 3 100
z 1z 5 i i i i
=
3 + 4i 5i 3 4i
43. Demuestre que
39. Resuelva el sistema: z w
Re + Re =1
2wi + (1 i) z= 3 z+w w + z
(1 i) w + 4z = 2 + i
44. Demuestre que
40. Encuentre z E k tal que z + 1 = 0 Re (zw) = Re (z) Re (w) Im (z) Im (w)
z
41. Calcule el valor de: 45. Demuestre que
1 1 1 1
+ + + ..... +
i2 i3 i50 Im (zw) = Re (z) Im (w) + Im (z) Re (w)
i

Soluciones

1. a) 5i b) 9i c) 6i d) 12i
2. a) i b) 24i
3. a) 2 + 3i b) 1+i c) 6 + 2i d) i
4. a) 7 3i b) 5 5i c) 4 + 3i d) 6
e) 6 5i f) 2 5i g) 8 + 2i h) 2i
5. a) 5 14i b) 7 32i c) 17 17i d) 15 16i
e) 7i f) 5 + 5i g) 22 6i h) 23 + 11i
6. a) 13 b) 26 c) 37 d) 25 e) 2 f) 34 g) 1 h) 4
4 19 7 11 1 5 3 1
7. a) 13
+
13
i b) 20 20 i c) 2 2 i d) 10 + 10 i
9 7 3 4
e) 13 + 13 i f) 5 + 5 i g) 1 + i h) 2 6i
1 2 1 2 4 1 3 1 1
8. a) 5 + 5 i b) 5 5 i c) 17 + 17 i d) 10 10 i e) i f) 2 i

9. a) i b) 1 c) 1 d) i e) i f) 1 g) i h) 1 i) i j) 1
10. a) 5 + 12i b) 16 30i c) 2i d) 3 4i e) 2i f) 8i

1
11. a) 64 b) 16 c) 2i d) 4
13. a) 0 b) 0
14. a) (8, 1) b) (1, 12) c) ( 4, 12) d) ( 25, 21)

23 11 2 7 3 41 1
e) , f) , g) , h) , 2
65 65 5 5 65 65 4

15. a) 3 i b) 19 40i c) 264 + 12i d) 89 114i

213 84 99 67
e) 19 + 33i f) 270 + 570i g) 1.258 + 629 i h) 485 97 i

388 Nmeros complejos

388-389 388 09/11/2001, 12:07


CAPITULO 9

16. a) 12 + 3 2 2 + 3 6 e)

10 3 6 -3 2
b) + i
11 11 2
17. 3 2i
y 3 2 3i
3 z1
z5
z3
2 En todos los casos se observa que el
1
x
giro es de 90.
8 5
1
2 En general (a + bi) i = b + ai
z6
2
z2 27. a) 3 + 2i b) 2 i c) 4 + 3i d) 1 3i
3 z4

28.
5 1
18. a) x = 1, y = 1 b) x = , y = 3 + 2i
6 2
2
c) x = 2, y = 1 d) x = 2, y = 3
3 2+i
1
1 3 3
x= , y=6 x= , y=8 2
2
1
2
3 4 3 2i 2 i
19. 1 + 2i; 1 2i
4 + 3i
1 1 + 3i
20. x = 3
2
21. No existe, porque no cumple la condi-
4
cin pedida. 1

3
5 4 3i 1 3i
22. x =
2
23. x = 3 y = 5 Se observa un giro de 180.
24. a) 3 + 2i b) 2 + 4i c) 3 + 5i
d) 1 2i e) 2 3i 3
29. a = 10
25. a) b) 4

2
3+2i 30. 3 i, 3 + i
4+2i
2
31. x = 1; y = 0
2 3
-2 4 32. 8 6i
3
23i
33. 1 3i

34. 18 + 15i

c) 3+5i d) 1 3 8 1
5
2 + i 35. a) z = 2 + 2 i b) z = 13 13 i
1

2 1 2
c) z = 3 + i d) z = 1
2
1 2i
9 13
3 5
e) 5 + 5 i

3 7
53i 36.
625

Nmeros complejos 389

388-389 389 09/11/2001, 12:08


37) a) (2 + i) b) (5 + 2i) 1 9 7 1
39) w = 10 10 i; z =
10
+
2
i
c) (1 + 4i) d) (3 + 2i)
40) z = i
e) (2 + 2i) f) (1 + i)
38) z = 1 + 2i 41) 1 i 42) 0

Conjugado y mdulo
de un complejo 9.2

9.2.1 Conjugado de un complejo


Sea z = a + bi un nmero complejo.

Definicin:
Se llama conjugado del complejo z al complejo
z = a bi

PROPIEDADES.
Sean z y w dos nmeros complejos, entonces, se cumplen las
siguientes propiedades:
1. z = z El conjugado del conjugado de z es z.
El conjugado de una suma es igual
2. z + w = z + w a la suma de los conjugados de los
sumandos.

El conjugado de un producto es igual


3. z w = z w al producto de los conjugados de los
factores.

4. z z
, w (0, 0) El conjugado de un cociente es igual al
=
w w cociente de los conjugados.
La suma de un complejo con su conju-
5. z + z = 2 Re (z) gado es igual a dos veces la parte real
del complejo.
La diferencia de un complejo con su
6. z z = 2 Im (z) i conjugado es igual a dos veces la parte
imaginaria del complejo.
Un complejo es real si y slo si es igual
7. z E R z = z
a su conjugado.
Para las demostraciones, ver ejercicios resueltos.

390 Nmeros complejos

390-391 390 10/11/2001, 16:51


CAPTULO 9

9.2.2 Mdulo de un complejo


Sea z = a + bi un nmero complejo.

Definicin:
Se llama mdulo o valor absoluto de z al nmero real
| z | definido por | z | = a2 + b 2

PROPIEDADES.
Sean z y w dos nmeros complejos, entonces, se cumplen las
siguientes propiedades:
El valor absoluto de la parte real de
1. | Re (z) | | z | un complejo es menor o igual al valor
absoluto del complejo.
El valor absoluto de la parte imaginaria
2. Im z z de un complejo es menor o igual al valor
absoluto del complejo.
Un complejo es cero si y slo si su valor
3. z = 0 z = 0
absoluto es cero.
El valor absoluto de un complejo es igual
4. z = z = z al valor absoluto de su inverso aditivo y
de su conjugado.
El valor absoluto de un producto de
5. z w = z w complejos es igual al producto de los
valores absolutos de los factores.

z z El valor absoluto de un cociente de


6. = nmeros complejos es igual al cociente de
w w
los valores absolutos de los nmeros.
El valor absoluto de una suma de nmeros
7. z + w z + w complejos es menor o igual a la suma
de los valores absolutos de los nmeros
complejos.
La propiedad nmero 7 recibe el nombre de Desigualdad
Triangular.
Para las demostraciones, ver los ejercicios resueltos.

1. Dados los nmeros complejos Ejercicios


z1 = 3 5i, z2 = 6 + 3i, z3 = 2i, z4 = 5. Encontrar: resueltos
a) el conjugado de cada uno b) el valor absoluto de cada uno

c) z1 + z 2 2 z 3 d) z1 z3

Nmeros complejos 391

390-391 391 10/11/2001, 16:52


Solucin:
a) Si z1 = 3 5i, entonces z1 = 3 + 5i
Si z2 = 6 + 3i, entonces z2 = 6 3i
Si z3 = 2i, entonces z3 = 2i
Si z4 = 5, entonces z4 = 5
b) Si z1 = 3 5i, entonces z1 = 32 + 5 2
= 34

Si z2 = 6 + 3i, entonces z2 = 6 2 + 32 = 45

2
Si z3 = 2i, entonces z3 = 2 = 4 =2

Si z4 = 5, entonces z4 = 52 = 25 = 5
c) z1 + z2 2 z3 = 3 5i + 6 + 3i 2 2i

= 3 + 2i = 3 2i
d) z1 z3 = 3 5i 2 i = 10 6 i = 100 + 36 = 136

2. Demostrar que z = z. (Propiedad 1)


Solucin: Sea z = a + bi E k
z = a + bi = a bi = a + bi = z

3. Demostrar que z + w = z + w (Propiedad 2)


Solucin: Sean z = a + bi, w = c + di E k

z + w = a + bi + c + di = a + c + b + d i

= (a + c) (b + d)i = (a bi) + (c di) = z + w

4. Demostrar que z w = z w (Propiedad 3)


Solucin: Sean z = a + bi, w = c + di E k

z w = a + bi c + di = ac bd + ad + bc i
= (ac bd) (ad + bc)i = ac adi + bdi2 bci
= a (c di) bi (c di) = (a bi) (c di) = z w

z
5. Demostrar que = z (propiedad 4)
w w
Solucin: Sean z = a + bi, w = c + di E k

z a + bi bc ad
w
= = ac + bd2 + 2
i
c + di c2 + d c2 + d
ac + bd bc ad
= i
2
c2 +d c2 + d2
ac bdi2 bci + adi
=
c2 + d2

392 Nmeros complejos

392-393 392 10/11/2001, 16:57


CAPTULO 9

c a bi + di a bi
=
c + di c di

a bi c + di a bi z
= = =
c + di c di c di w

6. Demostrar a) z + z = 2 Re (z) (Propiedades 5 y 6)


b) z z = 2 Im (z)i
Solucin: Sea z = a + bi
a) z + z = (a + bi) + (a bi) = 2a = 2 Re (z)
b) z z = (a + bi) (a bi) = 2bi = 2 Im (z) i

7. Demostrar z E R z = z (Propiedad 7)
Solucin: Sea z = a + bi E k
) Hip: zER
Tesis: z=z
z E R a + bi E R b = 0 z = a z = a

\z=z
) Hip: z=z
Tesis: zER
z = z a + bi = a bi a = a
y b = b 2b = 0
b=0
\z=a luego z E R

8. Determinar un nmero complejo tal que su cuadrado sea


igual a su conjugado.
Solucin: Sea x + yi el nmero complejo pedido.
(x + yi)2 = x yi
x2 y2 + 2xyi = x yi

x2 y2 = x
2xy = y } cuya solucin es x = 1
2
y= 3
2

luego los nmeros buscados son

1 3 1 3
+ i y i Comprobarlo.
2 2 2 2

9. Probar que a) | Re (z) | |z|


b) | Im (z) | |z| (Propiedades 1 y 2)
Solucin: Sea z = a + bi

Nmeros complejos 393

392-393 393 10/11/2001, 16:58


a) |Re (z)| = |a| = a2 a2 + b2 = |z|
luego |Re (z)| |z|

b) |Im (z)| = |b| = b2 a2 + b2 = |z|


luego |Im (z)| |z|
10. Probar que z = 0 |z| = 0 (Propiedad 3)
Solucin: Sea z = a + bi

) z = 0 a + bi = 0 a = 0 A b = 0

luego a2 + b2 = |z| = 0

) |z| = 0 a2 + b2 = 0 a2 + b2 = 0 a = 0 A b = 0
luego z = a + bi = 0
11. Probar que |z| = | z| = | z | (Propiedad 4)
Solucin: Sea z = a + bi

|z| =|a + bi| = a2 + b2

| z| = | a bi| = a 2+ b 2
= a2 + b2

| z | =|a bi| = a2 + ( b) 2 = a2 + b2
luego |z| = | z| = | z |
12. Demostrar |z w| = |z| |w| (Propiedad 5)
Solucin: Sean z = a + bi, w = c + di E k
|z w|= |(a + bi) (c + di)| = |(ac bd) + (ad + bc) i|

= ac bd 2 + ad + bc 2

= a2 c2 2 abcd + b2 d2 + a2 d2 + 2 abcd + b2 c2

= a 2 c 2 + d2 + b2 c 2 + d2 = a 2 + b2 c 2 + d2

= a 2 + b2 c2 + d2 = |z| |w|
z
13. Demostrar z = (Propiedad 6)
w w
Solucin: Sean z = a + bi, w = c + di E k.
z a + bi ac + bd bc ad
= = + i
w c + di c2 + d2 c2 + d2
2 2
= ac + bd bc ad
+
c2 + d2 c2 + d2

= a 2 c 2 + 2 abcd + b2 d2 + b2 c 2 2 abcd + a 2 d2
2
c2 + d2

394 Nmeros complejos

394-395 394 09/11/2001, 12:27


CAPTULO 9

a2 c2 + d2 + b2 c2 + d2 a2 + b2 c2 + d2
= =
2
c2 + d2 c2 + d2 2

a2 + b2 a2 + b2 z
= = =
c2 + d2 c2 + d2 w

14. Probar que z z = |z|2


Solucin: Sea z = a + bi
2
z z = (a + bi) (a bi) = a2 + b2 = a2 + b2 = |z|2
15. Probar que |z + w| |z| + |w| (Propiedad 7)
Solucin: Sean z, w E k
|z + w|2 = (z + w) ( z + w ) (ver ejercicio 14)
= (z + w) ( z + w) (Prop. 2 de conjugado)
= zz + zw + wz + ww

= |z|2 + zw + zw + |w|2 wz = zw
= |z|2 + 2 Re zw + |w|2 (Prop. 5 de conjugado)
|z|2 + 2 |Re zw | + |w|2 (x |x| I x E R)
|z|2 + 2 |z w | + |w|2 (Prop. 1 de valor absoluto)
= |z|2 + 2 |z| | w | + |w|2 (Prop. 5 de valor absoluto)
= |z|2 + 2 |z| |w| + |w|2 (Prop. 4 de valor absoluto)
= (|z| + |w|)2
Luego:
|z + w|2 (|z| + |w|)2 extrayendo raz cuadrada.
|z + w| |z| + |w|
16. Determinar el valor de a para que el valor absoluto del cociente
(3 2i) : (a + i) sea 3.
3 2i 13
Solucin: = 3 = 3
a + i a2 + 1
3 2i a i 13
= 3 = 9
a +i a i a2 + 1

3a 2 2a + 3 i
2
= 3 9a2 + 9 = 13
a + 1 a2 + 1
2 2
3a 2 2a + 3 4
+ = 3 a2 =
2 2 9
a2 + 1 a2 + 1

9a2 12a + 4 + 4a2 + 12a + 9


2
= 3 a=2
a2 + 1 3

13 a2 + 1
= 3
2
2
a + 1

Nmeros complejos 395

394-395 395 09/11/2001, 12:29


Ejercicios
1. Dados los siguientes nmeros comple- 9. Calcule el conjugado y el valor
jos, encuentre su conjugado. absoluto de (i4 i 11)3.
a) 6 2i b) 4 i c) 3 + 4i
10. Si z = a + bi, encuentre la parte real y
d) 2i e) 5 f) 1 + 2 i 1 z
2 la parte imaginaria de
2. Calcule el valor absoluto de los 1 + z
siguientes nmeros complejos. 11. Determine los nmeros complejos
a) 3 2i b) 9 i c) 3 + 5i tales que su mdulo sea 5 y la parte
real de su cuadrado sea 7.
d) 1 i e) i f) 3 i
12. Encuentre z complejo tal que
3. Dados los nmeros complejos z1 = 3 2i,
|z|2 = 37 y Im (z2) = 12.
z2 = 4 + i, z3 = 2i y z4 = 1 i,
encuentre: 13. Demuestre que I z, w E k, se
cumple que
a) |z3 z2| f) z 1 z 2 z + w 2
+ z w 2
= |z|2 + |w|2
2
b) z1 z3 g) z1 z2 + z3 z4 14. Demuestre que si dos complejos z
z1 z2
y w tienen mdulo 1, entonces
c) h) |z1 (z2 z4 )| 1 z w
z4 = 1
z w
d) |(z2 z3 ) z4| i) z1 + z2 z3 15. Demuestre que I z E k, z2 = z
2

z3 1 1 16. Encuentre z E k tal que:


e) j)
z3 + z4 z1 z2 |1 z| = 1 = |z|.
z
4. Calcule el valor absoluto de i8 17. Demuestre que i 4n + q = i q
i4 i3 I n, q E N.
5. Determine el valor de a para que el 18. Determine un nmero z E k tal
valor absoluto de (a 2i) sea 3. que su cuadrado sea el triple de
6. Encuentre x para que el valor absoluto su conjugado.
de (1 xi) (1 + i) sea 10. 19. Encuentre los complejos que satis-
7. Encuentre el valor de b para que el fagan que la mitad de su cuadrado es
valor absoluto del cociente (b 2i) : igual a un tercio de su conjugado.
(3 i) sea 2. 1
20. Pruebe que si w + w es real,
8. Determine x para que el conjugado de
(x i) (1 3i) sea igual a (1 + 7i). entonces Im (w) = 0 V |w| = 1.

Soluciones
1
1. a) 6 + 2i b) 4 + i c) 3 4i d) 2i e) 5 f) + 2 i
2
2. a) 13 b) 82 c) 34 d) 2 e) 1 f) 10
2
3. a) 2 17 b) 3 c) 1 + 2i d) 5 2 e) f) 2
5
1 1
g) 153 h) 117 i) 5 + 10i j)
13 17
1 1 1 2
4. 5. a = 5 6. x = 7 7. x = 6 8. x = 2 9. i;
2 4 4 4

396 Nmeros complejos

396-397(2003) 396 20/11/02, 5:53 PM


CAPTULO 9

1 a2 b2 1 3
10. ;
2b 16. z1 = + i ; z2 = 1 3
i
2 2 2 2 2 2 2 2
1 + a + b 1 + a + b

11. (4 + 3i), ( 4 3i), (4 3i), ( 4 + 3i) 17. Recuerde que i4a + q = (i4)u iq y i4 = 1
3 3 3
12. (6 + i), ( 6 i), ( 1 6i), (1 + 6i) 18. z1 = + i; z2 = 3 3 3 i
2 2 2 2
1 3
13. Use la propiedad |z|2 = z z 19. z1 = 1 + 3
i ; z2 = i
3 3 3 3
14. Eleve al cuadrado la igualdad y considere |z|2 = z z

Representacin trigonomtrica o
forma polar de un nmero complejo 9.3

9.3.1 Denicin de razones trigonomtricas


Sea P (x, y) un punto del plano cartesiano Y
tal que OP forma un ngulo a con el

}
eje x. P (x, y)
Sea OP = r.
Recordemos que:
} }
0
a
r
y

X
y x
Seno a = = sen a
r
x
Coseno a = = cos a
r
y
Tangente a = = tg a
x

9.3.2 Representacn trigonomtrica del complejo z = a + bi


El complejo z = a + bi geomtricamente representa un punto
en el plano cartesiano.

z = a2 + b2 es la distancia del origen del plano al punto


representado por z. A esta distancia la llamamos r.

z = a + bi
r= z = a 2 + b2 (1) b

|z|
Sea a el ngulo definido por el vector oz y
el eje x. a
0 a
El ngulo a se llama argumento del complejo z.

a est definido para todo z 0

Nmeros complejos 397

396-397(2003) 397 20/11/02, 5:54 PM


En la figura se ve que
a = |z| cos a y b = |z| sen a (2)
Sea z = a + bi, aplicando las relaciones (1) y (2) se tiene que
z = |z| cos a + |z| sen a i
z = r (cos a + i sen a)

Para escribir un complejo z = a + bi en su forma trigonomtrica


calculamos
1 r = a 2 + b2
2 Ubicamos a tal que cos a = ar y sen a = b
r
o bien ubicamos a tal que tg a = b , pero se debe cuidar de
a
determinar el cuadrante donde se encuentra el complejo z.
Ejemplo: Escribir z = 2 + 2i en forma trigonomtrica.
r= 22 + 22 = 2 2

tg a = 2 = 1 a = 45
2
luego z = 2 2 (cos 45 + i sen 45)

9.3.3 Producto y cociente de complejos en forma polar


PRODUCTO.
Si z1 = r1 (cos a1 + i sen a1) y cos (a + b) = cos a cos b - sen a sen b
z2 = r2 (cos a2 + i sen a2) y sen (a + b) = sen a cos b - cos a sen b
entonces: z1 z2 = r1 r2 [cos (a1 + a2) + i sen (a1 + a2)]

COCIENTE.
Si z1 = r1 (cos a1 + i sen a1)
z2 = r2 (cos a2 + i sen a2) , z2 0
z1 r1
entonces: = [cos (a1 a2) + i sen (a1 a2)]
z2 r2

9.3.4 Potenciacin de nmeros complejos en forma polar


Si z = r (cos a + i sen a) y n E N,

entonces: zn = rn (cos n a + i sen n a)

Nota: Si r = 1 entonces z = cos a + i sen a

(cos a + i sen a)n = cos na + i sen na

(Demostracin pg. 404 - ejercicios propuestos).

Esta frmula se conoce con el nombre de frmula de De Moivre


y tambin es vlida si n es negativo.

398 Nmeros complejos

398-399 398 10/11/2001, 17:01


CAPTULO 9

9.3.5 Radicacin de nmeros complejos en forma polar


Si z = r (cos a + i sen a) y n E N, entonces:
n n + 2k + 2k
z = r cos + i sen , k = 0, 1... n 1
n n

NOTA: Se pueden obtener todas las races n-simas de un complejo z si se


multiplica una de estas races por todas las races n-simas de la unidad.

1. Encontrar la forma polar de los nmeros complejos siguientes: Ejercicios


a) 1 3 i b) 1 + i c) 2i d) 4 resueltos
Solucin:

a) z = 1 3 i
2
r= 1+ 3 =2 1
3 3 z=1 3
tg a = a = 300o
1
luego: z = 1 3 i = 2 (cos 300o + i sen 300)

b) z = 1 + i z = 1 + i 1
2 2
r= 1 +1 = 2
1
1
tg a = = 1 a = 135o
1
luego: z = 1 + i = 2 (cos 135o + i sen 135o)

c) z = 2i
2 z = 2i
2 2
r= 0 + 2 =2
2
tg a = no est definida,
0
significa que a vale 90o

luego: z = 2 (cos 90o + i sen 90o)

d) z = 4

r= 42 + 02 = 4 z=4
4
tg a = 0 = 0 a = 0o
4
luego: z = 4 (cos 0o + i sen 0o)

Nmeros complejos 399

398-399 399 10/11/2001, 17:02


2. Escribir los siguientes complejos en su forma rectangular (a + bi).
a) 3 (cos 30o + i sen 30o) b) 2 (cos 135o + i sen 135o)
o
c) 2 (cos 90 + i sen 90 ) o d) 3 (cos 180o + i sen 180o)
Solucin:
Para transformar los nmeros complejos de su forma polar en
su forma rectangular basta calcular los valores del seno y del
coseno del ngulo sealado.
Para resolver estos problemas es conveniente tener a mano la
siguiente tabla:
a 30 45 60
1 1 3
sen a
2 2 2
3 1 1
cos a
2 2 2
1
tg a 1 3
3

a) z = 3 (cos 30o + i sen 30o) = 3


( 2
3
+ 1 i =
2 )
3 3
+ 3 i
2 2

b) z = 2 (cos 135o + i sen 135o) = 2 1 + 1 i = 1 + i


( 2 2 )
cos 135o = cos (90o + 45o) = cos 45o = 1
2
2
sen 135o = sen (90o + 45o) = sen 45o = 1
2

c) z = 2 (cos 90o + i sen 90o) = 2 (0 + i) = 2 i


d) z = 3 (cos 180o + i sen 180o) = 3 ( 1 + 0 i) = 3

3. Efectuar el producto de los siguientes nmeros complejos:


a) z1 = 2 (cos 30o + i sen 30o) b) z1 = 2 (cos 60o + i sen 60o)
z2 = 3 (cos 15o + i sen 15o) z2 = 3 (cos 240o + i sen 240o)

Solucin:
a) z1 z2 = [2 (cos 30o + i sen 30o)] [3 (cos 15o + i sen 15o)]
= 6 (cos 45o + i sen 45o)
b) z1 z2 = [ 2 (cos 60o + i sen 60o)] [ 3 (cos 240o + i sen 240o)]
= 6 (cos 300o + i sen 300o)

4. Efectuar el cociente de los siguientes nmeros complejos:

a) z1 = 4 (cos 25o + i sen 25o) b) z1 = 2 (cos 120o + i sen 120o)

z2 = 2 (cos 80o + i sen 80o) z2 = 2 (cos 90o + i sen 90o)

400 Nmeros complejos

400-401 400 10/11/2001, 17:08


CAPTULO 9

Solucin:
z1 4 cos 25 + i sen 25
a) = = 2 (cos ( 55o) + i sen ( 55o)
z2 2 cos 80 + i sen 80
= 2 (cos 305o + i sen 305o)
z1 2 cos 120 + i sen 120
b) = = cos 30o + i sen 30o
z2 2 cos 90 + i sen 90
5) Encontrar la potencia indicada de cada nmero complejo:
a) (5 + 2i)6 b) (3 2i)4 c) ( 8 i)5
Solucin:
Es conveniente transformar primero los nmeros complejos
a su forma polar.
a) z = 5 + 2i es un punto del primer cuadrante y por lo tanto el
argumento ser un ngulo menor que 90.
Si tg a = 2 , entonces a = arctg 2 = 21,8o
5 5
r= 25 + 4 = 29

luego z = 5 + 2i = 29 (cos 21,8o + i sen 21,8o)

y z6 = ( 29 )6 (cos (6 21,8o) + i sen (6 21,8o))


= 24.389 (cos 130,8 + i sen 130,8o)
Pasando este complejo a su forma cartesiana
z6 = 24.389 ( 0.6534 + 0,7570 i) = 15.935,8 + 18.462,5 i
\ (5 + 2i)6 15.935,8 + 18.462,5 i
b) z = 3 2i es un punto del cuarto cuadrante, luego 270o < a < 360
2
Si tg a = , entonces a = 326,3o
3
r = 9+4 = 13

z = 3 2i = 13 (cos 326,3o + i sen 326,3o)

z4 = (3 2i)4 = ( 13 )4 (cos (4 326,3o) + i sen (4 326,3o))

= 169 (cos 225,2o + i sen 225,2o)


que en la forma cartesiana es:
z4 = 169 ( 0,7046 0,7095 i) = 119,1 119,9 i
\ (3 2i)4 = 119,1 119,9 i 119 120 i
c) z = 8 i es un punto del tercer cuadrante y por lo tanto
180 < a < 270
Si tg a = 1 , entonces a = arctg 1 = 187,1
8 8

r= 64 + 1 = 65

Nmeros complejos 401

400-401 401 10/11/2001, 17:09


z = 8 i = 65 (cos 187,1o + i sen 187,1o)

z5 = ( 65 )5 (cos (5 187,1o) + i sen (5 187,1o))


= 34.063 (cos 215,5o + i sen 215,5o)
que en la forma cartesiana es:
z5 = 34.063 ( 0,814 0,581 i) = 27.727,3 19.790,6 i
\ ( 8 i)5 = 27.727 19.790 i
6. Calcular el valor de (1 + i)100
Solucin: z = 1 + i = 2 (cos 45o + i sen 45o)

z100 = (1 + i)100 = ( 2 )100 (cos 4.500o + i sen 4.500o)


(1 + i)100 = 1,11015 (cos 180o + i sen 180o)
= 1,11015 ( 1 + 0i) = 1,11015
\ (1 + i)100= 1,11015
7. Encontrar todas las races indicadas de:
5 3
a) 1 b) 1 i c) 4
3 + i y representarlas grficamente.
Solucin: a) z = 1 = (cos 0 + i sen 0o)
5 2 k + i sen 2 k
1 = cos
5 5
Si k = 0 : z1 = cos 0o + i sen 0o = 1

Si k = 1 : z2 = cos 2 + i sen 2 = 0,31 + 0,95 i


5 5
4 4
Si k = 2 : z3 = cos + i sen = 0,81 + 0,59 i
5 5
6 6
Si k = 3 : z4 = cos + i sen = 0,81 0,59 i
5 5

Si k = 4 : z5 = cos 8 + i sen 8
= 0,31 0,95 i
5 5

Grficamente: 1 z2
0,9
0,8
0,7
z3 0,6
0,5
5 0,4
8p/
/5 0,3
6p
0,2 4
p/5

0,1
2p/

1 0,9 0,8 0,7 0,6 0,5 0,4 0,3 0,2 0,1


5

z1
0,1 0,1 0,2 0,3 0,4 0,5 0,6 0,7 0,8 0,9 1
0,2
0,3
0,4
0,5
z4 0,6
0,7
0,8
0,9
1 z5

402 Nmeros complejos

402-403 402 09/11/2001, 13:19


CAPTULO 9

b) z = 1 i = 2 (cos 315o + i sen 315o)


3 3 315 + k 360 315 + k 360
1 i = 2 cos + i sen
3 3

Si k = 0 : 1,1 (cos 105o + i sen 105o) = 0,29 + 1,06 i


Si k = 1 : 1,1 (cos 225o + i sen 225o) = 0,78 0,78 i
Si k = 2 : 1,1 (cos 345o + i sen 345o) = 1,06 0,29 i
Grficamente: 1,0
0,9
0,8
0,7
0,6
0,5
0,4
0,3
225 1
0,2 05

0,1
-1,0 -0,9 -0,8 -0,7 -0,6 -0,5 -0,4 -0,3 -0,2 -0,1
0,1 0,2 0,3 0,4 0,5 0,6 0,7 0,8 0,9 1,0
-0,1
345

-0,2
-0,3
-0,4
-0,5
-0,6
-0,7
-0,8
-0,9
-1,0

c) z = 3 + i = 2 (cos 30o + i sen 30o)

4 4 30 + k 360 30 + k 360
3 + i = 2 cos + i sen
4 4

Si k = 0 : 1,19 (cos 7,5o + i sen 7,5o) = 1,18 + 0,16 i


Si k = 1 : 1,19 (cos 97,5o + i sen 97,5o) = 0,16 + 1,18 i
Si k = 2 : 1,19 (cos 187,5o + i sen 187,5o) = 1,18 0,16 i
Si k = 3 : 1,19 (cos 277,5o + i sen 277,5o) = 0,16 1,18 i

Grficamente: 1,2
1,1
1,0
0,9
0,8
0,7
0,6
0,5
5 0,4
,
187

0,3 9
0,2
7,5

-1,2 -1,1 -1,0 -0,9 -0,8 -0,7 -0,6 -0,5 -0,4 -0,3 -0,2 -0,1 0,1

-0,1 0,1 0,2 0,3 0,4 0,5 0,6 0,7 0,8 0,9 1,0 1,1 1,2
-0,2
-0,3
277 -0,4
-0,5
-0,6
-0,7
-0,8
-0,9
-1,0
-1,1
-1,2

Nmeros complejos 403

402-403 403 09/11/2001, 13:19


8. Usando la frmula de De Moivre, hallar una expresin para
sen 5 a y para cos 5 a en funcin de sen a y cos a
Solucin:
Se tiene, segn la frmula de De Moivre que (cos a + i sen
a)5 = cos 5 a + i sen 5 a
Desarrollando el primer miembro de esta ecuacin de acuerdo al
teorema del binomio e igualando tenemos:
(cos a + i sen a)5 = cos 5 a + i sen 5 a
5 5 5
cos5a + cos4a i sen a + cos3a (i sen a)2 +
0 1 2

5 5 5
cos2a (i sen a)3 + cos a (i sen a)4 + (i sen a)5
3 4 5
= cos 5a + i sen 5a
cos5a + 5 i cos4a sena 10 cos3a sen2a 10i cos2a sen3a +
5 cosa sen4a + i sen5a = cos 5a + i sen 5a
cos5a 10 cos3a sen2a + 5 cosa sen4a + (5 cos4a sena 10
cos2a sen3a + sen5a) i = cos 5a + i sen 5a
De donde:
cos 5a = cos5a 10 cos3a sen2a + 5 cosa sen4a
sen 5a = sen5a 10 sen3a cos2a + 5 sena cos4a
9. Demostrar que si z1 = r1 (cos a1 + i sen a1 ) y
z2 = r2 (cos a2 + i sen a2 ), entonces
z1 z2 = r1 r2 [cos (a1 + a2 ) + i sen (a1 + a2 )]
Solucin:
z1 z2 = [r1 (cos a1 + i sen a1 )] [r2 (cos a2 + i sen a2 )]
= r1 r2 [cos a1 cos a2 sen a1 sen a2 + i (sen a1 cos a2
+ cos a1 sen a2 )]
= r1 r2 [cos (a1 + a2 ) + i sen (a1 + a2 )]
10. Demostrar que si z1 = r1 (cos a1 + i sen a1 ) y
z2 = r2 (cos a2 + i sen a2 ), entonces
z1 r1
=
z2 r2
[cos (a1 a2 ) + i sen (a1 a2 )]
Solucin:
z1 r1 cos 1 + i sen 1 cos 2 i sen 2
=
z2 r2 cos 2 + i sen 2 cos 2 i sen 2

r1 cos 1 cos 2 + sen 1 sen 2 + i sen 1 cos 2 cos 1 sen 2


=
r2 cos2 2 + sen2 2
r1
= [cos (a1 a2) + i sen (a1 a2)]
r2
11. Demostrar que si z = r (cos a + i sen a), entonces
zn = rn (cos n a + i sen n a) I n E N

404 Nmeros complejos

404-405 404 20/11/02, 5:57 PM


CAPTULO 9

Solucin:
La demostracin de esta frmula la haremos por induccin. Es
decir, la verificamos para n = 1, la suponemos verdadera para n = k
y con esta hiptesis la demostramos para n = k + 1.
Si n = 1 z = r (cos a + i sen a), verdadero.
Hip: si n = k
zk = rk (cos ka + i sen ka)
Tesis: si n = k + 1
zk + 1 = rk + 1 [cos (k + 1) a + i sen (k + 1) a]
Demostracin:
Por hiptesis:
zk = rk (cos k a + i sen k a) / z
z k + 1 k
= r (cos k a + i sen k a) r (cos a + i sen a)
zk + 1 = rk r [cos (k a + a) + i sen (k a + a)]
zk + 1 = rk + 1 [cos (k + 1) a + i sen (k + 1) a]
\ Se ha demostrado que:
zn = rn [cos (n a) + i sen (n a)]
12. Demostrar que si z = r (cos a + i sen a), entonces:

n n + 2k + 2k
z = r cos + i sen
n n
k = 0, 1, ... n 1
Solucin:
n
Supongamos que r cos + i sen = (cos b + i sen b)
y encontremos y b
n
r cos + i sen = (cos b + i sen b) ( )n
r (cos a + i sen a) = n [cos (n b) + i sen (n b)]

r cos a = cos (n b)
n

r sen a = n sen (n b) ( )2

r2 cos2 a = 2n cos2 (n b)
r2 sen2 a = 2n sen2 (n b) sumando
n
r2 = 2n = r
reemplazando en la segunda ecuacin del sistema:
n n
r sen a = r sen (n b) /:r
sen a = sen (n b)

n b = a + 2k p
b = + 2k k = 0, 1, 2...
n

Nmeros complejos 405

404-405 405 20/11/02, 5:58 PM


Luego,
n n + 2kp + 2kp
r (cos + i sen ) = r cos + i sen
n n

con k = 0, 1, 2, n 1 ya que para valores de n enteros


menores que 0 o mayores que n 1 los valores empiezan
a repetirse.

13. Resolver la ecuacin x6 2x3 = 2


Solucin:
Haciendo u = x3 nos queda u2 2u + 2 = 0 cuyas
soluciones son: u1 = 1 + i y u2 = 1 i
Como u = x3 debemos resolver
3
a) x3 = 1 + i x = 1 + i
3
b) x3 = 1 i x = 1 i
1 1
a) 1 + i 3 = 2 (cos 45 + i sen 45) 3

1
3
45 + k 360 45 + k 360
= 2 cos + i sen
3 3
Si k = 0 : 1,12 (cos 15 + i sen 15) = 1,08 + 0,29i
Si k = 1 : 1,12 (cos 135 + i sen 135) = 0,79 + 0,79i
Si k = 2 : 1,12 (cos 255 + i sen 255) = 0,29 1,08i
1 1
3
b) 1 i 3 = 2 (cos 315 + i sen 315)
1
3 315 + k 360 315 + k 360
= 2 cos + i sen
3 3
Si k = 0 : 1,12 (cos 105 + i sen 105) = 0,29 + 1,08i
Si k = 1 : 1,12 (cos 225 + i sen 225) = 0,79 0,79i
Si k = 2 : 1,12 (cos 345 + i sen 345) = 1,08 0,29i
As vemos que las 6 soluciones de la ecuacin x6 2x3 + 2 = 0 son
x1 = 1,08 + 0,29i x2 = 0,79 + 0,79i x3 = 0,29 1,08i
x4 = 0,29 + 1,08i x5 = 0,79 0,79i x6 = 1,08 0,29i

Ejercicios
1. Escriba los siguientes complejos en su forma polar.
a) 1 i b) 3 + i c) 3 + 3 i d) 2 + i
e) 2 2i f) 7 + 3i g) 6 7i h) 1 + 9i
i) 10 + i j) 6i k) 5 l) 12i
2. Escriba los siguientes complejos en su forma rectangular (a + bi)
a) 4 (cos 45 + i sen 45) b) 3 (cos 135 + i sen 135)

406 Nmeros complejos

406-407 406 09/11/2001, 13:25


CAPTULO 9

c) 3 (cos 30 + i sen 30) b) z1 = 2 (cos 350 + i sen 350)


d) cos 60 + i sen 60 z2 = 4 (cos 140 + i sen 140)
e) 4 (cos 270 + i sen 270) c) z1 = 8 (cos 10 + i sen 10)
f) 2 (cos 210 + i sen 210) z2 = 4 (cos 340 + i sen 340)
g) 5 (cos 240 + i sen 240) d) z1 = 6 (cos 345 + i sen 345)
h) 3 (cos 330 + i sen 330) z2 = 2 (cos 300 + i sen 300)
z1
i) 12 (cos 180 + i sen 180) 6. Efecte los siguientes cocientes
z2
j) 1 (cos 315 + i sen 315) a) z1 = 3 (cos 25 + i sen 25)
2
k) 2 (cos 225 + i sen 225) z2 = 2 (cos 30 + i sen 30)
l) 5 (cos 270 + i sen 270) b) z1 = 3 (cos 12 + i sen 12)
z2 = 2 (cos 10 + i sen 10)
3. Efecte los siguientes productos z1 z2
c) z1 = 12 (cos 130 + i sen 130)
a) z1 = 2 (cos 240 + i sen 240)
z2 = 4 (cos 52 + i sen 52)
z2 = 1 (cos 60 + i sen 60)
2
d) z1 = 5 (cos 250 + i sen 250)
b) z1 = 2 (cos 30 + i sen 30)
z2 = 4 (cos 20 + i sen 20)
z2 = 6 (cos 150 + i sen 150)
e) z1 = 2 (cos 15 + i sen 15)
c) z1 = 3 (cos 25 + i sen 25) z2 = 1 (cos 350 + i sen 350)
z2 = 2 (cos 65 + i sen 65) 2

d) z1 = 2 (cos 130 + i sen 130) 7. Calcule las siguientes potencias:


a) (3 + 2i)4 e) (2 i)5
z2 = 5 (cos 95 + i sen 95)
b) (5 i)5 f) (3 + i)4
4. Efecte los siguientes productos z1 z2 c) (5 + 2i) g) (1 + 2i)6
6

a) z1 = 3 (cos 22 + i sen 22) d) (1 i)4 h) (5 2i)3

z2 = cos 35 + i sen 35 8. Calcule el valor de (1 i)50


b) z1 = 2 (cos 30 + i sen 30) 9. Encuentre todas las races indicadas y
represntelas grficamente
z2 = 12 (cos 20 + i sen 20)
5 4 3
c) z1 = 2 (cos 120 + i sen 120) a) 2 b) 1+ i c) 8i
z2 = 4 (cos 100 + i sen 100) 10. Resuelva la ecuacin x4 + 1 = 0
d) z1 = 3 (cos 220 + i sen 220)
11. Resuelva la ecuacin x3 + 2 + 2i = 0
z2 = 3 (cos 130 + i sen 130)
12. Pruebe que:
e) z1 = 2 (cos 190 + i sen 190)
a) sen 3x = 3 sen x 4 sen3x
z2 = 2 (cos 330 + i sen 330)
b) cos 3x = 3 cos x + 4 cos3x
z1 13. Resuelva la ecuacin
5. Efecte los siguientes cocientes
z2
a) z1 = 3 (cos 210 + i sen 210) x6 + 4x3 + 5 = 0

z2 = 3 (cos 60 + i sen 60) 14. Encuentre dos nmeros que sumados


den 4 y multiplicados den 8.

Nmeros complejos 407

406-407 407 09/11/2001, 13:26


Soluciones

1. a) 2 (cos 315 + i sen 315) c) 3 +i d) 3 + 3 i


2 2
6. a) 1,5 0,13 i b) 1,22 + 0,04 i
b) 2 (cos 30 + i sen 30)
c) 0,62 + 2,93 i d) 0,80 0,96 i
c) 2 3 (cos 150 + i sen 150)
e) 1,81 + 0,85 i
d) 5 (cos 26,5 + i sen 26,5) 7. a) 119 + 120 i b) 1.900 2.876 i
c) 15.939 18.460 i d) 4
e) 2 2 (cos 225 + i sen 225)
e) 38 41 i f) 28 96 i
f) 58 (cos 156,8 + i sen 156,8)
g) 117 + 44 i h) 65 142 i
g) 85 (cos 310,6 + i sen 310,6) 8. 225 i

h) 82 (cos 83,7 + i sen 83,7) 9. a) 1,15; (0,36 + 1,09 i); ( 0,93 + 0,68 i);
( 0,93 0,68 i); (0,36 1,09 i)
i) 101 (cos 5,7+ i sen 5,7)
b) (3,92 + 0,78 i); ( 0,78 + 3,92 i);
j) 6 (cos 270 + i sen 270)
( 3,92 0,78 i); (0,78 3,92 i)
k) 5 (cos 180 + i sen 180) c) ( 1,86 + 3,56 i); ( 2,15 3,39 i);

l) 12 (cos 90 + i sen 90) (4,02 0,08 i)

1 1 1 1
2. a) 2 2 + 2 2 i b) 3 + 3 i 10. + i ; + i
2 2 2 2 2 2
3 3 1 3
c) + i d) + i
1 1 1 1
2 2 2 2
i ; i
2 2 2 2
e) 4 i f) 3 i
11. (0,37 + 1,37 i); ( 1,37 0,37 i); (1 i)
5 5 3
g) i h) 3 3

3
i
2 2 2 2 12. Usar frmula de De Moivre para
i) 12 j) + 1

1
i
(cos + i sen )3
2 2 2 2
13. x1 = 1,29 + 0,20 i
k) 1 i l) 5 i
x2 = 0,82 + 1,02 i
3. a) 0.5 0.87 i b) 2 3
x3 = 0,47 1,22 i
c) 6i d) 5 2 5 2 i
x4 = 0,47 + 1,22 i
4. a) 1,63 + 2,52 i b) 10,9 + 13 i
x5 = 0,82 1,02 i
c) 6,13 5,14 i d) 5,12 0,9 i
x6 = 1,29 0,20 i
e) 3,76 + 1,37 i
14. 2 + 2 i; 2 2i
1 3 3 1
5. a) + i b) i
2 6 4 4

408 Nmeros complejos

408-409 408 10/11/2001, 17:13


CAPTULO 9

Prueba de seleccin mltiple


1. El valor de 5. Si z1 = 3 2 i y 9. El valor de i 13 es:
25 + 2 4 36 z2 = 4 + i entonces
A. 0
es: z 1 z2 =
B. 1
A. 3i A. 14 + 5 i
C. 1
B. 4i B. 14 5 i
D. i
C. 5i C. 14 5 i
D. 14 + 5 i E. i
D. 6 i
E. 5 14 i
E. 6 i
6. Si z1 = 4 2 i y 10. El valor de
2. El inverso aditivo de z2 = 3 + 6 i entonces (i11 + i 5)6 es:
2 5 i es: z 1 : z2 = A. 64
8 2
A. 2 + 5 i A. + i B. 64
15 5
B. 25i 8 2 C. 32
B. i
15 5
C. 2+5i 8 2 D. 32
C. + i
D. 5 2 i 15 5 E. 16
8 2
E. 5+2i D. i
15 5
2 8
E. + i
5 15 11. El valor de
3. Si z1 = 4 2 i y
z2 = 3 + 5 i entonces 7. El inverso multiplicativo ( i17 + i126)2 es:
z1 + z2 = de 1 + 2 i es:
A. 1
A. 13i 1 2 B. 1
A. 5 5 i
B. 7+3i C. i
1 2
B. i
C. 1+3i 5 5 D. i
D. 1 + 3 i
1
+
2
i
C. E. 2i
5 5
E. 7 3 i
D. 12i
4. Si z1 = 2 5 i y 12. Si z = 1 + 3 i
z2 = 5 i entonces E. 1 2 i
entonces z2 es:
z1 z2 =
8. El valor de i112 es: A. 86i
A. 2 + 10 i
A. 0 B. 8 + 6 i
B. 2 10 i B. 1 C. 8 6 i
C. 2 + 10 i C. 1
D. 6 + 8 i
D. 2 10 i D. i
E. 6 + 8 i
E. 2 E. i

Nmeros complejos 409

408-409 409 10/11/2001, 17:13


Prueba de seleccin mltiple
13. Si z = 3 + 5i, entonces 17. Son soluciones de la 21. En la igualdad
1 + z + z2 = ecuacin 2x 1 + i = 3 + i, x vale
A. 18 25i x2 2x + 5 = 0 A. 0
B. 18 25i I) (1 + 2i) II) (1 2i) B. 1
C. 18 + 25i III) 2
C. 2
D. 20 + 25i
A. I y II D. 1
E. 20 + 25i
B. I y III E. i
14. El valor de C. II y III
1 1 1 1 1 D. slo III 22. En la igualdad
+ 2+ 3+ 4+ 5 es: (x 2yi) (1 i) = 7 + i
i i i i i E. ninguna los valores de x e y
A. 0
respectivamente son:
B. 1 18. La diferencia entre los
complejos z1 y z2 es: A. 2 ; 3
C. 1
3 + 6i, si z2 = 2 z1
B. 3 ; 2
D. i entonces z2 vale
E. i C. 2 ; 3
A. 3 6i
D. 3 ; 2
15. Si z1 = 2 i , B. 6 12i
E. 2 ; 3
z2 = 2i y z3 = 4 + 2i, C. 3 6i
D. 6 12i 23. El nmero complejo
entonces 1 (z2 + z3) =
z1 E. 6 + 12i cuyo cuadrado es
8 3 4i es:
A. + 8 i
5 5 19. Si z = 1 i y A z2 = 1,
A. 2 i
entonces A vale
B. 8 4 i
5 5 B. 2 + i
A. 1 i
4 2
C. + 8 i 1
C. 2 i
5 5 B. i
2 D. 3 + i
4 8 C. 1 + 2i
D. i
5 5 E. 3 i
4 D. 1 2i
E. 8 i
5 5 E. 1 i x + i
24. Para que sea un
1 + i
16. Si z1 = 4 2i y imaginario puro, x debe
z2 = 5 + 6i, entonces 20. El valor de (i2 i1 )2 es:
valer:
Re (z1 z2) es: A. 2i
A. 9 A. 1
B. 2i
B. 12 B. 1
1
C. 2i C. 0
C. 14
1
D. 20 D. D. 2
2i

E. 32 E. 1i E. 2

410 Nmeros complejos

410-412 410 10/11/2001, 17:17


CAPTULO 9

1 2i 33. El conjugado de
25. Para que sea un 29. El valor de z E k que
3x i (i5 + i12)1 es:
nmero real, x debe satis face la ecuacin
z 1 =0 A. 1 + i
tomar el valor: z
B. 1 i
A. cualquier complejo
A. 6
B. 1 y 1 C. 1 + 1 i
B. 6 2 2
1
C. 1 2i D. 1 1 i
C. 6 2 2
1
D. i
D. 6 E. 1 1 i
E. i 2 2
E. 1 34. Un complejo cuya parte
30. Si z = 1 3i, entonces
real es 3 y cuyo valor
z es:
26. Si z = 1 i, entonces absoluto es 13 es:
2z2 z + 1 vale: A. 1 + 3i A. 3 + 2i
B. 1 + 3i B. 3 + 2i
A. 3
C. 1 3i C. 3 2i
B. 3 D. 3 3i
D. 1 + 1i
C. 3i 3 E. 3 + 3i
D. 3i E. 1 1i
3 35. Un nmero complejo
E. 3 + 3i 31. Si z1 = 1 2i y z2 = 3i, tal que su cuadrado es la
z1 mitad de su conjugado
entonces es:
z2 es:
27. En la ecuacin 5 1 3
z (1 i) + 3 = 1 2i + 2z, A. A. + i
2 4 4
z vale: 3
B.
5
B. 1 + i
3 4 4
A. 2 1 1 3
C. C. i
3 4 4
B. 2 1 3
D. + i
C. 2i D. 2 4 4
3 1 3
E. i
D. 2i E. 1 4 4

E. 1 2i 36. El complejo 2 + 2i en
32. El valor absoluto de forma polar es:
i10
es: A. 2 2 (cos 45 + i sen
28. Si z = 2 3i, la parte i4 + i3
45)
imaginaria de 1 es: A. 2
z2 B. 2 (cos 45 + i sen
1 45)
A. 12 B.
2
C. 2 2 (cos 135 +
B. 12 13 C.
2
2 i sen 135)
C. 12 132
D. 1 D. 2 2 (cos 225 +
D. 12 132 i sen 225)
2
E. 13 E. E. 2 2 (cos 315 +
2
i sen 315)

Nmeros complejos 411

410-412 411 10/11/2001, 17:19


Prueba de seleccin mltiple
37. El complejo 5 (cos 180 40. Si z1 = 4 (cos 100 + 43. Son races cuarta de 1:
+ i sen 180) en su forma i sen 100) y z2 = 2
1
(cos 55 + i sen 55), I) + 1 i
cartesiana es: z1 2 2
A. 5 entonces z =
2
II) 1 + 1 i
B. 5 2 2
A. 2 + 2i 1 1
C. 5i III) i
2 2
D. 5i B. 2 2i 1 1
IV) i
E. 5 + 5i C. 2 + 2 i 2 2
A. slo I
38. Si z1 = 2 (cos 40 + D. 2 2 i
B. slo I y IV
i sen 40) y z2 = E. 2 + 2 i
1
C. slo II y III
(cos 20 + i sen 20), D. todas
2 41. Si z1 = 6 (cos 40 +
entonces z1 z2 = E. ninguna
i sen 40) y z2 = 3 (cos
1 3
A. + i 280 + i sen 280), enton- 44. Dos nmeros cuya suma
2 2
z es 5 y su producto es
3 1 ces z 1 =
B. + i 2 25 son:
2 2
A. 2 2 3 i 5+2 3 52 3
C. 1
+ 3 i A.
2
i; i
2 2 2
3 B. 1 3i 5+3 3 53 3
D. + 1 i B.
2
i;
2
i
2 2
1 C. 1 + 3i 5+4 3 54 3
E. 3 i C. i; i
2 2 2 2
D. 1+ 3i 55 3
39. Si z1 = 2 (cos 290 + 5+5 3
D. i; i
1 1 2 2
i sen 290) y z2 =3 (cos E. 3i
2 2 E. 2+ 3 i;3 3 i
100 + i sen 100),
entonces z1 z2 = 42. El valor de (2 + i)5 45. Es solucin de la ecua-
cin x3 + 2 = 2i
A. 3 3 3i A. 38 + 41i
A. 1+i
B. 3 3 + 3i B. 38 41i
B. 1+i
C. 3+3 3i C. 38 + 41i C. 2+i
D. 33 3i D. 32 + 41i D. 2+i
E. 3 3 3i E. 1i
E. 32 41i

Soluciones

1. A 8. B 15. A 22. D 29. B 36. C 43. D


2. C 9. E 16. E 23. A 30. B 37. B 44. D
3. C 10. B 17. A 24. B 31. B 38. A 45. E
4. E 11. E 18. B 25. C 32. C 39. B
5. B 12. C 19. B 26. D 33. D 40. C
6. D 13. B 20. C 27. A 34. C 41. C
7. B 14. E 21. C 28. D 35. B 42. C

412 Nmeros complejos

410-412 412 10/11/2001, 17:21


CAPTULO 10
V ectores

Deniciones 10.1

Llamamos vector a un segmento dirigido. A su punto inicial


lo llamamos origen y a su punto nal extremo. Distinguimos el
extremo porque en l dibujamos una punta de echa.
Denotamos un vector: AB, PQ, RS , u, v.
B S
v
u
P Q
A R
Cada vector se caracteriza por tener magnitud, direccin y
sentido.
La magnitud o longitud es la distancia entre su origen y su
extremo y se llama valor absoluto, mdulo o norma del vector. Se
designa por ||AB|| o ||v||.
La direccin es la direccin de la recta que contiene al vector y
de todas sus paralelas. Se representa por el ngulo , que se forma
entre la horizontal y la recta que contiene al vector.
Cada direccin admite dos sentidos, y ste est dado por la
punta de la echa.
Todos los vectores que trasladados paralelamente coinciden,
constituyen el mismo vector. En un sistema de referencia
cartesiano, los asimilamos con el vector cuyo origen es el (0, 0) y
cuyo extremo es (u1, u2).

u2
u u1 y u2 se llaman coordenadas
q del vector u = (u1, u2).
u1

Vectores 413

413-421(2003).indd 413 01-02-2006 13:32:24


El valor absoluto, mdulo o norma del vector u = (u1, u2) es:
|| u|| = u21 + u22 y corresponde a la longitud de la echa que lo
representa.

Existen mltiples situaciones de la vida real que se representan


con vectores; stas se llaman magnitudes vectoriales. Por ejemplo:
un avin se dirige a aterrizar a una velocidad constante de 160 Km/hr
en una trayectoria que forma un ngulo de 15 con la horizontal.
El sentido est dado hacia la pista.

15
160

Otras situaciones como el nmero de hijos en una familia, la


edad de una persona o el valor de una casa quedan perfectamente
definidas con un nmero. No requieren de una direccin ni
de un sentido para precisarlas. Estas se llaman MAGNITUDES
ESCALARES.

10.2 Operaciones con Vectores

10.2.1 Suma de vectores

v2 Sean u = (u1, u2) y v = (v1, v2)


v vectores, entonces:
+
v u
u + v = (u1 + v1, u2 + v2)

u2 Geomtricamente, el vector suma


u de dos vectores es la diagonal del
paralelogramo que se forma con
v1 u1
ambos vectores.

414 Vectores

413-421(2003).indd 414 01-02-2006 13:32:26


CAPTULO 10

Tambin podemos decir que el vector


suma est representado por la flecha
+v
que resulta al unir el origen de u con el u
v
extremo de v, despus de haber trasladado
paralelamente v hasta que su origen
coincida con el extremo de u. u
v

10.2.2 Producto por escalar

Sea u = (u1, u2) un vector y


k R un escalar, entonces:
k u = (ku1, ku2)
3u2
Geomtricamente, el vector
3u producto por escalar resulta de
u2 poner el vector a continuacin
u
de s mismo tantas veces como
u1 3u1 indica el escalar. En la gura
k = 3.

Si el escalar es negativo (k < 0),


entonces el vector producto por
escalar resulta de sentido inverso v2
2v1 v
al vector u.
v v1
2
Nota: El vector 0 = (0, 0)
El vector u = 1 u 2v2

10.2.3. Propiedades de la suma y


el producto por escalar
Sean u, v y w vectores y k1, k2 escalares en R.

1. u + v = v + u
2. u + ( v + w) = ( u + v) + w
3. u + 0 = u
4. u + ( u) = 0
5. k1 ( u + v) = k1 u + k1 v
6. (k1 + k2) u = k1 u + k2 u
7. (k1 k2) u = k1(k2 u)
8. 1u=u

Ver demostraciones en ejercicios resueltos N3.

Vectores 415

413-421(2003).indd 415 01-02-2006 13:32:27


10.2.4 Resta de vectores
Sean u = (u1, u2) y v = (v1, v2)
dos vectores, entonces:

u v = u + ( v)

u
v

v
Geomtricamente, para
restar el vector v del vector u
u dibujamos desde el mismo origen
de u el vector v y la diagonal del

u
paralelogramo as formado es el
v
v
vector u v.
Observacin: el vector que va
del extremo de v al extremo de u
tambin es u v.

v
Adems, podemos decir que
si a continuacin del vector u
dibujamos v, la echa que une u
el origen de u con el extremo de v
v representa el vector u v. uv

10.3 Vector Unitario

10.3.1 Denicin

Se llama VECTOR UNITARIO al vector cuyo valor absoluto o norma es 1.

Ejemplo: u =
( 12 , 5
13 13 ) || u|| =
144 25
+
169 169
=
169
169
=1

v=
( 2 , 5
3 3 ) || v|| =
4 5
+ =
9 9
9
9
=1

416 Vectores

413-421(2003).indd 416 01-02-2006 13:32:29


CAPTULO 10

Hay dos vectores unitarios especiales denotados por:


= (1, 0) J = (0, 1)

Cualquier vector puede ser escrito como combinacin lineal


(C.L) de ambos:
Ejemplo: (5, 2) = 5(1, 0) + 2(0, 1) = 5+ 2J
Observamos que indica la componente horizontal y J seala la
componente vertical del vector.

10.3.2 Normalizar un vector

Se llama normalizar un vector u al procedimiento utilizado para


conseguir otro vector con la misma direccin y sentido que el
vector original pero de magnitud, mdulo o norma 1.

Para ello, basta multiplicar el vector dado por el inverso de su


norma:

Ejemplo 1:

Sea u = (3,4); || u|| = 9 + 16 = 25 = 5

Entonces,

= 1
|| u||
u=
1
5
(3,4) =
( )
3 , 4
5 5

9 16
en efecto, ||u|| = + =1
25 25

Ejemplo 2:

Sea u = 2 + 4J ; || u|| = 4 + 16 = 20 = 2 5

Entonces,

1 1 1 2
= u= (2 + 4J) = +
5 J

|| u|| 2 5 5

1 4
en efecto, ||u|| = + =1
5 5

Vectores 417

413-421(2003).indd 417 01-02-2006 13:32:30


10.4 Descomposicin de un vector

Sea u = (u 1 , u 2 ) un vector cualquiera. Este puede


descomponerse en su parte horizontal y su parte vertical como
sigue: llamamos al ngulo medido desde el eje horizontal al
vector u , entonces:

u1 u2
cos = y sen = , luego
|| u|| || u|| u2

u1 = || u|| cos y u2 = || u|| sen


u
Es decir, el vector u = (u1, u2) puede escribirse:

u= || u|| cos + || u|| sen J
u1

Ejercicios 1. Sean u = (2, 4), v = (3, 1) y los escalares k1= 3 y k2 = 1


a) Hallar y gracar k1 u + v
resueltos
b) Calcular k2( u + v)
Solucin:
a) k1 u + v = 3(2, 4) + (3, 1)
= (6, 12) + (3, 1)
= (3, 13)

Grcamente:

12


u
8
v


3u +


u

4


u
v
| | | | | | | | | |
3 2 1 2 4 6

418 Vectores

413-421(2003).indd 418 01-02-2006 13:32:34


CAPTULO 10

b) k2( u + v) = 1((2, 4) + (3, 1))


= 1(1, 5)
6
_
= (1, 5)
5
_
Grcamente: _
_
_ u
v _

_
_
_
_
_

_
_
_
_
_
3 _ 1 2
_
_
_ 1( u + v)
_
6 _

2. Determinar la norma del vector u = (5, 3)

Solucin: || u|| = ||(5, 3)|| = (5)2 + 32 = 25 + 9 = 34

3. Demostrar las siguientes propiedades de la suma y el producto por


escalar.
a) u + v = v + u
Demostracin:
u + v = (u1, u2) + (v1, v2) (sumando los vectores)
= (u1 + v1, u2 + v2) (aplicando conmutatividad de la
suma en R)
= (v1 + u1, v2 + u2) (Descomponiendo una suma
de vectores en sumandos)
= (v1,v2) + (u1 , u2)
=v+u
b) (k1 + k2) u = k1 u , k2 u

Demostracin:
(k1 + k2) u = (k1 + k2) (u1, u2) (multiplicando por escalar)
(
= (k1 + k2)u1, (k1 + k2)u2 ) (aplicando distributividad
del producto sobre la
suma en R.)
= (k1u1 + k2u1, k1u2 + k2u2) (descomponiendo la suma
de vectores en sumandos)
= (k1u1, k1u2) + (k2u1, k2u2) (aplicando producto por
escalar)
= k1(u1, u2) + k2(u1, u2)
= k1 u + k2 u

Vectores 419

413-421(2003).indd 419 01-02-2006 13:32:37


4. Normalizar el vector u = 3 + 7J _
7
6
_
Solucin:
1 5
_
u= u
|| u|| 4
_
3_
|| u|| = 9 + 49 = 58
2_
1 3 7
u= (3 + 7J) = + J 17 _
58 58 58 58

_
_
_
3
u (0.4, 0.9) 58 1 2 3

5. Hallar la norma y la direccin del vector u = (4, 3)


Solucin:

a) || u|| = (4)2+32 = 25 = 5
3_
u 4
2_ b) cos = 1 =
_ || u|| 5
u 1

= arcos ( 45 ) = 143,1
_
_
_
_

4 3 2 1

La norma del vector u es 5 y su direccin es de 143,1.

6. Hallar las componentes horizontal y vertical del vector de norma


12 y direccin 54.
Solucin:
Sabemos que
u2 u1 = || u||cos y u2 = || u||sen
u Por lo tanto:
54 u1 = 12 cos 54 3,4641 0,5878 = 2,0362
u1
u2 = 12 sen 54 3,4641 0,8090 = 2,8025

3
7. Determinar el valor de m para que el vector u = + 2mJ sea
4
un vector unitario.
Solucin: un vector unitario es aquel cuya norma es 1.
Por lo tanto:

( 34 ) + (2m) = 1
2
|| u|| = 2

9 2
16 + 4m = 1
7
4m2 =
16
2 7
m =
64
7
m=
8

420 Vectores

413-421(2003).indd 420 01-02-2006 13:32:40


CAPTULO 10

Luego los vectores unitarios que resultan son:

3 3
u= + 7 J y u= 7 J
4 8 4 8

8. Determinar el valor de m para que el vector u = 2 + mJ sea un


vector unitario.
Solucin: Para que u sea unitario debemos hacer || u|| = 1.
|| u|| = 22 + m2 = 4 + m2 = 1
4 + m2 = 1
m2 = 3
No existe un nmero real tal que su cuadrado sea 3. Esto se
debe a que ningn vector de R2 cuya primera coordenada sea 2
puede ser unitario.

1 Observe que el lugar


geomtrico de todos
los vectores unitarios es
-1 1 2 circunferencia de radio 1.

-1

9. Dados los vectores: u v w

mostrar geomtricamente que ( u + v) + w = u + ( v + w)

Solucin: v w
v+w
u v
u+ )+ w
(u + v
+ w)
u + (v
10. Dado los vectores u = (3, 2) y v = (4, 5), hallar la norma del
vector u v. Gracarlo.

Solucin: || u v|| = (3 4)2 + (2 5)2 = 1 + 49 = 50

5


v
uv


_
_
_
_

3 4
u
2

Vectores 421

413-421(2003).indd 421 01-02-2006 13:32:43


11. En un punto A de un objeto se aplica una fuerza de 42N en
direccin 60. Calcular la magnitud de la fuerza que acta en
forma horizontal y de la fuerza que acta en forma vertical.

2 N
42 Solucin:
A 60 1 = 42 cos 60 = 21N
1
2 = 42 sen 60 = 21 3 N

La fuerza que acta en direccin vertical es de 21 3 N y la


que acta en forma horizontal es de 21N.

12. Un mvil se desliza en un plano inclinado en 32 con una


velocidad instantnea de 12 m/seg. Calcular la componente
horizontal y la componente vertical de dicha velocidad.

v1
32
v2 v1 = 12 cos 32 = 10,18 m/seg
32 v2 = 12 sen 32 = 6,36 m/seg

Ejercicios
1. Determine cules de las siguientes 3. Determine la direccin y la norma
medidas se representan por vectores de los siguientes vectores.
y cules por escalares: a) v = 2 + 7J
a) 5 hijos b) v = 5 3J
b) 25 panes c) v = 2 9J
c) 18 km/hr d) v = 4 + 5J
d) $25.000 1 25
e) v = +
e) La velocidad de aproximacin 3 4 J
de un avin. 1
f) v = 3 + J
f) Desaceleracin a 5 m/seg2. 2
4. Realice las siguientes operaciones
2. Determine la direccin y la norma entre los vectores u = 2 + 3J ,
de los siguientes vectores. Dibjelos v = 4 J , w = 3 2J , p = 5,
en el plano cartesiano. s = 6J y grafiquelos en el sistema
a) u = (5,2) cartesiano.
b) u = (2,7) a) u + v
b) 2u 3v
c) u = (5,1)
c) u + v + w
d) u = (6,3) d) u 5p
e) u = (5, 2) e) 3(u + s )
f) u = ( 5, 3 ) f) p 2u + 3w

422 Vectores

422-431 422 25/11/02, 12:00 PM


CAPTULO 10

5. Escriba los siguientes vectores como 10. Encuentre la longitud y direccin


combinacin lineal de los vectores de los siguientes vectores:
= (1, 0) y J = (0, 1). a) u = (5, 7)
a) u = (2, 5) b) v = (3, 2)
b) v = (3, 2) c) w= (10, 8)
c) w = (8, 3) d) u = (6, 3)
d) u = (5, 0) e) v = (5, 0)
e) v = (7, 0) f) w = (0, 3)
f) w = (0, 3)
11. Encuentre la longitud y direccin de
6. Encuentre un vector unitario que tenga los siguientes vectores:
la misma direccin que el vector dado. a) u = 2 5J
a) u = (4, 2) b) v = + 6J
b) u = (6, 1) c) w= 2 + 2J
c) u = (5, 3) d) u = 5 4J
d) u = (3, 4) e) v = 2
e) u = (3, 8) f) w= 4J
f) u = (5, 0)
12. Encuentre las componentes horizontal
7. Encuentre un vector unitario que y vertical de los vectores, conocida su
tenga la misma direccin que el magnitud y su direccin.
vector dado. a) 2, 45
a) v = 3 + 5J b) 5, 30
b) v = + 4J 1
c) , 60
c) v = 2 6J 3
d) v = 4 J d) 10, 12

e) v = 6 e) 12,
4
f) v = 4J f) 100, 115
8. Normalice los siguientes vectores g) 25, 90
a) u = (6, 2) h) 4, 135
b) v = (9, 3) i) 8, 212
c) w = (5, 8) j) 3, 270
d) u = (1, 3) 3
k) 18,
4
e) v = (1, 1) l) 5,
f) w = (0, 5)
13. Dados los vectores u = 3 + 5J y
9. Normalice los siguientes vectores. v = 6 2J, encuentre la norma del
a) u = 3 5J vector 2u 3v.
b) v = 2 + J 14. Para trasladar horizontalmente un
c) w = 3J objeto desde un punto A hasta un
d) u = 2 + 2 J punto B se aplica una fuerza de
30N en direccin 30. Calcular la
1 3
e) v = magnitud de la fuerza que acta en
2 4 J
la direccin AB y la fuerza que acta
f) w = 5 en direccin normal a AB.

Vectores 423

422-431 423 25/11/02, 12:00 PM


15. Un avin se desplaza en direccin 16. Una persona desea cruzar un ro
60 NO a una velocidad de 800 en bote a una velocidad media de
km/hr y no hay viento. Al llegar 32 km/hr y sabe que el agua fluye
a cierto punto de su trayectoria se a 8 km/hr. Qu velocidad y en
encuentra con un viento sobre l qu direccin debe imprimir a
de 60 km/hr en direccin 30 NO. su bote para mantener un rumbo
Calcule la velocidad real del avin al perpendicular a la orilla del ro?
ser expuesto a ese viento.

Soluciones

1. a) escalar b) escalar c) vectorial


d) escalar e) vectorial f) vectorial

2 _
1 _ 6 5 4 3 2 1
2. a) ; 29; 21,8 d) ; 3 5; 206,6
_
_
_
_
_

_
_
_
_
_
_
1 2 3 4 5
1 _
2 _
3 _


_
_

1 2
b) ; 53; 285,9 _

e) 2 _ ; 3 3 ; 15,8


_
_
_
_
_

2 1 2 3 4 5

c) ; 26; 168,7
1 _ 3
_
_
_
_
_

_
5 4 3 2 1 f) ; 2 2 ; 142,2
_
_

3. a) 53; 74,05 b) 34; 210,96 c) 85; 282,5


d) 41 ; 141,34 e) 6,26; 86,95 f) 3,0414; 189,5

4. a) 6 + 2j b) 8 + 9j
9

3_
2_ u
2u

6
v
1_

u+
3v

u
_
_
_
_
_
_

1 _ 1 2 3 4 5 6 3
v -v
-v u
- v
| | | | | | | | | | | | | | | | |
12 8 4 - v

424 Vectores

422-431 424 25/11/02, 12:01 PM


CAPTULO 10

c) 9 d) 27 + 3j
3_
2_
u
1_ u+ v
u+v+ w
_
_
_
_
_
_
u 5p
_ v 9
-1
w 5 p -p -p -p -p -p 25 27
-2 _
-3 _

e) 6 + 27j f) 12j

5. a) 2 + 5J d) 5 10. a) 8,6; 125,50 d) 3 5; 206,57


b) 3 2J e) 7J b) 3,6; 213,69 e) 5; 180
c) 8 3J f) 3J c) 12,81; 38,66 f) 3, 90

6. a) ( 2 5, 5
5 5 ) d) ( 3 , 4
5 5 ) 11. a) 5,39; 21,80
b) 6,08; 80,54
d) 6,40; 321,34
e) 2; 0
b) ( 6 , 1
37 37 ) e)
( 10 , 4 10
10 5 ) c) 2; 45 f) 4; 270

12. a) (1,1) g) (0, 25)


c) ( 5 , 3
34 34 ) f) (1,0)

7. a)
3
+
5
j d) 4

1
j
b) ( 5 3, 5
2 2 ) h) (2 2, 2 2)

34 34 17 17

b)
1
+
4
j e)
c) ( 1 , 3
2 6 ) i) (6,78; 4,24)

17 17 d) (9, 78; 2,08) j) (0, 3)


1 3
c)
10

10
j f) j
e) ( 12 , 12
2 2 ) k) (9 2, 9 2)

8. a) ( 3 , 1
10 10 ) d) ( 1 , 3
10 10 ) f) (42,26; 90,63) l) (5, 0)

13. 20
b) (
9 , 3
10 10 ) e) ( 2 , 2
2 2 ) 14. f1 = 15 3 f2 = 15

c) ( 5 , 8
93 93 ) f) (0,1)
15. El avin se desplaza a 852,47 km/hr en
direccin 147,98 NO.

5 2 16. 32,98 km/hr; 104 con la orilla.


9. a) 3 j d) +
1
j
34 34 6 3
2 1 2 3
b) + j e) j
5 5 13 13
c) J f)

Vectores 425

422-431 425 25/11/02, 12:01 PM


Producto Punto (o producto escalar) 10.5

10.5.1. Denicin
Sean u = (u1, u2) = u1 + u2J y
v = (v1, v2) = v1 + v2J vectores.

Se llama PRODUCTO PUNTO entre los vectores u y v al


escalar que se obtiene de la siguiente forma:

u v = u1v1 + u2v2

Ejemplo: u = (3, 2) = 3 + 2J
v = (5, 6) = 5 6J

u v = 3 5 + 2 (6) = 15 12 = 3

Nota: El producto punto se llama tambin producto escalar (no


confundir con el producto por escalar) debido a que su resultado
es un escalar.

10.5.2. Propiedades
Sean u, v y w vectores y k un escalar en R, entonces:

1. (u + v) w = u w+v w
2. u v=v u
3. (ku) v = u (kv) = k(u v)
4. u u= ||u||2 > 0; u u=0u=0

10.5.3 Angulo entre vectores

Como sabemos, cualquier vector se puede


representar por un segmento dirigido con origen en
el origen del sistema cartesiano.
Llamamos q al ngulo formado por los vectores v
u y v . El ngulo 0 q ser cero si ambos
vectores tienen igual direccin y sentido y ser q u
si ambos vectores tienen igual direccin y sentidos
opuestos.

426 Vectores

422-431 426 25/11/02, 12:01 PM


CAPTULO 10

El ngulo q formado por los vectores u y v, ambos distintos de


cero, se obtiene a travs de la expresin:

uv
cos q = Ver ejercicio resuelto n2
|| u || || v ||

Dos vectores se dicen PARALELOS si estn contenidos en la


misma recta que pasa por el origen. Si u y v son vectores paralelos,
entonces existe k R tal que u = k v

Dos vectores se dicen ORTOGONALES si estn contenidos en


rectas perpendiculares que pasan por el origen. Dos vectores u y
v son ortogonales si y slo si u v = 0.

Ejemplo:

6_
_
_ Sean u = (4, 2) = 4 2J
_ v v = (3, 6) = 3 + 6J
_
_
_
_
_
_

_ 3
u u v = 12 + 12 = 0
2 _

10.5.4 Proyeccin de un vector sobre otro

Sean u y v dos vectores distintos de


cero. Se llama:
proyeccin vectorial de u sobre
v al vector:
uv
pr(u , v) = v
|| v ||2

u
) u
pr( v ,

proyeccin vectorial de v sobre


, v) v u al vector:
pr( u
vu
pr(v , u) = u
|| u ||2

Ver ejercicio n4

Vectores 427

422-431 427 25/11/02, 12:01 PM


Ejercicios 1. Dados los vectores u = 3 + 2J , v = + 5J y w = 6 3J.
resueltos Hallar (u + v) w

Solucin:
Primero sumamos u + v = 3 + 2J + 5J = 2 + 7J
Ahora hacemos (u + v) w = (2 + 7J) (6 3J)
= 2 6 + 7(3) = 9

2. Determinar el ngulo formado por los vectores u = (2, 5) y


v = (3, 2)
5_
_
_ Solucin:
_ u Sabemos que el coseno del
ngulo entre dos vectores
_
est dado por:
_
_
_

_
_
_

3 2

uv 2 3 + 5 2 4
cos = = = 0,2060
|| u || || v || 29 13 377
= 78,1

3. Hallar el valor de m para que los vectores u = m + 5J y


v = 4 (1 + m)J sean ortogonales.
Solucin:
Para que dos vectores sean ortogonales, el ngulo q formado entre
ellos debe ser de 90; por lo tanto, cos q = cos 90 = 0

uv
Sabemos que cos =
|| u || || v ||
Para que este valor sea cero, u v debe ser cero,
u v = 4m 5(1 + m) = 0
4m 5 5m = 0
5 m = 5
m = 5

4. Determinar la proyeccin de u sobre v si u = 3 + 5J y


v = 6 + 2J.
Solucin: Sabemos que pr(u, v) = u v2 v
|| v ||
36+52
Es decir, pr(u, v) = (6 + 2J)
40
28 21 7
= 40 (6 + 2J) = + .
5 5 J

428 Vectores

422-431 428 25/11/02, 12:01 PM


CAPTULO 10

Grficamente:

5_
4_
3_
u
2_
1_ )
_ pr(u,v v
_
_
_
_
_
_
1 2 3 4 5 6

5. Determinar el ngulo obtuso del tringulo ABC sabiendo que


A = (3, 1), B = (6, 6) y C = (2 ,2).

Solucin:
Tenemos que: OA + AB = OB
AB = OB OA
6_ B AB = (6, 6) (3, 1) = (3, 5)
5 _
y OA + AC = OC
4_
3_ AC = OC OA
C 2_ u AC = (2, 2) (3, 1) = (5, 1)
v 1_ A

_
_
_
_
_

_
_
_
_
_
_

5 4 3 2 1 1 2 3 4 5 6

Si llamamos u = AB = (3, 5) y v = AC = (5, 1) el problema se


reduce a calcular el ngulo entre los vectores u y v.

uv 15 + 5 10
Luego cos = = = 0,3363
|| u || || v || 34 26 2 221
_1
As = cos (0,3363) = 109,65

Ejercicios

1. Dados los vectores u = (3, 1), v = (5, 2) y w = (1, 6). Calcule.


a) u v d) (u + v) w g) 3u 2v
b) u w e) 3u v h) u u
c) v w f) u 3v i) w w

Vectores 429

422-431 429 25/11/02, 12:01 PM


Ejercicios
2. Encuentre el ngulo formado por cada uno de los siguientes pares de vectores.
a) u = 2 + J v = + 2J
b) u = 3 J v = 5 + J
c) u = 2J v = 3J
d) u = 4 + J v = + 5J
e) u = 3 + 5J v = 5 + 3J

3. Determine si los siguientes pares de vectores son paralelos. Si lo son, determine


si tienen igual o distinto sentido.
a) u = 3 + 2J v = 4,5 + 3J
b) u = + 5J v = 5J
c) u = 12 6J v = 16 + 8J
d) u = 2 + 2J v = 2 + 2J
e) u = 2 4J v = 3 + 6J
f) u = 5 + J v = 6 + 2J

4. Determine cules de los siguientes pares de vectores son ortogonales y


cules no lo son.
a) u = 2 3J v = 4 + J
b) u = 6 J v = 3 18J
c) u = 6 J v = 3 18J
d) u = 4 + 2J v = J
e) u = 5 v = 8J
f) u = 6 v = 12

5. Encuentre el valor de m para que los siguientes pares de vectores sean


ortogonales.
a) u = 3 mJ v = + 2J
b) u = m + 2J v = 2 3J

c) u = 2 + J
3
v = m + ( 1 + 3
2 )J

d) u = 1 + v = mJ
2 4 J
1 1
e) u = J v = (m + 1) J
2 2
f) u = 2m J v = 4 + 6mJ

6. Encuentre el o los valores de m para que los siguientes pares de vectores sean
ortogonales. Analice la pertinencia de las soluciones.
a) u = 2m 3J v = m (m + 1)J
b) u = 4 2mJ v = 5 mJ
c) u = (3m 1) 2 v = m + 6J
d) u = m 3mJ v = 6 +(2m 1)J
e) u = + J v = m 2mJ
f) u = 5 mJ v = 3 + (m + 2)J

430 Vectores

422-431 430 25/11/02, 12:02 PM


CAPTULO 10

7. Demuestre que u v=v u

8. Determine la proyeccin del vector u sobre el vector v y la proyeccin del


vector v sobre el vector u. Grafique.
a) u = 5 + j v = 6j
b) u = 5 + 2j v = + 8j
c) u = + 3j v = 6 + 5j
3
d) u = + 9j v = 2 3j
4
e) u = 6 v = 3 5j

9. Dado el vector u = 5 + 2j , encuentre su proyeccin sobre el eje x y sobre


el eje y.

10. Dado el vector u = 3 + 5j


a) Encuentre u
b) Encuentre u j
c) Compare estas soluciones con las del ejercicio anterior. Establezca alguna
conclusin.

11. Pruebe que si u = u1 + u2j , entonces:


i) u es la proyeccin de u sobre el eje x.
ii) u j es la proyeccin de u sobre el eje y.

12. Encuentre un vector cualquiera que sea ortogonal al vector dado:


a) u = (3, 6)
b) u = (1, 15)
c) u = (2, 3)
d) u = (5, 1)

e) u = ( )
1 1
,
2 3
f) u = ( 2, 3)
g) u = (2,5; 3)
h) u = (0,5; 3,5)

13. Un vector de R2 es un par ordenado u = (u1, u2). Generalizando, un vector


de Rn es una n-upla ordenada.
u = (u1, u2, u3, ..un)
Pruebe que en Rn: u v = v u

14. Sean u = (3 3,3), v = (3, 3 3) y w = (3, 3 3). Determine el ngulo formado


por u y v, el ngulo formado por u y w y el ngulo formado por v y w.

Vectores 431

422-431 431 25/11/02, 12:02 PM


Ejercicios
15. Determine el ngulo obtuso del tringulo formado por los puntos A=(2, 2),
B=(5, 1) y C=(1, 5)

16. Se tienen los puntos A=(2, 1), B=(0, 3) y C=(6, 0). Determine el ngulo obtuso
del tringulo ABC.

17. Sea ABC el tringulo formado por los vrtices A=(4, 2), B=(1, 4) y C=(3, 6).
Determine la medida de los segmentos en que hc divide al lado AB.

18. Sea ABC el tringulo formado por los vrtices A=(1, 1), B=(4, 2) y C=(4, 3).
Calcule medida de los segmentos en que la altura hb divide al lado AC.

19. Sea ABC el tringulo formado por los vrtices A=(1, 3), B=(4, 2) y C=(8, 3).
Determine la medida de los segmentos que ha genera en el lado BC.

20. Encuentre ambas alturas del paralelogramo formado por los vectores u = (6, 2)
y v = (3, 4).

21. Sea ABCD el paralelogramo cuyos vrtices son A=(0, 0), B=(6, 4), C=(9, 2),
D=(3, 2). Determine su rea.

Soluciones

1. a) 13 b) -3 c) 17 d)14 e) 39 f) 39 g) 78 h) 10 i) 37

2. a) 90 b) 172,9 c) 45 d) 64,65 e) 90

3. a) s, igual. b) s, distinto. c) s, distinto. d) no. e) no. f) no.

4. a) no. b) no. c) s. d) no. e) s. f) no.

3 1 3
5. a) b) 3 c) d) 2 e) 2 f) 0.
2 4 4 3

6. a) No existe. b) No existe.c) 1 73
6
d) 1 ; si m = 0, u = 0 y no podemos hablar de ortogonalidad.
2
e) No existe. f) 3, 5.

432 Vectores

432-442 432 20/11/02, 6:06 PM


CAPTULO 10

7. u v = (u1, u2) (v1, v2) = u1v1 + u2v2 = v1u1 + v2u2 = (v1, v2) (u1, u2) = v u.

8. a) J b) 0,3 + 2,6J c) 1,08 + 0,9J d) 3,33 + 7,27J e)1,59 2,65J .

9. 5 ; 2J .

10. a) 3 b) 5J .

11. a) Representemos el eje x por el vector v = (x, 0) = x, pr(u,v) = u1 x+ u2 0 x = u1


que es la proyeccin del vector u sobre el eje x. x2
b) Idem considerando al eje y como el vector v = (0, y).

6 12
12. a) (2, 1) b) (15, 1) c) (6, 4) d) (2, 10) e) (2, 3) f) ( , 1) g) ( , 2) h) (7, 1)
2 15
13. Ver respuesta ejercicio 7. 14. 30, 90, 60.
15. 126,87 16. 120,9.
17. 1,42; 6,39. 18. 5,6; 0,6.
19. 0,78; 5,62. 20. 2,85; 3,6.
21. 24.

3
Vectores en el espacio IR 10.6

10.6.1 Deniciones
Un vector de R 3 es un segmento dirigido en el espacio
tridimensional. Anlogamente a lo enunciado en el punto 10.1,
todos los vectores de R3 pueden considerarse con origen en
el origen del sistema cartesiano y extremo en cualquier punto
del espacio.
Z
7

5 Z
4
5
3
4
u2
4

3
1
3

2
2

1 Y
1 u
1

2 1 2 3 4
3 Y
4 1 2 3 4 5 6 7
5 X

u = (5, 2, 7) u = (3, 5, 1)
X

Vectores 433

432-442 433 20/11/02, 6:06 PM


Llamamos:
= (1, 0, 0) al vector unitario en la direccin x.
J = (0, 1, 0) al vector unitario en la direccin y.
k= (0, 0, 1) al vector unitario en la direccin z.
Cualquier vector de R3 se puede escribir como combinacin
lineal de los vectores , J y k.
Las operaciones vectoriales mencionadas en el presente captulo
se extienden todas a R3. Ver ejercicios resueltos.
Adems:
=1 J = 0
J J = 1 k = 0
k k = 1 J k = 0

10.6.2. Producto vectorial o producto cruz


En forma especial se define el producto vectorial o producto cruz para vectores
de R3 de la siguiente forma:

Sean: u = u1 + u2J + u3k y


v = v1 + v2J + v3k vectores de R3

entonces el producto vectorial entre u y v es el vector:

w = u x v = (u2v3 u3v2) (u3v1 u1v3)J + (u1v2 u2v1)k

Obsrvese que los coeficientes de , J y k son el desarrollo de determinantes


2 x 2 (ver cap. 9.5), por lo tanto:

u2 u3 u1 u3 u1 u2
| | |
u x v = v v v v J + v v k
2 3 1 3 1 2
| | | o

J k
|
u x v = u1 u2 u3
v1 v2 v3
|
Observaciones:
1. El vector w = u x v es ortogonal al vector u y al vector v.
2. El valor absoluto del producto mixto u v x w representa el volumen del
paraleleppedo generado por los tres vectores.
u1 u2 u3
3. El valor de u v x w se encuentra al calcular el determinante v1 v2 v3 , cuyo
w 1 w2 w3
valor absoluto corresponde al volumen del paraleleppedo formado por los vectores
u, v y w.
4. Sean u y v dos vectores de R3 que forman un ngulo , entonces,
||u x v|| = ||u|| ||v|| sen . ( Ver ejercicio resuelto n 2).

434 Vectores

432-442 434 20/11/02, 6:07 PM


CAPTULO 10

Ejercicios
1. Dados u = (3, 2, 5) y v = (3 ,1, 0), encuentre u x v y v x u.
resueltos
Solucin:
J k
| |
u x v = 3 2 5 = 5 15J + 9k
3 1 0

J k
| |
v x u = 3 1 0 = 5 + 15J 9k
3 2 5

Observamos que el vector u x v = (v x u), es decir, al conmutar


dos vectores en un producto cruz se generan vectores en
sentidos opuestos y con igual magnitud.

2. Dados los vectores u = (3, 1, 5) y v = (4, 2 ,3).


a) Calcular el ngulo formado por ellos.
b) Calcular ||u|| ||v|| sen .
c) Calcular ||u x v||.
d) Comparar los valores de b) y c).

Solucin:
uv
a) Sabemos que cos = .
|| u || || v ||
3 4 + 1 2 + 5 3 1
cos = = 0,0314
9 + 1 + 25 16 + 4 + 9 1.015

de donde = 91,7987.

b) ||u|| ||v|| sen = 35 29 sen 91,7987 = 31,843.

j k
c) u x v =
| |
3 1 5 = (3 10) (9 20)J + (6 4)k
4 2 3

u x v = (13, 29, 2) y

||u x v|| = 169 + 841 + 4 = 1.014 31,843

d) Los resultados de b) y c) son iguales. Con ello hemos dado


un ejemplo de que ||u x v|| = ||u|| ||v|| sen , donde es
el ngulo formado por los vectores u y v.

Vectores 435

432-442 435 20/11/02, 6:07 PM


3. Determinar el volumen del paraleleppedo formado por los
vectores u, v y u x v en R3 si u = (2a, 0, 0), v = (a, a 3, 0).

Solucin: Veremos dos formas.

a)

Calculemos u x v = w .
w

J k
| |

0 a 3
w= 2a 0 0 = 0 0J + 2 3a2k

a a 3 0
u v

2a
w = (0, 0, 2 3 a2)
B
A
Se trata entonces de un paraleleppedo
C cuya base es el paralelogramo ACBO y
su altura es ||w|| = OD

El rea de las bases OA BA (base altura), donde


OA = ||u|| y BA = ||v|| sen .

a 3
Sen = BA = a 3
|| v ||

||u|| = 4a2 = 2a AACBO = 2a x a 3 = 2a2 3.

El volumen del paraleleppedo es:

V = AACBD OD = 2a2 3 2 3a2 = 12a4

b) Otra forma.
Sabemos que el volumen del paraleleppedo formado por los
vectores u, v y w es el valor absoluto del clculo de u v x w; y

| |
2a 0 0
uvxw= a a 3 0 = 2a a 3 2a2 3 = 12a4
0 0 2a2 3

Luego el volumen del paraleleppedo es 12a4.

436 Vectores

432-442 436 20/11/02, 6:07 PM


CAPTULO 10

Ejercicios
1. Grafique los siguientes vectores:
a) u = 3 + 2J + 5k
b) u = 5 2J + k
c) u = + 6J 3k
d) u = 2 3J 5k
e) u = 3 + 2J 4k
f) u = 3 6J + 5k
2. Dados los vectores u = (3, 2, 1) y v = (5, 4, 2), calcule u x v y v x u. Compare
ambos resultados.
3. Dadas las siguientes parejas de vectores en R3, calcule u x v y v x u en cada
caso y grafquelos.
a) u = 2 + 3j + 0k v = 4 + J + 0k
b) u = + 2J + 0k v = 5 3J + 0k
c) u = 3 + 5J + 0k v = 2 3J + 0k
4. Dadas las siguientes parejas de vectores de R3, calcule u x v y v x u en cada
caso y grafquelos.
a) u = 2 + 3k v= 4 5k
b) u = + 2k v = + 2k
c) u = 5 k v = 5 + 2k
5. Dadas las siguientes parejas de vectores de R3, calcule u x v y v x u en cada
caso y grafquelos.
a) u = 2J 3k v = J + 4k
b) u = 3J 6k v = 2J + 2k
c) u = 3J +k v = 4J 4k
6. Calcule un vector normal a los vectores u = (2, 2, 1) y v = (1, 1, 5)
7. Calcule m para que el vector w = (5, m+1, 2) sea normal a los vectores
u = (5, 3, 1) y v = (4, 2, 1)
8. Sean u = (5, 2, 0) y v = (2, 5, 0) dos vectores :
a) Calcule w = u x v
b) Calcule u v x w
c) Calcule v u x w
d) Grafique el paraleleppedo generado por los vectores u, v y w.
e) Calcule el volumen de dicho paraleleppedo.
9. Sean u = (0, 3, 4) y v = (0, 4, 3) dos vectores
a) Calcule w 1 = u x v
b) Calcule w 2 = v x u
c) Grafique el paraleleppedo formado por los vectores u, v y w en ambos casos.
d) Calcule u v x w en ambos casos.
e) Calcule el volumen de los paraleleppedos construidos en c.

Vectores 437

432-442 437 20/11/02, 6:07 PM


10. Sean u = 3 + k y v = 2 + 6k dos vectores
a) Calcule w 1 = u x v
b) Calcule w 2 = v x u
c) Grafique los paraleleppedos formados por los vectores u, v y w 1 y u, v y w 2.
d) Calcule u v x w 1 y v u x w 1
e) Calcule u v x w 2 y v u x w 2
f) Calcular el volumen de los paraleleppedos graficados en c.
11. Sean u = (1, 2, 3) y v = (5, 1, 2). Calcule:
a) w 1 = u x v
b) w 2 = v x u
c) Calcule el volumen de los paraleleppedos generados por los vectores
u, v y w 1 y u, v y w 2.

12. Dados los puntos A(3, 2, 1) y B(6, 4, 5). Calcule las coordenadas del vector AB .

13. Las componentes del vector AB son (3, 0, 4) y las coordenadas del punto B son
(0, 3, 1). Halle las coordenadas del punto A.

14. Calcule el ngulo formado por los vectores u = (4, 3, 1) y el eje vertical
representado por el vector w = (0, 0, 2).

15. Calcule el ngulo formado por los vectores u = 5 + 2J 3k y v = 2J + k.

Soluciones

2. u x v = (0, 1, 2); v x u = (0, 1, 2); u x v = (v x u)


3. a) 10k; 10k b) 13k; 13k c) 19k; 19k
4. a) 22J , 22J b) 2J ; 2J c) 5J ; 5J
5. a) 11 , 11 b) 18 ; 18 c) 8 ; 8
6. (11,11,0)
7. m = 2.
8. a) w = 29k b) 841 c) 841 e) 841
9. a) 21 b) 21 d) 525 e) Ambos tienen volumen 525
10. a) 20J b) 20J d) 400; 400 e) 400; 400.
f) Ambos tienen volumen 400.
11. a) (7, 13, 11) b) (7, 13, 11) c) 369
12. (3, 2, 4)
13. (3, 3, 3)
14. 78,69
15. 97,61

438 Vectores

432-442 438 20/11/02, 6:07 PM


CAPTULO 10

Prueba de seleccin mltiple


1. Si u = (2, 1) y v = (3, 2), 5. Los vectores u = (1, 3, 2) C. ||u|| cos a
entonces u + v es: y v = (1, 3, 2) difieren
en: D. ||u|| sen a
A. (2, 3) E. u u
A. Slo magnitud.
B. (6, 2)
B. Slo sentido. 9. El vector unitario en
C. (5, 3) la direccin y sentido
C. Slo direccin. del vector u se expresa
D. (2, 3) por:
D. Direccin y sentido.
E. (2, 1) A. u = ||u|| u
E. Magnitud y sentido.
2. Si u = (1, 0) y v = (3, 1) u
B. u =
entonces 2u 3v es: ||u||
6. Si u = (1, 1, 1), entonces
A. (10, 3) ||u|| vale: C. u = (1, 1)

B. (10, 3) A. 0 D. u = ||u||

C. (10, 3) B. 1 1
E. u =
||u||
D. (11, 3) C. 2
10. El vector unitario del
E. (11, 3) D. 2 vector u = (12,5) es:
12 5
3. El valor de k para que E. 3 A. ( , )
u = (k, 1, 1) y 13 13
7. Si u = (3, 1), entonces 5 12
v = (1, 1) sean iguales es: B. ( , )
el vector unitario en la
13 13
A. 0 direccin de u es:
5 12
C. ( , )
B. 1 A. u = (3, 1) 13 13
12 5
C. 2 3 1 D. ( , )
B. u = ( , ,) 13 13
D. 2 10 10
5 12
3 1 E. ( , )
C. u = ( , ) 13 13
E. 1 10 10
3 1 11. Si u = ( 3, 1), el ngulo
4. La longitud del vector D. u = ( , ) que forma con el eje x, es
u = (0, 1, 1) es: 10 10
decir, su direccin, es:
3 1
A. 1 E. u = ( , ,)
10 10 A. 60
B. 2 8. Si el vector u tiene B. 30
direccin a, entonces
C. 2 C. 45
la componente vertical
D. 3 de u es:
D. 120
E. 3 A. u cos a
E. 150
B. u sen a

Vectores 439

432-442 439 20/11/02, 6:07 PM


Prueba de seleccin mltiple
12. Sea u = m + 2J. El valor 18. De los vectores
de m para que u sea B u = (2, 1); v = (1, 2);
unitario es: A
w = (4, 2) y o = (4,2)
son ortogonales:
1 60 30
A.
2 A. w y o

B. 1 A. El sistema se des- B. u y w
2 plaza hacia el lado
1 C. v y o
C. de A.
4 D. u y v
B. El sistema se des-
D. 1 plaza hacia el lado E. u y o
4 de B.
E. No existe. 19. Si u = 2 + 5J; v = 2J y
C. El sistema est en
w = 3 J, entonces
equilibrio.
1 u + v w es:
13. Si u = + 3 J, el
m 2 D. No se puede deci-
A. 3
valor de m para que u dir.
sea unitario es: B. 3
E. En el desplazamiento
A. 1 slo interviene la incli- C. 6
B. 0 nacin del plano.
D. 6
C. 2 16. Si la magnitud del vector
E. 9
u es 2 y su direccin
D. 1 es 225, entonces sus
2 20. El valor de m para que
componentes son:
u = mJ y v = (1 m)
E. 1
3 A. (1, 1) + 2J sean ortogonales
es:

14. Si u = 2 5J y v = 3 B. (1, 1) A. 0
+ J, la norma del vector
u v es: C. (1, 1) B. 1

A. 11 C. 1
D. (1, 1)
B. 25 1
D.
E. (0, 2) 3
C. 36 1
17. Si u = (3, 2) y v = (5, 1), E.
3
D. 61 entonces u v es:
E. 1 21. Si u = 2 + 4J y v = 5
A. 0
+ 3J, entonces el vector
15. En la figura, el cuerpo A B. 3 pr (u , v) es:
pesa 15 kg y B pesa 10
C. 5 5 3
kg. Prescindiendo de la A. + J
fuerza de roce podemos 17 17
D. 12 2 4
decir que: B. + J
E. 15 17 17

440 Vectores

432-442 440 20/11/02, 6:08 PM


CAPTULO 10

55 33 B. (1, 2, 1) 28. Sean A = (2, 5, 1) y


C. + J
17 17 B =(3, 2, 1) dos puntos,
C. (1, 2 ,1)
33 55 entonces las coordena-
D. + J D. (1, 2, 1)
17 17 das del vector AB son:
E. (1, 2 ,1)
33 55 A. (5, 3, 1)
E. + J
17 17 25. Si u = 2 3J + k y
B. (5, 3, 0)
v = + J k entonces
22. Si u = 2 + 4J y v = 5
u x v es: C. (5, 3, 1)
+ 3J, entonces el vector
pr(v, u) es: A. (2, 5, 3) D. (5, 3, 0)
3 B. (2, 5, 3) E. (5, 3, 0)
A. + J
5
C. (2, 3, 5)
29. Los componentes del
2 4
B. + J D. (2, 3, 5) vector AB son (5, 2, 1)
5 5
y las coordenadas del
22 11 E. (2, 3, 5) punto A son (0, 3, 2). Las
C. + J coordenadas del punto B
5 5
26. El vector normal a los son:
11 22 vectores u = (2, 1, 1) y
D. + J v = (1, 1, 3) es: A. (5, 5, 1)
5 5

11 22 A. 4 5J + 3k B. (5, 1, 3)
E. + J
5 5 B. 4 5J 3k C. (5, 5, 1)

23. El vector (3, 4, 2) es C. 4 + 5J 3k D. (5, 1, 3)


igual a:
D. 4 5J + 3k E. (5, 5, 1)
A. 3(1,0,0) 4(0,1,0)
E. 4 + 5J + 3k 30. El ngulo formado por los
2(0,0,1)
vectores u = (1, 1, 1) y
27. Son normales al vector v = (1, 2, 1) es:
B. 3(1,0,0) + 4(0,1,0) +
u = 3 J + k los
2(0,0,1)
vectores: A. 30
C. 3(1,0,0) 4(0,1,0) I) 3 J 10k B. 45
2(0,0,1)
II) 2 3J + 3k C. 60
D. 3(1,0,0) + 4(0,1,0)
2(0,0,1) III) 2 + 3J 3k D. 90

A. Slo I E. 180
E. 3(1,0,0) + 4(0,1,0)
+ 2(0,0,1) B. Slo II

24. Si u = (1, 0, 1) y v = (2, C. Slo III


1, 0) entonces u x v es:
D. Slo I y II
A. (1, 2, 1)
E. I, II y III

Vectores 441

432-442 441 20/11/02, 6:08 PM


Soluciones

1) C 11) B 21) C

2) D 12) E 22) D

3) C 13) C 23) D

4) C 14) D 24) E

5) B 15) B 25) C

6) E 16) D 26) B

7) C 17) D 27) E

8) D 18) D 28) E

9) B 19) C 29) A

10) D 20) D 30) D

442 Vectores

432-442 442 20/11/02, 6:08 PM


CAPTULO 11
M
atrices
y determinantes

Conceptos bsicos 11.1

Definicin: Se denomina MATRIZ a una ordenacin rectangular de


elementos. Estos elementos sern en general nmeros reales.

En esta ordenacin rectangular distinguimos las FILAS de la


matriz, que son las lneas de elementos ordenados horizontalmente,
y las COLUMNAS de la matriz, que son las lneas de elementos
ordenados verticalmente.
Si la matriz tiene m filas y n columnas se dice que es de orden
m n o su dimensin es m n.
Cada elemento de la matriz est entonces identificado por
la posicin que ocupa, esto es, est en la interseccin de una
fila y una columna.
Denotamos la matriz A de dimensin m n por A = [aij] m n,
donde cada aij representa el elemento situado en la fila i y en la
columna j. Es claro que 1 i m
1jn
Si la matriz consta de slo 1 fila se llama matriz fila o vector
fila. Si consta de una sola columna, se llama matriz columna
o vector columna.
EJEMPLOS:
2 1 3
1. Sea A = 1 1 2
es una matriz

de orden 2 3 (2 filas y 3 columnas)

2. A = [1 2 2 3] es una matriz fila de orden 1 4.

Matrices y determinantes 443

443-445(2003) 443 20/11/02, 6:19 PM


Ejercicios
junio julio agos. sept.
1. Dada la siguiente tabla de notas:
Detergente $448 $452 $452 $490
P1 P2 P3 P4
Colonia $820 $779 $790 $800
Altamira 6.1 6.5 4.9 3.1
Lavaloza $375 $375 $390 $410
Contreras 2.3 3.9 4.9 6.4
Limpiavid. $440 $430 $450 $460
Fernndez 5.1 2.8 3.1 4.8
Lustramueb. $400 $410 $430 $470
Martnez 2.6 3.4 3.8 5.3
Valds 6.3 6.6 3.7 6.9 a) Precio de lavaloza en los meses
Zamora 4.6 4.8 5.3 6.6 de agosto y septiembre.
b) Diferencia de precios de limpia-
Responda las siguientes preguntas: vidrio entre junio y septiembre.
a) Qu nota obtuvo el 1er alumno c) Porcentaje de la colonia entre
en la 3a prueba? junio y julio.
d) Diferencia de precios entre lava-
b) Qu nota obtuvo el 3er alumno loza y detergente en el mes de
en la 1a prueba? agosto.
c) Cul es la mejor nota del 4 1 2 1 3
alumno? 5. Dada la matriz: 4 1 2 0
M= 3 2 2 4
d) Quin y en qu prueba sac la
2 1 3 6
nota ms baja?
5 2 1 1
e) Quin y en qu prueba sac la
nota ms alta? a) Determine el orden de M.
b) Escriba los elementos m12; m53;
2. Determine el orden de las siguientes m24; m33.
matrices: c) Escriba los elementos de la 5a
fila.
2 3 2 3 1 d) Escriba los elementos de la 3a
a) 1 0 1
b) 1 3 columna.
4 4
e) Calcule 2 m34 3 m43
1 2 1 1 1 1 3 f) Calcule m11 + m22 + m33 + m44
c) 4 1 0 d) 1 1 2 1
1 2 0
2 3 1 3 6. Escriba con los siguientes elementos:
2 3 5
2
3, 2, 5, 0, 3, 1, 9, 12, 3, 5, 1, 0,
e) 101 f) 1 (se supone que estn ledos por filas)
una matriz cuyo orden es:
3. Determine el nmero de elementos de a) 2 6 b) 3 4 c) 4 3 d) 6 2
una matriz si su orden es:
7. Escriba una matriz M de orden 3 3 que
a) 2 5
cumpla las siguientes condiciones:
b) 2 2 a) los elementos de la 2a columna
c) 3 4 son 1, 2, 3
d) 6 3 b) los elementos de la 3a fila son 2, 3, 1
c) a11 = a22 + a33
e) m n d) a13 = a31
f) n p e) a12 = a21
4. Dada la siguiente tabla, seale: f) a23 = a21 + a22

444 Matrices y determinantes

443-445(2003) 444 20/11/02, 6:20 PM


CAPTULO 11

8. Determine en cada caso una matriz 9. Construya una matriz M (3 3) de


que cumpla la condicin indicada. modo que cumpla las siguientes
condiciones:
a) A [3 4] ai = 0 si i = j
a) a22 = 0 d) a32 = a13
b) A [4 3] aij = 1 si i > j
b) a23 = 2 e) a12 = a33 = a32
c) A [2 3] aij = i + j
c) a32 = a21 = 1 f) a11 = a31 = a23
d) A [3 3] aij = i j2
e) A [4 3] aij = i + j 1

Soluciones
3 2 5 0
3 2 5 0 3 1
1. a) 4.9 b) 5.1 c) 5.3 6. a) b) 3 1 9 12
9 12 3 5 1 0
3 5 1 0
d) 2 alumno; 1a prueba.
e) 5 alumno, 4a prueba.
3 2
2. a) 2 3 b) 3 2 c) 4 3 3 2 5 5 0
d) 3 4 e) 1 3 f) 2 1 c) 0 3 1 d) 3 1
9 12 3 9 12
3. a) 10 b) 4 c) 12 5 1 0 3 5
1 0
d) 18 e) m n f) n p
7. 8.
4. a) $ 390 y $ 410 b) $ 20
c) El precio disminuy un 5% d) $ 62
2 1 1
5. a) 5 4 b) 2; 1; 0; 2. c) 5 2 1 1 9. 1 0 2
d) 1 2 2 3 1 e) 1 f) 4 2 1 1

Igualdad y adicin 111.2


de matrices
11.2.1 Matrices iguales
Dos matrices son iguales si se cumple:

a) Tienen el mismo orden, es decir, el mismo nmero de filas


y el mismo nmero de columnas.
b) Los elementos correspondientes a igual fila e igual columna
son iguales.

11.2.2 Adicin de matrices


Sean A y B dos matrices de orden m n.
Definimos la ADICIN de A y B por:

Matrices y determinantes 445

443-445(2003) 445 20/11/02, 6:20 PM


A + B = C si y slo si
aij + bij = cij donde 1 i m
1 j n.
Observacin:
1. La matriz resultante C tambin es de orden m n.
2. No es posible sumar dos matrices que difieran ya sea
en el nmero de filas, en el nmero de columnas o
en ambos.

11.2.3 Propiedades de
la adicin
Sean A, B y C matrices de orden m n. (escribimos
A E M m n, B E M m n,), entonces:

1. La matriz A + B tambin es de orden m n, es decir:


A E Mm n y B E Mm n, entonces A + B E Mm n.
sta es la propiedad de la CLAUSURA.
2. A + B = B + A
La adicin de matrices es conmutativa.
3. (A + B) + C = A + (B + C)
La adicin de matrices es asociativa.
4. Existe la matriz nula de orden m n tal que:
A+0=A
donde la matriz 0 = [aij] con aij = 0 Ii, Ij
5. Existe la matriz inversa
I A E Mm n, H ( A) E Mm n A + ( A) = 0
El conjunto de las matrices con la operacin ADICIN
recin definida y las propiedades que posee tiene estructura
de GRUPO ABELIANO O GRUPO CONMUTATIVO.

Ejercicios
1. Determinemos el valor de las incgnitas si A = B siendo:
resueltos 2y a 2
A= y B= b 5
3 x
Aplicando directamente la definicin de igualdad de
matrices tenemos que:
a=2 y=2 b = 3 x=5
2. Determinemos el valor de x e y si A = B y
y
A = [2 x] B =
3
En este caso A tiene orden 1 2 y B tiene orden 2 1; por lo
tanto, NO puede haber igualdad.
3. Determinemos los valores de las incgnitas si A = B siendo

446 Matrices y determinantes

446-447 446 10/11/2001, 13:37


CAPTULO 11

u+v 2 x 4 u v 3
A = w 4 B =
3 y 1 4

Aqu tenemos u + v = 4
u v = 2 } Q u=3 ; v=1
x = 3
y = 3
w = 1

4. Obtengamos la suma de A y B siendo:


1 3 0 2
A = 1 2 y B = 2 5
4 2 1 0
Ambas tienen el mismo orden; por lo tanto, se pueden sumar.
Sumamos los elementos correspondientes y obtenemos:
1+0 3+2 1 1
A + B = 1+2 2+5 = 3 7
4+1 2+0 5 2
5. Aplicando las propiedades de la adicin en las matrices
podemos resolver ecuaciones matriciales.
Resolvamos A + X = B siendo
3 4
A = 2 2 y B =
1 8 12 1

Si A + X = B, entonces sumando la matriz opuesta de A, que


se denota ( A) a ambos lados, obtenemos:
X = B + ( A)
para obtener (A) cambiamos el signo de cada elemento de A. As:

3 4 2 2
X = +
12 1 1 8

X = 5 6
y efectivamente, sumando A y X se obtiene B.
11 9

Ejercicios
1. Determine el valor de la(s) variable(s) si en cada caso A = B siendo:
a) A = 1 0 B = x y d) A = a +b ab B = 4 2

4 2
1 x 1 3 y+1
b) A = B = 2x 3
y 1 e) A = B =
4 1 0 3 0 3
2 x y a 3 4 x+y 1 y 3x 4
c) A = B = f) A = B =
1 0 1 b 0 1 0 2 0 2

Matrices y determinantes 447

446-447 447 10/11/2001, 13:38


Ejercicios
x2 5
g) A = 3y B= 2 xy x2 y2 x3
z+1 1
i) A = x 5y 3x2 2 x3
u+v 2 2x 2
h) A = B= 2x 3 2y x3
x 2y 2x + y 2x2
ux w 5 y+1 3

z
2 2 6z w 2x+y x2 + y2 2 x3
i) A = 3 B= 5 xy B= 5y 2x2 x3
x + y 12
x3y 3x2 0
j) A = [a2 + b2 2a] B = [a2 b2 9] 1+ 2+ 3 1 + 2 2 3
j) A= 1 2 3 B == 1 2+ 3
1+ 2+ 3 1 2+ 3
2. Dadas las matrices A y B, determine en
3. Dadas las matrices:
cada caso la suma A + B.
2 0 3 3 1 2
a) A = 2 4 5 B = 06 60 33 A= B=
1 4 1 0 1 4
1 1 0
C= 3 1 3
1
2 2 1 2 5 3
2
1 1 1 Obtenga g) A+B+C
b) A = B= 2
2 4 2 a) A d) A + B h) BA
1
1 4
1 2 b) B e) A C i) CA
3 2
c) C f) A + B C j) CBA
1 2 1 2 0 1
c) A = 2 1 2 B= 2 4 3
4. Determine el valor de las variables en
0 3 3 1 5 2
cada caso:
d) A = 2 2 3 3 5
1 +2x 3 5 3 2y 2
1 1 2 a) +
2 2y 3 z 1 3
3 2 3 3 5 5
B= = 9 5 u
0 1 1 8 v w

a + 2b 3a 2 ab 2b a2 b + a2 2 b 8 3
b) =
e) A = 2b a a + b B= 3 b 2a 3a 2b c 0 4 d 6
b 5 a 3a a + 4b b 7a
c) [6 + x 2 y 6 z] + [3 x 1 y 4 z]
f) A = [ x + y 2 x y 3 x 3y]
= [x y z]
B = [ x + y x 2 y 2 x + 4y] 1+a 4 2a 2 3 x
d) 1+b 3 + 2b 3 = 6 y
2 ab 3 a + 2b 1+c 3 2c 5 9 z
g) A =
b 5a b + 3a
5. Resuelva las siguientes ecuaciones
matriciales
a+b b
2a
3 5 2 3 +X= 1 0
B= a)
b 3a 1 5 0 1
3 a + 4b 2
5
2 3 1 1
2 2

[ ] +X= 2 5
h) A = 1 + 2 b)
1
B= 6 1 3 0
1 2 0
1 5 4 2

448 Matrices y determinantes

448-449 448 10/11/2001, 18:00


CAPTULO 11

c) X + [1 2 a 3 + a] 6. Exprese la siguiente matriz:


= [a 1 2 a a 2] A=
1 0 1 0 0 3
d) X 2 0 2 = 0 3 0
1 3 1 2 0 1
Como:
1 2 0 2
e) 3 1 X= 1 1 a) Suma de dos matrices cualesquiera.
7 2 2 2
b) Suma de dos matrices tales que la
a 2b 3b 2a 3b b
f) X= diagonal principal de una de ellas est
ba 2b b a+b b 2b
fomada slo por 1.
1+ 2 1 c) Suma de dos matrices tales que los
g) 3 X= 0 elementos que estn bajo la diagonal
2 3 1
principal de una de ellas sean todos
iguales a cero.
h) [2, 1 3, 7 0, 4] + X
d) Suma de dos matrices tales que los
= [ 3, 3 1, 7 2, 5]
elementos que estn sobre la diagonal
2 4 3 1 principal de una de ellas y los que estn
3 3 5 2 bajo la diagonal principal de la otra
i) X= sean todos iguales a cero.
3 5 1 2
4 2 7 9 e) Suma de dos matrices tales que la
1m m+5 m m diagonal principal de una de ellas est
j) X =
2+3m 2m 0 1 formada slo por ceros.

Soluciones

1. a) x = 1 y = 0 b) x = 3 y=4 3 2 0 4 2 3 8 5
c) 0 5 5 d)
c) a = 2 x = 3 d) a = 1 1 8 5
1 0 3
b=1 y=4 b=3
3a+b 3a+2b
2 3 e)
e) x = f) x = 5b3a b2a
3 2 5b4a b4a
y=3 y=3 f) [2 x + 2 y 3x3y 5 x + y]
2
g) x = 5 y= z=2 7a5b 29 b
3 5a+ 2
g) 3 5 h)
h) x = 3 y=2 u=2 1 2
21 a + 3 b
v=8 w=1 13 b 2 a
5
17 7 2 2
i) x = y= z= w=
2 2 3 9
3x 2x2 x3
9
j) a = b=0 i) x 10 y x2 3x3
2
5
3 x3
2 3x 2 y 5 x2
2 10 8 3 1 3
2. a) b)
7 1 3 2 4 3 2
5 7 j) 22 2
6 2 2 3

Matrices y determinantes 449

448-449 449 10/11/2001, 18:01


2 0 3 3 1 2 1 4
3. a) b) 0 1 4 1 4 7
1 4 1
5. a) X = 3 b) X = 3 1
1 6
3 3
c) 3 1 3 d) 1 1 5
2 5 3 1 5 3
c) X = [a 2 3a 2 5]
1 1 6 4 2 8
e) f)
1 1 2 1 0 6 1 0 4 1 0
d) X = 2 3 2 e) X = 2 0
5 1 1
g) 2 0 2 h) 1 3 5
3 3 2 5 4
3 10 0
5b 2
1 1 6 4 2 8 3a 4b
i) j) f) X = 2a 3b b
g) X = 3
1 1 2 1 0 6 1 3
5
4. a) x = z=6 v= 0 h) X = [ 5,4 5,4 2,1]
2
y=1 u=3 w = 6
b) a = 2 c=2
1 11
b= 3 d=6 5 6
i) X = 17 41
c) x = 2 y= 1 z = 1 28 18
2
4 7 10
d) a = b= c= 1 2m + 5
3 3 3 j) X =
2 + 3m 1 + 2m
x=6 y=0 z=2

11.3 Ponderacin de una matriz


por un escalar

11.3.1 Denicin
Ponderar una matriz A por un escalar es multiplicar cada
elemento de la matriz A por el elemento .
En general consideraremos a como un elemento del conjunto
de nmeros reales.
El resultado de esta nueva operacin nos da una nueva matriz
del mismo orden de la matriz original.

11.3.2 Propiedades
Notemos que la ponderacin de una matriz por un escalar es una
operacin entre elementos de diferentes conjuntos.
Sean A y B matrices de orden m n y sean a y b dos escalares.
(Consideremos a, b, E R).

450 Matrices y determinantes

450-451 450 20/11/02, 6:24 PM


CAPTULO 11

Se cumple:
1. a (A + B) = aA+aB
2. (a + b) A = aA+bA
3. (a b ) A = a (b A)
4. 1 A = A
Recordemos que (M m n , +) tena estructura de Grupo Abeliano.
Si agregamos al conjunto esta nueva operacin recin definida y
sus propiedades, ampliamos la estructura algebraica y obtenemos
un Espacio Vectorial.
As, tienen estructura de Espacio Vectorial aquellos sistemas
formados por un conjunto dotado de una operacin que tenga
estructura de Grupo Abeliano y de otra operacin que tenga las
propiedades 1 a 4 indicadas anteriormente.

1. Sea A =
1 6 2
y a=4 Ejercicios
2 0 3
resueltos
Obtengamos a A.
1 6 2 4 24 8
a A = 4A = 4 2 0 3
=
8 0 12
Para obtener a A simplemente multiplicamos cada elemento
de A por 4, que es el valor de a.
2. Determinemos el valor de las variables en:

1 0 4 y
x = z v2
2 3

Efectuemos la primera operacin y luego planteemos las


igualdades correspondientes:
x 0 4 y
= z
2x 3x v2
de donde: x=4
y=0
2x = z Q z=8
3x = v 2 Q v = 14

1 2 3 5
3. Dadas las matrices A= 3 1 y B= 2 6
3 4 5 4

Resolvamos la ecuacin X 5A = 3B

La solucin de esta ecuacin est dada por:

X = 5A 3B es decir:

Matrices y determinantes 451

450-451 451 20/11/02, 6:24 PM


1 2 3 5
X= 5 3 1 + 3 2 6
3 4 5 4

5 10 9 15
X= 15 5 + 6 18
15 20 15 12

14 25
X= 9 23
30 32

12 9 27
4. Expresemos la matriz A = 3 12 6

como el producto de un escalar por otra matriz.


Este problema tiene infinitas soluciones, pues podemos escoger
cualquier nmero real como el escalar, pero una simple inspeccin
de los elementos nos indica que podemos factorizar la matriz
por 3 y as mantener todos los elementos enteros.
12 9 27 4 3 9
=3
3 12 6 1 4 2

5. La siguiente es una tabla de precios de artculos all sealados.


pequeo mediano grande
leo 1.100 1.600 2.000
ltex 1.300 1.750 2.200
diluyente 700 1.000 1.300

Podemos considerar la informacin proveniente de una matriz


cuyas filas nos sealan el producto y cuyas columnas nos
sealan el tamao.

Determinemos ahora la matriz correspondiente a los precios de


los artculos si ellos estn rebajados en un 10%.

El nuevo precio se obtiene al multiplicar cada precio por 0.9, as


la matriz de los nuevos precios ser:
1.100 1.600 2.000 990 1.440 1.800
0.9 1.300 1.750 2.200 = 1.170 1.575 1.980
700 1.000 1.300 630 900 1.170

Ejercicios
1. Calcule:

1 4
a) 2 1 4 b) 1 0 1 5 1
1 2 3 1 3
e) 1
1 0 2 9
f)
1 6 3 1 8 1 2 2 6 3 4 2 2
c) 2 d) 5 8 4 3 7
2 1 2 3 2 5 4

452 Matrices y determinantes

452-453 452 10/11/2001, 13:47


CAPTULO 11

2 w + z
2 3 3 12 3
g) 2 h) e) x
4
= 1
z
1 0 2 27 3 3 y
6 12
ab a+b
i) a + b a b j) 0 1 0 1 2
5 3 15 3 x x1 9 p
b a
f) 2 y y+2 = 3 11 q
2. Dadas las matrices:
z z4 4 r
1 3 2 2 2 4
A= 4 1 3 B= 4. Resuelva las siguientes ecuacio-
3 1 5
3 3 4
nes matriciales:
C=
2 1 5
a) X + 2 1 0 1
= 3
2 3 4
Determine: 2 1 3 1 0 3

a) 2A B b) A 2B + 3C 1 0 3 3
b) 1
c) 3A 3B d) 4 (A B) 2 1 X= 1 2 5
5 3 2 4 6
e) 2
A
1
B f) 2 (A + B) 2 (A B)
5 2
c) 2 X 3 1 4 1
= 3
2 5 2
g) 4 B h) B 2 A 3 2 3 2 1 5

1 3 1
i) 1 C 1 B + 1 A j) 3
2
A + B d) 2 1 0 2
3 2 4 3 1 0 4
3 2 3
3. Determine el valor de las incgnitas 4 2 1
en cada caso: =X+ 5 6 3

x y 1 5 5. Dadas las matrices:


a) 2 =3 u v
3 2 1 2 4 5 2 1
A= 1 3 ;B= 1 1 yC= 1 0
b) 3 1 0 = 2 3 5 w
u v
8
2 4 3 2 1 3
2 x 5 4
resuelva las ecuaciones:
2x y x+9 y 12 a) 3A + X = B
c) 2 1 y 2 = u 2v
3 3+x w z b) 2A + C = B X
c) 2X + A = 3C
d) 3 xz uy = y + 1 x 2 d) B 2C X = 3A
2z+1 2z

Soluciones
3
3
2 8 0 1 2 6 2
1. a) b) g) h) 3
2 4 1 0 2 0 9
2 2
2 12 6 5 40 5 10
c) 4 2 4 d) 15 10 25 20
a 2 b2 a2 + 2ab + b2
5

1 3
3
i) a2 + ab ab + b2
2 2 4 ab b2 a2 ab
9 3 9
1
e)
2
f)
4 4 j) 0 0 0 0
3 3 3 0 0 0 0
3
2 2 2
4 2
9 21 0 4 0 12 8 6
4 4
2. a) b)
5 1 11 4 4 2

Matrices y determinantes 453

452-453 453 10/11/2001, 13:48


3 3 6 4 4 8
c) d) 4 0 32
e) x = 2 y=6 z=8 w=4
3 0 24
27 29 33 29
4 f) x = p= y= q=
0 0 2 3 2 3
5 8 8 16
e) f)
1
1 11 12 4 20
z=6 r= 4
5 5 3
16
8 8 16 0 4 0 3
g) 12 4 20
h) 5
5 1 11 8 9 10 12 26
4. a) X =
7 2 15
b) X = 5 5
5
1 0 17 28
i) 3 j) 8 12 16 5 5
1 1 11 17 5 9 9 3 3
2 2 6 5 5 6
c) X = 2 2 2 d) X =
3 4 15 9 1 10 2
15 3
3. a) x = y= u=2 v= 2 2
2 2 3
3 1 1 11 8 0
b) u = 1 v= w= x = 12 5. a) X = 2 8 b) X = 2 7
5 5
9 10 0 13
9
c) x = y=4 u=6 v=2
5 7 5
12 2 2
w=6 z= 3
11 3
5 c) X = 1 d) X = 2 10
5 7 1 2
2 1 20
d) x = y= z= u= 1 13
8 8 5 15
2 2

11.4 Multiplicacin de matrices

11.4.1 Procedimiento
Sean A y B dos matrices de orden 1 n y n 1, respectivamente.
El producto de A y B es una matriz de orden 1 1 y se obtiene
sumando todos los productos de la forma a1i bi1 donde 1 i j.
As: A = a11 a12... a1n y B = b11
b21



bn
entonces AB = a1 b1 + a2 b2 + ... + an bn
Una ampliacin de esta definicin es la siguiente:
Sea A una matriz de orden m n y B una matriz de orden n p. El producto AB es una
matriz C de orden m p, donde cada elemento Cij se obtiene sumando los productos de
los elementos de la i-sima fila de A con la j-sima columna de B.
1 2
Ejemplo: A = 1 2 1 B = 1 3
3 1 2
3 1

454 Matrices y determinantes

454-455 454 20/11/02, 6:32 PM


CAPTULO 11

A tiene orden 2 3 y B tiene orden 3 2, por lo tanto, la


matriz producto C tendr orden 2 2 (nmero de filas de A y
nmero de columnas de B).
El elemento C11 se obtiene multiplicando fila 1 de A por
columna 1 de B.
El elemento C12 se obtiene multiplicando fila 1 de A por
columna 2 de B.
El elemento C21 se obtiene multiplicando fila 2 de A por
columna 1 de B.
El elemento C22 se obtiene multiplicando fila 2 de A por
columna 2 de B.
Para cada elemento se suman los productos obtenidos.
1 1 + 2 1 + 1 3 1 2 + 2 3 + 1 1
AB = = 6 7
3 1 + 1 1 + 2 3 3 2 + 1 3 + 2 1 10 7

Observacin: La multiplicacin de matrices NO es una operacin


conmutativa.
Si el nmero de columnas de una matriz no coincide con
el nmero de filas de la otra entonces el producto no se puede
efectuar.

11.4.2 Propiedades de la multiplicacin

Como vimos anteriormente, no todas las matrices se pueden


multiplicar. Consideremos aqu el conjunto de matrices cuadradas,
de orden n n.
En este conjunto la multiplicacin cumple las siguientes
propiedades:
1. Asociatividad.
Sean A, B, C matrices de orden n n, entonces
(A B) C = A (B C).
2. Existe neutro multiplicativo.
Sea A E M n n, entonces existe la matriz identidad
I E M n n tal que:
A I = I A = A.
Esta matriz, llamada tambin matriz unitaria, est formada por
1 en la diagonal principal y 0 fuera de ella.
3. El producto es distributivo respecto de la suma.
Sean A, B y C matrices de orden n n; se cumple:
A (B + C) = AB + AC.
4. Existen divisores de cero.
Recordemos que en el conjunto de nmeros reales R si el producto
de dos nmeros es igual a cero, necesariamente uno de ellos es
cero. En el caso de las matrices, ello no ocurre y as podemos tener
dos matrices no nulas y tales que su producto es cero.
As, afirmamos que H A, B E M n n, A 0 A B 0 tales
que AB = 0

Matrices y determinantes 455

454-455 455 20/11/02, 6:33 PM


Observacin: El producto de matrices cuadradas de orden n
no es conmutativo.
No todas las matrices son invertibles, es decir, dada una
matriz A de orden n n no siempre es posible encontrar una
matriz A1 tal que A A1 = A1 A = I
Cuando es posible encontrar dicha matriz se dice que A es
invertible y A1 se llama la matriz inversa de A.

11.4.3 Matrices inversas y


ecuaciones multiplicativas
Sea A una matriz cuadrada de orden 2 2 dada por:

A= a b , donde a, b, c, d E R
c d
La matriz inversa de A viene dada por:
1 d b
A1 =
ad bc c a
La condicin entonces para que la matriz A = a b sea
c d
invertible es que ad-bc sea distinta de cero.
La determinacin de la matriz inversa de una matriz de orden
3 la dejaremos para el punto siguiente, pues con los determinantes
se facilita mucho su obtencin.
Aplicando las matrices inversas podemos resolver ecuaciones
multiplicativas. Es decir, sean A y B dos matrices cuadradas.
Resolvamos: A X = B
Si A es invertible, entonces existe A1 tal que
A1 A = I = A A1 As:
AX = B / A1
A1 A X = A1 B
Recordemos que la multiplicacin no es conmutativa; por lo
tanto, si multiplicamos un lado de la igualdad por la izquierda,
debemos multiplicar el otro lado tambin por la izquierda.
Entonces la solucin de la ecuacin est dada por:
X = A1 B

Ejercicios 2 3
1. Sea A = [ 1 2] y B= 1 3
resueltos
a)Es posible efectuar el producto AB y BA?
b)Qu orden tiene(n) la(s) matriz(ces) resultante(s)?

456 Matrices y determinantes

456-457 456 10/11/2001, 18:13


CAPTULO 11

c) Obtengamos los productos.


Soluciones:
a) A tiene orden 1 2 y B tiene orden 2 2; por lo tanto, se
puede obtener AB pero no BA.
b) AB tiene orden 1 2 (nmero de filas de A y nmero de
columnas de B).
2 3
c) 1 3

[1 2] = [x y]

Notemos que la distribucin as dispuesta nos entrega un


modo fcil de efectuar los productos, as:
AB = [1 2 + 2 1 1 3 + 2 3]
AB = [0 9]

2. Dada A = 3 2 y B = 2 3
1 5 3 4

Obtengamos AB y BA.

AB = 3 2 2 3 BA = 2 3 3 2
1 5 3 4 3 4 1 5

6 6 98 6 + 3 4 15
= =
2 + 15 3 + 20 9 4 6 + 20

0 17 9 19
AB = 13 23
BA =
5 14

Es claro que AB y BA son diferentes.


2
3. Sea A = 2 1 5 y B= 5
3 2 4
1
Obtengamos AB.
2 1 5 2 4 + 5 + 5 4
= =
3 2 4 5 6 + 10 + 4 8
1

4. A= 2 2 y B = 1 0 obtener AB y BA.
1 3 0 1

En este caso vemos que B es la matriz identidad y por lo


tanto es neutro multiplicativo, entonces AB = BA = A.

5. Obtengamos la matriz inversa de A = 4 1


1 2
y comprobemos.
a b
Recordemos que para que exista la matriz inversa de c d
debe ocurrir que ad bc 0.

Matrices y determinantes 457

456-457 457 10/11/2001, 18:14


En este caso ad bc = 8 + 1 = 9 0, por lo tanto, A-1 existe
y es: A 1 = 1 2 1
9 1 4
Comprobemos:
1 2 1 4 1
A 1 A =
9 1 4 1 2

1 8+1 2+2
=
9 4 +4 1+8

1 9 0
=
9 0 9

= 1 0 que es la matriz identidad.


0 1

Como el producto de matrices no es conmutativo debemos


comprobar que A A1 tambin es la matriz identidad.
4 1 1 2 1
A A 1 =
1 2 9 1 4

2 1
4 1 9 9
=
1 2 1 4
9 9

8 1 4 4
+
= 9 9 9 9 = 1 0
2 2 1 8 0 1
+
9 9 9 9

6. Determinemos si existe la matriz inversa de:


3 5
A=
6 10

En este caso vemos que la condicin para la existencia de la


matriz inversa no se cumple, pues ad bc = 30 30 = 0 y,
por lo tanto, decimos que A no es invertible.
7. Sean A y B dos matrices dadas por:
4 5 3 8
A= y B=
1 2 2 5

Resolvamos la ecuacin B X = A
Sabemos que si B es invertible, la ecuacin tiene por solucin
X = B1 A.
B cumple la condicin para tener matriz inversa y sta es:
5 8
B 1 =
2 3
1
y as, la solucin de BX = A se obtiene mediante B B X = B1 A
{

I X = B1 A
X = B1 A

458 Matrices y determinantes

458-459 458 10/11/2001, 13:55


CAPTULO 11

es decir X = 5 8 4 5
2 3 1 2

X = 20 + 8 25 + 16
83 10 6
28 9
X=
11 4

Ejercicios
1. Calcule los siguientes productos: 3. Dadas las siguientes matrices:

2
A = 2a a B=
a 2a
a) 1 2 1 1 a 3a a 2a
0
2 2 2 5 1
b) y 0 a
3 4 1 2 1 C=
a 0
1 1
c) 2 1
1 4 calcule:
1 2
3 2
a 2a a) AB AC b) A(B C) c) 2A 3AB
a b a b ab 2b
d) b2 a
d) 3 ABC e) (A B)C f) (B C) A
a b

2 3 2 3 6 g) 3 (A + B C) h) (2A + B) C
e)
3 2 3 6 8
1 a 4. Determine el valor de las variables
f) 2 2a a
3 3a
3a en cada caso:
ab ba
a+b ab 2a 3 2
g) a+b a+b 2b
a+b b+a
a) x y = 9 13
a b 5 3
9 4 4 8
h) 9 3 9 18

2. Encuentre la matriz inversa de las b) 2a 3b 1 2 = 4 8


siguientes matrices. Compruebe que 3 4

son inversas y en caso de no existir la


inversa explique por qu.
2 5 x 9
a) A= 2 2 b) A = 3 4 c) y
=
1 3 5 1 3 7 15

1
1
c) T = 2 d) M = 8 12
2a b
1 2 3 4 1
= 6 3
2 d) x 3y 1 3 4 9
2
e) P = x 2x f) Q = 0.5 0.6
x 3x 0.1 1
3 4 x y
2 3 e) = 1 2
g) M = b b h) N= 2 1 2z u 0 1
3b 2b 3 2

Matrices y determinantes 459

458-459 459 10/11/2001, 13:57


Ejercicios
1 16 22
d) X= 1 0
3 2 2 4 12 0 1
f) x 2y 1 1 3 = 4 3
2 4 1 2 2 = 1 1
e) X
3 5 1 1
a1 a2 a3 a4 4
5
4 b1 b2 b3 b4 3 6 5 12
g) 2 5 3 6 = c c c c f) 2
5 2
3 X=
2 1 2 3 4 9 4
1 d1 d2 d3 d4
8. Demuestre que en el conjunto
5. Sea A una matriz de orden 2 3 de matrices cuadradas de orden 2
Sea B una matriz de orden 2 4 el producto no es conmutativo.
Sea C una matriz de orden 3 4 9. Encuentre dos matrices cuadradas
Sea D una matriz de orden 4 3 de orden 2, A y B y ejemplifique
Sea E una matriz de orden 4 2 la siguiente proposicin:
Indique todos los productos que
A1 B A B
se puedan efectuar y d un
ejemplo de cada uno de ellos. 10. Encuentre un par de matrices cua-
Indique el orden o dimensin de dradas de orden 2 cuyo producto
cada producto. sea conmutativo.
6. Encuentre un par de matrices Nota: No considere la matriz inversa
no nulas cuyo producto sea nulo. ni la matriz nula.
Haga el caso en que ambas son 11. Sea A = 1 a
0 1
matrices cuadradas, es decir, de
orden n n y el caso en que Calcule A2, A3, A4, A5 ...
ambas no son cuadradas. Puede hacer alguna generalizacin?
7. Resuelva las siguientes ecuacio-
Nota: A2= A A ; A3= A2 A ;
nes matriciales multiplicativas:
A4 = A3 A
2 3 2 2 1 3
a) 1 6
X=
1 6 12. Sea A = 2 3 yB=
1 4 2 5

3 4 3 4
b) X
0 5
= Calcule A1 , B1 , AB y (AB)1
5 6
Verifique que: (AB)1 = B1 A1
6 11 4 8
c) X = (2) Ocurre siempre esto? Demustrelo.
9 4 6 12

Soluciones
2 b2 2 ab
g) 4 b2 2a2
h) 0 0
2 ab 9 18
1. a) 0 b) 2 14 4
10 7 1 2. a) A1 = 1 3 2
8 1 2
2 2
c) 3 6 d) [a2 + ab 3a2 + 3b2] 1 1 4
5 8 b) A1 =
23 5 3
5 6 3 2 2 3 + 2 6
e) 2
1
2 6 3 + 2 3 3 2 4 4
c) T1 = 2
7 1
a 3a2 1
2
f) 2a 6a2
3a 9a2 d) M no es invertible. (Ver 11.5.1)

460 Matrices y determinantes

460-461 460 10/11/2001, 14:00


CAPTULO 11

1 3x 2x 5. Se pueden efectuar los siguientes


e) P1 =
5x2 x x productos de 2 matrices:
3
1 AC de orden 24
f) Q1 = 25 5
11 1 1 BD de orden 23
10 2
BE de orden 22
g) M1 = 1 2 2b b
5b 3b b CD de orden 33
1 2 3
h) N1 = CE de orden 32
5 3 2
EB de orden 44
2a 2 0 2a2 0
3. a) a2 7a 2
b) a2 7a2 EA de orden 43

4a + 3a2 2a + 6a2
D C de orden 44
c)
2a 12a2 6a 24a2
Algunos productos de 3 matrices son:
6a3 3a3
d)
24a3 12a3 A C D de orden 23
3a 2 3a 2 a2 4a 2 A C E de orden 22
e) a2 2a 2
f) 2a 2 6a 2

3a 0 0 3a2
g) h) 8a2 a2 2 1 5 3
3a 15a 6. A = y B=
2 1 10 6

4. a) x = 92 y= 21
Es claro que A 0 y B 0 y AB = 0
19 19
2 8
b) a= b=
5 15 1 15 0
12 57 7. a) X = 1 9 6 b) X =
c) x= y= 15 4 14 15 25 38
29 29
18
d) a = 15 b=
26 13
2 88 4 1 6 11
21 32
c) X = d) X =
x= y= 123 84 138 26 2 8
13 39
1
x= 1 1 32 0
e) z= e) X = 2 0 f) 1 36
5 5 2 0 108
2 2
y= u=
5 5
11. A2 = 01 12 a A3 = 01 13 a A4 = 1 4 a
1 0 1
f) x= 7 y=
11 22
Generalizando: An = 1 n a
g) a1 = 10 a2 = 25 0 1
a3 = 15 a4 = 30
1 4 3 1 5 3
b1 = 8 b2 = 20 12. A 1 = B 1 =
5 1 2 11 2 1
b3 = 12 b4 = 24
c1 = 4 c2 = 10 AB = 8 9 (AB)1 = 1 17 9
9 17 55 9 8
c3 = 6 c4 = 12
d1 = 2 d2 = 5 1 17 9
B1 A1 =
d3 = 3 d4 = 6 55 9 8

Matrices y determinantes 461

460-461 461 10/11/2001, 14:02


11.5 Determinantes y sistemas
de ecuaciones

11.5.1 Determinantes y
sistemas lineales de orden 2
Dado el siguiente sistema de 2 ecuaciones:
ax + by = p
cx + dy = q

Las soluciones para x e y estn dadas por:


pd bq aq pc
x = e y =
ad bc ad bc

La solucin del sistema existir slo si el denominador comn


x e y es distinto de cero. Esto es, ad bc 0.
Vemos que el sistema original tambin puede ser planteado
como ecuacin matricial multiplicativa de la siguiente manera:
a b x = p
c d y q

Llamando A a la matriz de los coeficientes, X a la matriz de


las variables y B a la matriz de los trminos constantes, tenemos la
ecuacin A X = B que tiene por solucin X = A 1 B, siempre que
A 1 exista, es decir, siempre que A sea invertible y esta condicin
est dada por: ad bc 0.

La expresin ad bc se llama DETERMINANTE de orden 2


asociado a la matriz a b y se denota por det. A = a b
c d c d
Vemos que las soluciones encontradas para x e y a partir
del sistema pueden ser expresadas en forma de determinantes
de la siguiente manera:
p b a p
q d c q
x = y =
a b a b
c d c d

As, si llamamos al determinante formado por los coeficientes


del sistema, x al determinante obtenido cambiando la columna
correspondiente a la variable x por la columna de trminos
constantes y, y al determinante obtenido al cambiar la columna de
los coeficientes de y por la de trminos constantes, podemos escribir
la solucin del sistema de la siguiente manera:

x y
x = e y =

462 Matrices y determinantes

462-463 462 10/11/2001, 14:03


CAPTULO 11

11.5.2 Determinantes y
sistemas lineales de orden 3

Consideraciones similares a las hechas en el punto anterior nos


llevan a definir un determinante de orden 3 a partir del sistema
de 3 ecuaciones lineales:
a1x + b1y + c1z = d1
a2x + b2y + c2z = d2
a3x + b3y + c3z = d3
como el valor comn de los denominadores de x, y y z una
vez despejados stos.

as a1 b1 c1
a2 b2 c2 = a1 b 2 c3 + a2 b 3 c1 + a3 b 1 c2
a3 b3 c3 a3 b 2 c1 a1 b 3 c2 a2 b 1 c3

Si conmutamos los trminos obtenidos y factorizamos obtenemos


que el valor del determinante puede ser expresado:
a1 (b2 c3 b3 c2) b1 (a2 c3 a3 c2) + c1 (a2 b3 a3 b2),
lo cual es equivalente a reducir el determinante de orden 3 a 3
determinantes de orden 2, es decir:
a1 b1 c1
b2 c2 a2 c2 a b2
a2 b2 c2 = a1 b1 + c1 2
b3 c3 a3 c3 a3 b3
a3 b3 c3

Este proceso se llama DESARROLLO del Determinante por


MENORES.
Un proceso ms simple para el clculo de un determinante de
orden 3 es la llamada Regla de Sarrus, que consiste en agregar
al determinante las 2 primeras filas (en el mismo orden) o las dos
primeras columnas y luego formar todas las diagonales completas,
es decir, con 3 elementos. Los productos de las diagonales en
este sentido se suman y los productos de las diagonales en
el otro sentido se restan.
Ahora, para resolver un sistema de 3 ecuaciones lineales:
a1x + b1y + c1z = d1
a2x + b2y + c2z = d2
a3x + b3y + c3z = d3
formamos los siguientes determinantes:
formado por los coeficientes de las variables.
x obtenido de reemplazando la columna de los coeficientes
de x por la columna de los trminos libres.
y reemplazando en los coeficientes de y por los trminos libres.
z reemplazando en los coeficientes de z por los trminos libres.
a1 b1 c1
As: = a2 b2 c2
a3 b2 c3

Matrices y determinantes 463

462-463 463 10/11/2001, 14:03


d1 b 1 c1 a1 d1 c1 a1 b 1 d1
x = d2 b 2 c2 y = a2 d2 c2 z = a2 b 2 d2
d3 b 3 c3 a3 d3 c3 a3 b 3 d3

y la solucin del sistema est dada por:


y z
x = x y = z =

Este mtodo para obtener soluciones es conocido como REGLA
DE CRAMER.
Observacin: Claramente el sistema tiene solucin si el valor
de es distinto de cero.

Ejercicios 1. Calculemos el valor del determinante:


resueltos 2 2
1 3

De inmediato: 2 3 1 2 = 6 + 2 = 8
2. Apliquemos determinantes para resolver el sistema:
3x y =10
x + 2y = 8

Formemos los determinantes , x y y:

= 3 1 = 6 (1) = 7 Como es distinto de 0


1 2
el sistema tiene solucin y
calculamos x y y
x = 10 1 = 20 (8) = 28
8 2

y = 3 10 = 24 10 = 14
1 8
y la solucin del sistema es:

x = x = 28 = 4
7
y
y = = 14 = 2
7
3. Apliquemos determinantes para resolver el sistema:
2x 7y = 3
4x + 14y = 1
Formemos y calculemos el determinante principal
2 7
= 4 14
= 28 28 = 0

Como = 0 el sistema no tiene solucin. (En este caso se trata


de dos rectas paralelas.)

464 Matrices y determinantes

464-465 464 10/11/2001, 14:05


CAPTULO 11

4. Calculemos el valor del determinante:


1 2 2
3 3 4
2 1 5

Aplicando la regla de Sarrus, obtenemos:


1 2 2 1 2
3 3 4 3 3
2 1 5 2 1

= (15 + 16 6) (12 4 + 30)


= 25 38
= 13
5. Resolvamos el sistema de ecuaciones:
2x + y z = 6
3x 2y + 3z = 3
x y + z = 0

Formemos y calculemos el determinante principal:


2 1 1 2 1
= 3 2 3 3 2 = ( 4 + 3 + 3) (2 6 + 3) =
1 1 1 1 1 2+1=3
Como 0 el sistema tiene solucin y calculamos:
6 1 1 6 1
x = 3 2 3 3 2 = ( 12 + 3) ( 18 + 3) =
0 1 1 0 1 9 + 15 = 6
2 6 1 2 6
y = 3 3 3 3 3 = (6 + 18) ( 3 + 18) =
1 0 1 1 0 24 15 = 9
2 1 6 2 1
z = 3 2 3 3 2 = (3 18) ( 12 6) =
1 1 0 1 1 15 + 18 = 3
y las soluciones son:

x
x = Qx = 2

y
y = Qy = 3

z
z = Qz = 1

6. Resolvamos el siguiente sistema aplicando determinantes.
(Regla de Cramer).
3x 2y + 4z = 9
2x + 3y z = 2
x + y + 3z = 1

Matrices y determinantes 465

464-465 465 10/11/2001, 14:06


Formemos y calculemos el determinante principal.

El valor del determinante principal es cero, por lo tanto, el


sistema no tiene solucin.

Ejercicios
1. Calcule el valor de los siguientes c) 2 6
d) 5 2
determinantes: 1 3 10 2

2 3 3 2 h2 2h 1 a
a) b) e) f)
1 2 1 2 h 2 0 1
3 4 2 3
c) d) 1 5 g) 1 a h) 2 0
5 2
a 1 1 2
a 1 2 b
e) 1 a f) b 3 4. Determine en cada caso el valor de la
incgnita para que los determinantes
g) h 1 h) 2 1 tengan el mismo valor:
2 h 3 2
1 3 2 2
1 a) y
0.2 5 x 1 3
2 2 0.5
i) 1
j)
0.1 3 0.3 y 2y 4 1
4
b) y
3 5 3 3

k) 0 1
l) 1 3
5 3 2 x 3 2
2 2 c) y
3 1 1 2
2. Para cada uno de los siguientes
determinantes encuentre otro que 3 3 h+1 2
d) y
3 3
h 2h
tenga el mismo valor:
2x x x1 3x + 2
a) 2 3 b) 1 3 e) 3 2
y
1 5
1 4 5 6
5. Resuelva los siguientes sistemas de
5 5 3 2
c) d) ecuaciones aplicando determinantes:
2 2 5 6
a a a a2
e) f) a) x + y = 6 b) x 5y = 2
b b a 2a
2x + y = 9 3x + 2y = 6
3. Dadas las siguientes matrices aplique
determinantes para establecer si ellas
son invertibles o no. c) 3x 5y = 11 d) 5x + y = 2
3 1 2x + 4y = 1 x 4y = 3
a) 2 1
b) 1 4
1 5

466 Matrices y determinantes

466-467 466 10/11/2001, 14:08


CAPTULO 11

1 1 g) 2x y = 2 h) y z= 3
e) 2x 3y = 6 f) x+y=
2 3
1 x + 2z = 1 2x + 3z = 1
x 3y = 12 x y=4
2 2y 3z = 2 x 2z = 4
g) 4x 3y = 5 h) 9x + y = 1 8. Resuelva los siguientes sistemas
3 1
2x y = 2 x y=2 de ecuaciones aplicando deter-
2 10
minantes (Regla de Cramer).
i) 0.1x 1 y=4
3 a) 2x y z = 0 b) x + 3y z = 3
3x + 10y = 3 x + 3y + z = 5 2x y + 4z = 14
3x y + 2z = 4 x 5y + z = 15
6. Calcule el valor de los siguientes
determinantes:
c) x + 3y = 9 d) 3x 2y 3z = 12
2 3 2 1 2 1
a) 1 3 1 b) 3 1 0 2x z = 8 x+ y+ z=3
2 1 3 2 1 2
y + 4z = 6 2x y 2z = 3
2 1 5 2 1 2
c) 1 2 3 d) 0 3 1
3 1 2 3 1 2 e) 4x + y + z = 3 f) x + y z = 2
0 3 1 2 2 2
4x y + z = 3 x y + z = 2
e) 2 1 2 f) 1 1 1
0 1 2 3 3 3 2x + y + 4z = 3 x+y+z=0
7. Escriba la ecuacin matricial corres-
pondiente a cada uno de los siguientes g) x 3y 2z = 5
sistemas: 2x y + 3z = 27
3x + 2y z = 22
a) 3x y = 2 b) x + y = 7
9. Demuestre:
x + 4y = 3 2x 6y = 2
b+c ab a
a) det c+a bc b = 3 abc a2 + b2 + c2
c) 2x + 3y = a d) 5x + 2y = 1 a+b ca c

x 6y = b 2x 3y = 2 1 a 1+a
b)
det a 1 1 =a a1 a+1
e) x 2y 3z = 3 f) 2x y + z = 4 1 a 1
2x + y + 4z = 1 x+yz=2 abc 2a 2a
c) det 2b bca 2b = a+b+c 3
x 3y z = 4 2x y z = 3
2c 2c cab

Soluciones

1. a) 7 b) 8 c) 14 d) 7 e) No f) S g) S h) S
e) a2 1 f) 6 + b2 g) h h 2 15
4. a) x = 23 b) y = c) x = 2
11
h) 2 2 3 i) 0 j) 0.9
d) h = 1 e) x = 7
k) 2 l) 4 3 2 9

3. a) S b) S c) No d) S 5. a) x = 3 y=3 b) x = 26 y=
12
17 17

Matrices y determinantes 467

466-467 467 10/11/2001, 14:09


39 25 11 17 2 1 1 x 4
c) x = y= d) x = y= f) 1 1 1 y = 2
22 22 19 19 z
2 1 1 3
4
e) x = 6 y = 6 f) x = 10 y=
3 3 2 1 0 x 2
g) No tiene solucin h) x = 19 y = 170 g) 1 0 2 y = 1
0 2 3 z 2
i) No tiene solucin
6. a) 3 b) 15 c) 22 d) 25 e) 10 f) 0 0 1 1 x 3
h) 2 0 3 y = 1
1 0 2 z 4
3 1 x 2 1 1 x
7. a) y
= b) y
= 7
1 4 3 2 6 2 8. a) x = 1 y= 1 z= 1
2 3 x
= a
b) x = 4 y=2 z= 1
c) y b
1 6
c) x = 3 y= 2 z = 2
5 2 x 1 d) x = 3 y= 3 z = 3
d) y
=
2 3 2
e) x = 1 y= 1 z= 0
1 2 3 x 3 f) x = 0 y= 1 z = 1
e) 2 1 4 y = 1
1 3 1 z 4 g) x = 2 y=5 z= 6

Prueba de seleccin mltiple


1. Sea A = (aij) 2 3 una A. 6 y 7 4. De las siguientes propo-
matriz tal que aij = i j B. 6 y 4 siciones para A y B
entonces A es: C. 4 y 2 A y B E Ma a:
I AB=BA
1 0 1 D. 4 y 4
A. II A0 y B0
0 1 2 E. 6 y 8
entonces AB 0
B. 0 1 2 3. Sea III A 0 entonces
1 0 1
existe A1
0 +1 +2 1 3
C. A. y Son falsas:
1 0 1 3 1
A. Slo I
0 1 B. 2 1
D. 1 2 1 1 B. Slo II
2 1
AB= C. II y III
0 1 D. TODAS
E. 1 0 A.
1 4
E. NINGUNA
2 1 2 0
1 3
2. Qu valor deben tener x 3 4 5.
B. Sea A = 5 6
e y para que las matrices 2 0 2 1
A y B sean iguales? 3 4 3 3
C. y B = 0 2
2 2
A= 3 5 1 4
2 x+2 1 4 La matriz X que satisface
D.
2 2
3 x+1
B X = A es:
B= E. 3 4 4 0
2 y 2 2 A. 5 4
1 5
468 Matrices y determinantes

468-469 468 10/11/2001, 14:12


CAPITULO 11
CAPTULO

2 0 A. k=0 B. I y III
B. 5 4
C. II y III
1 5 B. k0
C. k9 D. Todas
4 0
C. 5 4 E. Ninguna
1 5
D. k=9
E. A siempre es 12. La ecuacin matricial
2 6
D. 5 8 invertible 2 1 0 x 2
1 3 1 0 1 y = 3
z
2 0 9. Si A = 1 x 0 2 3 4
E. 5 8 0 1
representa el sistema:
1 5 entonces An =
2 1 3
A. 2x + y = 2
6. Sea A =
x 2y = 3
1 2 2
1 nx
1 2 1 A. 0 1 y 3z = 4
y B =
0 1 2
B. 1 xn B. 2x y = 2
La matriz C que satisface 0 1 xz= 3
CAB= 0 0 0 C. n nx 2y + 3z = 4
0 0 0 0 1
1 1 4 C. 2x y = 2
A. D. 1 nx
1 3 0 0 n xy= 3
B. 1 1 2 E. Depende de n 2x + 3y = 4
1 3 0
3 3 2 10. Qu valor deben tener D. 2x + y = 2
C. 1 1 4 x 2z = 3
las variables a y b para
3 3 2 que se cumpla: y + 3z = 4
D.
1 1 4 A = B siendo E. Ninguno de los
E. 1 1 4 1 a b anteriores.
1 3 0
A =
0 2 5
y
7. El sistema 3x y = 2 1 a+2 b 3
13. La matriz inversa de
B =
2x + 3y = 1 0 2 5 A = 6 5 es
5 4
expresado en
A. a=1 b= 3
forma matricial es: 2 A. A1 = 4 5
3 5 6
A. 3 1 2 = x B. a=2 b=
y 2
2 3 1 B. A1 = 4 5
C. a= 2 b= 3 5 6
3 1 2
B. xy =
D. a=2 b= 3 6 5
2 3 1 C. A1 =
E. a= 2 b = 3 5 4
3 1 x = 2
C. y
2 3 1
11. Dada la matriz D. A1 = 1 4 5
29 5 6
D. 3 2 x = 2 1 3 4 2
1 3 y 1 A= 2 3 1 5 E. A1 = 1 4 5
3 3 6 1 29 5 6
3 1
E. x y = 2 1
14. La nica proposicin
2 3 Se puede afirmar:
8. Para que la matriz I a11 = a43 verdadera es:
A =
k1 2k A. El conjunto de matri-
5 9 II a22 = 2 a33
ces con la multiplica-
no sea invertible III a13 a31 = a23 cin forma un grupo
se debe cumplir: A. Slo III abeliano.

Matrices y determinantes 469

468-469 469 10/11/2001, 14:13


Prueba de seleccin mltiple
2
B. El neutro multipli- A. 3
21. Determine los valores
cativo en Ma a es 2 de a y b para que se
B.
3 cumpla la igualdad
I = 1 1 3
1 1 C. 2 A 3 B = C, siendo:
2
C. Si el producto de dos D. 3 2 2
A= 1 3 B=
matrices es 0, enton- 2 2 5
ces al menos una de E. 0 6 5
y C= 4 9
ellas debe ser cero. 18. La expresin
D. Todas las matrices 1 a b A. a = 4 b= 3
b a
es igual a:
tienen inverso adi- ab B. a = 4 b= 3
tivo. A. a b C. a = 6 b= 3
E. Si M es de orden B. a + b D. a = 4 b=3
3 4 y N es de orden 1 E. a = 6 b=3
3 5, entonces MN C. ab
es de orden 4 5. 1 22. El valor de
D. 1 m
a+b m
15. La condicin que E. a2 b2 det 0 m 1 m es:
0 0 m+1
debe cumplir k para
que el sistema 2 3 A. 0
3 1 B. m3
2kx+ y = 9 19. Calcule: =
3 1
3x+ 2y = 5 2 3
C. m3 1
tenga solucin es: A. 1 D. m3 m
A. k = 3 E. m2 1
4 B. 1
B. k 3 23. La matriz principal (o
C. 11
4 matriz de los coeficien-
3 D. 11 tes) del sistema
C. k
4
x + y + z = 1 es:
D. k 4 E. Otro
3 x y z = 2
E. k = 4 20. El valor de m que x + y z = 1
3 hace ver dadera la 1 1 1
2 3 5 igualdad en: A. 1 1 +1
16. Sea A = 1 2 1
,
m+2 1 1 1
entonces det (A) = [m 2 m + 2 2m] m 2 1 1 1
A. 10 m B. 1 1 1
= [ 8 m] es: 1 1 1
B. 10 1 1 1
A. 1
C. 0 C. 1 1 1
B. 1 1 1 1
D. Otro valor
C. 8 1 1 1
E. No existe D. 2 1 1
D. 8 1 1 1
17. El valor de m en : E. 0 1 1 1
m m+1 E. 1 2 1
2 5
= 0 es:
1 1 1

470 Matrices y determinantes

470-472 470 20/11/02, 6:41 PM


CAPTULO
CAPTULO 11

24. Si A = 1 2 N una matriz de orden de 30. En la operacin


1 2
5 3. Entonces el producto 2 1 1 3 a a
1 2 + = 11 12
B= , N M es de orden: 0 3 2 1 a21 a22
2 2
entonces A. 3 3 el valor de a21 a11 es:
2 (A + B) 2 (A B) = B. 4 5 A. 5

2 4 C. 5 4 B. 5
A. 4
2
4
D. 20 C. 3
2 4
B. D. 3
2 4 E. No se puede efectuar
4 0 1 E. 1
C.
6 0 2 28.Sea A una matriz
invertible y sea B = A1 31. El valor de 2a en
D. 4 0 se puede afirmar:
6 4 3 2a
4 5
= 23 es:
2 0 2 I. A = B1
E. 3 4 A. 1
II. Todos los eleme-
tos de A son B. 2
25. Sea A = 10 00
distintos de cero C. 1
0 1
B= 0 0 III. det (A) 0
D. 2
0 0 0 0 A. Slo I
C= 1 0
D= 0 1
, E. Otro
entonces la matriz B. I y II
2 3 32. El valor de
M= se puede C. Slo III
5 4 2 1 3 1
=
expresar por: D. I y III 1 3 2 1

A. 2A + 3B + 5C 4D E. Todos A. 1
B. 35
B. 2A 3B + 5C 4D
29. Los valores de las
C. 25
C. 2A + 3B + 5C 4D variables x, y, z para
que la igualdad se D. 35
D. 2A 3B + 5C + 4D
cumpla son respec- E. 25
E. 2A + 3B + 5C + 4D tivamente:
33. Sea A = (aij) dada
26. 1 m m 1 = [3x z y+2 x + y] =
1 m 1 m 3 2a 2b
[8 2y + 8 2x] por: A = 2b a 3b
A. (m + 1)2 a b ab
A. 2 2 10
B. (m 1)2 3
Si x = a21 a23 + 2a13 ,
8 entonces x =
C. m2 + 2m B. 2 8
3 A. 2a b
D. 2m m2 1 C. 2
2 10
3 B. b
E. (m 1)2
D. No se pueden deter- C. b
27. Sea M una matriz minar D. 2a + b
de orden 3 4 y E. Otros E. Otro

Matrices y determinantes 471

470-472 471 20/11/02, 6:42 PM


Prueba de seleccin mltiple
34. El valor de k en 38. Sea
3 2 = 2 es:
k+2 k1 M= 0 2
N= a b
2 0 0 1
A. 1 B. 1 3 4
para que N M =
33 1 2 0
C. D.
5 5 los valores de a y b son
E. 5 respectivamente:
35. El valor de k para que la matriz A A. 2 ; 3 B. 2 ; 3
2 2
no sea invertible debe ser: 3 3
3 0 0
C. 2 ; D. 2 ;
2 2
1 3k 0 E. Otros
A= 1
3 2k 1
2k
3 39. De las siguientes proposiciones,
A. 0 B. 1 si A, B y C son matrices de
C. 1 D. 3 orden n n, A y B invertibles, es
E. 3 verdadero:
A. (A + B) C = C (A + B)
36. A partir de la siguiente igualdad:
B. A + (B C) = (A + B)
u 2v 2 2 3 3
x 2y
= (A + C)
1 3 1 2
es falso: C. (A B) 1 = B 1 A 1
A. u=4 B. v = 5 D. (A + B) (A B) = A2 + B2
C. x=6 D. 2y = 3 E. A B = B A
E. 2u + v = 3
40. 5 3 2 1 3 2 1
37. El resultado de: +
2 1 6 2 1 6 3
1 2 1 2 1 2

1 3 2 2 2 1 es 2
1
A.
10 0
B. 0 0 A. 3 B. 1 0
7 11 1 0 1
3 0 2
3
C. 6 2 D. 6 2
9 6
1 3 1 3 C. 3 18
D. 1 1
1 1
4 2
E. 3 2
3 9 E.
1 6

Soluciones

1. B 6. B 11. A 16. E 21. C 26. B 31. B 36. B


2. B 7. C 12. B 17. A 22. D 27. C 32. D 37. D
3. C 8. D 13. B 18. B 23. C 28. D 33. C 38. C
4. D 9. A 14. D 19. B 24. A 29. A 34. D 39. C
5. C 10. A 15. B 20. A 25. B 30. B 35. A 40. E

472 Matrices y determinantes

470-472 472 20/11/02, 6:44 PM


CAPTULO 12
S umatoria
y progresiones

Sumatoria 12.1

Definicin: Sea 1, 2, 3, 4..., n E N los trminos de una sucesin.

Definimos el smbolo i de la siguiente forma:


1

i=1
i = 1
n + 1 n

i=1
i = i
i=1
+ n +1
n
El smbolo i se lee: sumatoria de los i desde 1 hasta n.
i=1

Ejemplo:
5 4 3
i = i
i=1 =1
i + 5 = i + 4 + 5
i=1
2
= i + 3 + 4 + 5
i=1
1
= i + 2 + 3 + 4 + 5
i=1

= 1 + 2 + 3 + 4 + 5

Propiedades:
n n n
1. (i + vi) = i
i=1 =1
i + vi
i=1
n
2. k=nk
i=1
k = constante

Sumatoria y progresiones 473

473 473 10/11/2001, 11:52


n n
3. k i = k i= 1 i
i=1
k = constante

n
4. (i i 1) = n o (propiedad telescpica)
i=1

5. Algunas sumas importantes y de uso frecuente:


n
a) i = 1 + 2 + 3 + ... + n =
i=1
n(n +1)
2
n
n n + 1 2n + 1
b) i2 = 1 + 4 + 9 + ... + n2 =
i=1 6
n 2

i3 = 1 + 8 + 27 + ... + n3 =
n n + 1
c)
i=1 2

Observacin:
Sean m y n nmeros naturales tales que m n, entonces:
n n m 1


i=m
i =
i=1
i
i=1
i

Ejercicios 1. Desarrollar las sumatorias:


n
resueltos a) ( 1)i 1 (i2 + 1) b)
k 1
i=1 k=1 k + 1

Solucin:
a) 2 5 + 10 17 + 26 ... + ( 1)n 1 (n2 + 1)
1 2 3 4
b) 0 + + + + +...
3 4 5 6
12
2. Calcular el valor de (i 1) (i + 1)
i=1

Solucin:
(i 1) (i + 1) = i2 1
12 12 12 12
(i 1) (i + 1) = i= 1 (i2 1) = i= 1 i2 i= 1 1
i=1

12 12 + 1 2 12 + 1
= 1 12
6

= 12 13 25

12 = 638
6
Se aplic la frmula
n
n n + 1 2n + 1
i2 = 6
i=1

474 Sumatoria y progresiones

474-475 474 10/11/2001, 18:33


CAPTULO 12

3. Calcular la suma de los n primeros trminos de:


1 6 + 2 7 + 3 8 + 4 9 +...
Solucin:
Observando los trminos de la suma nos damos cuenta que
el trmino general es ai = i (i + 5); por lo tanto, la suma se
expresa como:
n n
i
i=1
i+5 =
i=1
i2 + 5 i

n n
= i2 + 5 i
i=1 =1
i

= n n+1 2n + 1 n n+1
+5
6 2
3 2
= n +9n +8n
3
12
4. Calcular el valor de:
k=5
k+1 2k3

Solucin:
n n m 1
Sabemos que: k = k= 1 k k= 1 k
k=m

y (k + 1) (2 k 3) = 2 k2 k 3, luego
12 12 4

k=5
k+1 2k3 =
k=1
2 k2 k 3
k=1
2 k2 k 3

12 12 12 4 4 4
=2 k2 k= 1 k k= 1 3 2 k= 1 k2 + k=1 k + k= 1 3
k=1

12 13 25 12 13 459 45
= 2 3 12 2 + +34 =
6 2 6 2
= 1.300 78 36 60 + 10 + 12 = 1.148
12
\ k + 1 2 k 3 = 1.148
k=5

n
n2 + 3 n
5. Si i =
i=1 2
, hallar i

Solucin:
n n1
n = i i
i=1 =1
i

2
n2 + 3 n n1 +3 n1
=
2 2
n2 + 3 n n2 + 2 n 1 3 n + 3 2n+2
= = =n+1
2 2
Como n = n + 1, entonces i = i + 1

Sumatoria y progresiones 475

474-475 475 10/11/2001, 18:33


n
6. (ai ai1 ) = (a1 a0) + (a2 a1) + (a3 a2) + .... + (an an1)
i=1

Eliminando el parntesis y asociando en forma adecuada


obtenemos:

= an a0

n n n
O bien ai ai 1 = i
i=1 =1
ai ai 1
i=1

= (a1 + a2 + a3 + ... + an1 + an) (a0 + a1 + a2 + ... + an1)

Se cancelan todos los elementos, excepto el ltimo del


primer parntesis y el primero del segundo, y nos queda:

n
ai ai 1 = an a0
i=1

7. Calcular k2 (k 1)2
k=1

Al desarrollar nos queda: (1 0 ) + (22 1) + (32 22)


+ ... + (n2 (n 1)2)

Aplicando la propiedad telescpica, donde:


n

i=1
ai ai 1 = an a0 nos queda inmediatamente:

k2 (k 1)2 = n2 02
k=1

Comprobemos aplicando las propiedades anteriores.

n n

k2 (k 1)2 = k2 (k2 2k + 1) =
k=1 k=1

n n n

2k 1 = 2 k 1
k=1 k=1 k=1

2n (n + 1)
= n
2

476 Sumatoria y progresiones

476-480 476 25/11/02, 12:32 PM


CAPTULO 12

= n2 + n n

= n2

Obviamente el uso de la propiedad telescpica es mucho ms


directo en este caso para calcular esa sumatoria.

n
1
8. Calculemos k(k + 1)
k=1

Aplicaremos (y explicaremos) el mtodo de fracciones parciales


para solucionar problemas de este tipo.

El mtodo consiste en expresar la fraccin original como suma


de 2 fracciones, como sigue:

1 A B
= +
k(k + 1) k k+1

Debemos determinar A y B.

1 A(k + 1) + Bk k(A + B) + A
= =
k(k + 1) k(k + 1) k(k + 1)

Debemos igualar los numeradores.

A=1
A+B=0 } B = 1

1 1 1
Entonces: =
k(k + 1) k k+1

Y as la sumatoria pedida se transforma en:


k=1
( 1
k

1
k+1
= 1
1
2
+
) ( ) (
1
2

1
3
+
)
( 1
3

1
4 )
+ ... +
1
n

1
n+1

Sumatoria y progresiones 477

476-480 477 25/11/02, 12:32 PM


Cada trmino de la sumatoria es la diferencia de dos elementos
consecutivos y vemos que al desarrollarla se cancelan todos, excepto
el primero y el ltimo, por la propiedad telescpica.

As:

n n
1 1 1 1 n
k(k + 1)
= k

k+1
=1
n+1
=
n+1
k=1 k=1

9. Usemos el mtodo anterior para calcular.

n
1
(2k 1)(2k + 1)
k=1

Primero expresamos en fracciones parciales:

1 A B
= +
(2k 1)(2k + 1) 2k 1 2k + 1

1 A(2k + 1) + B(2k 1)
=
(2k 1)(2k + 1) (2k 1)(2k + 1)

1 2Ak + A + 2Bk B
=
(2k 1)(2k + 1) (2k 1)(2k + 1)

1 = k(2A + 2B) + ( A B) (igualando numeradores)

2A + 2B = 0 (Coeficiente de k no existe al lado izquierdo)

AB=1

AB=1 1 1
A+B=0 2A = 1 A = y B=
2 2

478 Sumatoria y progresiones

476-480 478 25/11/02, 12:33 PM


CAPTULO 12

As, la suma pedida se transforma en:

n 1 1 n
1 1 1 1

2 2
= =
k=1 2k 1 2k + 1 k=1
2 (2k 1) 2 (2k + 1)

n
1 1 1
2

k=1
(
(2k 1) (2k + 1) )
Y nuevamente podemos aplicar la propiedad telescpica, ya que
cada trmino de la sumatoria es la diferencia de dos elementos
consecutivos; por lo que, al desarrollar sta se cancelan casi
todos los trminos. Nos queda

n n
1 1 1 1

k=1
( (2k 1)(2k + 1) ) =
2

k=1
(
(2k 1) (2k + 1) )
1 1 1 1
=
2 [( 1
3 ) (
+
3

5 ) + ...

1 1
+
( 2n 1

2n + 1 )]
1 1
=
2
1
(
2n + 1 )
1 2n
=
2 ( 2n + 1 )
n
=
2n + 1

Sumatoria y progresiones 479

476-480 479 25/11/02, 12:34 PM


n

10. Calculemos k3 (k 1)3


k=1

Podemos aplicar directamente la propiedad telescpica y


obtenemos:

k3 (k 1)3 = n3
k=1

11. A partir de esta igualdad, podemos deducir la sumatoria de


los trminos al cuadrado. Observa:
n

k3 (k3 3k2 + 3k 1) = n3
k=1

3k2 3k + 1 = n3
k=1

n n n

3 k2 3 k+ 1 = n3
k=1 k=1 k=1

n
n(n + 1)
3 k2 = n3 + 3
2
n
k=1

2n3 + 3n2 + 3n 2n
=
2

n
n(2n2 + 3n + 1)
3 k2 =
2
k=1

n
n(n + 1) (2n + 1)
k2 =
6
k=1

Aqu obtuvimos la sumatoria de los cuadrados de los


nmeros desde 1 hasta n.

480 Sumatoria y progresiones

476-480 480 25/11/02, 12:35 PM


CAPTULO 12

Ejercicios
20
1. Desarrollar las siguientes sumatorias: c) (i2 i) =
i=1
6 5 15
1
a)
i=1 i
= b)
i=1
(2i 1) = d) (i 2i2 + i3) =
i=1
8 4
c) i2 = d) i3 = 5. Calcular el valor de las siguientes
i=1 i=1 sumas:
5 10
1
e) (-1)ii = f) (1)i 1 = 30
i=1 i=1 i a) k(k 3)
n n k = 20

g) 2i = h) 1 = k = 36
i=1 i=1 2i 1 b) (1 2k)
n n k = 12
1
i)
i=1
(-1)ii2 = j)
i=1
(1)n (1+ =
i ) c)
k=8
(k3 k2 + 1)
k=5
2. Escribir en forma de sumatoria las k = 10
siguientes series: d) 2k(1 k)
k=8
2 1
a) 1 + + + 4 6. Calcular el trmino general y el valor
3 9 27
de los n primeros trminos de las
1 2 3 4 5
b) + + + + siguientes sumas:
2 3 4 5 6
a) 2 3 + 3 4 + 4 5 + 5 6
1 1 1 1 1 1
c) + + + + + + 6 7 + ...
2 3 4 5 6 7
1 1 1 1 b) 3 + 5 + 7 + 9 + ...
d) + + +
3 6 9 12 c) 1 4 + 2 5 + 3 6 + 4 7 + ...
3. Escribir en forma de sumatoria las d) 1 2 + 4 3 + 9 4 + 25 5 + 36
siguientes series: 6 + 49 7 + ...
2 3 4 7. Dada la suma, hallar el trmino
a) 1 + + + + ...
3 5 7 general i:
4 6 8 10 n
b) 2 +
3
+ + +
5 7 9
+ ... a) i =
i=1
2n3 + 3n2 5n
6
n
4. Calcular las siguientes sumas:
5
b)
i=1

i = n3 + 2n
3
a) (2i3 i + 3) = n
i=1
7
c) i
i=1
= n2 + 4n
b) (3i3 2i2 + i 6) = n
i
2
i=1
d) = 3n + 7n
i=1 2

Soluciones
1 1 1 1 1 1 1 1 1
f) 1 + + + +
1 1 1 1 1 2 3 4 5 6 7 8 9 10
1. a) 1 + + + + +
2 3 4 5 6 g) 2 + 4 + 6 + 8 + 10 + ... + 2n
b) 1 + 3 + 5 + 7 + 9
1 1 1 1
c) 1 + 4 + 9 + 16 + 25 + 36 + 49 + 64 h) 1 + + + ... +
3 5 7 2n 1
d) 1 + 8 + 27 + 64 i) 1 + 4 9 + 16 25 + ... + 1nn2
3 4 5 6 1
e) 1 + 2 3 + 4 5 j) 2 + + + + + ... + ( 1)n (1 + )
2 3 4 5 n

Sumatoria y progresiones 481

481-483 481 25/11/02, 12:37 PM


4 5 6 4
i i 1 1
2. a) b) c) (1)i d)
i = 1 3i 1 i=1 i+1 i=1 i+1 i=1 3i

k 2k 4. a) 450 b) 2.058 c) 2.660 d) 12.040
3. a) b)
k=1 2k 1 k=1 2k 1
5. a) 6.160 b) 1.175 c) 1.026 d) 436
n
3 2
6. a) uk = (k + 1) (k + 2) ; (k + 1) (k + 2) = n + 6n + 11n
k=1
3
n
b) uk = 2k + 1 ; (2k + 1) = n2 + 2n
k=1
n
3 2
c) uk = k(k + 3) ; k(k + 3) = n + 6n + 5n
k=1
3
n
d) uk = k2(k + 1) ; k2(k + 1) = 3n4 + 10n3 + 9n2 + 2n
k=1
12
7. a) ui = i2 1 b) ui = i2 i + 1 c) ui = 2i + 3 d) ui = 3i + 2

12.2 Sucesiones

12.2.1 Denicin
Una sucesin es una funcin cuyo dominio es el conjunto de
nmeros naturales.
En general, se denota de la siguiente manera:
{an} = a1, a2, a3, ..., an,....
El trmino general de la sucesin es an; el subndice indica el lugar
que ocupa el trmino en la sucesin.

Ejemplos.
Determinar los cinco primeros trminos de la sucesin cuyo
trmino general es:
1. an = n
{an} = 1, 2, 3, 4, 5, ...

2. an = 1
n
{an} = 1, 1 , 1 , 1 , 1 , ...
2 3 4 5

482 Sumatoria y progresiones

481-483 482 25/11/02, 12:37 PM


CAPTULO 12

3. an = (1)n
{an} = 1, 1, 1, 1, 1, ...
n
4. an = (1)n
n+1
{an} = 1 , 2 , 3 , 4 , 5 , ...
2 3 4 5 6

Determinar el trmino general de las siguientes sucesiones:


Ejercicios
1) 1, 3, 5, 7, 9, ...
resueltos
an = 2n 1 (nmeros impares)

1 1 1 1
2) 1, , , , , ...
4 9 16 25
1
an = (numerador 1, denominador son
n2
los nmeros al cuadrado)

1 2 3 4 5
3) , , , , , ...
3 5 7 9 11
n
an = (el numerador es n, los
2n + 1
denominadores son impares, a
contar del 3)

1 1 1 1 1
4) , , , , , ...
2 4 8 16 32

1
an = (-1)n + 1 (los signos van intercalados,
2n
comenzando por +; los
numeradores son 1 y los
denominadores son potencias
de 2)
5) 2, 5, 8, 11, 14, ...
an = 3n 1 (la diferencia entre cada par de
trminos consecutivos es 3, y
parten del 2, que es 3 1)

1 3 5 7 9
6) , , , , , ...
3 5 7 9 11
2n 1
an = (los numeradores son impares
2n + 1
partiendo por el 1 y los
denominadores son impares
partiendo por el 3)

Sumatoria y progresiones 483

481-483 483 25/11/02, 12:37 PM


Ejercicios
1. Encuentra los cinco primeros trminos 2. Encuentra el trmino general de las
de las sucesiones cuyos trminos siguientes sucesiones:
generales son:
a) 3, 7, 11, 15, 19, ...
n3
a) an =
n2 b) 3, 9, 27, 81, 243, ...
n2
b) an = 3 5 7 9 11
n+1 c) , , , , , ...
4 7 10 13 16
2n + 3
c) an =
2n 1
d) 1, 1, 1, 1, 1, ...
n2 1
d) an =
n2 + 1 1 1 1 1 1
e) , , , , , ...
e) an = (1)n
2n a a2 a3 a4 a5
2n + 1

Soluciones

1. a) 0 , 1 , 2 , 3 , 4 2. a) an = 4n 1
2 3 4 5
b) 1 , 4 , 9 , 16 , 25 b) an = (-1)n 3n
2 3 4 5 6
2n + 1
c) 5 , 7 , 9 , 11 , 13 c) an =
3n + 1
1 3 5 7 9
d) 0 3
, , 8 , 15 , 24 d) an = (1)n + 1
2 5 10 17 26
e) 2 , 4 , 6 , 8 , 10 e) an = 1
3 5 7 9 11 an

12.2.2 Sucesiones convergentes


Una sucesin {an} es convergente si sus trminos se van
acercando cada vez ms a un cierto valor. Ese valor se llama
el lmite de la sucesin y se dice que la sucesin converge
a ese lmite.

Ejercicios 1. Tomemos la siguiente sucesin: an =


1
n
resueltos 1, 1 , 1 , 1 , 1 , ...
2 3 4 5
Los trminos de esta sucesin se van acercando cada vez ms a
cero, aunque nunca lo alcancen.

484 Sumatoria y progresiones

484-485 484 25/11/02, 12:38 PM


CAPTULO 12

1
La sucesin an = es convergente y su lmite es cero.
n
n
2. Tomemos la siguiente sucesin: an =
n+1
1 , 2 , 3 , 4 , 5 , ...
2 3 4 5 6
Los trminos de esta sucesin se van acercando cada vez ms a
1, aunque nunca lo alcancen.
n
La sucesin an = es convergente y su lmite es 1.
n+1
3. Consideremos la siguiente sucesin an = 2
2, 2, 2, 2, 2, ...
Todos los trminos son iguales a 2, esta sucesin es convergente
y su lmite es 2.

12.2.3 Sucesiones divergentes


Si una sucesin no es convergente se dice que es divergente.
Una sucesin puede divergir porque sus trminos oscilan o bien
porque sus trminos crecen o decrecen sin medida.
Una sucesin oscilante tambin puede ser convergente.

1. Consideremos la siguiente sucesin an = (1)n Ejercicios


1, 1, 1, 1, 1, .... resueltos
En este caso los trminos oscilan y no se acercan a ningn
valor; decimos que esta sucesin diverge.
2. Consideremos la sucesin an = 2n 1
1, 3, 5, 7, 9, ...
Los trminos de esta sucesin se van haciendo cada vez ms
grandes, no se acercan a ningn valor y por lo tanto esta
sucesin tambin diverge.
1
3. Consideremos la sucesin an = (-1)n + 1
2n
1 , 1 , 1 , 1 , 1 , ...
2 4 6 8 10
En este caso los trminos oscilan pero se van acercando a cero;
esta sucesin converge y su lmite es cero.

Sumatoria y progresiones 485

484-485 485 25/11/02, 12:38 PM


12.2.4 Sucesiones crecientes
y decrecientes
Una sucesin es creciente si cada trmino es mayor que el trmino
anterior; es decir an > an 1
Una sucesin es decreciente si cada trmino es menor que el
anterior an < an 1

Ejercicios
1
resueltos 1. La sucesin an = 2
n

1, 3 , 5 , 7 , 9 , 11 , ...
2 3 4 5 6

es una sucesin creciente, pero sus trminos no crecen


indefinidamente; ninguno de ellos es mayor que 2.
Esta es una sucesin convergente y su lmite es 2.

2n + 1
2. La sucesin an =
3
3 , 5 , 7 , 9 , 11 , 13 , ...
3 3 3 3 3 3

es una sucesin creciente y sus trminos crecen indefinidamente.


Esta es una sucesin divergente.
3. La sucesin an = 1 3n
2, 5, 8, 11, 14, ....
es una sucesin decreciente y sus trminos decrecen sin
medida. No converge.

1
4. La sucesin an =
2n 1

1, 1 , 1 , 1 , 1 , ...
3 5 7 9

es una sucesin decreciente, ya que el denominador crece sin


medida mientras que el numerador se mantiene constante. Esta
sucesin converge y su lmite es cero.

Nota: En muchos casos, para determinar la convergencia o


divergencia de una situacin es conveniente calcular los
trminos de ella para valores muy grandes de n.

486 Sumatoria y progresiones

486-487 486 25/11/02, 12:40 PM


CAPTULO 12

Ejercicios
1. Analiza las siguientes sucesiones 2) a) Escribe 3 sucesiones crecientes
determinando cules de ellas son que sean convergentes.
crecientes, decrecientes, oscilantes,
convergentes o divergentes. b) Escribe 3 sucesiones crecientes
que sean divergentes.
a) an = n + 2
b) an = 6 12n c) Escribe 3 sucesiones decrecientes
que sean convergentes.
3
c) an =
n+3
d) Escribe 3 sucesiones decrecientes
3n 1 que sean divergentes.
d) an =
n1
1 e) Escribe 3 sucesiones oscilantes
e) an = 1 que sean convergentes.
n
f) an = (1)n n f) Escribe 3 sucesiones oscilantes
n que sean divergentes.
g) an = (1)2n
n+1
(n + 1)2 3) Conjetura hiptesis sobre:
h) an = (1)n + 1
n2
a) Condicin necesaria (y suficiente)
n2
i) an = (1)n para que una sucesin creciente
(n + 1)2
sea convergente.
1
j) an = 1 + (compare con ej. e)
n b) Condicin necesaria (y suficiente)
n para que una sucesin decreciente
k) an = sea convergente.
n2 + 1

l) an =
{ n si n es par.
n si n es impar.
c) Averigua el significado del con-
cepto de sucesiones acotadas y
da ejemplos de ella.

Soluciones

1) a) creciente, diverge h) oscila, converge


b) decreciente, diverge i) oscila, converge
c) decreciente, converge j) decrece, converge (al mismo valor
d) creciente, converge que sucesin e)

e) creciente, converge k) decreciente, converge

f) oscila, diverge l) oscila, diverge.

g) oscila, converge

Sumatoria y progresiones 487

486-487 487 25/11/02, 12:40 PM


12.3 Progresin aritmtica

Definicin: Una Progresin Aritmtica (P.A.) es una sucesin


de trminos a1, a2, a3, ..., an, ... tal que cada uno se obtiene
de sumar un valor constante al anterior.

a1 : primer trmino de la P.A.

d : diferencia de la P.A.

an : trmino ensimo de la P.A.

n : nmero de trminos de la P.A.

Sn : suma de n trminos de la P.A.

Frmulas en una P.A.

an = a1 + (n 1) d

d = an an 1

n n
Sn = (a + an) = [2a1 + (n 1) d]
2 1 2

Los trminos de una P.A. que se encuentran entre dos trminos dados
se llaman medios aritmticos y el procedimiento para hallarlos se
denomina interpolacin de medio aritmticos.

1. Calcular el trmino que ocupa el lugar 50 en una P. A. si el primero


Ejercicios es 5 y la diferencia es 2.
resueltos Solucin:
a1 = 5 ; d=2 ; n = 50 ; an = ?
an = a1 + (n 1) d
a50 = 5 + 49 2 = 103
a50 = 103

488 Sumatoria y progresiones

488-489 488 25/11/02, 12:42 PM


CAPTULO 12

2. El undcimo trmino de una P. A. es 49 y su diferencia es 4.


Encontrar el primer trmino.
Solucin:
a11 = 49 ; d=4 ; a1 = ?
an = a1 + (n 1) d
49 = a1 + (11 1) 4
a1 = 49 40
a1 = 9

3. El primer trmino de una P. A. es 5, su diferencia es 4 y el trmino


ensimo es 53. Hallar el nmero de trminos.
Solucin:
a1 = 5 ; d=4 ; an = 53 ; n=?
an = a1 + (n 1) d
53 = 5 + (n 1) 4
4n = 52
n = 13

4. Determinar la P. A. cuyo quinto trmino es 14 y cuyo dcimo


trmino es 29.
Solucin:
Conocer una P. A. significa determinar su primer trmino
y su diferencia.
a5 = 14 ; a10 = 29
como an = a1 + (n 1) d
se tiene 14 = a1 + 4d
29 = a1 + 9d
Resolviendo el sistema:
d=3 y a1 = 2 luego la P. A. es:
2, 5, 8, 11, 14, 17, 20, 23, 26, 29, 32,...

5. En la P. A. tal que su sexto trmino es 15 y la diferencia es 3 ,


5
hallar el trmino del lugar 16.
Solucin:
3
a6 = 15 ; d= ; a16 = ?
5
a6 = a1 + 5d 15 = a1 + 3 a1 = 12
a16= a1 + 15d a16 = 12 + 9 a16 = 21

Sumatoria y progresiones 489

488-489 489 25/11/02, 12:42 PM


6. Interpolar 5 medios aritmticos entre los nmeros 3 y 6.
Solucin:
Se est pidiendo formar una P. A. de 7 trminos donde el primero
es 3 y el sptimo es 6. Se debe determinar la diferencia y con ello
calcular los 5 medios aritmticos pedidos:
a1 = 3 ; a7 = 6 ; a2, a3.... a6 = ?
a7 = a1 + 6d
1
6 = 3 + 6d d =
2
7
Luego: a2 = , a3 = 4, a4 = 9 , a5 = 5, a6 = 11
2 2 2

Son los cinco medios aritmticos pedidos.


7. Hallar tres nmeros que estn en P. A. y cuya suma sea 39.
Solucin:
Sean a1 , a2 y a3 los nmeros pedidos.
Hagamos a1 = a2 d y a3 = a2 + d.
Luego: (a2 d) + a2+ (a2 + d) = 39
3 a2 = 39
a2 = 13
a1 + a2 + a3 = 39
a1 + a3 = 26
Si a1 = 1, entonces a3 = 25 y los tres nmeros pedidos son 1, 13 y 25.
Si a2 = 2, entonces a3 = 24 y los tres nmeros pedidos son 2, 13 y 24.
Luego, este problema tiene infinitas soluciones. Los tros de
nmeros naturales que satisfacen la condicin son:
1, 13 y 25 6, 13 y 20 11, 13 y 15
2, 13 y 24 7, 13 y 19 12, 13 y 14
3, 13 y 23 8, 13 y 18 13, 13 y 13
4, 13 y 22 9, 13 y 17
5, 13 y 21 10, 13 y 16
8. Determinar una P. A. sabiendo que la suma del primer
y tercer trmino es 44 y el producto del segundo por el
primero es 418.
Solucin:
Sean a d, a , a + d los tres trminos de la P. A.

(a d) + (a + d) = 44 (1)
a (a d) = 418 (2)

De (1) 2a = 44 a = 22

490 Sumatoria y progresiones

490-491 490 25/11/02, 12:46 PM


CAPTULO 12

De (2) a2 ad = 418
484 22 d = 418
d=3
Luego a d = 22 3 = 19 y a + d = 22 + 3 = 25
La P.A. pedida es 19, 22, 25
9. Hallar la suma de los 100 primeros mltiplos de 3.

Solucin:
Observamos que los 100 primeros mltiplos de 3 equivalen
a la P.A. cuyo primer trmino es 3, la diferencia es 3 y tiene
100 trminos.
a1 = 3 ; d=3 ; n = 100
n
Sn = [2a1 + (n 1)d]
2
S100 = 50 (6 + 99 3) = 15.150
Luego, la suma de los 100 primeros mltiplos de 3 es 15.150.
10. Calcular cunto dinero tena para vacaciones un joven si el
primer da gast $ 7.000, fue disminuyendo el gasto en $200
diarios y el dinero le dur 30 das.

Solucin:
Vemos que la situacin planteada obecede a una P.A. donde el
primer trmino es 7.000, la diferencia es 200 y el nmero de
trminos es 30. El dinero con que el joven contaba para vacaciones
es la suma de los 30 trminos.
a1 = 7.000 ; d = 200 ; n = 30
n
Sn = [2a1 + (n 1)d]
2
S30 = 15 (14.000 + 29 (200)) = 123.000
El joven tena $123.000 para vacaciones.
11. Para hacer un tnel se sabe que el primer metro tiene un
costo de $1.000.000 y por cada metro ms se debe agregar
$80.000. Cul es el largo del tnel si se ha debido cancelar
$33.264.000.000?

Solucin:
Podemos pensar esta obra como una P.A. en que el primer trmino
es 1.000.000, la diferencia es 80.000 y la suma del costo de los n
metros que hubo que hacer es 33.264.000.000.
a1 = 1.000.000 ; d = 80.000 ; Sn= 33.264.000.000 ; n=?
n
Sn = [2a1 + (n 1)d]
2
33.264.000.000 = n [2.000.000 + 80.000 n 80.000]
2

Sumatoria y progresiones 491

490-491 491 25/11/02, 12:46 PM


66.528.000.000 = 1.920.000 n + 80.000 n2
8n2 + 192 n 6.652.800 = 0 /: 8
n2 + 24 n 831.600 = 0
n1 = 900
n = 24 3.326.976 =
2 n2 = 924

Luego el tnel tiene 900 m de largo.

Ejercicios 10. Hallar la suma de los 10 primeros


trminos de las siguientes sucesio-
1. Dadas las progresiones aritmticas nes:
siguientes, hallar el trmino que se a) 1, 2, 3,...
indica:
b) 2, 4, 6,...
a) 5, 8, 11, 14... a10 c) 5, 2, 1,...
1
d) 1, , 0,...
b) 5, 2, 1, 4... a20 2
1 7 11. Encontrar la suma de una P.A. de
c) , 2, , 5... a12 20 trminos si el primero es 5 y el
2 2
ltimo es 62.
d) 3, 8, 13, 18... a15 12. Hallar la suma de los 15 primeros
trminos de una P.A. si se sabe que
2. Determinar cuntos trminos tiene
el quinto trmino es 17 y el sptimo
una P.A. si el primero es 5, el ltimo
es 23.
es 50 y la diferencia es 3.
13. Calcular la suma de los mltiplos de
3. Hallar la P.A. tal que la suma de los siete que estn entre 100 y 200.
20 primeros trminos es 120 y su
diferencia es 2. 14. En una P.A. cuya diferencia es 4 el
trmino central vale 21 (tiene un
4. Determinar la diferencia en una P.A. nmero impar de trminos). Si su
cuyo trmino de lugar 27 es 32 y suma es 189, encontrar el nmero de
cuyo trmino de lugar 18 es 5. Hallar trminos y escribir la progresin.
tambin el primer trmino.
15. Hallar tres nmeros enteros que estn
5. Calcular el trmino de lugar 100 en la en P.A., cuyo producto es 15.000 y
P.A. 6, 4, 2, 0, 2, .... cuya suma es 75.
6. En la P.A. 5, 8, 11, 14... determinar 16. Determinar cuntos trminos de la
qu lugar ocupa el trmino de valor P.A., 2, 6, 10,... hay que sumar para
65. obtener 288.
7. Interpolar cinco medios aritmticos 17. Al sumar nmeros pares consecutivos
entre 12 y 42. a partir del 10 se obtiene 580.
8. Interpolar tres medios aritmticos Cuntos nmeros se han sumado?
entre 12 y 12. 18. La suma de cinco nmeros que estn
9. Interpolar tres medios aritmticos en P.A. es 20 y su producto es 720.
entre 1 y 7. Hallar dichos nmeros.

492 Sumatoria y progresiones

492-493 492 25/11/02, 12:48 PM


CAPTULO 12

19. Encontrar tres nmeros sabiendo que 30. Para construir una va elevada se
estn en P.A., que su suma es 51 y levanta sobre una superficie hori-
el mayor es dos unidades menor que zontal una rampa de pendiente
el doble del menor. uniforme, la cual se sostiene sobre
12 soportes igualmente espaciados.
20. La suma de los nueve trminos de La altura del primer soporte es 2
una P.A. es 9. La diferencia entre m y la del ms alto es de 51,5 m.
el primer y el ltimo es 16. Hallar Encontrar la altura de cada soporte.
dicha progresin.
31. Las medidas de los lados de un
21. Hallar una P.A. de 8 trminos tringulo rectngulo forman una P.A.
sabiendo que los cuatro primeros y suman 45 m. Calcular cunto mide
suman 40 y que los cuatro ltimos cada uno.
suman 72. 32. Se deja caer una bola de goma, la
22. En una P.A. el segundo y el dcimo cual en el primer bote se levanta 1
trmino suman 44 y el primero mas m del suelo, en el segundo bote slo
el noveno suman 34. Hallar los 95 cm, en el tercer bote 90 cm y as
trminos mencionados. sucesivamente. Calcular cunto ha
recorrido la bola desde que toca por
23. Si las expresiones 5x + 3, 3x + 2 primera vez el suelo hasta que llega
y 2x 1 estn en P.A., hallar el al punto ms alto despus del dcimo
valor de x. bote. Cuntos botes alcanza a dar
24. Interpolar dos medios aritmticos antes de detenerse?
entre a y b. Dar un ejemplo si 33. Las medidas de los ngulos de un
a = 5 y b = 8. tringulo forman una P.A. Si uno
de ellos mide 20o, cunto miden
25. Determinar d en una P.A. de n + 2
los otros dos?
trminos si a1 = a y an+2 = b.
34. Cuntos medios aritmticos se
26. Interpolar 5 medios aritmticos entre deben interpolar entre 4 y 40 para
2 y 10. que la suma de la P.A. resultante sea
27. Encontrar la suma de los n primeros 220? Cules son esos medios?
nmeros naturales. 35. Un espectador de teatro ve bien a
28. Encontrar la suma de los p primeros 26 m del escenario. En qu fila
nmeros naturales pares. debe sentarse si la primera dista 8
m del escenario y la decimoctava,
29. Encontrar la suma de los q primeros que es la ltima, est a 42 m del
nmeros naturales impares. escenario?

Soluciones

1. a) 32 b) 52 c) 17 d) 73 8. Los medios son 6, 0 y 6.

2. 16 3. 13, 11, 9, 7... 5 11


9. Los medios son ,4 y
2 2
4. d = 3 ; a1 = 46 5. 192 6. 21 10. a) 55 b) 110 c) 85 d) 12.5
7. Los medios son 17, 22, 27, 32 y 37 11. 670 12. 390 13. 2.107

Sumatoria y progresiones 493

492-493 493 25/11/02, 12:48 PM


14. n = 9; 5, 9, 13, 17, 21, 25, 29, 33, 37 n(n + 1)
27. Sn = 28. p + p2 29. q2
15. 20, 25 y 30 16. 12 17. 20 2
30. 2, 6.5,11, 15.5, 20, 24.5, 29, 33.5, 38,
18. 2, 3, 4, 5, 6 19. 12, 17 y 22
42.5, 47, 51.5 m.
20. 9, 7, 5, 3, 1, 1, 3, 5, 7, (a1 = 9 ; d = 2)
31. 11.25 m, 15 m y 18.75 m.
21. a1 = 7 d = 2
22. a1 = 3, a2 = 2, a9 = 37, a10 = 42, 32. 14.95 m alcanza a dar 20 botes

33. 60o y 100o


2a + b a + 2b
23. x = 2 24. , ; 6, 7
3 3 34. Se deben interpolar 8 medios y son 8,
12, 16, 20, 24, 28, 32 y 36.
ba
25. d = 26. 0, 2, 4, 6, 8
n+1 35. Deben sentarse en la dcima fila.

12.4 Progresin geomtrica

12.4.1 Denicin
Una Progresin geomtrica (P.G.) es una sucesin de
trminos a1, a2, a3, ..., an, ... tal que cada uno se obtiene de
multiplicar el antecesor por un valor constante r.
a1 : primer trmino de la P.G.

r : razn de la P.G.

an : trmino ensimo de la P.G.

n : nmero de trminos de la P.G.

Sn : suma de n trminos de la P.G.

Pn : producto de n trminos de la P.G.

an = a1 rn 1
a
r= a n
n1

a1(1 rn) Si r = 1 entonces


Sn = , r 1 S n = n a1
1r
Si n y | r | < 1, entonces
a1
Sn =
1r

494 Sumatoria y progresiones

494-495 494 25/11/02, 12:52 PM


CAPTULO 12

Pn = (a1 an)n

Los trminos de una P.G. que se encuentran entre dos trminos dados
se llaman medios geomtricos y el procedimiento para encontrarlos
se denomina interpolacin de medios geomtricos.

12.4.2 Clculo de intereses


de capital
Una aplicacin directa de las progresiones geomtricas la
encontramos en los problemas financieros de intereses.

Un capital inicial C produce durante un ao al i% de inters


Ci
anual los intereses de .
100
Se llama inters compuesto si al cabo del ao ese dinero que
se obtuvo como inters de capital pasa a formar parte del capital y
produce inters para el ao siguiente.
Entonces en t aos el capital inicial C se convierte en un capital
final que llamaremos S y se calcula segn la frmula.

i t
S = C (1 + )
100
Por cada ao que pasa el capital es un trmino de una P.G.,
donde el primer trmino es el capital inicial C y la razn es
i
1+ .
100
NOTA: t representa el perodo que se acuerda para el clculo
y pago de los intereses. Puede ser anual, semestral,
trimestral o mensual.

1. Calcule el noveno trmino de una P.G. cuyo primer trmino Ejercicios


es 1 y la razn es 3.
Solucin:
resueltos
an = a1 r n1 a1 = 1 ; r = 3 ; a9 = ?
a9 = 1 38 = 6.561
2. El quinto trmino de una P.G. es 162 y el primero es 2. Hallar
la razn.
Solucin:
an = a1 r n1 a1 = 2 ; a5 = 162 ; r = ?
4 4
162 = 2 r r = 81
r = 481
r =3

Sumatoria y progresiones 495

494-495 495 25/11/02, 12:52 PM


3. El sptimo trmino de una P.G. es 192 y la razn es 2. Hallar
el primer trmino:
Solucin:
an = a1 r n1
a7 = 192 ; r = 2 ; a1 = ?
192
192 = a1 26 a1 = =3
64
1
4. En una P.G. el primer trmino es 32 y la razn es . Determinar
2
1
qu lugar ocupa el trmino que vale .
8
Solucin:
an = a1 r n1
1 1
an = ; a1 = 32 ; r = ; n=?
8 2

( )
n1
1 1
= 32
8 2
23 = 25 21 n
23 = 26 n

3 = 6 n

n=9
1
El trmino ocupa el noveno lugar de la P.G. dada.
8
5. Demostrar que la suma de n trminos de una P.G. cuyo primer
trmino es a1 y cuya razn es r dada por la frmula
a (1 rn)
Sn = 1 Si |r| < 1 y n tiende a infinito, entonces la
1r a1
suma de infinitos trminos de la P.G. es S =
1r.
Solucin:
Los trminos de la P.G. son a1, a2 = a1 r
_
a3 = a1 r2, a4 = a1 r3 , ... an = a1 rn 1 , entonces
Sn = a1 + a1 r + a1 r2 + ... + a1 rn1 /r
2 n1
r Sn = a1 r + a1 r + ... + a1 r + a1 r n

restando miembro a miembro.


Sn r Sn = a1 a1 rn
Sn (1 r) = a1 (1 rn)
a (1 rn)
Sn = 1
1r
Si |r| < 1 entonces rn se hace cada vez ms chico a medida que
aumenta n. Por ejemplo si r = 0,5
r2 = (0,5)2 = 0,25 r7 = (0,5)7 = 0,0078125

r3 = (0,5)3 = 0,125

r4 = (0,5)4 = 0,0625
r15 = (0,5)15 = 0,0000305
5 5
r = (0,5) = 0,03125
r6 = (0,5)6 = 0,015625

496 Sumatoria y progresiones

496-497 496 10/11/2001, 12:22


CAPTULO 12

es decir, si n es muy grande, rn se hace muy pequeo y se puede


despreciar su valor por lo que en ese caso rn tiende a cero y:
a1
S=
1r
6. Hallar la suma de los 10 primeros trminos de la P.G. 4,
12, 36, 108...
Solucin:
36
a1 = 4 ; r = = 3 ; n = 10
12
a1(1 rn)
Sn =
1r
4(1 310) 4(1 59.049)
S10 = = = 118.096
13 2
7. Calcular la suma de infinitos trminos de la P.G.
1, 1 , 1 , 1 , 1 , ...
2 4 8 16
Solucin:
1
a1 = 1 ; r = <1 ; n
2
a1
S=
1r
1 1
S= = =2
1 1 1
2 2
8. Demostrar que el producto de n trminos de una P.G. es
Pn = (a1 an)n
Solucin:
Sea la P.G. cuyo primer trmino es a1, y la razn es r, entonces
los trminos siguientes son:
a2 = a1r
a3 = a1r2
a4 = a1r3


an = a1r n1
Pn = a1 a1 r a1 r2 .... a1 rn2 a1 rn1
Pn = a1 rn1 a1 rn2 a1 rn3 ... a1 r a1
Multiplicando trmino a trmino:
Pn2 = a1a1rn1 a1a1rn1 a1a1rn1 ... a1a1rn1 a1a1rn1
Pn2 = (a1 an)n
Pn = (a1 an)n
9. Calcular el producto de los 10 primeros trminos de la P.G.
3 , 3 , 3 , ...
2 4

Sumatoria y progresiones 497

496-497 497 10/11/2001, 12:22


Solucin.
Pn = (a1 an)n
1 1 9 3
a1 = 3 ; r =
2
; a10 = 3 ( )
2
= 9
2
P10 = (3 3 2 ) = (3 3 2 )5 = 310 245
9 10 9

P10 = 310 245

10. Interpolar 5 trminos entre 1 y 512 para que resulte una


8
progresin geomtrica.
Solucin:
1
Se trata de formar una P.G. donde el primer trmino sea y
8
el sptimo trmino sea 512.
a1 = 1 = 2 3
8
a7 = 512 = 29

a7 = a1 r6 29 = 2 3 r6 r6 = 212
6
r = 4.096 r = 4

Con una calculadora podramos aplicar logaritmo para calcular r.


1 1
a2 = 4=
8 2
1
a3 = 42 = 2
8
1
a4 = 43 = 8
8
1
a5 = 44 = 32
8
a6 = 1 45 = 128
8
11. Probar que en una P.G. que tiene un nmero impar de trminos,
el trmino central es igual a la raz cuadrada del producto
de los extremos.
Solucin:
Sea p = 2 n 1 el nmero de trminos de la P.G.
El primer trmino es a1.
El p-simo trmino es ap = a1 r p1
El trmino central ac es el de lugar p + 1 , es decir, es
p1 2
ac = a1r 2 1
p1
ac = a1r 2

ac = a1rn1

a c = an

498 Sumatoria y progresiones

498-499 498 25/11/02, 12:53 PM


CAPTULO 12

Por demostrar que ac = a1 ap = a1 a1r p1 =


a1 a1r 2n 2 = a12 r 2(n 1) = (a1 r n 1)2 = a1 r n1 = an
luego ac = a1 ap p nmero impar.
12. Hallar el quinto trmino de una P.G. si se sabe que el primer
trmino es 2 y el noveno es 13.122.
Solucin.
Como es una P.G de 9 trminos, el quinto es el trmino central
y por lo tanto es la raz del producto del primero por el ltimo
(ver ejercicio anterior).
a5 = a1 a9 = 2 13.122 = 162
13. Se toma una hoja de papel de 0,1 mm de grosor. Se corta en dos
y se pone una encima de la otra; queda un grosor de 0,2 mm.
Este fajo se corta en dos y se pone una parte encima de la otra;
queda de grosor de 0,4 mm. La siguiente vez que se efecta
el mismo procedimiento, el fajo queda con un grosor de 0,8
mm. Suponiendo que el papel es de un tamao tal que permite
efectuar el mismo procedimiento 30 veces, averiguar el grosor
que tendra el fajo formado.
Solucin.
El grosor se duplica cada vez que se efecta el procedimiento
descrito: 0,1; 0,2; 0,4; 0,8, ... estamos frente a una P.G.
cuyo primer trmino a1 = 0,1 mm y cuya razn r = 2. La
altura del fajo pedida es la suma de los 30 primeros trminos
de esta P.G.
a (1 r29) 0,1(1 229)
S30 = 1 = = 53.687.091 mm
1r 12
Por lo tanto, el grosor del fajo obtenido luego de cortar 30 veces
el papel es de aproximadamente 53,68 km.
14. Se estima que el crecimiento de la poblacin de una regin
ser de un 2% anual. Calcular el porcentaje en que habr
crecido la poblacin al cabo de 20 aos. A los cuntos aos
se duplicar la poblacin?
Solucin.
Sea p la cantidad de poblacin existente. Si el crecimiento se
estima en un 2% anual, entonces al cabo del:
1er ao ser 1,02 p
2do ao ser (1,02)2 p
3er ao ser (1,02)3 p


20o ao ser (1,02)20 p = 1,49 p.
Por lo tanto, en 20 aos la poblacin crecer en un 49%.
Estamos frente a una P.G. cuyo primer trmino es p (poblacin
existente en el momento de efectuar la medicin) y cuya razn
es 1,02 (crecimiento de 2% anual).

Sumatoria y progresiones 499

498-499 499 25/11/02, 12:54 PM


Queremos averiguar en cuntos aos se duplicar la poblacin.
an = 2 p
an = a1 rn 1
2p = p (1,02)n 1
(1,02)n
2 =
1,02
(1,02)n = 2,04
log 2,04
n =
log 1,02
n = 36
Es decir, al cabo de 36 aos se duplicar la poblacin existente
si la tasa de crecimiento es de un 2% anual.
15. Expresar en forma racional los decimales infinitos siguientes:
a) 0,424242...
b) 025323232...
Solucin:
42 42 42
a) 0,424242... = + + + ...
100 10.000 1.000.000
Es decir, se trata de una suma de infinitos trminos de una P.G.
42 1
cuyo primer trmino a1 = y cuya razn r =
100 100
Como r = 1 < 1, entonces:
100
42 42
a1
= 99 = 42 = 14
100 100
S= =
1r 1
1 99 33
100 100

As; 0,42 = 14
33
32 32
b) 0,25323232... = 0,25 + + +...
10.000 1.000.000

Se trata de sumar infinitos trminos de una P.G. cuyo primer


32 1
trmino a1 = y su razn r = . Luego sumamos 0,25
10.000 100
y obtenemos la fraccin pedida:
32 32
10.000 10.000 32
S= 1
= 99
=
1 100 100
9.900
32
As; 0,25323232... = 0,25 +
9.900
1 32 2.475 + 32
= + =
4 9.900 9.900
2.507
=
9.900
2.507
Luego 0,25323232... =
9.900

500 Sumatoria y progresiones

500-501 500 10/11/2001, 11:10


CAPTULO 12

16. Probar que dado un capital C, puesto a un inters compuesto del

i% en un perodo determinado, al cabo de t perodos iguales el


t
i
capital C se ha transformado en C 1 +
100
Solucin.
Capital: C; inters: i%; t = perodos
Llamaremos S al capital acumulado al cabo de t perodos.
Al finalizar el primer perodo:
i
inters = C i ; Capital = C + Ci = C 1 +
100 100 100
Al finalizar el segundo perodo:
i i i i i
inters = C 1 + ; Capital= C 1 + + C 1+
100 100 100 100 100
2
i
= C 1+
100
Al finalizar el tercer perodo:
2 2 2

inters = C 1 +
i

i ; Capital= C 1 + i + C 1+
i
i
100 100 100 100 100
3
i
= C 1+
100
Al finalizar el t-simo perodo:
t1 t
i i i
inters = C 1 + ; Capital = C 1 +
100 100 100
del t-simo perodo.
17. Se invierten $ 500.000 al 6% anual. Calcular el capital que se habr
formado al cabo de 5 aos si el inters es compuesto:
a) anual, b) semestral, c) cuatrimestral.
Solucin.
C = 500.000 ; i = 6% anual ; t = 5 aos
a) Al cabo de 5 aos el capital se transforma en:
t 5
i 6
C 1+ = 500.000 1 + =
100 100

= 500.000 (1,06)5 = $ 669.111


b) Si los intereses se calculan semestralmente (dos veces al ao).
C = 500.000 ; i = 3% semestral ; t = 2 5 = 10
En estas condiciones al cabo de 5 aos el capital se transforma en:
t 10

C 1+
i = 500.000 1 + 3 =
100 100
= 500.000 (1,03)10 = $ 671.958
c) Si los intereses se calculan cuatrimestralmente (cuatro veces al ao)
C = 500.000 ; i = 2% trimestral ; t = 3 5 = 15

Sumatoria y progresiones 501

500-501 501 10/11/2001, 11:11


Ahora el capital, al cabo de 5 aos, se transforma en:
t 15
i 2
C 1+ = 500.000 1 + =
100 100

=500.000 (1,02)15 = $ 672.933


18. Calcular el capital que debe invertirse para que con un inters
compuesto del 10% anual, pagado semestralmente, al cabo de 3
aos se logre un capital de $1.000.000.
Solucin.
Nos preguntan por el capital inicial C. Llamaremos S al capital
logrado en los 3 aos.
S = 1.000.000 ; i = 5% ; t=6
t
i
S = C 1+
100

1.000.000 = C (1,05)6
1.000.000
C= = $ 746.268
1,34
Luego, para lograr $ 1.000.000 en 3 aos en las condiciones
sealadas, debe invertirse un capital inicial de aproximadamente
$ 746.268.
19. A qu porcentaje de inters compuesto deber ponerse un capital
para que se duplique en 5 aos?
Solucin:
Se desea que el capital inicial se duplique en 5 aos: S = 2C
t
i
S= C 1+ ; i=?; t = 5 aos
100
5
i
2C = C 1+
100
5
i
2 = 1+
100

5
100 2 = 100 + i
5
i = 100 2 100
i = 100 1,1487 100
i = 114,87 100 = 14,87
Es decir, un capital puesto a un inters compuesto del 15% anual
se duplica en 5 aos (ver ejercicio siguiente).
20. Calcular el capital que se obtiene al depositar $ 400.000 a un
inters compuesto del 15% anual durante 5 aos.
Solucin.
C = 400.000 ; i = 15% ; t=5; S=?
t 5
i 15
S = C 1+ S = 400.000 1 +
100 100

502 Sumatoria y progresiones

502-503 502 25/11/02, 12:58 PM


CAPTULO 12

= 400.000 (1, 15)5


= 400.000 2,0113571 = $ 804.543
Vemos que en 5 aos al 15% el capital aproximadamente se
duplica.
21. Un banco concede un prstamo de $ 1.000.000 pagadero en 3
aos con un inters anual del 15%. Cunto se debe cancelar
mensualmente? Cunto se cancelar en total?
Solucin:
El 15% anual corresponde al 1,25% de inters mensual y son 36
meses; por lo tanto, el capital de $ 1.000.000 se convertir al
cabo de los tres aos en S.
t
i C = 1.000.000
S = C 1 +
100 i = 1,25
t = 36
S = 1.000.000 (1,0125)36 = 1.563.942
lo que dividido en 36 cuotas nos da $ 43.443 cada mensualidad.
Se deben pagar 36 mensualidades de $ 43.443 cada una, lo que
da un total de $ 1.563.942.

Ejercicios

1. Determine si las siguientes sucesiones 4. Encuentre tres nmeros que formen


son o no progresiones geomtricas. Si una P. G. tales que su producto sea
lo son, hallar la razn. 1.728 y su suma sea 52.

a) 4, 12, 36, ... 5. En una P. G. el primer trmino es 23


y la razn es 2. Cuntos trminos se
b) 24, 18, 14, ... deben sumar para que el resultado
sea 1.449?
c) 3, 9, 27, ...
1 1 1
6. Una P. G. consta de cuatro trminos.
d) , , , ... Si la suma del primero y el ltimo es
2 4 8
168 y la suma de los dos centrales es
e) q, 1, 1 ,
q 2
1
, ... 72, encuentre esta progresin.
q
1 1 1 3
f) r
+ r, + 2 + r2, + + 3 r + r3 ,
r
.... 7. Si un muchacho ganara $ 1 el primer
r2 r3
da de su trabajo, $ 2 el segundo
2. Encuentre el quinto trmino y la suma
da, $ 4, el tercer da, $ 8 el cuarto,
de los 10 primeros trminos de la P.
G. 8, 4, 2, ... cunto habra ganado al cabo de
20 das de trabajo?
3. El segundo trmino de una P. G. es
32 8. Calcule la media geomtrica entre
y el quinto es . Encuentre el
3 p y q.
octavo trmino.

Sumatoria y progresiones 503

502-503 503 25/11/02, 12:58 PM


9. Encuentre los 5 primeros trminos de la razn y escribir los cuatro primeros
una P. G. de modo que el primero sea trminos.
2 y el segundo sea 3.
22. Calcule la suma de:
10. Interpole 4 trminos entre 1 y 243 de
3 + 0, 3 + 0, 03 + 0, 003 +...
modo que formen una P. G.
23. La suma de tres nmeros en P. G. es 35.
La diferencia entre el mayor y el menor
11. Interpole tres trminos entre 3 y 1
4 108 es 15. Encuentre la progresin.
de modo que se forme una P. G.
x4
24. Encuentre x para que , x + 2
12. Interpole dos medios geomtricos 2
entre a y b. y 2 (x 2) estn en P. G. Cunto
vale la razn?
13. Interpole dos medios geomtricos
entre 6 y 162 (ver ejercicio anterior).
25. Determine x para que x + 1, 2 x + 2 y
4 x + 4 formen una P. G.
14. Encuentre tres nmeros en P. G. tales
que su producto sea 512 y que el
26. Determine x para que x 1, x + 2
menor sea la dcima parte del doble
y x + 5 estn en P. G.
del central.
27. Encuentre la suma de n trminos de
15. Determine el producto de los once
la P. G. 1, 1, 1, 1,...
trminos de una P. G. si el trmino
central es 3. 28. Si se sabe que una determinada
bacteria se reproduce por biparticin
16. Determine el producto de los 2n1
cada 20 minutos, es decir, de cada
trminos de una P. G. si su trmino
bacteria aparecen 2 cada 20 min,
central es q.
cuntas bacterias habr pasadas
10 horas desde que se detect la
17. Determine cuatro nmeros que estn primera?
en P. G. tales que el primero y el
29. Se tiene un cuadrado de lado a. Se
cuarto sumen 91 y los dos centrales inscribe en l un cuadrado uniendo los
108
sumen 7 . puntos medios del cuadrado original
9
y as se van inscribiendo cuadrados
18. En la P. G. 4 , 2 , 1 ..., encuentre el cada vez ms chicos. Calcule la suma
5 5 5 de las reas y de los permetros de los
valor de la razn y la suma de infinitos
infinitos cuadrados as obtenidos.
trminos.
30. Calcule la suma de las reas y de los
1 1 1 permetros de todos los cuadrados que
19. Calcule la suma de 1+ + + + ...
3 9 27 se pueden inscribir sucesivamente
20. Determine cuntos trminos tiene a partir de un cuadrado de 4 m de
la P. G. cuyo primer trmino es 2 lado.
y cuyo ltimo trmino es 512 si su
suma es 682. 31. El crecimiento de una poblacin es
del 4% anual. Qu poblacin habr
21. La suma de una P. G. infinita es 6,25 dentro de 10 aos si hoy hay 150.000
y el segundo trmino es 1. Encuentre habitantes?

504 Sumatoria y progresiones

504-505 504 10/11/2001, 11:20


CAPTULO 12

32. Una regin tiene 1.000.000 de 36. Una persona decide ahorrar $ 500.000,
habitantes. En cuntos aos se los que ingresar al banco al inicio
duplicar esta poblacin si la tasa de de cada ao. Si el banco le paga un
crecimiento es del 5% anual? 12% de inters compuesto anual, qu
capital tendr al cabo de 6 aos?
33. Calcule el capital final que se obten-
dr si se invierten $ 2.000.000 al 37. Para la adquisicin de un departa-
16% anual al cabo de 3 aos. mento una persona debe contraer
una deuda por $ 4.000.000. El banco
34. Determine el capital que se obtiene se la presta, pero le cobra un 15%
en 2 aos colocando $ 500.000 a un anual. Si la persona decide pagar en
inters del 8% anual si los intereses 20 aos, cunto deber cancelar en
se liquidan trimestralmente. total al cabo de los 20 aos? Cunto
le saldr la cuota mensual si se fija
35. Qu capital se debe invertir si
se quiere que al cabo de 5 aos, el valor con el 15% anual?
colocado a un inters del 12% anual,
ste se convierta en $ 1.000.000?

Soluciones
20. n = 5 21. r = 1 ; 5, 1, 1 , 1
1. a) es P.G. r = 3 5 5 25
4
b) no es P.G. 14 18 r = ; 1,25 ; 1 ; 0,8 ; 0,64
5
18 24
c) es P.G. r = 3 22. S = 10 23. 5, 10, 20.
3

d) es P.G. r = 1 24. x = 2 ; r = 4
2 5 3
e) es P.G. r = 1 25. Cualquier x 1 hace de este tro
q
una P.G.
1
f) es P.G. razn = r
+r 26. No existe x E R tal que x 1, x + 2 y
2. a5 = 1 ; S10 = 1.023 x + 5 estn en P.G.
2 64 27. Si n es par Sn = 0. Si n es impar Sn = 1.
3. 256 4. 4, 12 y 36
3 28. Habr ms de 1.073 millones de bacte-
5. Los seis primeros 6. 6, 18, 54, 162.
rias.
trminos.
29. Sreas = 2a2; Spermetros = 8a
7. $ 1.048.575 8. pq = 4a 2 + 2
9 27 81 30. Srea = 32 m2; 2 2
9. 2, 3, , , 10. 3, 9, 27, 81
2 4 8
1 1 1 3 6
Spermetros = 16 2 + 2 = 54,6 m.
11. , , 12. a2b , a2 b4
4 12 36 31. 222.033 habitantes.
13. 18 y 54 14. 1.6, 8, 40 32. En aproximadamente 15 aos.
15. 177.147 16. q2n1 33. $ 3.121.792 34. $ 585.526
1 1 4 16 35. $ 567.427 36. $ 4.544.508
17. , , ,
4 3 9 27
37. $ 65.466.128
18. r = 1 S=
8
19. S = 3
2 5 2 Cuota mensual fija: $ 272.776.

Sumatoria y progresiones 505

504-505 505 10/11/2001, 11:21


12.5 Progresin
armnica

Definicin: Una Progresin Armnica (P.H.) es una


sucesin de trminos a1, a2, a3,... an,...
tal que 1 1 1 1
, , , ... , ...
a1 a2 a3 an
estn en progresin aritmtica.

Nota: No disponemos de frmulas elementales para el clculo


de progresiones armnicas; por ello resolveremos los ejercicios
ocupando la definicin y, por lo tanto, las frmulas para
progresiones aritmticas.

1. Determinar si las siguientes sucesiones son o no progresiones


Ejercicios
armnicas (P.H.).
resueltos a) 1 1 1 1
, , , , ... b) 1, 2 1 2 1 2
, , , , , ...
2 4 6 8 3 2 5 3 7
Solucin.
Debemos analizar si la sucesin de los trminos recprocos
es una P.A.
a) 2, 4, 6, 8,... es P.A. con a1 = y d = 2

luego 1 1 1 1 es P.H.
, , ,
2 4 6 8
3 5 7 1
b) 1, , 2, , 3, es P.A. con a1 = 1 y d =
2 2 2 2
2 1 2 1 2
luego 1, , , , , es P.H.
3 2 5 3 7
2. Hallar el trmino siguiente en cada P.H.:
1 1 1
a) , 1, , b) 2 , 1, 3 , 3 , 1
2 4 7 3 4 5 2
Solucin.
Debemos trabajar con los recprocos de los trminos dados y
buscando la diferencia de la P.A.
a) 2, 1, 4, 7, es P.A. de d = 3, luego el trmino siguiente
es 10.
Luego el trmino siguiente de la P.H. es 1
10
b) 2 , 1, 4 , 5 , 2 es P.A. de d = 1 , luego el trmino siguiente es 7 .
3 3 3 3 3
Por lo tanto, el trmino siguiente de la P.H. es 3
7
3. Interpolar un medio armnico entre a y b.
Solucin.
Sea x el medio armnico que debemos interpolar.

506 Sumatoria y progresiones

506-507 506 10/11/2001, 11:24


CAPTULO 12

Entonces a, x, b, deben estar en P.H., lo que significa que


1 1 1
, , deben ser P.A., luego
a x b
1 1 1 1
=
b x x a
2 1 1
= +
x b a
2 a+b
= x = 2ab
x ab a+b
4. Interpolar 2 medios armnicos entre 2 y 12.

Solucin.
Sean x e y los medios armnicos que se desea interpolar.
Entonces 2, x, y, 12 deben ser P.H., por lo que 1 , 1 , 1 , 1
2 x y 12
deben ser P.A. y 1 1 = 1 1 = 1 1
12 y y x x 2
1 1
Sean x
=a , y
=b
1 1
b=ba=a
12 2
Tomando dos igualdades:
1 1
b=ba a 2b =
12 12
1 1
ba=a b 2a =
2 2

2
de donde a = 13 y b=
36 9
13 2 2
Luego 1
x
= , 1y = y 1
, 13 , , 1
estn en P.A.
36 9 2 36 9 12

Luego los medios armnicos pedidos son


9
x = 36 e y= as
13 2
36 9
2, , , 12 es una P.H.
13 2
5. Encontrar tres nmeros que estn formando una P.A., una
P.G. y una P.H.
Solucin.
Sean x, y, z los nmeros buscados, entonces:

zy =yx (P.A.)
z y
=
y x
(P.G.)
1 1 1 1
=
z y y x
(P.H.)

La solucin del problema planteado est en resolver este


sistema:

Sumatoria y progresiones 507

506-507 507 10/11/2001, 11:25


x 2y + z = 0 x 2y + z = 0 (1)
y2 xz = 0 y2 xz = 0 (2)
xy2 2xyz + zy2 = 0 /:y xy 2xz + yz = 0 (3)

Reemplazando (2) en (3) y dividiendo por y obtenemos la


ecuacin (1). De (2) y = x z

Reemplazando en (1)
x 2 xz + z = 0 ( x z )2 = 0 x z =0
En (1) x + z = 2 xz Hagamos a = x y b = z
x z = 0

a2 + b2 = 2ab
ab= 0 a=b

2 a2 = 2ab
a= b x = z x=z
En cualquiera de las ecuaciones del sistema vemos que x = y = z.
Luego cualquier tro de nmeros iguales estn P.A. (d = 0), P.G.
(r = 1) y P.H. (sus recprocos estn en P.A.).

Ejercicios (Mezclado P.A., P.G. y P.H.)

1. En cada una de las sucesiones Qu progresin se forma? Determine


siguientes, determine si son P.A., la diferencia. El primer trmino y el
P.G. o P.H. de lugar 52.

a) 2,
2 2 2
, , , ....
5. Encuentre tres nmeros en P.G. sabiendo
3 9 27 que si se suma 2 al del medio se
b) 1, 0.1, 0.01, 0.001, ... forma una P.A. y que los tres nmeros
sumados dan 28.
c) 3 , 1, 5 , 3 , ....
4 4 2 6. Encuentre tres nmeros en P.A. sabiendo
d) 5, 5, 5, 5, ... que si se resta 2 al del medio se
transforman en P.G. y que los tres
e) a, a , a , a , .... aEN nmeros sumados dan 51.
2 4 8
a3 a6 a 9
f) 3 3 3
, 1, , , .... 7. Estudie la sucesin , ,
a a a
2 4 5
Encuentre el dcimo trmino y la suma
2. En la sucesin x, x2, 4x2 determine
qu valor debe tomar x para que de los diez primeros trminos.
sta sea a) P.A., b) P.G., c) P.H. 8. En una plantacin hay 50 filas de
3. Dada la funcin f(n) = 2n + 5, rboles, si cada fila tiene 1 rbol ms
donde n E N, escriba los cinco que la anterior y la fila 12 tiene 38
primeros valores de f(n). Forman rboles, calcule cuntos rboles hay
ellos alguna progresin? en la primera fila, en la ltima fila y
4. Escriba los cinco primeros trminos cuntos rboles hay en la plantacin
de la funcin f(n) = 3n 2 con n E N. completa.

508 Sumatoria y progresiones

508-509(2003) 508 25/11/02, 1:03 PM


CAPTULO 12

9. Un refrigerador cost $ 154.000. toma una copa de 100 cc. y para que
Despus de cuatro aos se vendi no se note la reemplaza con agua.
en la mitad de su valor. Si esto se Otras personas realizan el mismo
repite cada cuatro aos, cunto procedimiento, quedando cada vez
pag por l la quinta persona que de inferior calidad el vino. Cuntas
lo compr? personas debern tomarse 100 cc. de
vino y reemplazarlo por agua para
10. Una garrafa de 5 litros contiene vino que la garrafa contenga 50% de agua
de buena calidad. Una persona se y 50% del vino original?

Soluciones

1 3. 7, 9, 11, 13, 15 P.A. d=2


1. a) P.G. r=
3
b) P.G. r=
1 4. 1, 4, 7, 10, 13 P.A. d=3
10

c) P.A. d= 1 a1 = 1 a52= 154


4
d) P.A. d = 0, P.G., r = 1 5. 4, 8 y 16 6. 9, 17 y 25
1 1 1 1 a3
P.H. , , , es P.A. d = 0 7. Es una P.A. con a1 = a
, d= 3
5 5 5 5 a
1 a 30 10a 165
e) P.G. r= a10 = a
S10 =
2 a

f) P.H. { 23 , 33 , 43 , 53 , es P.A. d = }
1
3
8. a1 = 27, a50 = 76 S50 = 2.575

2. a) 1
b) 4 c) 7 9. $ 9.625 10. 6
2 4

Induccin 12.6
matemtica

El principio de Induccin matemtica es un mtodo


de demostracin vlido para subconjuntos infinitos de
nmeros naturales.
Se basa en los axiomas de Peano que afirman:

i) 1 E N (El uno es un nmero natural)

ii) k E N (k + 1) E N (si k es un nmero natural


entonces (k + 1) tambin lo es)

Sumatoria y progresiones 509

508-509(2003) 509 25/11/02, 1:04 PM


El principio de induccin es el siguiente:
Si P(n) es una proposicin asociada al nmero natural n
y se verifica:
i) P(1) es verdadera, es decir, n = 1 hace verdadera la
proposicin.
ii) Siempre que P(k) sea verdadera se cumple que P(k+1)
tambin lo es.
Entonces la proposicin P(n) se cumple para todo nmero
natural n.

Ejercicios 1. Demostrar por induccin que la suma S de los n primeros


resueltos nmeros naturales es S =
n(n + 1)
2
Solucin:
n(n + 1)
Debemos demostrar que 1 + 2 + 3 + ... + n =
2
n
n(n + 1)
Sea P(n): i=
i=1 2
i) Verificamos si se cumple para n = 1
1
12
=1 = s se cumple.
i=1 2
ii) Suponiendo que se cumple para n = k, verificamos si se
cumple para n = k + 1, es decir:
k
k(k + 1)
hip.: i =
i=1 2
k+1
(k + 1)(k + 2)
tesis: i =
i=1 2
k
k(k + 1)
Dem.: i = /sumamos a ambos lados
i=1 2
el trmino siguiente a k.
k
k(k + 1)
i + (k + 1) = + (k + 1)
i=1 2
k+1
k(k + 1)
i = + (k + 1)
i=1 2
k(k + 1) + 2(k + 1)
=
2
2
k + 2k + 2k + 2
=
2
2
k + 3k + 2
=
2
k+1
(k + 1) (k + 2)
i =
i=1 2

510 Sumatoria y progresiones

510-515(2003) 510 25/11/02, 1:07 PM


CAPTULO 12

que es lo que queramos demostrar; por lo tanto, afirmamos


que para todo nmero natural n se cumple que la suma S es:
n(n + 1)
y P(n) es verdadera para todo n.
2
2. Demostrar que la suma de los n primeros nmeros impares
es S = n2
Solucin:
Por demostrar.
1 + 3 + 5 + .+ (2n 1) = n2
n
Sea P(n): (2j 1) = n2
j=1

i) Verificamos si se cumple para n = 1


1
2j 1 = 1 = 12 s se cumple.
j=1

ii) Suponiendo que se cumple para n = k, verificamos si se


cumple para n = k + 1; es decir;
k
hip: 2j 1 = k2
j=1
k+1
tesis: (2j 1) = (k + 1)2
j=1

Dem.: Por la hiptesis, tenemos:


k
(2j 1) = k2 sumamos a ambos lados el
j=1
impar siguiente a 2k 1, que
es 2k + 1
k
(2j 1) + (2k + 1) = k2 + (2k + 1)
j=1

k+1
(2j 1) = k2 + 2k + 1
j=1

k+1
(2j 1) = (k +1)2
j=1

que es nuestra tesis: entonces por el principio de induccin


afirmamos que la proposicin es vlida para todo nmero
natural n.

3. Demostrar que la expresin n2 + 5n es siempre par.

Solucin:

Sea P(n): n2 + 5n = 2p para p E N.

i) Verificamos si se cumple para P(1)

n = 1; 12 + 5 1 = 6 = 2 3 (es par)

ii) Suponiendo que se verifica para n = k, demostremos que


tambin se cumple para n = k + 1:

Sumatoria y progresiones 511

510-515(2003) 511 25/11/02, 1:07 PM


hip.: k2 + 5k = 2p (para algn p E N)

tesis: (k + 1)2 + 5(k + 1) = 2q (para algn q E N)

En este caso vamos a trabajar con la tesis, de la siguiente


forma:
Debemos demostrar (k + 1)2 + 5(k + 1) = 2q
(k + 1)2 + 5(k + 1) = k2 + 2k + 1 + 5k + 5
= k2 + 5k + 2k + 6

{
= 2p + 2(k + 6) por hiptesis de
induccin
= 2(p + k + 6)
= 2q
Entonces si se cumple para k tambin se cumple para (k + 1) y
por lo tanto se cumple para todo natural n.
4. Demostrar que se cumple para todo nmero natural n el
siguiente enunciado.
1 1 1 1 n
+ + + ... + =
12 23 34 n (n + 1) n+1
Solucin:
i) Verificamos primero que se cumple la igualdad para
n = 1, es decir,
1 1
= correcta
12 2
ii) Suponiendo que se verifica para n = k, debemos demostrar
que se verifica para n = k + 1

1 1 1 k
hip.: + = ++
12 23 k (k + 1) k+1
1 1 1 1 k+1
tesis: + ++ + =
12 23 k (k + 1) (k + 1)(k + 2) k+2
dem.: Por hiptesis tenemos:
1 1 1 k
+ + = +
12 23 k (k + 1) k+1
Sumamos a ambos lados el trmino siguiente que
1
es :
(k + 1)(k + 2)

1 1 1 1 k
+ ++ + = +
12 23 k (k + 1) (k + 1)(k + 2) k+1
1
(k + 1)(k + 2)

512 Sumatoria y progresiones

510-515(2003) 512 25/11/02, 1:07 PM


CAPTULO 12

1.Demostrar la induccin que la sumas de los n primeros


1
nmeros naturales 1 1 k (k + 2) + 1
+ es S+= + =
12 23 (k + 1)(k + 2) (k + 1)(k + 2)
Solucin: Debemos demostrar que 1 + 2 + 3+ .+ n = ..
(sacando
Sea P(n): .. i = n(n + 1) denominador comn)
i)Verificamos si se cumple
k2 + 2k + 1 para n = 1
=
.. = 1 (k=+11)(k x 2 + 2) si se cumple.
(k + 1)2
ii)Suponiendo
= que se cumple(factorizando)
para n = k, verificamos si se
(k + 1)(k
cumple para n = + 1, es decir: + 2)
k+1
hip. ..=i = k(k(simplifi
+ 1) cando)
k+2
tesis
que es lo que. debamos
i = . (k + 1) ( kpor
demostrar; + 2)lo tanto, afirmamos que
para todo
Dem. .. i = nmero natural n se cumple
k(k + 1) /sumamos a ambos la proposicin.
lados el trmino
5. Demostrar
siguiente a k. que para todo entero n, n 2, se verifi ca:
n
i + (k + 1) = k(k (1+ 1) + (k>+1 1)
+ a) + na
siendo a un nmero real distinto de cero y mayor que -1.
. i = k(k + 1) + (k + 1)
(a E R, a 0, a > 1)
= k(k + 1) + 2(k + 1)
Solucin:
= k2
Sea P(n)+ la + 2k + 2 (1 + a)n > 1 + na
2kdesigualdad
= k2 + 3k
(notemos que+ para
2 n = 1 es falsa, sin embargo debemos verificar
que la +proposicin
= (k 1) (k + 2) es verdadera a partir de n = 2)
i)i n = 2 = (k + 1) (k + 2)
(1 lo
que es a)2 =queramos
+ que 2 > 1 + 2a
1 + 2a + ademostrar, por lo pues
2
tantoaafi>rmamos
0, ya que
que
a 0)
para todo nmero natural n se cumple que la suma S es: n(n + 1)
y y P(n) es verdadera
entonces para todo n.se cumple para n = 2, es decir,
la proposicin
P(2) es verdadera.
2.Demostrar que la suma de los n primeros nmeros impares
es S =ii)
n2Suponiendo que se verifica para n = k, debemos demostrar
que se verifica para n = k + 1, es decir, si P(k) es verdadera,
Solucin:
entoncesPor demostrar.
tambin debe serlo P(k + 1).
1 + 2Hip.:
+ 5 +(1.+
+ a) > 1 +
(2n
k 1)ka
= n2
Tesis:.
Sea P(n): + a)k+1
(1 (2j 1) => n2
1 + (k + 1)a
Dem.: (1 + a) k+1 = (1 + a)k (1 + a) (prop. de exponente)
i)Verificamos si se cumple para n = 1
> (1 + ka) (1 + a) (hiptesis de induccin)
. 2j 1 = 1 = 12 si se cumple.
pero observamos que:
ii)Suponiendo que se cumple para n 2= k, verificamos si se
(1 + ka) (1 + a) = 1 + a + ka + ka (multiplicando)
cumple para n = k + 1; es decir
= 1 + a(k + 1) + ka 2 (factorizando)
hip2j 1 = k2
> 1 + a (k + 1) (pues ka2 > 0, k > 0
tesis ..(2j 1) = (k + 1)2 y a 0)
Por Por
Dem. lo la
tanto, por propiedad
hiptesis, tenemos: transitiva de la desigualdad
concluimos que si
(2j 1) = k2 sumamos a ambos lados el impar siguiente
(1 + a)k > (1 + k)a entonces (1 + a)k+1 > 1 + (k + 1)a
a 2k 1, que es 2k + 1
Y podemos afirmar que la proposicin es vlida para todo
(2j 1) + (2k + 1) = k2 + (2k + 1)
n 2 (a > 1, a 0)

Sumatoria y progresiones 513

510-515(2003) 513 25/11/02, 1:08 PM


Ejercicios

Demuestre que los siguientes enunciados son verdaderos para todo entero
positivo n:

1. 3 + 5 + 7 + + (2n + 1) = n(n + 2)

(3n 1)
2. 1 + 4 + 7 + + (3n 2) =
2
3. 3 + 7 + 11 + + (4n 1) = n(2n + 1)

4. 3 + 9 + 15 + + (6n 3) = 3n2

n(5n 1)
5. 2 + 7 + 12 + + (5n 3) =
2
6n 1
6. 1 + 6 + 62 + + 6n1 =
5

7. 2 + 6 + 18 + + 2 3n1 = 3n 1

1 1 1 n (n + 3)
8. + + ... + =
123 234 n(n + 1) (n + 2) 4(n + 1) (n + 2)
n

9. i=1
2i = n(n + 1)

n
i(i + 1) n(n + 1) (n + 2)
10. =
i=1 2 6
n

11. i=1
2i = 2(2n 1)

n
1 n
12. =
i=1 (2k 1) (2k +1) 2n + 1
n
2k + 1 1
13. =1
k=1 [k(k + 1)]2 (n + 1)2
n
n(n + 1) (2n + 1)
14. k2 =
k=1 6
n
n(n + 1)
15. k3 =
k=1 2
16. n3 n + 3 es divisible por 3.

17. n2 + n es divisible por 2.

18. n2 n + 2 es divisible por 2.

19. 4n 1 es divisible por 3.

514 Sumatoria y progresiones

510-515(2003) 514 25/11/02, 1:08 PM


CAPTULO 12

20. 5n 1 es divisible por 4.

21. 42n 1 es divisible por 3.

22. n3 + 3n2 + 2n es divisible por 6.

23. 10n+1 + 3 10n + 5 es divisible por 9.

24. 23n 1 es divisible por 7.

25. xn yn es divisible por (x y).


(sugerencia xk+1 yk+1 =
xk(x y) + (xk yk) y )

26. x2n1 + y2n1 es divisible por x + y.

27. n < 2n

28. n2 < 2n para n 5

29. 2n + 1 < n2 para n 3.

30. 1 + 2n 3n

n(n + 1) (2n + 1)2


31. <
2 8
32. 3n 3 n

Prueba de seleccin mltiple


1. El trmino siguiente en A. slo I D. a
la P.A. 4, 1, 2 es: B. slo II E. 1 a
A. 1 C. slo III
B. 2 4. Si 1x , 1y , 1z estn en P. H.
D. todas
C. 3 E. ninguna podemos decir que x,
D. 4 y, z estn
E. 5 3. En la P.A. siguiente la
diferencia es: A. en P.A.
2. De las siguientes suce-
a a2 2a 2 a B. en P.G.
siones son P.G. , ,
a1 a1 a1
1 C. en P.H.
III) 1, 1
,1 A.
a
2 4 D. en P.A. y P.G.
III) 1, 2, 4 B. 1
a E. no forman ninguna
III) 1, 1 , 1 C. a progresin.
2 4

Sumatoria y progresiones 515

510-515(2003) 515 25/11/02, 1:08 PM


Prueba de seleccin mltiple
5. La suma de todos los E. 18 E. 3 1 , 10, 30
3
nmeros impares de dos
1 2
cifras es: 9. ,
3 3
y 1 estn en P.A., 13. En la sucesin 1, 3 , 9 ,
4 16
A. 2.225 entonces estn en P.H. 27
, 81 , ... el trmino
64 256
B. 2.350 A. 3 2 1 siguiente es:
C. 2.475 B. 3 1,5 1 108
A.
D. 2.525 192
C. 3 2 1,5
243
E. 2.550 B.
D. 2 1,5 1 1.024
6. Dados los nmeros 3 E. 2 1 0,5 C.
324
y 12, qu nmero se 768
debe intercalar entre 10. La suma de n nmeros 162
D.
ambos para obtener una 192
impares consecutivos,
P.G.? partiendo del 1, es 2.500. 324
E.
1.024
Entonces n vale:
A. 4
14. La suma de los trminos
B. 5 A. 10 1 1
de la sucesin 1, , ,
3 9
C. 6 B. 20 1
,... es:
27
D. 8 C. 25
A. 1
E. 10 D. 30 B. 1,5
1 E. 50 C. 2
7. Dados los nmeros y
5 D. 2,5
1 11. El quincuagsimo ml-
, qu nmero se debe
9 tiplo de 3 es: E. 3
intercalar entre ambos
para obtener una P.H.? A. 141 15. En una P.G. si a5 = 9
y a7 = 1, entonces a6
A. 7 B. 144
vale:
1 C. 147 A. 8
B.
7 D. 150 B. 5
C. 6
E. 153 C. 7
1
D.
6 12. Si el producto de tres D. 3
1 nmeros que estn en
E. E. 1
8
P.G. es 1.000 y la razn
8. Si el producto de tres es 3, los tres nmeros 16. La suma de los cincuenta
nmeros en P.G. es 27, son primeros nmeros natu-
cul es el trmino cen- rales es:
tral? 1
A. 1 , 4, 12 A. 1.025
3
A. 1 B. 2, 6, 18 B. 1.125
B. 3 1 C. 1.275
C. 2 , 7, 21
C. 6 3 D. 1.575
D. 9 D. 3, 9, 27
E. 1.750

516 Sumatoria y progresiones

516-517 516 10/11/2001, 11:43


CAPTULO
CAPITULO12
CAPTULO 12

1
17. La suma de los prime- C. (2n3 + 6n2 + 4n) B. 4
8
ros veinticinco nmeros
C. 16
pares es: 1
D. (n3 + 3n2 + 2n) D. 64
8
A. 600
E. 81
1
B. 625 E. (2n3 + 6n2 4n)
6 25. La suma de infinitos tr-
i = 20
C. 650 21. El valor de i2 + 5 minos de la progresin
es:
i = 10
5, 5 , 5 , 5
, ... es:
2 4 8
D. 1.125
A. 2.640 A. 5
E. 1.275 B. 8
B. 2.650
18. La suma de los prime- C. 9
C. 2.675
ros veinticinco nmeros D. 10
impares es: D. 2.685
E. 12
A. 600 E. 2.690
26. La suma de tres nmeros
22. El producto de los diez en P.A. es 45 . Si al del
B. 625
primeros trminos de la 2
3
1 centro se le resta se
C. 650 progresin 1, , 1 , 1 , 2
2 4 8
... es: transforma en una P.G.
D. 1.125 Los nmeros son:
A. 215
E. 1.275 A. 3, 15 25
,
B. 220 2 12

19. La expresin C. 225 15


B. 3, , 12
2
20 D. 230
i1 i + 1 =
i=1
E. 2 45 C. 3, 6, 12
A. 2.250
23. En la P.A. 7, 10, 13, ... 25
D. 3, 6,
B. 2.500 el trmino de valor 37 2

ocupa el lugar: 7 15 23
C. 2.650 E. , ,
2 2 2
A. 8
D. 2.700 27. Para que el medio arit-
B. 9 mtico sea igual al medio
E. 2.850 geomtrico entre dos
C. 10 nmeros a y b debe
20. La suma de los n prime- D. 11 cumplirse que:
ros trminos de la serie
1 2 + 2 3 + 3 4 E. 12 A. a + b = 0
+ ... es:
24. El sexto trmino de una B. a b = 0
P.G. es 1.024 y la razn
1 C. a + b > 0
A. (2n3 + 6n2 + 4n) es 4. Entonces el tercer
6
trmino es: D. a b > 0
1 A. 1
B. (n3 + 3n2 + 2n) E. a b < 0
6

Sumatoria y progresiones 517

516-517 517 10/11/2001, 11:44


Prueba de seleccin mltiple
28. Si tres nmeros a, b y veces el medio aritm- tercero, $ 8 el cuarto y as
c estn en P.H. y P.G., tico, entonces: sucesivamente. Cunto
entonces: A. ab = 4 (a + b)2 habr ganado al cabo
A. ac (a 2b + c) = 0 de 25 das?
B. ab = 4a2 + b2
B. ac (a + 2b + c) = 0 A. $ 616.777.216
C. ab = 4 (a2 + b2)
C. ac (a 2b c) = 0 B. $ 633.554.431
D. ab = 2 (a + b)2
D. ac (a + 2b c) = 0 C. $ 267.108.864
E. ab = 2a2 + 2b2
E. ac ( a 2b c) = 0 D. $ 134.217.728
30. Una persona gana $ 1
29. Si el medio geomtrico el primer da de trabajo, E. $ 268.435.456
entre a y b es cuatro $ 2 el segundo, $ 4 el

Soluciones

1. E 6. C 11. D 16. C 21. D 26. B


2. D 7. B 12. E 17. C 22. E 27. B
3. C 8. B 13. B 18. B 23. D 28. A
4. A 9. B 14. B 19. E 24. C 29. A
5. C 10. E 15. D 20. A 25. D 30. B

518 Sumatoria y progresiones

518 518 10/11/2001, 13:00


CAPTULO

nlisis
combinatorio,
13
A
Teorema del binomio y
Elementos de probabilidades
Anlisis combinatorio 13.1

13.1.1 Conceptos bsicos


Sea n un nmero natural. Definimos el FACTORIAL de n,
denotado por n! en forma recursiva, por 1! = 1.
(n + 1) ! = n! (n + 1)
Definimos tambin 0! = 1
Sean n y k dos elementos de N0 tales que n > k. Se define el
n n n!
nmero por =
k k k! ( n k) !

Se lee "n sobre k".


PRINCIPIO DE LA MULTIPLICACIN
Supongamos que un suceso puede ocurrir de n maneras y
otro suceso puede ocurrir de m maneras, entonces ambos sucesos
pueden ocurrir de m n maneras.

PRINCIPIO DE LA SUMA
Supongamos que un suceso puede ocurrir de m maneras y otro
suceso puede ocurrir de n maneras, entonces hay m + n maneras
en que pueda ocurrir slo uno de ellos.

13.1.2 Permutaciones
Una permutacin de los elementos de un conjunto es
cualquier cambio en el orden de estos elementos sin repetirlos
ni omitirlos.

Anlisis combinatorio, Teorema del binomio y Elementos de probabilidades 519

519 519 25/11/02, 1:12 PM


Segn el principio multiplicativo, el nmero de permutaciones
que se pueden efectuar en un conjunto de n elementos es n!
Si denotamos por P (n) el nmero de permutaciones en un
conjunto de n elementos, tenemos:

(n) = n !P
Vemos que en las permutaciones importa la posicin relativa
de los elementos entre s, por lo tanto, si queremos permutar
en forma circular n elementos, el nmero de maneras en que
podemos hacerlo es:
Pc (n) = (n 1) !
(puesto que rotar todos juntos en una direccin no constituye
una permutacin diferente).

13.1.3 Arreglos o variaciones


Un arreglo o variacin de k elementos tomados de un conjunto
de n elementos (k n) es cualquier ordenacin que puede hacerse
con esos k elementos.
Notemos que dos arreglos o variaciones diferentes pueden
incluir los mismos elementos, slo es necesaria una ordenacin
distinta.
El nmero de arreglos o variaciones que pueden efectuarse de k
elementos tomados de un conjunto de n est dado por:

A n =
k
n!
(n k) !
Ejemplo: En el conjunto {a, e, i, o, u} podemos formar:

V52 = A 52 = (5 5 !2) ! =
5!
3!
=
1
1


2
2


3
3
4 5
= 20

variaciones diferentes tomando 2 elementos cada vez.


Estos son:
ou, ae, ai, ao, au, ei, eo, eu, io, iu,
uo, ea, ia, oa, ua, ie, oe, ue, oi, ui
(au y ua son dos arreglos diferentes)

13.1.4 Combinaciones
Una combinacin de k elementos tomados de un conjunto
de n elementos (k n) es cualquier subconjunto que se puede
formar con esos k elementos.
Notemos que al hablar de subconjunto no estamos considerando
el orden en que estn dispuestos los elementos.
As, dos combinaciones sern distintas si al menos tienen
un elemento distinto.
El nmero de combinaciones que pueden formarse de k
elementos a partir de un conjunto de n elementos est dado
por:
n n!
Ck = n
k
=
k ! ( n k) !

520 Anlisis combinatorio, Teorema del binomio y Elementos de probabilidades

520-521 520 25/11/02, 1:14 PM


CAPTULO 13

Ejemplo: El nmero de combinaciones de 2 elementos que


pueden formarse a partir del conjunto M = {a, e, i, o, u} es:

C 52 = 2 ! ( 55 ! 2) ! =
5!
2! 3!
1
1
2
2
3
1
4
2
5
3
= 10

y ellos son:
{a e}, {a i}, {a o}, {a u}, {e i}, {e o}, {e u}, {i o}, {i u}, {o u}
(en este caso {a u} y {u a} son la misma combinacin).

1. Calculemos 5 ! y 8 ! Ejercicios
5!
Segn la definicin: resueltos
5! = 1 2 3 4 5 = 120
8! 12345678
= = 336
5! 12345
2. Disponemos de 3 lneas de buses para viajar de la ciudad A a
la ciudad B y de 5 para viajar de B a C. De cuntas maneras
podemos viajar de A a C pasando por B?
Vemos que cada una de las 3 lneas de A a B las podemos
combinar con cada una de las 5 que hay para viajar de B a C.
Por lo tanto, aplicamos el principio multiplicativo y el total
de posibilidades es 3 5 = 15.
3. Cuntos mens diferentes podemos escoger si en el restaurant
se dispone de 5 entradas diferentes, 4 platos de fondo y
6 postres?
Aplicando la regla del producto vemos que la eleccin del
men que debe constar de 1 entrada, 1 plato de fondo y 1
postre se puede efectuar de:
5 x 4 x 6 = 120 maneras.
4. De cuntas maneras puedo elegir 2 fichas de colores diferentes
si cuento con 3 fichas rojas, 4 azules y 7 amarillas?
Las fichas pueden ser: roja-azul; roja-amarilla y azul-amarilla.
1 roja y 1 azul se pueden escoger de 3 x 4 = 12 maneras
distintas.
1 roja y 1 amarilla, de 3 x 7 = 21 maneras, y 1 azul y 1 amarilla,
de 4 x 7 = 28 maneras.
En total, aplicando el principio de la suma puedo escoger 2 fichas
de colores diferentes de 12 + 21 + 28 = 61 maneras.
n n
5. Demostremos que k
=
n k
(n > k)
Apliquemos la definicin para obtener cada miembro de
la igualdad:
n n! (1)
=
k k ! ( n k) !

n n! n! (2)
= =
n k (n k) ! (n (n k)) ! (n k) ! k !
y comparando (1) y (2) vemos que la igualdad se cumple.

Anlisis combinatorio, Teorema del binomio y Elementos de probabilidades 521

520-521 521 25/11/02, 1:15 PM


6. Determinemos el valor de x de modo que se cumpla:
x
= 10
2
Aplicando la definicin tenemos:
x!
= 10
2 ! x 2 !
Esto es:
1 2 3 x 3 x 2 (x 1) x
= 10
1 2 1 2 3 x 3 x 2
Simplificando nos queda:
x x 1
= 10 es decir, x2 x 20 = 0
2
Las soluciones de esa ecuacin son x1 = 5 y x2 = 4,
pero en este caso no nos sirven las soluciones negativas, por lo
tanto, el valor de x es x = 5.

7. Cuntas palabras no necesariamente pronunciables pueden


formarse con las letras de la palabra VESTIDO? (no pueden
repetirse las letras ni pueden omitirse).
Se trata de formar permutaciones con 7 elementos, y como
sabemos, el nmero de ellos est dado por 7 ! = 1 2 3
4 5 6 7 = 5.040
8. Cuntas de las palabras obtenidas en el ejercicio anterior
empiezan con V y terminan con O?
Aqu, de las 7 letras de que disponemos, debemos dejar 2 fijas
(la primera y la ltima) y por lo tanto debemos permutar las otras
cinco restantes. El nmero de ellas es:
5 ! = 1 2 3 4 5 = 120
9. Cuntos nmeros diferentes de 3 cifras se pueden formar
con los dgitos del 1 al 9 si no se permite la repeticin de
un dgito?
Nos piden el nmero de variaciones o arreglos de 3 elementos
tomados de un conjunto de 9. El nmero est dado por:
9! 9!
V 9
3
=
9 3 !
=
6!
= 7 8 9 = 504

El problema puede ser planteado tambin de la siguiente


manera:
Debemos llenar 3 casilleros y para ello disponemos de 9
elementos.
El primero puede ser llenado con cualquiera de los 9 nmeros una
vez llenado ste, el segundo se puede ocupar con cualquiera de
los ocho restantes, pues no podemos repetir los nmeros y una vez
llenado ste, para el tercero nos quedan 7 posibilidades.
Aplicando el principio de la multiplicacin, la cantidad de
nmeros que podemos formar es: 9 8 7 = 504.

522 Anlisis combinatorio, Teorema del binomio y Elementos de probabilidades

522-523 522 10/11/01, 12:34 PM


CAPITULO 13

10. Resolvamos el problema anterior si se permite la repeticin


de los dgitos.
Procedamos a llenar 3 casilleros con 9 elementos diferentes.
El primero se puede llenar con cualquiera de los 9 elementos;
como se permite la repeticin, el segundo se puede llenar tambin
con cualquiera de los 9 y, por la misma razn, el tercer casillero
tambin se puede llenar de 9 maneras diferentes.
Por el principio multiplicativo la cantidad de nmeros que se
puede formar est dada por 9 9 9 = 729.

11. Cuntos subconjuntos de 4 elementos se pueden formar a


partir de un conjunto de 6?
Debemos encontrar el nmero de combinaciones de 4 elementos
tomados de un conjunto de 6. Este nmero est dado por:
6 6! 6! 123456
C 4
=
4 ! (6 4) !
=
4! 2!
=
123412
= 15

Ejemplo: Consideremos el conjunto A = {a, b, c, d, e, f}. Los


subconjuntos de 4 elementos son:

{a, b, c, d} {a, b, c, e} {a, b, c, f} {a, c, d, e} {a, c, d, f}


{b, c, d, e} {b, c, d, f} {b, d, e, f} {a, d, e, f} {c, d, e, f}
{a, b, d, e} {a, b, d, f} {a, b, e, f} {a, c, e, f} {b, c, e, f}

12. De un total de 15 nios y 6 nias se desea escoger un grupo de 6.


De cuntas maneras puede hacerse esta eleccin?
En total contamos con 21 nios (entre hombres y mujeres). Como
no hay restriccin de ningn tipo debemos formar conjuntos de 6
a partir de 21. El total de maneras est dado por:
21 21 21 ! 16 17 18 19 20 21
C 6
=
6
=
6 ! 21 6 !
=
1 2 3 4 5 6
= 54.264

13. Resolvamos el ejercicio anterior si el grupo que se desea formar


debe constar de 3 nios y de 3 nias exactamente.
Debemos elegir 3 nios de un total de 15, esto es:
15 15 13 14 15
C3= 3
=
1 2 3
= 455

y debemos elegir 3 nias entre 6.


6 6 456
C3 = 3
=
123
= 20

En total el grupo se puede formar de acuerdo con el principio


multiplicativo de:

C 15
3 C 6
3 = 455 20 = 9.100 maneras distintas.

14. Resolvamos el problema anterior con la condicin de que en el


grupo debe haber siempre 2 nios fijos y una nia fija (y debe
constar de 3 nios y 3 nias).

Anlisis combinatorio, Teorema del binomio y Elementos de probabilidades 523

522-523 523 10/11/01, 12:34 PM


Como debe haber 2 nios fijos, debemos escoger 1 de un
total de 13.

C 13
1
= 13

Como debe haber 1 nia fija, debemos escoger las otras 2


de un total de 4.
4 34
C 4
2
=
2
=
12
= 6

y el total de maneras en que se puede formar el grupo con


esas condiciones es:

C 13
1
C 42 = 13 6 = 78

Ejercicios
1. Calcule:
x x x
a) 4 ! b) 10 ! c) 7 ! c) + = 10 d) =3
2 3 2

d) 10 ! e) 15 ! f) 30 ! x x
7! 11 ! 28 ! e) = 15 f) = 28
2 x2
n! k + 1 !
g) 5 ! 7 ! h) i) x
3! 4! n 2 ! k 1 ! x
g) + = 28
5 6
5 12
j) k) l) 10
4 9 3 x x x
h) 2 +3 =0
1 2 3
7 7 5 5
m) + n) +
2 3 3 2 x x x
i) +2 + = 20
1 2 3
6 4 6 5 5
o) + p) +2
4 2 3 3 2 x
j) = 36
x2
6
7! + 8!
3 3. Compare:
n
q) n + 1 : r)
n n1 6
7
4
6 5 5 6
a) + con
3 4 4
5
10 ! + 9!
2
s) t) 3 ! + 4 ! + 5 ! 9 9 10
5 25 b) + con
8!
3
9 3 4 4

n! + n+1 ! + n+2 ! 15 15 16
u) c) + con
2
n+2 6 7 7
2. Determine el valor de x de modo que
la igualdad se cumpla: 8 8 9
d) + con
4 5 5
x x
a) =7 b) =6
6 2

524 Anlisis combinatorio, Teorema del binomio y Elementos de probabilidades

524-525 524 10/11/01, 12:39 PM


CAPITULO 13

4. Demuestre: 13. De cuntas maneras se pueden


ordenar en un estante 12 libros?
n n n+1
+ = 14. De cuntas maneras se pueden
k k+1 k+1
ordenar en un estante 12 libros si
5. Compare: tres de ellos deben estar siempre
juntos?
a) 2 ! + 3 ! + 4 ! con 2 ! 15. Hay 5 libros de qumica, 4 de fsica
16 y 6 de matemtica. De cuntas
b) 6! + 7! + 8! con 6 ! maneras se pueden ordenar en un
64 estante si deben quedar juntos los
c) 4! + 5! + 6! con 4! libros de cada tema?
36 16. Cuntas palabras se pueden formar
d) 8! + 9! + 10! con 8! con las letras de la palabra LIBRO?
100 17. Cuntas de las palabras del ejercicio
6. Demuestre: anterior empiezan con consonante?
n! + n + 1 ! + n + 2 !
18. Cuntas de las palabras del ejercicio
= n! 16 terminan con o?
2
n + 2 19. Cuntas palabras de 4 letras se
7. Compare: pueden formar con las letras de la
palabra CLAUDIO si no se pueden
5 4 5 repetir?
a)
3
: 2
con
3 20. Cuntos nmeros de 4 cifras se
pueden formar con los dgitos 0, 1, 2,
7 6 7 ..., 9 si deben empezar con 4 y no se
b) 5 : 4
con
5 pueden repetir los dgitos?
21. Ejercicio 19: si se permite la repe-
c) ticin?
3 2
1
: 0
con 3 22. Cuntos nmeros pares de 2 cifras
se pueden formar con los dgitos
{0, 1, 2, 3, 4} si no se permite la
d) 9 8 9 repeticin?
7
: 6
con
7 (Recuerde, el nmero 04 no se
considera de 2 cifras).
8. Demuestre:
23. Ejercicio 20: si se permite la repe-
n + 1 n n + 1 ticin?
n 1
: n 2
=
n 1 24. Cuntas permutaciones pueden
hacerse con las letras de la palabra
9. Cuntos nmeros diferentes de 5 PINCEL?
cifras se pueden formar con los dgitos 25. Cuntas de las permutaciones del
1, 2, ..., 9 sin repetir? ejercicio anterior comenzarn con P?
10. De cuntas maneras se pueden ubicar 26. Cuntas de las permutaciones del
6 personas en una fila? ejercicio anterior comenzarn con P
11. De cuntas maneras se pueden ubicar y terminarn con L?
6 personas en una mesa redonda? 27. Cuntas palabras pueden formarse
12. De cuntas maneras se pueden sentar con 24 consonantes y 5 vocales si
en una mesa redonda 3 matrimonios cada una debe estar formada por
si cada esposo debe tener a su lado 3 consonantes y 3 vocales y no se
a su mujer? permite la repeticin?

Anlisis combinatorio, Teorema del binomio y Elementos de probabilidades 525

524-525 525 10/11/01, 12:39 PM


28. Cuntas palabras pueden formarse 39. Ejercicio 38: si del total de nios se
con 24 consonantes y 5 vocales si excluyen 8 nios y 5 nias?
cada una debe estar formada por 3
consonantes y 3 vocales y deben ir 40. Ejercicio 38: si deben estar incluidos
intercaladas? siempre 2 nios y 1 nia?

29. Cuntas patentes se pueden formar 41. Cuntos grupos de 5 personas se


si stas constan de 2 letras seguidas de pueden formar entre 4 nios y 7
4 dgitos y no se permite la repeticin? nias si debe haber por lo menos 2
(Considere 27 letras y 10 dgitos) nias incluidas?

30. Ejercicio 29: si se permite la repeti- 42. Ejercicio 41: si debe haber a lo ms
cin? dos nias incluidas?

31. Cuntos nmeros de 5 dgitos se 43. En un curso hay 19 nias y 21 nios.


pueden formar si la cifra de las De cuntas maneras se pueden
unidades es 5 y la de la unidad de mil formar grupos de 5 personas que
es 3 y no se permite la repeticin? incluyan:
a) 2 nias y 3 nios.
32. De cuntas maneras se pueden
b) 3 nias y 2 nios.
seleccionar 5 discos de un total de
15? c) 4 nias.
d) 4 nios.
33. De cuntas maneras se pueden
seleccionar 3 personas de un total 44. En un curso hay 15 nios y 16 nias
de 8? y se desea seleccionar un grupo de
14 alumnos. De cuntas maneras se
34. De cuntas maneras se pueden puede hacer esta seleccin si:
ordenar 8 hermanos en una fila si el a) no hay restriccin.
menor debe estar en primer lugar y el
b) debe haber 7 nios y 7 nias.
mayor en ltimo lugar?
c) hay 5 nios y 5 nias excluidos.
35. De cuntas maneras se pueden d) hay 4 nios y 5 nias incluidos.
ordenar 12 libros en un estante si
2 de ellos deben quedar siempre 45. La primera divisin de la Liga de
separados? Ftbol consta de 25 equipos. Cuntos
partidos deben jugarse para completar
36. Cuntos colores diferentes se pueden la primera rueda?
obtener mezclando 2 cada vez de
un total de 12? 46. En una sala de clases hay 22 sillas. De
cuntas maneras se pueden ubicar
(Se supone que la mezcla se hace 15 alumnos?
siempre en la misma proporcin).
47. Cuntos nmeros menores o iguales
37. De un total de 16 nios y 12 nias de 3.000 y mltiplos de 5 se pueden
se desea escoger un grupo de 6. De formar con los dgitos 0, 1, 2, 3,
cuntas maneras puede hacerse esta 4, 5?
eleccin?
48. De cuntas maneras se puede formar
38. Ejercicio 37: si el grupo debe constar un comit de 3 personas de un total
de 3 nios y 3 nias? de 8?

526 Anlisis combinatorio, Teorema del binomio y Elementos de probabilidades

526-527 526 25/11/02, 1:16 PM


CAPTULO 13

49. De un total de 8 personas, de cuntas 55. Cuntas diagonales tiene un oct-


maneras pueden ocupar 3 cargos gono?
diferentes?
56. Cuntas diagonales tiene un polgono
50. De un total de 8 personas, de cuntas de n lados?
maneras se puede formar un comit
de 4 personas si una de ellas debe 57. Se dispone de 10 puntos donde no
estar siempre incluida? hay 3 colineales. Cuntos tringulos
51. En una liga de 12 equipos de ftbol, se pueden formar?
cuntos partidos deben jugarse si
58. De un total de x personas se pueden
cada equipo juega 2 veces con cada
formar 10 grupos de 2. Determine x.
uno de los restantes?
52. De cuntas maneras se pueden 59. De un total de x personas se pueden
seleccionar 3 libros de un total de formar 21 grupos de 5. Determine x.
10?
60. Cuntos nmeros pares de 3 cifras,
53. Cuntas diagonales tiene un pent- entre 400 y 700, se pueden formar si
gono? no se permite la repeticin?
54. Cuntas diagonales tiene un hex-
gono?

Soluciones

1. a) 24 b) 3.628.800 c) 5.040 9. 9 8 7 6 5 = 15.120 10. 6! = 720


d) 720 e) 32.760 f) 870
11. 5! = 120 12. 2! 23 = 16
g) 4.200 h) n (n 1) i) k (k + 1)
j) 5 k) 220 l) 120 13. 12! 14. 3! 10!
m) 56 n) 20 o) 21
15. 5! 4! 6! 3! 16. 5! = 120
p) 30 q) n + 1 r) 224
n
17. 3 4 3 2 1 = 72 18. 4! = 24
s) 2 t) 3! u) n!
19. 4 5 6 7 = 840 20. 9 8 7 = 504
2. a) x=7 b) x = 4 c) x = 4
d) x=3 e) x = 6 f) x = 8 21. 7 7 7 7 = 2.401
g) x=7 h) x = 3 i) x = 4
j) x=9 22. 10 (22 y 44 estn excluidos)

3. a) ambos son iguales a 15 23. 1.000 24. 6! = 720


b) ambos son iguales a 210
25. 5! = 120 26. 4! = 24
c) ambos son iguales a 11.440
d) ambos son iguales a 126 27. 24
5
6! = 14.572. 800
3 3
5. a) son iguales b) son iguales
c) son iguales d) son iguales 28. 2 (24 23 22 5 4 3) = 1.457.280

7. a) son iguales b) son iguales 29. 27 26 10 9 8 7 = 3.538.080


c) son iguales d) son iguales
30. 27 27 10 10 10 10 = 7.290.000

Anlisis combinatorio, Teorema del binomio y Elementos de probabilidades 527

526-527 527 25/11/02, 1:16 PM


31.7 7 6 = 294 32. (154) = 3.003 33. ( 83 ) = 56 34. 6! = 720

36. ( ) = 66 (286) = 376.740


12
35.12! 11! 2 = 399.168.000 37.
2
38. (163) (123) = 123.200 39. ( 83 ) ( 73 ) = 1.960 40. ( ) ( ) = 770
14
1

11
2

41. 2 m y 3 h. Q ( ) ( ) = 84
7 4
2 3
3 m y 2 h. Q ( ) ( ) = 210
7 4
3 2
4 m y 1 h. Q ( ) ( ) = 140
7 4 84 + 210 + 140 + 21 = 455
4 1
Q ( )
7
5m = 21
5
42. 1 m y 4 h. Q ( ) ( ) =
7 4
7
1 4
7 + 84 = 91
2 m y 3 h. Q ( ) ( ) = 84
7 4
2 3

43. a) ( ) ( ) = 227.430
19 21
47. 1 dgito 1
2 3
2 dgitos 10
b) ( ) ( ) = 203.490
19 21 1 + 10 + 60 + 145 = 216
3 2 3 dgitos 60
4 dgitos 145
c) ( ) ( ) =81.396
19 21
4 1
48. ( ) = 56
8
49. 8 7 6 = 336
d) ( ) ( ) = 113.715
19 21 3

1 4
50. ( ) = 35
7
51. 11 12 = 132
44. a) ( ) b) ( ) ( )
31 15 16 3

14 7 7
52. ( ) = 120
10
53. 5
c) ( ) d) ( )
21 22 3
14 5
25 24
54. 9 55. 20
45. = 300
2 n n3

46. V 22
15
56.
2
57. (103) = 120
58. x = 5 59. x = 7 60. 104

13.2 Teorema
del binomio

13.2.1 Conceptos y
observaciones bsicas
En el punto anterior nos familiarizamos con la expresin ( nk )
que leamos "n sobre k"; nos ser de gran utilidad en esta seccin
donde desarrollaremos cualquier potencia de binomio sin hacer
multiplicaciones tediosas.

La expresin
n
k ( )
recibe el nombre de coeficiente binomial.
Desarrollando algunas potencias de binomios tenemos:
528 Anlisis combinatorio, Teorema del binomio y Elementos de probabilidades

528-529 528 10/11/2001, 19:11


CAPITULO 13

(a + b)0 = 1
(a + b)1 = a + b
(a + b)2 = a2 + 2ab + b2
(a + b)3 = a3 + 3a2 b + 3 ab2 + b3
(a + b)4 = a4 + 4a3 b + 6 a2 b2 + 4 ab3 + b4

Observaciones.

1. El desarrollo de (a + b)n tiene n + 1 trminos.


2. El primer trmino es an y el ltimo es bn.
3. El exponente de a decrece mientras el de b aumenta en
1 unidad.
4. La suma de ambos exponentes es siempre constante, igual
a n.
5. Los coeficientes de los trminos equidistantes del centro
son iguales.
6. Si comparamos los coeficientes de los trminos obtenidos
en los desarrollos con los coeficientes binomiales, podemos
ver que son iguales.
Ejemplo:

n = n =1
0 n

n = n =n
1 n1

n = n n n1
2 n2
= ... etc.
2

Con estas observaciones estamos en condiciones de formular


el teorema del binomio.

13.2.2 Teorema del binomio


El desarrollo de la potencia nsima del binomio (a + b)
corresponde a:
n
(a + b)n =
k=0
n
k an k bk

en forma extensiva;

(a + b)n = n an + n1 an 1 b + n2 an 2 b2 + ...
0

+ ... + n
n1 abn 1 + nn bn
Ejemplo:
Desarrollemos (a + b)4 aplicando el teorema:

(a + b)4 = 4 a4 + 4 a3 b + 4 a2 b2 + 4 ab3 + 44 b4
0 1 2 3

Anlisis combinatorio, Teorema del binomio y Elementos de probabilidades 529

528-529 529 10/11/2001, 19:12


siendo: 4 = 4 = 1
0 4
4
1
= 4 =4
3
4 =6
2
entonces:
(a + b)4 = a4 + 4 a3 b + 6a2 b2 + 4 ab3 + b4

Si queremos determinar el trmino de orden r (o lugar r)


en el desarrollo de (a + b)n, se est dado por:
n
tr = r1
an r + 1 br 1
es decir, se obtiene haciendo k = r 1
Por ejemplo, el tercer trmino de (a + b)6 est dado por:

t3 = 6 a4 b2 (k = 2)
2
Recordemos que en el desarrollo del binomio, (a + b)n
k vara desde 0 hasta n;
As, el primer trmino se obtiene para k = 0
2 trmino se obtiene para k = 1
3 trmino se obtiene para k = 2
4 trmino se obtiene para k = 3
r trmino se obtiene para k = r 1
En forma ms general, podemos expresar:
tk + 1 = n an k bk
k

13.2.3 El tringulo de Pascal


Los coeficientes de los trminos de un binomio tambin se
pueden obtener construyendo el siguiente ordenamiento, conocido
como tringulo de Pascal-Tartaglia:

n= 0
n= 1
n= 2
n= 3
n= 4
n= 5

La construccin de este tringulo se efecta partiendo y terminando


con 1 y cada trmino se obtiene sumando en la fila anterior los dos
nmeros ms prximos.

530 Anlisis combinatorio, Teorema del binomio y Elementos de probabilidades

530-531 530 25/11/02, 1:18 PM


CAPITULO 13

As, para n = 6 hacemos:

1 6 15 20 15 6 1

y, por lo tanto, si queremos obtener (a + b)6, simplemente


completamos los factores literales de cada trmino tomando en
cuenta las consideraciones anteriores: As:

(a + b)6 = a6 + 6a5b + 15a4b2 + 20a3b3 + 15a2b4 + 6ab5 + b6

1. Desarrollemos (2a + 3b)4 Ejercicios


Sabemos que el desarrollo tiene 5 trminos y, de acuerdo al
tringulo de Pascal (o usando los coeficientes binomiales), los resueltos
coeficientes son 1 4 6 4 1.
Ahora, el primer trmino es 2a y el segundo es 3b
El desarrollo es:
(2a + 3b)4 = 1 (2 a)4 + 4 (2 a)3 (3 b) + 6 (2 a)2 (3 b)2 + 4
(2 a)1 (3 b)3 + (3 b)4
completando el desarrollo, nos queda:
(2 a + 3 b)4 = 16 a4 + 96 a3 b + 216 a2 b2 + 216 a b3 + 81 b4
2. Determinemos el 6 trmino en el desarrollo de (a + 2b)11

Sabemos que: t6 = 11 a6 (2b)5


5
t6 = 462 a6 32 b5
t6 = 14.784 a6 b5
3. Cul es el coeficiente de x14 en el desarrollo de (x2 + x3)6.
Sabemos que el trmino general est dado por:

n
tk = k 1 (x2)n k + 1 (x3)k 1

en este caso:

6
tk = k 1 (x2)6 k + 1 (x3)k 1

6
= k 1 x14 2k x3k 3

6
= k 1
xk + 11

queremos el coeficiente de x14, es decir, debemos igualar


exponentes:
xk + 11 = x14
o sea k=3
y el trmino que contiene a x14 en ese desarrollo es el tercero
y su coeficiente es:
6 = 15
2

Anlisis combinatorio, Teorema del binomio y Elementos de probabilidades 531

530-531 531 25/11/02, 1:19 PM


4. Determinemos el exponente de x en el 8 trmino del desarrollo
de:
10
1
x3 +
x2
7
t8 = 10 3 1
x3
7 x2

= 10 x9 1
7 x14
= 120 x5
y el exponente de x en el 8 trmino es 5.
5. Determinemos el trmino independiente de x en el desarrollo
de:
6
1
x4
x2

El trmino general es:


6 k
1
t k + 1= 6 x4 k
k x2
6 k
1
= 6 1 k
x4 k
k x2
1
= 6 1 k
x4 k
k x12 2 k

= 6 1 k
x6 k 12
k
para que el trmino sea independiente de x es necesario que el
exponente de x sea igual a 0.
es decir: 6k 12 = 0
k=2
y para k = 2 tenemos:
4
6 1 x4 2
t3 = 2 x2

1
= 62 1 2 x8
x8

= 6
2

= 15
el trmino independiente de x en el desarrollo es el 3er trmino
y es igual a 15.

Ejercicios

1. Obtenga los siguientes desarrollos: c) (1 + 3a)7 d) (1 b)11

a) (x + y)5 b) (x 2y)5 e) (2a 3b)4 f) (x 1 )6


x

532 Anlisis combinatorio, Teorema del binomio y Elementos de probabilidades

532-533 532 10/11/01, 12:49 PM


CAPITULO 13

7
h) penltimo trmino de a2 a
4 8
g) 12 + z2 h) m 1 3
z 2 2
6
3 x3 6 i) ltimo trmino de 1)11(3x11
p 2q
i) j)
j) trmino central de (1 5x)6
2q p x4 4
2. Determine el trmino indicado en el 4. En el desarrollo de 1 5
deter-
3 x2
desarrollo correspondiente: x
mine:
a) 7 trmino en (x y)11
a) el coeficiente numrico del cuarto
b) 5 trmino en (a + b)21 trmino
10
c) 10 trmino en 1 1 b)el trmino que contiene x4
a b
2 7 c) el trmino independiente de x
d) 8 trmino en x2 y
x y2 5. En el desarrollo de
e) 11 trmino en (2a b)10 2
3 x y3
12

x2 y
determine:
4
1
f) 2 trmino en 1 xyz
a) trmino general
11
g) 9 trmino en 2 2 +3 3
b) trmino que contiene x 3
6
h) 4 trmino en 2 3 c) trmino que contiene y12
9
i) 6 trmino en 3 d) trmino independiente de x
a a

x5 2
15 e) trmino independiente de y
j) 13 trmino en 2
5
x 6. Determine el trmino que contiene q9
3. Determine el coeficiente numrico en los siguientes desarrollos:
del trmino indicado:
a) (2p + q)11
a) 2trmino en (2x y)4
10
b) 3ertrmino en (3a + 4b)6 b) 1
q pq
10
c) 9trmino en x2 y2
y

x
c) (p2 q3)7
d) 5trmino en ( a + 12)5 1 3
d) 3 q5 q
e) 8trmino en (p2v2 1)14
f) trmino central (2x2 y + xy3 )8
17 7. Encuentre los 3 primeros trminos en
2 1
g) ltimo trmino 10
a2 b2 ab el desarrollo de 2 x + 3

Soluciones

1. a) x5 + 5x4 y + 10x3 y2 + 10x2 y3 + 5xy4 + y5

b) x5 10x4 y + 40x3 y2 80x2 y3 + 80xy4 32y5

c) 1 + 21a + 189a2 + 945a3 + 2.835a4 + 5.103a5 + 5.103a6 + 2.187a7

d) 1 11b + 55b2 165b3 + 330b4 462b5 + 462b6 330b7 + 165b8 55b9 + 11b10 b11

e) 16a4 96a3b + 216a2b2 216ab3 + 81b4

Anlisis combinatorio, Teorema del binomio y Elementos de probabilidades 533

532-533 533 10/11/01, 12:50 PM


f) x6 6x4 + 15x2 20 + 15 6 + 1
2 4 6 x x x
g) 1 + 4 + 6 + 4z4 + z8
z8 z4
h) 1 (m8 8m7 + 28m6 56m5 + 70m4 56m3 + 28m2 8m + 1)
256
p6 3 p4 15 p2 q2 q4 q6
i) + 20 + 60 96 + 64
64 q6 8 q4 4 q2 p2 p4 p6
18
j) 729 729 + 1.215 135 + 135 x4 9 x11 + x
x24 2 x17 16 x10 16 x3 256 512 4.096

2. a) a7 = 462x5 y 6 h)
7
i) 1 j) 2.500
729
b) a5 = 5.985a17 b4 4. a) 90 b) a3 = 270x4
10 c) no existe
c) a10 =
a b9
128 5. a) ak + 1 = 12 212k ( 3)k x3k 24 y4k 12
d) a8 = k
x7 y14
b) a8 = 12 37 25 x3 y16
e) a11 = b10 7
4 c) a7 = 12 26 36 x 6 y12
f) a2 = xyz 6
g) a9 = 2.640 312 2 d) a9 = 12 24 38 y20
8
h) a4 = 120 6
e) a4 = 12 29 33 x15
3
i) a6 = 30.618 6. a) a10 = 220 p2q9
a

j) a13 = 232.960 b) no hay


x45
3. a) 32 b) 19.440 c) 45 c) a4 = 73 p6 q9

d) 5 124 e) 3.432 d) a2 = 27 q9

f) 1.120 g) 1 7. 32x10 + 160 6 x9 + 2.160 x8

13.3 Elementos de
probabilidades

13.3.1 Conceptos bsicos


Se llama experimento determinstico a aquel en el cual el
resultado se puede predecir, es decir, siempre que se realice en
condiciones semejantes se obtendr el mismo resultado.
Un experimento aleatorio es aquel en el cual no es posible
predecir el resultado aunque ste se realice en las mismas
condiciones.
Al conjunto de resultados posibles de obtener a partir de un
experimento aleatorio se le llama Espacio muestral.

534 Anlisis combinatorio, Teorema del binomio y Elementos de probabilidades

534-535 534 10/11/01, 12:53 PM


CAPITULO 13

Cualquier subconjunto del Espacio muestral se denomina


Evento o suceso. Se pueden clasificar en:
a) evento cierto o seguro es aquel que est formado por
todo el Espacio muestral.
b) evento imposible es el subconjunto vaco del espacio
muestral.
c) eventos incompatibles o mutuamente excluyentes son
aquellos que no pueden suceder simultneamente.
d) eventos complementarios son aquellos cuya unin es
el Espacio muestral y cuya interseccin es el conjunto
vaco.

La probabilidad de que un evento o suceso ocurra es el


cociente entre el nmero de casos favorables y el nmero
de casos posibles.

As, si la probabilidad de que ocurra el suceso A, designada


P (A), est dada por:
Nf
P A =
Np
donde Nf denota el nmero de casos favorables y
Np denota el nmero de casos posibles de ocurrencia
del experimento.
Observaciones.
1. La probabilidad de un evento cierto es 1.
2. La probabilidad de un evento imposible es 0.
3. Para cualquier evento A, se tiene que la probabilidad de
A, denotada P (A), es:
0 P (A) 1
4. Si A y B son eventos complementarios entonces:
P (A) = 1 P (B)

13.3.2 Probabilidad de la unin y


de la interseccin de dos eventos
Sean A y B dos eventos del Espacio muestral E.
Si A y B son mutuamente excluyentes, entonces la probabilidad
de que ocurra la unin de los dos est dada por:
P (A K B) = P (A) + P (B)
Si A y B no son mutuamente excluyentes entonces la
probabilidad de ocurrencia de A K B est dada por:
P (A K B) = P (A) + P (B) P (A J B)

Anlisis combinatorio, Teorema del binomio y Elementos de probabilidades 535

534-535 535 10/11/01, 12:53 PM


Ahora, si A y B son eventos independientes, la probabilidad de
ocurrencia de A J B est dada por:

P (A J B) = P (A) P (B)

Si A y B no son eventos independientes, es decir, la ocurrencia


de uno de ellos, por ejemplo de A, influye sobre la ocurrencia del
otro, en este caso de B, entonces la probabilidad de ocurrencia
de A J B est dada por:

P (A J B) = P (A) P (B | A)

donde P (B | A) es la probabilidad de ocurrencia de B condicionada


por la ocurrencia de A.

Ejercicios 1. Calculemos la probabilidad de que al lanzar una moneda


resueltos salga cara.
Al lanzar una moneda tenemos dos resultados posibles: CARA -
SELLO. Ese es entonces nuestro Espacio muestral. La cantidad de
elementos del Espacio muestral nos indica el nmero de casos
posibles Np. El nmero de casos favorables en este ejemplo
es uno solo: sale cara.
As, la probabilidad pedida es:

1 n casos favorables
P=
2 n casos posibles
2. Calculemos la probabilidad de que al lanzar un dado nos salga
un nmero menor que 4.
Al lanzar el dado podemos obtener 6 resultados distintos. El
Espacio muestral es E = {1, 2, 3, 4, 5, 6}.
Los casos favorables son 3: {1, 2, 3}
La probabilidad pedida entonces es:
3 1
P= =
6 2

3. Al lanzar un dado, cul es la probabilidad de obtener un nmero


par o bien obtener un nmero menor que 3?
Sea E el Espacio muestral: E = {1, 2, 3, 4, 5, 6}
Es decir, el N de casos posibles es Np = 6
Sea A el evento: A = obtener nmero par.
Es decir, A = {2, 4, 6} Q NA = 3
Sea B el evento: B = obtener nmero menor que 3.
B = {1, 2} Q NB = 2
Vemos que A J B = {2} Q N (A J B) = 1

536 Anlisis combinatorio, Teorema del binomio y Elementos de probabilidades

536-537 536 10/11/01, 12:54 PM


CAPITULO 13

Claramente A y B no son mutuamente excluyentes, entonces la


probabilidad de que ocurra A K B es:
P(AKB) = P(A) + P(B) P(A J B)
3 + 2 1
=
6
P(AKB) = 2
3
4. Calculemos la probabilidad de que al lanzar 3 monedas,
obtengamos una cara y dos sellos.
Cada moneda origina 2 resultados posibles: cara o sello; como
son 3 monedas, el Espacio muestral consta de 8 elementos. Tres
de esos ocho elementos cumplen la condicin pedida, as la
probabilidad buscada es 3 .
8

Ejercicios
1. Determine cules de los siguientes comprando 4 nmeros, si hay 10 listas,
experimentos son predeterminados cada una con 20 nmeros?
(o determinsticos) y cules son 4. Cul es la probabilidad de obtener
aleatorios. siete puntos en el lanzamiento de
a) jugar una cartilla de apuesta dos dados?
deportiva 5. Cul es la probabilidad de no obtener
b) mezclar azcar y agua nmero par en el lanzamiento de
c) enfriar agua a 0 C un dado?

d) lanzar una piedra y medir 6. Al lanzar dos monedas, qu probabi-


su alcance lidad hay de:
e) comprar un nmero de rifa a) obtener dos caras
f) apostar en una carrera de caballos b) obtener 1 cara y 1 sello
g) preguntarle a un desconocido c) obtener lados iguales
si fuma 7. En el lanzamiento de 1 dado, cul es
2. Seale el Espacio muestral de los la probabilidad de:
siguientes experimentos: a) obtener el N 5
a) lanzar una moneda b) no obtener el N 5
b) lanzar dos monedas c) obtener 3 o 5
c) lanzar 3 monedas d) obtener nmero menor que 5
d) lanzar 1 dado
8. En el lanzamiento de dos dados, cul
e) lanzar 2 dados es la probabilidad de que:
f) asignar el premio en una rifa para a) la suma sea 11
la cual hay 10 listas con 10 nmeros
b) la suma sea mayor que 9
cada una
c) la suma sea menor que 4
3. Cul es la probabilidad de ganar
uno de los 5 premios de una rifa d) no salgan nmeros iguales

Anlisis combinatorio, Teorema del binomio y Elementos de probabilidades 537

536-537 537 10/11/01, 12:55 PM


9. Al lanzar 1 dado dos veces consecu- 15. En una caja hay 10 bolitas rojas y 6
tivas, qu probabilidad hay de: azules. Cul es la probabilidad de
obtener 3 bolitas azules en 3 sacadas
a) obtener 2 ases
diferentes sin reposicin?
b) obtener primero un 3 y luego un
nmero par 16. En un grupo de 30 personas todos
tienen edades diferentes menores
c) obtener primero un 3 y luego no
de 50 aos. Si se integra una nueva
obtener 3
persona, tambin menor de 50, cul
d) obtener nmero par primero y el es la probabilidad de que su edad
3 despus coincida con la de alguno de los
30?
10. En un naipe ingls (52 cartas) qu
probabilidad hay de: 17. Hay 150 nmeros en una rifa. Cuntos
a) obtener un trbol al sacar una habr que comprar para tener un 8%
carta de probabilidad de ganarla?
b) obtener dos ases en una entrega 18. Qu probabilidad hay de que al
(13 cartas) lanzar 2 dados se obtenga una suma
11. En una caja hay 12 bolas negras y 8 menor que 6?
rojas, qu probabilidad hay de:
19. Hay 16 monedas de $ 100, 22 monedas
a) sacar 1 negra de $ 50 y 12 monedas de $ 10. Al sacar
b) sacar 1 roja una moneda, cul es la probabilidad
de sacar una de $ 100 o de $ 50?
c) sacar 1 negra y, sin reponerla, sacar
luego una roja 20. En un curso la mitad de los alumnos
d) sacar 1 negra y luego de reponerla, son hombres. Si el 40% de los hombres
sacar una roja. sabe ingls y el 50% de las mujeres,
francs, cul es la probabilidad de
12. Se lanza un dado y sale 4. Qu que al elegir un alumno sepa ingls
probabilidad hay de que al lanzarlo de o francs?
nuevo sume con el primer resultado
un nmero menor que 9? 21. En un grupo de 100 personas, 40
gustan de la msica, 30 gustan del
13. Al comprar clavos, la probabilidad deporte, 10 gustan de la msica y
de obtener 1 defectuoso es de 0,015. del deporte y los otros no gustan
Cuntos clavos defectuosos habr ni de la msica ni del deporte. Al
en un paquete que contiene 10 cajas y elegir una persona al azar, cul es la
cada caja contiene aproximadamente probabilidad de que:
40?
a) guste slo de la msica
1 b) guste slo del deporte
14. En un curso de 60 alumnos, de
3
los alumnos habla ingls, 1 habla c) guste de la msica o del deporte
4
1
francs y habla los dos idiomas. d) no guste ni de la msica ni del
10
deporte
Cul es la probabilidad de que un
alumno elegido al azar hable slo e) guste de ambos.
un idioma?

538 Anlisis combinatorio, Teorema del binomio y Elementos de probabilidades

538-539 538 10/11/01, 12:56 PM


CAPTULO13
CAPITULO 13

Soluciones

1. a) aleatorio 1
b) predeterminado 4. o 16, 6 %
6
c) predeterminado 1
d) aleatorio 5. o 50%
2
e) aleatorio 6. a) 1 b) 1 c) 1
f) aleatorio 4 2 2
g) aleatorio 1 5
7. a) b)
6 6
2. a) E = {cara, sello} c) 1 d) 2
3 3
b) E = {cara cara; cara sello; sello cara; 1 1
sello sello} 8. a) b)
18 6
c) E = {cara cara cara; cara cara sello; c) 1
d) 5
cara sello cara; cara sello sello; sello 12 6
1
cara cara; sello cara sello; sello sello 9. a) b) 1
cara; sello sello sello} 36 12

d) E = {1, 2, 3, 4, 5, 6} c) 5 d) 1
36 12
e) E = {1-1, 1-2, 1-3, 1-4, 1-5, 1-6
4 : 48
2-1, 2-2, 2-3, 2-4, 2-5, 2-6 2 11
3-1, 3-2, 3-3, 3-4, 3-5, 3-6 10. a) 1 b)
4 52
4-1, 4-2, 4-3, 4-4, 4-5, 4-6
13
5-1, 5-2, 5-3, 5-4, 5-5, 5-6 11. a) 3 b) 2
6-1, 6-2, 6-3, 6-4, 6-5, 6-6} 5 5
24 6
f) E = {L1,1; L1,2; L1,3 ...; L1,10 c) d)
95 25
L2,1; L2,2; L2,3 ...; L2,10 2
12. 13. 6 14. 23
3 60

3
15. 1 16. 17. 12 nmeros
28 5
5 19
18. 19.
L10,1; L10,2; L10,3 ...; L10,10} 18 25
4 196 20. 45% o 9
1 4 20
3. 21. a) 30% b) 20% c) 60%
200
5 d) 40% e) 10%

Prueba de seleccin mltiple


11! 7 + 7 =
1. Si 2x = 10, entonces el 2. = 3.
2! 9! 2 3
valor de x es: A. 2! B. 2 7 14
A. 5 B. 5
A. 5 B. 4
C. 55 D. 110 8
C. 3 D. 6 C. 4 D. 8
3
E. 10 E. 220 E. Otro

Anlisis combinatorio, Teorema del binomio y Elementos de probabilidades 539

538-539 539 10/11/01, 12:58 PM


Prueba de seleccin mltiple
4. De cuntas maneras se 8. El coeficiente numrico 11. El trmino indepen-
pueden escoger 3 libros del 2 trmino en el diente de x en el desa-
4
de un total de 9? desarrollo de (2 a + b)5 1
es: rrollo de x es:
A. 27 x2
B. 504 A. 16 A. 2

C. 84
B. 32 B. 3
D. 12
C. 80
E. 729 C. 4
D. 10
7 6 = D. ltimo
5.
5 : 4 E. 50
7
A. 9. El trmino central en el E. No hay trmino in-
5
y dependiente de x
7 desarrollo de (3x )7
B. es:
2
6

7 12. De cuntas maneras


C.
4 A. 2.835 x4 y3 se pueden seleccionar
8
3 personas de un total
6 de 7?
D. 2.835
5 B. x4 y3
5 8
E. A. 35
4
C. 945 x3 y4
16 B. 40
6. El ltimo trmino en
el desarrollo de (x 3 945 3 4
y)5 es: D. x y C. 56
16
A. 15 y5 D. 9
E. No hay trmino cen-
B. 15 y5 tral
E. 21
C. 243 y5 10. El trmino central en el
desarrollo de (2 x y)6
D. 243 y5 13. De cuntas maneras
es: se pueden ordenar 5
E. 243 x y5 libros en un estante si
A. 60 x2 y4
2 de ellos deben estar
7. El coeficiente numrico siempre juntos?
del 8 trmino del desa- B. 60 x2 y4
rrollo de (2 x)11 es: A. 4!
C. 160 x3 y3
A. 330 B. 2 4!
B. 330 D. 160 x3 y3
C. 5!
C. 5.280 E. No hay trmino D. 2 5!
central
D. 5.280 5
E. !
2
E. Otro

540 Anlisis combinatorio, Teorema del binomio y Elementos de probabilidades

540-544 540 25/11/02, 1:22 PM


CAPTULO 13

14. De cuntas maneras 17. El valor de B. 18


se puede seleccionar
un grupo de 4 personas 5 + 2 5 + 5 es: C. 27
0 1 5
entre un total de 3 hom-
bres y 5 mujeres? A. 3 D. 120

B. 5 E. 216
A. 32
C. 10
B. 60 21. Una liga de ftbol consta
D.12 de 15 equipos. Cuntos
C. 140 partidos deben jugarse
E. 125 para completar la pri-
D. 70 mera rueda?
18. n + 2 : n+1 =
n+1 n+1 A. 15
E. 144
A. n
B. 30
15. El coeficiente numrico
del tercer trmino del B. n + 1 C. 105
desarrollo de (1 + x)5
es: C. n + 2 D.150

A. 1 D.(n + 1) (n + 2) E. 225
B. 3 n+2
E.
C. 5 n+1 22. De cuntas maneras
se puede escoger un
D.10 19. Cuntas palabras de men (1 entrada-plato
5 letras pueden formarse de fondo-postre) si se
E. Otro. dispone de 3 entradas,
con las letras de la pala-
bra TIGRE? (sin repetir) 3 platos de fondo y 5
16. De las siguientes afir-
postres?
maciones:
A. 60
n n A. 45
I =
k nk B. 120
B. 15
II n + n = n+1 C. 240
k k+1 k+1 C. 11
D.600
n+1 n
III n1
=
n2 D.14
E. 720
son verdaderas: E. 125

A. Slo I
20. Cuntas palabras de 3
B. Slo II letras pueden formarse 23. Cuntas diagonales se
con las letras de la pueden trazar en un
C. I y II palabra FTBOL? (sin polgono de 10 lados?
repetir)
D. Todas A. 10
A. 6
E. Ninguna B. 30

Anlisis combinatorio, Teorema del binomio y Elementos de probabilidades 541

540-544 541 25/11/02, 1:22 PM


Prueba de seleccin mltiple
C. 35 D. 60 D. 1
9
D. 70 E. 120
1
E.
3
E. Otro 27. De cuntas maneras se
puede viajar de A a C 30. Al lanzar 2 dados, cul
pasando por B si hay 6 es la probabilidad de
24. De un grupo de n per- posibilidades de ir de que la suma sea mayor
sonas hay 45 maneras A a B y 4 para ir de B o igual a 10?
de formar parejas. Cul a C?
es el valor de n? 1
A. 24 A.
3
A. 9
B. 12 1
B.
B. 10 4
C. 10
C. 12 C.
1
D. 6 6
D. 15
E. 36 1
D.
E. 20 9
28. Cul es la probabilidad
de obtener un nmero 1
menor que 3 al lanzar E. 12
25. Cuntas palabras se 1 dado?
pueden formar con las 31. La probabilidad de
letras de la palabra obtener 2 nmeros igua-
A. 10%
LPIZ si la A y la P les al lanzar 2 dados
deben aparecer juntas? es:
B. 20%
A. 48
C. 30% 1
A.
2
B. 24
1 1
D. B.
C. 120 3 3
1
D. 60 E. 1
2 C.
4
E. 96 29. Cul es la probabilidad
de obtener 3 caras al D. 1
lanzar 3 monedas? 6
26. De cuntas maneras se
pueden ubicar 6 perso- E.
1
nas en un auto si slo 1 A. 1 12
4
de ellas sabe manejar? 32. La probabilidad de no
1 obtener el 5 o el 3 al
A. 5 B. lanzar 1 dado es:
8

B. 6 1
C.
1 A.
6 4
C. 30 1
B.
3

542 Anlisis combinatorio, Teorema del binomio y Elementos de probabilidades

540-544 542 25/11/02, 1:23 PM


CAPTULO 13

2 1 38. Cul es la proba-


C. B.
3 18 bilidad de obtener
1 1
2 nmeros pares al
D. C.
6 12 lanzar 2 dados?
5
E. 1
6 D. 9 A. 1
6
33. La probabilidad de no
1
obtener una carta de E. 1
corazn de un naipe
6 B.
2
ingls (52 cartas) es: 36. En una caja hay 5 mone-
das de $ 100 y 6 de $ 50. 1
C.
Cul es la probabilidad 3
A. 1
4 de sacar 2 monedas de
$ 100 si se sacan 2 veces 1
3 D.
B. sin reponerlas? 12
4
1
C. 1 A.
3
E. 1
2 4

3 2 39. Al lanzar 3 dados, cun-


D. B.
8 5 tos resultados posibles se
1 2 pueden obtener?
E. C.
8 11
A. 18
34. Al lanzar 2 veces segui- 5
D. 11 B. 30
das 1 dado, cul es la
probabilidad de sacar
E.
4 C. 108
2 ases?
11

37. Una caja contiene 8 D. 196


1
A.
36 bolas rojas y 4 negras.
E. 216
Cul es la probabilidad
1
B. de no sacar una bola
6 40. Al lanzar 1 dado, cul
negra?
1 es la probabilidad de
C. obtener un nmero im-
12
1
A. par o un nmero menor
1 3
D. que 4?
18
B. 2
1 3 1
E. A.
30 3
1
C. 2
35. Al lanzar 2 dados, cul 4 B.
3
es la probabilidad de
1
obtener una suma menor D. C.
1
6
que 3? 2

1 1 D.
A. E.
36 2

Anlisis combinatorio, Teorema del binomio y Elementos de probabilidades 543

540-544 543 25/11/02, 1:26 PM


3 1
E. E.
4 40
41. Al lanzar 2 veces un dado, cul 43. En una carrera corren 8 caballos signados
es la probabilidad de obtener un 6 por nmeros 1, 2, 3, , 8. Cul es la
en el primer lanzamiento y un 2 en probabilidad de que gane el caballo
el segundo? 3 o el caballo 6 si todos tienen igual
probabilidad de ganar?
1
A.
6 1
A.
1
4
B. 3
1
B.
1 3
C.
36
1
C.
1 8
D. 18
1
D.
1 64
E.
12
E. Otro.
42. Cul es la probabilidad de ganar el
premio de una rifa para la cual se venden
20 listas y cada lista tiene 20 nmeros, si
se compran 4 nmeros?
1
A.
10

i
B.
100

1
C.
200

1
D.
50

Soluciones
1. A
2. C 9. E 16. C 23. C 30. C 37. B
3. D 10. D 17. D 24. B 31. D 38. E
4. C 11. E 18. C 25. A 32. C 39. E
5. A 12. A 19. B 26. E 33. B 40. B
6. D 13. B 20. D 27. A 34. A 41. C
7. D 14. D 21. C 28. D 35. A 42. B
8. C 15. D 22. A 29. B 36. C 43. A

544 Anlisis combinatorio, Teorema del binomio y Elementos de probabilidades

540-544 544 25/11/02, 1:27 PM


CAPTULO 14
Problemas

Introduccin
En esta seccin aplicaremos los conocimientos adquiridos
en el planteamiento y resolucin de problemas. Para esto es
necesario seguir algunos pasos:

1. Leer muy bien el enunciado y asegurarse de comprenderlo cabalmente.


2. Identificar los elementos conocidos y desconocidos que all intervengan.
3. Asignar variables correspondientes a los elementos mencionados.
4. Plantear la o las ecuaciones necesarias de acuerdo con el problema.
5. Resolver la o las ecuaciones planteadas.
6. Entregar la solucin del problema.
El ltimo punto es importante porque muchas veces la solucin del problema no
est dada por la solucin directa de las ecuaciones. Muchas veces es conveniente
efectuar una comprobacin de la solucin.

Aplicacin de ecuaciones
lineales enteras 14.1

Ejercicios 1. El doble de un nmero ms 5 es igual al triple del mismo nmero


menos 2. Cul es el nmero?
resueltos
El enunciado es muy simple y hay slo un elemento en cuestin.
Sea x el nmero pedido.
Escribamos los datos en forma de ecuacin:

Problemas 545

545 545 25/11/02, 1:29 PM


2x + 5 = 3x 2
resolviendo la ecuacin, obtenemos:
x=7
que es la solucin del problema.

2. El triple de un nmero disminuido en 18 es igual al mismo nmero


aumentado en 8. Cul es el nmero?
Sea x el nmero pedido:
Tenemos: 3x 18 = x + 8
y resolviendo obtenemos:
2x = 26
x = 13
y sa es la solucin del problema.

3. Al doble de un nmero disminuido en 5 se le agrega el triple del


mismo nmero disminuido en 10 y se obtiene 25. Calcular el
cudruple de dicho nmero.
Sea x el nmero en cuestin:
Tenemos: 2x 5 + 3x 10 = 25
resolviendo la ecuacin:
5x 15 = 25
5x = 40
x =8
el nmero es 8, pero se nos pide el cudruple de l; por lo tanto,
la solucin del problema es 4x = 32.

4. Encontrar tres enteros consecutivos cuya suma sea 54.


Como se trata de nmeros consecutivos se pueden representar
por x, x + 1 y x + 2.
(Tambin, y es ms simple en este caso, por x 1, x, x + 1).
Planteando la ecuacin de acuerdo al problema, tenemos:
x+x+1+x+2 = 54
3x + 3 = 54
3x = 51
x = 17
y los nmeros pedidos son 17, 18 y 19.

5. La suma de 3 nmeros pares consecutivos es 402. Cules


son los nmeros?
Aunque formalmente la expresin de un nmero par est dada
por 2n (n E N), aqu podemos usar la variable simple x, dadas
las condiciones del problema. Como se trata de nmeros pares,
la diferencia entre dos consecutivos de ellos es 2. As, los
nmeros son:
x, x + 2 , x + 4 o (x 2 , x, x + 2).

546 Problemas

546-547 546 10/11/01, 1:33 PM


CAPITULO 14

Entonces la ecuacin es:


x+x+2+x+4 = 402
3x + 6 = 402
3x = 396
x = 132
y los nmeros son 132, 134 y 136.

6. La suma de tres nmeros impares consecutivos es 297. Cul


es el doble del mayor?
Formalmente la expresin que identifica a un nmero impar es
2n + 1 (n E N), pero en este caso, dadas las condiciones del
problema, podemos identificarlo con x. Entonces los nmeros
son x, x + 2 y x + 4.
As: x+x+2+x+4 = 297
3x + 6 = 297
3x = 291
x = 97
Los nmeros son 97, 99 y 101 y el doble del mayor es 202.

7. La suma de las edades de 2 hermanos es 31 aos, si la diferencia


entre ellos es de 3 aos, calculemos la edad de ambos.
Sea x la edad del mayor. Como la diferencia entre sus edades es de
3 aos, el hermano menor tendr entonces x 3 aos.
As: x+x3 = 31
2x 3 = 31
2x = 34
x = 17
Entonces la edad del mayor es 17 aos y la del menor es
14 aos.

8. Dividir el nmero 550 en tres partes tales que la segunda exceda a


la primera en 25 y la tercera exceda a la segunda en 50.
Sea x la primera parte, entonces la segunda ser x + 25 y la
tercera (x + 25) + 50 = x + 75
La ecuacin correspondiente es:
x + x + 25 + x + 75 = 550
3x + 10 = 550
3x = 450
x = 150
y las partes sern, respectivamente: 150, 175 y 225.

9. La edad de Ana es el doble de la edad de Mara y hace 6 aos era 4


veces la de Mara. Determinemos las edades actuales.
Sea x la edad actual de Mara. Entonces
2x es la edad actual de Ana.

Problemas 547

546-547 547 10/11/01, 1:33 PM


Hace 6 aos la edad de Mara era
x 6 y la edad de Ana era 2x 6
y en ese momento la edad de Ana era el cudruple de la edad
de Mara. Es decir:
2x 6 = 4 (x 6)
Resolviendo la ecuacin, tenemos:
2x 6 = 4x 24
18 = 2x
9 =x
Entonces, actualmente Mara tiene 9 aos y Ana tiene 18 aos
(hace 6 aos Mara tena 3 aos y Ana tena 12, es decir, el
cudruple de la edad de Mara).

Ejercicios
1. La suma de un nmero con su doble Cules son los nmeros?
es 18. 9. La suma de 3 nmeros impares
Cul es el nmero? consecutivos es 279.
2. La diferencia de un nmero con su Cul es el mayor?
triple es 16. 10. La suma de 4 nmeros pares consecu-
Cul es el nmero? tivos es 4.620.
3. El doble de un nmero aumentado Cul es el nmero menor?
en 1 es igual al triple del mismo 11. La suma de 4 nmeros consecutivos
nmero disminuido en 4. es 546.
Cul es el nmero? Cul es el doble del menor?
4. El triple de un nmero disminuido 12. Compro 1 kg de azcar y 1 litro de
en 8 es igual al doble del mismo aceite. Si el kg de azcar cuesta $ 30
nmero aumentado en 4. menos que el litro de aceite y en total
Cul es el nmero? gasto $ 650.
5. Siete veces un nmero disminuido Cul es el precio de cada artculo?
en 1 es igual a 6 veces dicho nmero 13. Se compra una docena de pinceles y
aumentado en 3. media docena de cuadernos. Si cada
Cul es el nmero? cuaderno es $ 60 ms caro que cada
pincel y el total de la compra
6. La suma de tres nmeros enteros es de $ 2.520, cul es el precio de
consecutivos es 636. cada artculo?
Cules son los nmeros? 14. Se compran 2 kg de lentejas y 3 kg
7. La suma de tres nmeros enteros de garbanzos. Si el kg de garbanzos
consecutivos es 135. cuesta $ 60 ms que el de lentejas y
Cules son los nmeros? el total de la compra es de $ 1.930,
calcule el precio de cada artculo.
8. La suma de tres nmeros impares
consecutivos es 369.

548 Problemas

548-549 548 10/11/01, 1:35 PM


CAPITULO 14

15. Pedro y Luis tienen en total $ 3.800. Si cuntos hay de cada tipo?
Pedro gasta $ 600 y Luis gana $ 400, 27. En un corral hay conejos y gallinas,
Luis tendr el doble de lo que tendr si en total hay 40 patas y 14 cabezas.
Pedro. Cunto tiene cada uno? Cuntos conejos hay? y Cuntas
16. Hay 91 rosales rojos, blancos y gallinas?
amarillos. Si hay el doble de rosales 28. Un padre reparte una herencia a sus
rojos que amarillos y el doble de 3 hijos otorgndoles $ 2.000.000
rosales blancos que rojos, cuntos por cada ao que ellos tienen. Si el
rosales hay de cada color? hermano mayor tiene 3 aos menos
17. Cuntas parras se necesitan para que la suma de las edades de sus dos
producir 12.800 kg de uva si cada hermanos, el hermano del medio tiene
parra produce aproximadamente 160 el doble de la edad de su hermano
kg? menor y ste, el menor, tiene 4
18. Dividir el nmero 1.200 en dos partes aos, determine cul fue la cantidad
tales que el triple de la parte menor repartida.
exceda en 96 a la mayor. 29. Las edades de 2 hermanas suman
19. Dividir el nmero 164 en dos partes 26 aos y el doble de la edad de
tales que el doble del mayor aumentado la menor disminuida en 1 es igual
en 40 sea igual al triple del menor a la edad de la mayor. Determine
disminuido en 7. ambas edades.

20. Dividir el nmero 750 en 3 partes tales 30. La edad actual de una madre es el
que la segunda sea igual al doble de la cudruple de la edad de su hija ms
primera y la tercera sea igual a la suma 3 aos y hace 3 aos la edad de la
de la primera y la segunda. madre era 8 veces la edad de la hija.
Determine ambas edades.
21. Dividir el nmero 196 en tres partes
tales que la segunda sea igual al doble 31. Las edades de dos hermanos suman 35
de la primera menos 3 unidades, la aos. Dentro de 15 aos la edad del
tercera sea igual al doble de la segunda menor ser la edad actual del mayor.
ms 2 unidades. Determine ambas edades.

22. El permetro de un cuadrado es 52 cm. 32. Tengo en mi bolsillo 37 monedas de


Cunto mide el lado? $ 50 y $ 100 y en total tengo $ 2.600.
Cuntas monedas de $ 50 y cuntas
23. El permetro de un rectngulo es 64 cm de $ 100 tengo?
y su largo tiene 2 cm ms que su ancho.
Cules son sus dimensiones? 33. Encontrar 3 nmeros enteros consecuti-
vos tales que el doble del menor ms el
24. Calcule las dimensiones de un rectn- triple del mediano menos el cudruple
gulo sabiendo que el largo es el del mayor sea igual a 19.
triple del ancho y que su permetro
es 72 cm. 34. La suma de dos nmeros es 77 y la
diferencia entre el triple del menor y
25. Se desea cercar un terreno rectangular. el doble del mayor es 16. Determine
Si se necesitan 30 m de alambre y se dichos nmeros.
sabe que el largo tiene 3 m ms que
el ancho, determine las dimensiones 35. Dentro de 15 aos la edad de un padre
del terreno. ser el doble de la edad de su hijo.
Si las edades difieren en 30 aos,
26. En un corral hay conejos y gallinas. determine las edades actuales.
Si en total hay 44 patas y 14 cabezas,

Problemas 549

548-549 549 10/11/01, 1:35 PM


Soluciones
(Ecuaciones enteras)

1. x = 6 2. x = 8 3. x = 5 4. x = 12 5. x = 4 6. 211 , 212 , 213


7. 44, 45, 46 8. 121, 123, 125 9. 91 10. 1.152
11. 270 12. $ 310 , $ 340 13. $ 120 pincel; $ 180 cuaderno.
14. $ 350 lentejas; $ 410 garbanzos
15. $ 1.800 Pedro; $ 2.000 Luis 16. 13 26 52. 17. 80.
18. 324 876. 19. 89 75 20. 125 250 375
21. 29 55 112 22. 13. 23. largo = 17 cm; ancho = 15 cm
24. l = 27 cm; a = 9 cm 25. a = 6 m; l = 9 m
26. 8 conejos; 6 gallinas 27. 6 conejos; 8 gallinas
28. $ 42.000.000 29. 9 17 30. h = 6 aos; m = 27 aos
31. 25 y 10 32. 22 de $ 50 y 15 de $ 100
33. 24 25 26 34. 34 43 35. P = 45 aos; h = 15 aos

Aplicacin de ecuaciones
14.2 lineales fraccionarias

El procedimiento y los pasos a seguir son anlogos al caso


anterior.
Los desarrollos para obtener las soluciones dependen de
las ecuaciones planteadas y, como se trata de expresiones
fraccionarias, debern hacerse todas las amplificaciones
necesarias para resolver el problema.

1. Un nmero ms su mitad, ms su tercera parte y ms su cuarta


Ejercicios parte es igual a 25. Cul es el nmero?
resueltos Denotemos por x el nmero pedido. Tenemos que el planteamiento
del problema nos queda:
x+ x
+ x
+ x = 25
2 3 4
Y resolvemos la ecuacin amplificando por el m.c.m. entre los
denominadores, que es 12.
x+ x
+ x
+ x = 25 / 12
2 3 4
12x + 6x + 4x + 3x = 300
1
25x = 300 /
25
x = 12
El nmero pedido es 12.
2. De una pieza de tela de 60 m se venden los 2 . Cunto queda?
2 5
Quedan 60 60 = 60 24
5
= 36 m

550 Problemas

550-551 550 10/11/01, 1:37 PM


CAPITULO 14

2
3. La edad de un hijo equivale a los de la edad de su padre y
7
1
hace 6 aos la edad del hijo era de la del padre. Determine
6
las edades actuales.
Sea x la edad actual del padre. Entonces 2 x es la edad
7
actual del hijo.
Hace 6 aos las edades eran x 6 y 2 x 6, respectivamente,
7
y la relacin estaba dada por:
2 1
x6 = (x 6) / 42
7 6
12x 252 = 7x 42
5x = 210
x = 42
La edad del padre es 42 aos y la de su hijo es 12 aos.

Ejercicios
1. Dividir el nmero 150 en dos partes, del nmero disminuido en 15. Cul
de modo que una de ellas sea la es el nmero?
mitad de la otra.
8. Se desea repartir $ 1.020 en tres partes,
2. Dividir el nmero 150 en tres partes, de modo que la primera sea igual a
de modo que la primera sea la mitad tres cuartos de la segunda ms $ 180
de la segunda y la segunda sea igual a y la tercera sea igual a cinco sextos
los dos tercios de la tercera. de la primera ms $ 120. Cunto
corresponde a cada parte?
3. Dividir el nmero 150 en dos partes,
de modo que la primera sea igual a los 9. En un curso la mitad de los alumnos
tres medios de la segunda. habla ingls y espaol, la sexta parte
habla francs y espaol, la octava
4. Dividir el nmero 150 en 4 partes,
parte habla alemn y espaol y los 25
de modo que la primera sea igual a
alumnos restantes hablan slo espaol.
un tercio de la segunda, la segunda
Cuntos alumnos tiene el curso?
sea igual a un medio de la tercera
y la tercera, igual a la suma de la 10. Hace 15 aos la edad de Juan era
cuarta y la primera. igual a los tres cuartos de la edad
de Pedro; sabiendo que Pedro tiene
5. Si pago los dos quintos de una deuda
10 aos ms que Juan, determine las
quedo debiendo $ 45.000 ms los
edades actuales.
tres dcimos de la deuda. A cunto
asciende sta? 11. Un padre reparte una cantidad de
dinero entre sus hijos. El mayor recibi
6. La cuarta parte de un nmero dismi-
la mitad del total, el segundo recibi
nuido en 2 es igual a la sexta parte
la cuarta parte del total; el tercero
del mismo nmero aumentada en 1.
recibi la sexta parte del total y el
Cul es el nmero?
cuarto recibi el dinero restante que
7. El doble de un nmero disminuido en eran $ 5.400. Cunto fue el dinero
la mitad del mismo es igual al triple repartido?

Problemas 551

550-551 551 10/11/01, 1:38 PM


12. Hace 10 aos la edad de Mara era 2
23. Los de una cantidad de fruta son
igual a la dcima parte de la edad de 5 3
Ana y actualmente es la cuarta parte. manzanas, los son naranjas y hay 12
7
Qu edades tienen Ana y Mara? duraznos. Cul es el total de fruta?
13. Dos nmeros enteros consecutivos 3
son tales que la diferencia entre los tres 24. Los de los animales de una granja
8
cuartos del menor y los dos quintos son corderos y el resto son vacas.
del mayor equivalen al nmero mayor En total hay 64 animales. Cuntos
disminuido en 17. corderos y cuntas vacas hay?
14. Tengo cierta cantidad de dinero. Gasto 25. Una mezcla de concreto se prepara con
1 arena y cemento en la proporcin 3:4.
del total en un primer artculo;
5 Si se necesitan 105 kilos de concreto,
luego gasto la mitad de lo que cuntos kilos de arena y de cemento
me queda en un segundo artculo se requieren?
y an me quedan $ 4.800. Cunto 26. Si con 134 litros de agua se llena
dinero tena? 2
un estanque slo en sus 3
partes,
4 3
15. La diferencia entre los y los determine la capacidad del estanque.
5 4
de un nmero es 6. Determine el 27. La quinta parte de los asistentes a una
nmero. conferencia son alemanes, la mitad son
16. Un vehculo recorri 210 kilmetros ingleses y hay 21 brasileos. Cuntos
en 2 horas y 20 minutos. Cuntos son los asistentes a la conferencia?
kilmetros recorri en 1 minuto? 2 de los 3 de un nmero equivalen
28. Los
(Suponga una velocidad constante). 5 2
7 a las 3 partes del mismo nmero dismi-
17. Un hijo tiene de la edad de su 4
16
nuido en 6. Determine el nmero.
padre. Si ambas edades suman 92
aos, determnelas. 29. Se compran 3 artculos A, B y C.
18. Un kilmetro corresponde aproxima- El artculo A cost $ 150.
damente a los ocho quintos de una El artculo A y el artculo B costaron las
milla. Cuntos km hay en 150 tres cuartas partes del valor de C y el
millas? artculo C ms el A costaron $ 50 ms
19. De un depsito de agua se ocupan las que el doble del valor de B. Cunto
dos quintas partes; luego se ocupan cost cada uno?
los dos tercios de lo que queda, 1
30. Tengo una cantidad de dinero. Gasto
luego se ocupa un medio del resto 8
y an quedan 144 litros. Cul es la de esa cantidad en un artculo y 5 de la
capacidad del depsito? 12
20. Un reloj se adelanta 90 segundos por misma en otro. Luego gasto 4
de lo
da. En cuntos das el reloj marcar 5

un aumento de 12 minutos? que me queda y an conservo $ 5.500.


Cul era mi capital inicial?
21. Los 3 de los 2 de una deuda
4 5
31. Dos vehculos viajan en la misma
corresponden a $ 1.710. A cunto carretera acercndose en sentidos
asciende la deuda? opuestos. El primero mantiene una
22. El doble de un nmero ms la mitad velocidad de 65 km/h y el segundo
del mismo nmero, ms un cuarto de 55 km/h. Si se encuentran entre s
del nmero ms uno equivale a 100. a 520 km de distancia, en cuntas
Cul es el nmero? horas estarn a 40 km entre ellos, antes
de cruzarse?

552 Problemas

552-553 552 10/11/01, 1:40 PM


CAPITULO 14
Soluciones
(Ecuaciones fraccionarias)

1. 50 y 100 2. 25 - 50 - 75 3. 90 - 60 4. 10 - 30 - 60 - 50
5. $ 150.000 6. 36 7. 10 8. 360 - 240 - 420 9. 120
10. Juan: 45 aos; Pedro: 55 aos 11. $ 64.800
12. Mara tiene 15 aos y Ana tiene 60 aos 13. 24 ; 25 14. $ 12.000 15. 120
16. Recorri 1,5 km 17. 28 y 64 aos, respectivamente 18. 93,75 km 19. 1.440 litros.
20. 8 das 21. $ 5.700 22. 36 23. 70 24. 24 corderos y 40 vacas
25. 45 kg de arena y 60 kg de cemento 26. 201 litros 27. 70 28. 40
29. $ 150, $ 450 y $ 800, respectivamente 30. $ 60.000 31. 4 horas

Aplicacin de sistemas
de ecuaciones lineales 14.3

Muchos problemas se pueden resolver en forma mucho ms


simple si se plantean mediante un sistema de ecuaciones.
Esto implica el uso de 2 o ms variables, cuyos valores deben
ser encontrados aplicando los mtodos conocidos para la
solucin de sistemas.

1. La suma de dos nmeros es 17 y su diferencia es 7. Determine


Ejercicios los nmeros.
resueltos Como se trata de dos nmeros, stos sern denotados por x
e y. As tenemos:
x + y = 17
xy = 5
Sumando ambas ecuaciones obtenemos:
2x = 22
x= 11
y sustituyendo el valor obtenido tenemos:
y=6
Entonces los nmeros pedidos son 11 y 6.
2. La cuarta parte de la diferencia de dos nmeros es igual a
9 y la diferencia entre el mayor y el triple del menor es 4.
Determine los nmeros.
Sean x e y los nmeros pedidos, con x > y.
Planteamos las ecuaciones de acuerdo con el enunciado:
xy
=9
4
x 3y= 4

Problemas 553

552-553 553 10/11/01, 1:40 PM


y resolvemos el sistema. Amplifiquemos la primera ecuacin
por 4. Nos queda:
x y = 36
x 3y = 4
Restando ambas obtenemos:
2y = 32
y = 16
Y reemplazando el valor obtenido en cualquier ecuacin
tenemos:
x = 52 y los nmeros pedidos son 52 y 16.
1
3. Hace 6 aos la edad de un hijo era de la edad de su padre y
5 2
dentro de 9 aos la edad del hijo ser los de la de su padre.
5
Determinar las edades actuales.
Sean x e y las edades actuales del padre y del hijo respectiva-
mente.
Hace 6 aos, las edades eran x 6 e y 6, respectivamente.
Dentro de 9 aos ambos tendrn x + 9 e y + 9, respectiva-
mente.
Planteamos entonces las ecuaciones de acuerdo con el enunciado.
1
y 6 = (x 6)
2
2
y + 9 = (x + 9)
5
Haciendo las amplificaciones adecuadas tenemos:
5y 30 = x 6
5y + 45 = 2x + 18
5y x = 24
5y 2x = 27

Resolviendo el sistema obtenemos las soluciones x = 51


y = 15
lo que corresponde a las edades actuales de un padre y de
su hijo.

4. Dividir 90 en tres partes tales que la parte menor sea igual


1 9
a de la parte intermedia y la intermedia sea igual a
9 20
de la parte mayor.
Sean x, y, z las tres partes ordenadas de menor a mayor.
Tenemos: x + y + z = 90
1
x= y
9
9
y= z
20
20
de la 3 ecuacin: z = y
9
de la 2 ecuacin: y = 9x

554 Problemas

554-555 554 10/11/01, 1:41 PM


CAPITULO 14

20
de ambas igualdades obtenemos: z = 9x
9
z = 20x
podemos expresar tanto y como z en trminos de x.
Reemplazando en la 1 ecuacin:
x + 9x + 20x = 90 30x = 90
x =3
y sustituyendo obtenemos: y = 27
z = 60
y as queda 90 dividido en esas tres partes.

Ejercicios
1. La suma de dos nmeros es 12 y la 10. Si al doble de un nmero le resto
diferencia entre ellos es 6. Determ- el triple de otro obtengo 5 y si al
nelos. primer nmero le sumo el doble del
2. La suma de dos nmeros es 6 y su segundo obtengo 1. Cules son
diferencia es 17. Determnelos. los nmeros?
3. La diferencia de dos nmeros es 24 11. Si Pedro le da a Juan $ 400, ambos
y la suma del mayor con el doble quedan con la misma cantidad y si
del menor es 6. Determine ambos Juan le da a Pedro $ 300, entonces
nmeros. Pedro tendr exactamente el doble
4. Tengo $ 4.050 en monedas de $ 50 y de lo que tiene Juan. Cunto tiene
de $ 100. Si en total tengo 46 monedas, cada uno?
cuntas tengo de cada valor? 12. Si sumamos el dinero mo y el de mi
5. Dividir 60 en dos partes tales que la hermano hacemos $ 1.700 y si mi
parte menor sea igual a la tercera parte hermano me regala $ 155, tendremos
de la parte mayor. lo mismo cada uno. Cunto tiene
cada uno?
6. Tres kilos de un artculo A ms un kilo
de un artculo B cuestan $ 955. Tres 13. Hace 10 aos la edad de un hijo era
kilos del artculo B ms un kilo del un sptimo de la edad de su madre y
artculo A cuestan $ 1.225. Cul es el dentro de 10 aos la edad de la madre
precio de cada artculo por kilo? ser el doble de la edad de su hijo.
Determine las edades actuales.
7. Determine dos nmeros de modo que
la diferencia entre el mayor y el doble 14. Si a los dos trminos de una fraccin
del menor sea 1 y la suma del doble se le agrega 1, la fraccin que resulta
del mayor con el menor sea 8. 1
es equivalente con y si a los
8. Tengo 18 aves entre patos y gallinas y 2
dos trminos de la misma fraccin
la diferencia entre el doble de patos
y el triple de gallinas es 1. Cuntas se les resta 1, resulta una fraccin
tengo de cada tipo? 1
equivalente a . Determine la
4
9. Si al triple de un nmero le agrego el fraccin original.
doble de otro obtengo como resultado
8 y si al doble del segundo nmero le 15. La razn entre dos nmeros es 2:3
5
agrego el primero obtengo 0. Cules y su suma es . Cules son los
4
son los nmeros? nmeros?

Problemas 555

554-555 555 10/11/01, 1:41 PM


16. Dos nmeros estn en la razn 3:1. 20. La suma de tres nmeros es 27. Si
La diferencia entre el doble del mayor el mayor de ellos se divide por 5, se
y el menor es 15. Cules son los obtiene el primero y si el primero se
nmeros? multiplica por 3, se obtiene el segundo.
Cules son los nmeros?
17. La edad de Ana es igual al triple de la
edad de Rosa. La diferencia entre la 21. La suma de dos ngulos de un tringulo
mitad de la edad de Ana y un tercio es igual a 86 y la diferencia entre
14 ellos es 30. Determine los tres ngulos
de la edad de Rosa es . Qu edad
3 del tringulo.
tiene cada una?
22. A, B y C renen en total $ 11.000.
18. La suma de tres nmeros enteros
La diferencia entre el doble de lo que
positivos es 20. Si el menor se
tiene C y lo que tiene A es $ 400 y la
multiplica por 5, se obtiene el doble
diferencia entre el doble de lo que tiene
del mayor y si el mediano se multiplica
B y el triple de lo que tiene C es $ 570.
por 5, se obtiene el triple del mayor.
Cunto tiene cada uno?
Determine los nmeros.
19. La suma de tres nmeros enteros 23. Compr tres artculos por $ 1.855. El
(distintos) es 3. La suma del menor primero y el tercero juntos costaron
con el mayor es 2 y la diferencia $ 205 ms que el segundo, y el tercero
entre ellos es 14. Cules son los cost $ 250 menos que el primero.
nmeros? Determine el precio de cada uno.

Soluciones
1. x = 9 7. x = 3 13. hijo: 14 aos 19. x = 6
y=3 y=2 madre: 38 aos y=1
z=8
11 2
2. x = 8. x = 11 14.
2 5 20. a = 3
23 b=9
y = y =7
2 c = 15
1 21. a = 58
3. x = 14 19. x = 4 15. x =
2
3 b = 28
y = 10 y = 2 y= d = 94
4
4. 11 monedas de $ 50 10. x = 1 16. x = 9 22. a = $ 4.540
35 monedas de $ 100 y = 1 y=3 b = $ 3.990
c = $ 2.470
5. 15 y 45 11. x = 2.500 17. Ana: 12 aos
y = 1.700 Rosa: 4 aos 23. a = $ 640
b = $ 825
6. a = $ 205 12. x = 1.005 18. x = 4 c = $ 390
b = $ 340 y = 695 y=6
z = 10

556 Problemas

556-559 556 10/11/2001, 19:15


CAPITULO 14

Problemas 14.4
miscelneos

1. Un trabajador tiene un contrato por 48 horas semanales


o 192 horas mensuales. Cada mes recibe un sueldo bruto
consistente en:
Sueldo base $330.240
Asignacin zona $ 66.048
Antigedad $ 39.625
Determine su valor hora base, su porcentaje de asignacin de
zona y su porcentaje correspondiente a antigedad.

2. Un alumno ha sacado las siguientes notas en lgebra: 5,5; 6,2;


5,8; 6,6 en una escala de 1 a 7. Debe dar una prueba ms.
Averige qu nota debe obtener en esa prueba para lograr
un promedio final 6,5.

3. Se deben preparar 100 ml de un jarabe para la tos, con una


droga que viene con una concentracin de 5 mg/ml y un elixir
saborizante de miel para distraer el sabor de la droga. Cunto
se debe usar de cada ingrediente si la solucin debe quedar
con una concentracin de 2mg/ml?

4. Dados los siguientes enunciados, exprselos como una


frmula donde intervengan las variables y una constante
k. En cada caso estime el valor de la constante segn las
condiciones dadas.
a) x es directamente proporcional a y. Si x es 22, entonces
y es 66.
b) r vara en forma directa respecto de t. Si r = 24; entonces
t = 6.
c) p es inversamente proporcional a la suma de r y t. Si
r = 0,4 y t = 0,8, entonces p = 1,4
d) La energa cintica E c vara en forma directamente
proporcional a la masa y al cuadrado de la velocidad con
que se mueve un cuerpo. Si m = 6, v = 10, entonces
Ec = 30.
e) La velocidad de desplazamiento v de un cuerpo vara en
forma directamente proporcional a la distancia recorrida
d, e inversamente proporcional al tiempo t que se
demora en recorrerla. Si d = 10 m y t = 2 seg, entonces
v = 5 m/seg.

5. Una empresa de distribucin de correspondencia debe repartir


x cartas en 3 das. El primer da reparte a sobres, el segundo da

Problemas 557

556-559 557 10/11/2001, 19:15


reparte 6 sobre menos que el primer da. Determine cuntos
sobres le quedan por repartir el tercer da.

6. Sea A = x + y y b = x y. Encuentre el valor de x2.


1 2 1 1
7. Se sabe que + =m y + = 2.
x x m m
Determine el valor de x.
xx
8. Si x + 2x + 3x = 48, encuentre el valor de
x+x
9. Si t + 2t + 3t + 4t = 1, encuentre el valor de
1 2 3 4 5
+ + + +
t t t t t
10. Con a litros de bencina un automvil alcanza a recorrer las
tres quintas partes del camino entre dos ciudades. Determine
cuntos litros se necesitan para recorrer el trayecto de
ida y vuelta.

11. De un total de n alumnos que rindi la prueba de ingreso


a la educacin superior reprobaron b alumnos. Calcule el
porcentaje de alumnos que aprob.

12. Si r es el 25% de A y t es el 75% de A, escriba r en


funcin de t.

13. Para preparar 120 kg. de concreto (cemento con arena), en


los primeros 40 kg se usa la proporcin 3:2 y en el resto, se
usa la proporcin 2:3 de cemento y arena, respectivamente.
Determine cunta arena se us.

14. Tres amigos reparten equitativamente entre ellos $960.000 en


billetes de 1.000, 5.000 y 10.000. Si cada uno recibe la misma
cantidad de billetes de cada denominacin, cunto dinero
recibe cada amigo en billetes de 1.000?

15. En un mapa se lee: escala 1 : 120.000. Se mide con una regla


la distancia entre dos pueblos y se encuentra que es 5,4 cm.
Calcule la distancia real.

16. Se dispone de un presupuesto mensual de $10.000 para


comprar dulces o chocolates. Si cada dulce cuesta 10 pesos y
cada chocolate cuesta 50 pesos.
a) Escriba una frmula que relacione la cantidad de dulces con
la cantidad de chocolates que se pueden comprar.
b) Grafique la funcin que result.
c) Compare las intersecciones con los ejes del grfico con
la cantidad de dulces que podra comprar si no comprara
chocolates y con la cantidad de chocolates que se podra
comprar si no comprara dulces.
558 Problemas

556-559 558 10/11/2001, 19:15


CAPITULO 14

17. Una empresa invierte $3.000.000 en una maquinaria que


tiene una vida til de 5 aos; despus quedar totalmente
desvalorizada. Escriba una frmula que relacione el valor de
la maquinaria con el tiempo transcurrido.

18. La diagonal de un cuadrado mide 10 cm ms que su lado.


Calcule la medida del lado.

19. Un paraleleppedo (slido rectangular) mide 3 cm de largo,


2 de ancho y 1 de alto. Si duplico sus magnitudes lineales,
qu pasa con su volumen?

20. Una caera de agua de 1 cm de radio transporta lquido a razn


de 1,2 m/s. Cunta agua sale en 1 hora?

Soluciones

1. 1.720, 20%, 12% 1


12. x = t
3
2. No puede subir a 6.5. Tendra que
obtener un 8.2 en la prueba que le 13. 64 kg
queda.
14. 20.000 pesos.
3. 40 ml de concentrado de droga.
15. 6,48 km.
60 ml de saborizante.
16. a) x + 5y = 1.000
4. a) x = ky ; k = 3
b)
b) r = kt ; k = 4
Chocolates
1
c) p = k ; k = 1.68
r+t
1
d) Ec = k mv2; k =
2
d 200
e) v = k ;k=1
t
Dulces
5. x 2a + b 1.000

6. A B y2
17. y = 3.000.000 600.000x
7. 3 ( x = tiempo y = valor)
8. 4 18. 24,142 cm
9. 150
19. a) Se aumenta ocho veces.
10
10. a lt 20. 604.800 c.c.
3
nb
11. 100
n

Problemas 559

556-559 559 10/11/2001, 19:15


Indice
Captulo 1
lgebra en los Nmeros Reales
1.1 LENGUAJE ALGEBRAICO .................................................................................. 7
1.2 VALORIZACIN DE EXPRESIONES ALGEBRAICAS ........................................... 12
1.3 REDUCCIN DE TRMINOS SEMEJANTES Y USO DE PARNTESIS .................. 14
1.4 MULTIPLICACIN ALGEBRAICA ....................................................................... 19
1.5 PRODUCTOS NOTABLES .................................................................................. 24
1.6 FACTORIZACIN .............................................................................................. 29
1.6.1 Factor comn (Monomio y Polinomio) ............................................................... 29
1.6.2 Factor comn compuesto ................................................................................... 32
1.6.3 Diferencia de cuadrados .................................................................................... 34
1.6.4 Trinomios ordenados .......................................................................................... 37
1.6.5 Sumas o diferencias de cubos ............................................................................ 41
1.7 FRACCIONES ALGEBRAICAS ............................................................................. 43
1.7.1 Simplificacin .................................................................................................... 43
1.7.2 Multiplicacin y Divisin de fracciones algebraicas ........................................... 45
1.7.3 Adicin y Sustraccin de fracciones algebraicas ................................................. 50
PRUEBA DE SELECCIN MLTIPLE ................................................................... 56

Captulo 2
Ecuaciones e inecuaciones de primer grado
2.1 ECUACIONES .................................................................................................... 60
2.1.1 Ecuaciones de primer grado con coeficientes enteros ......................................... 61
2.1.2 Ecuaciones de primer grado con coeficientes fraccionarios ................................ 65
2.1.3 Ecuaciones fraccionarias de primer grado .......................................................... 69
2.1.4 Ecuaciones literales de primer grado .................................................................. 73
2.1.5 Ecuaciones con valor absoluto ........................................................................... 79
2.2 PROBLEMAS ...................................................................................................... 80
2.3 DESIGUALDADES E INECUACIONES ................................................................ 89
2.3.1 Desigualdades .................................................................................................... 91
2.3.2 Inecuaciones ...................................................................................................... 94
2.3.3 Inecuaciones simultneas ................................................................................... 97
2.3.4 Inecuaciones con valor absoluto ........................................................................ 100
PRUEBA DE SELECCIN MLTIPLE ................................................................... 104

Captulo 3
Relaciones y funciones
3.1 LGICA ............................................................................................................. 111
3.2 CONJUNTOS ..................................................................................................... 123
3.2.1 Conceptos bsicos ............................................................................................. 123
3.2.2 Operaciones entre conjuntos .............................................................................. 129
3.3 RELACIONES ..................................................................................................... 136
3.3.1 Conceptos bsicos ............................................................................................. 136
3.3.2 Relacin de equivalencia y de orden .................................................................. 144
3.4 FUNCIONES ...................................................................................................... 151
3.4.1 Conceptos bsicos ............................................................................................. 151
3.4.2 La funcin de primer grado (Ecuacin de la recta) .............................................. 162
3.4.3 Tipos de funciones. Funcin inversa ................................................................... 175
3.4.4 Funciones de primer grado simultneas. Sistemas de ecuaciones de primer grado .............. 186

560 ndice

indice final 560-568 560 25/11/02, 1:33 PM


3.4.5 Inecuaciones con dos variables. Sistemas y problemas de programacin lineal ......... 211
PRUEBA DE SELECCIN MLTIPLE ................................................................... 221

Captulo 4
Ecuaciones e inecuaciones de segundo grado
4.1 ECUACIN CUADRTICA ................................................................................ 227
4.1.1 Solucin de la ecuacin por factorizacin ......................................................... 227
4.1.2 Solucin de la ecuacin cuadrtica aplicando la frmula general ...................... 230
4.1.3 Ecuaciones bicuadrticas ................................................................................... 233
4.1.4 Relacin entre los coeficientes de una ecuacin cuadrtica
y sus races o soluciones y naturaleza de ellas .................................................... 235
4.2 LA FUNCIN CUADRTICA ............................................................................. 240
4.3 INECUACIONES DE SEGUNDO GRADO .......................................................... 246
4.4 SISTEMAS DE ECUACIONES DE SEGUNDO GRADO ....................................... 251
4.4.1 Sistemas que contienen una ecuacin lineal y una ecuacin cuadrtica ............ 251
4.4.2 Sistemas en que ambas ecuaciones son de la forma ax2 by2 = c ...................... 253
4.4.3 Sistemas formados por una ecuacin de la forma x2 y2 = a
y la otra ecuacin, de la forma xy = b .............................................................. 256
4.4.4 Sistemas homogneos formados por ecuaciones cuyos trminos
son todos de segundo grado ............................................................................... 259
4.4.5 Otros sistemas y problemas ................................................................................ 262
PRUEBA DE SELECCIN MLTIPLE ................................................................... 267

Captulo 5
Polinomios y teora de ecuaciones
5.1 DEFINICIN Y OPERACIONES CON POLINOMIOS ......................................... 272
5.1.1 Suma .................................................................................................................. 273
5.1.2 Resta .................................................................................................................. 273
5.1.3 Producto ............................................................................................................ 273
5.1.4 Divisin ............................................................................................................. 274
5.2 TEORA DE ECUACIONES ................................................................................. 283
5.2.1 Clculo de las races de un polinomio. Factorizacin ........................................ 283
5.2.2 Relacin entre los coeficientes de una ecuacin P (x) = 0 y sus races ................ 284
PRUEBA DE SELECCIN MLTIPLE ................................................................... 291

Captulo 6
Potencias y Races
6.1 POTENCIAS ....................................................................................................... 295
6.1.1 Potencias de exponente natural .......................................................................... 295
6.1.2 Potencias de exponente cero y exponente entero negativo ................................. 295
6.2 PROPIEDADES DE LAS POTENCIAS .................................................................. 299
6.2.1 Multiplicacin de potencias de igual base .......................................................... 299
6.2.2 Divisin de potencias de igual base ................................................................... 299
6.2.3 Elevacin de potencia a potencia ....................................................................... 299
6.2.4 Multiplicacin de potencias de igual exponente ................................................ 299
6.2.5 Divisin de potencias de igual exponente .......................................................... 300
6.2.6 Potencia de un producto ................................................................................... 300
6.2.7 Potencia de un cociente ..................................................................................... 300
6.3 ECUACIONES EXPONENCIALES ........................................................................ 304
ndice 561

indice final 560-568 561 25/11/02, 1:33 PM


Indice
6.4 RACES ............................................................................................................... 307
6.5 PROPIEDADES ................................................................................................... 307
6.5.1 Potencia de exponente fraccionario ................................................................... 307
6.5.2 Multiplicacin de races de igual ndice ............................................................. 308
6.5.3 Divisin de races de igual ndice ....................................................................... 308
6.5.4 Raz de una raz ................................................................................................. 308
6.6 RACIONALIZACIN .......................................................................................... 318
6.6.1 Tcnicas de racionalizacin ............................................................................... 318
6.7 ECUACIONES IRRACIONALES ........................................................................... 320
PRUEBA DE SELECCIN MLTIPLE ................................................................... 323

Captulo 7
Logaritmos
7.1 DEFINICIN DE LOGARITMO .......................................................................... 329
7.2 PROPIEDADES ................................................................................................... 330
7.3 ECUACIONES EXPONENCIALES Y LOGARTMICAS .......................................... 340
PRUEBA DE SELECCIN MLTIPLE ................................................................... 350

Captulo 8
Trigonometra
8.1 SISTEMAS DE MEDICIN DE NGULOS ........................................................... 353
8.2 RAZONES TRIGONOMTRICAS PARA NGULOS AGUDOS ............................ 354
8.3 IDENTIDADES TRIGONOMTRICAS .................................................................. 354
8.4 FUNCIONES TRIGONOMTRICAS DE UN NGULO CUALQUIERA ................ 355
8.5 FUNCIONES TRIGONOMTRICAS DE 60, 30 y 45, 0, 90, 180 y 270 ...... 355
8.6 FUNCIONES PERIDICAS.................................................................................. 356
8.7 FUNCIONES PARES E IMPARES ......................................................................... 356
8.8 ECUACIONES TRIGONOMTRICAS .................................................................. 356
8.9 RESOLUCIN DE TRINGULOS NO RECTNGULOS ..................................... 357
8.9.1 Teorema del seno (o de los senos) ...................................................................... 357
8.9.2 Teorema del coseno (o de los cosenos) ............................................................... 357
8.9.3 ngulos de elevacin y depresin ...................................................................... 357
PRUEBA DE SELECCIN MLTIPLE ................................................................... 376

Captulo 9
Nmeros Complejos
9.1 DEFINICIONES Y PROPIEDADES ....................................................................... 379
9.1.1 Igualdad ............................................................................................................. 379
9.1.2 Representacin geomtrica ................................................................................ 379
9.1.3 Forma cannica de un complejo ........................................................................ 380
9.1.4 Operaciones con nmeros complejos ................................................................ 380
9.1.5 Estructura del conjunto (k , + , ) ..................................................................... 380
9.1.6 Potencias de i ..................................................................................................... 381
9.2 CONJUGADO Y MDULO DE UN COMPLEJO ............................................... 390
9.2.1 Conjugado de un complejo ................................................................................ 390
9.2.2 Mdulo de un complejo ..................................................................................... 391
9.3 REPRESENTACIN TRIGONOMTRICA O FORMA POLAR DE UN
NMERO COMPLEJO ....................................................................................... 397
9.3.1 Definicin de razones trigonomtricas ............................................................... 397

562 ndice

indice final 560-568 562 25/11/02, 1:33 PM


9.3.2 Representacin trigonomtrica del complejo z = a + bi ...................................... 397
9.3.3 Producto y cociente de complejos en forma polar .............................................. 398
9.3.4 Potenciacin de nmeros complejos en forma polar .......................................... 398
9.3.5 Radicacin de nmeros complejos en forma polar ............................................. 399
PRUEBA DE SELECCIN MLTIPLE ................................................................... 409

Captulo 10
Vectores
10.1 DEFINICIONES .................................................................................................. 413
10.2 OPERACIONES CON VECTORES ....................................................................... 414
10.2.1 Suma de vectores ............................................................................................... 414
10.2.2 Producto por escalar .......................................................................................... 415
10.2.3 Propiedades de la suma y el producto por escalar ............................................... 415
10.2.4 Resta de vectores ................................................................................................ 416
10.3 VECTOR UNITARIO ........................................................................................... 416
10.3.1 Definicin .......................................................................................................... 416
10.3.2 Normalizar un vector ......................................................................................... 417
10.4 DESCOMPOSICIN DE UN VECTOR ................................................................ 418
10.5 PRODUCTO PUNTO (O PRODUCTO ESCALAR) .............................................. 426
10.5.1 Definicin .......................................................................................................... 426
10.5.2 Propiedades ....................................................................................................... 426
10.5.3 ngulo entre vectores ......................................................................................... 426
10.5.4 Proyeccin de un vector sobre otro .................................................................... 427
10.6 VECTORES EN EL ESPACIO R3 .......................................................................... 433
10.6.1 Definiciones ....................................................................................................... 433
10.6.2 Producto vectorial o producto cruz ..................................................................... 434
PRUEBA DE SELECCIN MLTIPLE ................................................................... 439

Captulo 11
Matrices y determinantes
11.1 CONCEPTOS BSICOS ...................................................................................... 443
11.2 IGUALDAD Y ADICIN DE MATRICES ............................................................. 445
11.2.1 Matrices iguales ................................................................................................. 445
11.2.2 Adicin de matrices ........................................................................................... 445
11.2.3 Propiedades de la adicin .................................................................................. 446
11.3 PONDERACIN DE UNA MATRIZ POR UN ESCALAR ..................................... 450
11.3.1 Definicin .......................................................................................................... 450
11.3.2 Propiedades ....................................................................................................... 450
11.4 MULTIPLICACIN DE MATRICES ...................................................................... 454
11.4.1 Procedimiento .................................................................................................... 454
11.4.2 Propiedades de la multiplicacin ....................................................................... 455
11.4.3 Matrices inversas y ecuaciones multiplicativas ................................................... 456
11.5 DETERMINANTES Y SISTEMAS DE ECUACIONES .............................................. 462
11.5.1 Determinantes y Sistemas lineales de orden 2 .................................................... 462
11.5.2 Determinantes y Sistemas lineales de orden 3 .................................................... 463
PRUEBA DE SELECCIN MLTIPLE ................................................................... 468

ndice 563

indice final 560-568 563 25/11/02, 1:33 PM


Indice
Captulo 12
Sumatoria y progresiones
12.1 SUMATORIA ...................................................................................................... 473
12.2 SUCESIONES ..................................................................................................... 482
12.2.1 Definicin .......................................................................................................... 482
12.2.2 Sucesiones convergentes .................................................................................... 484
12.2.3 Sucesiones divergentes ....................................................................................... 485
12.2.4 Sucesiones crecientes y decrecientes ................................................................. 486
12.3 PROGRESIN ARITMTICA ............................................................................... 488
12.4 PROGRESIN GEOMTRICA ............................................................................ 494
12.4.1 Definicin .......................................................................................................... 494
12.4.2 Clculo de intereses de capital ........................................................................... 495
12.5 PROGRESIN ARMNICA ................................................................................ 506
12.6 INDUCCIN MATEMTICA .............................................................................. 509
PRUEBA DE SELECCIN MLTIPLE ................................................................... 515

Captulo 13
Anlisis combinatorio, Teorema del binomio y Elementos de probabilidades

13.1 ANLISIS COMBINATORIO ............................................................................... 519


13.1.1 Conceptos bsicos ............................................................................................. 519
13.1.2 Permutaciones .................................................................................................... 519
13.1.3 Arreglos o variaciones ........................................................................................ 520
13.1.4 Combinaciones .................................................................................................. 520
13.2 TEOREMA DEL BINOMIO ................................................................................. 528
13.2.1 Conceptos y observaciones bsicas .................................................................... 528
13.2.2 Teorema del binomio ......................................................................................... 529
13.2.3 El tringulo de Pascal ......................................................................................... 530
13.3 ELEMENTOS DE PROBABILIDADES ................................................................... 534
13.3.1 Conceptos bsicos ............................................................................................. 534
13.3.2 Probabilidad de la unin y de la interseccin de dos eventos ............................. 535
PRUEBA DE SELECCIN MLTIPLE ................................................................... 539

Captulo 14
Problemas
14.1 APLICACIN DE ECUACIONES LINEALES ENTERAS ......................................... 545
14.2 APLICACIN DE ECUACIONES LINEALES FRACCIONARIAS ............................ 550
14.3 APLICACIN DE SISTEMAS DE ECUACIONES LINEALES .................................. 553
14.4 PROBLEMAS MISCELNEOS ............................................................................. 557

564 ndice

indice final 560-568 564 25/11/02, 1:33 PM


Indice Analtico
lgebra, concepto de, 3 Logartmica, 340
Anlisis combinatorio, 519 Lineal, 545
Principio de la Lineal entera, 545
multiplicacin en el, 519 Linealmente dependiente, 186
Principio de la suma en el, 519 Linealmente independiente, 186
ngulo, 357 Literales, 73
Arreglos, 520 Sistema de, 186, 251, 553
de depresin, 357 Eliminacin por igualacin, 191
de elevacin, 357 Eliminacin por reduccin, 189
Axioma, 123 Eliminacin por sustitucin, 190
Axioma de Peano, 509 Espacio muestral, 534
Coeficiente binomial, 528 Experimento aleatorio, 534
Combinaciones, 520 Experimento determinstico, 534
Conjunto, 123 Expresin algebraica, 11
Cardinalidad de un, 124 Divisin de, 45
Clase de equivalencia entre, 144 Factorizacin de, 29
Conjunto universo,124 Multiplicacin de, 19, 45
Conjunto vaco, 123 Trmino de una,11
Diferencia simtrica,130 Factor comn compuesto, 32
Equivalencias de, 124 Factorial de un nmero, 519
Esquema de los Factorizacin, 29
conjuntos numricos, 5 Fracciones algebraicas, 43
Idempotencia,117 Adicin y sustraccin de, 50
Igualdad de, 124 Multiplicacin y divisin de, 45
Leyes de la asociatividad entre, 117 Funcin, 151
Leyes de la conmutatividad entre, 117 Composicin de, 152
Leyes de la distributividad entre, 118 Dominio de una, 152
Leyes de Identidad, 117 Funcin biyectiva, 175
Leyes de Morgan, 117 Funcin constante, 152
Operaciones entre, 129 Funcin cuadrtica, 240
Potencia de un, 124 Funcin epiyectiva, 175
Propiedades de la relacin Funcin idntica, 152
de inclusin entre, 130 Funcin inversa, 175
Subconjunto, 124 Funcin inyectiva, 175
Conjuntos numricos, 4 Funcin preposicional o
Esquema de los , 4 preposicin abierta, 111
Contradiccin, 113 Funciones de primer grado, 186
Desigualdad, 89 Parte entera de una, 152
Determinantes, 462 Rango o recorrido de una, 152
Ecuacin, 60 ,187, 227 Valor absoluto de una, 152
Bicuadrtica, 233 Induccin matemtica, 509
Con valor absoluto, 79 Inecuacin, 89
Cuadrtica, 230 con dos variables, 211
De primer grado, 61 con valor absoluto, 100
Exponencial, 304 de primer grado, 60
Fraccionaria, 550 de segundo grado, 246
Irracional, 320

ndice Analtico 565

indice final 560-568 565 25/11/02, 1:33 PM


Indice Analtico
simultneas, 97 de la unin y de
Inters compuesto, 495 la interseccin, 535
Intersecciones con los ejes, 240-241 de un evento cierto, 535
Lenguaje algebraico, 7 de un evento imposible, 535
Logaritmo, 329 Producto cartesiano, 136
Matrices, 443 Productos notables, 24
Adicin de, 445 Programacin lineal, 211
Determinante de orden 2, 462 Progresin aritmtica, 488
Determinante de orden 3, 463 Progresin armnica, 506
Igualdad de, 445 Progresin geomtrica, 494
Multiplicacin de, 454 Propiedad telescpica, 474
Orden o dimensin de una, 443 Proposicin, 111
Producto matriz-escalar, 450 Dominio o universo de una, 111
Regla de Cramer, 464 Negacin de una, 111
Medios aritmticos, 488 Races, 307
Interpolacin de, 488 Racionalizacin, 318
Medios geomtricos, 495 Radin, 353
Interpolacin de, 495 Recta, 162-163
Multiplicacin algebraica, 19 Coeficiente de
Nmeros complejos, 379-381 posicin de una, 162
Conjugado y sus propiedades, 390 Ecuacin de la recta
Mdulo y sus propiedades, 391 dados dos puntos, 162
Propiedades de la suma de, 380 Interseccin de la recta
Propiedades de producto de, 381 con los ejes, 165
Optimizacin, 211 Familia de, 166
Parbola, 240 Pendiente de la, 162
Concavidad de la, 240 Rectas paralelas, 163
Discriminante de la, 241 Rectas perpendiculares, 163
Vrtice de la, 241 Regla de Cramer, 464
Permutaciones, 520 Relacin, 137,144,149
Polinomio, 272-273 Dominio de una, 137
Definicin de, 272 Grfico cartesiano de una, 137
Grado de un, 272 Grfico Sagital de una, 137
Operaciones con, 273 Propiedades de una, 144
Races complejas e Rango o recorrido de una, 137
irracionales de un, 284 Relacin de equivalencia, 144
Races racionales de un, 284 Relacin de orden, 145
Raz de un, 283 Relacin inversa, 137
Potencia, 295 Relaciones y funciones, 111
Potencia de exponente Sistema de ecuaciones
fraccionario, 307 de primer grado,186
Potencia de un nmero, 295 Sistema de ecuaciones
Propiedades de las, 299 de segundo grado, 251
Principio multiplicativo, 519 Sistema inconsistente, 186
Principio de la suma, 519 Sistema indeterminado, 186
Probabilidad, 534 Sistemas lineales en orden 2, 462
de eventos complementarios, 535 Sistemas lineales en orden 3, 463

566 ndice Analtico

indice final 560-568 566 25/11/02, 1:34 PM


Sucesiones, 482 Valoracin de expresiones
Sumatoria, 473 algebraicas, 12
Tautologa, 113 Valor absoluto, 79
Teorema, 123 Vector unitario, 416
Teorema del binomio, 529 Vectores, 413,427
Teorema del Coseno, 357 Descomposicin de, 418
Teorema del Seno, 357 Magnitud, direccin
Teora de ecuaciones, 283 y sentido de, 413
Trmino semejante, 14 Mdulo o norma de, 417
Tringulo de Pascal, 530 Operaciones con, 414
Trigonometra, 353 Producto Punto o escalar, 426
Ecuacin trigonomtrica, 356 Vectores en R3, 433
Identidad trigonomtrica, 354 Vectores ortogonales, 427
Trinomio ordenado, 37 Vectores paralelos, 427

ndice Analtico 567

indice final 560-568 567 25/11/02, 1:34 PM


indice final 560-568 568 25/11/02, 1:34 PM

You might also like